Sei sulla pagina 1di 454

Problem Books in Mathematics

Hayk Sedrakyan
Nairi Sedrakyan

Geometric
Inequalities
Methods of Proving
Problem Books in Mathematics
Series Editor:
Peter Winkler
Department of Mathematics
Dartmouth College
Hanover, NH 03755
USA

More information about this series at http://www.springer.com/series/714


Hayk Sedrakyan • Nairi Sedrakyan

Geometric Inequalities
Methods of Proving

B
r1

r3

r2

A C
r1 + r 2 > r 3
Hayk Sedrakyan Nairi Sedrakyan
University Pierre and Marie Curie Yerevan, Armenia
Paris, France

ISSN 0941-3502 ISSN 2197-8506 (electronic)


Problem Books in Mathematics
ISBN 978-3-319-55079-4 ISBN 978-3-319-55080-0 (eBook)
DOI 10.1007/978-3-319-55080-0

Library of Congress Control Number: 2017937367

Mathematics Subject Classification (2010): 00A07

© Springer International Publishing AG 2017


This work is subject to copyright. All rights are reserved by the Publisher, whether the whole or part of
the material is concerned, specifically the rights of translation, reprinting, reuse of illustrations,
recitation, broadcasting, reproduction on microfilms or in any other physical way, and transmission
or information storage and retrieval, electronic adaptation, computer software, or by similar or
dissimilar methodology now known or hereafter developed.
The use of general descriptive names, registered names, trademarks, service marks, etc. in this
publication does not imply, even in the absence of a specific statement, that such names are exempt
from the relevant protective laws and regulations and therefore free for general use.
The publisher, the authors and the editors are safe to assume that the advice and information in this
book are believed to be true and accurate at the date of publication. Neither the publisher nor the
authors or the editors give a warranty, express or implied, with respect to the material contained
herein or for any errors or omissions that may have been made. The publisher remains neutral with
regard to jurisdictional claims in published maps and institutional affiliations.

Printed on acid-free paper

This Springer imprint is published by Springer Nature


The registered company is Springer International Publishing AG
The registered company address is: Gewerbestrasse 11, 6330 Cham, Switzerland
To Margarita,
a wonderful wife and a loving mother

To Ani,
a wonderful daughter and a loving sister
Preface

Geometric inequalities are one of the most interesting sections of elementary


mathematics and have a wide range of applications in geometry and the other fields
of mathematics, such as algebra and trigonometry. To prove geometric inequalities
one often has to use, besides the geometric reasoning, algebraic transformations,
trigonometric relations and inequalities, calculus and mathematical analysis.
This book is the third book of the authors about inequalities. The first two books
were dedicated to algebraic inequalities and were published in 2015 in South Korea.
All these books reflect long years of experience of the authors in teaching. Most of
the problems were created or proved by the authors during those classes.
The authors have tried not to use, whenever possible, the concept of a derivative,
therefore making the solutions of many problems understandable to students.
The book contains more than 1000 problems. Approximately 800 problems in the
book are with thorough solutions. Basically, these are non-standard problems.
The majority of problems are for mathematics competitions and Olympiads.
Many problems in the book and the majority of the solutions belong to the authors.
Some of those problems were used by the authors to teach their students interested
in mathematical Olympiads. In few cases, the solution was proposed by a student,
so his/her name is indicated. Some problems of the authors, included in this book,
were proposed in mathematical Olympiads (in different countries). Some problems
were proposed in different mathematical journals, such as the American Mathe-
matical Monthly (MAA), Crux Mathematicorum with Mathematical Mayhem
(Canadian Mathematical Society), Mathematical Reflections (USA), and Kvant
(Russia).
The book is divided into eight chapters, each of the chapters consists of one, two,
three, four or five paragraphs. The basis of the classification is usually the method of
the solution.

vii
viii Preface

The authors have tried to find common approaches to different problems. The
goal of the book is to teach the reader new and classical methods for proving
geometric inequalities.
The authors would like to express their gratitude to their family for the support.

Hayk Sedrakyan
Nairi Sedrakyan
Contents

1 Theorem on the Length of the Broken Line . . . . . . . . . . . . . . . . . . 1


1.1 Triangle Inequality . . . . . . . . . . . . . . . . . . . . . . . . . . . . . . . . . . 2
Solutions . . . . . . . . . . . . . . . . . . . . . . . . . . . . . . . . . . . . . . . . . 5
Problems for Self-Study . . . . . . . . . . . . . . . . . . . . . . . . . . . . . . 20
1.2 Theorem on the Length of the Broken Line . . . . . . . . . . . . . . . . 24
Solutions . . . . . . . . . . . . . . . . . . . . . . . . . . . . . . . . . . . . . . . . . 26
Problems for Self-Study . . . . . . . . . . . . . . . . . . . . . . . . . . . . . . 36
2 Application of Projection Method . . . . . . . . . . . . . . . . . . . . . . . . . . 39
2.1 Convex Polygon Lying Inside of Another Polygon . . . . . . . . . . . 40
Solutions . . . . . . . . . . . . . . . . . . . . . . . . . . . . . . . . . . . . . . . . . 41
Problems for Self-Study . . . . . . . . . . . . . . . . . . . . . . . . . . . . . . 58
2.2 Sufficient Conditions for Comparison of Lengths
of Two Broken Lines on the Plane . . . . . . . . . . . . . . . . . . . . . . . 59
Solutions . . . . . . . . . . . . . . . . . . . . . . . . . . . . . . . . . . . . . . . . . 60
Problems for Self-Study . . . . . . . . . . . . . . . . . . . . . . . . . . . . . . 65
2.3 Inscribed Polygons with the Least Perimeter . . . . . . . . . . . . . . . 66
Solutions . . . . . . . . . . . . . . . . . . . . . . . . . . . . . . . . . . . . . . . . . 67
Problems for Self-Study . . . . . . . . . . . . . . . . . . . . . . . . . . . . . . 74
2.4 Method of Projections . . . . . . . . . . . . . . . . . . . . . . . . . . . . . . . . 74
Solutions . . . . . . . . . . . . . . . . . . . . . . . . . . . . . . . . . . . . . . . . . 76
Problems for Self-Study . . . . . . . . . . . . . . . . . . . . . . . . . . . . . . 88
3 Areas . . . . . . . . . . . . . . . . . . . . . . . . . . . . . . . . . . . . . . . . . . . . . . . . 93
3.1 Inequalities with Areas . . . . . . . . . . . . . . . . . . . . . . . . . . . . . . . 93
Solutions . . . . . . . . . . . . . . . . . . . . . . . . . . . . . . . . . . . . . . . . . 98
Problems for Self-Study . . . . . . . . . . . . . . . . . . . . . . . . . . . . . . 135

ix
x Contents

4 Application of Vectors . . . . . . . . . . . . . . . . . . . . . . . . . . . . . . . . . . 139


4.1 Application of Vectors for Proving Geometric
and Trigonometric Inequalities . . . . . . . . . . . . . . . . . . . . . . . . . 139
Solutions . . . . . . . . . . . . . . . . . . . . . . . . . . . . . . . . . . . . . . . . . 143
Problems for Self-Study . . . . . . . . . . . . . . . . . . . . . . . . . . . . . . 163
5 Application of Trigonometric Inequalities . . . . . . . . . . . . . . . . . . . 167
5.1 Inequalities for the Angles of a Triangle . . . . . . . . . . . . . . . . . . 168
Solutions . . . . . . . . . . . . . . . . . . . . . . . . . . . . . . . . . . . . . . . . . 170
Problems for Self-Study . . . . . . . . . . . . . . . . . . . . . . . . . . . . . . 189
5.2 Inequalities for the Angles of Acute
and Obtuse Triangles . . . . . . . . . . . . . . . . . . . . . . . . . . . . . . . . 191
Solutions . . . . . . . . . . . . . . . . . . . . . . . . . . . . . . . . . . . . . . . . . 191
Problems for Self-Study . . . . . . . . . . . . . . . . . . . . . . . . . . . . . . 198
5.3 Some Relations for a Triangle . . . . . . . . . . . . . . . . . . . . . . . . . . 198
Solutions . . . . . . . . . . . . . . . . . . . . . . . . . . . . . . . . . . . . . . . . . 200
Problems for Self-Study . . . . . . . . . . . . . . . . . . . . . . . . . . . . . . 205
5.4 Trigonometric Inequalities . . . . . . . . . . . . . . . . . . . . . . . . . . . . 205
Solutions . . . . . . . . . . . . . . . . . . . . . . . . . . . . . . . . . . . . . . . . . 206
Problems for Self-Study . . . . . . . . . . . . . . . . . . . . . . . . . . . . . . 217
5.5 Using Trigonometric Inequalities for Proving
Geometric Inequalities . . . . . . . . . . . . . . . . . . . . . . . . . . . . . . . 218
Solutions . . . . . . . . . . . . . . . . . . . . . . . . . . . . . . . . . . . . . . . . . 223
Problems for Self-Study . . . . . . . . . . . . . . . . . . . . . . . . . . . . . . 258
6 Inequalities for Radiuses . . . . . . . . . . . . . . . . . . . . . . . . . . . . . . . . . 261
6.1 Inequalities for Radiuses of Circles . . . . . . . . . . . . . . . . . . . . . . 262
Solutions . . . . . . . . . . . . . . . . . . . . . . . . . . . . . . . . . . . . . . . . . 263
Problems for Self-Study . . . . . . . . . . . . . . . . . . . . . . . . . . . . . . 273
6.2 Integer Lattice . . . . . . . . . . . . . . . . . . . . . . . . . . . . . . . . . . . . . 275
Solutions . . . . . . . . . . . . . . . . . . . . . . . . . . . . . . . . . . . . . . . . . 276
Problems for Self-Study . . . . . . . . . . . . . . . . . . . . . . . . . . . . . . 283
7 Miscellaneous Inequalities . . . . . . . . . . . . . . . . . . . . . . . . . . . . . . . . 285
7.1 Miscellaneous Inequalities . . . . . . . . . . . . . . . . . . . . . . . . . . . . 286
7.2 Solutions . . . . . . . . . . . . . . . . . . . . . . . . . . . . . . . . . . . . . . . . . 300
7.3 Problems for Self-Study . . . . . . . . . . . . . . . . . . . . . . . . . . . . . . 407
8 Some Applications of Geometric Inequalities . . . . . . . . . . . . . . . . . 413
8.1 Application of Geometric Inequalities
for Solving Geometric problems . . . . . . . . . . . . . . . . . . . . . . . . 413
Solutions . . . . . . . . . . . . . . . . . . . . . . . . . . . . . . . . . . . . . . . . . 416
Problems for Self-Study . . . . . . . . . . . . . . . . . . . . . . . . . . . . . . 428
Contents xi

8.2 Using Geometric Inequalities for Proving


Algebraic Inequalities . . . . . . . . . . . . . . . . . . . . . . . . . . . . . . . . 430
Solutions . . . . . . . . . . . . . . . . . . . . . . . . . . . . . . . . . . . . . . . . . 432
Problems for Self-Study . . . . . . . . . . . . . . . . . . . . . . . . . . . . . . 443

Basic Notations . . . . . . . . . . . . . . . . . . . . . . . . . . . . . . . . . . . . . . . . . . . . 447

References . . . . . . . . . . . . . . . . . . . . . . . . . . . . . . . . . . . . . . . . . . . . . . . 449
Chapter 1
Theorem on the Length of the Broken Line

This chapter consists of two sections. Section 1.1 is devoted to the applications of
one of the most important geometric inequalities, called the triangle inequality.
As a generalization of triangle inequality, Section 1.2 is devoted to the theorem
on the length of the broken line.
Let us recall the triangle inequality: for any triangle, the sum of the lengths of
any two sides is greater than or equal to the length of the remaining side. In other
words, if A, B, C are arbitrary points, then AB  AC þ BC. Moreover, the equality
holds true, if and only if point C is on segment AB.
The goal of Section 1.1 is to get the reader acquainted with the triangle
inequality.
This section consists of problems that can be proved by using, if necessary
several times, the triangle inequality.
In Section 1.1 selected problems are those that can be proved using the following
techniques and statements:
1. For any convex quadrilateral ABCD it holds true AC þ BD > AB þ CD.
2. For any triangle with side lengths a, b, c and a median ma drawn to the side with
length a it holds true ma < bþc
2 .
3. If M is an arbitrary point inside of triangle ABC, then MA þ MB < CA þ CB.
4. For any points A, B, C, D it holds true AC  BD  AB  CD þ BC  AD.
In some problems, the triangle inequality is not always applied directly. In some
cases, at first one needs to do some geometric constructions or translations and only
after that apply the triangle inequality.
In Section 1.2, selected problems are those that can be proved using the theorem
on the length of the broken line, that is: if A1, A2, . . . , An are arbitrary n points, then
A1An  A1A2 þ A2A3 þ . . . þ An  1An. Moreover, the equality holds true if and
only if point Ai is on the segment Ai  1Ai þ 1, for i ¼ 2, 3, . . . , n  1.
Some problems in this chapter were inspired by [7]. Nevertheless, even for these
problems the authors have mostly provided their own solutions.

© Springer International Publishing AG 2017 1


H. Sedrakyan, N. Sedrakyan, Geometric Inequalities, Problem Books
in Mathematics, DOI 10.1007/978-3-319-55080-0_1
2 1 Theorem on the Length of the Broken Line

1.1 Triangle Inequality

1.1.1. Consider a triangle ABC. Prove that


(a) ∠C < ∠ A, if AB < BC,
(b) AB < BC, if ∠C < ∠ A,
(c) AB < BC þ AC,
(d) the length of any side of the triangle is less than its semiperimeter.
1.1.2. Let A, B and C be arbitrary points. Prove that AC  |AB  BC|.
1.1.3. Given a convex quadrilateral ABCD and a point M on a plane. Prove that
(a) MA þ MB þ MC þ MD  AC þ BD,
(b) MA < MB þ MC þ MD, if AB ¼ CD,
(c) MA < MB þ MC þ MD, if AC ¼ BD.
1.1.4. (a) Prove that in a convex quadrilateral the sum of its diagonals is greater than
its semiperimeter and is smaller than its perimeter.
(b) Let ABCD be a convex quadrilateral such that AB þ BD is not greater than
AC þ CD. Prove that the length of side AB is less than the length of diagonal AC.
(c) Prove that, if we connect the middle of all the adjacent sides of a convex polygon
A1A2 . . . An, then the perimeter of the obtained polygon is not less than the half
of the perimeter of the polygon A1A2 . . . An.
1.1.5. Let n>4 be an integer. For a convex n-gon A1A2...An consider the quadri-
laterals AiAiþ1Aiþ2Aiþ3, i¼1,2,...,n, where Anþj¼Aj. Prove that from those quad-
rilaterals no more than n/2 have an inscirbed circle. Give an example of an octagon
that has such 4 quadrilaterals.
1.1.6. Let ABCD be an inscribed quadrilateral. Prove that
(a) |AB  CD| þ |AD  BC|  2| AC  BD|,
(b) AB þ BD  AC þ CD, if ∠A  ∠ D.
(c) Prove that among all triangles inscribed in a given circle, the largest perimeter
has the equilateral triangle.
1.1.7. Prove that
(a) bþca
2 < ma < bþc
2 ,
(b) 4 ða þ b þ cÞ < ma þ mb þ mc < a þ b þ c, where ma, mb, mc are the lengths of
3

the medians drawn from vertices A, B, C and a, b, c are, respectively, the lengths
of sides BC, CA, AB of triangle ABC.
1.1.8. Let M be a point inside of triangle ABC. Prove that
(a) MA þ MB < CA þ CB,
(b) min(MA, MB, MC) þ MA þ MB þ MC < ma þ mb þ mc,
(c) MA þ MB þ MC  max (AB þ BC, AC þ BC, AC þ AB),
pffiffi
(d) MA þ MB þ MC  23 minðAB þ BC; AC þ BC; AC þ ABÞ.
1.1 Triangle Inequality 3

1.1.9. (a) Let ABCD be a quadrilateral and E,F be the midpoints of sides AB, CD,
respectively. Prove that EF  BCþAD
2 .

(b) Let M be the intersection point of medians AD and BE of triangle ABC. Prove
that, if ∠AMB  π/2, then AC þ BC > 3AB.
(c) Let C1, A1 be points (different from the vertices) on sides AB, BC of triangle
ABC. Let K be the midpoint of A1C1 and I be the in center of triangle ABC.
Given that A1BC1I is an inscribed quadrilateral. Prove that AKC is an obtuse
angle.
1.1.10. Let ABCD be a quadrilateral such that angles A and C are equal to 90 . Prove
that the perimeter of the inscribed quadrilateral in the quadrilateral ABCD is not
smaller than 2AC.
1.1.11. Let M be a point inside of an equilateral triangle ABC. Prove that
(a) MA þ MB > MC ,
(b) MA2 þ MB2 þ MC2 < 2 AB2.
1.1.12. Let E be a point on side AC of triangle ABC. Prove that
(a) BE  AC  AE  BC þ CE  AB,
(b) (EB  BA)  AC  (BC  AB)  AE.
1.1.13. Let D be a point on side BC of triangle ABC such that AD > BC. Let point
E on side AC be defined by the following condition EC AE
¼ ADBC
BD
. Prove that
AD > BE.
1.1.14. (a) Prove that for any distinct points A, B, C and D it holds true AC  BD 
AB  CD þ BC  AD.
(b) Let a square with the center O be externally constructed on the side AB of
triangle ABC. Let M, N be the midpoints of sides BC, AC and the lengths of
these sides be equal to a, b, respectively. Find the possible greatest value of the
sum OM þ ON when angle ∠ACB changes.
OAþOC
(c) Given a rectangle ABCD on a plane. Find the smallest value of OBþOD , where
O is an arbitrary point in that plane.
(d) Prove that for any points A, B, C and D it holds true
AB þ BC þ AC  2AD sin ∠ BDC.
(e) Let D, E, F be points on sides BC, CA, AB of triangle ABC, respectively. Prove
that
 
1 1 1 AB þ BC þ AC
þ þ ðDE þ EF þ FDÞ  ,
AD BE CF R

where R is the circumradius of triangle ABC.


(f) Given a triangle ABC and points D, E, F , such that ∠DBC ¼ ∠ ECA ¼ ∠ FAB > 0
and ∠DCB ¼ ∠ EAC ¼ ∠ FBA > 0.
4 1 Theorem on the Length of the Broken Line

Prove that AFþFBþBDþDCþCEþEA  ADþBEþCF.


(g) Given a point M and a parallelogram ABCD.
Prove that MA  MC þ MB  MD  AB  AD.
(h) Prove that for any distinct points A, B, C and D it holds true
DA  DB  AB þ DB  DC  BC þ DC  DA  AC  AB  BC  AC.
(i) Let ABCDA1B1C1D1 be a parallelepiped.
Prove that AB1 þ AD1 þ AC < AA1 þ AB þ AD þ AC1.
(j) Let SABC be a tetrahedron. Prove that
AB
SAþSB þ SBþSC
BC
> SAþSC
AC
:
(k) Let SABC be a tetrahedron. Prove that SA þ SB þ SC > 3  min (MA, MB, MC),
where M is the intersection point of the medians of triangle ABC.
1.1.15. For any point T of a given triangle (the interior of the triangle included)
denote by m(T) the smallest of segments TA, TB, TC. Find all points of triangle
ABC, such that the value of m(T ) is the greatest possible.
1.1.16. Let A1, A2, . . . , A8 be the vertices of a parallelepiped and O be its center of
 
symmetry. Prove that 4 OA21 þ OA22 þ ::: þ OA28 < ðOA1 þ OA2 þ ::: þ OA8 Þ2 .
1.1.17. Let G be the intersection point of the medians of triangle ABC. Prove that
(a) if AB > AC, then AC þ BG < AB þ CG,
(b) OG < 13 ðOA þ OB þ OCÞ, where O is an arbitrary.
1.1.18. Let O be a point in the hexagon A1A2A3A4A5A6, such that all its sides are
visible under the angle of 60 . Prove that, if OA1 > OA3 > OA5 and OA2 > OA4 >
OA6, then A1A2 þ A3A4 þ A5A6 < A2A3 þ A4A5 þ A6A1.
1.1.19. Given n distinct points. Prove that among those points there are points A, B,
C, such that 1  AC
AB
< 1 þ 2n, if (a) n ¼ 3, (b) n  5.
1.1.20. Given n (n  3) distinct points with the pairwise distances between them
equal to a1 , a2 , :::, anðn1Þ . Given that λn > 0 and λn1
n þ λn2
n ¼ 1. Prove that there
2

exist numbers i and j (i 6¼ j), such that 1  aaij  λ1n . For n ¼ 4, prove that the estimate
qffiffiffiffiffiffiffiffiffiffi
pffiffiffiffiffi qffiffiffiffiffiffiffiffiffiffi
pffiffiffiffiffi
¼ þ 18 ¼ 1, 32471795:::: is not possible to make smaller.
1 3 9 69 3 9þ 69
λ4 18

1.1.21. Given n (n  3) distinct points with the pairwise distances between them
equal to a1 , a2 , :::, anðn1Þ , where a1  a2  :::  anðn1Þ : Given that δn > 0 and δn
2 2
nðn1Þ
ð 1 þ δn Þ 2 1
> 2: Prove that there exist numbers i and j(i 6¼ j), such that
ai 
aj  1 < δn :

1.1.22. Let M, N be distinct points on side BC of triangle ABC, such that


∠MAN
∠BAM ¼ ∠ CAN. Prove that pffiffiffiffiffiffiffiffiffiffi
ffi pffiffiffiffiffiffiffiffiffiffiffi 2 < sin
MNBC
sin ∠BAC < pffiffiffiffiffiffiffiffiffiffi
ffi pffiffiffiffiffiffiffiffiffiffiffi 2 :
MNBC
ð BMCNþ BNCMÞ ð BMCN  BNCMÞ
1.1 Triangle Inequality 5

Solutions

1.1.1. (a) Let D be a point on side BC, such that AB ¼ BD. Then, we have that
∠A > ∠ BAD ¼ ∠ BDA ¼ ∠ C þ ∠ DAC > ∠ C. Therefore ∠A > ∠ C.
(b) Proof by contradiction argument. Assume that AB  BC. If AB ¼ BC, then
∠C ¼ ∠ A. This leads to a contradiction. If AB > BC, then according to prob-
lem 1.1.1а we have that ∠C > ∠ A. This leads to a contradiction.
(c) Let D be a point on line AC, such that point C belongs to segment AD and
CD ¼ BC. Hence, as ∠ABD ¼ ∠ B þ ∠ CBD > ∠ CBD ¼ ∠ BDC, then for
triangle ABD, according to problem 1.1.1b, we obtain that AB < AD. Thus, it
follows that AB < AC þ BC.
2 , where p ¼
(d) We have that p  a ¼ bþca 2 . Therefore, p > a.
aþbþc

1.1.2. Note that AC  AB  BC  AC. Hence, |AB  BC|  AC.


1.1.3. (a) As MA þ MC  AC and MB þ MD  BD, then MA þ MC þ MB þ
MD  AC þ BD.
(b) We have that MA  MB þ AB ¼ MB þ CD  MB þ MC þ MD. Note that the
equality MA ¼ MB þ MC þ MD cannot hold true. Otherwise, M belongs to
segment CD, then MA < MB þ AB.
(c) We have that MA  MC þ AC ¼ MC þ BD  MC þ MB þ MD. It is clear that
the following equalities MA ¼ MC þ AC and BD ¼ MB þ MD simultaneously
cannot hold true. Therefore, MA < MC þ MB þ MD.
1.1.4. (a) Let M be the intersection point of the diagonals of the given convex
quadrilateral ABCD. Then, using the triangle inequality, we obtain that
MA þ MB > AB, MB þ MC > BC, MC þ MD > CD, MA þ MD > AD.
Summing up these inequalities, we deduce that AC þ BD > 12 ðAB þ BCþ
CD þ ADÞ.
From AC < AB þ BC and AC < AD þ DC, it follows that AC < 12 ðAB þ BCþ
CD þ ADÞ. Therefore, AC þ BD < AB þ BC þ CD þ AD.
Remark We have that AC þ BD ¼ (MA þ MB) þ (MC þ MD) > AB þ CD.
(b) We have that AB þ BD  AC þ CD and AB þ CD < AC þ BD. Therefore, 2AB þ
CD þ BD < 2AC þ CD þ BD or AB < AC.
(c) Let B1, B2, . . . , Bn be the midpoints of sides A1A2, A2A3, . . . , AnA1, respec-
tively (Figure 1.1).

Figure 1.1
6 1 Theorem on the Length of the Broken Line

Then, for n  4, we have that

2B1 B2 þ 2B2 B3 þ ::: þ 2Bn B1 ¼ A1 A3 þ A2 A4 þ A3 A5 þ ::: þ An A2 ¼


1
¼ ððA1 A3 þ A2 A4 Þ þ ðA2 A4 þ A3 A5 Þ þ ::: þ ðAn A2 þ A1 A3 ÞÞ >
2
1
> ððA1 A2 þ A3 A4 Þ þ ::: þ ðAn A1 þ A2 A3 ÞÞ ¼ A1 A2 þ A2 A3 þ ::: þ An A1
2

(according to the remark in problem 1.1.4a). Hence, it follows that B1 B2 þ B2 B3


þ::: þ Bn B1 > 12 ðA1 A2 þ A2 A3 þ ::: þ An A1 Þ: For n ¼ 3

1
B1 B2 þ B2 B3 þ ::: þ Bn B1 ¼ ðA1 A2 þ A2 A3 þ ::: þ An A1 Þ:
2

1.1.5. If it would be possible to cut off more than n2 circumscribed quadrilaterals,


then among them there would be two neighboring quadrilaterals having two
common sides. Let us denote these quadrilaterals by ABCD and BCDE (Figure 1.2).
For any of them the sum of the opposite side is equal to AB þ CD ¼ BC þ AD,
BC þ DE ¼ CD þ BE.
Hence, we obtain that

AB þ DE ¼ AD þ BE: ð1:1Þ

The initial n-gon is convex; therefore its diagonals AD and BE intersect at some
point P. According to the triangle inequality AD þ BE ¼ AP þ BP þ PD þ
PE > AB þ DE. This leads to the contradiction with (1.1).
In order to construct the required octagon, let us circumscribe an isosceles
trapezoid A1A2A3A4 around the circle, such that its base is A1A4 and the base
angle is equal to 45 . Then, construct it up to octagon A1A2 . . . A8, as it is shown
in Figure 1.3.
 way, one can construct n-gon, such that it is possible to cut off by its
In a similar
diagonals n2 circumscribed quadrilaterals.
1.1.6. (a) Let M be the intersection point of diagonals AC and BD of a quadrilateral
ABCD. Note that ΔABM ~ ΔDCM. Therefore,

Figure 1.2
1.1 Triangle Inequality 7

Figure 1.3

 
     
AC  BD ¼ AM þ MC  BM  DM ¼ AM þ BM  CD  BM  AM  CD ¼
 AB AB 
 
AM  BM    
¼  AB  CD  AB  CD
AB

(see problem 1.1.2). In a similar way, we obtain that |AC  BD|  |AD  BC|, thus
|AB  CD| þ |AD  BC|  2|AC  BD|.
In the last inequality the equality holds true if and only if quadrilateral ABCD is a
rectangle.
(b) Note that ∠MAD  ∠ MDA. Therefore, MD  MA. As CD AB ¼ MB ¼ MA ¼ k  1
CM DM

(see the proof of problem 1.1.6а), then AC þ CD  AB  BD ¼ (k  1)


(AB þ BM  AM)  0.
(c) Let an irregular triangle ABC be inscribed in the given circle, such that
∠A  ∠ B  ∠ C. Note that ∠A > 60 > ∠ C.
Let D be a point on arc ABC, such that ∠DAC ¼ 60 . As ∠A > ∠ A þ
∠ C  60 ¼ ∠ ACD, then according to problem 1.1.6b (see the proof), AB þ BC þ
AC < AD þ DC þ AC.
If triangle ADC is equilateral, then this ends the proof. Otherwise, if triangle
ADC is irregular, then repeating the above proof for triangle ADC, we obtain that its
perimeter is smaller than the perimeter of the equilateral triangle inscribed in the
given circle. Therefore, in this case also the perimeter of triangle ABC is smaller
than the perimeter of the equilateral triangle inscribed into the given circle.
Other proofs of this problem one can obtain using problems 5.1.6 and 8.2.1i.
1.1.7. (a) Consider Figure 1.4.
We have that 2ma < b þ c, thus ma < bþc
2 . Let c  b, then ma þ 2 > c  2 .
a cþb

Hence, it follows that ma > 2 .


cþba

(b) We have that ma < bþc 2 , mb < 2 and mc < 2 , thus ma þ mb þ mc < a þ b þ c.
aþc aþb

Note that 23 ma þ 23 mc > b. In a similar way, we obtain that 23 ma þ 23 mb > c and


3 mc þ 3 mb > a. Summing up these inequalities, we deduce that ma þ mb þ
2 2

mc > 34 ða þ b þ cÞ.
8 1 Theorem on the Length of the Broken Line

Figure 1.4

Figure 1.5

Figure 1.6

1.1.8. (a) Let N be the intersection point of lines AM and BC; then AM þ BM <
AM þ MN þ BN ¼ AN þ NB < AC þ CN þ BN ¼ AC þ BC.
Therefore, AM þ BM < AC þ BC.
(b) Let M be a point inside of triangle AGB1 (Figure 1.5), where G is the intersection
point of the medians of triangle ABC.
Using the result of problem 1.1.8а, we obtain that AM þ BM  b2 þ mb and
AM þ MC  23 ma þ 23 mc . Therefore,

minðAM; BM; CMÞ þ AM þ BM þ CM  2AM þ BM þ CM 


b 2 2 ma mc 2 2
 þ mb þ ma þ mc < þ þ mb þ ma þ mc ¼ mb þ ma þ mc :
2 3 3 3 3 3 3

We obtain that min(AM, BM, CM) þ AM þ BM þ CM < mb þ ma þ mc.


Remark If the triangle is not obtuse, then taking the point M in the center of the
circumcircle of triangle ABC, we obtain that ma þ mb þ mc > 4R, where R is the
circumradius of triangle ABC.
(c) Let us draw through point M segments parallel to AB, AC and BC (Figure 1.6).
1.1 Triangle Inequality 9

Let AB  AC  BC. Since triangles C2MC1, MB1B2, MA1A2 are similar to


triangle ABC, then the smallest sides of these triangles are C1C2, MB1 and MA2,
respectively. We have that

MA þ MB þ MC < ðAB1 þ B1 MÞ þ ðMA2 þ A2 BÞ þ ðMA1 þ A1 CÞ 


 AB1 þ B1 B2 þ A1 A2 þ A2 B þ CB2 þ A1 C ¼ AC þ BC:

Hence, it follows that MA þ MB þ MC < AC þ BC.


One can easily prove that the inequality holds true if M belongs to one of the
sides of the triangle.
(d) Let us consider two cases.
Case 1 Let there be a point M0 inside of the triangle, such that ∠AM0B ¼
∠ BM0C ¼ ∠ AM0C ¼ 120 . Then, prove that AM þ BM þ CM  AM0 þ
BM0 þ CM0.
Consider an equilateral triangle BCA1 constructed externally on the side BC of
triangle ABC. We have that ∠BM0C þ ∠ BA1C ¼ 120 þ 60 ¼ 180 . Hence,
M0BA1C is an inscribed quadrilateral. Thus, it follows that M0A1 ¼ M0B þ M0C
(see the proof of problem 1.1.14а) and ∠AM0A1 ¼ ∠ AM0B þ ∠ BM0A1 ¼
120 þ ∠ BCA1 ¼ 180 . Hence, AM0 þ BM0 þ CM0 ¼ AM0 þ M0A1 ¼ AA1.
According to problem 1.1.14а, we have that MB þ MC  MA1. Therefore, we
deduce that MA þ MB þ MC  MA þ MA1  AA1 ¼ AM0 þ BM0 þ CM0.
pffiffi
Now, let us prove that AM0 þ BM0 þ CM0  23 minðAB þ BC; ABþ
AC; BC þ ACÞ. Let max(∠A, ∠B, ∠C) ¼ ∠ C. Note that ∠AM0B > ∠ C. Hence,
60  ∠ C < 120 . Thus, it follows that 120  ∠ ACA1 < 180 . Therefore,
pffiffiffiffiffiffiffiffiffiffiffiffiffiffiffiffiffiffiffiffiffiffiffiffiffiffiffiffiffiffiffiffiffiffiffiffiffiffiffiffiffiffiffiffiffiffiffiffiffiffiffiffiffiffiffiffiffiffiffiffiffiffiffiffiffiffiffiffiffiffiffiffiffi
AA1 ¼ AC2 þ BC2  2AC  BC  p cos ffiffiffi ∠ACA1 
pffiffiffiffiffiffiffiffiffiffiffiffiffiffiffiffiffiffiffiffiffiffiffiffiffiffiffiffiffiffiffiffiffiffiffiffiffiffiffiffiffiffiffiffi 3
 AC2 þ BC2 þ AC  BC  ðAC þ BCÞ:
2

Case 2 If there does not exist such point M0 (see Case 1), then one can easily prove
that ∠C  120 .
Note that either ∠MCB < 120 or ∠MCA < 120 . Without loss of generality one
can assume that ∠MCB < 120 ; then according to problem 1.1.8а, MA þ MA1 
AC þ CA1. Therefore, MA þ MB þ MC  MA þ MA1  AC þ BC.
This ends the proof.
1.1.9. (a) Let us denote by K the midpoint of diagonal BD. We have that
EF  EK þ KF ¼ AD 2 þ 2 , and the equality holds true if and only if K belongs to
BC

the segment EF, that is AD||BC.


(b) Note that point M cannot be inside of the circle with diameter AB. Hence,
MO  AB 2 , where O is the midpoint of segment AB. According to problem
4 þ 4 > 3  2 . Thus, it follows that AC þ BC > 3  AB.
1.1.9a, we have that AC BC MO
10 1 Theorem on the Length of the Broken Line

Figure 1.7

Figure 1.8

(c) Let O be the midpoint of segment AC. Note that AC ¼ AC1 þ CA1. According to
problem 1.1.9а, we have that OК  AC1 þCA
2
1
¼ AC
2 . Therefore, AKC is an obtuse
angle.
1.1.10. Let a quadrilateral MNPK be inscribed in a quadrilateral ABCD (Figure 1.7).
Let E and F be the midpoints of segments KM and NP, respectively. Since
AE ¼ KM 2 , EF 
MNþKP
2 (see problem 1.1.9а) and CF ¼ NP 2 , then
MNþNPþKPþKM
2
 AE þ EF þ CF  AF þ CF  AC. Thus, it follows that MN þ NP þ KP þ
KM  2AC.
Remark If ∠A  90 , ∠ C  90 , then MN þ NP þ KP þ KM  2AC.
1.1.11. (a) Let us consider the Figure 1.8.
Consider segments MA1, MB1, MC1, such that MA1||AB, MB1||BC and MC1||AC.
One can easily prove that AC1MB1, BA1MC1 and CA1MB1 are isosceles trapezoids.
Therefore, MA ¼ C1B1, MB ¼ A1C1 and MC ¼ A1B1. Hence, MA þ MB > MC .
(b) Let MA1 ¼ x, MB1 ¼ y, MC1 ¼ z, AB ¼ a (Figure 1.8). Therefore, x þ y þ z ¼ a
and
1.1 Triangle Inequality 11

Figure 1.9

MA2 þ MB2 þ MC2 ¼ zða  xÞ þ y2 þ xða  yÞ þ z2 þ yða  zÞ þ x2 ¼


¼ a2 þ x2 þ y2 þ z2  zx  xy  yz < a2 þ ðx þ y þ zÞ2 ¼ 2a2 :

1.1.12. (a) If point E coincides with points A and C, then


AB  EC þ BC  AE ¼ BE  AC.
If point E does not coincide with points A and C, then consider the following
figures (Figure 1.9а, b), where MP ¼ AE  EC, NP ¼ BE  EC, MN ¼ AB  EC,
PK ¼ EB  AE, MK ¼ BC  AE.
We have that MN þ MK > NK or AB  EC þ BC  AE > AC  BE.
(b) Note that the inequality of problem 1.1.12b is equivalent to the inequality of
problem 1.1.12a.
1.1.13. We have that

AE  AD ¼ BD  EC þ AE  BC: ð1:2Þ

From problem 1.1.12а, it follows that AC  BE < AE  BC þ AB  EC. From the


condition of the problem, it follows that point E does not coincide with points
A and C.
Let AD  BE, then AC  AD  AC  BE < AE  BC þ AB  EC. Using (1.2), we
obtain that AC  AD ¼ AD  AE þ AD  EC ¼ BD  EC þ AE  BC þ AD  EC. There-
fore, EC(BD þ AD  AB) < 0. This leads to a contradiction. Hence, AD > BE.
1.1.14. (a) Let us take a point A1 on a ray DA, such that DA1 ¼ DA
1
. In a similar way,
take points B1 and C1 on the rays DB and DC. One can easily prove that
A1 B1 ¼ DADB
AB
, B1 C1 ¼ DBDC
BC
and C1 A1 ¼ DCDA
CA
. We have that A1B1 þ B1C1  A1C1.
Thus, AB  DC þ BC  DA  AC  BD (see problem 4.1.9).
(b) According to problem 1.1.14а, for points A, N, K, O we have that
pffiffi
NO  AK  AO  NK þ AN  OK or NO  b2 þ 22 a, where K is the midpoint of
pffiffi
side AB. In a similar way, we obtain that МО  a2 þ 22 b. Hence, it follows
12 1 Theorem on the Length of the Broken Line

pffiffi
that МО þ NO  22þ1 ða þ bÞ. The equality holds true if only ∠ANO ¼
∠ CMO ¼ 90 , that is ∠C ¼ ∠ A þ ∠ B þ 90 . Therefore ∠C ¼ 135 .
(c) According to problem 1.1.14a, for points A, B, C, O we have that
AC  BO  AB  CO þ BC  AO . In a similar way, for points A, D, C, O we
have that AC  DO  AD  CO þ AO  CD. Summing up these inequalities, we
AOþCO
obtain that BOþDO  ABþADAC
. Taking O  A, we obtain that the left-hand side of
AC
the last inequality is equal to ABþAD . Thus, the possible smallest value is equal
AC
to ABþAD.
OAþOC
Remark One can prove that OBþOD  ABþBC
BD .

(d) If points B, C, D are on one line, then sin ∠ BDC ¼ 0; therefore AB þ BC þ


AC  2AD sin ∠ BDC.
If points B, C, D are not on one line and point O is the circumcenter of triangle
BCD, then, according to problem 1.1.14а, for points A, B, O, C we have that

AB  OC þ AC  OB  AO  BC:

Hence,

AB  OC þ AC  OB þ BC  OD  BCðAO þ ODÞ: ð1:3Þ

We have that AO þ OD  AD and OC ¼ OB ¼ OD. Thus from (1.3) we obtain


that

BC
AB þ AC þ BC  AD  ¼ 2AD sin ∠BDC:
OD

(e) According to problem 1.1.14d, we obtain that EDþDFþFE


AD  2 sin ∠A ¼ BCR ; in a
similar way we obtain that EDþDFþFE
 AC
and EDþDFþFE
 AB
. Summing up
1
BE R  CF R
these inequalities, we deduce that AD þ BE þ CF ðED þ DF þ EFÞ 
1 1 ABþBCþAC
R .
(f) Let BC ¼ mx, BD ¼ nx, DC ¼ kx, AC ¼ my, CE ¼ ny, AE ¼ ky, AB ¼ mz, AF ¼ nz,
FB ¼ kz.
According to problem 1.1.14а, for points A, B, D, C, we have that
AB  DC þ AC  DB  AD  BC or kz þ ny  AD; that is, FB þ CE  AD. In a similar
way we obtain that BD þ AE  CF and AF þ DC  BE. Summing up these inequal-
ities, we deduce that AF þ FB þ BD þ DC þ CE þ EA  AD þ BE þ CF.

!
!
(g) Consider a point M0 , such that MM0 ¼ AD . For points M, C, M0 , D, according to
problem 1.1.14a, we have that MC  M0 D þ CM0  MD  CD  MM0 .
As M0 D ¼ AM, CM0 ¼ BM, MM0 ¼ AD, then MA  MC þ MB  MD  AB  AD.

!
!
(h) Consider a point A0 , such that BA0 ¼ AC .
1.1 Triangle Inequality 13

According to problem 1.1.14g, we have that


AD  DA0 þ BD  DC  AB  AC; thus it follows that

AD  BC  DA0 þ BD  DC  BC  AB  AC  BC: ð1:4Þ

According to problem 1.1.14a, we have that

BD  CA0 þ CD  BA0  BC  DA0 :

Thus, we deduce that

BD  AD  AB þ CD  AD  AC  BC  DA0  AD: ð1:5Þ

Summing up inequalities (1.4) and (1.5), we obtain that

DA  DB  AB þ DB  DC  BC þ DC  DA  AC  AB  BC  AC:

(i) For points A, A1, B1, C1 according to problem 1.1.14a, we have that

AB1  A1 C1 < AA1  B1 C1 þ A1 B1  AC1 ¼ AA1  AD þ AB  AC1

(see the proof of problem 1.1.14а).


Therefore,

AB1  AC < AA1  AD þ AB  AC1 : ð1:6Þ

In a similar way, for points A, D, D1, C1 and A, B, C, C1 we have that


AD1  DC1 < AD  D1C1 þ AC1  DD1 and AC  BC1 < AB  CC1 þ AC1  BC, or

AD1  AB1 < AD  AB þ AC1  AA1 , ð1:7Þ

and

AC  AD1 < AB  AA1 þ AC1  AD: ð1:8Þ

Let O, O1 be the intersection points of the diagonals of parallelograms ABCD


and A1B1C1D1, respectively. Let line AC1 intersect with segments A1O and CO1 at
points M and M1, respectively. Since AA1C1C is a parallelogram, then A1O1 ¼ OC
and A1O1||OC, thus quadrilateral A1O1CO is also a parallelogram. Therefore, A1O||
O1C. We have that A1O1 ¼ O1C1 and A1M||O1M1. Hence MM1 ¼ M1C1. In a similar
way, we obtain that AM ¼ MM1.
Note that ΔAOM  ΔC1A1M, thus AMO 1M
¼ AAO
1 C1
¼ AO
AC
¼ 2. It follows that M is the
intersection point of the medians of triangle A1BD. In a similar way, we obtain that
M1 is the intersection point of the medians of triangle B1D1C.
According to Stewart’s theorem, we obtain that
14 1 Theorem on the Length of the Broken Line

 
1 2 2 2 1 2 AB2 þ AD2 BD2
AM ¼ AA1 þ AO  A1 O ¼ AA1 þ
2 2 2 2
 
 32 3 9
 3 3 2 4
2 A1 B þ A1 D2 BD2 1  1 
  ¼ AA1 2 þ AB2 þ AD2  A1 B2 þ A1 D2 þ BD2 :
9 2 4 3 9

In a similar way, we obtain that

1  1 
AM1 2 ¼ AB1 2 þ AD1 2 þ AC2  B1 D1 2 þ D1 C2 þ CB1 2 :
3 9

Therefore

1 
AM1 2  AM2 ¼ AB1 2 þ AD1 2 þ AC2  AB2  AD2  AA1 2
3

as BD ¼ B1D1, A1D ¼ B1C, A1B ¼ D1C.


Note that AM1 ¼ 23 AC1 , AM ¼ 13 AC1 . Hence,

AB1 2 þ AD1 2 þ AC2 ¼ AB2 þ AD2 þ AA1 2 þ AC1 2 ;

from this equality and inequalities (1.6), (1.7), (1.8) we obtain that

AB1 2 þ AD1 2 þ AC2 þ 2AB1  AC þ 2AD1  AB1 þ 2AC  AD1 <


< AB2 þ AD2 þ AA1 2 þ AC1 2 þ 2AA1  AD þ 2AB  AC1 þ 2AD  ABþ
þ2AC1  AA1 þ 2AB  AA1 þ 2AC1  AD,

or (AB1 þ AC þ AD1)2 < (AA1 þ AB þ AD þ AC1)2. Therefore,

AB1 þ AC þ AD1 < AA1 þ AB þ AD þ AC1 :

(j) Note that

AB BC AB2 BC2
þ ¼ þ 
SA þ SB SB þ SC SA  AB þ SB  AB SB  BC þ SC  BC
ðAB þ BCÞ2
 :
SA  AB þ SC  BC þ SBðAB þ BCÞ

It is sufficient to prove that

ðAB þ BCÞ2 ðSA þ SCÞ > SA  AB  AC þ SC  BC  AC þ SB  ACðAB þ BCÞ


ðAB þ BCÞðAB  SA þ BC  SC þ AB  SC þ BC  SA  SB  ACÞ >
> ACðSA  AB þ SC  BCÞ,
1.1 Triangle Inequality 15

ðAB þ BC  ACÞðAB  SA þ BC  SCÞþ


þðAB þ BCÞðAB  SC þ BC  SA  SB  ACÞ > 0:

This holds true, as AB  SC þ BC  SA  SB  AC  0 (see problem 1.1.14а) and


AB þ BC > AC.
(k) Let us consider two cases.
Case 1 Assume that there exists a point M0 inside of the triangle ABC, such that
∠AM0B ¼ ∠ BM0C ¼ ∠ AM0C ¼ 120 . We have that SA þ SB þ SC > M0A þ
M0B þ M0C (see the solution of problem 1.1.8d). Let us prove that M0A þ M0B þ
M0C  3  min (MA, MB, MC).
Let M0A ¼ x, M0B ¼ y, M0C ¼ z (x  y  z), then 3  minðMA; MB; MCÞ ¼
pffiffiffiffiffiffiffiffiffiffiffiffiffiffiffiffiffiffiffiffiffiffiffiffiffiffiffiffiffiffiffiffiffiffiffiffiffiffiffiffiffiffiffiffiffiffiffiffiffiffiffiffiffiffiffiffiffiffiffiffiffi
x2 þ y2 þ 4z2 þ 2xz þ 2yz  xy: Therefore,
pffiffiffiffiffiffiffiffiffiffiffiffiffiffiffiffiffiffiffiffiffiffiffiffiffiffiffiffiffiffiffiffiffiffiffiffiffiffiffiffiffiffiffiffiffiffiffiffiffiffiffiffiffiffiffiffiffiffiffiffiffi
xþyþz x2 þ y2 þ 4z2 þ 2xz þ 2yz  xy:

Case 2 Assume that there does not exist such a point M0. Let max
(∠A, ∠B, ∠C) ¼ ∠ C. We have that (see the proof of problem 1.1.8d)
pffiffiffiffiffiffiffiffiffiffiffiffiffiffiffiffiffiffiffiffiffiffiffiffiffiffiffiffiffiffiffiffiffiffiffiffiffiffiffiffiffi
SA þ SB þ SC > AC þ BC > 2AC2 þ 2BC2  AB2 ¼ 3  minðMA; MB; MCÞ:

See also problem 7.1.107d.


1.1.15. Let ∠A  ∠ B  ∠ C.
If ∠A  90 , then the required point T is the circumcenter O of the triangle. Let
the radius of the circumcircle be equal to R; then m(O) ¼ R.
If point T does not coincide with point O, then it is in one of the triangles AOB,
AOC, BOC. Let point T is in triangle AOB; then mðT Þ  TAþTB 2 < OAþOB
2 ¼ R (see
problem 1.1.8а).
If ∠A > 90 , then let us take on side BC points M and N, such that ∠BAM ¼ ∠ B
and ∠CAM ¼ ∠ C. Then ∠BAM ¼ ∠ B < ∠ BAN ¼ ∠ A  ∠ C (Figure 1.10).
If T 2 ΔABM, then mðT Þ  ATþBT 2  AMþBM
2 ¼ AM  AN ¼ mðN Þ, as ∠AMN ¼
2 ∠ B  ∠ ANB ¼ 2 ∠ C and BN ¼ BM þ MN ¼ AM þ MN > AN.

Figure 1.10
16 1 Theorem on the Length of the Broken Line

0
If T 2 ΔAMN, then m(T )  AT  AA  max (AM, AN) ¼ AN ¼ m(N ), as max
0 0
(∠MA A, ∠NA A)  90 .
If T 2 ΔANC, then mðT Þ  ATþTC
2  ANþCN
2 ¼ AN ¼ mðN Þ.
Thus, the greatest value of m(T )is equal to m(N ). From the proof, it follows that,
if ∠B ¼ ∠ C, then points M and N are the required points. Otherwise, point N is the
required point.
Remark In the case, if ∠A > 90 , mðT Þ  CN < BC
2.

1.1.16. Let OA1 ¼ a, OA2 ¼ b, OA3 ¼ c, OA4 ¼ d and a ¼ max(a,b,c,d). According


to problem 1.1.3b [it is true also, if M does not belong to the plane (ABC)], we have
that b þ c þ d > a. Therefore

ðOA1 þ ::: þ OA8 Þ2 ¼ 4ða þ b þ c þ dÞ2 > 


> 4 a2 þ b2 þ c2 þ d 2 þ aðb þ c þ d Þ þ ab þ ac þ ad >
 2   
> 4 a þ b2 þ c2 þ d 2 þ a2 þ b2 þ c2 þ d 2 ¼ 4 OA21 þ ::: þ OA28 :

1.1.17. (a) Using the triangle inequality, we obtain that BG þ 12 CG > 12 AB and
CG þ 12 BG > 12 AC. Summing up these inequalities, we deduce that 3(BG þ CG) >
AB þ AC.
2
CG2 AB2 AC2 AB2 AC2
We have that BG  CG ¼ BG BGþCG ¼ 3ðBGþCGÞ < ABþAC . Hence, AC þ BG <
AB þ CG.
(b) Let A1 be the midpoint of side BC. According to problem 1.1.7a, we have that
OA1  OBþOC
2 . Point A2 is on the segment OA1 and A1A2 : A2O ¼ 1 : 2.

Then OG  A2 G þ A2 O ¼ AO
3 þ 3 OA1  3 þ 3 þ 3 . It is clear that the equal-
2 AO BO CO

ities OG ¼ A2G þ A2O and OA1 ¼ 2 do not hold true simultaneously. Thus, it
OBþOC

follows that OG < 13 ðAO þ BO þ COÞ.


0
1.1.18. Let A05 be a point on ray OA5, such that OA 5 ¼ OA3 and let A04 be a point on
0
the segment OA4, such that OA 4 ¼ OA6. Then, we have that A04 A05 þ A4 A5 > A4 A05
þA5 A04 ; that is, A05 A6 þ A4 A5 > A4 A05 þ A5 A6 . Therefore,

A1 A 2 þ A3 A 4 þ A5 A6  A2 A3  A4 A5  A6 A1
< A1 A2 þ A3 A4 þ A05 A6  A2 A3  A4 A05  A6 A1 : ð1:9Þ
0 0
Let A01 be a point on the segment OA1, such that OA 1 ¼ OA3 ¼ OA 5 and let A02 be
0
a point on the segment OA2, such that OA 2 ¼ OA6. Then, we have that
A1 A2 þ A01 A02 < A01 A2 þ A02 A1 , thus A1 A2 þ A01 A6 < A01 A2 þ A1 A6 . Hence,

A1 A2 þ A3 A4 þ A05 A6  A2 A3  A4 A05  A6 A1 <


< A01 A2 þ A3 A4 þ 0 0 0
 A5A6  A2 A3 0 A 4 A50 A1 A6 ¼0  ð1:10Þ
0
¼ A1 A2  A2 A3 þ A3 A4  A4 A5 þ A5 A6  A1 A6 ¼ 0 þ 0 þ 0 ¼ 0:

From (1.9) and (1.10) it follows that A1A2 þ A3A4 þ A5A6< A2A3 þ A4A5 þ A6A1.
1.1 Triangle Inequality 17

1.1.19. Proof by contradiction argument. Let M and N be two points among those n,
such that the distance MN ¼ d1 is the greatest. Consider plane Π, passing through
the midpoint of segment MN and perpendicular to it.
One of the half-spaces with boundary Π contains k points from the given
n points, where k  n2. Let us denote these points by N, N1, . . . , Nk  1 and
MN ¼ d1  MN1 ¼ d2  . . . .  MNk  1 ¼ dk. According to our assumption, we
have that
     
2 2 2
d1  1 þ d2 , d2  1 þ d3 , :::, dk1  1 þ d k :
n n n

Multiplying these inequalities, we obtain that


 
2 k1
d1  1 þ dk : ð1:11Þ
n
 
As MNk  1  NNk  1, then according to our assumption, MN k1  1 þ 2n NN k1 .
On the other hand, by the triangle inequality NNk  1  MN  MNk  1. Therefore,
it follows that
   
2 2
2 þ dk  1 þ d1 : ð1:12Þ
n n

From (1.11) and (1.12), we deduce that


 k
2 2
2þ  1þ : ð1:13Þ
n n
 k
Note that for n  6, we have that 1 þ 2n ¼ 1 þ k  2n þ kðk1Þ2  n42 þ . . . > 2 þ n2
2n :
From the last inequality and from 1.13, we deduce that 2 þ n > 2 þ 2n . From this
2 n2

inequality it follows that n < 6. This leads to a contradiction.


7k 73
For n ¼ 5, from (1.13) we obtain that 12 5  5  5 . This leads to a
contradiction.
For n ¼ 3, we have that MN  53 MN 1 and MN 1  53 NN 1 . Therefore, it follows
that MN  MN 1 þ NN 1  35 MN þ 25 9
MN. This leads to a contradiction.
pffiffi
Remark 1. For n ¼ 4, there exist points A, B and C, such that 1  AC AB
 1þ2 5.
pffiffi
Besides, in this inequality the estimate 1þ2 5 is not possible to make smaller.
2. In a similar way, one can prove that among n points it is possible to choose points
A, B, C, such that 1  ACAB
 λ10 , where λ0 is the positive root of λk þ λk1 ¼ 1 and
nþ1
k¼ 2 .
18 1 Theorem on the Length of the Broken Line

It is known that the estimate λ10 is not possible to make smaller for 3  n  7.

1.1.20. The proof by contradiction method. Consider given points A1, A2, . . . , An,
such that AiAj  A1An ¼ d1, i, j 2 {1, ..., n} and max ðA1 Ai ; Ai An Þ ¼ di .
i2f2;:::;n1g
Without loss of generality one can assume that d2  d3  . . .  dn  1. Then
according to our assumption, we obtain that dd12 > λ1n , dd23 > λ1n , :::, ddn2
n1
> λ1n .
Multiplying these inequalities, we deduce that

n d 1 > d n1 :
λn2 ð1:14Þ

Let A1An  1 ¼ dn  1. Then according to our assumption, we have that A1 An1


> λ1n An1 An and according to the triangle inequality A1An  1 þ An  1An  A1An.
Therefore,

ð1 þ λn Þdn1 > d1 : ð1:15Þ

Using (1.14) and (1.15), we obtain that λn2


n þ λn1
n > 1.
This leads to a contradiction.
Remark If n  3, λn > 0 and λn2
n þ λn1
n ¼ 1, then λ1n < 1 þ 2n.
Indeed, if λn  nþ2, then using Bernoulli’s inequality, we obtain that
n

 n2  n1 2nþ2 2nþ2


n n
λn2 þ λn1  þ ¼ nþ2
n2  nþ2
n n
nþ2 nþ2 1 þ 2n 1 þ 2n ðn  2Þ < 1:

This leads to a contradiction.


qffiffiffiffiffiffiffiffiffiffi
pffiffiffiffiffi qffiffiffiffiffiffiffiffiffiffi
pffiffiffiffiffi 3
¼ þ ¼ 1 þ λ10 or
1 3 9 69 3 9þ 69 1
One can easily prove that if λ0 18 18 , then λ0
λ30 þ λ20 ¼ 1. Therefore, λ4 ¼ λ0.
Let us prove that for n ¼ 4, the estimate λ14 is not possible to make smaller.
Indeed, let A1, A2, A3, A4 be such points on one line, that A1A2 ¼ 1, A2A3 ¼ λ4,
A3 A4 ¼ λ24 , A2 belongs to the segment A1A3 and A3 belongs to segment A2A4. Then
a1 ¼ 1 þ λ4 þ λ24 , a2 ¼ 1 þ λ4, a3 ¼ λ4 þ λ24 , a4 ¼ 1, a5 ¼ λ4, a6 ¼ λ24 .
It is enough to note that aa12 ¼ aa23 ¼ aa34 ¼ aa45 ¼ aa56 ¼ λ14 .
1.1.21. Proof by contradiction
 argument. According to our assumption, for all i > j
ai 
we have that aj  1  δn . Therefore,
nðn1Þ
anðn1Þ  ð1 þ δn Þanðn1Þ1  ð1 þ δn Þ2 anðn1Þ2  :::  ð1 þ δn Þ 2 1 a1 ,
2 2 2

hence
nðn1Þ
anðn1Þ  ð1 þ δn Þ 2 1 a1 :
2
1.1 Triangle Inequality 19

Let AB ¼ anðn1Þ , CD ¼ a1 : Using the triangle inequality, we have that


2

AC þ BC  AB. Then, without loss of generality one can assume that AC  AB


2
and A 6¼ D. Note that
    
 minðAC; ADÞ


 maxðAC; ADÞ  min AC; AD 
maxðAC; ADÞ  1 ¼ maxðAC; ADÞ
CD CD
 
2  < δn ,
AC AB 2
nðn1Þ
1
ð1þδn Þ 2

 
 minðAC;ADÞ 
therefore max ðAC;ADÞ  1  < δn . This leads to a contradiction.

1.1.22. Let R and r be the circumradiuses of triangles ABC and MAN, respectively.
∠MAN
sin ∠BAC ¼ 2r  BC.
MN 2R
According to the law of sines, we have that sin
2 2
pffiffiffiffiffiffiffiffiffiffiffi pffiffiffiffiffiffiffiffiffiffiffi 2 < r < pffiffiffiffiffiffiffiffiffiffiffi pffiffiffiffiffiffiffiffiffiffiffi 2 :
BC R BC
Hence, it is enough to prove that
ð BMCNþ BNCMÞ ð BMCN BNCMÞ
Let the circumcircle of triangle MAN intersect sides AB and AC of triangle ABC
at points K and E, respectively (Figure 1.11).
We have that ∠KEM ¼ ∠ BAM ¼ ∠ CAN ¼ ∠ EMC. Therefore, KE k BC.
Thus, it follows that ΔKAE  ΔBAC.
pffiffiffiffiffiffiffiffiffiffiffi pffiffiffiffiffiffiffiffiffiffiffi
Hence, KE ¼ Rr BC, AK ¼ pBMBN ffiffiffiffiffiffiffiffiffiffiffiffi , AE ¼ pffiffiffiffiffiffiffiffiffiffiffiffi
CMCN
Using the triangle inequal-
r ð r 1Þ r ð r 1Þ:
R R R R
pffiffiffiffiffiffiffiffiffiffiffi pffiffiffiffiffiffiffiffiffiffiffi
j BMBN  CMCN j
ity, we obtain that |AK  AE| < KE < AK þ AE. Therefore, <
ffi pffiffiffiffiffiffiffiffiffiffiffi
pffiffiffiffiffiffiffiffiffiffirffi pffiffiffiffiffiffiffiffiffiffi BC
BMBN þ CMCN BC2 BC2
1R< . Thus, pffiffiffiffiffiffiffiffiffiffiffi pffiffiffiffiffiffiffiffiffiffiffi 2 < r < pffiffiffiffiffiffiffiffiffiffiffi pffiffiffiffiffiffiffiffiffiffiffi 2 :
R
BC
ð BMCNþ BNCMÞ ð BMCN  BNCMÞ

Figure 1.11
20 1 Theorem on the Length of the Broken Line

Problems for Self-Study

1.1.23. There are 50 correct clocks on the table. Prove that at some moment the sum
of the distances from the center of the table to the end of the minute hands will be
greater than the sum of the distances from the center of the table to the center of the
clocks.
1.1.24. Let M be a point inside of the parallelogram ABCD. Prove that

AC þ BD  MA þ MB þ MC þ MD < 2ðAB þ BCÞ:

1.1.25. Prove that the sum of the lengths of the diagonals of a convex pentagon is
greater than its perimeter and is smaller than twice the perimeter.
1.1.26. Prove that for any convex pentagon there are three diagonals that are sides
of a triangle.
1.1.27. Prove that in a convex polygon there are no three sides that are greater than
the largest diagonal of that polygon.
1.1.28. Prove that the arithmetic mean of the lengths of the sides of an arbitrary
convex polygon is less than the arithmetic mean of the lengths of its diagonals.
1.1.29. Prove that h1a < h1b þ h1c , where ha, hb, hc are the altitudes of some triangle.

1.1.30. Prove the following inequalities.


pffiffiffiffiffiffiffiffiffiffiffiffiffiffiffiffiffiffiffiffiffiffiffiffiffiffi pffiffiffiffiffiffiffiffiffiffiffiffiffiffiffiffiffiffiffiffiffiffiffiffiffi pffiffiffiffiffiffiffiffiffiffiffiffiffiffiffiffiffiffiffiffiffiffiffiffiffi
(а) a2 þ b2  ab þ b2 þ c2  bc  a2 þ c2 þ ac, where a, b, c > 0.
pffiffiffiffiffiffiffiffiffiffiffiffiffiffiffiffiffiffiffiffiffiffiffiffiffiffi pffiffiffiffiffiffiffiffiffiffiffiffiffiffiffiffiffiffiffiffiffiffiffiffiffi pffiffiffiffiffiffiffiffiffiffiffiffiffiffiffiffiffiffiffiffiffiffiffiffiffi
(b) c a2 þ b2  ab þ a b2 þ c2  bc  b a2 þ c2 þ ac, where a, b, c > 0.
1.1.31. Let the bisectors (lines) of angles A, B, C intersect the circumcircle
of triangle ABC at points A1, B1, C1, respectively. Prove that
AA1 þ BB1 þ CC1 > AB þ BC þ AC.
1.1.32. Consider a triangle ABC, such that AB > AC and BM, CN are its medians.
Prove that 12 ðAB  ACÞ < BM  CN < 32 ðAB  ACÞ.

ABAC ¼ 4  BMþCN .
Hint Prove that BMCN 3 ABþAC

1.1.33. Given that in triangle ABC side AC is the largest side. Prove that for any
point M of plane AM þ CM is not less than BM. When does the equality hold true?
1.1.34. Given that points A1, A2, . . . , An are not on the same line. Let P and Q be
distinct points, such that A1P þ A2P þ . . . þ AnP ¼ A1Q þ A2Q þ . . . þ AnQ ¼ S.
Prove that A1K þ A2K þ . . . þ AnK < S for some point K.
1.1.35. Given a tetrahedron ABCD. Prove that there exists a triangle with sides
AB  CD, AC  BD and AD  CB.
1.1.36. Let a, b, c be the side lengths of some triangle. Prove that
a2 þ2bc
þ bc2þ2ca c2 þ2ab
2

b2 þc2 þa2 þ a2 þb2 > 3.


1.1 Triangle Inequality 21

1.1.37. Let ABCD be a tetrahedron, such that ∠CAB þ ∠ DAC þ ∠ BAD¼


¼ ∠ ABC þ ∠ CBD þ ∠ DBA ¼ 180 . Prove that CD  AB.
1.1.38. Given that the acute angle of a parallelogram is equal to α, m and n are its
diagonals, m > n. Prove that ctg α2  mn.
Hint Let ABCD be a parallelogram, such that ∠BAD ¼ α. Let O be the
circumradius of triangle ABD. Let A1 be the midpoint of arc BAD. As α < π2,
point O is on the segment A1K, where K is the intersection point of diagonals AC
0
and BD. We have that A1K ¼ A1O þ OK ¼ AO þ OK  AK. Let A be on the segment
0
A1K and A K ¼ AK, then ∠BA0 K  α2. Therefore ctg α2  ctg ∠BA0 K ¼ mn (AK > BK,
as the circle with diameter AC contains points B and D).
1.1.39. On sides AB and BC of triangle ABC are taken points D and F, respectively.
Let E be the midpoint of segment DF. Prove that AD þ FC  AE þ EC.
1.1.40. (a) A convex quadrilateral MNPQ is inside a convex quadrilateral ABCD.
Line MP intersects the sides of quadrilateral ABCD at points K and L. Prove that the
sum of the distances from one of these points to the vertices of the external
quadrilateral is greater than the sum of the distances to the vertices of the inner
quadrilateral.
(b) Let four points be marked inside of a convex quadrilateral. Prove that on one of
the sides of the quadrilateral there is a point, such that the sum of the distances
from that point to the vertices of the quadrilateral is greater than the sum of the
distances from that point to the marked points.
1.1.41. Let points H, I, K, M, O be the midpoints of sides AB, BC, CD, DE, EA of a
convex pentagon, respectively. Prove that the length of a closed polyline HKOIMH
is less than the length of polyline ACEBDA.
Hint Prove that HK  BCþAD
2 < ACþBD
2 :
1.1.42. Let on the side CD of a parallelogram ABCD be constructed an equilateral
triangle CDE. Let X be an arbitrary point on a plane. Prove that
XA þ XB þ AD  XE.
0
!
!
Hint Let X be such a point that XX0 ¼ AD . According to problem 1.1.14а, we
0 0 0 0 0 0
have that CX þ X D  EX . Thus, it follows that XA þ XB þ AD ¼ X C þ X D þ XX
0 0
 X E þ XX  XE.
1.1.43. Given circles ω(0, r) and ω1(0, r1). Let quadrilateral ABCD be inscribed in
circle ω and the rays AB, BC, CD, DA intersect circle ω1 at points A1, B1, C1, D1,
respectively (r1 > r). Prove that
(a) A1 B1 þ B1 C1 þ C1 D1 þ D1 A1  rr1 ðAB þ BC þ CD þ DAÞ,
2
(b) SA1 B1 C1 D1  rr12 SABCD .
22 1 Theorem on the Length of the Broken Line

Hint
(a) See problem 1.1.14а.
(b) Note that SD1 AA1  SBA1 B1  SCB1 C1  SDD1 C1 ¼ ðr 1 2  r 2 Þsin 2 ∠A  sin 2 ∠B and
pffiffiffiffiffiffiffiffiffiffiffiffiffiffiffiffiffiffiffiffiffiffiffiffiffiffiffiffiffiffiffiffi
SABCD  ACBD
2  2r 2 sin ∠A  sin ∠B  2r 2 sin ∠A  sin ∠B.
1.1.44. Given a quadrilateral ABCD, such that ∠A  ∠ D and ∠B þ ∠ D  180 .
Prove that AC þ CD  AB þ BD.
Hint Let C1 be the intersection point of line AC and a circle passing through points
A, B, D.
Prove that C1 is on segment AC, and see problem 1.1.6b.
1.1.45. Prove that if a1, a2, . . . , an (n  3) are positive numbers, such that
 2 2  
a1 þ a22 þ ::: þ a2n > ðn  1Þ a41 þ a42 þ ::: þ a4n , then one can construct a tri-
angle with sides ai, aj, ak for any 1  i < j < k  n.
2
Hint If m > 3 and a2i1 þ ::: þ a2im > ðm  1Þ a4i1 þ ::: þ a4im , then ðm  2Þ
2
a4im  2a2im a2i1 þ ::: þ a2im1 þ ðm  1Þ a4i1 þ ::: þ a4im1  a2i1 þ ::: þ a2im1

< 0:

Therefore, D > 0 or
2
a2i1 þ ::: þ a2im1 > ðm  1Þðm  2Þ a4i1 þ ::: þ a4im1
2 2 :
ðm  2Þ a2i1 þ ::: þ a2im1 ; a2i1 þ ::: þ a2im1 > ðm  2Þ a4i1 þ ::: þ a4im1

1.1.46. Let a, b, c be side lengths of some triangle. Prove that


bþca þ cþab þ aþbc  3.
a b c


a
Hint We have that bþca ¼ 12 cþab
bþca þ aþbc
bþca .

1.1.47. Consider a triangle ABC.


(a) Prove that for any point M on a plane AM sin ∠ A  BM sin ∠ B þ CM sin ∠ C.
(b) Let A1, B1, C1 be points on sides BC, AC, AB respectively, such that angles of
triangle A1B1C1 are equal to α, β, γ. Prove that AA1 sin α þ BB1 sin β þ
CC1 sin γ < BC sin α þ AC sin β þ AB sin γ.
Hint See problem 1.1.14a.
1.1.48. Given n points A1, A2, . . . , An and a circle of radius 1. Prove that one can
choose a point M on this circle, such that MA1 þ MA2 þ . . . þ MAn  n.
1.1.49. Let d be the smallest of the distances between the skew edges of tetrahedron
ABCD and h be the smallest of its altitudes. Prove that 2d > h.
1.1 Triangle Inequality 23

Hint Let d ¼ MN, where M 2 BD and N 2 AC. As BM þ DM  BD, then without


loss of generality one can assume that BM  BD
2 , MH ⊥ (ABC) H 2 (ABC). Thus, it
follows that d > MH  h2.
1.1.50. From two points A and B, the distance between them is equal to d km, are
simultaneously observing during one second after the aircraft flying in a straight line at
a constant speed. From point A the observer reports that the plane has moved during
that second by angle α, and from point B the observer reports that the plane has moved
by angle β (α and β are acute angles). What could be the lowest speed of the plane?
Hint Let during 1 second of observation the plane moved from point C to point
α
D and M be the midpoint of the segment CD. Prove that AM  CD 2 ctg 2 and
β
 α β

BM  2 ctg 2. Then d ¼ AB  2 ctg 2 þ ctg 2 . Thus, CD  ctg αþctg β.
CD CD 2d
2 2

1.1.51. Let O be the circumcenter of triangle ABC. On sides AB and BC are given
points M and N, respectively, such that 2 ∠ MON ¼ ∠ AOC. Prove that
NB þ BM þ MN  AC.
Hint See problem 1.1.14d.
1.1.52. The sum of the distances from point M to the two neighboring vertices of the
square is equal to a. What is the largest value of the sum of the distances from point
M to the other two vertices of the square?
Hint See problem 1.1.14а.
1.1.53. Given that the perimeter of a convex quadrilateral is equal to 2004 and one
of its diagonals is equal to 1001. Can the second diagonal be equal to 1? Can the
second diagonal be equal to 2? Can the second diagonal be equal to 1001?
1.1.54. Let ABC be a triangle. Prove that
(a) maa2 þ mbb2 þ mcc2  mcab þ mabc þ mbac.
Hint According to problem 1.1.14а, we have that a2  23 ma  2c  m3b þ b2  m3c .
(b) ma mb þ mb mc þ mc ma  2p2  34 ðab þ bc þ acÞ.
Hint According to problem 1.1.14а, we have that ma mb  c  2c þ a2  b2.
(c) ma þ mb þ mc þ min (a, b, c)  la þ lb þ lc þ max (a, b, c).
   
Hint We have that ma  la  a2  ðp  bÞ ¼ bc 
2 :

1.1.55. Let A, X, D be points on a line, such that X is in between A and D. Let point
B be such that ∠ABX  120 and point C be in between B and X. Prove that
pffiffiffi
2AD  3ðAB þ BC þ CDÞ.
Hint We have that AX2  AB2 þ BX2 þ AB  BX  34 ðAB þ BXÞ2 .
1.1.56. Let K and L be points on side AB of triangle ABC, such that AK ¼ KL ¼ LB.
Prove that
24 1 Theorem on the Length of the Broken Line

(a) if AC < BC, then CK < CL.


(b) ∠KCL > min (∠ACK, ∠BCL).
Hint
(b) Let AC  BC, then CK < CB. Consider a parallelogram KCBD.
1.1.57. Let the diagonals of a convex quadrilateral ABCD intersect at point P.
Prove that BC þ AD  BD sin β þ AC sin γ, where β, γ > 0 and β þ γ ¼ ∠ BPC.
Hint Consider a point D1, such that quadrilateral BCD1D is a parallelogram. Then
DD1¼BC. Note that BC þ AD  AD1  BD sin β þ AC sin γ.
1.1.58. Prove that if a, b, c are the side lengths of some triangle, then
1 1 1
(a) aþb , bþc , aþc ,
a b c
(b) 1þa , 1þb , 1þc ,
(c) ð1þaÞð1þbÞ
aþbþ2ab ,
ð1þbÞð1þcÞ ð1þaÞð1þcÞ
bþcþ2bc , aþcþ2ac , are also side lengths of some triangle.
Hint
1
(a) aþb þ bþc
1
> aþbþc
2
,
a
(b) 1þa þ 1þb
b
> 1þaþb
aþb
.
1.1.59. Let ABCD be a tetrahedron. Prove that AB  CD þ BC  AD  S, where S is
the total surface area of the tetrahedron.
Hint Prove that AB  CD þ BC  AD  2SABC þ 2SADC.
Consider the layout of facets ABC, ADC and see problem 1.1.14а.
1.1.60. Prove that any polygon having a perimeter equal to 2a can be convered by a
square whose diagonal is equal to a.
Hint Inscribe the polygon into a square and use problem 1.1.10.
1.1.61. Given a triangle ABC and a point M. Let G be the intersection point of the
medians of triangle ABC. Prove that MA þ MB þ MC þ 3MG  2(MA1 þ MB1 þ
MC1), where points A1, B1, C1 are the midpoints of sides BC, AC, AB.
Hint See problem 1.1.14f.
1.1.62. Let A1A2 . . . An be a convex hexagon and M, N be given distinct points
inside of it. Prove that max NA1 ; NA2 ; :::; NAn > 1:
MA1 MA2 MAn

Hint See problem 1.1.8a.

1.2 Theorem on the Length of the Broken Line

If A1, A2, . . . , An are distinct points, then A1An  A1A2 þ A2A3 þ . . . þ An  1An.
Note that the equality holds true only if simultaneously hold true the following
conditions: point Ai is on the segment Ai  1Ai þ 1 for all i ¼ 2, 3, . . . , n  1.
1.2 Theorem on the Length of the Broken Line 25

1.2.1. Given two circles with radiuses R1 and R2, such that for the distance between
their centers it holds true the following inequality d > R1 þ R2. Prove that
d  R1  R2  XY  d þ R1 þ R2, where X and Y are arbitrary points of these
two circles.
1.2.2. Prove that in any polygon there are at least two sides a and b, such that
1  ba < 2.
1.2.3. Given a convex hexagon ABCDEF, such that ∠A  90 , ∠ D  90 . Prove
that BC þ CE þ EF þ FB  2AD.
1.2.4. Given the points A(a, 0), B(0, b), C(c, d) and O(0, 0) on a coordinate plane.
Prove that AB þ BC þ CA  2CO.
1.2.5. Prove the following inequalities:
qffiffiffiffiffiffiffiffiffiffiffiffiffiffiffi qffiffiffiffiffiffiffiffiffiffiffiffiffiffiffi qffiffiffiffiffiffiffiffiffiffiffiffiffiffiffiffiffiffiffiffiffiffiffiffiffiffiffiffiffiffiffiffiffiffiffiffiffiffiffiffiffiffiffiffiffiffiffiffiffiffiffiffiffiffiffiffiffiffiffiffiffiffiffiffiffiffi
(a) a21 þ b21 þ ::: þ a2n þ b2n  ða1 þ ::: þ an Þ2 þ ðb1 þ ::: þ bn Þ2 .
qffiffiffiffiffiffiffiffiffiffiffiffiffiffiffiffiffiffiffiffiffiffiffiffiffiffiffiffiffiffiffiffi qffiffiffiffiffiffiffiffiffiffiffiffiffiffiffiffiffiffiffiffiffiffiffiffiffiffiffiffiffiffiffiffi
a21 þ ka1 b1 þ b21 þ ::: þ a2n þ kan bn þ b2n 
(b) qffiffiffiffiffiffiffiffiffiffiffiffiffiffiffiffiffiffiffiffiffiffiffiffiffiffiffiffiffiffiffiffiffiffiffiffiffiffiffiffiffiffiffiffiffiffiffiffiffiffiffiffiffiffiffiffiffiffiffiffiffiffiffiffiffiffiffiffiffiffiffiffiffiffiffiffiffiffiffiffiffiffiffiffiffiffiffiffiffiffiffiffiffiffiffiffiffiffiffiffiffiffiffiffiffiffiffiffiffiffiffiffiffiffiffiffiffiffiffiffiffiffiffiffiffiffiffiffiffiffi
 ða1 þ ::: þ an Þ2 þ kða1 þ ::: þ an Þðb1 þ ::: þ bn Þ þ ðb1 þ ::: þ bn Þ2 ,
where |k|  2.
(c) ða þ b þ cÞða  b þ cÞ þ ða  b þ cÞða þ b  cÞ þ ða þ b  cÞða þ b þ cÞ
pffiffiffiffiffiffiffi pffiffiffi pffiffiffi pffiffiffi
 abcð a þ b þ cÞ, where a, b, c > 0:
1.2.6. On the legs of a right-angled triangle with hypotenuse c and acute angle α are
chosen points P and Q. Let PK and QH be perpendiculars to the hypotenuse drawn
from points P and Q. Prove that KP þ PQ þ QH  c sin 2α.
1.2.7. Prove that from all triangles inscribed in an acute triangle ABC, the smallest
perimeter has the orthic triangle (the triangle whose vertices are the endpoints of the
altitudes of triangle ABC).
1.2.8. Given a triangle ABC. Prove that ∠ACB  120 is a necessary and sufficient
condition for the following inequality to hold true MA þ MB þ MC  AC þ BC, for
any point M of plane ABC.
1.2.9. (a) Prove that MAA11MA
A2
2
þ ::: þ MAAn1 MAn  MA1 MAn , where M, A1, . . . , An are
n1 An A1 An

distinct points and n  3. When does the equality hold true?


(b) Given a square ABCD inscribed in a circle. Let M be a point on the minor arc
^
A B : Prove that
pffiffiffi
1. MC  MD  3 3MA  MB,
 pffiffiffi
2. MC  MD  3 þ 2 2 MA  MB.
1.2.10. On sides AB and CD of a convex quadrilateral ABCD are constructed
externally equilateral triangles ABE and CDF. Prove that for any points M and
N it holds true AM þ BM þ MN þ CN þ DN  EF.
26 1 Theorem on the Length of the Broken Line

1.2.11. Let ABCDEF be a convex hexagon such that AB ¼ BC ¼ CD,


DE ¼ EF ¼ FA and ∠BCD ¼ ∠ EFA ¼ 60 . Let G and H be two arbitrary points.
Prove that AG þ GB þ GH þ DH þ HE  CF.
1.2.12. (a) Let point M be the midpoint of side BC of a convex quadrilateral ABCD.
Given that ∠AMD ¼ 120 . Prove that AB þ 12 BC þ CD  AD.
(b) Let points A1, B1, C1, D1, E1, F1 be the midpoints of sides AB, BC, CD, DE, EF,
FA of a convex hexagon ABCDEF, respectively. Given that all angles of
pffiffi
hexagon A1B1C1D1E1F1 are equal. Prove that p  2 3 3 p1 , where p and p1 are
the perimeters of hexagons ABCDEF and A1B1C1D1E1F1, respectively.
1.2.13. Prove that any polygon with perimeter 2a can be covered by a circle whose
diameter is equal to a.
1.2.14. Prove that the sum of the planar angles at a vertex of a tetrahedron:
(a) is equal to 180 ,
(b) is greater than 180 , then any lateral edge of the tetrahedron is less than half-
perimeter of its base.
1.2.15. Prove that any hexagonal cross-section plane of the unit cube has a perim-
pffiffiffi
eter greater than or equal to 3 2.
1.2.16. Given that a rectangle ABCD is inside of a convex polygon A1 . . . An. Prove
that

minðAB; BCÞ þ AB þ BC < A1 A2 þ A2 A3 þ ::: þ An1 An :

1.2.17. Inside of the convex polygon with perimeter P are given two rectangles with
the perimeter with sides a, b and c, d which do not have any common interior point.
Prove that min(a, b) þ a þ b þ min (c, d ) þ c þ d < P.
1.2.18. (а) Let points A and B be outside of circle ω. Through points A and B are
drawn tangents AM and BN to circle ω. Prove that if segment AB intersects the
circle ω, then AB > AM þ BN, and if it does not intersect, then AB  AM þ BN.
(b) Let points A, B, C be outside of circle ω, such that ω intersects segments AC, BC
and does not intersect segment AB. Through points A, B, C are drawn tangents
AM, BN, CK to circle ω. Prove that AM  BC þ BN  AC > CK  AB.

Solutions

1.2.1. Let point X belong to a circle with center O1 and radius R1.
Let point Y belong to a circle with center O2 and radius R2.
For the broken line O1XYO2 we have that R1 þ XY þ R2  d.
Thus, it follows that XY  d  R1  R2.
1.2 Theorem on the Length of the Broken Line 27

Similarly, for the broken line XO1O2Y we obtain that XY  d þ R1 þ R2.


This ends the proof.
1.2.2. Let a1  a2  . . .  an be lengths of the sides of a given polygon.
We proceed with the proof by a contradiction argument. Assume that such two
sides do not exist; then a2  2a1, a3  2a2, . . . , an  2an  1.
We have that a1 þ a2 þ . . . þ an  1 > an.
Therefore, a1 > (...((an  an  1)  an  2)  ...  a1)  (...(an  1  an  2)  ... 
a1) (...(an  2  an  3)  ...  a1)  . . .  a1. Hence a1 > a1. This leads to a
contradiction.
1.2.3. Let points M, N, P be the midpoints of the segments BF, BE, CE, respectively.
As ∠A  90 , then point A is in the circle with diameter BF.
Therefore, AM  BF 2 . In a similar way, we obtain that DP  2 .
CE

Thus, it follows that 2AD  2AM þ 2MN þ 2NP þ 2DP  BF þ EF þ BC þ CE.


This ends the proof.
   
1.2.4. Let us consider points M a2; b2 and N aþc 2 ; 2 . As
d
qffiffiffiffiffiffiffiffiffi qffiffiffiffiffiffiffiffiffiffiffiffiffiffiffiffi2
a2 þb2 c2 þðbd Þ
OM þ MN þ NC  OC and OM ¼ 4 ¼ 2 , MN ¼
AB
¼ BC
2 and
qffiffiffiffiffiffiffiffiffiffiffiffiffiffiffiffi
2
4

NC ¼ ðac4Þ þd ¼ AC
2

2 , then AB þ BC þ AC  2OC.
See problem 1.1.14d.
1.2.5. (a) The proof follows straightforwardly from 1.2.5b for k ¼ 0.
qffiffiffiffiffiffiffiffiffiffiffiffi
2
(b) Let us consider the points Ai 1  k4 ða1 þ ::: þ ai Þ; b1 þ ::: þ bi þ 2kða1 þ :::
þai ÞÞ, i ¼ 1, . . . , n.
Note that
vffiffiffiffiffiffiffiffiffiffiffiffiffiffiffiffiffiffiffiffiffiffiffiffiffiffiffiffiffiffiffiffiffiffiffiffiffiffiffiffiffiffiffiffiffiffiffiffiffiffiffiffiffiffiffiffiffiffiffiffiffiffiffiffiffiffiffiffiffiffiffiffiffiffiffi
u0sffiffiffiffiffiffiffiffiffiffiffiffiffi 12
u  2
u k 2
k
Ai Aiþ1 ¼ t@ 1  aiþ1 A þ biþ1 þ aiþ1
4 2
qffiffiffiffiffiffiffiffiffiffiffiffiffiffiffiffiffiffiffiffiffiffiffiffiffiffiffiffiffiffiffiffiffiffiffiffiffiffiffiffiffiffiffiffiffiffi
¼ a2iþ1 þ kaiþ1 biþ1 þ b2iþ1 , i ¼ 0, :::, n  1,

where A0(0; 0) and


qffiffiffiffiffiffiffiffiffiffiffiffiffiffiffiffiffiffiffiffiffiffiffiffiffiffiffiffiffiffiffiffiffiffiffiffiffiffiffiffiffiffiffiffiffiffiffiffiffiffiffiffiffiffiffiffiffiffiffiffiffiffiffiffiffiffiffiffiffiffiffiffiffiffiffiffiffiffiffiffiffiffiffiffiffiffiffiffiffiffiffiffiffiffiffiffiffiffiffiffiffiffiffiffiffiffiffiffiffiffiffiffiffiffiffiffiffiffiffiffiffiffiffiffiffiffiffiffiffiffi
Ao An ¼ ða1 þ ::: þ an Þ2 þ kða1 þ ::: þ an Þðb1 þ ::: þ bn Þ þ ðb1 þ ::: þ bn Þ2 . As
A0A1 þ A1A2 þ . . . þ An  1An  A0An, then
qffiffiffiffiffiffiffiffiffiffiffiffiffiffiffiffiffiffiffiffiffiffiffiffiffiffiffiffiffiffiffiffi qffiffiffiffiffiffiffiffiffiffiffiffiffiffiffiffiffiffiffiffiffiffiffiffiffiffiffiffiffiffiffiffi
a21 þ ka1 b1 þ b21 þ ::: þ a2n þ kan bn þ b2n 
qffiffiffiffiffiffiffiffiffiffiffiffiffiffiffiffiffiffiffiffiffiffiffiffiffiffiffiffiffiffiffiffiffiffiffiffiffiffiffiffiffiffiffiffiffiffiffiffiffiffiffiffiffiffiffiffiffiffiffiffiffiffiffiffiffiffiffiffiffiffiffiffiffiffiffiffiffiffiffiffiffiffiffiffiffiffiffiffiffiffiffiffiffiffiffiffiffiffiffiffiffiffiffiffiffiffiffiffiffiffiffiffiffiffiffiffiffiffiffiffiffiffiffiffiffiffiffiffiffiffi
 ða1 þ ::: þ an Þ2 þ kða1 þ ::: þ an Þðb1 þ ::: þ bn Þ þ ðb1 þ ::: þ bn Þ2 :

2 ,b ¼ 2 ,c ¼ 2 .
(c) Let a þ b þ c ¼ m, a  b þ c ¼ n, a þ b  c ¼ k, then a ¼ nþk mþk mþn

Therefore, if mn þ nk þ mk  0, then
28 1 Theorem on the Length of the Broken Line

qffiffiffiffiffiffiffiffiffiffiffiffiffiffiffiffiffiffiffiffiffiffiffiffiffiffiffiffiffiffiffiffiffiffiffiffiffiffiffiffiffiffiffiffiffiffiffiffiffiffiffiffiffiffiffiffiffiffiffiffiffiffiffiffiffiffiffiffiffiffiffiffiffiffiffiffiffiffiffiffiffiffi !
pffiffiffiffiffiffiffi pffiffiffi pffiffiffi pffiffiffi 1  pffiffiffiffiffiffiffiffiffiffiffiffiffiffiffiffiffiffiffiffiffiffiffiffiffiffiffiffi2
abcð a þ b þ cÞ ¼ ððn þ kÞmÞ2 þ ðn þ kÞ mn þ nk þ mk þ
4
qffiffiffiffiffiffiffiffiffiffiffiffiffiffiffiffiffiffiffiffiffiffiffiffiffiffiffiffiffiffiffiffiffiffiffiffiffiffiffiffiffiffiffiffiffiffiffiffiffiffiffiffiffiffiffiffiffiffiffiffiffiffiffiffiffiffiffiffiffiffiffiffiffiffiffiffiffiffiffiffiffiffi
 pffiffiffiffiffiffiffiffiffiffiffiffiffiffiffiffiffiffiffiffiffiffiffiffiffiffiffiffi2ffi
þ ððm þ kÞnÞ2 þ ðm þ kÞ mn þ nk þ mk
qffiffiffiffiffiffiffiffiffiffiffiffiffiffiffiffiffiffiffiffiffiffiffiffiffiffiffiffiffiffiffiffiffiffiffiffiffiffiffiffiffiffiffiffiffiffiffiffiffiffiffiffiffiffiffiffiffiffiffiffiffiffiffiffiffiffiffiffiffiffiffiffiffiffiffiffiffiffiffiffiffiffiffi
 pffiffiffiffiffiffiffiffiffiffiffiffiffiffiffiffiffiffiffiffiffiffiffiffiffiffiffiffi2
þ ððn þ mÞkÞ2 þ ðn þ mÞ mn þ nk þ mk 
qffiffiffiffiffiffiffiffiffiffiffiffiffiffiffiffiffiffiffiffiffiffiffiffiffiffiffiffiffiffiffiffiffiffiffiffiffiffiffiffiffiffiffiffiffiffiffiffiffiffiffiffiffiffiffiffiffiffiffiffiffiffiffiffiffiffiffiffiffiffiffiffiffiffiffiffiffiffiffiffiffiffiffiffiffiffiffiffiffiffiffiffiffiffiffiffiffiffiffiffiffiffi
1
 4ðmn þ nk þ mkÞ2 þ 4ðm þ n þ kÞ2 ðmn þ nk þ mkÞ 
4
qffiffiffiffiffiffiffiffiffiffiffiffiffiffiffiffiffiffiffiffiffiffiffiffiffiffiffiffiffiffiffiffiffiffiffiffiffiffiffiffiffiffiffiffiffiffiffiffiffiffiffiffiffiffiffiffiffiffiffiffiffiffiffiffiffiffiffiffiffiffiffiffiffiffiffiffiffiffiffiffi
1
 4ðmn þ nk þ mkÞ2 þ 12ðmn þ nk þ mkÞ2
4
¼ mn þ nk þ mk,

according to problem 1.2.5а, and the following inequality (m þ n þ k)2


 3mn þ 3nk þ 3mk for mn þ nk þ mk < 0. The proof is straightforward.
1.2.6. Let ABC be a right-angled triangle. Given that ∠C ¼ 90 and points M, N,
R are the midpoints of segments PQ, QK, KH, respectively.
Then CM þ MN þ NR CR  h, where h is the altitude of triangle ABC drawn
from vertex C.
Note that h ¼ c  cos α  sin α ¼ 2c sin 2α, CM ¼ PQ
2 , MN ¼ 2 , NR ¼ 2 . Hence,
PK QH

it follows that KP þ PQ þ QH  c  sin 2α.


1.2.7. Let points M, N, P belong to sides AB, BC, AC, respectively. Let us draw from
point P perpendiculars PE and PF to sides AB and BC. As points E and F belong to
a circle whose diameter is BP, thus using the law of sines we obtain that
EF ¼ BP  sin ∠ B. If Q, T are the midpoints of segments MP and NP, then
EF  EQ þ QT þ TF ¼ MP 2 þ 2 þ 2.
MN NP

Thus, MP þ MN þ NP  2BP  sin ∠ B.


Let AA1, BB1, CC1 be the altitudes of triangle ABC; see Figure 1.12 and
B1E1 ⊥ AB, B1F1 ⊥ BC. As points A, C1, A1, C are on a circle whose diameter is

Figure 1.12
1.2 Theorem on the Length of the Broken Line 29

AC, thus ∠BC1A1 ¼ ∠ C. In a similar way, we deduce that ∠AC1B1 ¼ ∠ C and


∠BA1C1 ¼ ∠ CA1B1 ¼ ∠ A.
Let Q1, T1 be the midpoints of B1C1, A1B1, respectively. Then ∠E1Q1C1 ¼
2 ∠ E1B1Q1 ¼ 2 ∠ B1C1C ¼ ∠ B1C1A1.
Therefore E1Q1 k A1C1 and F1T1||C1A1.
Hence
E1 F1 ¼ E1 Q1 þ Q1 T 1 þ T 1 F1 ¼ 12 ðB1 C1 þ A1 C1 þ A1 B1 Þ ¼ BB1 sin ∠B.
Thus MP þ MN þ NP  2BP  sin ∠ B  2BB1  sin ∠ B ¼ B1C1 þ A1C1 þ A1B1.
Note that if points P and B1 do not coincide, then MP þ MN þ NP >
B1C1 þ A1C1 þ A1B1. Therefore, MP þ MN þ NP ¼ B1C1 þ A1C1 þ A1B1 if and
only if points M, N, P coincide with points C1, A1, B1, respectively.
Second Proof Let points M, N, P belong to sides AB, BC, AC, respectively.
According to problem 1.1.14d, for points M, N, P, B, we have that

MN þ NP þ MP  2BP sin ∠ sin B: ð1:16Þ

Let AA1, BB1, CC1 be the altitudes of triangle ABC. As BP  BB1, then from
(1.16) we obtain that

MN þ NP þ MP  2BB1 sin ∠B: ð1:17Þ

Let the altitudes AA1, BB1, CC1 intersect at point H. Note that

2BB1 sin ∠B ¼ BB1 sin ∠B þ BB1 sin ∠B ¼ BB1 sin ∠B þ CC1 sin ∠C ¼¼ BH sin ∠B
þ CH sin ∠C þ B1 H sin ∠B þ C1 H sin ∠C ¼ A1 C1
þ A1 B1 þ þB1 H cos ∠C1 B1 H þ C1 H cos ∠B1 C1 H
¼ A1 C1 þ A1 B1 þ C1 B1 :
ð1:18Þ

From (1.17) and (1.18), we obtain that

MN þ NP þ MP > A1 C1 þ A1 B1 þ C1 B1 :

Note that if points P and B1 do not coincide, then BP > BB1.


Therefore, MN þ NP þ MP > A1C1 þ A1B1 þ C1B1.
Hence, MP þ MN þ NP ¼ B1C1 þ A1C1 þ A1B1 if and only if points M, N,
P coincide with points C1, A1, B1, respectively.
This ends the proof.
1.2.8. Let the rotation by angle 180  ∠ C and the rotation center C (Figure 1.13)
point B moves to point B1, point M goes to point M1.
We have that CB ¼ CB1. Therefore AC þ CB ¼ AB1.
If ∠C  120 , then ∠MCM1 ¼ 180  ∠ C  60 .
Hence CM ¼ CM1, then CM  MM1. Thus, it follows that
30 1 Theorem on the Length of the Broken Line

Figure 1.13

AM þ CM þ BM ¼ AM þ CM þ B1 M1  AM þ MM1 þ B1 M1  AB1
¼ AC þ CB:

If for any point M of plane ABC it holds true that AM þ CM þ BM  AC þ CB,


then in the case when M coincides with A, we obtain that AB  CB. On the other
hand, if M coincides with B, then AB  AC. Therefore, ∠C  ∠ A, ∠C  ∠ B.
Now, assume that ∠C < 120 . Then inside angle CAB and outside triangle ABC
one can find a point B2, such that ∠B2CB ¼ ∠ B2BC ¼ 60 . Let M0 be the second
intersection point of line AB2 and circumcircle of triangle CBB2. Then
AM0 þ BM0 þ CM0 ¼ AM0 þ M0B2 ¼ AB2 < AC þ CB2 ¼ AC þ CB. (See the proof
of problem 1.2.9.). Hence, we have obtained that there exists a point M such that
AM þ CM þ BM < AC þ CB. This leads to a contradiction. Thus ∠C  120 .
Remark If max(∠A; ∠B; ∠C) < 120 , then

AM þ BM þ CM 
 AM0 þ BM0 þ CM0 :

CB2
Indeed, according to problem 1.2.9a, we have that MCMB2
þ MBMB
B2 B
2
 MCMB
BC
or
MB þ MC  MB2. Therefore MA þ MB þ MC  MA þ MB2  AB2 ¼ AM0 þ M0B2 ¼
AM0 þ BM0 þ CM0.
1.2.9. (а) Let us choose a point Bi on ray MAi, such that MBi ¼ MA
1
i
, i ¼ 1, . . . , n.
AA
Note that Bi Bj ¼ MAiiMA
j
j
.
Indeed, if rays MAi and MAj are not on the same line, we obtain that triangle
MAiAj is similar to triangle MBiBj.
BB
Hence, it follows that Aii Ajj ¼ MB
MAi ¼ MAi MAj .
i 1

AA
Therefore, Bi Bj ¼ MAiiMA
j
j
.
If rays MAi and MAj are on the same line, then
   

   1 1  MA MA Ai Aj
Bi Bj ¼ MBi MBj  ¼  ¼ i j
 ¼ :
MAi MAj MAi  MA MAi  MAj
1.2 Theorem on the Length of the Broken Line 31

Then, the inequality we need to prove is equivalent to the following inequality


B1B2 þ B2B3 þ . . . þ Bn  1Bn  B1Bn. The equality holds true if B1, B2, . . . , Bn
are on the same line (in the mentioned order).
If M 2 B1Bn, then A1, A2, . . . , An 2 B1Bn. It is clear that points M, A1, A2, . . . ,
An are on the same line, in the following order: M, A1, A2, . . . , An or M, An, An  1,
. . . , A1 or Ak, Ak  1, . . . , A1, M, An, An  1, . . . , Ak þ 1 or Ak þ 1, . . . , An, M, A1,
. . . , Ak, where k is an arbitrary positive integer not greater than n.
If M 2= B1Bn, then

∠A1 A2 A3 þ ∠A1 MA3 ¼ ∠A1 A2 M þ ∠A3 A2 M þ ∠A1 MA3 ¼



¼ ∠MB1 B3 þ ∠B1 B3 M þ ∠B1 MB3 ¼ 180 :

Therefore, points M, A1, A2, A3 are on the same circle. In a similar way, one
obtains that points M, A2, A3, A4, ...,M, An  2, An  1, An are on the same circle. Then,
points M, A1, A2, . . . , An are on the same circle (in the given order).
1
(b) 1. We have that MAMB ¼ MBMC
1
þ MCMD
1
þ MDMA
1

(see the proof of problem 1.2.9а).


qffiffiffiffiffiffiffiffiffiffiffiffiffiffiffiffiffiffiffiffiffiffiffiffiffiffi
1
Hence, using the Cauchy’s inequality, we obtain that MAMB  3 3 MAMBMC 1
2
MD2
.
pffiffiffi
Therefore, MC  MD  3 3MA  MB.
(c) 2. Note that ∠AMB ¼ 135 , ∠CMD ¼ 45 . Thus, it follows that MCMD
MAMB ¼ SMAB ¼
SMCD

ρðM;CDÞ
¼ ρðM;AB
ρðM;ABÞ
AB
Þ þ 1 is the smallest if ρ(M, AB) is the largest, i.e. M is the
midpoint of arc AB.
pffiffiffi
MAMB  ctg 8 ¼ 3 þ 2 2.
Therefore, MCMD 2π

1.2.10. For points M, A, B, E and N, C, D, F using problem 1.2.9а we obtain that


AM þ BM  ME and CN þ DN  NF.
Thus, it follows that

AM þ MB þ MN þ CN þ DN  ME þ MN þ NF  EF:

This ends the proof.


0 0
1.2.11. Let points G and H be symmetric to points G and H with respect to line BE,
respectively. As AB ¼ BD and AE ¼ ED, then points A and D are symmetric with
0 0 0 0 0
respect to the line BE. Using problem 1.2.10, we obtain that BG þ G D þ G H þ H
0
E þ H A  CF.
0 0 0 0 0 0
Now, note that BG þ G D þ G H þ H E þ H A ¼ BG þ GA þ HG þ HE þ HD.
Therefore, AG þ BG þ HG þ HE þ HD  CF.
0 0
1.2.12. (a) Let points B and C be symmetric to points B and C with respect to lines
0 0 0 0
AM and DM, respectively. Note that BM ¼ MB , CM ¼ C M, ∠B MC ¼ 120 
(∠BMA þ ∠ CMD) ¼ 60 .
Therefore B0 C0 ¼ B0 M ¼ BM ¼ BC 2.
0 0 0 0
We have that AB þ B C þ C D  AD or AB þ BC 2 þ CD  AD.
32 1 Theorem on the Length of the Broken Line

(b) Note that ∠A1B1C1 ¼ ∠ B1C1D1 ¼ . . . ¼ ∠ F1A1B1 ¼ 120 .

2 þ 2 þ 2  A1 C1 :
According to problem 1.2.12а, we have that AB BC CD

In a similar way we obtain that 2 þ 2 þ 2  B1 D1 , :::, AF


BC CD DE
2 þ 2 þ 2  F1 B1 :
AB BC

Summing up all these inequalities, we deduce that (see problem 1.2.5b)


3
p  A1 C1 þ B1 D1 þ . .. þ F1 B1 ¼
2
pffiffiffiffiffiffiffiffiffiffiffiffiffiffiffiffiffiffiffiffiffiffiffiffiffiffiffiffiffiffiffiffiffiffiffiffiffiffiffiffiffiffiffiffiffiffiffiffiffiffiffiffiffiffiffiffi pffiffiffiffiffiffiffiffiffiffiffiffiffiffiffiffiffiffiffiffiffiffiffiffiffiffiffiffiffiffiffiffiffiffiffiffiffiffiffiffiffiffiffiffiffiffiffiffiffiffiffiffiffiffiffiffiffiffi
¼ A 1 B 1 2 þ B 1 C 1 2 þ A 1 B 1  B 1 C 1 þ B 1 C 1 2 þ C 1 D1 2 þ B 1 C 1  C 1 D1
pffiffiffiffiffiffiffiffiffiffiffiffiffiffiffiffiffiffiffiffiffiffiffiffiffiffiffiffiffiffiffiffiffiffiffiffiffiffiffiffiffiffiffiffiffiffiffiffiffiffiffiffiffiffiffiffi
þ ::: þ A1 F1 2 þ A1 B1 2 þ A1 F1  A1 B1 ¼
qffiffiffiffiffiffiffiffiffiffiffiffiffiffiffiffiffiffiffiffiffiffiffiffiffiffiffiffiffiffiffiffiffiffiffiffiffiffiffiffiffiffiffiffiffiffiffiffiffiffiffiffiffiffiffiffiffiffiffiffiffiffiffiffiffiffiffiffiffiffiffiffiffiffiffiffiffiffiffiffiffiffiffiffiffiffiffiffiffiffiffiffiffiffiffiffiffiffiffiffiffiffiffiffiffiffiffiffiffiffiffiffiffiffiffiffiffiffiffiffiffiffiffiffiffiffiffiffiffiffiffiffiffiffiffiffiffiffiffiffiffiffiffiffiffiffiffiffiffiffiffiffiffiffiffiffiffiffiffiffiffiffiffiffiffiffiffiffiffiffiffiffiffiffiffiffiffiffiffiffiffiffiffiffiffiffiffiffiffiffiffiffiffiffiffiffiffiffiffiffiffiffiffiffiffiffiffiffiffiffiffiffiffiffiffiffiffiffiffiffiffiffiffiffiffiffiffiffi
¼ ðA1 B1 þ B1 C1 þ . . . þ A1 F1 Þ2 þ ðB1 C1 þ C1 D1 þ . . . þ A1 B1 Þ2 þ ðA1 B1 þ B1 C1 þ .. . þ A1 F1 ÞðB1 C1 þ C1 D1 þ . . . þ A1 B1 Þ
pffiffiffi
¼ 3p1 ,

pffiffi
thus, p  2 3 3 p1 :
This ends the proof.
1.2.13. Points A, B are on the sides of a polygon and divide its perimeter to two equal
parts. Note that for any point C, belonging to any side of the polygon, we have that
OC  OM þ MC  ACþBC 2  a2, where O is the center of segment AB (Figure 1.14).
Thus, a circle with radius a2 and center O covers this polygon.
This ends the proof.
1.2.14. (a) Given a tetrahedron SA1A2 . . . An with a vertex S. Consider scanning its
lateral surface, making the “cross section” along the edge SA1 (see Figure 1.15).
Given that the sum of the planar angles at vertex S is equal to 180 . Then
∠A1SA10 ¼ 180 and 2SA1 < A1A2 þ A2A3 þ . . . þ An  1An þ AnA10 ¼ A1A2 þ A2A3
þ . . . þ An  1An þ AnA1.
(b) Let A be the intersection point of line A1S with a side of polygon A1A2 . . . A0n A1,
different from A1 (see the solution of problem 1.2.14а). Hence, if A 2 AkAk þ 1,
(An þ 1  A10 ), then A1A2 þ . . . þ AkA > A1S þ SA, SA þ AAk þ 1 þ . . .
þ AnA10 > SA10 .
Therefore, A1A2 þ . . . þ An  1An þ AnA1 > 2SA1.
1.2.15. Consider the layout of the side surface of the cube (Figures 1.16 and 1.17).
pffiffiffi
Note that AA0 ¼ 3 2 and

Figure 1.14
1.2 Theorem on the Length of the Broken Line 33

Figure 1.15

Figure 1.16

Figure 1.17
34 1 Theorem on the Length of the Broken Line

AB þ BC þ CD þ DE þ EF þ FA ¼ AB þ BC þ CD þ DE þ EF þ FA0  AA0
pffiffiffi
¼ 3 2:

Second Proof We have that

AB þ BC þ CDq þffiffiffiffiffiffiffiffiffiffiffiffiffiffi
DE þ ffiEF þ FA ¼
pffiffiffiffiffiffiffiffiffiffiffiffiffiffiffi pffiffiffiffiffiffiffiffiffiffiffiffiffiffi qffiffiffiffiffiffiffiffiffiffiffiffiffiffiffi pffiffiffiffiffiffiffiffiffiffiffiffiffiffi qffiffiffiffiffiffiffiffiffiffiffiffiffiffi
¼ a1 þ a2 þ b21 þ b22 þ c21 þ c22 þ d21 þ d 22 þ e21 þ e22 þ f 21 þ f 22 
2 2
qffiffiffiffiffiffiffiffiffiffiffiffiffiffiffiffiffiffiffiffiffiffiffiffiffiffiffiffiffiffiffiffiffiffiffiffiffiffiffiffiffiffiffiffiffiffiffiffiffiffiffiffiffiffiffiffiffiffiffiffiffiffiffiffiffiffiffiffiffiffiffiffiffiffiffiffiffiffiffiffiffiffiffiffiffiffiffiffiffiffiffiffiffiffiffiffiffiffiffiffiffiffiffiffiffiffiffiffiffiffiffiffiffiffiffiffiffiffiffiffiffiffiffiffiffiffi
 ða1 þ b1 þ c1 þ d1 þ e1 þ f 1 Þ2 þ ða2 þ b2 þ c2 þ d 2 þ e2 þ f 2 Þ2 ¼ m,

(see problem 1.2.5а).


Note that if a1 þ b1 þ c1 þ d1 þ e1 þ f1 ¼ a, then a2 þ b2 þ c2 þ d2 þ e2 þ f2¼
qffiffiffiffiffiffiffiffiffiffiffiffiffiffiffiffiffiffiffiffiffiffiffiffiffiffiffi pffiffiffiffiffiffiffiffiffiffiffiffiffiffiffiffiffiffiffiffiffiffiffiffiffiffiffiffiffiffiffi qffiffiffiffiffiffiffiffiffiffiffiffiffiffiffiffiffiffiffiffiffiffiffiffiffiffiffiffiffiffi
¼6  a, then m ¼ a2 þ ð6  aÞ2 ¼ 2a2  12a þ 36 ¼ 2ða  3Þ2 þ 18 
pffiffiffi
3 2.
pffiffiffi
Therefore, it follows that AB þ BC þ CD þ DE þ EF þ FA  3 2.
This ends the proof.
1.2.16. Let P ¼ A1A2 þ A2A3 þ . . . þ An  1An.
Consider Figure 1.18.
We have that P > P1 þ P2 þ P3 þ P4 and P1  AB  sin α, P2  BC, P3  CD,
P4  AD  sin (90  α).
Thus, it follows that

P > AB  sin α þ BC þ CD þ AD  cos α 


 minðAB; ADÞð sin α þ cos αÞ þ BC þ CD 
 minðAB; ADÞðsin 2 α þ cos 2 αÞ þ BC þ CD 
¼ minðAB; ADÞ þ BC þ CD:

Hence, we obtain that

A1 A2 þ A2 A3 þ ::: þ An1 An > minðAB; BCÞ þ AB þ BC:

This ends the proof.

Figure 1.18
1.2 Theorem on the Length of the Broken Line 35

1.2.17. Lemma Let convex polygons B1B2 . . . Bp and C1C2 . . . Cm not have any
common interior points. Then, there exists a line l passing through one of the sides
of the polygons and separating these polygons.
Indeed, let us choose a point O inside of the polygon B1B2 . . . Bp, such that O is
not on lines BiCj. Let k be the smallest positive integer, such that the image of the
polygon B1B2 . . . Bp by homothety with the center Oand ratio k has a common point
with the polygon C1C2 . . . Cm.
Then, the following two cases are possible (Figure 1.19a, b).
In both cases the proof of the lemma is straightforward.
The proof of the problem follows from the lemma and problem 1.2.16.
1.2.18. (a) Let segment AB intersect circle ω, CD is tangent to the circle ω and CD||
AB (Figure 1.20a).
We have that AC þ AB þ BD > CP þ PD ¼
¼ AC þ AM þ BD þ BN. Therefore, AB > AM þ BN.
If segment AB does not intersect circle ω (Figure 1.20b), then

AB  AC þ CD þ BD ¼ AC þ CP þ PD þ DB ¼ AC þ CM þ BD þ DN
¼ AM þ BN

In this case, such placement of the points is also possible, as it is shown in


Figure 1.21. Then, we have that AB < BK < BN < BN þ AM.
(c) According to problem 1.2.18а, we have that

Figure 1.19
36 1 Theorem on the Length of the Broken Line

Figure 1.20

Figure 1.21

AM  BC þ BN  AC  AMðBN þ CK Þ þ BN ðAM þ CK Þ > CK ðAM þ BN Þ


 CK  AB:

Therefore, AM  BC þ BN  AC > CK  AB.


This ends the proof.

Problems for Self-Study

1.2.19. Given two circles with radiuses R1, R2, such that the distance between their
centers is equal to d and d þ R1 < R2. Prove that R2  R1  d  XY  d þ R1 þ R2,
where X and Y are arbitrary points of these two circles.
1.2.20. Prove that in any quadrilateral there are at least two sides a and b, such that
1  ba < 1, 875.
1.2.21. Prove that any hexagonal cross section of a unit cube by a plane passing
pffiffi
through its center has an area not less than 3 4 2.
1.2 Theorem on the Length of the Broken Line 37

1.2.22. In a quadrilateral ABCD angles A and C are not less than 90 . Prove that the
perimeter of the inscribed quadrilateral in the quadrilateral ABCD is not less than
2AC (see problem 2.3.6).
1.2.23. (a) In a tetrahedron SABC all planar angles at vertex S are equal to 60 .
Prove that AB þ BC þ AC  SA þ SB þ SC.
(b) In a convex hexagon ABCDEF any two of diagonals AD, BE and CF make
an angle of 60 . Prove that AB þ BC þ CD þ DE þ EF þ FA  AD þ BE þ CF.
(c) Among all convex quadrilaterals with given diagonals and given angles between
them find the quadrilateral with the smallest perimeter.
Hint
(а) Let SA ¼ a, SB ¼ b, SC ¼ c; then
sffiffiffiffiffiffiffiffiffiffiffiffiffiffiffiffiffiffiffiffiffiffiffiffiffiffiffiffiffiffiffiffiffiffiffiffiffiffiffiffiffiffiffi sffiffiffiffiffiffiffiffiffiffiffiffiffiffiffiffiffiffiffiffiffiffiffiffiffiffiffiffiffiffiffiffiffiffiffiffiffiffiffiffiffiffiffiffi
a 2 pffiffi3ffi 2 
b
2 pffiffiffi 2
3
AB þ BC þ AC ¼ b þ a þ c þ b
2 2 2 2
sffiffiffiffiffiffiffiffiffiffiffiffiffiffiffiffiffiffiffiffiffiffiffiffiffiffiffiffiffiffiffiffiffiffiffiffiffiffiffiffiffiffiffi
c 2 pffiffi3ffi 2
þ a þ c ,
2 2

and it is left to use the inequality of problem 1.2.5а.


pffiffiffiffiffiffiffiffiffiffiffiffiffiffiffiffiffiffiffiffiffiffiffiffiffi
(b) Note that x2 þ y2  xy  xþy 2 :
(c) Use the inequality of problem 1.2.5b.
1.2.24. Given a convex n-gon on a plane. Let ak be the length of its k-th side and dk
be the length of its projection on a line containing that side. Prove that
d1 þ d2 þ ::: þ dn > 2.
a1 a2 an

1.2.25. Prove that for any polyhedron there are three edges, from which you can
construct a triangle.
1.2.26. Given an infinite set of points S on a plane, such that in any 1
1 square
there are finitely many number of points of the set S. Prove that there are two
distinct points A and B from the set S, such that for any other point X belonging to
the set S it holds true min(XA; XB)  0, 999AB.
Hint Proceed the proof by contradiction argument.
1.2.27. Given a triangle ABC, such that ∠BAC  60 . Let M be the midpoint of side
BC and P be any point in plane ABC. Prove that PA þ PB þ PC  2AM.
Hint Let the rotation by the angle 60 and center A, point P moves to point P1 and
point C moves to point C1. Prove that PA þ PB þ PC ¼ PB þ PP1 þ P1C1 
BC1  2AM.
1.2.28. On the sides of a unit square, as on hypotenuses, are externally constructed
right-angled triangles. Let A, B, C, D be the vertices of the right angles and O1, O2,
38 1 Theorem on the Length of the Broken Line

O3, O4 be the incenters of these triangles, respectively. Prove that the area of
quadrilateral:
(a) ABCD is not greater than 2.
(b) O1O2O3O4 is not greater than 1.
Hint
(a) Let M and N be the midpoints of the opposite sides of the square. Then
AC  AM þ MN þ CN ¼ 2.
(b) Prove that if points O1, O2, O3, O4 are on a circle, circumscribed around the
pffiffiffi
square, then O1 O3  2.

1.2.29. Let SABC be a tetrahedron. Prove that


sffiffiffiffiffiffiffiffiffiffiffiffiffiffiffiffiffiffiffiffiffiffiffiffiffiffiffiffiffiffiffiffiffiffiffiffiffiffiffiffiffiffiffiffiffiffiffiffiffiffiffiffiffiffiffiffiffiffiffi
H2
SSAB þ SSBC þ SSAC  ðAB þ BC þ ACÞ2 þ S2ABC ,
4

where H is the length of the altitude of tetrahedron SABC, drawn from the vertex S.
Hint See problem 1.2.5а.
1.2.30. Let P be the projection of point M on a plane containing points A, B, C.
Prove that if from segments PA, PB, PC one can construct a triangle, then one can
construct a triangle from segments MA, MB, MC too.
pffiffiffiffiffiffiffiffiffiffiffiffiffiffiffiffiffiffiffiffiffiffiffi pffiffiffiffiffiffiffiffiffiffiffiffiffiffiffiffiffiffiffiffiffiffiffi
Hint We have that MA þ MB ¼ AP2 þ MP2 þ BP2 þ MP2 
qffiffiffiffiffiffiffiffiffiffiffiffiffiffiffiffiffiffiffiffiffiffiffiffiffiffiffiffiffiffiffiffiffiffiffiffiffiffiffiffiffi
ðAP þ BPÞ2 þ 4MP2 (see problem 1.2.5a).

1.2.31. Let ABC be a triangle. Prove that 2pr  ama  pR.


Hint See the solution of problem 1.2.7.
1.2.32. Let A1, B1, C1 be points on sides BC, AC, AB of triangle ABC, respectively.
pffiffiffi
Prove that A1 B1 þ B1 C1 þ A1 C1  3minðAA1 ; BB1 ; CC1 Þ.
Hint See the solutions of problems 1.2.7 and 1.2.3.
Chapter 2
Application of Projection Method

This chapter consists of four sections and is devoted to the projection method.
The projection method is one of the fundamental methods applied in order to deal
with geometric inequalities.
In this chapter we learn some techniques on how the projection method can be
applied and to prepare a background for the application of projection method: in
Section 2.1 we consider several problems and some properties of convex polygon
lying inside of another polygon, in Section 2.2 we consider some problems with a
sufficient condition for comparison of lengths of two broken lines on the plane; in
Section 2.3 we deal with the inscribed polygons with the least perimeter and in
Section 2.4 we consider problems that can be proved using these properties and the
projection method.
The main statement that we are going to use in order to apply the projection
method is the following: if A0 B0 is the orthogonal projection of segment AB onto a
line l or onto a plane α, then AB  A0 B0 .
In Section 2.1 selected problems are those that deal with geometric inequalities
related to two figures, such that one of them is inside of the other one. In this
section, problems 2.1.1 and 2.1.10 are used as the main techniques of proofs.
In Section 2.2 is provided a sufficiency condition of comparing the sum of the
length of the segments belonging to two sets of segments on the plane. This
beautiful condition is given in problem 2.2.18. Moreover, the following problems
demonstrate the application of this condition.
In Section 2.3 are given the solutions, for the triangle and quadrilateral, of the
following problem: given a convex n-gon A1A2 . . . An. Inscribe to A1A2 . . . An a
polygon B1B2 . . . Bn, such that B1B2 . . . Bn has the least perimeter (in Section 4.1
are provided some generalizations of this problem, see problem 4.1.18).
The last section of this chapter (Section 2.4) is devoted to the application of
projection method in different problems.
Some problems in this chapter were inspired by [4, 8, 9, 12, 13,
15]. Nevertheless, even for these problems the authors have mostly provided their
own solutions.

© Springer International Publishing AG 2017 39


H. Sedrakyan, N. Sedrakyan, Geometric Inequalities, Problem Books
in Mathematics, DOI 10.1007/978-3-319-55080-0_2
40 2 Application of Projection Method

2.1 Convex Polygon Lying Inside of Another Polygon

2.1.1. Prove that, if a convex polygon lays inside of other polygon, then the
perimeter of an internal polygon is less than the perimeter of the external polygon.
2.1.2. Prove that, if the sum of the plane angles at the top of a pyramid is greater than
180 , then each lateral edge of a pyramid is smaller than the half-perimeter of its base.
2.1.3. Consider a convex quadrilateral with sides a , b , c , d laying inside of a unit
1 1 1 1
square. Prove that þ þ þ > 4.
a b c d
2.1.4. Prove that, if a regular hexagon with side length b lays inside of an equilateral
a
triangle with side length a, then b < .
3
2.1.5. Prove that, if a rectangle is covered by a right-angled triangle, so that its
smaller side is on a hypotenuse, then it is possible to cover this rectangle by the
same right-angled triangle, such that its two sides are situated on the legs.
2.1.6. (a) Let S1 and S2 be squares with sides a and b such that they are inside of a
unit square. Given that they have no common points. Prove that a þ b < 1.
(b) Given that regular n-gons with sides b and c are located inside of a regular n-gon
with side a and have no common points. Prove that b þ c < a.
2.1.7. If a regular polygon B1B2 . . . Bn is inscribed into polygon A1A2 . . . An
(B1 2 A1A2, B2 2 A2A3, . . ., Bn 2 AnA1), where ∠A1 ¼ ∠A2 ¼ . . . ¼ ∠An, then
A1A2 . . . An is also a regular polygon and one can place in polygon B1B2A3 . . .
AnA1 a polygon equal to B1B2 . . . Bn, so that its sides are parallel to the sides of
polygon A1A2 . . . An.
2.1.8. Let two non-intersecting triangles with altitudes h1 , h2 , h3 and h01 , h02 , h03 lay
pffiffiffi  
inside of a unit square. Prove that 2  minðh1 ; h2 ; h3 Þ þ min h01 ; h02 ; h03 .
2.1.9. (a) Prove that, if a regular 2n -gon with a side length a2 lays inside of a regular
2n-gon with a side length a and center O, then it covers point O.
(b) A regular n-gon with a side length b is inside of a regular n-gon with a side
a
length a and does not contain its circumcenter. Prove that b <  π .
2cos 2 2n
2.1.10. Let a convex polygon M is inside of triangle ABC. Prove that polygon M can
be covered by triangle ABC, so that one of the sides of polygon M lays on one of the
sides of the triangle.
2.1.11. Consider two triangles, such that one triangle has sides a, b, c, the other has
sides a0 , b0 , c0 . Which relations between numbers a, b, c and a0 , b0 , c0 are necessary
and sufficient in order the first triangle to be covered by the second one?
2.1.12. Prove that, if triangle A1B1C1 is covered by a triangle ABC, then

(a) A1 B1 þ B1 C1 þ A1 C1  maxðA1 B1 ; B1 C1 ; A1 C1 Þ 
 AB þ BC þ AC  maxðAB; BC; ACÞ,
2.1 Convex Polygon Lying Inside of Another Polygon 41

(b) T(A1B1C1)  T(ABC), where for a triangle XYZ T(XYZ) ¼ min


(MX þ MY þ MZ), and M is any point of a plane XYZ.
2.1.13. Let a triangle altitudes h01 , h02 , h03 be inside of a triangle with altitudes h1, h2,
 
h3. Prove that min h01 ; h02 ; h03  minðh1 ; h2 ; h3 Þ.
2.1.14. Let a triangle be inside of the non-acute-angled triangle. Prove that the first
triangle can be in the second one in such a way that two of its vertices are on the
largest side of the second triangle.
2.1.15. Let an equilateral triangle A1B1C1 be inside of the non-acute-angled triangle
pffiffiffi
ABC (∠C  π2). Prove that AB  3A1 B1 .\
2.1.16. Given on a plane an equilateral triangle XYZ with the side length equal to
1 and an equilateral triangle DEF, such that ∠DEF ¼ 20 . Prove that the area of a
convex figure containing triangles XYZ and DEF is greater than or equal to 12 cos 10 .
2.1.17. Let 1  2 rectangle be inside of three mutually non-intersecting squares.
Given that the side lengths of the squares are equal to a, b, and c. Prove that
a þ b þ c  2.
pffiffi
2.1.18. Given that a triangle ABC is covered by a unit square. Prove that r  541,
where r is the inradius of triangle ABC.

Solutions

2.1.1. Let us construct on the sides of the internal polygon, outside of it, half-strips,
with parallel edges perpendicular to corresponding sides of the polygon (Figure 2.1).
It is clear, that the perimeter of the internal polygon does not exceed that part of
the perimeter of the external polygon which is inside of these strips.
Hence, the perimeter of the internal polygon is less than the perimeter of the
external polygon.
2.1.2. Given a pyramid SA1A2 . . . An with a vertex S. Consider the layout of its
lateral surface making the cut along the edge SA1 (Figure 2.2).

Figure 2.1
42 2 Application of Projection Method

Figure 2.2 A 1'

Аn
A1
S

A2

A3

Figure 2.3 K
N B C

M A D L

Since according to the statement of the problem, the sum of the plane angles at the
vertex S is greater than 180 , then triangle SA1 A01 lays inside of polygon A1 A2 :::An A01 .
According to problem 2.1.1 2SA1 þ A1 A01 < A1 A2 þ A2 A3 þ :::þ An A01 þ A1 A01 .
2.1.3. According to problem 2.1.1, we have that a þ b þ c þ d < 4. If
a þ bþ 1c þ d1  4, then by summing up these two inequalities, we deduce that
1 1
      
a þ 1a þ b þ 1b þ c þ 1c þ d þ d1 < 8. This leads to a contradiction, as
x þ 1x  2 (x > 0). Therefore, 1a þ 1b þ 1c þ d1 > 4.
2.1.4. It is sufficient to notice that the radius of the circle inscribed into a hexagon is
less than the radius of the circle inscribed into the triangle.
2.1.5. Without loss of generality we can assume that rectangle is inscribed in a
right-angled triangle (Figure 2.3).
Since AB  AD, then we have that MN  BC > KC. Thus, it follows that
pffiffiffiffiffiffiffiffiffiffiffiffiffiffiffiffiffiffiffiffiffiffiffiffiffi pffiffiffiffiffiffiffiffiffiffiffiffiffiffiffiffiffiffiffiffiffiffiffiffi
NC ¼ MC2  MN 2 < MC2  KC2 ¼ MK, where MNBA is a rectangle, such
that triangle NCF can be placed inside of a triangle MKL similar to it.
2.1.6. (a) At first, let us prove the following lemma.
Lemma If a square PQRS with a side length a is inscribed in a rectangle ABCD
(Figure 2.4), then ABCD is also a square and the square with a side a can be placed
inside of pentagon ABQRD, such that the sides of the square would be parallel to
AB and AD.
2.1 Convex Polygon Lying Inside of Another Polygon 43

Figure 2.4 B Q C

P1 Q1
Q2
P P2

R2 R

S2
S1 R1

A S D

Figure 2.5

a b

Indeed, consider the rotation of square PQRS around point O by an angle 90 .
Then point R will turn into point Q, hence line CD will turn into a line BC.
Consequently, point O is equidistant from sides CD and BC. Similarly, we obtain
that point O is equidistant from sides AB and BC, AD and AB. From above it follows
that ABCD is a square with center O.
Consider a square P1Q1R1S1 with center O, such that its sides are parallel to AB,
AD, and P1Q1 ¼ PQ (Figure 2.4).
!
Perform a translation by a vector Q1 Q2 . Therefore, square P1Q1R1S1 turns into a
square P0 Q0 R0 S0 that is inside of pentagon ABQRD, as one can easily prove that
Q1Q2 ¼ PP2 ¼ SS2. This ends the proof of the lemma.
Using the lemma, we can place the square with a side a inside of a unit square, so
that their sides are parallel and again the square with a side a would not have
common points with a square with a side b (Figure 2.5).
Using the lemma once again, we obtain that the sides of the squares with sides
a and b, which do not have common points, are parallel to the sides of the unit
square. It is not difficult to prove that a þ b < 1.
(b) Let a regular n-gon B1B2 . . . Bn with a side b is inside of a regular n-gon with a
side length a. Let us draw lines passing through the vertices of B1B2 . . . Bn and
parallel to the sides of the n-gon with side a (Figure 2.6).
44 2 Application of Projection Method

Figure 2.6
а

c B2
B3
A2

B1
b
A1 Bn An

Figure 2.7

l
Πc
c
dc
db b

Πb Πa

According to problem 2.1.7 A1A2 . . . An is a regular n-gon. It is not difficult to


prove that a regular n-gon with a side c can have a common point only with one of
the triangles B1A2B2, B2A3B3 , . . . , BnA1B1. Then according to problem 2.1.7,
without loss of generality, one can assume that the regular n-gon with a side b is
inside of the regular n-gon with a side a, so that the sides of these polygons are
parallel. Similar statement is true for a regular n-gon with a side c (Figure 2.7).
Let l be a line passing along one of the sides of the polygons with sides
b and c and separating these polygons (see the solution of problem 1.2.17). Let
strips Πa, Πb, and Πc be those with minimal widths that contain these polygons
having widths da, db, dc and their boundaries are parallel to line l.
It is clear that db þ dc < da and ddba ¼ ba, ddac ¼ ac. Thus, b þ c < a.

2.1.7. Note that ∠B1 A2 B2 ¼ ∠B2 A3 B3 ¼ ::: ¼ ∠Bn A1 B1 ¼ π  2π n , (Figure 2.8). 


Let ∠A2B1B2 ¼ α, then ∠A2 B2 B1 ¼ 2π n  α, ∠A 3 B 2 B 3 ¼ π  π  2πn þ n α

¼ α, ∠A3 B3 B2 ¼ 2π n  α. Hence ΔA2B2B1 ¼ ΔA3B3B2. Consequently, B1A2 ¼ B2A3,


A2B2 ¼ A3B3. Similarly, we obtain that A1B1 ¼ A2B2 ¼ . . . ¼ AnBn and B1A2 ¼
B2A3 ¼ . . . ¼ BnA1. Therefore A1A2 ¼ A2A3 ¼ . . . ¼ AnA1, this means that
A1A2 . . . An is a regular n-gon. Let O be the center of a regular n-gon A1A2 . . . An.
2.1 Convex Polygon Lying Inside of Another Polygon 45

Figure 2.8 А3
B3¢
B2 B3

A2 B2¢

O
B1

B1¢
A1 Bn An

Figure 2.9

P1
l
P2
d1

d2 P

Note that OB1 ¼ OB2 ¼ . . . ¼ OBn, or in the other words, O is the center of
regular n-gon B1B2 . . . Bn. Choose points B01 , B02 , :::, B0n on segments OA1,
OA2, . . . , OAn correspondingly, so that OB0 1 ¼ OB0 2 ¼    ¼ OB0 n ¼ OB1. Since
points B1, B2, . . . , Bn, B01 , B02 , :::, B0n lay on the same circumference, then from the
condition B1 B2 ¼ B02 B03 it follows that B1 B02 B2 B03 is an equilateral trapezoid.
   
Consequently, ρ B02 ; B1 B2 ¼ ρ B2 ; B02 B03 . Similarly, one can prove that
         
ρ B1 ; B01 B02 ¼ ρ B02 ; B1 B2 ¼ ρ B2 ; B02 B03 ¼    ¼ ρ Bn ; B0n B01 ¼ ρ B01 ; Bn B1 .
!
Let B02 M⊥B1 B2 and M 2 B1B2, then at translation by a vector B02 M the image of
the regular n-gon B01 B02 :::B0n will be in a polygon B1B2A3 . . . AnA1.
2.1.8. Let straight line l contains one of the sides of the given triangles and separates
these triangles (Figure 2.9).
Consider bands Π1, Π2, Π containing these triangles and the square with minimal
widths d1, d2, and d, their boundaries being parallel to line l. It is clear that
pffiffiffi  
d  d1 þ d2. Since 2  d and d1  min (h1, h2, h3), d2  min h01 ; h02 ; h03 , then
pffiffiffi  
2  minðh1 ; h2 ; h3 Þ þ min h01 ; h02 ; h03 .
46 2 Application of Projection Method

Figure 2.10

2.1.9. (a) Suppose that the polygon with a side a2 does not contain point O, then the
polygon symmetric to the given polygon with respect to point O is also inside of the
polygon with a side a. Then, according to problem 2.1.6(b), a2 þ a2 < a. This leads to
a contradiction.
(b) At first, let us prove that the circumcircle of the regular n-gon with side b does
not have points outside of the circumcircle of the regular n-gon with side a.
Indeed, assume that these circles intersect. Let us choose point M on the larger
circle, such that it is not a vertex of the regular n-gon with a side a and is inside
of the smaller circle (Figure 2.10).
Let given regular n-gons be A1A2 . . . An (A1A2 ¼ a) and B1B2 . . . Bn. Assume that
point M is on the smaller arc A1A2.
We have that ∠A1 MA2 ¼ πðn1 n
Þ
and that polygon A1A2 . . . An is inside of angle
∠A1MA2.
Consequently, polygon B1B2 . . . Bn also is inside of that angle.
Hence, ∠A1 MA2 > max ∠Bi MBj > π ðn1 Þ
n . This leads to a contradiction.
i, j
Let us denote the radiuses of these circles by Ra and Rb and their centers by
O and O1. From aforesaid, it follows that Ra  Rb þ OO1, and since point O is not
inside of polygon B1B2 . . . Bn, then OO1 > rb, where rb is the radius of the circle
inscribed in B1B2 . . . Bn.
a b a
Thus, Ra > Rb þ rb, or > . We deduce that b < π.
2 sin πn π b 2cos 2 2n
2 sin þ
n 2tg π
n
2.1.10. It is enough to prove the problem for triangle A1B1C1 (Figure 2.11a, b),
where A1B1 k AB, B1C1 k BC, A1C1 k AC.
Consider the following two cases: Figure 2.11a, b.
Let us consider first the case of Figure 2.11a. Perform a rotation around vertex C2
by some angle, so that one of the sides with vertices C2, B2, A2 of polygon
M becomes for the first time parallel to lines A1B1, A1C1, or B1C1, respectively.
Let u 2 [u1, u2], u1 and u2 being the values of u for aforesaid rotations (in positive
and negative directions). Let us denote by Mu the image obtained by a rotation of
2.1 Convex Polygon Lying Inside of Another Polygon 47

Figure 2.11 B
B1

C2 A2
u nf(u) j-u
mf(u) Mu
a g
A1 B2 C1
A C
a

B
B1

u
mf(u)

Mu
a g
A1 B2 C1
A C
b

polygon M around C2 with subsequent similarity transformations with similitude


centers A1 and B1. Note that ΔA2B2C2 is inscribed in triangle A1B1C1
It is clear that some of the sides of polygons Mu1 and Mu2 are on one of the sides
of triangle A1B1C1. Let C2B2 ¼ mf(u), A2B2 ¼ nf(u) .
Let us prove that, on segment [u1, u2], function f(u) accepts its maximal value at
point u1 or u2. Indeed, we have that

mf ðuÞ sin u nf ðuÞ sin u


A1 C1 ¼ A1 B2 þ B2 C1 ¼ þ sin ðφ  uÞ:
sin α sin γ

Thus, it follows that

A1 C1 sin α sin γ A1 C1 sin α sin γ


f ð uÞ ¼ ¼ ,
m sin u sin γ þ n sin ðφ  uÞ sin α a sin ðu þ φ1 Þ

where a, φ, and φ1 are constants. Since, in case u 2 [u1, u2] f(u) > 0, or equivalently,
sin(u þ φ1) > 0, then on segment [u1, u2], function sin(u þ φ1) accepts its minimal
value at point u1 or u2. Hence, on segment [u1, u2], function f(u) accepts its
minimal value at points u1 or u2.
The proof is similar for the case of Figure 2.11b. This ends the proof.
48 2 Application of Projection Method

Figure 2.12 C'

a' b'

a b

c
B' c' A'

C'

a' b'
ha'

a b

c
B' c' A'
Π a'

Figure 2.13

2.1.11. According to problem 2.1.10, if the first triangle is covered by the second
triangle, then one can assume that one of the sides of the first triangle lays on the
side of the second triangle. Let us find necessary and sufficient conditions for the
first triangle to be covered by the second one in a way shown in Figure 2.12.
Let ha0 , hb0 , hc0 be the altitudes of the triangle with sides a0 , b0 , c0 drawn to the
sides a0 , b0 , c0 , correspondingly, and Πa0 , Πb0 , Πc0 be the projections of the first
triangle on the straight lines, containing heights ha0 , hb0 , hc0 (Figure 2.13).
Let us prove that, or order that, one can place the triangle with sides a, b, c inside
of the triangle with sides a0 , b0 , c0 in a way shown in Figure 2.12, it is necessary and
sufficient that conditions

ha0  Πa0 , hb0  Πb0 , hc0  Πc0 ð2:1Þ

hold true.
The necessity of the conditions ha0  Πa0 follows from the fact that the triangle
0 0 0 0 0
 sides a, b, c is inside of a band with boundaries B C and l1 , where A 2 l1 ,
with
0  0 0
l1 B C , (see Figure 2.12). Similarly, one can prove the necessity of conditions
hb0  Πb0 and hc0  Πc0 .
2.1 Convex Polygon Lying Inside of Another Polygon 49

C'

a'
a
b b'

β' β α α'
B' B A A'
Figure 2.14

Figure 2.15
C

d j l
E F
l1

Now, if condition (2.1) is satisfied, then it is clear that c  c0 , otherwise Πa0 > ha0 .
Let us consider Figure 2.14.
!
If the parallel displacement of point C by a vector BB0 has moved it in the shaded
half-band, then it means that hb0 < Πb0 . This leads to a contradiction. This means
that, if condition (2.1) is satisfied, then the first triangle can be placed inside of the
second one.
Note that, if point C and straight line l (Figure 2.15) are given, then the
projection of segment CE (E 2 l ) on line l1 is equal to CE| cos (φ  δ)|.
Thus, we obtain that Πc0 ¼ hc , Πb0 ¼ 12 ða  j sin ðα0 þ βÞj þ b  j sin ðα  α0 Þjþ
c sin α0 Þ, Πa0 ¼ 12 ða  j sin ðβ0  βÞj þ b  j sin ðβ0 þ αÞj þ c sin β0 Þ (see the proof of
problem 2.2.1a). Then, condition (2.1) can be rewritten as:

1
hc0  hc , hb0  ða  j sin ðα0 þ βÞj þ b  j sin ðα  α0 Þj þ c sin α0 Þ,
2 ð2:2Þ
1
ha  ða  j sin ðβ0  βÞj þ b  j sin ðβ0 þ αÞj þ c sin β0 Þ:
0
2

It is clear that for condition (2.2) one needs only the values of a, b, c, a0 , b0 , c0 .
Note that, for two triangles 18 variants of dispositions (similar to Figure 2.12)
are possible and for the first triangle to be covered by the second triangle, it is
50 2 Application of Projection Method

necessary and sufficient that condition (2.2) is satisfied at least for one of these
18 variants.
2.1.12. We have to prove that f(A1B1C1)  f(ABC), if ΔABC covers ΔA1B1C1,
where f(ABC) 2 R and that f(ABC) ¼ f(MNK), if ΔABC ¼ ΔMNK. According to
problem 2.1.10, one can assume that one of the sides of triangle A1B1C1 lays on
one of the sides of triangle ABC. Without loss of generality, we can assume that two
sides of triangle A1B1C1 lay on two sides of triangle ABC, one of the two sides of
triangle A1B1C1 coincides with one of the sides of triangle ABC.
Indeed, in the case of Figure 2.16a, we have that

f ðA1 B1 C1 Þ  f ðA1 B1 A2 Þ  f ðAB1 A2 Þ  f ðAB1 CÞ  f ðABCÞ:

Therefore, f(A1B1C1)  f(ABC), while in the case of Figure 2.16b, we have that
f(A1B1C1)  f(A1B1B2)  f(A1BB2)  f(A1BC)  f(ABC). Thus, it follows that
f(A1B1C1)  f(ABC).
(a) Let triangles A1B1C1 and ABC are as it is shown in Figure 2.17.

Figure 2.16 C C

A2 B2
C1

C1

B B1 A1 AB B1 A1 A
a) б)

Figure 2.17 Cº C1

g1

g
b=b1 a1 a

a=a1 b1 b
Aº A1 c1 B1 B
c
2.1 Convex Polygon Lying Inside of Another Polygon 51

Let us consider the following three cases: max(α1, β1, γ 1) ¼ γ 1, max(α1, β1, γ 1) ¼ α1,
and max(α1, β1, γ 1) ¼ β1.
I. If max(α1, β1, γ 1) ¼ γ 1. Therefore, max(α, β, γ) ¼ γ. Note that ∠CB1B ¼
180  β1 > 90 , which means that a > a1. Hence, a þ b ¼ a þ b1 > a1 þ b1.
II. If max(α1, β1, γ 1) ¼ α1, then α ¼ α1  β1 > β. Consequently, max(α, β, γ) ¼ α
or max(α, β, γ) ¼ γ.
If max(α, β, γ) ¼ α, then b þ c ¼ c þ b1 > b1 þ c1.
If max(α, β, γ) ¼ γ, then γ  α ¼ α1  γ 1. Therefore, ∠CB1B ¼ 180  β1 >
90 . Consequently, a > a1, this means that a þ b ¼ a þ b1 >
a1 þ b 1  b 1 þ c 1.
III. Let max(α1, β1, γ 1) ¼ β1. Consider the following three cases:
1. max(α, β, γ) ¼ β, then a þ c ¼ a þ BB1 þ c1 > a1 þ c1.
2. max(α, β, γ) ¼ α, then β1  α1. Consequently b1  a1, hence b þ c ¼
b1 þ c  a 1 þ c > a 1 þ c 1.
3. max(α, β, γ) ¼ γ. In this case, let us notice that if point E moves from
point B1 to point B along segment B1B, then the value of
∠AEC  ∠ACE decreases from β1  γ 1 (β1  γ 1  0) to the value of
β  γ (β  γ  0). Consequently, on segment B1B exists such point
E that ∠AEC ¼ ∠ACE. Then a1 þ c1  AE þ CE ¼ AC þ CE ¼
b þ CE  a þ b. Consequently, a1 þ c1  a þ b (∠CEB > 90 , if point
E does not coincide with point B).
This ends the proof of (a).
(b) In order to end the proof, note that if max(α, β, γ)  120 , then T(ABC) ¼
AB þ BC þ AC  max (AB, BC, AC) (see problem 1.2.8) and if max(α, β, γ) <
120 , then T(ABC) ¼ BB0 (Figure 2.18).
Indeed, since max(α, β, γ) < 120 , then quadrilateral ABCB0 is convex and point
B is outside of the circumcircle of the equilateral triangle ACB0 . Let point М0 be such

Figure 2.18 B

M0
M
A 600 C
0 0
60 60


52 2 Application of Projection Method

that AМ0 ¼ AM and ∠MAM0 ¼ 60 (Figure 2.18). Then ΔAMC ¼ ΔAM0 B0 and conse-
quently, AM þ BM þ CM ¼ MM0 þ BM þ M0 B0  BB0 ¼ BM0 þ M0B0 ¼ BM0 þ
AM0 þ CM0. This means that T(ABC) ¼ BB0 . Note that T(ABC) ¼ BM0 þ AM0 þ
CM0 > BM0 þ AM0 > AB and T(ABC) ¼ BB0 < AB þ AB0 ¼ AB þ AC. Similarly, one
can prove that T(ABC) > BC, T(ABC) > AC, T(ABC) < BC þ AC, T(ABC) <
AB þ BC.
Without loss of generality, we can assume that side A1C1 is on side AC.
Consider the following cases:
I. If max(α, β, γ) < 120 , max(α1, β1, γ 1) < 120 , then convex quadrilateral
A1B1C1B10 has a diameter T(A1B1C1) and is located inside of convex quadri-
lateral ABCB0 with a diameter T(ABC). Hence T(A1B1C1)  T(ABC).
II. If max(α, β, γ)  120 , max(α1, β1, γ 1) < 120 , then according to problem
2.1.12a, we have that

T ðABCÞ ¼ AB þ BC þ AC  maxðAB þ BC þ ACÞ 


 A1 B1 þ B1 C1 þ A1 C1  maxðA1 B1 þ B1 C1 þ A1 C1 Þ > T ðA1 B1 C1 Þ:

Therefore, T(ABC) > T(A1B1C1).


III. If max(α, β, γ)  120 , max(α1, β1, γ 1)  120 , then according to problem
2.1.12a, we have that T(ABC)  T(A1B1C1).
IV. If max(α, β, γ) < 120 , max(α1, β1, γ 1)  120 , then without loss of generality,
one can assume that these triangles are ABC and AB1C, with vertex B1 being on
side AB and ∠AB1C ¼ 120 . Then, the proof is similar to the proof of case I.
2.1.13. Let min(h1, h2, h3) ¼ h1 and Π is a band with a minimal width containing the
triangle with altitudes h01 , h02 , h03 and with boundaries perpendicular to the altitude
with a length h1 (see Figure 2.19).  
Then, minðh1 ; h2 ; h3 Þ  h  min h01 ; h02 ; h03 , where h is the width of the band Π.
2.1.14. According to problem 2.1.10, one can assume that one side of the second
triangle is on one side of the first triangle and that the second triangle is in the first
triangle.
Without loss of a generality, we can assume that possible cases are those
presented in Figure 2.20. In the case of Figure 2.20a the proof of the problem is
obtained by rotating triangle AB1C around point A by angle ∠B1AB.
In the case of Figure 2.20b, if AB1  AC, then it is sufficient to consider the
triangle, symmetric to triangle A1B1C with respect to the bisector of angle ∠CAB.

Figure 2.19

h1
2.1 Convex Polygon Lying Inside of Another Polygon 53

Figure 2.20 B
B

B1
B1

A≡A1 C1 C A A1 C≡C1
a b

Figure 2.21 B
B1

A C

Figure 2.22 C

C1

A A1 H M B1 B

Now, let AB1 > AC, then ∠AB1C < ∠ACB1. Consequently, ∠BB1 C > π2.
If ∠AB1A1  ∠BCB1, then the proof of the problem is obtained by rotating
triangle A1B1C around point C by angle  ∠AB1A1.
Let AB1 > AC and ∠AB1A1 > ∠BCB1, then ∠AB1C1 < ∠ABC. Since
B1C < BC, A1B1 < AB, ∠AB1C1 < ∠ABC, then if triangle A1B1C1 is placed in
such a way that points B1 and B coincide, while point A1 is on side AB, then vertex
C1 of triangle A1B1C1 is inside of triangle ABC. This ends the proof.
Remark If ABC is an acute triangle, then the statement of the problem is wrong
(see the example in Figure 2.21).
2.1.15. According to problem 2.1.10, one can assume that points A1 and B1 lay on
segment AB (Figure 2.22).
54 2 Application of Projection Method

Note that ∠AC1 B  ∠ACB  π2. Therefore, if M is the midpoint of segment AB,
pffiffiffi
then AB  2C1 M  2C1 H ¼ 3A1 B1 .
Remark Similarly, one can prove that, if the triangle with altitudes h1, h2, h3 is
inside of non-acute triangle ABC, then the inequality max(AB, BC, AC)  2 min (h1,
h2, h3) holds true.
Another proof of the problem can be obtained by using problem 2.1.13.
2.1.16. (Solution of M.D. Kovalev.) First we need to prove that the side of an

equilateral triangle, containing ΔDEF, is not less than p2ffiffi3 cos 10 . According to
problem 2.1.10, it is sufficient to consider the following two cases (see Figure 2.23).
pffiffi  
In case (a), we have that 23 a  cos 10 . Consequently, a  p2ffiffi3 cos 10 .
 
In case (b), we have that a  D1 E ¼ sin 100
sin 60
 ¼ p2ffiffi3 cos 10 , where ∠FD1E ¼ 60 .

Consider now the minimal equilateral triangle X1Y1Z1 with sides parallel to the
sides of triangle XYZ, which contains triangles XYZ and DEF (Figure 2.24).

a E a a a

1
F

200
D F D1 D E
a b
Figure 2.23

Y1 Y1 Y1

Y x Y y
a x a a
Y
х

X Z X Z
y z
XºX1 Z Z1 X1 Z1 X1 Z1
a b c
Figure 2.24
2.1 Convex Polygon Lying Inside of Another Polygon 55

The area of a convex figure containing ΔXYZ and ΔDEF is not less than:
pffiffiffi pffiffiffi pffiffiffi
3 1 3 3
in case (a): þ x¼ þ ða  1Þ,
4 2 4 4
pffiffiffi pffiffiffi pffiffiffi
3 xþy 3 3
in case (b): þ ¼ þ ða  1Þ,
4 2 4 4
pffiffiffi pffiffiffi pffiffiffi
3 xþyþz 3 3
in case (c): þ ¼ þ ða  1Þ.
4 2 4 4
pffiffi 
It remains to note that 43 a  12 cos 10 .
Remark Among all convex figures covering any triangle with the sides not exceed-
ing 1, the least area has a triangle ABC, such that ∠A ¼ 60 , AB ¼ 1, and the altitude

drawn to AB is equal to cos10 . The area of that triangle is equal to 12 cos 10 .
2.1.17. Let sides AB and BC of rectangle ABCD be equal to 1 and 2 respectively,
and squares with sides a, b, c are not mutually intersecting and lay inside of
rectangle ABCD.
Note that, for each side of a rectangle one can find a “good” square, such that,
while moving it in a direction perpendicular to the given side, it does not intersect
with the other squares before intersecting with that side. According to the
Dirichlet’s principle there is a “good” square simultaneously for two sides of the
rectangle. If these are opposite sides, then this ends the proof (see Figure 2.25). In
the case of Figure 2.25a, we have that a  x þ y ¼ MN and b  ND. Consequently,
a þ b  CD ¼ 1. We have that c  1. Thus, a þ b þ c  2.
In the case of Figure 2.25b, according to problem 2.1.6a, we have that
b þ c  2  MN, a  MN. Therefore, a þ b þ c  2  MN þ MN ¼ 2.
If these sides are adjacent, then it is possible to assume that the “good” square is
located on one of these sides (see the proof of problem 2.1.6a). By repeating these
reasonings for the new squares we eventually find that one of these squares lies in
one of the corners of the rectangle (see Figure 2.26).

B C B C
x М
y b
2-MN
x N
b c b

A D A M N D
2-MN
a b

Figure 2.25
56 2 Application of Projection Method

Figure 2.26 B P C

A Q D

b a b a c b a
c

c
a b c

Figure 2.27

b x y
c y
c x
а c b
d
a
a
a b
Figure 2.28

If the projections of any two squares on side CD do not intersect, then as it has
been proven above, a þ b þ c  2. Otherwise, there exists a straight line parallel to
AD crossing all three squares. If none of the squares with sides b and c crosses
segment PQ (see Figure 2.26), then as we have already proven, a þ b þ c  2. Thus,
possible positions of the squares are presented in fig 2.27.
From the proof of problem 2.1.6a, it follows that instead of Figure 2.27 one can
consider the variants presented in Figure 2.28. In the case of Figure 2.28a, we have
that a þ b þ c  a þ b þ d  2 and, in the case of Figure 2.28b, we have that a þ b þ
c  a þ x þ y þ c ¼ (a þ x) þ (c þ y)  1 þ 1 ¼ 2.
2.1.18. At first, note that if point B is on side AD of triangle ACD, then rABC  rADC
(by rXYZ we denote the radius of the incircle of triangle XYZ).
Indeed, let O and O1 be the centers of the incircles of triangles ABC and ADC,
respectively. Since, these points belong to bisector AA1 of triangle ADC, then
∠ACO ¼ 12 ∠ACB  12 ∠ACD ¼ ∠ACO1 . Therefore, AO  AO1. Let E, E1 2 AC
and OE ⊥ AC, O1E1 ⊥ AC. Then, ΔAOE  ΔAO1E1, thus rrADC ABC
¼ AO
AO
1
 1.
2.1 Convex Polygon Lying Inside of Another Polygon 57

Let triangle ABC is covered by a unit square 1, and vertex B does not belong to
any of the sides of the square. Then, if ray AB intersects one of the sides of the
square at point D, we have that rABC  rADC.
From the aforesaid, it follows that, it is sufficient to prove the inequality
pffiffi
r  541, for triangle ABC, such that all of its vertices are on the sides of the square.
Let the vertices of the triangle are on the sides of square MNPQ and MN ¼ 1.
Consider the following cases:
(a) A, B 2 MN, C 2 PN, with B being on segment AN.
pffiffi pffiffi pffiffi
Since r ¼ r ABC  r ACN  r APN  r MPN ¼ 1  22 < 541, then r < 541.
(b) A, B 2 MN, C 2 PQ with B being on segment AN. Let C0 be the midpoint of the
segment PQ, then
pffiffiffi
1 1 51
r ¼ rABC  rACN  rMCN ¼  ¼ r MCN0 ¼ :
MC þ CN þ 1 MC0 þ C0 N þ 1 4

(c) A 2 MQ, B 2 PQ, C 2 PN, and AM  CN.


Without loss of generality, one can assume that, the point A and M coincide.
Indeed, it is sufficient to consider a unit square M1N1P1Q1, where M1 A,
N1 2 PN and M1N1 k MN, Q 2 M1Q1.
pffiffi
Let us prove that, if A M, B 2 PQ, C 2 PN, then r  541.
pffiffiffiffiffiffiffiffiffiffiffiffiffi pffiffiffiffiffiffiffiffiffiffiffiffiffi
Denote BQ ¼ x, CN ¼ y, then we have to prove that 1 þ x2 þ 1 þ y2 þ
qffiffiffiffiffiffiffiffiffiffiffiffiffiffiffiffiffiffiffiffiffiffiffiffiffiffiffiffiffiffiffiffiffiffiffiffiffiffi  p ffiffi
ffi 
ð1  xÞ2 þ ð1  yÞ2  1 þ 5 ð1  xyÞ, where x, y 2 [0; 1].
pffiffiffiffiffiffiffiffiffiffiffiffiffi pffiffiffiffiffiffiffiffiffiffiffiffiffi qffiffiffiffiffiffiffiffiffiffiffiffiffiffiffiffiffiffiffiffiffiffiffiffiffiffiffiffiffiffiffiffiffiffiffiffiffiffi
Firstly, we prove that 1 þ x2  1 þ y2 þ ð1  xÞ2 þ ð1  yÞ2 
pffiffiffi
5ð1  xyÞ.
pffiffiffiffiffiffiffiffiffiffiffiffiffi pffiffiffiffiffiffiffiffiffiffiffiffiffi qffiffiffiffiffiffiffiffiffiffiffiffiffiffiffiffiffiffiffiffiffiffiffiffiffiffiffiffiffiffiffiffiffiffiffiffiffiffi
If xy  12, then 1 þ x2  1 þ y2 þ ð1  xÞ2 þ ð1  yÞ2  1 þ xy  32 >
pffiffi p ffiffi

2  5ð1  xyÞ.
5

If xy < 12, then


pffiffiffiffiffiffiffiffiffiffiffiffiffi pffiffiffiffiffiffiffiffiffiffiffiffiffi qffiffiffiffiffiffiffiffiffiffiffiffiffiffiffiffiffiffiffiffiffiffiffiffiffiffiffiffiffiffiffiffiffiffiffiffiffiffi qffiffiffiffiffiffiffiffiffiffiffiffiffiffiffiffiffiffiffiffiffiffiffiffiffiffiffiffiffiffiffiffiffiffiffiffiffiffiffiffi
1 þ x2  1 þ y2 þ ð1  xÞ2 þ ð1  yÞ2 ¼ ð1  xyÞ2 þ ðx þ yÞ2
qffiffiffiffiffiffiffiffiffiffiffiffiffiffiffiffiffiffiffiffiffiffiffiffiffiffiffiffiffiffiffiffiffiffiffiffiffiffiffiffiffiffiffiffiffiffiffiffiffiffiffiffiffiffiffi
pffiffiffiffiffiffiffiffiffiffiffiffiffiffiffi 2
þ ð 1  2xyÞ þ ð1  x  yÞ2
qffiffiffiffiffiffiffiffiffiffiffiffiffiffiffiffiffiffiffiffiffiffiffiffiffiffiffiffiffiffiffiffiffiffiffiffiffiffiffiffiffiffiffiffiffiffiffiffiffiffiffiffiffiffi
pffiffiffiffiffiffiffiffiffiffiffiffiffiffiffi 2
 ð1  xy þ 1  2xyÞ þ 12
qffiffiffiffiffiffiffiffiffiffiffiffiffiffiffiffiffiffiffiffiffiffiffiffiffiffiffiffiffiffiffiffiffiffiffiffiffiffiffiffiffiffiffiffiffiffiffiffiffiffiffiffiffiffiffiffiffiffiffiffiffiffiffiffiffiffiffiffiffiffiffiffiffiffiffiffiffiffiffiffiffiffiffiffiffiffiffiffiffiffiffi
pffiffiffiffiffiffiffiffiffiffiffiffiffiffiffi
¼ ð1  xyÞ2 þ 2ð1  xyÞ 1  2xy þ 1  2xy þ 1
qffiffiffiffiffiffiffiffiffiffiffiffiffiffiffiffiffiffiffiffiffiffiffiffiffiffiffiffiffiffiffiffiffiffiffiffiffiffiffiffiffiffiffiffiffiffiffiffiffiffiffiffiffiffiffiffiffiffiffiffiffiffiffiffiffiffiffiffiffiffiffiffiffiffiffiffiffiffiffiffiffiffiffiffiffiffiffiffiffiffiffi
 ð1  xyÞ2 þ 2ð1  xyÞð1  2xyÞ þ 1  2xy þ 1
pffiffiffi
¼ 5ð1  xyÞ

(see problem 1.2.5a).


58 2 Application of Projection Method

Note that
pffiffiffiffiffiffiffiffiffiffiffiffiffi pffiffiffiffiffiffiffiffiffiffiffiffiffi qffiffiffiffiffiffiffiffiffiffiffiffiffiffiffiffiffiffiffiffiffiffiffiffiffiffiffiffiffiffiffiffiffiffiffiffiffiffi
1 þ x2 þ 1 þ y2 þ ð1  xÞ2 þ ð1  yÞ2
 
pffiffiffiffiffiffiffiffiffiffiffiffiffipffiffiffiffiffiffiffiffiffiffiffiffiffi qffiffiffiffiffiffiffiffiffiffiffiffiffiffiffiffiffiffiffiffiffiffiffiffiffiffiffiffiffiffiffiffiffiffiffiffiffiffi
2 2
 1 þ x 1 þ y þ ð1  x Þ þ ð1  y Þ
2 2

pffiffiffiffiffiffiffiffiffiffiffiffiffi  pffiffiffiffiffiffiffiffiffiffiffiffiffi

¼  1 þ x2  1 1 þ y2  1 þ 1
x2 y2
¼ 1  pffiffiffiffiffiffiffiffiffiffiffiffiffi  pffiffiffiffiffiffiffiffiffiffiffiffiffi  1  x  y,
1 þ x2 þ 1 1 þ y2 þ 1

therefore
pffiffiffiffiffiffiffiffiffiffiffiffiffi pffiffiffiffiffiffiffiffiffiffiffiffiffi qffiffiffiffiffiffiffiffiffiffiffiffiffiffiffiffiffiffiffiffiffiffiffiffiffiffiffiffiffiffiffiffiffiffiffiffiffiffi pffiffiffiffiffiffiffiffiffiffiffiffiffipffiffiffiffiffiffiffiffiffiffiffiffiffi
1 þ x2 þ 1 þ y 2 þ ð1  x Þ2 þ ð1  y Þ2  1 þ x 2 1 þ y 2
qffiffiffiffiffiffiffiffiffiffiffiffiffiffiffiffiffiffiffiffiffiffiffiffiffiffiffiffiffiffiffiffiffiffiffiffiffiffi
þ ð1  xÞ2 þ ð1  yÞ2 þ 1  xy 
 pffiffiffi
 1 þ 5 ð1  xyÞ:

Problems for Self-Study

2.1.19. Let a triangle with sides a1, b1, c1 be inside of the triangle with sides a, b, c.
Is it true that a21 þ b21 þ c21  a2 þ b2 þ c2 ?
2.1.20. Given points A1, B1, and C1 on sides BC, CA, AB of triangle ABC, respec-
tively, such that ∠AB1C1 þ ∠BC1A1 þ ∠CA1B1 ¼ 180 . Prove that triangle
A1B1C1 can be placed inside of one of the triangles AB1C1, BC1A1, CA1B1.
2.1.21. Given a point O inside of triangle ABC and points A1, B1, C1 on sides BC,
CA, AB, respectively, such that OB1 ⊥ AC, OA1 ⊥ BC, OC1 ⊥ AB.
Prove that triangle A1B1C1 can be placed inside of one of triangles AB1C1,
BC1A1, CA1B1.
2.1.22. Let a convex n-gon be inside of a unit square. Prove that one can find three
vertices A, B, C of this n-gon, so that the area of triangle ABC does not exceed
(a) 12, for n ¼ 3, 4, (b) 14, for n ¼ 5, (c) 18, for n ¼ 6, (d)n82 , for n  7.
2.1.23. Let a convex quadrilateral with the sum of the lengths of the diagonals equal
to d0 be inside of a convex quadrilateral with the sum of the lengths of the diagonals
equal to d. Prove that d0 < 2d.
2.1.24. Let a convex polygon with area S2 and perimeter P2 be inside of the convex
polygon with area S1 and perimeter P1. Prove that 2PS11 > PS22 .
2.1.25. Let a convex quadrilateral with the sum of the lengths of the pairwise
distances of its vertices (i.e., the sum of all its sides and diagonals) equal to S2 be
inside of a convex quadrilateral with the same sum equal to S1. Prove that S2 < 4S31 .
2.2 Sufficient Conditions for Comparison of Lengths of Two Broken Lines on the Plane 59

2.1.26. Is it possible to place inside of a unit square two regular triangles with sides
qffiffi
greater than 23, so that they do not intersect?
2.1.27. Prove that a regular n-gon with a side a can be placed inside of a regular
n þ 1-gon with a side a.
2.1.28. Let a parallelogram with altitudes h1 and h2 be inscribed in a convex
quadrilateral ABCD. Prove that quadrilateral ABCD can be placed inside of a
rectangle with one of the sides equal to h1 þ h2.
2.1.29. (a) Prove that, if a convex polygon lays inside of a circle, then the perimeter
of the polygon is less than the circumference of the circle.
(b) Prove that, if a circle lays inside of a convex polygon, then the circumference of
the circle is less than the perimeter of the polygon.
Hint Inscribe regular polygons in the circle.
2.1.30. Let O be a given point in triangle ABC with the lengths of sides AB ¼ c,
BC ¼ a, CA ¼ b, a  b  c. Prove that there exists a vertex of the triangle, such that
the distance from point O does not exceed pbffiffi2.
Hint Consider cases when the point is either in triangle ADC or in ABD, where
AD ⊥ BC(D 2 BC).

2.2 Sufficient Conditions for Comparison of Lengths


of Two Broken Lines on the Plane

2.2.1. (a) Given on a plane two sets of segments A1B1, . . . , AnBn and C1D1, . . . ,
CmDm, the sum of lengths of projections of the segments of the first set on any
straight line of this plane is greater than the sum of the lengths of projections of the
segments of the second set on the same straight line. Prove that the sum of the
lengths of the segments of the first set is greater than the sum of the lengths of the
segments of the second set.
(b) Given on a plane two sets of segments A1B1, . . . , AnBn and C1D1, . . . , CmDm.
Let l1, . . . , ln be a straight line on that plane, such that AiBi ⊥ li, i ¼ 1, . . . , n.
Given that for any i (i ¼ 1, . . . , n) the sum of the lengths of projections of the
segments of the first set on any straight line li is greater than the sum of the
lengths of projections of the segments of the second set on the same straight
line. Prove that the sum of lengths of segments of the first set is greater than the
sum of lengths of the segments of the second set.
(c) Given on a plane two sets of segments A1B1, . . . , AnBn and C1D1, . . . , CmDm.
Let l1, . . . , ln be such straight line on that plane that AiBi ⊥ li, i ¼ 1, . . . , n. It is
known that for any value of i (i ¼ 1, . . . , n) the sum of the lengths of pro-
jections of the segments of the first set on any straight line li is greater than the
60 2 Application of Projection Method

sum of the lengths of projections of the segments of the second set on the same
straight line. Prove that the sum of the lengths of the segments of the first set on
any line on the plane is greater than the sum of the lengths of the segments of
the second set on the same line.
2.2.2. Prove that for points A, B, C, D, E on a plane the following inequality
AB þ CD þ DE þ EC  AC þ AD þ AE þ BC þ BD þ BE holds true.
2.2.3. Four points on the straight line are denoted by letters A, B, C, D. Prove that
AE þ ED þ |AB  CD| > BE þ CE holds true for any point E outside that line.
2.2.4. Given on a plane vectors ~ a, ~ c, ~
b,~ d, whose sum is equal to ~ 0. Prove that
             
~
a þ ~
b þ ~
c þ ~
d   ~ d þ ~
a þ~ b þ~d  þ ~ d .
c þ~
2.2.5. Given on a plane two arbitrary triangles ABC and A1B1C1. Prove that

AA1 þ AB1 þ AC1 þ BA1 þ BB1 þ BC1 þ CA1 þ CB1 þ CC1 


 AB þ BC þ AC þ A1 B1 þ B1 C1 þ C1 A1 :

2.2.6. Given on a plane several segments, whose sum of lengths is equal to π. Prove
that it is possible to choose a straight line, so that the sum of lengths of projections
of the segments on this straight line is (a) less than 2, (b) more than 2.
2.2.7. Given on a plane several vectors, whose sum of lengths is equal to π. Prove
that it is possible to choose several of these vectors, so that the length of their sum is
greater than 1.

Solutions

2.2.1. (a) Given on a plane a segment AB and a straight line l. Consider projection of
segment AB on line l and denote by l(AB) the length of that projection.
If AB ⊥ l, then l(AB) ¼ 0. Similarly, for polygon ϕ denote by l(ϕ) the length of
the projection of polygon ϕ on line l.
It is not difficult to prove that, if point C belongs to segment AB, then l(AB) ¼ l
(AC) þ l(CB). On the other hand, if polygon A1A2 . . . An is convex, then l(A1A2) þ l
(A2A3) þ . . . þ l(An  1An) þ l(AnA1) ¼ 2l(A1A2 ... An) and if segments AB and CD
are either on the same line or on the parallel lines, then
CD  l(AB) ¼ AB  l(CD).
(b) Let us first prove the following lemma.
Lemma If on a plane are given such segments M1N1, M2N2, . . . , MpNp ( p  2),
that no two of them are on the same or parallel lines, then there exists a convex and
M N M N
centrally symmetric polygon with sides M12N1 , M22N2 , :::, p2 p , M12N 1 , M22N2 , :::, p2 p ,
so that the sides with length M2i Ni are parallel to segments MiNi, for any 1  i  p.
We proceed the proof by mathematical induction.
2.2 Sufficient Conditions for Comparison of Lengths of Two Broken Lines on the Plane 61

Figure 2.29 M2 N2
N1

M1

Figure 2.30 d d’
fk Nk+1
fk ¢ A¢
О
A fk ¢¢
Mk+1

Figure 2.31

For p ¼ 2 the proof is presented in Figure 2.29


Assume that the statement holds true for p ¼ k, prove that it holds true for p ¼ k þ 1.
Given segments M1N1, M2N2, . . . , MkNk, Mk þ 1Nk þ 1. Consider a convex cen-
trally symmetric polygon ϕk, the sides of which are equal to M12N1 , :::, Mk2Nk , M12N1 ,
. . . , Mk2N k and are parallel to segments M1N1, M2N2, . . . , MkNk.
Consider a line d||Mk þ 1Nk þ 1 which has a common point with a polygon ϕk and
one of the half-planes with a boundary d contains the ϕk polygon (Figure 2.30).
It is clear that line d contains just one vertex of polygon ϕk, let us denote it by point
A. Let lines d and d0 are symmetric with respect to point O (O is the center of symmetry
of polygon ϕk) and A0 is the symmetric point of point A with respect to point O. Take
on lines d and d0 points B and C, such that AB ¼ A0 C ¼ Mk þ 1Nk þ 1/2 (Figure 2.31).
0
Note that polygon ϕk þ 1, which is the sum of figures ϕ0k , ABCA0 , and ϕ0 k , satisfies
to the conditions of the lemma. This ends the proof of the lemma.
If among segments A1B1, . . . , AnBn there are segments, for example, AiBi and
AjBj, i 6¼ j on the same or parallel lines, then instead of them we consider the
62 2 Application of Projection Method

Figure 2.32 fA

M¢ Bi

M O

Ai

li
E F

segment with length AiBi þ AjBj (parallel to segment AiBi). So, one obtains that
segments A1B1, . . . , AnBn satisfy to the conditions of the lemma.
Let ϕA be a convex polygon with a center of symmetry at point O, with the sides
parallel to segments A1B1, . . . , AnBn and equal to A12B1 , :::, An2Bn , A12B1 , :::, An2Bn .
The polygon (or segment) ϕC, for segments C1D1, . . . , CmDm with the same
symmetry center, is defined similarly.
Let us prove that polygon ϕC is inside of polygon ϕA. Otherwise, there exists
such a point M 2 ϕC, as it is shown in Figure 2.32.
Let M0 be the symmetric point to point M with respect to point O.
We have that

li ðA1 B1 Þ þ . . . þ li ðAn Bn Þ ¼ 2li ðϕA Þ  2MM0  2li ðϕC Þ ¼ li ðC1 D1 Þ þ . . . li ðCm Dm Þ:

Thus li(A1B1) þ . . . þ li(AnBn)  li(C1D1) þ . . . þ li(CmDm).


This leads to a contradiction.
Thus, ϕC is inside of ϕA. Then, according to problem 2.1.1.
A1B1 þ . . . þ AnBn > C1D1 þ . . . þ CmDm.

Remark
1. Similarly, one can prove that if among segments A1B1, . . . , AnBn there are two
located on non-parallel (crossing) lines and li(A1B1) þ . . . þ li(AnBn) 
li(C1D1) þ . . . þ li(CmDm), i ¼ 1, . . . , n, then A1B1 þ . . . þ
AnBn  C1D1 þ . . . þ CmDm.
2. If for any line l it holds true l(A1B1) þ . . . þ l(AnBn)  l(C1D1) þ . . . þ l
(CmDm), then A1B1 þ . . . þ AnBn  C1D1 þ . . . þ CmDm.
(c) Since ϕC is inside of ϕA (see the solution of problem 2.2.1b), then l(ϕC) < l(ϕA),
so 2l(ϕC) < 2l(ϕA). Therefore, l(A1B1) þ . . . þ l(AnBn) > l(C1D1) þ . . . þ l
(CmDm).
Remark One can prove that, if there are given segments A1B1, . . . , AnBn on a plane
and lines li ⊥ AiBi, i ¼ 1, . . . , n, then for any line l of that plane and for arbitrary
mi > 0, i ¼ 1, . . . , n, holds true the following inequality
2.2 Sufficient Conditions for Comparison of Lengths of Two Broken Lines on the Plane 63

m1 lðA1 B1 Þ þ . . . þ mn lðAn Bn Þ  minðm1 l1 ðA1 B1 Þ þ . . . þ mn li ðAn Bn ÞÞ:

2.2.2. According to remark 2 (see the solution of problem 2.2.1b) it is enough to prove
the inequality for the projections of points A, B, C, D, E on a line, i.e., it is enough to
solve the problem for the case when points A, B, C, D, E are on the same line.
As the inequality is symmetric with respect to points C, D, E, then we can take
that E lays between points C and D, then DE þ EC ¼ CD.
According to the triangle inequality AC þ AD  CD, BC þ BD  CD,
AE þ BE  AB. Summing up these three inequalities, we deduce that

AC þ AD þ AE þ BC þ BD þ BE  AB þ CD þ DE þ EC:

2.2.3. Let AB > CD (in the case of AB < CD the proof is similar). One has to prove
that
AE þ ED þ AB > CD þ BE þ CE: ð2:3Þ

According to problem 2.2.1b, it is enough to prove inequality (2.3) for the


projections of points A, B, C, D, E on a line li, where li is perpendicular to one of
the segments AE, ED, AB (see problem 2.2.1b; must be used as follows: if
li(A1B1) þ . . . þ li(AnBn)  li(C1D1) þ . . . þ li(CmDm), i ¼ 1, . . . , n, and there
exists a number j, such that lj(A1B1) þ . . . þ lj(AnBn) > lj(C1D1) þ . . . þ
lj(CmDm), then A1B1 þ . . . þ AnBn > C1D1 þ . . . þ CmDm).
If l ⊥ AE or AB, then we have an equality. While, if l ⊥ ED, then we have a strict
inequality.
In the case of AB ¼ CD, then we have to prove that AE þ ED > BE þ CE. Indeed,
parallelogram EBNC (see Figure 2.33) is inside of parallelogram AEDN.
Hence, 2BE þ 2CE < 2AE þ 2ED (see the solution of problem 2.1.1).
Remark In the case of AB ¼ CD, for the set of segments AE, ED, AB and the set of
segments CD, BE, CE, we have that l(AE) þ l(ED) þ l(AB) ¼ l(CD) þ l(BE) þ l
(CE), at l ⊥ AE or l ⊥ ED or l ⊥ AB. On the other hand, AE þ ED þ
AB > CD þ BE þ CE.

Figure 2.33 E

A B C D

N
64 2 Application of Projection Method

! ! !
2.2.4. Consider points A, B, C, D, such that a, BC ¼ ~
AB ¼ ~ b, CD ¼ ~
c, then
! ~
DA ¼ d. We have to prove that

AB þ BC þ CD þ DA  AC þ BD þ 2MN, ð2:4Þ

where M and N are the midpoints of segments BD and AC. According to remark
2 (see the solution of problem 2.2.1b) it is sufficient to prove inequality (2.4) for
projections of points A, B, C, D on a line, i.e., it is enough to solve the problem for
the case, when points A, B, C, D are on the same line.
We can assume that A(0), B(b), C(c), D(d), and b, c, d  0, d  b, then we must
prove that |c  b| þ |c  d|  c  2b þ |d þ b  c|.
If d þ b  c, then |c  b| þ |c  d|  |(c  b)  (c  d)| ¼ d  b ¼ c  2b þ |
d þ b  c|.
If d þ b < c, then |c  b| þ |c  d| ¼ c  b þ c  d  c  2b þ |d þ b  c|.
2.2.5. According to remark 2 (see the solution of problem 2.2.1b) it is sufficient to
prove the inequality for the projections of points A, B, C, A1, B1, C1 on a line, i.e., it
is enough to prove the problem for the case, when points A, B, C, A1, B1, C1 are on
the same line.
As the inequality is symmetric with respect to points A, B, C (A1, B1, C1), without
loss of generality one can assume that point B lays between points A and C and point
B1 lays between points A1 and C1, A1C1  AC.
Then AB þ BC þ AC þ A1B1 þ B1C1 þ A1C1 ¼ 2AC þ 2A1C1. Since AC  AB1
þ B1C, A1C1  A1B þ BC1, AC  AA1 þ CA1, A1C1  AC  AC1 þ CC1, then by
summing up these inequalities we obtain that

2AC þ 2A1 C1  AA1 þ AB1 þ AC1 þ BA1 þ BC1 þ CA1 þ CB1 þ CC1 :

Therefore

AB þ BC þ AC þ A1 B1 þ B1 C1 þ A1 C1 
 AA1 þ AB1 þ AC1 þ BA1 þ BB1 þ BC1 þ CA1 þ CB1 þ CC1 :

2.2.6. Given segments A1B1, . . . , AnBn. We can assume that any two segments do
not belong to the same or parallel straight lines. For n ¼ 1 the statement of the
problem is evident, let n  2.
Construct a polygon ϕA (see the solution of problem 2.2.1b).
(a) Let O be the center of symmetry of polygon ϕA, and d be the minimal of all
distances between opposite sides (Figure 2.34).

Figure 2.34 l fA

O d
2.2 Sufficient Conditions for Comparison of Lengths of Two Broken Lines on the Plane 65

Figure 2.35
fA

d1
O
l1

Since the circumference with a center O and radius d2 is inside of polygon ϕA, the
length of that circumference is less than the perimeter of polygon ϕA, i.e., πd < π
(see problem 2.1.29b). Then the sum of the lengths of the projections of the
segments A1B1, . . . , AnBn on line l is equal to 2d and 2d < 2 (Figure 2.34).
(b) Consider all diagonals of the ϕA polygon which pass through point
O (Figure 2.35) and let d1 be the length of the largest of them.
Since polygon ϕA is inside of the circumference with a center O and radius d21 , the
perimeter of polygon ϕA is less than the length of that circumference, i.e., πd1 > π
(see problem 2.1.29a). Then the sum of the lengths of the projections of the
segments A1B1, . . . , AnBn on line l1 (Figure 2.35) is equal to 2d1. Thus, it follows
that the sum is larger than 2.
! !  !   ! 
2.2.7. Let be given vectors A1 B1 , :::, An Bn , such that A1 B1  þ ::: þ An Bn  ¼ π.
Consider segments A1B1, . . . , AnBn, then according to problem 2.2.6b there
exists a straight line l, such that l(A1B1) þ . . . þ l(AnBn) > 2 (see notations in the
!
solution of problem 2.2.1). Let ~ e be a unit vector on line l and Ai Bi ,~ e  90 ,
!
i ¼ 1, . . . , k and Ai Bi ,~e > 90 , i ¼ k þ 1, . . . , n. Then, either l(A1B1) þ . . . þ l
(AkBk) > 1 or l(Ak þ 1Bk þ 1) þ . . . þ l(AnBn) > 1. Let l(A1B1) þ . . . þ l(AkBk) > 1.
 ! ! 
We have that A1 B1 þ ::: þ Ak Bk   lðA1 B1 Þ þ ::: þ lðAk Bk Þ > 1. Therefore,
 ! ! 
A1 B þ ::: þ A k Bk > 1.
1

Problems for Self-Study

2.2.8. Points C1, A1, B1 are taken correspondingly on sides AB, BC, AC of triangle
ABC, such that BA1 ¼ λBC, CB1 ¼ λCA, AC1 ¼ λAB, where 12 < λ < 1. Prove that for
perimeter P of triangle ABC and for perimeter P1 of triangle A1B1C1 it holds true
(2λ  1)P < P1 < λP.
2.2.9. Point O is inside of triangle ABC with a perimeter P. Prove that
P
2 < AO þ BO þ CO < P.
66 2 Application of Projection Method

2.2.10. Prove that, if the lengths of all sides and diagonals of a convex polygon is
less than d, then its perimeter is less than πd.
2.2.11. Given several convex polygons and it is known that it is impossible to draw
a straight line, so that it does not intersect any polygon and on both sides of it there
is at least one polygon. Prove that all these polygons can be confined within a
polygon with the perimeter not exceeding the sum of their perimeters.
2.2.12. Given on a plane several vectors, whose sum of the length is equal to 1.
Prove that they can be broken into three groups (possibly empty), so that the sum
pffiffi
of the lengths of the vectors in these groups is more than 32π3.
Remark Add to these n vectors another 2n vectors obtained from the vectors of the
given set by rotation by angle 120 clockwise or counterclockwise.
2.2.13. Given on a plane a convex n-gon. Let ak be the length of its k-th side and dk be
the length of the projection of n-gon on a line containing that side (k ¼ 1, 2, . . . , n).
Prove that da11 þ ::: þ dann  4.

Remark First prove the statement of the problem for centrally symmetric polygons.

2.3 Inscribed Polygons with the Least Perimeter

2.3.1.
 Given convex polygon A1A2 . . . An and  points B1, B2, . . . , Bn on its sides
B1 2 A1 A2 ; B2 2 A2 A3 ; :::; Bn 2 An A1 ; Bi Aj , so that for any points C1, . . . , Cn on
the side of polygon A1 . . . An (C1 2 A1A2, . . ., Cn 2 AnA1, Ci Aj ) the inequality
B1B2 þ B2B3 þ . . . þ Bn  1Bn þ BnB1  C1C2 þ C2C3 þ . . . þ Cn  1Cn þ CnC1
is true.
Prove that

∠Bn B1 A1 ¼ ∠B2 B1 A2 , ∠B1 B2 A2 ¼ ∠B3 B2 A3 , :::, ∠Bn1 Bn An


¼ ∠B1 Bn A1 : ð2:5Þ

2.3.2.
 Given convex polygon A1A2 . . . An and  points B1, B2, . . . , Bn on its sides
B1 2 A1 A2 ; B2 2 A2 A3 ; :::; Bn 2 An A1 ; Bi Aj , so that condition (2.5) of problem
2.3.1 is satisfied. Prove that for any points C1, . . . , Cn on the sides of polygon
A1A2 . . . An (C1 2 A1A2, ..., Cn 2 AnA1) the following inequality is true:
B1B2 þ B2B3 þ . . . þ Bn  1Bn þ BnB1  C1C2 þ C2C3 þ . . . þ Cn  1Cn þ CnC1.
2.3.3. Prove that if A1A2A3 is not an acute-angled triangle, then on its sides do not
exist such points B1, B2, B3 (B1 2 A1A2, B2 2 A2A3, B3 2 A3A1, Bi Aj) for which the
condition (2.5) of problem 2.3.1 is satisfied.
2.3.4. Prove that if A1A2A3 is an acute-angled triangle, then on its sides exist such
points B1, B2, B3 (B1 2 A1A2, B2 2 A2A3, B3 2 A3 A1 , Bi Aj for which the condition
(2.5) of problem 2.3.1 is satisfied.
2.3 Inscribed Polygons with the Least Perimeter 67

2.3.5. Given convex quadrilateral A1A2A3A4 and points B1, B2, B3, B4 on the sides of
that quadrilateral (B1 2 A1A2, B2 2 A2A3, B3 2 A3A4, B4 2 A4A1, Bi Aj ) so that the
condition (2.5) of problem 2.3.1 is satisfied. Prove that quadrilateral A1A2A3A4 is
inscribed and that

maxð∠A1 A2 A4 ; ∠A3 A2 A4 ; ∠A2 A3 A1 ; ∠A4 A3 A1 ; ∠A3 A4 A2 ; ∠A1 A4 A2 ; ∠A4 A1 A3 ; ∠A2 A1 A3 Þ


π
< :
2
ð2:6Þ

2.3.6. Prove that if a convex quadrilateral A1A2A3A4 is inscribed and for it condition
(2.6) of problem 2.3.5 is satisfied, then on the sides of that quadrilateral exist such
points B1, B2, B3, B4 (B1 2 A1A2, B2 2 A2A3, B3 2 A3A4, B4 2 A4A1, Bi Aj), for which
the condition (2.5) of problem 2.3.1 is satisfied.
2.3.7. Given a regular tetrahedron A1A2A3A4 with edge 1 and points B1, B2, B3, B4 on
its faces A2A3A4, A1A3A4, A1A2A4, and A1A2A3, respectively. Prove that
pffiffiffiffiffi
B1 B2 þ B2 B3 þ B3 B4 þ B4 B1  0, 4 10.
2.3.8. Given a cub ABCDA0 B0 C0 D0 with edge 1 and points B1, B2, B3, B4, B5, B6 on its
faces ABCD, AA0 B0 B, BB0 C0 C, A0 B0 C0 D0 , DD0 C0 C, AA0 D0 D, respectively. Prove that
pffiffiffi
B1 B2 þ B 2 B3 þ B 3 B4 þ B 4 B5 þ B 5 B6 þ B 6 B1  2 3 .

Solutions

2.3.1. Assume that condition (2.5) is not correct and let

∠Bn B1 A1 6¼ ∠B2 B1 A1 : ð2:7Þ

Consider point B02 symmetric to B2 with respect to straight line A1A2


(Figure 2.36).
Denote the intersection point of straight lines B02 Bn and A1A2 by B. Since
∠B2 BM ¼ ∠B02 BM ¼ ∠Bn BA1 , then according to (2.7) points B and B1 differ.
0 0
Take a point B1 on segment BB1 so that point B1 would lay on side A1A2. Let us
0
consider points B1 , B2 , :::, Bn . According to problem 1.1.8a,

Figure 2.36 An

Bn
A3
B2

M A2 B B1 A1
B'2
68 2 Application of Projection Method

Figure 2.37 ln
Bn-1
An

Bn

A3 bn l1

B2 b1 C1 A1
C1'
b1 B1
A2
C1''
l2

B02 B01 þ B01 Bn < B02 B1 þ B1 Bn , thus B2 B01 þ B01 Bn < B2 B1 þ B1 Bn , which means
that

B01 B2 þ B2 B3 þ ::: þ Bn1 Bn þ Bn B01 < B1 B2 þ B2 B3 þ Bn1 Bn þ Bn B1 :

This leads to a contradiction.


2.3.2. Let ∠BnB1A1 ¼ ∠B2B1A2 ¼ β1, ∠B1B2A2 ¼ ∠B3B2A3 ¼ β2, . . ., ∠Bn  1BnAn ¼
∠B1BnA1 ¼ βn. Draw through vertices A1, A2, . . . , An straight lines l1, l2, . . . , ln
parallel to lines B1Bn, . . , Bn  1Bn, respectively (Figure 2.37).
00 0
Let C10 and C1 be projections of point C1 on lines l1 and l2, C20 and C0 2 -
00
projections of point C2 on lines l2 and l3, etc., Cn0 and Cn - projections of point Cn
on lines ln and l1. We have that
00 00 00 00 00
C1 C2 þ C2 C3 þ ::: þ Cn1 Cn þ Cn C1  C1 C2 þ C2 C02 þ ::: þ Cn1 C0n þ Cn C01 ¼
¼ ðA2 C1 cos β1 þ A2 C2 cos β2 Þ þ ::: þ ðC1 A1 cos β1 þ Cn A1 cos βn Þ ¼
¼ A1 A2 cos β1 þ A2 A3 cos β2 þ ::: þ An A1 cos βn ¼
¼ ðA2 B1 cos β1 þ B1 A1 cos β1 Þ þ ::: þ ðA1 Bn cos βn þ An Bn cos βn Þ ¼
¼ B1 B2 þ B2 B3 þ ::: þ Bn1 Bn þ Bn B1 :

hence

C1 C2 þ C2 C3 þ ::: þ Cn1 Cn þ Cn C1  B1 B2 þ B2 B3 þ ::: þ Bn1 Bn þ Bn B1 :


2.3 Inscribed Polygons with the Least Perimeter 69

Remark
1. The equality holds true if and only if

C1 C2 k B1 B2 , C2 C3 k B2 B3 , :::, Cn1 Cn k Bn1 Bn , Cn C1 k Bn B1 :

2. For odd n the equality holds if C1 B1, C2 B2, . . . , Cn Bn.


2.3.3. Let ∠A1  90 and for points B1, B2, B3 condition (1) of problem 2.3.1 is
satisfied (Figure 2.38).
We have that

∠A2 þ ∠A3 ¼ 360  ð∠B2 B3 A3 þ ∠B2 B1 A2 Þ  ð∠B3 B2 A3 þ ∠B1 B2 A2 Þ ¼


¼ 180  ð∠B1 B3 A1 þ ∠B3 B1 A1 Þ þ 180  ð∠B3 B2 A3 þ ∠B1 B2 A2 Þ
¼ ∠A1 þ ∠B1 B2 B3 > ∠A1 :

Hence ∠A2 þ ∠A3 > ∠A1, which is wrong.


2.3.4. Let A3B1, A1B2, A2B3 be the altitudes of acute triangle A1A2A3 (Figure 2.39).

Figure 2.38 A1

B3
B1

A3 B2 A2

Figure 2.39 A1

B1
B3

A3 B2 A2
70 2 Application of Projection Method

Figure 2.40 A2

B2
B1
P
O A3
A1
B3
B4

A4

Since points B1 and B3 lay on the circumference with a diameter A1H, we have
∠B3B1A1 ¼ ∠B3HA1. Similarly we get that ∠B2B1A2 ¼ ∠B2HA2 and since
∠B3HA1 ¼ ∠B2HA2, then ∠B3B1A1 ¼ ∠B2B1A2.
The other two equalities are proved similarly.
Remark According to problems 2.3.1 and 2.3.3, a triangle with a minimum perim-
eter can be inscribed in the given triangle A1A2A3 if it is an acute triangle (necessary
condition).
On the other hand, according to problems 2.3.2 and 2.3.4 this is also a sufficient
condition.
2.3.5. We have that (Figure 2.40)
∠A2 þ ∠A4 ¼ π  ∠B2 B1 A2  ∠B1 B2 A2 þ π  ∠B4 B3 A4  ∠B3 B4 A4
¼ π  ∠B4 B1 A1  ∠B1 B4 A1 þ π  ∠B3 B2 A3  ∠B2 B3 A3 ¼ ∠A1 þ ∠A3 ,

thus quadrilateral A1A2A3A4 is inscribed.


Let us assume, that one of these angles is not less than π2, let ∠A1 A3 A4  π2.
Let ρ(M, l ) be the distance from point M to straight line l. Note that ρ(A3,
B1B2) ¼ ρ(A3, B2B3) ¼ ρ(A3, B3B4), similarly ρ(A1, B1B2) ¼ ρ(A1, B3B4). Hence line
A1A3 is a locus of points equidistant from straight lines B1B2 and B3B4 (in the region
of Π). Thus the bisectors of angles ∠B1B2B3 and ∠B4B3B2 intersect on line A1A3,
let that point be O. Note that points O and A3 are on the different sides of line B2B3,
which means that point O belongs to ray A3A1 and since ∠OB3 A3 ¼ π2, then
∠A1 A3 A4 < π2, which is wrong.
2.3.6. Let the diagonals of inscribed quadrilateral A1A2A3A4 intersect at point O and
points B1, B2, B3, B4 be the orthogonal projections of point O on the sides of
quadrilateral A1A2A3A4 (Figure 2.41).
We will prove that points B1, B2, B3, B4 satisfy condition (1) of problem 2.3.1.
2.3 Inscribed Polygons with the Least Perimeter 71

Figure 2.41 A2
B2
A3
B1

O
B3

А1

B4

A4

Since points B1 and B4 are on the circle with diameter A1O, then
∠B4B1A1 ¼ ∠B4OA1. We have ∠B4 OA1 ¼ π2  ∠B4 A1 O ¼ π2  ∠B2 A2 O ¼
∠B2 OA2 .
Since points B1 and B2 are on the circle with diameter A2O, then
∠B2OA2 ¼ ∠B2B1A2. Thus ∠B4B1A1 ¼ ∠B2B1A2. The other three inequalities
one can prove similarly.
Remark
1. According to problems 2.3.1 and 2.3.5, a quadrilateral with a minimum perim-
eter can be inscribed into the given quadrilateral A1A2A3A4 if the latter is
inscribed and condition (2.6) of problem 2.3.5 is satisfied (necessary condition),
and according to problems 2.3.2 and 2.3.6 these are also sufficient conditions.
2. If a quadrilateral with a minimum perimeter can be inscribed into the given
quadrilateral A1A2A3A4 then the number of these would be infinite. According to
remark 1 to problem 2.3.2, corresponding sides of all minimum possible perim-
eter quadrilaterals inscribed into A1A2A3A4 quadrilateral will be parallel to each
other (Figure 2.42).

2.3.7. Let M be the midpoint of edge A1A3. Denote points symmetric to B1, B2, B3,
B4 with respect to plane A2MA4 by B01 , B02 , B03 , B04 , respectively, and the midpoints of
the segments B03 B1 , B2 B02 , B3 B01 , B4 B04 by C1, C2, C3, C4,respectively.
Note that points C1 and C3 are on facets A2A3A4 and A1A2A4 respectively, and
points C2 and C4 are on segments MA4 and MA2, respectively
According to problem 1.1.9a (see the solution) we have that

B 1 B2 þ B 2 B3 B 2 B3 þ B1 B2 B3 B4 þ B4 B1 B1 B 4 þ B3 B 4
¼ þ þ þ
2 2 2 2
¼ B 1 B2 þ B 2 B3 þ B 3 B4 þ B4 B1 ,

thus C1C2 þ C2C3 þ C3C4 þ C4C1  B1B2 þ B2B3 þ B3B4 þ B4B1.


72 2 Application of Projection Method

Figure 2.42 A2

B1

B2
A1

A3
B4
B3

A4

Let N be the midpoint of edge A2A4.


Denote points symmetric to C1, C2, C3, C4 with respect to plane A1NA3 by
C01 , C02 , C03 , C04 , respectively, and the midpoints of segments C1 C01 , C2 C04 , C3 C03 , C4
C02 by D1, D2, D3, D4,respectively. Note that points D1, D2, D3, D4 are on segments
NA3, MA4, NA1, MA2, respectively. According to problem 1.1.9a we get

D1 D2 þ D2 D3 þ D3 D4 þ D4 D1 
C1 C2 þ C04 C01 C2 C3 þ C03 C04 C3 C4 þ C02 C03 C1 C4 þ C01 C02
 þ þ þ
2 2 2 2
¼ C1 C2 þ C2 C3 þ C3 C4 þ C4 C1 :

Thus D1D2 þ D2D3 þ D3D4 þ D4D1  C1C2 þ C2C3 þ C3C4 þ C4C1, which
means that D1D2 þ D2D3 þ D3D4 þ D4D1  C1C2 þ C2C3 þ C3C4 þ C4C1 
B1B2 þ B2B3 þ B3B4 þ B4B1, consequently,

B1 B2 þ B2 B3 þ B3 B4 þ B4 B1  D1 D2 þ D2 D3 þ D3 D4 þ D4 D1 
 ρðNA3 ; MA4 Þ þ ρðMA4 ; NA1 Þ þ ρðNA1 ; MA2 Þ þ ρðMA2 ; NA3 Þ,

where ρ(l1, l2) is the distance between straight lines l1 and l2.
We shall prove that one can choose on segments MA4 and NA3 points K and E,
respectively, so that KE ⊥ MA4, KE ⊥ NA3, then ρ(NA3, MA4) ¼ KE (Figure 2.43).
! ! !
Denote A3 A ¼ ~
1 a, A3 A ¼ ~
4 b, A3 A2 ¼ ~
c, A3 E ¼ λ, A4 K ¼ μ.
A3 N A4 M
We have that

! ¼ ! ! !
EA 3 þ A3 M þ MK ¼
EK
 
! ~ a ! λ
~
a ~
a
¼ λA3 N þ þ ð1  μÞMA 4 ¼¼  ~ c þ þ ð1  μ Þ ~
b þ~ b ¼
2 2 2 2
1

¼ μ~a þ ðλ þ 2  2μÞ~


b  λ~
c :
2
2.3 Inscribed Polygons with the Least Perimeter 73

Figure 2.43 A4

A2

K b

A1

M c

a

A3

 
  ! ~
!
Since aj ¼ ~
j~ b ¼ j~ a~
cj ¼ 1,~b ¼~ c ¼~
b~ a ¼ 12
c~ and EK ⊥ b þ~ c , EK ⊥


2~
b ~a , then μ~ a þ ð2  λ  2μÞ~ c ~
b  λ~ b þ~ c ¼ 0 and ðμ~
a þ ð2  λ  2μÞ

~ cÞ 2~
b  λ~ b ~a ¼ 0, thus 3λ  2μ þ 3 ¼ 0 and 2λ  3μ þ 3 ¼ 0. Hence, we
obtain that λ ¼ μ ¼ 35.
  r ffiffiffiffiffiffiffiffiffiffiffiffiffiffiffiffiffiffiffiffiffiffiffiffiffiffiffiffiffiffiffi

2ffi
 ! pffiffiffiffiffi
Then ρðNA3 ; MA4 Þ ¼ EK ¼  EK  ¼ 0, 1 3~ ~
a þ b  3~ c ¼ 0, 1 10. Simi-
pffiffiffiffiffi
larly we get that ρðMA4 ; NA1 Þ ¼ ρðNA1 ; MA2 Þ ¼ ρðMA2 ; NA3 Þ ¼ 0, 1 10, thus
pffiffiffiffiffi
B1 B2 þ B2 B3 þ B3 B4 þ B4 B1  0, 4 10.
Another solution of this problem can be obtained by using a problem 4.1.24b.
Remark If points B1, B2, B3, B4 are on segments A3N, A4M, A1N, A2M, respectively,
pffiffiffiffiffi
and AA33BN1 ¼ AA44BM2 ¼ AA11BN3 ¼ AA22BM4 ¼ 35, then B1 B2 þ B2 B3 þ B3 B4 þ B4 B1 ¼ 0, 4 10.

2.3.8. Consider Cartesian coordinate system in the space and let B(0; 0; 0), A
(1; 0; 0), C(0; 1; 0), B0 (0; 0; 1), B1(x; y; 0).
Denote the points symmetric to B1, D1, D2, D3, D4 with respect to planes AA0 B0 B,
BB0 C0 C, A0 B0 C0 D0 , DD0 C0 C, AA0 D0 D by D1, D2, D3, D4, and D5, respectively. Then
D1(x; y; 0), D2(x; y; 0), D3(x; y; 2), D4(x; 2 þ y; 2), D5(2 þ x; 2 þ y; 2).
Note that

B1 B2 þ B2 B3 þ B3 B4 þ B4 B5 þ B5 B6 þ B6 B1 ¼
¼ D 1 B2 þ B2 B3 þ B3 B4 þ B4 B5 þ B5 B6 þ B6 B1 
 D 1 B3 þ B3 B4 þ B4 B5 þ B5 B6 þ B6 B1 ¼ D 2 B3 þ B3 B4 þ B4 B5 þ B5 B6 þ B6 B1 
 D 2 B4 þ B4 B5 þ B5 B6 þ B6 B1 ¼ D 3 B4 þ B4 B5 þ B5 B6 þ B6 B1 
 D 3 B5 þ B5 B6 þ B6 B1 ¼ D 4 B5 þ B5 B6 þ B6 B1  D 4 B6 þ B6 B1 ¼
¼ D 5 B6 þ B6 B1  D 5 B1 ,
74 2 Application of Projection Method

pffiffiffi
consequently B1 B2 þ B2 B3 þ B3 B4 þ B4 B5 þ B5 B6 þ B6 B1  D5 B1 ¼ 2 3.
(See also problem 4.1.24c.)

Problems for Self-Study

2.3.9. Given an inscribed quadrilateral ABCD. Prove that the perimeter of the
quadrilateral inscribed into ABCD is greater than or equal to 2AC sin ∠A.
2.3.10. Given straight lines l, m, and n. Find the triangle with the minimum possible
perimeter and the vertices on lines l, m, and n.
2.3.11. Given an inscribed quadrilateral ABCD. Find on lines AB, BC, CD, DA, such
points B1, B2, B3, B4, respectively, for which the sum B1B2 þ B2B3 þ B3B4 þ B4B1
would be minimal.

2.4 Method of Projections

2.4.1. Prove that radius of the incircle of the right-angled triangle is less than half of
its leg.
2.4.2. Several circles, the sum of lengths of which is equal to 10, are located inside a
unit square. Prove that one can find a straight line crossing at least four of these circles.
2.4.3. Non-self-crossing broken line of length 1000 is located inside a unit square.
Prove that that one can find a straight line parallel to one of the sides of a square
which would cross at least 500 segments of this broken line.
2.4.4. Given two straight lines a and b. A1B1, A2B2, A3B3 are perpendiculars drawn
from three consecutive points A1, A2, A3 on line a to line b.
Prove that A2B2  max (A1B1, A3B3).
2.4.5. Diagonals of a convex quadrilateral ABCD intersect at point O. Let P and Q be
the centers of circumcircles of triangles AOB and COD. Prove that PQ > ABþCD 4 .

2.4.6. Points A1, B1, and C1 are taken on sides BC, CA, and AB of non-obtuse
triangle ABC, respectively. Prove that 2(B1C1 cos α þ C1A1 cos β þ A1B1 cos γ) 
BC cos α þ AC cos β þ AB cos γ, where ∠A ¼ α, ∠B ¼ β, ∠C ¼ γ. Give an example
of an obtuse triangle for which the inequality does not hold.
2.4.7. Let circles of unit radiuses have no common internal points and be inside of a
band S. The band is formed by two parallel lines having a distance w. Let us call
these circles a k-cloud, if each line that intersects S, intersects also more than or
pffiffiffi
equal to k circles. Prove that for k-cloud w  2 þ 3 2k , where k 2 N and [a] is the
integer part of the number a.
2.4 Method of Projections 75

2.4.8. Let ON be the radius of the circle with a center at point O, which intersects
chord AB at point M at a right angle. Let P be an arbitrary point on the larger arc AB,
not coinciding with the point diametrically opposite to point N. Straight lines PM
and PN define points Q and R on the circumference and on chord AB, respectively.
Prove that RN > MQ.
2.4.9. In convex pentagon ABCDE, side AB is perpendicular to side CD, and side BC
is perpendicular to side DE. Prove that, if AB ¼ AE ¼ ED ¼ 1, then BC þ CD < 1.
2.4.10. (a) Opposite sides of convex hexagon ABCDEF are parallel (AB||DE, BC||
EF, CD||FA). Prove that RA þ RC þ RE  P, where RA, RC, RE are the radiuses of the
circumcircles of triangles FAB, BCD, and DEF, respectively, and p is the half-
perimeter of hexagon ABCDEF.
(b) Let M be an arbitrary point inside triangle ABC, Ra, Rb, Rc the distances of point
M from A, B, C, da, db, dc the distances from point M to lines BC, AC, and AB,
respectively. Prove that Ra þ Rb þ Rc  2da þ 2db þ 2dc.
(c) Let M be a point inside triangle ABC. Prove that one of the angles
∠MAB, ∠MBC, ∠MCA is less than or equal to 30 .
2.4.11. Prove that for an acute triangle ha þ hb þ hc  4R þ r.
2.4.12. Given points U and V on sides AB and CD of square ABCD, respectively.
Let straight lines DU and AV intersect at point P, and lines CU and BV at point Q.
Prove that PQ  12 AB.
2.4.13. Given 110 unit vectors on a plane, the sum of these being a zero vector.
Prove that of these 110 vectors one can choose such 55 vectors, that the modulus of
the sum of which will not be greater than 1.
2.4.14. Let the diagonals of convex quadrilateral ABCD intersect at point P. Points
Q, R, S, and T are the feet of the perpendiculars drawn from point P to lines AB, BC,
CD, and DA, respectively. Prove that

1
PQ þ PR þ PS þ PT  ðAB þ BC þ CD þ DAÞ:
2

2.4.15. Let ABCDEF be a convex hexagon. Given that AB ¼ CD ¼ EF. Prove that
AD þ BE þ CF  AB þ BC þ CD þ DE þ EF þ FA. For which hexagon does equal-
ity hold true?
2.4.16. The vertices of tetrahedron KLMN lay either (inside) on the facets or on the
edges of another tetrahedron ABCD. Prove that the sum of the lengths of all edges of
tetrahedron KLMN is less than 43 of the sum of the lengths of all edges of
tetrahedron ABCD.
2.4.17. Let M be a point inside a convex n-gon A1A2 . . . An and p be half-perimeter
∠An
of that n-gon. Prove that MA1 cos ∠A ∠A2
2 þ MA2 cos 2 þ ::: þ MAn cos 2  p.
1

2.4.18. (a) Points M, N, P are chosen on edges A1B1, C1C and AD of unit cube
ABCDA1B1C1D1, respectively. Prove that the perimeter of triangle MNP is not less
pffiffiffi
than 3 6=2.
76 2 Application of Projection Method

(b) Given a unit cube ABCDA1B1C1D1. Prove that the distance from the arbitrary
point in the space to one of straight lines A1B1, C1C, AD is not less than p1ffiffi2.
(c) Given two spheres with diameters d and D inside a unit cube so that they do not
pffiffiffi
have common points. Prove that d þ D < 3  3.

Solutions

2.4.1. Consider the projection of a circle on the edge.


2.4.2. Let us consider the projection of these circles on side AB of the square. The
sum of the lengths of these projections is equal to 10 π > 3. Thus, several segments
Δ1, Δ2, . . . , Δk, with the sum of their lengths greater than 3, are located on the unit
segment. We shall prove that there exists a point which belongs to at least four
segments.
Let different points A1, A2, . . . , Am be all endpoints of segments Δ1, Δ2, . . . , Δk
(Figure 2.44).
Replace segments Δ1, Δ2, . . . , Δk by segments of type AiAi þ 1, i ¼ 1, 2, . . . ,
m  1, segment AiAi þ 1 being taken as many times as it belongs to segments
Δ1, Δ2, . . . , Δk. It is clear that, by this replacement, the sum of the lengths of all
segments will not change.
Thus we can assume that any two segments either do not have any common
internal point or do not coincide. Let M1, M2, . . . , Mn be the left endpoints of these
segments. If each point Mi is a left endpoint of no more than three of those
segments, then taking one of each segments with left endpoints M1, M2, . . . , Mn
we obtain that the sum of the lengths of these segments is not greater than 1. Then
the sum of the lengths of all segments will be not greater than 3. This means that
there exists a point Mi which is a left endpoint of at least four segments. Thus there
exists a point M which is an internal point of at least four segments. Then straight
line l which passes through point M and is perpendicular to side AB of the square
would intersect at least four of these circles.
2.4.3. Consider the projections of the segments of the broken line on the two perpen-
dicular sides of the square. Let the lengths of the projections of the segments on one of
the sides be a1, a2, . . . , an and those on the other side b1, b2, . . . , bn (Figure 2.45).
Note that ai þ bi  ci, where ci is the length i-th segment of the broken line.
Therefore (a1 þ b1) þ . . . þ (an þ bn)  1000. Hence, it follow that a1 þ . . . þ
an  500 or b1 þ b2 þ . . . þ bn  500. To conclude the proof, see the solution of
problem 2.4.2.
2.4.4. If straight lines a and b are in the same plane, then it is easy to see that min
(A1B1, A3B3)  A2B2  max (A1B1, A3B3).

Figure 2.44 A B
A1 A2 Am
2.4 Method of Projections 77

Figure 2.45

bi ci bi
ai

ai

Figure 2.46 A1¢

B A2¢

A3¢

If the straight lines a and b are not in the same plane then we shall consider the
projections of points A1, A2, A3, B1, B2, B3 on the plane perpendicular to line b. Let
those points are A01 , A02 , A03 , and B (Figure 2.46).
  
Since max ∠BA0 2 A03 ; ∠BA0 2 A01  90 , hence it follows that max(BA0 1, BA0 3) >
BA0 2 and we have that BA0 1 ¼ B1A1, BA0 2 ¼ B2A2, BA0 3 ¼ B3A3. Therefore
A2B2 < max (A1B1, A3B3).
Remark Given segments AB and CD, then inequality SABM  max (SABC, SABD)
holds true for any point M of segment CD.
2.4.5. Note that the lengths of the projections of segment PQ on lines AC and BD
2 and 2 , respectively. Thus PQ þ PQ > 2 þ 2 , consequently, PQ
are equal to AC BD AC BD

> 4 > 4 (see the remarks of problem 1.1.4a).


ACþBD ABþCD

2.4.6. Let A2C2 be the projection of segment A1C1 on side AC. Since β  π2 and
A1C1  A2C2, therefore we deduce that 2A1C1 cos β  2A2C2 cos β ¼ 2 cos β
(AC  AC1 cos α  CA1 cos γ).
Similarly we get that 2A1B1 cos γ  2 cos γ(AB  AB1 cos α  BA1 cos β) and
2B1C1 cos α  2 cos α(BC  BC1 cos β  CB1 cos γ).
78 2 Application of Projection Method

Summing up these three inequalities we obtain that

2ðB1 C1 cos α þ C1 A1 cos β1 þ A1 B1 cos γ Þ  2AC cos β þ 2AB cos γ þ 2BC cos α
 2cos α  cos βðAC1 þ C1 BÞ  2cos β cos γ ðBA1 þ A1 CÞ  2 cos γ cos αðAB1 þ B1 CÞ
¼ 2BC cos α þ 2AC cos β þ 2AB cos γ  cos αðAB cos β þ AC cos γ Þ
 cos βðAB cos α þ BC cos γ Þ  cos γ ðAC cos α þ BC cos βÞ ¼
¼ 2BC cos α þ 2AC cos β þ 2AB cos γ  BC cos α  AC cos β  AB cos γ ¼
¼ BC cos α þ AC cos β þ AB cos γ:

Hence, it follows that 2(B1C1 cos α þ C1A1 cos β1 þ A1B1 cos γ)  BC cos α þ
AC cos β þ AB cos γ.
For α ¼ γ ¼ π6, AC1 ¼ AB 3 ¼ 3, and C1B1 k CB, B1A1 k AB we have that C1 B1 ¼ 3,
a a
pffiffi
A1 B1 ¼ 23 a , A1 C1 ¼ 37 a. Thus

2ðB1 C1p ffiffiffi α p


cos þffiffiC pffiffiffi β1 þ A1 B1 cos γ Þ
ffi 1 A1 cos
3 3 7 3a
¼ a< ¼ BC cos α þ CA cos β þ AB cos γ:
3 2

2.4.7. Draw through the center of a certain circle of the k-cloud a straight line l,
which is perpendicular to the boundaries of band S. Then that line l should intersect
not less than k  1 other circles. Line l divides the plane into two half-planes. It is
not difficult to understand that one of the half-planes contains not less than 2k þ 1
centers of those circles (including k the centers which are on line l ). Let O1, O2, . . . ,
On be these centers, where n ¼ 2 þ 1 (Figure 2.47).
Let points O01 , O02 , :::, O0n be the projections of points O1, O2, . . . , On on line l.
Denote by d i ¼ Oi O0i , i ¼ 1, . . . , n. Note that for i ¼ 1, . . . , n  1 we have that
qffiffiffiffiffiffiffiffiffiffiffiffiffiffiffiffiffiffiffiffiffiffiffiffiffiffiffiffiffiffiffiffiffiffiffiffiffiffiffiffiffiffiffi qffiffiffiffiffiffiffiffiffiffiffiffiffiffiffiffiffiffiffiffiffiffiffiffiffiffiffiffiffiffiffiffi pffiffiffiffiffiffiffiffiffiffiffi pffiffiffi
O0i O0iþ1 ¼ Oi O2iþ1  ðdi  diþ1 Þ2  4  ðd i  diþ1 Þ2  4  1 ¼ 3,
pffiffiffi pffiffiffi
since 0  di  1, i ¼ 1, . . . , n. Consequently O01 O0n  3ðn  1Þ ¼ 3 2k , where
O01 and O0n are, respectively, the lowest and the highest points on line l.

Figure 2.47 l

On¢ On
O¢j O j
O¢2 O2
O¢1 O1
2.4 Method of Projections 79

Figure 2.48 L

K
A T M R B

Q N

Since band S should extend, at least by the radius of the circle, on each side of the
ends of segment O01 O0n , to contain the circles with centers O1 and On, then it should
pffiffiffi
have a width w  2 þ 3 2k .
2.4.8. Consider the figure below (Figure 2.48).
Segments MR and KM are projections of segments NR and QM on lines AB and
KM, respectively. We have that ∠QMK ¼ π2  ∠KQM ¼ π2  ∠MNR ¼ ∠MRN.
Therefore, it will be sufficient to prove that MR > MK. Note that
_ _ _ _
∠BRP ¼ BPþAN
2 ¼ BPþBN
2 ¼ ∠MLP.
This means that MLPR is an inscribe quadrilateral. Since ∠MLR ¼
∠MPR ¼ ∠TLM, then MR ¼ MT > MK.
2.4.9. Let ∠CDB ¼ α, ∠CBD ¼ β, α  β, and A0 E0 is the projection of segment AE
on line BD. We have that AE  A0 E0 ¼ BD þ sin α þ sin β, consequently

1  sin α  sin β 1  sin α 1  sin 2 α


BD  1  sin α  sin β  < < <
αβ αβ αβ
cos cos cos
2 2 2
αþβ
cos α cos
<  2 ¼ sin ðα þ βÞ :
αβ αβ sin α þ sin β
cos cos
2 2

Hence

sin ðα þ βÞ
BD < : ð2:8Þ
sin α þ sin β

BD sin α BD sin β
Since BC þ CD ¼ sin ðαþβÞ þ sin ðαþβÞ, then according to (2.8) we get
BC þ CD < 1.
2.4.10. (a) Consider Figure 2.49.
We have that BF  MN and BF  PK. Thus, we deduce that BF  MNþPK
2 or
2RA sin α  ða sin βþf sin γÞþ2 ðc sin γþd sin βÞ. Therefore, it follows that
80 2 Application of Projection Method

Figure 2.49 M B b C P
b g
a c
A a

D
f a
d
g b
N F e E K

1 sin β 1 sin γ
RA  ða þ dÞ þ ðf þ cÞ : Similarly we get that
4 sin α 4 sin α
1 sin α 1 sin β
RC  ðf þ cÞ þ ðb þ eÞ ,
4 sin γ 4 sin γ
1 sin γ 1 sin α
RE  ðb þ eÞ þ ða þ dÞ :
4 sin β 4 sin β

Summing up these inequalities and making use of inequality x þ 1x  2ðx > 0Þ,
we obtain that RA þ RC þ RE  12 ða þ dÞ þ 12 ðb þ eÞ þ 12 ðf þ cÞ ¼ p. This ends
the proof.
Remark Equality holds if and only if hexagon ABCDEF is regular.
(b) Draw perpendiculars MB1, MD1, MF1 to lines BC, AC, AB, respectively, and
construct parallelograms B1MF1A1, D1MB1C1, F1MD1E1.
Since the radiuses of the circumcircles of triangles D1MF1, B1MF1 B1MD1 are
equal to R2a , R2b , R2c , with R2a ¼ RE1 , R2b ¼ RA1 , R2c ¼ RC1 , then according to problem
2.4.10a we have that R2a þ R2b þ R2c  p ¼ da þ db þ dc . Thus

Ra þ Rb þ Rc  2da þ 2db þ 2d c :

(c) According to problem 2.4.10b MA þ MB þ MC  2MC1 þ 2MA1 þ 2MB1,


where MA1 ⊥ BC, MB1 ⊥ AC, MC1 ⊥ AB (Figure 2.50). Thus, at least one of
the following inequalities MA  2MC1, MB  2MA1, MC  2 MB1 is correct.
Let MA  2MC1, consequently either ∠MAB  30 or ∠MAB  150 . If
∠MAB  150 , then ∠MBC < 30 .
2.4 Method of Projections 81

Figure 2.50 B
A1

C1
M

A B1 C

Figure 2.51 B
A2

C1 A1
kc ka
O
kb
A B1 C

2.4.11. Let O be the center of the circumcircle of acute triangle ABC and ka, kb, kc be
the distances from point O to its sides (Figure 2.51).
Lemma Prove that ka þ kb þ kc ¼ R þ r.
Let A1, B1, and C1 be the midpoints of sides BC, CA, and AB, respectively. By
Ptolemy’s theorem akc þ cka ¼ bR akb þ bka ¼ cR, ckb þ bkc ¼ aR. On the other
hand, aka þ bkb þ ckc ¼ 2S ¼ (a þ b þ c)r. By adding these equalities and reducing
a þ b þ c, we obtain that ka þ kb þ kc ¼ R þ r. This ends the proof of the lemma.
Projection of broken line AOA1 on straight line AA2 is equal to ha, where
AA2 ⊥ BC, thus R þ ka  ha. Similarly we get that R þ kb  hb, R þ kc  hc.
By summing up these inequalities we get according to lemma that
4R þ r ¼ 3R þ ka þ kb þ kc  ha þ hb þ hc (see problem 5.5.8b).
2.4.12. Let BU  CV, then AU  DV. Draw a median MN of the square (Figure 2.52)
and P1P2||Q1Q2||AB.
Projections of segments PP1 and QQ1 on side AB do not have common internal
points, thus
   
P1 P2 Q1 Q2 KN KM
PQ  AB  ðPP1 þ QQ1 Þ ¼ AB  þ  AB  þ
2 2 2 2
1
¼ AB:
2
82 2 Application of Projection Method

Figure 2.52

Figure 2.53 x
    
a1 S - r = a1 + ... + an

an

a2

2.4.13. First we will prove the following lemma.


Lemma 2n unit vectors are drawn from point O on the plane. They are painted
alternately into red and blue colors. Let ~ S be the sum of n red vectors,~ r be the sum of
 
n blue vectors. Prove that ~ S ~ r   2.
Indeed let ~S ~r 6¼ ~
0. Introduce a coordinate system with Ox axis along vector
~
S ~ r. Since the sum of projections of vectors ~ a2 , :::,~
a1 ,~ an (Figure 2.53) on the Oy
axis is equal to zero, then the length of vector ~ S ~ r is equal to the modulus of the
difference of the sums of the lengths of the positive projections of these vectors on
the Ox and that of the negative projections.
Consequently, the length of the vector ~ S ~r does not exceed either the sums of
the lengths of the positive projections or sums of the lengths of the negative
projections. It is clear that the sums of the lengths of the positive projections as
well as sums of the lengths of the negative projections of vectors~ a2 , :::,~
a1 ,~ an on any
axis does not exceed the diameter of the circle, i.e., does not exceed 2. (See also
problem 7.1.72.)
Consider unit vectors~ e1 , :::,~
e110 , the sum of which is a zero vector. According to
the lemma they can be divided into two groups so that each group would contain
55 vectors. The sum of the vectors of the first group is ~ S, the sum of the vectors of
   
the second group is ~ r, such that ~ S ~r   2. Since ~ S þ~ r ¼~ 0, ~S ~ r   2, then
     
2 ~
S ¼ ~S ~r   2, consequently ~ S  1. This ends the proof.
2.4 Method of Projections 83

Figure 2.54 B

B1
A2
A1
A P C
C2
D1 C1

2.4.14. We shall make use of the following fact: if XX1 is a bisector of angle X of
triangle XYZ and ∠XX1Z  90 , then the midpoint of side YZ belongs to segment
X1Z. Indeed, since ∠XX1Z  90 , then ∠Y  ∠Z.consequently,

YX1 XY
¼  1:
X1 Z XZ

Let straight line A1C1 contain the bisector of angle APB and points A2, C2 be,
respectively, the midpoints of sides AB, CD (see Figure 2.54).
Let also points A02 , C02 be the projections of points A2, C2 on line A1C1.
We have that A1 C1  A02 C02 , thus it follows that A02 C02  A2 C2  ADþBC 2 (see
problem 1.1.9a), consequently A1 C1  ADþBC 2 . Similarly we get that B D
1 1  ABþCD
2 .
Summing up these inequalities we obtain that
ABþBCþCDþDA
2  A1 C1 þ B1 D1 ¼ A1 P þ B1 P þ C1 P þ D1 P  PQ þ PRþ
PS þ PT, which means that ABþBCþCDþDA 2  PQ þ PR þ PS þ PT.
Remark The equality in the last inequality holds true if and only if quadrilateral
ABCD is a rectangular.
2.4.15. Let points B0 , C0 be projections of points B, C on line AD. Then AD ¼
AB cos ∠BAD þ B0 C0 þ CD cos ∠ADC  AB cos ∠BAD þ BC þ CD cos ∠ADC.
Similarly we obtain that BE  AB cos ∠ABE þ AF þ EF cos ∠FEB and
CF  CD cos ∠FCD þ ED þ FE cos ∠CFE.
Summing up these two inequalities we deduce that

AD þ BE þ CF  ABð cos ∠BAD þ cos ∠ABEÞ þ CDð cos ∠ADC þ cos ∠FCDÞþ
∠BAD þ ∠ABE
þ EFð cos ∠FEB þ cos ∠CFEÞ þ BC þ DE þ AF  2AB cos
2
∠ADC þ ∠FCD ∠FEB þ ∠CFE
þ 2CD cos þ 2EF cos þ BC þ DE þ AE 
2 2
 3AB þ BC þ DE þ AE ¼ AB þ BC þ CD þ DE þ EF þ FA:
84 2 Application of Projection Method

Figure 2.55 N C D P

M A F K

Since AB ¼ CD ¼ EF and ∠BADþ∠ABE2 þ ∠ADCþ∠FCD


2 þ ∠FEBþ∠CFE
2 ¼ π (see prob-
lem 5.1.4a). The equality holds if and only if ∠A ¼ ∠B ¼ ∠C ¼ ∠D ¼ ∠E ¼ ∠F.
Remark We give an example of the centrally symmetric hexagon ABCDEF, for
which AD þ BE þ CF > AB þ BC þ CD þ DE þ EF þ FA.
Let MNPK be a unit square and let MA ¼ NB ¼ NC ¼ DP ¼ EK ¼ FK ¼ x
(Figure 2.55).
qffiffiffiffiffiffiffiffiffiffiffiffiffiffiffiffiffiffiffiffiffiffiffiffiffiffiffi
Then AD þ BE þ CF ¼ 3 1 þ ð1  2xÞ2 and AB þ BC þ CD þ EF þ FA þ
pffiffiffi qffiffiffiffiffiffiffiffiffiffiffiffiffiffiffiffiffiffiffiffiffiffiffiffiffiffi
DE ¼ 2 2x þ 2ð1  2xÞ þ 2 x2 þ ð1  xÞ2 . Note that at x ¼ 0 we have that
qffiffiffiffiffiffiffiffiffiffiffiffiffiffiffiffiffiffiffiffiffiffiffiffiffiffiffi pffiffiffi pffiffiffiffiffi qffiffiffiffiffiffiffiffiffiffiffiffiffiffiffiffiffiffiffiffiffiffiffiffiffiffi
3 1 þ ð1  2xÞ2 ¼ 3 2 > 4 ¼ 2 2x þ 2ð1  2xÞ þ 2 x2 þ ð1  xÞ2 . There-
 
fore, there exists such a number x 0 < x < 12 , such that

AD þ BE þ CF > AB þ BC þ CD þ DE þ EF þ FA:

2.4.16. (I. Ziganshin, Russia)


Let KLM be the facet of tetrahedron KLMN which has the largest of the
perimeters of all the facets of the tetrahedron, then

1
KL þ LM þ KM þ KN þ LN þ MN ¼ KL þ LM þ KM þ ðKN þ LN þ KLÞþ
2

þ ðKN þ MN þ KMÞ þ ðLN þ MN þ LMÞ
4ðKL þ LM þ KMÞ
 ¼ 2ðKL þ LM þ KMÞ:
2

Project tetrahedron ABCD on the plane of facet KLM. Denote the projections of
vertices A, B, C, D of the tetrahedron by A1, B1, C1, D1, respectively. Let Γ be the
convex envelope of points A1, B1, C1, D1 and PΓ be the perimeter of polygon Γ.
Since triangle KLM is inside Γ, then according to problem 2.1.1 we have
KL þ LM þ KM  PΓ. Thus, it follows that
2.4 Method of Projections 85

Figure 2.56 B1 B1 C1

D1
A1
A1 C1
D1
a b

KL þ LM þ KM þ KN þ LN þ MN  2ðKL þ LM þ KMÞ  2PΓ :

Now consider cases when Γ is a triangle or quadrilateral.


In the first case (Figure 2.56a) we have that
PΓ ¼ A1 B1 þ B1 C1 þ A1 C1 ¼
2 1 1 1
¼ ðA1 B1 þ B1 C1 þ A1 C1 Þ þ A1 B1 þ B1 C1 þ A1 C1 <
3 3 3 3
2 1
< ðA1 B1 þ B1 C1 þ A1 C1 Þ þ ðD1 A1 þ D1 B1 Þ
3 3
1 1
þ ðD1 B1 þ D1 C1 Þ þ ðD1 C1 þ D1 A1 Þ ¼
3 3
2
¼ ðA1 B1 þ B1 C1 þ A1 C1 þ D1 A1 þ D1 B1 þ D1 C1 Þ:
3

In the second case (Figure 2.56b) we have that

PΓ ¼ A1 B1 þ B1 C1 þ C1 D1 þ D1 A1 ¼
2 1
¼ ðA1 B1 þ B1 C1 þ C1 D1 þ D1 A1 Þ þ ðA1 B1 þ B1 C1 þ C1 D1 þ D1 A1 Þ <
3 3
2 2
< ðA1 B1 þ B1 C1 þ C1 D1 þ D1 A1 Þ þ ðA1 C1 þ B1 D1 Þ
3 3

See problem 1.1.4a.


In both cases PΓ < 23 ðA1 B1 þ B1 C1 þ A1 C1 þ D1 A1 þ D1 B1 þ D1 C1 Þ. On the
other hand, A1B1  AB, B1C1  BC, A1C1  AC, D1A1  DA, D1B1  DB, D1C1 
DC. Therefore, KL þ LM þ KM þ KN þ LN þ MN  2PΓ < 43 ðAB þ BC þ ACþ
DA þ DB þ DCÞ.
Remark Consider a regular pyramid DABC, where AB ¼ BC ¼ AC ¼ 1,
DA ¼ DB ¼ DC ¼ n (n > 2). Choose on edges DA, DC, BA, BC points K, L, M, N,
respectively, such that DK ¼ DL ¼ BM ¼ BN ¼ 1n. Then we have that
KM þ KD þ MB > BD. Consequently KM > n  2n. Similarly we deduce that
KN, ML, NL > n  2n. Then
86 2 Application of Projection Method

 
KL þ KM þ KN þ ML þ NL þ MN 4 n  2n
> : ð2:9Þ
AB þ AC þ AD þ BC þ BD þ CD 3n þ 3

4ðn2Þ
Since at n ! 1 we have that 3nþ3n ! 43, then it is clear that the value 4
3 in
inequality 2.4.16 cannot be lowered.
2.4.17. Let ∠MAiAi þ 1 ¼ βi, ∠MAiAi  1 ¼ γ i, ∠Ai ¼ αi, i ¼ 1, . . . , n, An þ 1 Ai,
A0 An, it is clear that αi ¼ βi þ γ i. Note that

X
n X
n
βi þ γ i β  γi
2p ¼ MAi ð cos βi þ cos γ i Þ ¼ 2MAi cos cos i
i¼1 i¼1
2 2
X
n
αi
 2MAi cos ,
i¼1
2

∠A1 ∠An
consequently, MA1 cos 2 þ ::: þ MAn cos 2  p.
2.4.18. (a) Let M0, N0, and P0 be the midpoints of edges A1B1, C1C, and DA,
pffiffi
respectively. From right-angled triangles AA1P0 and A1M0P0 we have A1 P0 ¼ 25,
pffiffi
M0 P0 ¼ 26, and cos ∠A1 M0 P0 ¼ p1ffiffi6. Similarly, we get that the cosines of angles
∠B1M0N0, ∠C1N0M0, ∠CN0P0, ∠DP0N0, and ∠AP0M0 are equal to p1ffiffi. 6
Let point X0 be the projection of point X on line M0P0. Then, we have that

B01 M0 þ M0 P0 þ P0 D0 ¼ B01 M0 þ M0 P0 þ P0 D0 , M0 P0
1 1
¼ M0 P0 þ pffiffiffi  ðB1 M þ PDÞpffiffiffi :
6 6

Since MP  M0 P0 , we get that

1 1
MP  M0 P0 þ pffiffiffi  ðB1 M þ PDÞpffiffiffi : ð2:10Þ
6 6

In a similar way, we obtain that

1 1
MN  M0 N 0 þ pffiffiffi  ðA1 M þ CN Þpffiffiffi , ð2:11Þ
6 6

and

1 1
PN  P0 N 0 þ pffiffiffi  ðAP þ NC1 Þpffiffiffi : ð2:12Þ
6 6
2.4 Method of Projections 87

Summing up (2.10), (2.11) and (2.12) we get


pffiffiffi
3 6
MP þ MN þ PN  M0 P0 þ M0 N 0 þ P0 N 0 ¼ :
2

(b) Let O be the center of the cube and M be an arbitrary point of the space.
Let’s consider projections of point M and straight lines A1B1, C1C, AD on plane
M0N0P0 (see notations in the solution of problem 2.4.18a).
Denote the projection of point X on plane M0N0P0 by X0 .
Since OM0 ⊥ A1B1, we have OM0 ⊥A01 B01 . Let A01 B01 \ A0 D0 ¼ A2 ,
0 0 0 0 0 0 0 0
A1 B1 \ C1 C ¼ B2 , A D \ C1 C ¼ C2 , then A2B2C2 is a regular triangle with
center O.
We have that SM0 A2 B2 þ SM0 B2 C2 þ SM0 A2 C2  SA2 B2 C2 , consequently

M0 M0 2  A2 B2 M0 N 0 2  B2 C2 M0 P0 2  A2 C2 1 A2 B2 þ B2 C2 þ C2 A2
þ þ  pffiffiffi  ,
2 2 2 2 2

thus maxðM0 M0 2 ; M0 N 0 2 ; M0 P0 2 Þ  p1ffiffi2, where MP2 ⊥ AD, MM2 ⊥ A1B1, MN2 ⊥ C1C,
and P2 2 AD, M2 2 A1B1, N2 2 C1C, which means that

 0 1
maxðMM2 ; MN 2 ; MP2 Þ  max M0 M0 2 ; M0 N 0 2 ; M0 P2  pffiffiffi :
2

(c) Let O1 and O2 be the centers of these spheres and projections of segment O1O2
on the edges emerging from the same vertex are equal to a, b, and c. We have
that a2 þ b2 þ c2 ¼ O1O22. Let a  b, a  c, in that case a  Op1 Offiffi3 2 . Note
(Figure 2.57) that d2 þ a þ D2  1.
Consequently
pffiffiffi
1  Op1 Offiffi3 2 þ d2 þ D2 > dþD
pffiffi þ dþD. Hence, we deduce that d þ D < 3 
2 3 2 3.

Figure 2.57

D
О2
2
а
О1
d
2
88 2 Application of Projection Method

Problems for Self-Study

2.4.19. Prove that the distance from one of the vertices of the convex quadrilateral
to the opposite diagonal does not exceed the half of that diagonal.
2.4.20. Let parallelogram P2 be inscribed in parallelogram P1 and parallelogram P3
be inscribed in parallelogram P3, such that the sides of P3 are parallel to the sides of
P1. Prove that the length of at least one of the sides of P1 does not exceed the double
length of the corresponding parallel side of P3.
2.4.21. Prove that inside a convex n-gon (n  7) one can find a point, the sum of the
distances from which to the vertices are greater than the perimeter.
2.4.22. Given a unit square and such a broken line, that inside it every line parallel
to the side of the square intersects it in no more than one point. Prove that the length
of the broken line is less than 2.
2.4.23. (a) Let us consider on a plane a finite set of segments, the sum of lengths of
pffiffiffi
which is less than 2. Prove that there exists an infinite net of unit squares, the sides
of which do not intersect with any of these segments.
Remark The statement of the problem holds true also if the number of segments is
not finite.
(b) A figure on the coordinate plane has an area S, where S > 1. Prove that it can be
translated by a vector with the integer number of coordinates, so that the figure
and its image do not intersect.
2.4.24. Let ABCDEF be a convex hexagon, such that AB|| ED, BC||EF, CD||AF, and
AB þ DE ¼ BC þ EF ¼ CD þ AF. Prove that

AD þ BE þ CF  AB þ BC þ CD þ DE þ EF þ AF:

2.4.25. Let ABCDEF be a convex hexagon. Prove that

BC  DE BC  AF AF  DE
AD þ BE þ CF  AB þ CD þ EF þ þ þ :
AF DE BC

Hint See the solutions of problems 2.4.15 and 5.1.22a.


2.4.26. Let φ, θ > 0 and φ þ θ ¼ π  α. Prove that ma  b sin θþc
2
sin φ
, where ma is the
length of the median from vertex A of triangle ABC, AB ¼ c, AC ¼ b, α ¼ ∠A.
pffiffiffiffiffiffiffiffiffiffiffiffiffi
2.4.27. Let ABCDA1B1C1D1 be a parallelepiped. Prove that V  S1 S2 S3 , where
V is the volume of the parallelepiped and S1, S2, S3 are the areas of facets ABCD,
AA1B1B, AA1D1D.
2.4.28. Given points A1,B1, and C1 on sides BC, CA, and AB of triangle ABC
respectively. Prove that
2.4 Method of Projections 89

R1 sin α sin β sin γ


 qffiffiffiffiffiffiffiffiffiffiffiffiffiffiffiffiffiffiffiffiffiffiffiffiffiffiffiffiffiffiffiffiffiffiffiffiffiffiffiffiffiffiffiffiffiffiffiffiffiffiffiffiffiffiffiffiffiffiffiffiffiffiffiffiffiffiffiffiffiffiffiffiffiffiffiffiffiffiffiffiffiffiffiffiffiffiffiffiffiffiffiffiffiffiffiffiffiffiffiffiffiffiffiffiffiffiffiffiffiffiffiffiffiffiffiffiffiffiffiffiffiffiffiffiffiffiffiffiffiffiffiffiffiffi ,
R
ð sin α1 sin γ Þ þ ð sin α sin γ 1 Þ2  2 sin α sin α1 sin γ sin γ 1 cos ðβ þ β1 Þ
2

where ∠A ¼ α, ∠B ¼ β, ∠C ¼ γ, ∠B1A1C1 ¼ α1, ∠A1B1C1 ¼ β1, ∠A1C1B1 ¼ γ 1,


and R1, R are the radiuses of the circumcircles of triangles A1B1C1 and ABC,
respectively.
Hint Let OA and OC be the centers of the circumcircles of triangles AC1B1 and
CA1B1, then OAOC  R sin β. It remains to prove that
ffiffiffiffiffiffiffiffiffiffiffiffiffiffiffiffiffiffiffiffiffiffiffiffiffiffiffiffiffiffiffiffiffiffiffiffiffiffiffiffiffiffiffiffiffiffiffiffiffiffiffiffiffiffiffiffiffiffiffiffiffiffiffiffiffiffiffiffiffiffiffiffiffiffiffiffiffiffiffiffiffiffiffiffiffiffiffiffiffiffiffiffiffiffiffiffiffiffiffiffiffiffiffiffiffi
s    ffi
sin α1 2 sin γ 1 2 sin α1 sin γ 1
O A O C ¼ R1 þ 2 cos ðβ þ β1 Þ:
sin α sin γ sin α sin γ

2.4.29. Let p be the sum of the lengths of the edges of convex polyhedron and d be
the maximal distance between its vertices. Prove that p > 3d.
Hint Consider the projection of the polyhedron on straight line AB, where A and
B are the vertices of the polyhedron and AB ¼ d.
2.4.30. The closed broken line passes on a surface of a unit cube and has common
points with all its facets. Prove that the length of the broken line is not less than
pffiffiffi
3 2.
Hint Consider the projections of the broken line on three mutually non-parallel
edges of the cube.
2.4.31. A number of segments is located inside a unit cube, so that any plane
parallel to one of the facets of the cube crosses not more than one of them. Prove
that the sum of the lengths of these segments is not greater than 3.
Hint Consider projections of these segments on three edges of a cube with
common vertex.
2.4.32. A number of polygons is located inside a unit cube, so that any line parallel
to one of the edges of the cube intersects with not more than one of them. Prove that
the sum of the areas of these polygons is not greater than 3.
Hint Consider projections of polygons on three facets of a cube with common
vertex.
2.4.33. Prove that for any tetrahedron one can find two such planes that the ratio of
pffiffiffi
the areas of the projections of the tetrahedron on these planes is not less than 2.
Hint For the tetrahedron SABC let MN be the common perpendicular of straight
lines SA and BC. Consider planes α, β, γ, passing through line MN, where
α ⊥ SA, β ⊥ BC, such that the angles between γ and straight lines SA, BC are equal.
90 2 Application of Projection Method

2.4.34. Given a triangular pyramid ABCD. R is the radius of its circumsphere, r is


the radius of its insphere, a is the length of its longest edge, h is the length of the
least altitude (to one of its facets). Prove that Rr > ah.
2.4.35. Given 75 points inside a unit cube. Prove that the area of one of the triangles,
with vertices belonging to these points, does not exceed 7/72.
2.4.36. The angles at base AD of trapeze ABCD satisfy condition ∠A < ∠D < 90 .
Prove that then AC > BD.
2.4.37. Given a plane n (n  2) unit vectors
the sum of which is a zero vector. Prove
that of these n vectors one can choose n2 vectors, such that the sum of their modules
is not greater than 1.
 
Hint Prove that if one has chosen k (k  n  2) vectors with a sum~ S, where ~
S  1,
then from remaining vectors we can choose two vectors ~ a and ~b, such that at least
     
one of the inequalities ~ a  1, ~
S þ~ b  1, ~
S þ~ a þ~
S þ~ b  1 holds true.
(See also problem 2.4.13.)
2.4.38. Points A1, B1, C1 are taken on sides BC, CA, AB of triangle ABC, respec-
tively. Given that AB1 þ AC1 ¼ BC1 þ BA1 ¼ CA1 þ CB1.
Prove that A1 B1 þ B1 C1 þ C1 A1  12 ðAB þ BC þ ACÞ.
Hint See the solutions of problems 2.4.6 and 5.1.4a.
2.4.39. Let ABCDE be a convex pentagon. Given that AB ¼ BC ¼ CD ¼ DE ¼ EA
and max(∠A, ∠B, ∠C, ∠D, ∠E) < 120 . Prove that min(∠A, ∠B, ∠C, ∠D, ∠E) >
90 .
Hint Let ∠A  90 , and M and N are the midpoints of segments AC and AD,
respectively. Let X0 be the projection of point X on straight line MN. Then
pffiffiffiffiffi 0 0
   a a pffiffi

2a  BE  B E > 2 þ 2 cos α þ 2 cos 30  α > 2 þ 2 1 þ 2 ,


a a a 3
where
AB ¼ a, ∠BMB0 ¼ α. This leads to a contradiction.
2.4.40. Let A1A2    An be a polygon, such that ∠A1 ¼ ∠A2 ¼    ¼ ∠An and
A1A2  A2A3      An  1An  AnA1. Prove that A1A2    An is a regular polygon.
2.4.41. Given points D, E, F on sides BC, CA, AB of triangle ABC, respectively. It is
known that DE ¼ EF ¼ DF, AB ¼ c, BC ¼ a, CA ¼ b. Prove that
pffiffiffiffiffiffiffiffiffiffiffiffiffiffiffiffiffiffiffiffiffiffiffiffiffiffiffiffiffiffiffiffiffiffiffiffiffiffiffiffiffi
pffiffiffi ffi pffiffiffi
DE  a2 þ b2 þ c2 þ 4 3S  2 2S, where S is the area of triangle ABC.
Hint See problem 2.4.28.
2.4.42. Given points A1, B1, and C1 on sides BC, CA, and AB of triangle ABC,
respectively. Let IA be the incenter of triangle AC1B1, and GA the centroid of
triangle AC1B1. Points IB, GB and IC, GC are defined similarly. Prove that
IAIB þ IBIC þ ICIA  GAGB þ GBGC þ GCGA.
2.4 Method of Projections 91

Hint Consider the projections of segments ICIA, IAIB, and IBIC on the straight lines
AC, AB, and BC, respectively. Then, one can easily deduce that
IAIB þ IBIC þ ICIA  A1B1 þ B1C1 þ A1C1. Prove that GC GA  13 ðA1 C1 þ ACÞ and
GAGB þ GBGC þ GCGA  A1B1 þ B1C1 þ A1C1.
2.4.43. Prove the inequality cos α  Rr for a triangle, where α ¼ max (α, β, γ).
Hint Prove that cosβ þ cos γ  1.
Chapter 3
Areas

This chapter is devoted to the inequalities related to areas and it consists of only one
paragraph, that is Section 3.1. One of the methods for proving the inequalities
related to areas (of some figures on the plane) is the following: if the figures with
areas S1 , S2 , . . . , Sk cover a figure with area S, then S1 þ S2 þ . . . þ Sk  S.
Let us give a simple example. Prove that the area of parallelogram AMNK
(M 2 AB, N 2 BC, K 2 AC) inscribed to triangle ABC is not greater than the half of
the area of ABC. Consider parallelograms BNKE and NCKF; note that triangles
AEK and KNF are, respectively, equal to triangles MBN and KNC. Moreover,
triangles AEK and KNF cover parallelogram AMNK; therefore the sum of the
areas of triangles MBN and KNCis not smaller than the area of parallelogram
AMNK. This ends the proof of given example.
One of the main methods of proving geometric inequalities related with areas is
rewriting them as algebraic inequalities by introducing some notations.
In order to compare areas of two figures, often one needs to consider consequent
figures, such that the first figure and the last figure are the given figures. Afterwards,
one needs to compare consequently areas of any two of the considered figures.
Some problems in this chapter were inspired by [2, 4, 9, 11, 13–16].
Nevertheless, even for these problems the authors have mostly provided their
own solutions.

3.1 Inequalities with Areas

3.1.1. (a) In a convex quadrilateral ABCD diagonals intersect at a point O and BC||
AD. Prove that SOCD  14 SABCD .
(b) Given points E and F on parallel sides BC and AD of the convex quadrilateral
ABCD, respectively. Given also that AE, BF intersect at point P and segments
CF, ED at point Q. Prove that SPEQF  14 SABCD .

© Springer International Publishing AG 2017 93


H. Sedrakyan, N. Sedrakyan, Geometric Inequalities, Problem Books
in Mathematics, DOI 10.1007/978-3-319-55080-0_3
94 3 Areas

(c) In a convex quadrilateral ABCD diagonals AC, BD intersect at point O.


pffiffiffiffiffiffiffiffiffiffiffiffi pffiffiffiffiffiffiffiffiffiffi pffiffiffiffiffiffiffiffiffiffi
Prove that SABCD  SAOB þ SCOD .
3.1.2. Prove that the area of the polygon obtained by sequentially connecting the
midpoints of a convex n-gon (n  5) will be greater than the half of the initial
polygon.
3.1.3. Points K and M are taken on sides AB and CD of a convex quadrilateral
ABCD, respectively. Let L be the point of intersection of segments AM and KD,
N be the point of intersection of segments KC and BM. Prove that, if AK :
KB ¼ CM : MD ¼ m : n, then SKLMN < m2 þmnþn
mn
2 SABCD .

3.1.4. Let M and N be the midpoints of sides BC and CD of a convex quadrilateral


ABCD, respectively. Prove that 14 SABCD < SAMN < 12 SABCD .
3.1.5. Bisectors ME , MF , MG , MH of triangles ABM, BMC, CMD, DMA are drawn
from point M inside rectangle ABCD with area S. Prove that for area S0 of a
pffiffiffi 
quadrilateral EFGH, the inequality 2  1 S  S0  0, 5S holds true. For which
points does equality S0 ¼ 0 , 5S hold true?
3.1.6. Let M be a given point inside of an angle with a vertex O. Draw a segment
AB passing through the point M and having the endpoints on the sides of the angle,
such that
(a) the area SOAB is minimal.
1
(b) MA þ MB1
is maximal.
3.1.7. (a) Given a convex quadrilateral ABCD with area S. Perpendiculars MA1,
MB1, MC1, MD1 are drawn from the point of intersection of diagonals M to the sides
AB, BC, CD, DA, respectively. Prove that SA1 B1 C1 D1  S2.
(b) Diagonal BD of the convex quadrilateral ABCD divides it into two isometric
parts. Perpendiculars MA1, MB1, MC1, MD1 are drawn from the point M inside
of the given quadrilateral to straight lines AB, BC, CD, and DA, respectively.
Prove that SMA1 D1 þ SMB1 C1 < 12 SABCD .
3.1.8. Let M, N, and P be the feet of the perpendiculars drawn from the centroid of
the triangle to the straight lines AB, BC and CA, respectively. Prove that
SMNP  14 SABC .
3.1.9. Points D, E and F are on sides BC, CA and AB of triangle ABC, respectively,
such that they do not coincide with points A, B, C. Prove that, if quadrilateral AFDE
EF2
SABC  AD2 .
is inscribed into a circle, then 4S DEF

3.1.10. Given D and E on sides AB and BC of triangle ABC, respectively. Points


K and M divide segment DE into three equal segments (DK ¼ KM ¼ ME). The
straight lines BK and BM intersect side AC at points T and P, respectively. Prove
pffiffiffiffiffiffiffiffiffiffiffiffiffiffiffiffiffiffiffiffiffiffi
that SBTP  SABT  SPBC .
3.1.11. Let P, Q, R be points in a triangle ABC, such that P 2 [AB], Q 2 [BC], R 2
pffiffiffiffiffiffiffiffiffiffi pffiffiffiffiffiffiffiffiffi pffiffiffiffiffiffiffiffiffiffi
[PQ]. Prove that 3 SABC  3 SAPR þ 3 SQRC .
3.1 Inequalities with Areas 95

3.1.12. Let A1, B1, C1 be points on sides BC, CA, AB of a triangle ABC, respectively.
Prove that
(a) SABC  S2A1 B1 C1  4SAB1 C1  SBA1 C1  SCA1 B1 ,
(b) minðSAB1 C1 ; SBA1 C1 ; SCA1 B1 Þ  SA1 B1 C1 ,
(c) minðSAB1 C1 ; SBA1 C1 ; SCA1 B1 Þ  14 SABC ,
pffiffiffiffiffiffiffiffiffiffiffiffi pffiffiffiffiffiffiffiffiffiffiffiffi pffiffiffiffiffiffiffiffiffiffiffiffi 3 pffiffiffiffiffiffiffiffiffiffi
(d) SAB1 C1 þ SBA1 C1 þ SCA1 B1  2 SABC ,
 
1 BB1 CC1
(e) SA1 B1 C1  min 14SABC ; 2AAðABþBCþACÞ , if segments AA1, BB1 and CC1 intersect at
one point,
 
(f) SA1 B1 C1  SAB1 C þ SBA1 C þ SCA1 B  3,
rffiffiffiffiffiffiffiffiffiffiffiffiffiffiffiffiffiffiffiffiffiffiffiffiffiffiffiffiffiffiffiffiffiffiffiffiffiffiffi
1 1 1 1
ffi 1 1
rffiffiffiffiffiffiffiffiffiffiffiffiffiffiffiffiffiffiffiffiffiffiffiffiffiffiffiffiffiffiffiffiffiffiffiffiffiffiffi
SAB1 C1 SBA1 C1 SCA1 B1 SAB1 C1 SBA1 C1 SCA1 B1
(g) max S  S  ,
ð AB1 C1 ;SBA1 C1 ;SCA1 B1 Þ A1 B1 C1 minðSAB1 C1 ;SBA1 C1 ;SCA1 B1 Þ

if segments AA1, BB1 and CC1 intersect at one point.


3.1.13. Two points M and N are chosen on sides AB and BC of triangle ABC,
respectively. Three parallel straight line passing through points M, B and N intersect
side AC at points K, D and L, respectively. Prove that the area of the trapezoid
(or parallelogram) KMNL is not greater than the area of one of the triangles ABD
and DBC.
3.1.14. Two tangents PB and PC are drawn from point P to the circle, such that

∠BPC  90 . Point A is chosen on the smaller arc BC. Prove that the area of the
triangle, cut from the angle BPC by the tangent to the circle drawn from point A,
does not exceed the area of triangle ABC.
3.1.15. Let bisectors of the internal angles of triangle ABC intersect the circumcircle
of triangle ABC. Given that they intersect the circle for the second time at points A1,
 
B1, C1. Prove that SASABC
B C
 min cos 2 ∠A∠B
2 ; cos 2 ∠B∠C
2 ; cos 2 ∠C∠A
2 .
1 1 1

3.1.16. Prove that the area


(a) of the parallelogram placed inside the triangle does not exceed half the area of
the triangle.
(b) of the triangle placed inside the parallelogram does not exceed half the area of
the parallelogram.
3.1.17. Prove that the convex polygon with the area S can be placed inside
(a) a rectangle with the area not greater than 2S.
(b) a triangle with the area not greater than 2S.
3.1.18. Prove that one can inscribe into a convex polygon with the area S
(a) a parallelogram with the area not less than S2,
(b) a triangle with the area not less than 3S
8.
(c) a hexagon with the area not less than 3S4.
96 3 Areas

3.1.19. Given a convex polygon in which one cannot place any triangle with area
1. Prove that this polygon can be placed into a triangle with area 4.
3.1.20. Given an acute triangle ABC. Let A1, B1, C1 be the points symmetric to
points A, B, C with respect to straight lines BC, AC, AB, respectively. Prove that
SA1 B1 C1  4SABC .
3.1.21. Given a triangle ABC. Segments BB1, CC1, AA1 are placed on rays AB, BC,
CA starting at points B, C, A, respectively, such that BB1 ¼ AC, CC1 ¼ AB,
AA1 ¼ BC. Prove that SAA1 B þ SBB1 C þ SCC1 A  3SABC .
3.1.22. Three secants are drawn for each internal point X of triangle ABC parallel to
its sides. As a result of this, one obtains three triangles each bounded by two secants
and a side of the triangle. Let the areas of these triangles be S1, S2, S3. Prove that
S1 þ S2 þ S3  13 S.
3.1.23. In a convex hexagon ABCDEF the opposite sides are parallel to each other
(AB||DE, BC||EF, CD||AF). Prove that 2SBDF  SABCDEF.
3.1.24. Given points P, Q, R on sides AB, CD, EF of the centrally symmetric convex
hexagon ABCDEF, respectively. Prove that 2SPQR  SABCDEF.
3.1.25. The triangle is inscribed into a regular hexagon; one of the sides of the
triangle passes through the center of symmetry of the hexagon. Prove that the area
of the triangle does not exceed 13 of the area of the hexagon.
3.1.26. Points D and E are chosen on the sides AB and AC of the triangle ABC,
respectively. Let segments BE and CD intersect at point P. Prove that
pffiffi
(a) SPDE  5 5211 SABC ,
 pffiffiffi 
(b) SPDE  5 2  7 SABC , if SBCED ¼ 2SPBC.
3.1.27. The segment PQ passes through the centroid of triangle ABC and points P,
Q are on sides BC, AC, respectively. Prove that SMPQ  29 SABC , where M is the
midpoint of side AB.
3.1.28. Prove that the area of any section of the cube by a plane passing through its
center is not less than the area of the faces of the cube.
3.1.29. The part of a plane between two parallel straight lines is called a “strip”. Let
several strips be given on a plane, and no two of them are parallel, i.e. no two
boundary straight lines of different strips are parallel. How one should move the
strips parallel to themselves to make the area of the polygon formed by the
intersection of the strips the greatest possible?
 
3.1.30. (a) Given a circle ω with a center A 12; 12 and radius R, where R < 12. Points
M and N are on the positive semiaxes Oy and Ox so that segment MN is tangent to
circle ω. Prove that
3.1 Inequalities with Areas 97

pffiffi 2 pffiffi
(1) SMON  2
2
R , if 0 < R  2
4 ,
pffiffi
(2) SMON  14  R2 , if 42 < R < 12.
pffiffiffi
(b) Prove that any section of the unit cube by a plane has an area not greater than 2.
3.1.31. (a) Given points B2, . . . , Bn and C2, . . . , Cn on the sides of triangle B1AC1
(B2, . . . , Bn 2 AB1, C2, . . . , Cn 2 AC1). The ray with a vertex at A intersects seg-
ments B1C1, B2C2, . . . , BnCn at points D1, D2, . . . , Dn, respectively. Prove that

AD1 þ AD2 þ ::: þ ADn  maxðAB1 þ AB2 þ ::: þ ABn ; AC1 þ AC2 þ ::: þ ACn Þ:

(b) Prove that in a convex quadrilateral the length of any segment with ends on the
sides of the quadrilateral, which passes through the point of intersection of
diagonals do not exceed the length of one of the diagonals.
(c) Prove that the area of any cross section of the tetrahedron by a plane does not
exceed the area of one of its faces.
3.1.32. Given a convex quadrilateral ABCD. Prove that

(a) CD2  SABC þ BC2  SACD > AC2  SBCD, if ∠A þ ∠ C > 180 ,

(b) CD2  SABC þ BC2  SACD ¼ AC2  SBCD, if ∠A þ ∠ C ¼ 180 ,

(c) CD2  SABC þ BC2  SACD < AC2  SBCD, if ∠A þ ∠ C < 180 .
3.1.33. In a convex hexagon ABCDEF opposite sides are parallel (AB||DE, BC||EF,
CD||AF). Denote by A1, B1, C1, D1, E1, and F1 the midpoints of sides AB, BC, CD,
DE, EF and FA, respectively. Prove that
(a) segments A1D1, B1E1 and C1F1 can be sides of a triangle.
(b) 12 SBDF < S1  SBDF , where S1 is the area of the triangle with sides having
lengths A1D1, B1E1 and C1F1.
3.1.34. Given that the rectangle with sides a and b is inside the rectangle with sides
c and d, such that max(a, b) > max (c, d ). Prove that 2ab < cd.
3.1.35. O is an internal point of the convex quadrilateral ABCD with area S. Let
points K, L, M and N be on segments AB, BC, CD and DA, respectively. Prove that if
OKBL and OMDN are parallelograms, then
pffiffiffi pffiffiffiffiffi pffiffiffiffiffi
(a) S  S1 þ S2 :
pffiffiffi pffiffiffiffiffi pffiffiffiffiffi
(b) 1, 25 S > T 1 þ T 2 :
pffiffiffi pffiffiffiffiffi pffiffiffiffiffi
(c) C0 S  T 1 þ T 2 ,
where S1, S2, T1, T2 are the areas of the quadrilaterals ONAK, OLCM, OKBL,
OMDN, respectively and
 
sin 2α þ π4
C0 ¼ max :
½0;π4 cos α
98 3 Areas

3.1.36. Let A1, B1, C1, D1 be the midpoints of sides BC, CD, DA, AB of a convex
quadrilateral ABCD, respectively, S0 be the area of the quadrilateral, formed by the
segments AA1, BB1, CC1, DD1, and S be the area of the quadrilateral ABCD. Prove
that 16 S < S0  15 S.
3.1.37. Consider a triangle with the area S. Prove that if one places externally
equilateral triangles on each side of the given triangle, then centers of these tri-
angles are vertices of an equilateral triangle with the area not less than S.
3.1.38. Given on a plane mutually nonintersecting triangles all obtained from
triangle ABC using a translation by a certain vector. All these triangles are contin-
ued to become parallelograms, such that any of these parallelograms has a diagonal
parallel to AB. Prove that S0  1, 5S, where S0 is the area of the union of these
parallelograms and S is the sum of the areas of all triangles.
3.1.39. Given that in a convex hexagon ABCDEF triangles ACE and BDF are
pffiffiffiffiffiffiffiffiffiffiffiffiffiffiffiffiffiffiffiffiffiffi
similar. Prove that SABCDEF  Rr SACE  SBDF , where r and R are, respectively,
inradius and circumradius of triangle ACE.
3.1.40. Given on a plane five points, so that the area of each of 10 triangles defined
by these points is greater than 2. Prove that among those triangles exists a triangle,
pffiffiffi
such that its area is greater than 1 þ 5.
3.1.41. The vertices of a convex hexagon A1A2A3A4A5A6 are on the sides of the unit
square. Prove that the area of one of triangles A1A2A3, A2A3A4, A3A4A5, A4A5A6,
A5A6A1, A6A1A2 is not greater than 18.
3.1.42. Given on a plane a triangle ABC and a point P, such that ∠A ¼ 600, PA ¼ 1,
 pffiffiffiffiffi pffiffi
PB ¼ 2, PC ¼ 3. Prove that SABC  13 þ 73  83.
3.1.43. Given on a plane a triangle ABC and a point P, such that ∠A ¼ α  π2,
PA ¼ r1, PB ¼ r2, PC ¼ r3, with 0 < r1 < r2 < r3, where r1, r2, r3, α are constants.
Find the possible highest and lowest values of the area of triangle ABC.
3.1.44. Consider two parallelograms that intersect exactly in eight points. Prove that
the common area of these parallelograms is greater than or equal to half of the area
of one of them.

Solutions

3.1.1. (a) Consider the midpoints of segments AC, CD and BD and let those points
be M, N and P, respectively. We have that MN||AD and NP||BC and also
SOCD  SMCDP ¼ SMCN þ SPND ¼ 14 SACD þ 14 SBCD ¼ 14 SACD þ 14 SABC ¼ 14 SABCD .
Consequently, SOCD  14 SABCD .
(b) Making use of problem 3.1.1a we get that
3.1 Inequalities with Areas 99

Figure 3.1 B C
N
M

K
L

1 1 1
SPEQF ¼ SPEF þ SEQF  SABEF þ SFECD ¼ SABCD :
4 4 4

(c) We have that SSDOA


AOB
¼ OD
BO
¼ SSCOD
BOC
, consequently,
pffiffiffiffiffiffiffiffiffiffiffiffiffiffiffiffiffiffiffiffiffiffiffi
SABCD ¼ SAOB þ SBOC þ SCOD þ SDOA  SAOB þ SCOD þ 2 SBOC  SDOA ¼
pffiffiffiffiffiffiffiffiffiffi pffiffiffiffiffiffiffiffiffiffi 2
¼ ð SAOB þ SCOD Þ ,
pffiffiffiffiffiffiffiffiffiffiffiffi pffiffiffiffiffiffiffiffiffiffi pffiffiffiffiffiffiffiffiffiffi
we obtain that SABCD  SAOB þ SCOD .
3.1.2. Let points B1, B2, . . . , Bn be the midpoints of sides A1A2, A2A3, . . . , AnA1 of
the initial polygon A1A2 . . . An, respectively. Denote by C1, C2, . . . , Cn the inter-
section points of the diagonalеs AnA2 and A1A3, A1A3 and A2A4,. . ., An  1A1 and
AnA2, respectively. We have that

1
SB1 B2 :::Bn ¼ SA1 A2 :::An  SB1 A2 B2  :::  SBn A1 B1 ¼ SA1 A2 :::An  ðSA1 A2 A3 þ ::: þ SAn A1 A2 Þ ¼
4
1 1
¼ SA1 A2 :::An  ð2SA1 A2 :::An  SC1 A2 C2  :::  SCn A1 C1  2SC1 C2 :::Cn Þ > SA1 A2 :::An :
4 2

3.1.3. We have that SSMKD


MKC
¼ mn (Figure 3.1).
Let SMKC ¼ mS1, SMKD ¼ nS1. Similarly we have SAKM ¼ mS2, SBKM ¼ nS2.
Denote by ρ(X, l ) the distance of the point X from the straight line l. Calculate
 
1 1 m n  
SCKD ¼ CD  ρðK; CDÞ ¼ CD  ρðB; CDÞ þ ρ A; CD ¼
2 2 mþn mþn
1 1
¼ CM  ρðB; CDÞ þ MD  ρðA; CDÞ ¼ SBCM þ SAMD :
2 2

Therefore, SKLMN ¼ SBNC þ SALD.


Let SKNM ¼ x, SKLM ¼ y, then SNCM ¼ mS1  x, SMLD ¼ nS1  y, SAKL ¼ mS2  y,
SBKN ¼ nS2  x.
It is clear that SSNCM
BNC
¼ NM
BN
¼ SSKNM
BKN
, consequently, SBNC ¼ ðmS1 xÞxðnS2 xÞ.
100 3 Areas

Similarly, we deduce that SALD ¼ ðnS1 yÞyðmS2 yÞ. Since SKLMN ¼ SBNC þ SALD,
then x þ y ¼ ðmS1 xÞxðnS2 xÞ þ ðnS1 yÞyðmS2 yÞ or 1 ¼ ðS1 þS mS1 nS2
2 ÞðmþnÞx
þ ðS1 þS
mS1 nS2
2 ÞðmþnÞy
. Now
we have to find the maximal value of x þ y at x < min (mS1, nS2).
Let x  y. Denote ðS1 þSmS1 nS2
2 ÞðmþnÞx
¼ sin 2 α, ðS1 þS
mS1 nS2
2 ÞðmþnÞy
¼ cos 2 α, where α0 < α  π4,
sin 2 α0 ¼ ðS1 þS2 Þðmþn
mS1 nS2
ÞminðmS1 ;nS2 Þ.
Then x þ y ¼ ðS1 þS24mS
ÞðmþnÞsin 2 2α < ðS1 þS2 ÞðmþnÞsin 2 2α0 .
1 nS2 4mS1 nS2

If mS1  nS2, then sin 2 α0 ¼ ðS1 þSnS 2


2 ÞðmþnÞ
and

4mS1 nS2 mS1 mS1


¼ ¼ ¼
ðS1 þ S2 Þðm þ nÞsin 2 2α0 1  sin 2 α0 nS2
1
ðS1 þ S2 Þðm þ nÞ
mðm þ nÞS1 ðS1 þ S2 Þ mðm þ nÞS1 ðS1 þ S2 Þ mnðm þ nÞðS1 þ S2 Þ
¼  ¼ ¼
mS1 þ mS2 þ nS1 m2 m2 þ mn þ n2
mS1 þ S1 þ nS1
n
mn
¼ 2 SABCD :
m þ mn þ n2

Thus, we obtain that


mn
SKLMN < SABCD , ð3:1Þ
m2 þ mn þ n2

In the case of nS2 < mS1 the inequality (3.1) is obtained similarly.
3.1.4. We have that SABMND < SABCD,

1 1 1
SABMND ¼ SAMN þ SABM þ SAND ¼ SAMN þ SABC þ SACD ¼ SAMN þ SABCD ,
2 2 2

consequently, SAMN < 12 SABCD . Note that SABCD ¼ SABD þ SBCD ¼ SABD þ 4SCMN.
Therefore, 4SAMN ¼ SABCD þ SABD, so SAMN > 14 SABCD .
3.1.5. (Solution of G. Khotsanyan, 9th grade)
First we shall prove two lemmas which are used at the proof of the inequality.
Lemma 1 A straight line perpendicular to AB is drawn from point H of segment
AB, where AH < BH. Points C and D are taken on that perpendicular so that
CH < DH. Points E and F are the feet of the bisectors drawn from vertices C and D
in triangles ABC and ABD. Prove that AE < AF.
Proof Indeed, according to the interior angle bisector theorem, for triangles ABC
pffiffiffiffiffiffiffiffiffiffiffiffiffiffiffi pffiffiffiffiffiffiffiffiffiffiffiffiffiffiffi
CH 2 þAH 2 AF DH2 þAH 2
and ABD we get: AE
EB ¼ AC
BC ¼ p ffiffiffiffiffiffiffiffiffiffiffiffiffiffiffi , ¼ AD
¼ pffiffiffiffiffiffiffiffiffiffiffiffiffiffiffi .
CH 2 þBH 2 FB BD DH2 þBH 2
pffiffiffiffiffiffiffiffiffiffiffiffi qffiffiffiffiffiffiffiffiffiffiffiffiffiffiffiffiffiffiffiffiffiffiffi
x2 þAH 2
Consider now a function f ðxÞ ¼ pffiffiffiffiffiffiffiffiffiffiffiffi BH 2
2
¼ 1 þ AH x2 þBH 2
. Note that it grows in
2x þBH2

the interval x 2 [0 ; 1 ) (since AH2  BH2 < 0). Consequently, from the condition
3.1 Inequalities with Areas 101

CH < DH, it follows that f(CH) < f(DH). Hence, we deduce that AE
EB < AF
FB or
EB < FB . From this inequality we obtain that AF > AE.
EBþAE FBþAF

Lemma 2 Given that points E and F are the feet of the bisectors drawn from
vertices C and D in triangles ABC and ABD, respectively, with CD||AB. Prove that,
if segments CE and DF do not intersect, then CD > EF.
Proof First introduce following notations (see also Figure 3.2) ∠ACB ¼ 2α,
∠ADB ¼ 2β, ∠BCD ¼ u, ∠ADC ¼ v. It is known that ∠ABC ¼ ∠BCD ¼ u,
∠BAD ¼ ∠ADC ¼ v.

From triangles ABC and ABD we deduce that 2α þ u þ v þ ∠CAD ¼ 180 ,
 
2β þ u þ v þ ∠CBD ¼ 180 . Hence, we have that 2α þ u þ v < 180 ,

2β þ u þ v < 180 . Summing up these two inequalities, it follows that
 
2α þ 2β þ 2u þ 2v < 360 , or ∠ECD þ ∠FDC < 180 . If segments CE and DF do
not intersect, then CD > EF. This ends the proof of the lemma.
Now, we continue the proof of the problem. Let us introduce following notations
(see Figure 3.3), AE ¼ a, AH ¼ b, DG ¼ c, BF ¼ d, AB ¼ CD ¼ e, BC ¼ AD ¼ f.
We have that

S0 ¼ S  ðSAEH þ SBEF þ SCFG þ SDGH Þ ¼


1 ef 1
¼ ef  ðab þ ðe  aÞd þ ðf  dÞðe  cÞ þ ðf  bÞcÞ ¼  ðc  aÞðd  bÞ:
2 2 2

Figure 3.2 C D
u v
a b b
a

v u
A E F B

Figure 3.3 B d F f-d C

e-a e-c

E M G

a c

A b H f-b D
102 3 Areas

Figure 3.4 B C1 C

B1 M D1

A A1 D

Thus,

S 1
S0 ¼  ðc  aÞðd  bÞ: ð3:2Þ
2 2

If point M belongs to one of the medians of rectangle ABCD, then c ¼ a or d ¼ b,


i.e. S0 ¼ S2.
Now, assume that point M does not belong to any median and that point A is the
vertex of the rectangle closest to M.
Then, it is clear that M belongs to one of the four rectangles (obtained by
dividing the given rectangle by medians) which contains vertex A (Figure 3.4).
Let us now perform a translation of triangle MCD by a vector CB. ~ Then side CD
coincides with side BA and according to the first lemma c > a.
Similarly, we obtain that d > b (Figure 3.4).
From aforesaid and (3.2) we get that S0 < S2.
From formula (3.2) it follows that S0 reaches its least value when the expression
(c  a)(d  b) has a maximal value. We shall prove that both (c  a) and (d  b)
reach their maximal values at point A.
Compare the values of the (c  a) at points M, B1 and A.
According to the first lemma the value of c at point B1 is greater than the value at
point M, while the value of a is less.
Consequently the value of the expression (c  a) at B1 is greater than at M.
Denote the values of the c and a at points B1 and A by c1, a1 and c2, a2, respectively
(Figure 3.5). Applying the second lemma to the case when bisectors drawn from the
vertices B1 and A of triangles CB1D and CAD do not intersect, we obtain that
a1 ¼ a1  a2 > c1  c2 or c2  a2 > c1  a1. If the bisectors intersect, then c2 > c1.
Hence, it follows that c2  a2 ¼ c2 > c1  a1. We have proven that the value of
(c  a) at point A is greater than the value at point B1, while the value at point B1 is
greater than the value at point M (inside the rectangle). Similarly one can prove that
the expression (d  b) reaches its maximal value at point A.
Thus, S0 reaches its minimal value when the point M coincides with the vertex A,
i.e. a ¼ 0, b ¼ 0. Therefore c ¼ pefffiffiffiffiffiffiffiffi
ffi, d ¼ pefffiffiffiffiffiffiffiffi2ffi.
fþ e þf
2 2
eþ e þf
2

From formula (3.2) we deduce that


3.1 Inequalities with Areas 103

Figure 3.5 B C

c2 c1
a1

A D

Figure 3.6 A
E

A1 M

O
F B

!
S ef
S0  1 pffiffiffiffiffiffiffiffiffiffiffiffiffiffi : ð3:3Þ
2 ef þ ðe þ f Þ e2 þ f 2 þ e2 þ f 2
pffiffiffiffiffi
Since e þ f  2 ef , e2 þ f2  2ef, then
qffiffiffiffiffiffiffiffiffiffiffiffiffiffi  pffiffiffi
ef þ ðe þ f Þ e2 þ f 2 þ e2 þ f 2  ef 3 þ 2 2 : ð3:4Þ

From inequalities (3.3) and (3.4) we get that


!
S ef pffiffiffi 
S0  1  pffiffiffi ¼ 2  1 S:
2 ef 3 þ 2 2

3.1.6. Let us draw ME ⊥ OA, MF ⊥ OB (Figure 3.6).


We have that

1 MA  MB 1 2 sin ∠A  sin ∠B
SMEF ¼ ME  MF  sin ∠EMF ¼  AB  sin 2 ∠O ¼
2 AB2 2 sin ∠O
MA  MB
¼  SABO  sin 2 ∠O:
AB2
ð3:5Þ
104 3 Areas

(a) According to (3.5) SABO would be minimal if the value of the expression MAMB
AB2
2
¼ 14  14  ðMAMB
AB2
Þ
would be maximal i.e. at MA ¼ MB.
To construct segment AB one has to note that OA1 ¼ A1A, where MA1||OF.
Remark 1. SMEF  14 SABO .
Indeed, according to (3.5), SMEF  MAMB
AB2
 SABO  14 SABO .

2. SOA1 M  14 SOAB .
(b) According to (3.5) we have that

1 1 AB SABO sin 2 ∠O 1 sin 2 ∠O


þ ¼ ¼  ¼ ON  ,
MA MB MA  MB AB SMEF 2 SMEF

where ON ⊥ AB.
If the straight line passing through point M perpendicular to line OM intersects
the sides of the given angle at points A1 and B1, then segment A1B1 is what was
required. Indeed, we have that

1 1 sin 2 ∠O sin 2 ∠O 1 1
þ ¼ ON   OM  ¼ þ :
MA MB SMEF SMEF MA1 MB1

But if the straight line passing through point M perpendicular to line OM does
not intersect the sides of the given angle, then the required segment cannot be
constructed. Indeed, since N 2 OK (Figure 3.7), where KM||OE, then ON does not
have a maximal value.

N K

O F

Figure 3.7
3.1 Inequalities with Areas 105

Figure 3.8
D2

D3
D D'
D1 C1 C

M B1

B'

A A1 B B3 B2

3.1.7. (a) We have that

1
SA1 B1 C1 D1 ¼ SMA1 B1 þ SMB1 C1 þ SMD1 C1 þ SMA1 D1  SABC
4
1 1 1
þ SBCD þ SACD þ SABD ¼
4 4 4
1 S
¼ SABCD ¼
2 2

(see the remark 1 of problem 3.1.6a). This means that SA1 B1 C1 D1  S2.
(b) Draw from points C and M straight lines parallel to diagonal BD (Figure 3.8).

According to the remark 1 of problem 3.1.6a

1 1 1
SMA1 D1 þ SMB1 C1  SAB3 D3 þ SCB0 D0 < SAB3 D3
4 4 4
1 1 1
þ SB2 B3 D3 D2 ¼ SAB2 D2 ¼ SABCD :
4 4 2

Since from the statement of the problem, it follows that points C and A are
equidistant from line BD, i.e. BD is the midline of triangle AB2D2.
Remark 1. If ABCD is a square and point M coincides with C, then

1
SMA1 D1 þ SMB1 C1 ¼ SCBD ¼ SABCD :
2
106 3 Areas

Figure 3.9 B

B1
M
G

A P C1 C

Figure 3.10 C
E

E' D

A F B

2. In general the inequality does not hold true: take in a square ABCD a point
M close to point C and move vertices B and D along the sides in the direction
of A.  
S2ABCD S2ABCD
3. One can prove that, in general case, SMA1 D1 þ SMB1 C1 < 14 max SBDC ; SABD .

3.1.8. Draw through point G a straight line B1C1||BC (Figure 3.9).


We have that SMGP  14 SAB1 C1  sin 2 ∠A (see the solution of problem 3.1.6a), and
SAB1 C1 AB 2 4
SABC ¼ AB ¼ 9. Therefore, SMGP  SABC 9  sin ∠A. Similarly, we get that
1 2

SMGN  SABC
9  sin ∠B, SNGP  9  sin ∠C. Thus
2 SABC 2

SABC  2  SABC
SMNP ¼ SMGP þ SMGN þ SNGP   sin ∠A þ sin 2 ∠B þ sin 2 ∠C  :
9 4

(see problem 9.1).


0 0
3.1.9. Let DE k AB (Figure 3.10), then ∠ADE ¼ ∠DAF ¼ ∠DEF and
0
EF2
∠DAE ¼ ∠DFE. Hence, it follows that ΔAE D  ΔFDE. Thus SSDEF0 ¼ AD 2 and we
AE D
3.1 Inequalities with Areas 107

Figure 3.11 B

E
K M
D E1

M1
K1
A T P C

have to prove that SAE0 D  14 SABC . Note that, this follows from the solution of
problem 3.1.6a (see remark 2).
3.1.10. Draw from the point A a straight line parallel to DE (Figure 3.11).
Since DK ¼ KM ¼ ME, then AK1 ¼ K1M1 ¼ M1E1. Now by using Menelaus’
theorem for triangle AE1C and straight lines BT, BP, we obtain that

AK 1 BE1 CT AM1 BE1 CP


  ¼1¼   :
K 1 E1 BC TA M1 E1 BC PA

Consequently,

1 SBPT þ SBPC SBPC


 ¼2 :
2 SABT SABT þ SBPT

Hence, it follows that (SBPT þ SBPC)(SABT þ SBPT) ¼ 4SABT  SBPC.


Using further the Cauchy inequality, we deduce that 4SABT  SBPC 
pffiffiffiffiffiffiffiffiffiffiffiffiffiffiffiffiffiffiffiffiffiffi pffiffiffiffiffiffiffiffiffiffiffiffiffiffiffiffiffiffiffiffiffiffi pffiffiffiffiffiffiffiffiffiffiffiffiffiffiffiffiffiffiffiffiffiffi
2 SBPT  SBPC  2 SABT  SBPT . Thus SBPT  SABT  SBPC .
Remark From the last inequality it follows that SBPT  SABT þS
2
BPC
. Hence, we obtain
that TP  3 . This problem was suggested on the 16th Russian Mathematical
AC

Olympiad.

PA ¼ λ, QC ¼ μ, then using the Cauchy inequality, we deduce that


BQ
3.1.11. Let PB

rffiffiffiffiffiffiffiffiffiffi sffiffiffiffiffiffiffiffiffiffiffi sffiffiffiffiffiffiffiffiffiffiffiffiffiffiffiffiffiffiffiffiffiffiffiffi sffiffiffiffiffiffiffiffiffiffiffiffiffiffiffiffiffiffiffiffiffiffiffiffi


3 SAPR 3 SQRC 3 SAPR SPBQ 3 SQRC SPBQ
þ ¼  þ  ¼
SABC SABC SPBQ SABC SPBQ SABC
sffiffiffiffiffiffiffiffiffiffiffiffiffiffiffiffiffiffiffiffiffiffiffiffiffiffiffiffiffiffiffiffiffiffiffiffiffiffiffiffi sffiffiffiffiffiffiffiffiffiffiffiffiffiffiffiffiffiffiffiffiffiffiffiffiffiffiffiffiffiffiffiffiffiffiffiffiffiffiffiffiffi
λS 1 μ 3 μSQRC λ 1
¼ 3 APR   þ   
SPBQ 1 þ λ 1 þ μ SPBQ 1 þ λ 1 þ μ
!
1 λSAPR þ μSQRC 1 μ λ 1
 þ þ þ þ ¼ 1,
3 SPBQ 1þλ 1þμ 1þλ 1þμ

since λSAPR þ μSQRC ¼ SPBR þ SQBR ¼ SPBQ.


pffiffiffiffiffiffiffiffiffi pffiffiffiffiffiffiffiffiffiffi pffiffiffiffiffiffiffiffiffiffi
Consequently 3 SAPR þ 3 SQRC  3 SABC .
108 3 Areas

AB ¼ α, BC ¼ β, CA ¼ γ, then SAB1 C1 ¼ SABC  αð1  γ Þ, SBA1 C1 ¼


3.1.12. Let AC 1 BA1 CB1

SABC  ð1  αÞβ, SCA1 B1 ¼ SABC  γ ð1  βÞ and SA1 B1 C1 ¼ SABC ð1  αð1  γ Þ


βð1  αÞ  γ ð1  βÞÞ.
(a) We have to prove that (1  α(1  γ)  β(1  α)  γ(1  β))2  4αβγ(1  α)
(1  β)(1  γ), or

ð1  α  β  γ þ αγ þ βγ þ αβÞ2  4αβγ ð1  α  β  γ þ αγ þ βγ þ αβ  αβγ Þ,


ð1  α  β  γ þ αγ þ βγ þ αβ  2αβγ Þ2  0:

(b) Assume that SA1 B1 C1 < minðSAB1 C1 ; SBA1 C1 ; SCA1 B1 Þ and let SAB1 C1  SBA1 C1 
SCA1 B1 . Then, taking into account the solution of the problem 3.1.12a, we get
that

SABC  S2AB1 C1 > SABC  S2A1 B1 C1  4SAB1 C1  SBA1 C1  SCA1 B1 


 S2AB1 C1  4SCA1 B1 > S2AB1 C1  ðSAB1 C1 þ SBA1 C1 þ SCA1 B1 þ SA1 B1 C1 Þ ¼ SABC  S2AB1 C1 :

Consequently SABC  S2AB1 C1 > SABC  S2AB1 C1 . This leads to a contradiction.


The Second Solution Consider the midpoints of sides BC, CA and AB. Let these
points be A0, B0 and C0, respectively. If among points A1, B1, C1 there exist two
points belonging to two of segments AC0, C0B, BA0, A0C, CB0, B0A with a common
endpoint, then the statement is evident. Indeed, let C1 2 [C0B] and A1 2 [A0B], then
M 2 BN, where M ¼ BB1 \ A1C1, N ¼ A0C0 \ BB1, consequently BM  BN ¼ NB1 
MB1. Therefore SA1 C1 B  SA1 B1 C1 .
Otherwise, one can assume that C1 2 [C0B], A1 2 [A0C], B1 2 [AB0], then

SA1 B1 C1  minðSA1 B1 C0 ; SA1 B1 B Þ  minðSA0 C0 B1 ; SC0 CB1 ; SA0 B1 B Þ 


1
 minðSA0 C0 B0 ; SCC0 B0 ; SA0 B0 B Þ ¼ SABC :
4

4 SABC  minðSAB1 C1 ; SBA1 C1 ; SCA1 B1 Þ. Thus, SA1 B1 C1  min


1
Consequently,
ðSAB1 C1 ; SBA1 C1 ; SCA1 B1 Þ.
Yet another solution can be obtained by using the solution of the problem
3.1.12e.
(c) According to 3.1.12b we have

minðSAB1 C1 ; SBA1 C1 ; SCA1 B1 Þ ¼ minðSAB1 C1 ; SBA1 C1 ; SCA1 B1 ; SA1 B1 C1 Þ 


1 SABC
 ðSAB1 C1 þ SBA1 C1 þ SCA1 B1 þ SA1 B1 C1 Þ ¼ :
4 4
3.1 Inequalities with Areas 109

(d) We have that


pffiffiffiffiffiffiffiffiffiffiffiffi pffiffiffiffiffiffiffiffiffiffiffiffi pffiffiffiffiffiffiffiffiffiffiffiffi pffiffiffiffiffiffiffiffiffiffipffiffiffiffiffiffiffiffiffiffiffiffiffiffiffiffiffi pffiffiffiffiffiffiffiffiffiffiffiffiffiffiffiffiffiffi pffiffiffiffiffiffiffiffiffiffiffiffiffiffiffiffiffi
SAB1 C1 þ SBA1 C1 þ SCA1 B1 ¼ SABC αð1 γ Þ þ βð1  αÞ þ γ ð1  βÞ 
pffiffiffiffiffiffiffiffiffiffi α þ 1  γ β þ 1  α γ þ 1  β
 SABC þ þ
2 2 2
3pffiffiffiffiffiffiffiffiffiffi
¼ SABC :
2

(e) According to Ceva’s theorem αβγ ¼ (1  α)(1  β)(1  γ). Hence

SA1 B1 C1 ¼ SABC ð1  αð1  γ Þ  βð1  αÞ  γ ð1  βÞÞ


¼ SABC ðð1  αÞð1  βÞð1  γ Þ þ αβγ Þ ¼
pffiffiffiffiffiffiffiffiffiffiffiffiffiffiffiffiffiffiffiffiffiffiffiffiffiffiffiffiffiffiffiffiffiffiffiffiffiffiffiffiffiffiffiffiffiffiffiffiffiffiffi SABC
¼ 2SABC αβγ ð1  αÞð1  βÞð1  γ Þ  ,
4

since xð1  xÞ  14.


According to Stuart’s theorem we have
 
AA21 ¼ βb2 þ ð1  βÞc2  βð1  βÞa2 ¼ βð1  βÞ b2 þ c2  a2
 
þ β2 b2 þ ð1  βÞ2 c2  βð1  βÞ b2 þ c2  a2
þ 2βð1  βÞbc ¼ 4ðp  aÞpβð1  βÞ,

therefore
pffiffiffiffiffiffiffiffiffiffiffiffiffiffiffiffiffiffiffiffiffiffiffiffiffiffiffiffiffiffiffiffiffiffiffiffiffiffiffiffiffiffiffiffiffiffiffiffiffiffiffi
AA1  BB1  CC1  4SABC  ða þ b þ cÞ αβγ ð1  αÞð1  βÞð1  γ Þ ¼ 2SA1 B1 C1 
1 BB1 CC1
ða þ b þ cÞ, hence SA1 B1 C1  2AA
ðABþBCþACÞ.

(f) We have to prove that ð1  αð1  γ Þ  βð1  αÞ  γ ð1  βÞÞ αð1γ 1
Þ þ βð1αÞþ
1

γ ð1βÞÞ  3, or
1

 
1 1 1
A ¼ ðð1  αÞð1  βÞð1  γ Þ þ αβγ Þ þ þ  3:
αð1  γ Þ βð1  αÞ γ ð1  βÞ

Note that

ð1  αÞð1  βÞ βγ ð1  β Þð1  γ Þ
A¼ þ þ
α 1γ β
αγ ð1  αÞð1  γ Þ αβ 1β
þ þ þ ¼
1α γ 1β α
β 1γ γ
þβ  1 þ βþ þγ1þ γ
1γ β  1  α 
1α α 1β α
þ þα1þ α¼ þ
 γ 1
   β α  1  β
β 1γ γ 1α
þ þ þ þ  3  3,
1γ β 1α γ
110 3 Areas

since a þ 1a  2, where a > 0.


(g) Let SABC ¼ 1, then according to problems 3.1.12e and 3.1.12f maxðSAB1 C1 ;
SBA1 C1 ; SCA1 B1 Þ  14 and S2 A1 B1 C1  maxðSAB1 C1 ; SBA1 C1 ; SCA1 B1 Þ  14 S2 A1 B1 C1 
SAB1 C1  SBA1 C1  SCA1 B1 .
Let α(1  γ)  β(1  α)  γ(1  β) (see the solution of problem 3.1.12a), then we
pffiffiffiffiffiffiffiffiffiffiffiffiffiffiffiffiffiffiffiffiffiffiffiffiffiffiffiffiffiffiffiffiffiffi
have to prove that 1  αð1  γ Þ  βð1  αÞ  γ ð1  αÞ  αβð1  γ Þð1  αÞ or
qffiffiffiffiffiffiffiffiffiffiffiffiffiffiffi
αβγ
2αβγ  αβ  1β , γ ð1  βÞ  14.
But this is the statement of problem 3.1.12c. This ends the proof.

AB ¼ λ, BC ¼ μ, SABD ¼ S1, SDBC ¼ S2. Hence, it follows


3.1.13. Let us denote that BM BN

that

SKMNL ¼ S1 þ S2  ð1  λÞ2 S1  ð1  μÞ2 S2  λμðS1 þ S2 Þ ¼


   
¼ S1 2λ  λ2  λμ þ S2 ð2μ  μ2  λμÞ  2λ þ 2μ  λ2  μ2  2λμ maxðS1 ; S2 Þ 
 maxðS1 ; S2 Þ,

since 2λ  λ2  λμ ¼ λ(1  λ) þ λ(1  μ)  0, 2μ  μ2  λμ  0 and 2λ þ 2μ  λ2  μ2


 2λμ  1 ¼  (λ þ μ  1)2  0.
3.1.14. Let the tangents to a circle at point A intersect PB and PC at points M and N,
respectively. Denote by MA ¼ a, NA ¼ b, ∠ABC ¼ β, ∠ACB ¼ α, then ∠MBA ¼
∠MAB ¼ ∠ACB ¼ α, ∠NAC ¼ ∠NCA ¼ ∠ABC ¼ β.

1 sin 2α  sin 2β
ða þ bÞ2
SPMN 2 sin ð2α þ 2βÞ 1
¼ ¼ ¼
SABC 1 2 cos α  cos β  cos ðα þ βÞ
2a cos α  2b cos β  sin ðα þ βÞ
2
1 1
¼ < pffiffi2 pffiffi2 ¼ 1,
cos ðα þ βÞ þ cos ðα þ βÞ  cos ðα  βÞ
2
2
2
þ 22


cos β 2a cos α
because 2bsin β ¼ sin α and 0  α  β < α þ β  45 .

3.1.15. Let ∠A ¼ α, ∠B ¼ β, ∠C ¼ γ and the radius of the circumcircle of triangle


ABC is equal to R. Then, note that ∠B1 A1 C1 ¼ βþγ 2 , ∠A1 B1 C1 ¼ 2
αþγ
and
αþβ
∠A1 C1 B1 ¼ 2 . Therefore,

SABC 2R2 sin α sin β sin γ α β γ


¼ ¼ 8 sin sin sin ¼
SA1 B1 C1 α þ β β þ γ γ þ α 2 2 2
2R2 sin sin sin
 2 2 2
αβ αþβ γ αβ γ γ αβ
¼ 4 cos  cos sin ¼ 4 cos sin  4sin 2  cos 2 ,
2 2 2 2 2 2 2

 αβ
2
because cos 2  2 sin 2γ  0.
3.1 Inequalities with Areas 111

Similarly, one can prove that SASABC  cos 2 βγ


2 and SA1 B1 C1  cos
SABC 2 γα
2 .

1 B1 C 1
αβ βγ γα

Thus, it follows that SA B C  min cos 2 2 ; cos 2 2 ; cos 2 2 .
SABC
1 1 1

3.1.16. Consider the straight lines which pass through the vertices of a triangle and
are parallel to one of the sides of the parallelogram (Figure 3.12).
Note that it is sufficient to solve the problem for the cases, when the triangle and
the parallelogram have sides belonging to the same straight line (Figure 3.13a, b).
In a case of Figure 3.13a we have that SXYZT  SAY 1 Z1 T 1  12 SABC (see the solution
of problem 3.1.6a, remark 2), where Z1T1||AB. In the case of Figure 3.13b, we have
that SABC  SXCT  12 SXYZT .
3.1.17. (a) Let points A and B be the most removed vertices of a polygon. Then it is
clear that the polygon is inside the strip formed by the perpendiculars to the
segment AB at points A and B.
Consider the strip with a minimum width having boundaries parallel to the
segment AB and which contains the given polygon (Figure 3.14).
Then the intersection of these two strips contains the polygon and has area
2SABC þ 2SABD  2S.
(b) Let A, B, C be such vertices of the given polygon that SABC is maximal. It is clear
that the given polygon is contained in the half-plane ΠA, the boundary of which
passes through point A and is parallel to CB (Figure 3.15).

Figure 3.12

B Y Z

Y1 Y Z Z1

A X T T1 C X A B T
a b
Figure 3.13
112 3 Areas

The half-planes ΠB and ΠC are defined similarly. The intersection of these three
half-planes contain the given polygon and have the area equal to 4SABC. Thus, if
SABC  S2, then this ends the proof. It remains to consider the case, whenSABC > S2.
Consider a triangle A1B1C1 with a minimum area having parallel sides to the
sides of triangle ABC and which contains the given polygon (Figure 3.16).

Figure 3.14
C

A B

Figure 3.15

PA

A
B

Figure 3.16 A1
D
A S2 B1
B
S1 M
F S3
E
C

C1
3.1 Inequalities with Areas 113

Figure 3.17
P
F

l1(t) l2(t)
A B
C
S1 S2
E

MC
It is clear that AA1, BB1 and CC1 intersect at point M. Let CC 1
¼ k, then SABC ¼ k
(S1 þ S2 þ S3), where S1 ¼ SFAC, S2 ¼ SDAB, S3 ¼ SECB, and D, E, F are some vertices
of the given polygon.
Because SABC > S2, consequently k > 1.
We have that

SA1 B1 C1 ¼ SABC þ SA1 ACC1 þ SA1 ABB1 þ SC1 CBB1 ¼


2k þ 1
¼ SABC þ ð S1 þ S2 þ S3 Þ <
k
< SABC þ 3ðS1 þ S2 þ S3 Þ <
< 2ðSABC þ S1 þ S2 þ S3 Þ  2S:

In the absence of some of the points DEF the proof can be done similarly.
3.1.18. (a) Let points A, B be the most removed vertices of the given polygon and
C be any point on segment AB (Figure 3.17).
Consider the function f(t) ¼ l1(t)  l2(t) on the segment [0; 1], where t ¼ AC AB ,
while l1(t) and l2(t) are the lengths of the segments, cut off by the given polygon
from the midpoint perpendiculars of segments AC and CB, respectively. It is not
difficult to understand that the graph of the function f(t) is a broken line and as
l1(0) ¼ l2(1) ¼ 0, then we have that f(0) < 0 and f(1) > 0. Hence, it follows that there
exists a certain point C0, such that l1(t0) ¼ l2(t0), where t0 ¼ AC
AB (i.e. f(t0) ¼ 0).
0

Consider the straight line passing through point C and perpendicular to segment
AB. Assume that it divides the given polygon into two polygons with areas S1 and S2
(Figure 3.17).
Let us draw through points E and F support lines,1 then S1  l1(t)  AC.
Similarly, we obtain that S2  l2(t)  CB. Consequently, l1(t)  AC þ l2(t)  CB  S.
For point C0 the quadrilateral EFPQ is a parallelogram, such that

1
If the line l has at least one common point with a figure F and whole figure F is located on one side
of l, then the line l is called a support line of the figure F.
114 3 Areas

AC0 þ C0 B AC0  l1 ðt0 Þ C0 B  l2 ðt0 Þ S


SEFPQ ¼ l1 ðt0 Þ ¼ þ  :
2 2 2 2
 1 AB
(b) At t ¼ 12 we have that l1 12  AB 2 þ l2 2  2  S. Note that SEFB þ SAPQ ¼
      3
3
AB  l 1
þ 3
AB  l 1
 3
S. Therefore, max S EFB ; SAPQ  8 S.
8 1 2 8 2 2 4    
(c) At t ¼ 12 we have that SAFPBQE ¼ 38 AB  l1 12 þ l2 12  3S 4.

3.1.19. See the solution of problem 3.1.17b.


3.1.20. Let AB ¼ c, BC ¼ a, AC ¼ b, ∠A ¼ α, ∠B ¼ β, ∠C ¼ γ. We have that

1 1 1
SA1 B1 C1 ¼ SABC þ SABC1 þ SBCA1 þ SACB1  ab sin 3γ  bc sin 3α  ac sin 3β ¼
2 2 2
1 1 1
¼ 4SABC  ab sin 3γ  bc sin 3α  ac sin 3β:
2 2 2

Because sin3x ¼ sin x(2 cos 2x þ 1), then

1 1 1
ab sin 3γ þ bc sin 3α þ ac sin 3β ¼ SABC ð2 cos 2γ þ 2 cos 2α þ 2 cos 2β þ 3Þ ¼
2 2 2
¼ SABC ð4cos 2 γ þ 4cos 2 α þ 4cos 2 β  3Þ  0

(see problem 5.1.1).


Consequently, SA1 B1 C1  4SABC .
This ends the proof.
3.1.21. Let AB ¼ c, BC ¼ a, AC ¼ b, ∠A ¼ α, ∠B ¼ β, ∠C ¼ γ. We have that

1 1 1
SAA1 B þ SBB1 C þ SCC1 A ¼ ac sin α þ ab sin β þ bc sin γ
 2  2 2
sin α sin β sin γ
¼ SABC þ þ 
sin β sin γ sin α !
rffiffiffiffiffiffiffiffiffiffiffiffiffiffiffiffiffiffiffiffiffiffiffi rffiffiffiffiffiffiffiffiffiffiffiffiffiffiffiffi
sin α sin β sin γ
 SABC 2  þ2 1 1 
sin β sin γ sin α
sr ffiffiffiffiffiffiffiffiffiffiffiffiffiffiffiffiffiffiffiffiffiffiffiffiffiffiffiffiffiffiffiffiffi
ffiffiffiffiffiffiffiffiffiffi rffiffiffiffiffiffiffiffiffiffi !
sin α sin γ
 SABC 4   1 ¼ 3SABC ,
sin γ sin α

pffiffiffiffiffi
because x þ y  2 xy (x  0, y  0).
2 2 2
3.1.22. We have that SS1 ¼ ðkþmþn
k
, S2 ¼ ðkþmþn
Þ2 S
n
, S3 ¼ ðkþmþn
Þ2 S
m
Þ2
(Figure 3.18).
Consequently
3.1 Inequalities with Areas 115

Figure 3.18 B

S3 S1
X
m k
S2
A n C

Figure 3.19 D E

K M F
N
C

B A

 2  2  2 
S1 þ S2 þ S3 ¼ S k
kþmþn þ n
kþmþn þ m
kþmþn 
 
2 k 1 2 n 1 2 m 1
S   þ   þ  
3 kþmþn 9 3 kþmþn 9 3 kþmþn 9
 
2 1 S
¼S 1 ¼ ,
3 3 3

since x2  23 x  19.
3.1.23. Let us draw through points B, D and F lines parallel to segments AF, EF and
AB, respectively (Figure 3.19). Note that SBCD ¼ SBND, SDEF ¼ SDKF, SABF ¼ SBMF.
Consequently 2SBDF ¼ SABCDEF þ SMNK  SABCDEF.
3.1.24. Denote the points symmetrical to P, Q and R relative to the symmetry center
0 0 0 0 0 0
of hexagon ABCDEF by P , Q and R , respectively. It is clear that points P , Q and R
0 0 0 0 0
2
belong, respectively, to sides DE, AF and BC. Since Q R#R Q, PQ #P Q and RP #
0
RP , we have 2SPQR ¼ SPR0 QP0 RQ0  SABCDEF (see the solution to problem 3.1.23).
3.1.25. Let ABCDEF be a regular hexagon with a symmetry center O and triangle
MNP an inscribed triangle, with side MN passing through point O. One can assume

2
AB # CD denotes that the segments AB and CD are parallel and equal.
116 3 Areas

that M 2 CD, N 2 AF and P 2 AB. We have that SMNP  max (SMNA, SMNB), SMNA
¼ 12 SAMDN  12 SACDF ¼ 13 SABCDEF and SMNB ¼ SBON þ SBOM ¼ SBAO þ SBCO
¼ 3 SABCDEF . Consequently, SMNP  3 SABCDEF .
1 1

AC ¼ λ, AB ¼ μ, SBPC ¼ x, SPDE ¼ y.
3.1.26. Let us introduce the following notations: EC BD

Let SABC ¼ 1. We have that SBEC ¼ λ, SBCD ¼ μ, SADE ¼ (1  λ)(1  μ). As


SPDB  SPEC ¼ SPDE  SPBC, then

ðλ  xÞðμ  xÞ ¼ xy: ð3:6Þ

(a) According to (3.6) and

SADE ¼ ð1  λÞð1  μÞ ¼ 1  λ  μ þ x  y, ð3:7Þ

we get that y ¼ λμðλþμλμ


1λÞð1μÞ λμ
¼ λþμλμ  λμ.
pffiffiffiffi
pffiffiffiffiffi λμ
Since λ þ μ  2 λμ, then y  pffiffiffiffi  λμ.
2 λμ
 pffiffi pffiffi
Note that max f ðtÞ ¼ f 32 5 ¼ 5 5211, where f ðtÞ ¼ 2t
t
 t2 .
½0;1
pffiffiffiffiffi pffiffi
Therefore SPDE ¼ y  f ð λμÞ  max f ðtÞ ¼ 5 5211, because 0 < λμ < 1.
½0;1

(b) Since SBCDE ¼ 2SBPC, then λ þ μ þ y  x ¼ 2x. We have that 3x  y ¼ λ þ μ and


(3.6), consequently x2 ¼ λμ 2 . According to (3.7) wepfind that 2x ¼ λ þ μ  λμ,
ffiffiffiffiffiffiffiffi
2y ¼ λ þ μ  3λμ, consequently, λ þ μ ¼ λμ þ 2λμ, we deduce that
pffiffiffiffiffiffiffiffi pffiffiffiffiffi
λ þ μ ¼ λμ þ 2λμ  2 λμ. From the last inequality follows that
 pffiffiffi2 pffiffiffi
λμ  2  2 ¼ 6  4 2.
pffiffiffiffiffi pffiffiffi pffiffiffi
2λμ2λμ
We have that y ¼ λþμ3λμ ¼  5 2  7, because λμ  6  4 2. Thus
pffiffiffi 2 2
SPDE ¼ y  5 2  7.

AC ¼ λ and CB ¼ μ. Let point G be the centroid of triangle ABC. We


3.1.27. Denote QC CP
S
have that SCAM ¼ SBCM ¼ S2, where S ¼ SABC. Because SCAM CQG
¼ 23 λ, SSCBM
CGP
¼ 23 μ and
SQCP pffiffiffiffiffi
S ¼ λμ, we have 3λμ ¼ λ þ μ. Therefore 3λμ  2 λμ, hence λμ  9.
4

We need to prove that SQCP þ SAQM þ SBMP  79 S, or λμS þ S2 ð1  λÞþ


2 ð1  μÞ  9 S.
S 7

Indeed λμ þ 12 ð1  λÞ þ 12 ð1  μÞ ¼ 1  λμ
2  9. It remains to note that
7
 
SQMP ¼ S  SQCP þ SAQM þ SBMP  S  79 S ¼ 29 S, consequently SQMP  29 S.
3.1.28. It is clear that the cross section of the cube by a plane passing through its
center is a centrally symmetric convex polygon, which has an even number of sides.
Thus, it is either a quadrilateral (Figure 3.20a), or a hexagon (Figure 3.20b).
In the first case (Figure 3.20a), we have that Sсеч ¼ bh  a  a ¼ a2.
3.1 Inequalities with Areas 117

b D
b
B a c
d g
c
h b d E

A
b
a
F
a b
Figure 3.20

Figure 3.21 B S2
C

S1

A
D

yi yi¢

In the second case (Figure 3.20b), we have that


pffiffiffi
Sсеч ¼ bd sin α þ bc sin β þ cd sin γ. Since 2b, 2c, 2d  2a and α þ β þ γ ¼ π,
then according to the problem 5.2.3 in the case α, β, γ < π2, we obtain that
2
Sсеч  a2 ð sin α þ sin β þ sin γ Þ > a2 .
If γ  π2, then DE2  a2 þ a2 ¼ a2 , consequently Sсеч > SABDE  DE  a  a2.
2 2

3.1.29. It is clear that one can leave two of the given strips as they are and move the
remaining k strips. Let coordinate straight line be parallel to non of the boundary
lines of these strips.
Denote by xi (i ¼ 1, 2, . . . , k) coordinates of the intersection points of the line li
with the coordinate line, where the straight line li is parallel to the bounding lines of
the i-th strip and is equidistant from them. It is clear that there exist numbers ai and
bi, such that ai  xi  bi [because the intersection of each of these k strips with
parallelogram ABCD is not empty (Figure 3.21)], where ABCD is the intersection of
two not moved strips.
118 3 Areas

b b
x
x x
a g a 1800-g

x b b=0
a a 1800 -d
d

a b
Figure 3.22

It is clear that to each figure which is the intersection of all these strips,
corresponds a set of numbers x1, x2, . . . , xk. Denote the area of all these k þ 2 strips
by S(x1, x2, ..., xk). If all these strips do not have common points we will set S(x1,
x2, ..., xk) ¼ 0.
We shall prove that there exists a constant number M, such that at any
xi , x0i 2 ½ai ; bi , i ¼ 1, 2, . . . , k the following inequality holds true
   
Sðx1 ; x2 ; :::; xk ÞS x0 ; x0 ; :::; x0  M jx1  x0 j þ jx2  x0 j þ ::: þ jxk  x0 j :
1 2 k 1 2 k
ð3:8Þ
   
Note that S y1 ; :::; yi1 ; yi ; yiþ1 ; :::; yk  S y1 ; :::; yi1 ; y0i ; yiþ1 ; :::; yk  S1 þ

S2 (Figure 3.21), and S1 , S2  y0i  yi  d, where d is the longest diagonal of the

parallelogram ABCD. Then S1 þ S2  2 y0i  yi  d, and thus
 
Sðx1 ; x2 ; :::; xk Þ  S x0 ; x0 ; :::; x0 
1 2 k
     
 Sðx1 ; x2 ; :::; xk Þ  S x01 ; x2 ; :::; xk þ S x01 ; x2 ; :::; xk  S x01 ; x02 ; x3 :::; xk þ
   
þ ::: þ S x01 ; x02 ; :::; x0k1 ; xk  S x01 ; x02 :::; x0k 
 
 2d x1  x01 þ 2d x2  x02 þ ::: þ 2d xk  x0k ¼ M jx1  x01 j þ jx2  x02 j þ ::: þ jxk  x0k j ,

where M ¼ 2d.
Using the inequality (3.8) one can prove that there exist numbers x01 , x02 , :::, x0k ,
 
such that for any xi 2 [ai, bi], i ¼ 1, 2, . . . , k, Sðx1 ; x2 ; :::; xk Þ  S x01 ; x02 ; :::; x0k .
We shall prove that all strips to which correspond numbers x01 , x02 , :::, x0k have a
common center of symmetry.
Proof by contradiction argument.  
Let certain two parallel sides of the intersection with the area S x01 ; x02 ; :::; x0k ¼ S
be not equal. Let a > b (Figure 3.22).
3.1 Inequalities with Areas 119

Figure 3.23 y

E
a
K
O N x

Let us shift the strip, one of the boarders of which contains a, by x (Figure 3.22).
0
Denote the area of intersection of new strips by S . Then for sufficiently enough
small x we have that

2a þ xðtgβ þ tgαÞ 2b  xðtgγ þ tgδÞ


S0  S ¼ x x¼
2 2
2a  2b þ xðtgβ þ tgα þ tgγ þ tgδÞ
¼ x > 0,
2
0
i.e. S > S. This leads to a contradiction.
3.1.30. (а) Let AE ⊥ MN, OK ⊥ MN (Figure 3.23).
pffiffi pffiffi
Denote ∠AOK ¼ α, then OA ¼ 22 and OK ¼ 22 cos α  R, where 0  α < π4.
Note that ON ¼ cosOKπα , OM ¼ cosOKπþα , consequently,
ð4 Þ ð4 Þ
pffiffi 2
2 cos α R
2

SMON ¼ :
cos 2α

pffiffi
pffi 2
pffiffi 2
2 cos αR
2
(1) For 0 < R  4
2
we have to prove that cos 2α  22  R , or
pffiffi cos αpffiffiffiffiffiffiffiffiffi ffi pffiffi cos αpffiffiffiffiffiffiffiffiffi ffi pffiffi
2 ð cos 2αÞ
pffiffiffiffiffiffiffiffiffiffi 2 ð cos 2αÞ
pffiffiffiffiffiffiffiffiffiffi
2  R (α ¼
6 0). We need to prove that 2  2
4 , or
1 cos 2α
pffiffiffiffiffiffiffiffiffiffiffiffiffi 1 cos 2α
2 cos α  1  cos 2α, i.e. (cosα  1)2  0.
(2) We shall prove that for 0 < R < 12 the following inequality holds true
p ffi 2
2 cos αR
2 pffiffiffi 2
cos 2α  14  R2 , or 2R cos α  12  0.
pffiffi
For 42 < R < 12 the equality may hold true, since p1ffiffi2 < 2p1ffiffi2R < 1, hence there
exists such an angle α, for which cos α ¼ p1ffiffi . 2 2R
120 3 Areas

B1 C1 B1 W C1
B1 C1
V
A1 D1 P A1
A1 N D1 D1
S
M N O
K U
B C B C B C
b
M O1
R T
K
A D A D A Q D
a b c
Figure 3.24

(b) According to problem 3.1.29, if one shifts the plane parallel to itself so that it
passes through the center of the cube, then the area of the cross section does not
decrease. Thus, it is sufficient to make the proof for the case of dissection of the
cube by a plane passing through its center. Consider the following cases
(Figure 3.23a, b, c).
pffiffiffi
I. (Figure 3.24a) Ssec ¼ 2SANC1  2maxðSAD1 C1 ; SADC1 Þ ¼ 2.
II. (Figure 3.24b) Ssec ¼ 2SNKP  2maxðSNKC1 ; SNKC Þ ¼ maxðSANC1 K ; SA1 NCK Þ
pffiffiffi
 2.
III. Let OO1 ⊥ ABC, where O is the center of the cube, O1K ⊥ QT and O1K ¼ R,
∠O1KO ¼ β (Figure 3.24c).
pffiffi
pffi 2
12STQD 12 22R
If 0 < R  4 , then Ssec ¼ cos β  pffiffiffiffiffiffi
2
R (see problem 3.1.30 a,1).
1þR2
pffiffi 2 pffiffi
4
pffiffiffi pffiffi
We need to prove that 1  2 22  R  pffiffiffiffiffiffiffi 2R ffi
, or 2 2R  2R2  p2ffiffiffiffiffiffiffiffiffi
2R ffi
,
4þR
1 2
1þ4R2
pffiffiffi pffiffi pffiffiffi p ffiffiffiffiffiffiffiffiffiffi pffiffiffi pffiffiffiffiffiffiffiffiffiffiffiffiffiffiffiffi
ffiffiffiffiffiffiffiffiffi1
2
2  R  pffiffiffiffiffiffiffiffiffi
2 ffi
2
, or 2 p1þ4R ffi  R, or 4 2R  1 þ 4R2 þ 1 þ 4R2 .
2
1þ4R
pffiffiffiffiffiffiffiffiffiffiffiffiffiffiffiffi
1þ4R
pffiffiffi
Indeed, 1 þ 4R2 þ 1 þ 4R2 > 4R þ 2R ¼ 6R > 4 2R.
pffiffi 12ð14R2 Þ
12S
IV. If 42 < R  12, then Ssec ¼ cosTQD β  pffiffiffiffiffiffi
R (see problem 3.1.30a)
1 2 þR
4

pffiffiffi pffiffiffiffiffiffiffiffiffiffi
1 1þ4R2 1
(3) We need to prove that 2 þ 2R R
2
 2, or (1 þ x)2(1 þ x)  8x, where
2

2 < x ¼ 4R  1. Indeed, we have that (1 þ x)  8x ¼ (x  1)(x þ 4x  1)  0,


1 2 3 2

because x  1  0, x2 þ 4x  1 > 14 þ 2  1 > 0.


pffiffi qffiffiffiffiffiffiffi2ffi pffiffiffi pffiffiffi
1
þR
V. If 2 < R  2 , Ssec < cos β ¼ 4 R < 2. Hence Ssec < 2
1 2 1

3.1.31. (a) Consider Figure 3.25.


3.1 Inequalities with Areas 121

Figure 3.25 A

a-x
x

B D C

We have that SABD þ SADC ¼ SABC, consequently


AB  AC sin α
AD ¼ : ð *Þ
AB sin x þ AC sin ðα  xÞ

Taking into account (*), we obtain that


 
AB1  AC1 ABn  ACn
AD1 þ ::: þ ADn ¼ þ ::: þ
AB1 sin x þ AC1 sin ðα  xÞ ABn sin x þ ACn sin ðα  xÞ
sin α,

where ∠B1AC1 ¼ α, ∠B1AD1 ¼ x.


For positive numbers a1, . . . , an, b1, . . . , bn one can prove by the method of
mathematical induction that

a1 b1 an bn ða1 þ ::: þ an Þðb1 þ ::: þ bn Þ


þ ::: þ  : ð3:9Þ
a1 þ b1 an þ bn a1 þ ::: þ an þ b1 þ ::: þ bn

Using further the inequality (3.9) for numbers ai ¼ ABi sin x and bi ¼ ACi sin
(α  x) at 0 < x < α, we deduce that

ðAB1 þ ::: þ ABn Þ  ðAC1 þ ::: þ ACn Þ


AD1 þ ::: þ ADn  sin α
ðAB1 þ ::: þ ABn Þ sin x þ ðAC1 þ ::: þ ACn Þ sin ðα  xÞ
AB  AC sin α
¼ ¼ AD,
AB sin x þ AC sin ðα  xÞ

where AB ¼ AB1 þ . . . þ ABn, AC ¼ AC1 þ . . . þ ACn.


One of angles ∠BDA and ∠CDA is not acute. Therefore AD < max (AB, AC),
hence

AD1 þ ::: þ ADn  AD <


< maxðAB1 þ ::: þ ABn ; AC1 þ ::: þ ACn Þ:
0 0 0
(b) Let B , E , A be points symmetrical to B, E, A relative to point M (Figure 3.26).
122 3 Areas

Figure 3.26 C

B А'

E M
E' F
A B'

D B1

C1
M N1
K1
L
L1

D1
B

C N
К
A A1
a b

Figure 3.27

0 0 0
From problem 3.1.31a we have that ME þ MF  max (MC þ MA , MB þ MD),
or EF  max (AC, BD).
(c) Let the section of a tetrahedron ABCD be a quadrilateral KLMN (Figure 3.27a).
We need to prove that SKLMN  max (SAMC, SDKB). Let us project the tetrahedron
on a plane perpendicular to KM (Figure 3.27b).
Since SKLMN ¼ 12 KM  L1 N 1 and from problem 3.1.31b we have that L1N1  max
 
(A1C1, B1D1), then SKLMN  max 12KM  A1 C1 ; 12KM  B1 D1  maxðSAMC ; SBKD Þ.
It remains to note that SAMC  max (SACD, SABC) and SBKD  max (SABD, SCBD),
(see remark to the solution of problem 2.4.4.).
In the case of triangular section one can assume that it is passing, e.g., through
vertex D, i.e., points L and M have coincided with point D.
3.1.32. Let points C1, B1, A1, D1 be H images of points C, B, A, D (Figure 3.28).
3.1 Inequalities with Areas 123

B1
D
AC×CD

AB×CD
H- image BD×AC
C А C1 BC×CD
A1

BC×AD
BC×AC
B

D1

Figure 3.28

Figure 3.29 C2 C C1 D

Δ1 B1 D1
B2
B E2 Δ2
A1 E
E1
A2 A F1 F

 
(a) If ∠A þ ∠C > 180 , then ∠B þ ∠D < 180 . Hence C1B1A1D1 is a convex quad-
rilateral, thus SA1 B1 C1 þ SA1 C1 D1 > SC1 B1 D1 . SA1 B1 C1 ¼ CD2  SABC , SA1 C1 D1 ¼
BC2  SACD , SC1 B1 D1 ¼ AC2  SBCD . Consequently CD2  SABC þ BC2  SACD > AC2 
SBCD.

(b) If ∠A þ ∠C ¼ 180 , then SA1 B1 C1 þ SA1 C1 D1 ¼ SC1 B1 D1 . Thus CD2  SABC þ BC2 
SACD ¼ AC2  SBCD.
 
(c) If ∠A þ ∠C < 180 , then ∠B þ ∠D > 180 , hence point A1 is inside the triangle
C1B1D1, consequently SA1 B1 C1 þ SA1 C1 D1 < SC1 B1 D1 . Therefore, it follows that
CD2  SABC þ BC2  SACD < AC2  SBCD.
3.1.33. Consider a hexagon A2B2C2C1E2F1, where CC2 ¼ AF1, AA2 ¼ CC1, C1E2||
A2B2||DE, C2B2||F1E2||BC (Figure 3.29).
We have that the opposite sides of the hexagon A2B2C2C1E2F1 are parallel to
each other and C1 C2 ¼ AFþCD
2 ¼ A2 F1 . Consequently, C1E2 ¼ B2A2, E2F1 ¼ B2C2.
Let points E and E3 be symmetric to each other with respect to point E2, then
ABCE3 is a parallelogram. Therefore, C1 E2 ¼ ABþDE2 , F1 E 2 ¼
BCþEF
2 . One can easily
prove that B2C1||A1D1 and B2F1||B1E1, B2C1 ¼ A1D1, B2F1 ¼ B1E1.
This ends the proof of the point (a) of the problem.
124 3 Areas

Figure 3.30 d
d1

c c1 b
a

Figure 3.31
N d C P

N1 P1 D

c O
B M1 Q1

M A Q

We have that S1 ¼ SC1 F1 B2 ¼ 12 SA2 B2 C2 C1 E2 F1 ¼ 12 ðSABCDEF þ SΔ1 þ SΔ2 Þ (see the


solution of problem 3.1.23). Note that Δ1 ¼ Δ2 and SABCDEF ¼ 2SBDF  4SΔ1 . Thus
S1 ¼ SBDF  SΔ1  SBDF and SABCDEF ¼ 4S1  2SBDF. Finally we have S1 > 12 SBDF .
3.1.34. Let a  b, c  d, then b > d. Without loss of generality, one can assume that
the rectangle with sides a and b is inscribed into the rectangle with sides c and
d (Figure 3.30).
Indeed, as c1  c and d1  d, then b > d  d1. Hence, if we prove that 2ab < c1d1,
then 2ab < cd.
It is clear that after symmetry transformation with a center at point O (O is the
center of symmetry of the rectangle with sides a and b) the rectangle with sides
c and d transforms into itself. Hence, the centers of these rectangles coincide. Since
the vertices of the rectangle with sides a and b are on the circle with the center at
pffiffiffiffiffiffiffiffiffi
O and a radius a2þb , then it is clear that the positioning shown in Figure 3.31 is
2 2

possible.
To conclude the proof it remains to draw through the vertices A, B, C, D of the
rectangle with the sides a and b a straight line, parallel to the sides of the rectangle
with the sides c and d. We have that

ab ¼ SABCD < SBCQ1 þ SADN1 þ SCDP1 þ SABM1 ¼ SBCN þ SADQ þ SCDP þ SABM
¼ cd  ab:

Thus 2ab < cd.


3.1.35. (a) If O is on AC, then ABCD, AKON, and OLCM are similar. Therefore
pffiffiffiffi pffiffiffiffi pffiffiffiffiffi pffiffiffiffiffi pffiffiffi
pSffiffi1 þ pSffiffi2 ¼ AO þ CO ¼ 1. Consequently, S1 þ S2 ¼ S. But if O does not
S S AC AC
belong to AC, then one can assume that O and D are on the same side of AC.
3.1 Inequalities with Areas 125

Denote the intersection points of a straight line passing through O with lines BA,
AD, CD and BC, respectively, W, X, Y and Z. Assume that W
X
A. Then OW OX ¼ 1,
and OY > 1. Now we rotate the line around point O in such a way that it does not
OZ

pass through B until it reaches the position, such that Y


Z
C. Then OW OX > 1, while

OY ¼ 1. Therefore there exists an intermediate position of the line, such that


OZ

OX ¼ OY . Let P1, P2, Q1 and Q2 denote


OW OZ
the areas of triangles WKO, OLZ, ONX and
pffiffiffi pffiffiffiffiffi pffiffiffiffiffi
OMY, respectively. The inequality S  S1 þ S2 is equivalent to the inequality
pffiffiffiffiffiffiffiffiffi
T 1 þ T 2  2 S1 S2 .
pffiffiffiffiffiffiffiffiffiffi pffiffiffiffiffiffiffiffiffiffiffi  2 OX2 Q
T 1 þ T 2 ¼ 2 P1 P2 þ 2 Q1 Q2 and PP12 ¼ OW ¼ OY ¼ Q1 , then T 1 þ T 2 ¼
pffiffiffiffiffiffiffiffiffiffi pffiffiffiffiffiffiffiffiffiffiffi pffiffiffiffiffiffiffiffiffiffiffiffiffiffiffiffiffiffiffiffiffiffiffiffiffiffiffiffiffiffiffiffiffiffiffiffiffiffiffiffi
OZ
pffiffiffiffiffiffiffiffiffi 2

2 P1 P2 þ 2 Q1 Q2 ¼ 2 ðP1 þ Q1 ÞðP2 þ Q2 Þ  2 S1 S2 (IMO, 1995, proposed


problems, Latvia).
(b, c) Let O belong to triangle ACD and SABC ¼ a, SACD ¼ b and SOAC ¼ x. We
1 =2 T 1 =2 T 1 =2
have that TSAOB  SBOC ¼ BK AB  BC ¼ SABC , Therefore
BL

2SAOB  SBOC
T1 ¼ :
a

Similarly we obtain that T 2 ¼ 2SAODbSCOD . Consequently,

pffiffiffiffiffi pffiffiffiffiffi SOAB þ SOBC SOAD þ SOCD a þ x b  x


T1 þ T2  pffiffiffiffiffi þ pffiffiffiffiffi ¼ pffiffiffiffiffi þ pffiffiffiffiffi
2a 2b 2a 2b
pffiffiffi pffiffiffi pffiffiffi pffiffiffi
aþ b a b
¼ pffiffiffi  pffiffiffiffiffiffiffiffi x:
2 2ab
pffiffiffiffiffi pffiffiffiffiffi pffiffi pffiffi pffiffiffiffiffiffiffiffiffiffiffi pffiffiffi
If a  b, then T 1 þ T 2  apþffiffi2 b  a þ b ¼ S.
If a < b, then, since point O cannot be outside of parallelogram ABCE, then
pffiffiffiffiffi pffiffiffiffiffi pffiffiaþpffiffib pffiffiapffiffib pffiffi pffiffi
x  a. Therefore we deduce that T 1 þ T 2  pffiffi2  pffiffiffiffiffi ffi a ¼ apþffiffi b 
pffiffi pffiffi pffiffiffi pffiffiffiffi
2ab 2
affiffiffiffi b bþ2pffiffiffi ffi a
p a¼ ab
.
pffiffiffi
2b 2b

bþ2pffiffiffi a sin ð2αþπ=4Þ
ab
Denote ab ¼ tg 2 α, where α 2 0; π4 . Then pffiffiffiffiffiffi 2b
¼ cos α  C0 .
pffiffiffiffiffi pffiffiffiffiffi bþ2pffiffiffi ffi aþb
p ffiffiffiffiffiffiffiffiffiffiffi p ffiffiffi
Consequently, T 1 þ T 2  pffiffiffi ffi a  C0 a þ b ¼ C0 S. When α ¼ π4,
ab

sin ð2αþπ=4Þ
2b
pffiffiffiffiffi pffiffiffiffiffi pffiffiffi
cos α ¼ 1, i.e. C0  1. Thus, in all cases T 1 þ T 2  C0 S.
Note that if in a quadrilateral the conditions AB ¼ BC, AD ¼ CD are satisfied and
ABCO is a parallelogram, then using the condition SSACD ABC
¼ tg 2 α0 , where 0  α0  π4
sin ð2α0 þπ=4Þ pffiffiffiffiffi pffiffiffiffiffi pffiffiffi
and C0 ¼ cos α0 we obtain that T 1 þ T 2 ¼ C0 S.
 
We need to prove that, if 0  α  π4, then sin 2α þ π4 < 54 cos α. This ends the
proof of the point (b)
of the problem.
Indeed, let φ 2 0; π4 and cos φ ¼ 45, then at 0  α < φ, we have that
 
sin 2α þ π4 < 1 ¼ 54 cos φ < 54 cos α. But if φ  α  π4, then tgφ ¼ 34 >
126 3 Areas

Figure 3.32 A D1 B
M4

M
O M1
C1
М3
A1

K
M2
D

B1
C

pffiffiffi     pffiffi
2  1 ¼ tg π8. Therefore, φ > π8 and sin 2α þ π4  sin 2φ þ π4 ¼ 22  31
25 <
pffiffi
2  4  1, 25 cos α.
2 5

Remark
 Using
 the concept of a derivative it is possible to prove that
tg 2α0 þ π4 tg α0 þ 2 ¼ 0, or tg3α0 þ 3tgα0  2 ¼ 0. Consequently, tg α0 ¼
p ffiffiffiffiffiffiffiffiffiffiffiffiffiffiffi
pffiffiffi p ffiffiffiffiffiffiffiffiffiffiffiffiffiffiffi
pffiffiffi
2 þ 1 2  1¼ 0, 59 . . ., then C0 ¼ 1, 11. . ..
3 3

3.1.36. (Solution of O. Sarkisyan, 9th grade) Let the diagonals of the quadrilateral
ABCD intersect at point O (Figure 3.32) and OC AO
¼ λ, OD
BO
¼ μ.
Let us express S0 through S, λ, μ. Since segment AA1, BB1, CC1 and DD1 medians
of triangles ABC, BCD, ACD and ABD, then
S
SAA1 CC1 ¼ SAA1 C þ SCC1 A ¼ SABA1 þ SCDC1 ¼ :
2
Similarly,
S
SBB1 DD1 ¼ SBCB1 þ SDAD1 ¼ :
2
Consequently,

S ¼ SAA1 CC1 þ SBB1 DD1 ¼ SABA1 þ SBCB1 þ SCDC1 þ SDAD1 :

Hence, we obtain that S0 ¼ SBA1 M1 þ SCB1 M2 þ SDC1 M3 þ SAD1 M4 . Let us now


calculate SBA1 M1 . From the similarity of triangles BMM1 and B1A1M1 we find that
BM1 BM
¼ : ð3:10Þ
M 1 B1 A1 B1
From the similarity of triangles AMO and AA1K, we have that
3.1 Inequalities with Areas 127

AK A1 K BM þ MO 1 BM
¼ ¼ ¼ þ :
AO MO 2MO 2 2MO

On the other hand, we have that

AK AO þ OK OC 1
¼ ¼1þ ¼1þ :
AO AO 2AO 2λ
λþ1
Therefore, 1 þ 2λ
1
¼ 12 þ 2MO
BM
, from which it follows that BM ¼ 2λþ1 BO. From
(3.10) we deduce that

BM1 BM λ þ 1 BO
¼ ¼  ¼
M1 B1 A1 B1 2λ þ 1 BD=2
ð2λ þ 2Þμ
¼ :
ð2λ þ 1Þðμ þ 1Þ

Consequently,

BM1 1 2ðλ þ 1Þμ


¼ ¼ :
BB1 1 þ MBM
1 B1 4λμ þ 3μ þ 2λ þ 1
1

SBM1 A1 1 BA1 ðλþ1Þμ


Since SBCB1 ¼ BM
BB1 BC ¼ 2BB1 ¼ 4λμþ3μþ2λþ1 and SBCD ¼ OC ¼ λ, then
BM1 SABD AO

ðλ þ 1Þμ ðλ þ 1Þμ SBCD


SBM1 A1 ¼ SBCB1 ¼
4λμ þ 3μ þ 2λ þ 1 4λμ þ 3μ þ 2λ þ 1 2
μ
¼  S ¼¼ f ðλ; μÞ  S,
2ð4λμ þ 3μ þ 2λ þ 1Þ

where f ðx; yÞ ¼ 2ð4xyþ3yþ2xþ1


y
Þ.
   
Similarly, we obtain that SCM2 B1 ¼ f μ; 1λ S, SDM3 C1 ¼ f 1λ; μ1 S,
 
SAM4 D1 ¼ f μ1; λ S. Therefore

1 μ 1 λ
S0 ¼ þ þ þ
2 4λμ þ 3μ þ 2λ þ1 4μ þ 3 þ 2μλ þ λ 4 þ 3λ þ 2μ þ λμ
ð3:11Þ
λμ
þ  S:
4λ þ 3λμ þ 2 þ μ

Denote 4λμ þ 3μ þ 2λ þ 1 ¼ a1, 4μ þ 3 þ 2μλ þ λ ¼ a2, 4 þ 3λ þ 2μ þ λμ ¼ a3.


Hence, it follows that λ ¼ 2a3 a
5
2 5
, μ ¼ 2a2 a
5
1 5
, λμ ¼ 2a1 a25a3 þ5. Now we have
to prove that
128 3 Areas

1 2a2  a1  5 1 2a3  a2  5 2a1  a2  a3 þ 5 2


< þ þ þ ¼A :
3 5a1 a2 5a3 5a4 5

It is not difficult to prove that


(a) a1 þ a3 ¼ a2 þ a4,
(b) a1 a3  a2 a4 ¼ 12 ð5ða2 þ a4 Þ þ a1 a2  2a2 a3 Þ,
(c) 3a1a3  2a2a4 > 0,
(d) 3a2a4  2a1a3 > 0.
Consider now the expression A  25. By reducing to a common denominator and
regrouping the similar members, using the property (a) and presenting the expres-
sion a1a2a3a4 in the form a1a2a3(a1 þ a3  a2), we deduce that A  25 ¼
5a1 a3 ða2 þa4 Þ5a2 a4 ða1 þa3 Þ2a2 a3 ða1 a3 a2 a4 Þþa1 a2 ða1 a3 a2 a4 Þ
5a1 a2 a3 a4 .
Replacing in the obtained expression a2 þ a4 by a1 þ a3 we obtain that
A  25 ¼ ða1 a3 a2 a4 Þð55aða11aþa 3 Þþa1 a2 2a2 a3 Þ
2 a3 a4
. According to the property (b), from the last
2
expression we obtain that A  25 ¼  2ða5a1 a13aa 2 a4 Þ
2 a3 a4
 0. Thus A  25. Note that the
equality holds true if and only if a1a3  a2a4 ¼ 0. However from the condition (a) it
follows that 2(a1a3  a2a4) ¼ (a2  a1)(a2 þ a1  a3  a4). Since a1a3  a2a4 ¼ 0,
then either a2  a1 ¼ 0 or a2 þ a1  a3  a4 ¼ 0. If a2 ¼ a1, then we obtain that
4λμ þ 3μ þ 2λ þ 1 ¼ 4μ þ 3 þ 2λμ þ λ, or μ ¼ 2λ1 2λ
.
But if a2 þ a1 ¼ a3 þ a4, then accounting to (a) it follows that a1 ¼ a4,
i.e. 4λμ þ 3μ þ 2λ þ 1 ¼ 4λ þ 3λμ þ 2 þ μ or μ ¼ 2λþ1 λþ2 .
Now, let us note that from a2  a1 ¼ 0 (or a2 þ a1  a3  a4 ¼ 0) and the condi-
tion (a) we deduce that a1a3  a2a4 ¼ 0. Therefore the last condition is equivalent to
either μ ¼ 2λ1
2λ
or μ ¼ 2λþ1
λþ2 .
2
Now, we need to prove that A > 13. Since A ¼ 25  2ða5a1 a13aa 2 a4 Þ
2 a3 a4
, then it
2
13a1 a2 a3 a4 6a21 a23 6a22 a24
remains to prove that 2
5  2ða5a1 a13aa 2 a4 Þ
2 a3 a4
> 13, or 15a1 a2 a3 a4 > 0, or
ð3a1 a3 2a2 a4 Þð3a2 a4 2a1 a3 Þ
15a1 a2 a3 a4 > 0.
The last inequality is holds true due to conditions (c) and (d).
3.1.37. Note that (see Figure 3.33).
pffiffiffi
c2 a2 2    a2 þ b2 þ c 2 2 3
O2 O3 ¼ þ  ac cos 60 þ β ¼
2
 S:
3 3 3 6 3
pffiffi
Similarly we find that O1 O3 2 ¼ O1 O2 2 ¼ a þb6 þc  2 3 3 S.
2 2 2

pffiffi pffiffi     pffiffi 


2 2
Thus SO1 O2 O3 ¼ 43 O2 O3 2 ¼ 43 c3 þ a3  23ac cos 60  β þ 4 3 3 S  S.
3.1.38. Let A1B1C1 and A2B2C2 be two nonintersecting triangles. Note that there
exists a straight line l, containing one of the sides of one of these triangles, so that
triangles A1B1C1 and A2B2C2 are on different sides of this line. Indeed, let O be any
point inside triangle A1B1C1 and k be the minimal positive number at which
3.1 Inequalities with Areas 129

Figure 3.33
B
O2
O3
b
c a

A b C

O1

А2

B¢ B¢ B2
A¢ A¢ B1
A1 B1 A1
C2
А2

C1 B2 C1 l

l C2
C¢ C¢
a b

Figure 3.34

0 0 0 0 0 0
triangles A B C and A2B2C2 have a common point, where A B C is the image of
triangle A1B1C1 under similarity transformation with a center O and similarity
coefficient k. The existence of line l follows from Figure 3.34.
Let A1B1C1 and A2B2C2 be two given triangles. Note that if line l contains sides
A1C1 or B1C1, then parallelograms A1C1B1D1 and A2C2B2D2 do not have any
common internal point, while if line l contains side A1B1, then hexagons
A1M1N1B1K1P1 and A2M2N2B2K2P2 do not have common internal points [here
MiNi and PiKi are midlines of triangles AiBiCi and AiBiDi (i ¼ 1, 2), respectively
(Figure 3.35)].
From the aforesaid it follows that, if given triangles AiBiCi, i ¼ 1, 2, . . . , n do
not have any common internal point, then any two of the hexagons AiMiNiBiKiPi,
i ¼ 1, 2, . . . , n also do not have any common internal point.
P
n P
n
Therefore, S0  SA i M i N i B i K i P ¼ 2 SAi Bi Ci ¼ 2 S.
3 3
i¼1 i¼1
130 3 Areas

Figure 3.35 D2

P2 K2

A2 B2
D1
M2 N2
P1 K1
A1 C2 B1 l

M1 N1

C1

Figure 3.36 N

A C
A1 C1

F E1 D

M E K

3.1.39. Let us draw through points A, C and E straight lines MN, NK and MK parallel
to BF, BD and DF, respectively (Figure 3.36).
Then ΔBFD  ΔMNK, and let MN ¼ λBF.
Note that

λ2 SBDF ¼ SMNK ¼
¼ SBDF þ SMNBF þ SNKDB þ SMKDF :

Thus, it follows that

ðλ þ 1ÞBF ðλ þ 1ÞBD ðλ þ 1ÞDF  


AA1 þ CC1 þ EE1 ¼ λ2  1 SBDF :
2 2 2
Therefore, we deduce that

BF  AA1 BD  CC1 DF  EE1


SABCDEF ¼ SBDF þ þ þ ¼ λSBDF :
2 2 2

We have that R > r1 (see the solution of problem 7.1.79), where r1 is the radius of
the incircle of triangle BDF. Hence, we obtain that
3.1 Inequalities with Areas 131

Figure 3.37 A

B M E

D
C

R R pffiffiffiffiffiffiffiffiffiffiffiffiffiffiffiffiffiffiffiffiffiffi
SABCDEF  SBDF ¼ SACE  SBDF :
r1 r
Remark 1. If center O of the circumcircle of a triangle ACE is in a convex hexagon
ABCDEF, then SABCDEF ¼ SABOF þ SBCDO þ SDEFO  BFR 2 þ 2 þ 2 ¼
BDR FDR
R
r1 SBDF .
qffiffiffiffiffiffiffiffiffiffiffiffiffiffiffiffiffiffiffiffiffiffiffiffiffiffiffiffiffiffiffiffiffiffiffi
2. If ABCDEF is a convex hexagon, then SABCDEF  Rr  Rr11  SACE  SBDF , where r,
R and r1, R1 are the inradius and circumradius of triangles ACE and BDF,
respectively.
3.1.40. Denote given five points by A, B, C, D, E. If the pentagon with vertices at A,
B, C, D, E is not convex, then we can assume that point D is inside triangle ABC,
pffiffiffi
then SABC ¼ SABD þ SDBC þ SDAC > 6 > 5 þ 1.
Consider now the case when pentagon ABCDE is convex. Let M be the point of
intersection of BE with AC (Figure 3.37).
pffiffi pffiffi pffiffi pffiffiffi
Suppose that BM ME
 1þ2 5, then SSACE ¼ BM
ME
 1þ2 5, SACE  SABC 1þ2 5 > 1 þ 5.
pffiffi ABC
pffiffi
51
ME
In the case BM < 1þ2 5, SMDE  min (SCDE, SADE) > 2, SSBMD ¼ BM
ME > 2 , SBMD >
pffiffi p ffiffi
ffi p ffiffi
MDE
ffi pffiffiffi
51
2 SMDE ¼ 5  1. Therefore SBDE ¼ SBMD þ SMDE > 5  1 þ 2 ¼ 5 þ 1.
3.1.41. In the case when four vertices of the hexagon A1A2A3A4A5A6 are on two
adjacent sides of the unit square ABCD, we have that if A1, A2 2 AB, A3, A4 2 BC,
 4
then SA1 A2 A3 SA2 A3 A4 ¼ 14 A1 A2  BA3  A2 B  A3 A4  14 A1 A2 þBA3 þA
4
2 BþA3 A4
 64
1
.
Consequently, minðSA1 A2 A3 ; SA2 A3 A4 Þ  18.
It remains to consider the case when A1, A2 2 AB, A3 2 BC, A4, A5 2 CD,
A6 2 AD.
Let A4C  A2B. If DA5  AA1, then we proceed to the proof by contradiction
argument. Suppose that the areas of all these triangles are greater than 18. Then by
approaching point A3 to C, and point A6 to D, one can obtain SA3 A4 A5 ¼ SA4 A5 A6 ¼ 18,
while other areas are greater than 18. Now by drawing together points A1 and A2, we
can reach SA6 A1 A2 ¼ SA1 A2 A3 ¼ 18, while areas of other two triangles are greater than 18.
Then we have that
132 3 Areas

1 þ A1 A2 1 þ A4 A5
SA1 A2 A3 A4 A5 A6 ¼ SA1 A2 A3 A6 þ SA6 A3 A4 A5 ¼ BA3 þ CA3 ¼
2 2
1 A1 A2  BA3 A4 A5  CA3 3
¼ þ þ ¼ :
2 2 2 4
On the other hand
1 A1 A2 þ A4 A5 1
SA1 A2 A3 A4 A5 A6 ¼ SA6 A1 A5 þ SA2 A3 A4 þ SA1 A2 A4 A5 > þ ¼
  4 2 4
1 1 1 1 1 1 1 3
þ þ ¼¼ þ  þ ¼ :
8 BA3 A3 C 4 8  BA3  A3 C 4 2  ðBA3 þ A3 CÞ 2 4

This leads to a contradiction.


If DA5 < AA1, then moving point A3 closer to C, and point A6 to A, one can obtain
that SA6 A1 A2 ¼ 18 , SA5 A3 A4 ¼ 18, while the areas of remaining four triangles are greater
than 18. Now, let us move points A3 and A6 in a direction parallel to AB, so that
SA6 A1 A2 ¼ SA6 A1 A5 ¼ 18, SA2 A3 A4 ¼ SA3 A4 A5 ¼ 18, SA1 A2 A3 > 18, SA6 A5 A4 > 18, PQ ¼ 1,
QR < 1 (Figure 3.38).
Note that A1A6||A2A5||A3A4. Let PO ¼ 1 and SV||A3A4. Denote A1A6 ¼ a,
A3A4 ¼ b, ∠QA1A6 ¼ α, then α > 45 and RA3 > QA6. Therefore,
a sin α þ b sin α < 1.
We have that
pffiffiffi
2 ¼ QO > QV ¼ QM þ MN þ NU þ UV ¼
a cos α sin α 1 1 b cos α sin α
¼   þ   þ   þ   ¼
sin α þ 45  4a sin α þ 45 4b sin α þ 45 sin α þ 45
1 aþb
¼    ða þ bÞ cos α sin α þ
sin α þ 45  4ab

1 1
    ða þ bÞ cos α sin α þ :
sin α þ 45 aþb

Figure 3.38 Q A1 A2 R
0 1
45 a
4a
M
a
N
A3
U
A6 1 b V
4b
a
P A5 A4 S O
3.1 Inequalities with Areas 133

 i
Since the function f ðxÞ ¼ x cos α sin α þ 1x decreases in 0; pffiffiffiffiffiffiffiffiffiffiffiffiffiffi
1
cos α sin α
ffi and
aþb< 1
< pffiffiffiffiffiffiffiffiffiffiffiffiffiffi
1 ffi, ða þ bÞ cos α sin α þ
then > cos α sin αþ
1 1
sin α cos α sin α sin α
pffiffiffi  aþb

sin α ¼ cos α þ sin α. Consequently, 2 > sin αþ45 1
 ða þ bÞ cos α sin α þ aþb
1
ð Þ
cos αþ sin α
pffiffiffi
> sin  ¼ 2 . This leads to a contradiction.
ðαþ45 Þ
This ends the proof.
Remark The number 18 cannot be made smaller, because if A1
A, A4
C,
A1 A2 ¼ A1 A6 ¼ 12 ¼ A3 A4 ¼ A4 A5 , then SA1 A2 A3 ¼ SA2 A3 A4 ¼ ::: ¼ SA6 A1 A2 ¼ 18.
0 0
3.1.42. Let us consider point B and P , such that BB ~ 0 ¼ PP~ 0 ¼ AC. ~ Let O be the
0
midpoint of segment BC and AB ¼ c, BC ¼ a, AC ¼ b, PB ¼ d.
0 0
According to problem 1.1.14a, for points P, B , P , C we have that bc  d þ 6.
0 0
Since quadrilateral ABB C is a parallelogram, then O is the midpoint of segment AB .
0 2 0
Consequently, PO2 ¼ 2þ2d 4AB , AB 2 ¼ 2b2 þ 2c2  a2, PO2 ¼ 8þ18a
2 2
4 .
Hence, we obtain that b þ c  a ¼ d  12, then d  12 ¼ bc  d þ 6. Thus
2 2 2 2 2
pffiffiffiffi pffiffi pffiffi  pffiffiffiffiffi pffiffi
d  1þ2 73, therefore SABC ¼ 43 bc  43 ðd þ 6Þ  13 þ 73 83.
3.1.43. First we prove that if given an angle ∠MAN ¼ α, then on sides AM, AN one
can find points B0, C0, respectively, and inside the given angle a point P, such that
PA0 ¼ r1, PB0 ¼ r2, PC ¼ r3 and ∠PB0A ¼ ∠PC0A.
Indeed, let point K be inside angle MAN and ρ(K, AM) ¼ r2, ρ(K, AN) ¼ r3.
Denote by P the intersection point of the circle with a center at A and radius r1
with ray AK. Then AP ¼ r1 and ρρððP;AM Þ
P;AN Þ ¼ r3 , and it remains to use the conditions
r2

r2 > r1 and r3 > r1 to choose on rays AM and AN points B0 and C0, such that
PB0 ¼ r2, PC0 ¼ r3 (Figure 3.39).
Then ΔPEB0 and ΔPFC0 are similar.
Hence ∠PB0A ¼ ∠PC0A. We need to prove that SABC  SAB0 C0 .
Let ΔABC and point P satisfy the conditions of the problem.
0 0
Consider points B and P , such that BB ~ 0 ¼ PP~ 0 ¼ AC~ 0 , then (see the solution of
0
problem 3.1.42), if PB ¼ d, we obtain that

Figure 3.39 B0
M

r2
E
P
r1 r3

a
A F C0 N
134 3 Areas

bc  dr 1 þ r 2 r 3 ð3:12Þ

and d2 ¼ r 22 þ r 23  r 21 þ 2bc cos α.


Consequently,

d2  r 22 þ r 23 
r 21 þ 2ðdr 1 þ r 2 r 3 Þ cos α:
pffiffiffiffiffiffiffiffiffiffiffiffiffiffiffiffiffiffiffiffiffiffiffiffiffiffiffiffiffiffiffiffiffiffiffiffiffiffiffiffiffiffiffiffiffiffiffiffiffiffiffiffiffiffiffiffiffiffiffiffiffi
Hence d  r 1 cos α þ r 22 þ r 23 þ 2r 2 r 3 cos α  r 21 sin 2 α.
Therefore,

1
SABC ¼ bc sin α 
2 qffiffiffiffiffiffiffiffiffiffiffiffiffiffiffiffiffiffiffiffiffiffiffiffiffiffiffiffiffiffiffiffiffiffiffiffiffiffiffiffiffiffiffiffiffiffiffiffiffiffiffiffiffiffiffiffiffiffiffiffiffi 
1 2
 r 1 cos α þ r 1 r 22 þ r 23 þ 2r 2 r 3 cos α  r 21 sin 2 α þ r 2 r 3 sin α ¼ SAB0 C0 ,
2

since for triangle AB0C0 the inequality (3.12) turns into equality.
Now we will find the minimal value of SABC. To do that, let us note that points A,
B, C are on the circles with a center at point P and with radiuses r1, r2, r3, respec-
tively (see Figure 3.40).
0 0
Note that triangle AB C also satisfies the  conditions of the problem and
SABC  SAB0 C0 ¼ 14 AB  AB0  AC  AC0 sin 2 α ¼ 14 r 22  r 21 r 23  r 21 sin 2 α.
Thus, it follows that
 2  
1 2  2  1 r 2  r 21 r 23  r 21 sin 2 α
SABC ¼ r 2  r 1 r 3  r 1  sin α
2 2 2
 :
4 SAB0 C0 4SAB0 Co

ðr22 r21 Þðr23 r21 Þsin 2 α


This means that the minimal value of SABC is equal to 4SAB0 Co .
This ends the proof.

Figure 3.40 C

a B
A
a
B¢ P

C'
3.1 Inequalities with Areas 135

Figure 3.41

3.1.44. Consider the following two cases.


(a) If any two of the constructed parallelograms (Figure 3.41а) do not have a
common point, denote by S0 the intersection area of the parallelograms and
let SA1 B1 C1 D1 ¼ S1 , then we have that S0 > S1  S0.

Thus, it follows that S0 > S21 :


(b) If two of the constructed parallelograms have common points (Figure 3.41b),
then any two among the other four constructed parallelograms do not have a
common point.
Therefore S0 > S2 , where S ¼ SABCD.

Problems for Self-Study

3.1.45. Prove that in any convex polygon one can place a rectangle having the area
not less than 14 of the area of the given polygon.
3.1.46. There are 5 patches placed on the shirt with area 1, the area of each of them
being not less than 12. Prove that one can find two patches so that the area of their
common parts is not less than 15.
3.1.47. Let T1 and T2 be two triangles with sides a1, b1, c1 and a2, b2, c2. Prove that
there exists a triangle T with sides a, b, c, such that if S1, S2 and S are the areas of
triangles T1, T2 and T, respectively, then
qffiffiffiffiffiffiffiffiffi qffiffiffiffiffiffiffiffiffi qffiffiffiffiffiffiffiffiffi
S1 þS2 a21 þa22 b21 þb22 c21 þc22
(a) S  2 , where a ¼ 2 , b¼ 2 ,c ¼ 2 ,
pffiffiffiffiffiffiffiffiffiffiffiffiffi
(b) S  4 S1  S2 , where a ¼ a1 þ a2, b ¼ b1 þ b2, c ¼ c1 þ c2.
136 3 Areas

3.1.48. The area of a convex quadrilateral ABCD is equal to S. Prove that the area of
the quadrilateral, with the vertices at the midpoints of segments AC, AD, BD and
BC, is less than S2.
3.1.49. Prove that in a triangle with area S one can inscribe a regular triangle, such
that its area is not greater than S4.
3.1.50. Prove that any acute triangle with area 1 can be placed into right-angled
pffiffiffi
triangle with area not greater than 3.
3.1.51. Given several squares the sum of area of which is equal to S. Prove that
(a) one can place them without overlapping inside a square with area 2S,
(b) with these squares one can always cover the square with area S4.

3.1.52. Let AD be the altitude of the right-angled triangle ABC, ∠A ¼ 90 . The
straight line passing through the centers of the incircles of triangles ABD and ACD
intersects sides AB and AC, respectively, at points K and L. Prove that
SABC  2SAKL.
3.1.53. Bisectors of angles A, B, C of an acute triangle ABC intersect its circumcir-
cle at points A1, B1, C1, respectively. The straight line AA1 intersects the bisectors of
the external angles B and Cof triangle ABC at point A0. Points B0, C0 are defined
similarly. Prove that SA0 B0 C0  4SABC .
3.1.54. Let ABC be an acute triangle, points M, N and P be the feet of the
perpendiculars drawn from the centroid of the triangle to sides AB, BC and CA,
respectively. Prove that SMNP > 27
4
SABC .
3.1.55. Let ABCD be a unit square. For any internal points M and N, such that line
MN does not contain any of the vertices of the square, denote by S(M, N ) the least of
the areas of the triangles with the vertices from the set of the points {A, B, C, D, M,
N}. Find the smallest number k, such that S(M, N )  k for all such points M and N.
Hint Let point N be inside of triangle CMD, then
SAMB þ SCMD ¼ 12, SCMD ¼ SCMN þ SMND þ SCND  3S(M, N ). Therefore
4SðM; N Þ  12. Hence, it follows that SðM; N Þ  18.
3.1.56. Points K, L, M and N are taken on sides AB, BC, CD and DA of a convex
quadrilateral ABCD, respectively. Denote by S1, S2, S3 and S4 the areas of
pffiffiffiffiffi pffiffiffiffiffi
triangles AKN, BKL, CLM and DMN, respectively. Prove that 3 S1 þ 3 S2 þ
p ffiffiffiffi
ffi p ffiffiffiffi
ffi p ffiffiffiffiffiffiffiffiffiffiffi

3
S3 þ 3 S4  2 3 SABCD .
sffiffiffiffiffiffiffiffiffiffiffiffiffiffiffiffiffiffiffiffiffiffiffiffiffiffiffiffiffiffiffiffiffiffiffiffiffiffiffiffiffiffiffiffiffiffiffiffiffiffiffiffiffiffiffiffiffiffiffiffiffiffiffiffiffiffiffiffiffiffiffiffiffiffiffiffiffiffiffiffiffiffiffiffiffiffi
rffiffiffiffiffiffiffiffiffiffiffiffiffiffiffiffiffiffiffiffiffiffiffiffiffiffiffiffiffiffiffiffiffiffiffiffiffiffiffiffiffiffiffiffiffiffiffiffiffiffiffiffiffiffiffiffiffiffiffiffiffiffiffiffiffiffi
3
 
Hint Prove that SABCD ¼ AK S1
 
AN SABD
 þ þ SABD
3 1 AK AN
AB AD SABCD 3 AB AD SABCD .

3.1.57. Prove that if a convex pentagon A1A2A3A4A5 is inside of a parallelogram


with area 1, then one of the triangles A1A2A3, A2A3A4, A3A4A5, A4A5A1, A5A1A2 has
an area that does not exceed 14.
3.1 Inequalities with Areas 137

Hint Draw a segment connecting the opposite sides of the parallelogram, then one
can assume that ΔA1A2A3 is inside the parallelogram with area 12.
Consequently, SA1 A2 A3  14.
3.1.58. (a) Let ABCDEF be a convex hexagon, such that AB||ED, BC||FE and CD||
AF. Prove that
 6
1
SABC  SBCD  SCDE  SDEF  SEFA  SFAB  SABCDEF : ð3:13Þ
6

Hint Prove that SABCDEF  2SABC þ 2SCDE þ 2SEFA.

Remark If BCEF is a rectangle, AB ¼ CD ¼ 3, AF ¼ DE ¼ 4, BF ¼ 5, BC ¼ 10,


then (3.13) is wrong.
(b) Let ABCDEF be a convex hexagon. Prove that minðSABC; SBCD; SCDE; SDEF; SEFA; SFABÞ
 16 SABCDEF .
3.1.59. Given points A, B, C, D inside a square with a side equal to 6, such that the
distance between each two of them is not less than 5. Prove that points A, B, C, D
form a convex quadrilateral with the area greater than 21.
3.1.60. Let ABCD be a convex quadrilateral and K be the intersection point of its
diagonals. Let also the conditions KL||AB, LM||DC, MN||AB hold true for the points
L 2 [AD], M 2 [AC], N 2 [BC]. Prove that SSKLMN
ABCD
< 27
8
.

3.1.61. Let S be the area of a convex quadrilateral ABCD, a, b, c, d be the lengths of


its consecutive sides, e, f be the lengths of the diagonals, and m, n the lengths of its
medians connecting the midpoints of the opposite sides of the quadrilateral.
Prove that:
(a) S  12 ef ,
(b) S  mn,
(c) S  abþcd
2 ,
(d) S  2 ,
acþbd

(e) S  14 ða þ cÞðb þ dÞ,


(f) S  14 p2 , where p ¼ aþbþcþd ,
 2
2
(g) S  4 ða þ cÞ þ bd .
1

3.1.62. Prove that none of the triangles inscribed into polygon M can have an area
greater than the maximal area of the triangles, the vertices of which coincide with
some of the vertices of M.
3.1.63. (a) Let M be a convex polygon and l be an arbitrary straight line. Prove that
one can inscribe into M a triangle, оne side of which is parallel to l and the area of
which is not less than 38 of the area of M.
138 3 Areas

(b) Let M be a regular hexagon and l be a line parallel to one of its sides. Prove that
one cannot inscribe into M a triangle, оne side of which is parallel to l and the
area of which is greater than 38 of the area of M.
3.1.64. Solve problem 3.1.43 in the case of α > π2.

Hint Prove that there exists a triangle PB0C0 and a point A inside it, such that
PA ¼ r1, PB0 ¼ r2, PC0 ¼ r3 and ∠BAC ¼ α, ∠AB0P ¼ ∠AC0P. Then
SABC  SAB0 C0 .
3.1.65. A triangle with the area 1 is cut out of paper. Prove that one can flex it over
pffiffiffi
the segment of a line, so that area S of obtained figure would be less than 2  2.
Hint Let AB  AC  BC, if we flex triangle ABC (SABC ¼ 1) over the bisector of
angle C, then S ¼ BCþAC
BC
. On the other hand, if we flex triangle ABC over a segment
perpendicular to side BC, then one can prove that

2
S ∠C
,
3 þ tg
tg ∠B

and that the equality can hold true. Then, prove that
!
1 2 pffiffiffi
min sin ∠B
; tg ∠C
<2 2:
1þ sin ð∠Cþ∠BÞ 3þ tg ∠B

3.1.66. Given a triangle ABC. Prove that there exists a straight line l, such that if
points A1, B1, C1 are symmetric to points A, B, C with respect to line l, then the area
pffiffiffi 
of the common parts of triangles ABC and A1B1C1 is greater than 2 2  1 .
Hint See the Hint of problem 3.1.64.
3.1.67. Let diagonals AD, BE and CF of the convex hexagon intersect at one point
and A1 ¼ AD \ BF, D1 ¼ AD \ CE, B1 ¼ BE \ AC, E1 ¼ BE \ DF, C1 ¼ CF \ BD,
F1 ¼ CF \ AE. Prove that
1
SA1 B1 C1 D1 E1 F1  SABCDEF :
4

Hint Let diagonals AD, BE, and CF intersect at a point O. Prove that
AOB SBOC SAOF
SA1 OB1 ¼ ðSAOBSþS BOC ÞðSAOB þSAOF Þ
 18 ðSAOF þ SBOC Þ.

3.1.68. The medians of triangle ABC intersect the circumcircle of the triangle for
the second time at points A1, B1, C1. Prove that SA1 B1 C1  SABC .
Remark Let point M be the centroid of triangle ABC. Then
ða2 þb2 þc2 Þ
2

SMA1 B1 ¼ 48m2 m2 SABC .


a b
Chapter 4
Application of Vectors

This chapter is devoted to the application of vectors for proving geometric


(or trigonometric) inequalities and consists of only one section, that is, Section 4.1.
One of the methods of proving geometric inequalities is the use of vectors and
the application of their properties. In particular, the following properties of the
scalar product of two vectors is widely used.
(a) ~
a  ~
a¼ j~
aj2 0. Moreover, the equality holds true if and only if ~a ¼~ 0.
 ~ ~
(b) ~a  b  j~ a and ~
aj  b. Moreover, the equality holds true if and only if ~ b are
collinear vectors (parallel to one line or lying on one line).
One of the main inequalities in Section 4.1 is the inequality of problem 4.1.8a.
Very important inequality is also the inequality of problem 4.1.13.
In Section 4.1 consider different types of problems and using vectors the authors
apply some famous algebraic inequalities in order to prove geometric inequalities.
Some problems in this chapter were inspired by [1, 7, 9, 10, 13]. Nevertheless,
even for these problems the authors have mostly provided their own solutions.

4.1 Application of Vectors for Proving Geometric


and Trigonometric Inequalities

4.1.1. Let R be the radius of the circumsphere of tetrahedron SABC. Prove that
(a) SA2 + SB2 + SC2 + 4R2  AB2 + BC2 + AC2,
(b) SA2 + SB2 + SC2 + AB2 + BC2 + AC2  16R2,
(c) xyAB2 + yzBC2 + xzAC2 + xtSA2 + ytSB2 + ztSC2  (x + y + z + t)2R2, where x, y, z, t
are arbitrary numbers.

© Springer International Publishing AG 2017 139


H. Sedrakyan, N. Sedrakyan, Geometric Inequalities, Problem Books
in Mathematics, DOI 10.1007/978-3-319-55080-0_4
140 4 Application of Vectors

4.1.2. Let O be the circumcenter of regular tetrahedron SABC. Prove that


cos φ þ cos ψ  23, where φ ¼ ∠AOS, ψ ¼ ∠AOB.
4.1.3. Given four points A, B, C, and D. Prove that
(а) AB2 + BC2 + CD2 + AD2  AC2 + BD2,
(b) 4(AP2 + BQ2 + CR2 + DS2)  5(AB2 + BC2 + CD2 + AD2), where P, Q, R, S are the
midpoints of segments BC, CD, DA, AB, respectively.
4.1.4. For arbitrary points A, B, C, D, E, and F prove the following inequalities:
(a) 2(AB2 + BC2 + CD2 + DE2 + EF2 + FA2)  AD2 + BE2 + CF2,
(b) 4(A1D12 + B1E12 + C1F12)  3((AB + DE)2 + (BC + EF)2 + (CD + AF)2), where A1,
B1, C1, D1, E1, F1 are the midpoints of segments AB, BC, CD, DE, EF, and FA,
respectively.
4.1.5. Given a trapezoid ABCD (AB k CD). Prove that, if AD > BC and AC > BD,
then AB > CD.
4.1.6. Prove that for any tetrahedron A1A2A3A4 and for any point M inside of it
min cos ∠Ai MAj  13  max cos ∠Ai MAj .
i<j i<j

4.1.7. (a) Given 7 points on a unit sphere. Prove that among them one can find
pffiffiffi
2 points, such that the distance between them is less than 2.
(b) Given 5 points on a unit sphere. Prove that among them one can find 2 points,
pffiffiffi
such that the distance between them is not greater than 2.
4.1.8. For any points A, B, C, M and arbitrary numbers x, y, z prove that
(a) (x + y + z)(xMA2 + yMB2 + zMC2)  xyc2 + yza2 + xzb2, where AB ¼ c, BC ¼ a,
AC ¼ b ,
(b) aMA2 + bMB2 + cMC2  abc,
(c) MA  MBc + MB  MCa + MC  MAb  abc,
(d) (x + y + z)(xMB2MC2 + yMA2MC2 + zMA2MB2)  xyc2MC2 + yza2MA2 + xzb2MB2,
(e) (a + b + c)(aMB2MC2 + bMA2MC2 + cMA2MB2)  a2b2c2,
(f) (MA  MB + MB  MC + MC  MA)(MA + MB + MC)  a2MA + b2MB + c2MC.
4.1.9. Prove the following inequality

A1 A2 A 2 A3 An1 An A1 An
þ þ ::: þ  ,
MA1  MA2 MA2  MA3 MAn1  MAn MA1  MAn

where M, A1, . . . , An are arbitrary distinct points.


4.1.10. Let n-gon with area S be inscribed into a circle with radius R, such that on
each side of the n-gon is marked one point. Prove that, if one connects consecu-
tively these marked points then the perimeter of the obtained n-gon is less than 2S
R.

4.1.11. Let M be a given point inside of triangle ABC and P, Q, R be points on


straight lines AB, BC, AC, respectively, such that ∠PMA, ∠PMB, ∠QMB, ∠QMC,
∠RMA, ∠RMC  π2. Prove that
4.1 Application of Vectors for Proving Geometric and Trigonometric Inequalities 141

pffiffiffiffiffiffiffiffiffiffiffiffiffiffiffiffiffiffiffiffiffiffiffiffiffiffiffiffiffiffiffiffiffiffiffiffiffiffiffiffiffiffiffiffiffiffiffiffiffiffiffiffiffiffiffiffiffiffiffiffiffiffiffiffiffiffiffiffi
MA þ MB þ MC  2 p
MA  MB  cos ∠PMA  cos ∠PMBþ ffi
ffiffiffiffiffiffiffiffiffiffiffiffiffiffiffiffiffiffiffiffiffiffiffiffiffiffiffiffiffiffiffiffiffiffiffiffiffiffiffiffiffiffiffiffiffiffiffiffiffiffiffiffiffiffiffiffiffiffiffiffiffiffiffiffiffiffiffiffiffi
þ2pMB  MC  cos ∠QMB  cos ∠QMCffi þ
ffiffiffiffiffiffiffiffiffiffiffiffiffiffiffiffiffiffiffiffiffiffiffiffiffiffiffiffiffiffiffiffiffiffiffiffiffiffiffiffiffiffiffiffiffiffiffiffiffiffiffiffiffiffiffiffiffiffiffiffiffiffiffiffiffiffiffiffi
þ2 MA  MC  cos ∠RMA  cos ∠RMC:

4.1.12. Let M be a given point inside of triangle ABC. Denote the distances from the
point M to lines BC, CA, AB by da, db, dc and the distances from point M to vertices
A, B, C by Ra, Rb, Rc, respectively. Prove that
(a) Ra + Rb + Rc  2da + 2db + 2dc,
(b) RaRb + RbRc + RcRa  2Rada + 2Rbdb + 2Rcdc.
4.1.13.1 For any points A1, A2, . . ., An, M and any numbers m1, . . ., mn prove the
following inequality m1 MA21 þ ::: þ mn MA2n  m1 GA21 þ ::: þ mn GA2n , where
! !
m1 + . . . + mn  0 and m1 GA 1 þ ::: þ mn GA n ¼ ~0.
4.1.14. Let points A1, A2, . . ., An be on the same circle, m1, . . ., mn > 0 and G be
! !
such a point that m1 GA 1 þ ::: þ mn GA n ¼ ~ 0. Given that straight lines GA1, . . .,
GAn intersect the circle for the second time at points B1, . . ., Bn. Prove that
(a) m1GB1 + . . . + mnGBn  m1GA1 + . . . + mnGAn,
(b) m1 GB21 þ ::: þ mn GB2n  m1 GA21 þ ::: þ mn GA2n .
4.1.15. (а) Let M be a point inside of the convex n-gon with vertices A1, A2, . . ., An.
Perpendiculars MB1, MB2, . . ., MBn are drawn from point M to lines A1A2, A2A3,
. . ., AnA1, respectively, and C1 2 A1A2, C2 2 A2A3, . . ., Cn 2 AnA1 are arbitrary
A2 ¼ A2 A3 ¼ ::: ¼ An A1 .
mn MBn
points. Given positive numbers m1, . . ., mn, such that mA11MB 1 m2 MB2
P P
Prove that mi mj Ci C2j  mi mj Bi B2j .
i<j i<j

(b) Let M be a point inside of convex n-hedron. Perpendiculars MB1, . . ., MBn are
draw from point M to planes Π1, . ., Πn containing the faces of the polyhedrons
with areas S1, . . ., Sn, respectively, and C1 2 Π1, . . ., Cn 2 Πn are arbitrary
points. Given positive numbers m1, . . ., mn such that m1SMB
1
1
¼ ::: ¼ mnSMB
n
n
.
P P
Prove that mi mj Ci C2j  mi mj Bi B2j :
i<j i<j
4.1.16. (a) Given a triangle ABC and positive numbers x, y, z. Points A1, B1, and C1
are taken on straight lines BC, CA, and AB, respectively. Prove that
xA1 B21 þ yB1 C21 þ zA1 C21  xAB42ðþyBC
xyþyzþzxÞ
2 S2 , and that the equality holds true.
þzAC2 ABC

1
The point Gis called center of mass for the system of points A1, A2, . . ., An with masses m1,
m2, . . ., mn, and the expression IM ¼ m1 MA21 þ ::: þ mn MA2n is called the moment of inertia of the
system of points A1, . . ., An with masses m1, . . ., mn relative to the point M.
142 4 Application of Vectors

(b) Given a tetrahedron A1A2A3A4. Find on planes A2A3A4, A1A3A4, A1A2A4,


and A1A2A3 such points B1, B2, B3, and B4, respectively, that the sum B1 B22
þB1 B23 þ B1 B24 þ B2 B23 þ B2 B24 þ B3 B24 is minimal.
4.1.17. Given different points A1, A2, . . ., An and positive numbers m1, m2, . . ., mn.
Pn
Find a point M, such that the sum mi MAi is minimal.
i¼1
4.1.18. Given a convex polygon A1A2 . . . An and positive numbers m1, m2, . . ., mn.
Inscribe in a polygon A1A2 . . . An a polygon C1C2 . . . Cn (C1 2 A1A2, C2 2 A2A3,
P
n
. . .,Cn 2 AnA1, Ci 6 Aj), so that the sum mi Ci Ciþ1 is minimal, where Cn + 1  C1.
i¼1
4.1.19. Let centers O1 and O2 of the incircles of triangles A1B1C1 and A2B2C2,
respectively, do not coincide. Prove that

O1 O2 p1 þ p2
< ,
A1 A2 þ B1 B2 þ C1 C2 2maxðp1 ; p2 Þ

where p1 and p2 are perimeters of triangles A1B1C1 and A2B2C2, respectively.


4.1.20. (a) Let O be the center of a unit sphere tangent to all faces of trihedral angle
with a vertex A. It is known that the measure of all linear angles of the trihedral
pffiffiffi
angle is not less than π2 and is not greater than π  arc cos 23. Prove that OA  3.
(b) Is it possible that the distances of all vertices of the centihedron outscribed
around the unit sphere from the center of the sphere is greater than 100?
4.1.21. Let ABCDA0 B0 C0 D0 be a cube. Consider points K, L, M on edges AB, CC0 ,
0 pffiffiffi
D0 A , respectively. Prove that KL þ LM þ MK  1, 5 6AB.
4.1.22. The radius of the circumsphere of tetrahedron ABCD is equal to R, the
lengths of the segments connecting vertices A, B, C, and D with centers of the
opposite faces are equal to ma, mb, mc, and md, respectively. Prove that ma þ mb þ
mc þ md  16R3 .

4.1.23. Let ABCDbe a tetrahedron, such that AC ⊥ BC and AD ⊥ BD. Prove that the
cosine of the angle between straight lines AC and BD is less than CD
AB .

4.1.24. (a) Given points A1 and B1, A2 and B2, . . ., An and Bn on faces Γ1, Γ2, . . ., Γn
of a convex m-hedron (m  n  3), respectively. Given that for any i 2 {1, 2, ..., n}
vector mi~ ei  mi1~ ei1 is perpendicular to the plane containing face Γi, where m1,
!
A Aiþ1
m2, . . ., mn are given positive numbers and~ ei ¼ Aii Aiþ1 , i ¼ 1, 2, . . ., n, An + 1  A1,
m0 ¼ mn, ~ e0 ¼ ~en . Prove that

m1 B1 B2 þ m2 B2 B3 þ ::: þ mn1 Bn1 Bn þ mn Bn B1


 m1 A1 A2 þ m2 A2 A3 þ ::: þ mn1 An1 An þ mn An A1 :

Given a tetrahedron C1C2C3C4, such that C1C2 ¼ C2C3 ¼ C3C4 ¼ C4C1. Find on
faces C2C3C4, C1C3C4, C1C2C4, and C1C2C3 points B1, B2, B3, and B4, respec-
tively, such that sum B1B2 + B2B3 + B3B4 + B4B1 is minimal.
4.1 Application of Vectors for Proving Geometric and Trigonometric Inequalities 143

(c) Given a cube ABCDA0 B0 C0 D0 . Find on faces ABCD, AA0 BB0 , BB0 C0 C, A0 B0 C0 D0 ,
DD0 C0 C, and AA0 D0 D points B1, B2, B3,B4, B5, and B6, respectively, such that the
sum B1B2 + B2B3 + B3B4 + B4B5 + B5B6 + B6B1 is minimal.
4.1.25. Given that circumcenter O of the tetrahedron is inside of that tetrahedron.
Prove that
(a) DA2 + DB2 + DC2 + AB2 + BC2 + AC2 > 12  OA2,
(b) DA + DB + DC + AB + BC + AC > 6  OA.

Solutions

4.1.1. (a) The proof follows from problem 4.1.1c, if we take x ¼ y ¼ z ¼ 1, t ¼  1.


(b) The proof follows from problem 4.1.1c, if we take x ¼ y ¼ z ¼ t ¼ 1.
(c) Let O be the circumcenter of tetrahedron SABC. Then, we have that

xyAB2 þ yzBC2 þ xzAC2 þ xtSA2 þ ytSB2 þ ztSC2  ðx þ y þ z þ tÞ2 R2


 ! !2  ! !2  ! !2  ! !2
¼ xy OB  OA þ yz OC  OB þ xz OC  OA þ xt OA  OS
 ! !2  ! !2
þyt OB  OS þ þ zt OC  OS  ðx þ y þ z þ tÞ2 R2
 ! ! ! !2
¼  xOA þ yOB þ zOC þ t OS  0:

4.1.2. We have that


 ! ! ! !2
4R2 þ 6R2 cos φ þ 6R2 cos ψ ¼ OA þ OB þ OC þ OS  0 ðR ¼ OAÞ:

Therefore, cos φ þ cos ψ  23.


! ! !
4.1.3. Let us denote AB ¼ ~
r B , AC ¼ ~
r C , AD ¼ ~
rD .
(a) We need to prove that ~ r B 2 þ ð~ r B Þ2 þ ð~
r C ~ r C Þ2 þ ~
r D ~ r 2D  ~
r 2C þ ð~ r B Þ2 ,
r D ~
or ð~
r B þ~ r C Þ2  0.
r D ~
(b) We need to prove that
  ~rC þ~rD  2 2  2 
~
rB þ~rC 2
4 þ þ ~
~
rBr C  ~r2D þ ~ r D  ~r2B 
2 2

5 ~r B 2 þ ð~ r B Þ2 þ ð~
r C ~ r D ~ r C Þ2 þ~r 2D ,

or ð~
r B þ~ r C Þ2  0.
r D ~
144 4 Application of Vectors

! ! ! ! !
a, BC ¼ ~
4.1.4. (a) Let us denote AB ¼ ~ c, DE ¼ ~
b, CD ¼ ~ d, EF ¼ ~
e, then one has
to prove that
 2

2 ~ 2 ~
a þ b þ~
2 2 ~
c þ d þ~
2
e þ ~
2 ~
a þ b þ~ ~
c þ d þ~e 
 2  2  2
 ~ a þ~b þ~ c þ ~ c þ~
b þ~ d þ ~ c þ~d þ~ e ,

But this inequality is equivalent to the following inequality


 2  2  2
ð~
a þ~ e Þ2 þ ~
c þ~ a þ~
d þ ~ e þ ~
b þ~ a þ~
b þ~
d þ~
e  0:

! ! ! ! ! !
(b) Let AB ¼ ~ a, BC ¼ ~
b, CD ¼ ~ c, DE ¼ ~
d, EF ¼ ~
e. Note that 2A1 D1 ¼
! ! ! !
BE þ AD , consequently, 4A1 D21 ¼ BE2 þ AD2 þ 2 BE  AD . Consider paral-
 2 !  ! !2
lelogram BADM. We have that a ~
~ d ¼ EM 2 ¼ BM  BE ¼
 ! !2    2
AD  BE , hence4A1 D21 ¼ 2 AD2 þ BE2  ~ a ~
d . Then, we have that

 
4 A1 D21 þ B1 E21 þ C1 F21 ¼ 4CF2 þ 4BE2 þ 4AD2 
 2  2  2
 ~ a ~d  ~ a þ~ c þ~
b þ 2~ d þ~e  ~ b ~e ¼
 2  2  2
¼4 ~ c þ~ d þ~ e þ4 ~ a þ~ b þ~c þ4 ~ c þ~
b þ~ d 
 2  2  2
 ~ a ~d  ~ a þ~ b þ~d þ~e þ 2~c  ~ b ~e ¼
 2  2  2
¼3 ~ a ~d þ3 ~ a þ~ b þ~d þ~ c þ3 ~
e þ 2~ e  4ð~
b ~ a þ~ e Þ2 
c þ~
 2  2  2
3 ~ a ~ d þ3 ~ a þ~ b þ~d þ~e þ 2~c þ3 ~ b ~
e 
 3ðAB þ DEÞ2 þ 3ðBC þ EFÞ2 þ 3ðCD þ AFÞ2 :

! ! ! !  
a, AC ¼ ~
4.1.5. Let AD ¼ ~ b, then DC ¼ ~
b ~ a, and AB ¼ k ~ b ~a , where k > 0.
 2
One has to prove that k > 1. We have that ~ a2 > ðk  1Þ~ b  k~
a and
0
 2 k1 ðk  1Þ~
b  k~
2
~ B aÞ
b2 > k~ b  ðk þ 1Þ~
a . Therefore, ¼ ~
a 2
 @  2 .
k þ 1‘ ~ ~
b  kb  ðk þ 1Þ~
2 a >0
Hence, k > 1.
4.1 Application of Vectors for Proving Geometric and Trigonometric Inequalities 145

!
ei ¼ MA
4.1.6. Let ~ MAi , i ¼ 1, 2, 3, 4, and ~
i e ¼~e1 þ~ e2 þ~e3 þ~ e4 . Let us draw through
point M a plane Π perpendicular to vector ~ e. It is clear that, if all points Ai were in
the same half-space with boundary Π, then point M would have been inside the
given tetrahedron. This means that there exists a vector~ ek , such that~e ~
ek  0. Then
we have that ð~ e ~ ek Þ~
ek  1. Hence, we deduce that min cos ∠Ai MAj 
i<j

3 ð~ ek Þ~
e ~  13.
1
ek
Similarly, we obtain that there exists a vector ~ em , such that ~
e ~em  0. In that
case, we have that ð~
e ~em Þ ~
em  1. Thus, it follows that
max cos ∠Ai MAj  13 ð~
e ~em Þ ~em  13. This inequality can be proven as follows:
i<j
P
max cos∠Ai MAj  16 cos∠Ai MAj ¼ 12 1
ð~
e2 ~e2 1 ~ e2 2 ~
e2 3 ~e2 4 Þ  13, beca-
i<j i<j
use ~
e2  0 and ~
e2 1 ¼ ~
e2 2 ¼ ~
e2 3 ¼ ~
e2 4 ¼ 1.
4.1.7. (а) Let points A1, A2, . . ., A7 are on the unit sphere with center O. Let us
pffiffiffi ! 2  2
assume that Ai Aj  2 ði 6¼ jÞ, then Ai Aj 2 ¼ Ai Aj ¼ ~ aj  ~
ai  2, where
!
OA i ¼ ~ai , i ¼ 1, . . ., 7. This means that for i 6¼ j we have that

~
ai  ~
aj  0: ð4:1Þ

Let ~e1 ¼ ~a1 ,~


e2 ,~e3 be mutually perpendicular unit vectors and
~
ai ¼ xi~ e1 þ yi~e2 þ zi~ e3i , i ¼ 1, . . ., 7. As x1 ¼ 1, y1 ¼ z1 ¼ 0, and ~ ai  ~
aj  0
(i 6¼ j), then x2, . . ., x7  0.
Let us consider vectors on a plane: ~ b2 ðy2 ; z2 Þ, :::, ~
b7 ðy7 ; z7 Þ. Note that there are
non-zero vectors among them. Indeed, if, for example, ~ b2 ¼ ~ b3 ¼ ~0, then some two
of the vectors ~ a3 coincide. This leads to a contradiction with (4.1). Let ~
a2 ,~
a1 ,~ b3 , ::
~
:, b7 be non-zero vectors. Then, some two of them form an acute angle, for example,
b3 and ~
~ b4 . Then, we have that ~ a4 ¼ x3 x4 þ y3 y4 þ z3 z4 ¼ x3 x4 þ ~
a3~ b3~b4 > 0,
~ ~
because x3x4  0 and b3 b4 > 0. This leads to a contradiction with (4.1).
Remark Vectors ~ a1 ð1; 0; 0Þ, ~
a2 ð1; 0; 0Þ, ~
a3 ð0; 1; 0Þ, ~
a4 ð0; 1; 0Þ, ~
a5 ð0; 0; 1Þ, ~
a6
ð0; 0; 1Þ satisfy condition (4.1).
 2
(b) If ~ aj  ~
ai > 2, (i 6¼ j), i, j, 2 {1, 2, 3, 4, 5}, then

~
ai  ~
aj < 0, ði 6¼ jÞ: ð4:2Þ

Then, among vectors ~ b2 , ~


b3 , ~
b4 , ~
b5 there is non-zero vector
 !  (see the proof of
4.1.7a), because otherwise, if ~ b2 ¼ ~ 0, then ~a2 ¼ 1; 0; 0 and 0 ¼ ~ a3  ~
a1 þ
a3~
~ a2 < 0.
This leads to a contradiction.
146 4 Application of Vectors

Therefore, among vectors ~ b2 , ~


b3 , ~
b4 , ~
b5 some two form an angle not greater than
 ~ ~
90 . Let b2 b3  0, then ~ a3 ¼ x2 x3 þ b2~
a2~ ~ b3 > 0, because x2, x3 < 0 and ~
b2~
b3  0.
This leads to a contradiction.
 pffiffi   pffiffi pffiffi 
Remark Vectors ~
a1 ð0; 0; 1Þ, ~
a2 2 3 2 ; 0; 13 , ~
a3  32;  36 ; 13 , and
 pffiffi pffiffi 
~
a4  32; 36 ; 13 satisfy condition (4.2).
4.1.8. (а) Note that
 
ðx þ y þ zÞ xMA2 þ yMB2 þ zMC2  xyc2  yza2  xzb2 ¼
   ! !2
¼ ðx þ y þ zÞ xMA2 þ yMB2 þ zMC2  xy MB  MA 
 ! !2  ! !2  ! ! !2
yz MC  MB  xz MC  MA ¼ xMA þ yMB þ zMC  0:

(b) Using problem 4.1.8a for x ¼ a, y ¼ b, z ¼ c, we deduce that aMA2 + bMB2 +


cMC2  abc.
(c) For four points M, A, B, C we call their H-image the points M1, A1, B1, C1, where
M1A1 ¼ MB  MC, M1B1 ¼ MA  MC, M1C1 ¼ MA  MB, A1C1 ¼ MB  AC,
B1C1 ¼ MA  BC, A1B1 ¼ MC  AB (see Figure 4.1).
Let us rewrite inequality 4.1.8b for points M1, A1, B1, C1, this means that
B1 C1  M1 A21 þ A1 C1  M1 B21 þ B1 A1  M1 C21  B1 C1  A1 C1  A1 B1 , or

MB2  MC2  MA  BC þ MA2  MC2  MB  AC þ MA2  MB2  MC  AB 


 MA  MB  MC  AB  BC  AC:

Therefore, MB  MCa + MA  MCb + MA  MBc  abc.


If MA  MB  MC ¼ 0, then the proof of the inequality is straightforward.
(d) Let M1, A1, B1, C1 be the H-image of points M, A, B, C. We have that ðx þ y þ zÞ
 
xM1 A21 þ yM1 B21 þ zM1 C21  xyB1 A21 þ yzC1 B21 þ xzA1 C21 (see problem
4.1.8a). Therefore, (x + y + z)(xMB2MC2 + yMA2MC2 + zMA2MB2)  xyc2MC2
+ yza2MA2 + xzb2MB2.

A1
B
MB×MC

MB×AC
H- image MC×AB
M C M1 C1
MA×MB
MA×BC
MA×MC
A

B1

Figure 4.1
4.1 Application of Vectors for Proving Geometric and Trigonometric Inequalities 147

(e) Using problems 4.1.8e and 4.1.8b, we obtain that for x ¼ a, y ¼ b, z ¼ c


 
ða þ b þ cÞ aMB2 MC2 þ bMA2 MC2 þ cMA2 MB2  abc2 MC2 þ bca2 MA2
þacb2 MB2  a2 b2 c2 :

Thus, (a + b + c)(aMB2MC2 + bMA2MC2 + cMA2MB2)  a2b2c2.


(f) If MA  MB  MC ¼ 0, then the proof of the inequality is straightforward. But, if
MA  MB  MC 6¼ 0, then we obtain the required inequality from inequality
4.1.8a by taking x ¼ MA
1
, y ¼ MB
1
, z ¼ MC
1
.

! !
4.1.9. Let us denote MA i ¼ ~ r i and MA MAi 2
i
¼!ρ i , i ¼ 1, . . ., n. Note that
rffiffiffiffiffiffiffiffiffiffiffiffiffi qffiffiffiffiffiffiffiffiffiffiffiffiffiffiffiffiffiffiffiffiffiffiffiffi
! ! 2 ! ! 
rj j ¼ ð~ri2~r2j Þ ¼
j~ri~
2

ρ i  ρ j ¼  ρ i  ρ j . We have to prove
Ai Aj
MAi MAj ¼ ~ 
ri ~ rj  ~r i ~
r j
       
that ! ρ1! ρ 2 þ ! ρ2! ρ 3  þ ::: þ !ρ n1  ! ρ n   !ρ1! ρ n . This can be
obtained, if we use the following inequality j~ a1 j þ j~
a2 j þ ::: þ j~
an1 j 
a1 þ ~
j~ a2 þ ::: þ ~ an1 j (see Section 1.2) for vectors ~ ! !
ai ¼ ρ iþ1  ρ i , i ¼
1, :::, n  1.
4.1.10. Let us draw tangents to the circle at the vertices of the inscribed n-gon
A1 . . . An, and choose unit vectors ~
e1 , :::,~
en on these tangents (Figure 4.2).
Let B1 2 A1An, B2 2 A1A2, . . ., Bn 2 An  1An be the marked points, then

B1 B2 þ B2 B3 þ ::: þ Bn1 Bn þ Bn B1 
! ! ! !
 B1 B2~
e1 þ B2 B3~ e2 þ ::: þ Bn1 Bn~
en1 þ Bn B1~ en ¼
 ! !   ! !   ! ! 
¼ B1 A1 þ A 1 B2 ~ e 1 þ B2 A2 þ A 2 B3 ~e2 þ ::: þ Bn An þ An B1 ~
en ¼
! ! ! 2S
¼ A1 A2~
e1 þ A2 A3~
e2 þ ::: þ An A1~
en ¼ ,
R
! ! ! !
because B2 A2~
e2 ¼ B2 A2~
e1 , :::, An B1~
en ¼ An B1~
e1 .

Figure 4.2 
A2 e2

e1 A3 
A1 B2 B3 e3

 B1
en
An
148 4 Application of Vectors

4.1.11. We have that cos ∠PMA cos ∠BMP  cos ð∠PMAþ∠PMB 2


Þþ1
¼
2 ∠PMAþ∠PMB 2 ∠AMB
cos 2  cos 2 , consequently, it is sufficient to prove that

MA þ MB þ MC 
pffiffiffiffiffiffiffiffiffiffiffiffiffiffiffiffiffiffi ∠AMB pffiffiffiffiffiffiffiffiffiffiffiffiffiffiffiffiffiffi ∠BMC pffiffiffiffiffiffiffiffiffiffiffiffiffiffiffiffiffiffi ∠AMC
 2 MA  MB cos þ 2 MB  MC cos þ 2 MA  MC cos :
2 2 2
ð4:3Þ
pffiffiffiffiffiffiffi pffiffiffiffiffiffiffi pffiffiffiffiffiffiffiffi
Let A1, B1, C1 be such points that MA1 ¼
MA, MB1 ¼ MB, MC1 ¼ MC and
∠A1 MB1 ¼ ∠AMB , ∠B1 MC1 ¼ ∠BMC
2 , ∠A1 MC1 ¼ π 
∠AMC. Then, from inequality
 ! ! 2 ! 2 2

MA1  MB1 þ MC1  0 we obtain inequality (4.3).

Remark Inequality (4.3) coincides with the inequality of problem 5.1.22a by


replacing MA ¼ x2, MB ¼ y2, MC ¼ z2, ∠AMB
2 ¼ α, ∠BMC
2 ¼ β, ∠AMC
2 ¼ γ.
4.1.12. (a) Let MP ⊥ AB, P 2 AB, MQ ⊥ BC, Q 2 BC, MR ⊥ AC, R 2 AC. Then
MA cos ∠PMA ¼ MB cos ∠PMB ¼ dc, MB cos ∠QMB ¼ MC cos ∠QMC ¼ da,
and MA cos ∠RMA ¼ MC cos ∠RMC ¼ db. Consequently, using problem 4.1.11,
we deduce that Ra + Rb + Rc  2da + 2db + 2dc.
(b) The required inequality for the H-image of points M, A, B, C can be obtained
from 4.1.12a (see the proof of problem 4.1.8c)).
4.1.13. We have that
Xn Xn  ! ! 2
IM ¼ mi MA2i ¼ mi MG þ GAi
i¼1 ! i¼1
X n
!2 !X n
! Xn
¼ mi MG þ 2MG mi  GAi þ mi GA2i ¼
i¼1 i¼1 i¼1
¼ m  MG2 þ I G ,

P
n
where mi ¼ m. Therefore, IM  IG.
i¼1

4.1.14. (a) As mR2 ¼ I0 ¼ mOG2 + IG (see the proof of problem 4.1.13), then
m(R2  OG2) ¼ IG, where O is the center and R is the radius of the given circle.
Hence, point Gis inside of the given circle, thus GAi  GBi ¼ R2  OG2.
4.1 Application of Vectors for Proving Geometric and Trigonometric Inequalities 149

We have that
! ! !2
X
n X
n n pffiffiffiffiffiffiffiffiffiffiffiffiffi2 X
X n pffiffiffiffiffiffiffiffiffiffiffiffiffi2 X
n pffiffiffiffiffiffiffiffiffiffiffiffiffiffiffiffiffiffiffi
mi GBi mi GAi ¼ mi GBi mi GAi  mi GAi  GBi ¼
i¼1 i¼1 i¼1 i¼1 i¼1
! ! !2
  n pffiffiffiffiffiffiffi
X X X
n
pffiffiffiffiffi 2
n
¼ m2 R2  OG2 ¼ mI G ¼ mi 2 ð mi GAi Þ  mi GAi
i¼1 i¼1 i¼1

P
n P
n
(see the proof of problem 4.1.15a). Therefore, mi GBi  mi GAi .
i¼1 i¼1

Remark Inequality holds true, if instead of a circle one considers a sphere.


(b) We have that
! ! ! ! !2
X
n X
n X
n
pffiffiffiffiffi 2
X
n
pffiffiffiffiffi 2
X
n
mi GB2i mi GA2i ¼ ð mi GBi Þ ð mi GAi Þ  mi GAi  GBi ¼
i¼1 i¼1 i¼1 i¼1 i¼1
!2
X
n     2
¼ mi R2  OG2 ¼ m R2  OG2 ¼ I 2G
i¼1
 2
¼ m1 GA21 þ ::: þ mn GA2n :

P
n P
n
Therefore, mi GB2i  mi GA2i (see the proof of problem 4.1.14 a)
i¼1 i¼1

P
n P
n
Remark If 0 < p  2, then mi GBip  mi GAip .
i¼1 i¼1
Indeed, we have that
! ! !2
X
n X
n X
n pffiffiffiffiffiffiffiffiffiffiffiffiffiffiffiffiffiffip  pffiffiffiffiffiffiffiffiffiffiffiffiffiffiffiffiffiffiffip 2
mi GBip mi GAip  mi GBi  GAi ¼ m R2  OG2 ¼
i¼1 i¼1 i¼1
 p
¼ m2 mm1 GA21 þ ::: þ mmn GA2n :

p
Because f ðxÞ ¼ x2 at 0 < p  2 is concave function on the interval [0 ; + 1 ) and
P
n
mi
m ¼ 1, then by using the Jensen’s inequality we deduce that
i¼1
m mn 2  m1  2  mn  2 
1
f GA21 þ ::: þ
GAn  f GA1 þ ::: þ f GAn , or
m m m m
m mn p2 m mn
1 1
GA21 þ ::: þ GA2n  GA1p þ ::: þ GAnp , consequently,
m m m m
150 4 Application of Vectors

! !
X
n X
n m mn p
1
mi GBip mi GAip  m2 GA21 þ ::: þ GA2n
i¼1 i¼1
m m
 2
 m1 GA1p þ ::: þ mn GAnp :

P
n P
n
Hence, it follows that mi GBip  mi GAip .
i¼1 i¼1
! !
4.1.15. (а) Note that mi MB 1 þ ::: þ mn MB n ¼ ~0. Indeed, let m1MB1 ¼ kA1A2, . . .,
! !
mnMBn ¼ kAnA1. Under rotations by 90 or –90 vectors mi MB 1 , :::, mn MB n trans-
! ! ! !
form into vectors kA1 A2 , :::, kAn A1 , and because kA1 A2 þ ::: þ kAn A1 ¼ ~
0, then
! ! ~
m1 MB 1 þ ::: þ mn MB n ¼ 0.
! !
Let C0 be such a point that m1 C0 C þ ::: þ mn C0 C ¼ ~
1 0, then n

X X  ! ! 2 X
mi mj Ci C2j ¼ C0 Cj  C0 Ci mi mj ¼ mi mj C0 C2j
i<j i<j i<j
X X ! !
þ mi mj C0 C2i  2mi mj C0 Ci  C0 Cj ¼
i<j i<j

!2
X
n X
n
! X
n
¼ ðm1 þ ::: þ mn Þ mi C0 C2i  mi C0 Ci ¼ ðm1 þ ::: þ mn Þ mi C0 C2i :
i¼1 i¼1 i¼1

P
n P
n
Similarly, we obtain that mi mj Bi B2j ¼ ðm1 þ ::: þ mn Þ
mi MB2i .
i<j i¼1
 
It is known that for any a1, . . ., an and b1, . . ., bn > 0 inequality a21 þ ::: þ a2n
 2 
b1 þ ::: þ b2n  ða1 b1 þ ::: þ an bn Þ2 holds true, the equality being reached only if
 2 
b1 ¼ ::: ¼ bn . Indeed, it is not difficult to check that a1 þ ::: þ a2n
a1 an
 2  P 2
b1 þ ::: þ b2n  ða1 b1 þ ::: þ an bn Þ2 ¼ ai bj  aj bi  0, where the equality
i<j
holds true, only if aibj  ajbi ¼ 0, ( i < j), or equivalently, if ab11 ¼ ::: ¼ abnn . Then, we
have that
4.1 Application of Vectors for Proving Geometric and Trigonometric Inequalities 151

X n

n
Ai A 2 X X Ai Aiþ1 2 X pffiffiffiffiffi 2
iþ1
mi mj Ci C2j ¼
pffiffiffiffiffi  ð mi Co Ci Þ  ðm1 þ ::: þ mn Þ 
i¼1
m i i<j i¼1
m i i<j
!2 !2
X
n X n
 ðm1 þ ::: þ mn Þ Ai Aiþ1 Co Ci  4ðm1 þ ::: þ mn Þ SCo Ai Aiþ1 
i¼1 i¼1
! 2
X
n
 4ðm1 þ ::: þ mn ÞS2A1 A2 :::An ¼ 4ðm1 þ ::: þ mn Þ SMAi Aiþ1 ¼
i¼1
!2
Xn
Ai Aiþ1 pffiffiffiffiffi
¼ ðm1 þ ::: þ mn Þ pffiffiffiffiffi mi MBi
i¼1
mi
! !2
X A i A2
n Xn
¼ ðm1 þ ::: þ mn Þ iþ1
mi MBi ¼
i¼1
mi i¼1
X n
Ai A2iþ1 X
¼ mi mj Bi B2j ,
i¼1
m i i<j

P P
where Cn + 1  C1. Therefore, mi mj Ci C2j  mi mj Bi B2j .
i<j i<j

! !
(b) It is sufficient to prove that mi MB 1 þ ::: þ mn MB n ¼ ~ 0 and to replace in the
proof of (a) term Ai A2iþ1 by S2i and areas by volumes.
Pn ! ! n !  ! 
P
Let N be any point inside, then V ¼ 13 NB i  MB
MBi Si ¼ 3
i k
NB i  mi MB i ,
i¼1 i¼1
where mi SMB
i
i
¼ 1k, i ¼ 1, . . ., n.
! ! !
By making (in the obtained equality) the replacement NB i ¼ NM þ MB i , we
! Pn !
deduce that the expression NM mi MB i is constant for any N. Hence,
i¼1
P
n !
mi MB i ¼ ~
0.
i¼1

4.1.16. (a) Let m1, m2, m3 be positive numbers, such that x ¼ m1m2, y ¼ m2m3,
z ¼ m1m3. Then,

xA1 B21 þ yB1 C21 þ zC1 A21 ¼ m1 m2 A1 B21 þ m2 m3 B1 C21 þ m1 m3 C1 A21 


4ðm1 þ m2 þ m3 ÞS2ABC 4ðxy þ yz þ zxÞS2ABC
 ¼
BC2 AC2 AB2 xAB2 þ yBC2 þ zAC2
þ þ
m1 m2 m3

(see the proof of problem 4.1.15a). To prove that the equality holds true, it is
sufficient to prove that inside of triangle ABC there exists point M, such that
m1 MA1 m2 MB1 m3 MC1
BC ¼ AC ¼ AB , where MA1 ⊥ BC, MB1 ⊥ AC, MC1 ⊥ AB, and A1 2 BC,
B1 2 AC, C1 2 AB.
152 4 Application of Vectors

Take a point M inside of triangle ABC, such that ctg ∠MAC ¼


ctg ∠A þ sin1∠A mm32AC
AB
and ctg ∠MCA ¼ ctg ∠C þ sin1∠C m
m2 BC
1 AC
. Then

MC1 MA  sin ð∠A  ∠MACÞ m2  AB


¼ ¼ and
MB1 MA  sin ∠MAC m3  AC
MA1 MC  sin ð∠C  ∠ACMÞ m2  BC
¼ ¼ :
MB1 MC  sin ∠ACM m1  AC
m1  MA1 m2  MB1 m3  MC1
Therefore, ¼ ¼ :
BC AC AB

(b) Let us choose points C1, C2, and C3 on edges A1A4, A2A4, and A3A4, respectively,
S2 S2 S2
so that A1 C 1
C1 A 4 ¼ SA2 1 A2 A3 , A 2 C2
C2 A 4 ¼ SA2 1 A2 A3 , A3 C 3
C3 A 4 ¼ SA2 1 A2 A3 . Then, it is not difficult to
A2 A3 A4 A1 A 3 A4 A1 A2 A4

construct common point M of planes A1C2A3, A2C1A3, and A1C3A2.


The point M is inside of tetrahedron A1A2A3A4. We need to prove that
MB1
S A2 A A
¼ SAMB 2
A A
¼ SAMB 3
A A
¼ SAMB 4
A A
, where B1 2 (A2A3A4), B2 2 (A1A3A4),
3 4 1 3 4 1 2 4 1 2 3
B3 2 (A1A2A4), B4 2 (A1A2A3) and MB1 ⊥ (A2A3A4), MB2 ⊥ (A1A3A4),
MB3 ⊥ (A1A2A4) и MB4 ⊥ (A1A2A3).
S2A V V S
Indeed, we have that S2A
1 A2 A3
¼ AC11CA14 ¼ V AA1 AA2 AA3 CC1 ¼ V AA1 AA2 AA3 MM ¼ MB 4 A1 A2 A3
MB1 SA A A , conse-
2 A3 A4 2 3 4 1 2 3 4 2 3 4

quently, SAMB 4
A A
¼ SAMB 1
A A
. Similarly, we obtain that SAMB 4
A A
¼ SAMB 2
A A
and SAMB 4
A A
¼ SAMB 3
A A
.
1 2 3 2 3 4 1 2 3 1 3 4 1 2 3 1 2 4
Now, by using problem 4.1.15b for numbers m1 ¼ m2 ¼ m3 ¼ m4 ¼ 1, we obtain
that the given sum is minimal at those points.
4.1.17. Lemma 1 Let M0 be such a point, that for any point M the following
P
n P
n
inequality holds true: mi MAi  mi M0 Ai . If
i¼1 i¼1

Pn !
M A
(a) M0 6 Ai , i ¼ 1, 2, . . ., n, then mi M00 Ai i ¼ ~
0, (4.4)
 i¼1
 P !
 n M A
(b) M0  Ak, then  mi M00 Ai i j  mk : (4.5)
i¼1, i6¼k

The Proof
P
n !
M A
(a) Let ei ¼ ~
mi~ S 6¼ ~
0, where ~
ei ¼ M00 Ai i . Take point Mt so that the equality
i¼1

!
M0 Mt ¼ t  ~
S 6¼ ~
0 holds true, where t > 0.
!
M t Ai
Let us denote ~
ei ðtÞ ¼ M t Ai , then we have that
4.1 Application of Vectors for Proving Geometric and Trigonometric Inequalities 153

!
X
n X
n
! ! X

n X
n
!
mi M0 Ai  mi M0 Ai ~
ei ðtÞ ¼ M0 Mt mi ~
ei ðtÞ þ mi Mt Ai ~
ei ðt Þ
i¼1 i¼1 i¼1 i¼1
X
n
¼ f ðt Þ þ mi Mt Ai : ð4:6Þ
i¼1

Now, we need to prove that there exists t0 > 0, such that f(t0) > 0, then from (4.6)
P
n Pn
it follows that mi M0 Ai > mi Mt0 Ai . This leads to a contradiction the condition
i¼1 i¼1
of the lemma. Hence ~ S ¼~ 0.
qffiffiffiffiffiffiffiffiffiffiffiffiffiffiffiffiffiffiffiffiffiffiffiffi
Because j~ ei j ¼ ð~
ei ðtÞ ~ ei ðtÞ ~ ei Þ2 ¼ 2 sin ∠Mt2Ai M0 < ∠Mt Ai M0 , then there
j~Sj
exists t0 > 0, such that j~ei ðt0 Þ ~ ei j < 2nmi , for all i ¼ 1, 2, . . ., n. Then,

! X ! X X
n
 
n n

!
f ðt0 Þ ¼ M0 Mt0 mi~
ei ðt0 Þ ¼ M0 Mt0 mi~
ei þ mi M0 Mt0 ð~
ei ðt0 Þ ~
ei Þ 
i¼1 i¼1 i¼1
X
n  
 
! 
 t0~
S2  mi M0 Mt0 j~
ei ðt0 Þ ~
ei j
i¼1
 
 X n ~
S t0 2
~ ~
> t0 S  t0 S
2  mi ¼ ~S > 0:
i¼1
2nm i 2

 
   P n 
 
!
~
(b) Let S ¼  ei  > mk . Take point Mt so that the equality Ak Mt ¼ t~
mi~ S
i¼1, i6¼k 
holds true, where t > 0. Then, we have that

X
n X
n
!
m i Ak Ai  mi Ak Ai~
ei ðtÞ ¼
i¼1, i6¼k i¼1, i6¼k
! X X X
n n n
 
!
¼ Ak M t mi~ei þ mi Ak Mt ð~
ei ðtÞ ~
ei Þ þ m i M t Ai :
i¼1, i6¼k i¼1, i6¼k i¼1, i6¼k

 
Let us denote ~ S ¼ mk þ q and choose t0, such that j~ ei j < ðn1q Þmi , i ¼ 1,
ei ðt0 Þ ~
2, . . ., n, i 6¼ k. Then,

Xn   X n
q X
n
S2  t0 ~
mi Ak Ai > t0~ S mi þ mi Mt0 Ai
i¼1 i¼1, i6¼k
ðn  1Þmi i¼1, i6¼k
Xn Xn
¼ mk Mt0 Ak þ mi Mt0 Ai ¼ m i M t 0 Ai :
i¼1, i6¼k i¼1
154 4 Application of Vectors

The obtained
 inequality
 contradicts to the conditions of the lemma. Conse-
 P 
 n

quently,  m~e   mk . This ends the proof of the lemma.
i¼1, i6¼k i i 

Remark By using the Weierstrass theorem one can prove that there exists a point
P
n
M, such that the sum mi MAi is minimal (see [2], problem 19).
i¼1

Lemma 2 If for point M0 the condition (4.4) or (4.5) is fulfilled and points A1, . . .,
An are not on the same line, then for any point M, different from M0, the inequality
Pn P
n
mi  MAi > mi  M0 Ai holds true.
i¼1 i¼1

Proof Indeed, if condition (4.4) is satisfied, then at M 6 M0 , we have that

X
n X
n
! ! X

n X ! n X n
mi  MAi > mi  MAi~ei ¼ MM0 mi~
ei þ mi M0 Ai~
ei ¼ mi  M0 Ai ,
i¼1 i¼1 i¼1 i¼1 i¼1

and if condition (4.5) is satisfied, then we have that

X
n X
n
! ! X
n Xn  ! ! 
mi  MAi > mi  MAi~ei  MAk mi~
ei ¼ mi MAi  MAk ~
ei
i¼1 i¼1, i6¼k i¼1, i6¼k i¼1, i6¼k

X n
¼ mi  Ak Ai :
i¼1

Remark 1. From lemmas 1 and 2 it follows that, if points A1, A2, . . ., An are not on
the same line, then point M0 is unique.
2. If in a statement of lemma 2 the condition that points A1, A2, . . ., An are not on
P
n Pn
the same line was not given, then mi  MAi  mi  M0 Ai
i¼1 i¼1

4.1.18. The proof of the problem follows from the following two lemmas.
Lemma 1 If B1B2 . . . Bn is a polygon inscribed into a polygon A1A2 . . . An,
( Bi 6 Aj and Bi 2 AiAi + 1, An + 1  A1), so that for any polygon C1C2 . . . Cn
(Ci 6 Aj , Ci 2 AiAi + 1) and any positive numbers mi the inequality
Pn Pn
mi Ci Ciþ1  mi Bi Biþ1 , where Cn + 1  C1, Bn + 1  B1, holds true, then the
i¼1 i¼1
following conditions are satisfied:
!
Ai Aiþ1 ðmi1~
ei1  mi~
ei Þ ¼ 0, i ¼ 1, :::, n, ð4:7Þ

!
Bi Biþ1
where ~
ei ¼ Bi Biþ1 , m 0 ¼ m n, ~
eo ¼ ~
en .
4.1 Application of Vectors for Proving Geometric and Trigonometric Inequalities 155

Proof Suppose that, at i ¼ k condition (4.7) does not hold true, that means that
! !
! 0 0 Bk1 B0k 0 B0k Bkþ1
~ ~
0 6¼ Ak Akþ1 ðmk1 ek1  mk ek Þ. Let Bk 2 Ak Akþ1 ,~
ek1 ¼ Bk1 B0 ,~
ek ¼ B0 Bkþ1 .
k k
If points B0k and Bk are different, then we have that

! 0 þ m B !
mk1 Bk1 Bk þ mk Bk Bkþ1 > mk1 Bk1 Bk~e k1 k k Bkþ1~ e0k ¼
! 0 ! 0 ! 0 !
¼ mk1 Bk1 B0k~
ek1 þ mk1 B0k Bk~
ek1 þ mk Bk B0k~ek þ mk B0k Bkþ1~e0k ¼
!  
¼ mk1 Bk1 B0k þ mk B0k Bkþ1 þ B0k Bk mk1~e0k1  mk~e0k : ð4:8Þ

! ! !   !  
Let B0k Bk ¼ λAk Akþ1 , B0k Bk mk1~ e0k1  mk~e0k ¼ λAk Akþ1 mk1~ e0k1  mk~
e0k .
!
Let us choose λ, so that λ  Ak Akþ1 ðmk1~ ek1  mk~ek Þ > 0. We need to prove that
!  
0 0
one can choose Bk , so that Bk 2 Ak Akþ1 and λ  Ak Akþ1 mk1~ e0k1  mk~e0k > 0.
!
Indeed, let Ak Akþ1 ðmk1~ ek1  mk~ ek Þ ¼ a 6¼ 0. Let us choose B0k , so that

 0  a
~
e ~ ei  < , at i ¼ k  1 ; k (see the proof of lemma 1 of problem 4.1.17),
i 4mi Ak Akþ1
then
 
 !   ! 
e0k1  mk~
λ  Ak Akþ1 mk1~ e0k  λ  Ak Akþ1 ðmk1~ek1  mk~ ek Þ ¼
 
 !   0   0 
¼ λ  Ak Akþ1 mk1 ~ ek1 ~ek1  mk ~ ek ~ek  
  
  !   0   0  aλ
  
 λ Ak Akþ1  mk1 ~ ek1 ~ ek1  mk ~ ek ~ 
ek  :
2

Therefore,
  
!   !  a  λ 
0 0
λ  Ak Akþ1 mk1~ek1  mk~
ek  λ  Ak Akþ1 ðmk1~
ek1  mk~
ek Þ 
   2
aλ
¼ > 0:
2

From the last expression and (4.8) we deduce that

mk1 Bk1 Bk þ mk Bk Bkþ1 > mk1 Bk1 B0k þ mk B0k Bkþ1 :

We have obtained that for polygon B1 B2 :::Bk1 B0k Bkþ1 :::Bn that the considered
sum is less than for polygon B1B2 . . . Bk . . . Bn. This leads to a contradiction.
Remark If m1 ¼ m2 ¼ . . . ¼ mn, then it is possible to prove lemma 1 more simply
than in the general case. On the other hand, Bi  Aj is impossible.
Lemma 2 If for the inscribed polygon B1B2 . . . Bn the condition (4.7) is satisfied,
P
n !
then for any inscribed polygon C1C2 . . . Cn the sum mi Ci Ciþ1~
ei is constant and
i¼1
156 4 Application of Vectors

P
n P
n
mi Ci Ciþ1  mi Bi Biþ1 .
i¼1 i¼1

Proof We have that

X
n
! X
n  ! ! 
mi Ci Ciþ1~
ei ¼ mi Ci Aiþ1 þ Aiþ1 Ciþ1 ~
ei
i¼1 i¼1
X
n
! X
n
!
¼ mi Ci Aiþ1~
ei þ mi Aiþ1 Ciþ1~
ei ¼
i¼1 i¼1
X
n
! X
n
!
¼ mi Ci Aiþ1~
ei þ mi1 Ai Ci~
ei1
i¼1 i¼1
X
n  ! !  X
n
!
¼ mi1 Ai Aiþ1  Ci Aiþ1 ~
ei1 þ mi Ci Aiþ1~
ei ¼
i¼1 i¼1
X
n
! X
n
!
¼ mi1 Ai Aiþ1~
ei1  Ci Aiþ1 ðmi1~
ei1  mi~
ei Þ
i¼1 i¼1
X
n
!
¼ mi1 Ai Aiþ1~
ei1 ,
i¼1

note that the last one is a constant.


Pn Pn ! P
n ! P
n
Thus, mi Ci Ciþ1  mi Ci Ciþ1~
ei ¼ mi Bi Biþ1~
ei ¼ mi Bi Biþ1 .
i¼1 i¼1 i¼1 i¼1

Remark If n is odd, then there exists no more than one polygon B1B2 . . . Bn, while
for even n there can exist an infinite number of polygons B1B2 . . . Bn.
4.1.19. Let O be the incenter of triangle ABC. We need the following property of
point O:
! ! !
BC  OA þ AC  OB þ AB  OC ¼ ~
0: ð4:9Þ

Let us consider points A1, B1, C1 (Figure 4.3). Since, the circle with diameter OA
passes through points B1 and C1, then B1 C1 ¼ OA sin ∠A ¼ BCOA 2R , where R is the
circumradius of triangle ABC. From the above said conditions OA ⊥ B1C1,
OB ⊥ C1A1, OC ⊥ A1B1, it follows that, under the rotation by 90 vectors
! ! ! ! ! !
2RB1 C1 , 2RC1 A1 , 2RA1 B1 transform into vectors BC  OA , AC  OB , AB  OC .
Thus, we have that condition (4.9) is satisfied. According to that condition
4.1 Application of Vectors for Proving Geometric and Trigonometric Inequalities 157

Figure 4.3 B B1
A1
C1

A B1 C


! 
! ! 
!
B1 C1  O1 A1 þ A1 C1  O1 B1 þ A1 B1  O1 C1 ¼ B2 C2  O2 A2
 ! !
þA2 C2  O2 B2 þ A2 B2  O2 C2 :
ð4:10Þ


! !  ! !  ! !
Taking into account that O2 A2 ¼ O2 O1 þ O1 A1 þ A1 A2 , O2 B2 ¼ O2 O1 þ

! ! ! ! !  !
O1 B1 þ B1 B2 , O2 C2 ¼ O2 O1 þ O1 C1 þ C1 C2 , from (4.10) it follows that
! 
! 
!
p2  O1 O2 ¼ O1 A1 ðB2 C2  B1 C1 Þ þ O1 B1 ðA2 C2  A1 C1 Þ
! ! ! ð4:11Þ
þO1 C1 ðA2 B2  A1 B1 Þ þ þB2 C2  A1 A2 þ A2 C2 B1 B2
!
þA2 B2  C1 C2 :

Therefore,
     
p2  O1 O2  O1 A1 B2 C2  B1 C1  þ O1 B1 A2 C2  A1 C1  þ O1 C1 A2 B2  A1 B1 þ
þB2 C2  A1 A2 þ A2 C2  B1 B2 þ A2 B2  C1 C2 :
ð4:12Þ

Let us further note that |B2C2  B1C1|  B1B2 + C1C2, |A2C2  A1C1| 
A1A2 + C1C2, |A2B2  A1B1|  A1A2 + B1B2, then from (4.12) it follows that

p2  O1 O2  ðO1 B1 þ B2 C2 þ O1 C1 ÞA1 A2 þ ðO1 A1 þ A2 C2 þ O1 C1 ÞB1 B2


þ ðO1 A1 þ A2 B2 þ O1 B1 ÞC1 C2 :
ð4:13Þ

Let us come back again to Figure 4.3.


Let point B01 be symmetric to B1 with respect to point O. For medians AO and CO
0
of triangles AB1 B01 and CB1 B01 we have the inequalities AO < (AB 1 + AB1)/2 and
0  0 
CO < (CB 1 + CB1)/2. Therefore, AO þ CO < AB 1 þ B01 C =2 þ AC=2. On the
other hand, point B01 is inside of triangle ABC, AB0 1 þ B01 C < AB þ BC, and
AB þ BC þ AC
AO þ CO < : ð4:14Þ
2
Then, using inequalities (4.13) and (4.14) we deduce that
158 4 Application of Vectors

p  p  p 
p2  O 1 O 2  1
þ B2 C2 A1 A2 þ 1 þ A2 C2 B1 B2 þ 1 þ A2 B2 C1 C2 <
2 2 2
p1 þ p2
< ðA1 A2 þ B1 B2 þ C1 C2 Þ:
2
Thus, A1 A2 þBO11BO22þC1 C2 < p12pþp2 .
2
It is clear that, from the very beginning we could assume that max( p1, p2) ¼ p2.
Remark If A1 6 A2, B1 6 B2, then (4.13) takes the following form
p2  O1O2  (O1A1 + A1B1 + O1B1)C1C2. Thus, p2  O1O2 < (C1A1 + A1B1 + C1B1)
C1C2 ¼ p1  C1C2.  
p1 p2
Hence, it follows that OC11 O
C2
2
< min ;
p p .
2 1

4.1.20. (a) Let P, Q, and R be the tangential points of the unit sphere with the faces
! ! !
 Then, ~
of trihedral angle. p ¼ OP, ~ q ¼ OQ , and ~r¼ OR  are unit vectors and
arccos3  ~
2 c
p,~ π
q  2, arccos3  ~ 2 c
q,~ π
r  2, arccos3  ~
2 c
p,~r  π2 . On the other
!
hand, for vector ~ a ¼ OA , we have that ~ p ¼~
a~ q ¼~
a~ a~r ¼ 1.
pffiffiffi
We needto prove that
        j~
a j  3 .
Let max ~ cq ; ~
p,~ cr ; ~
p,~ cr
q,~ ¼ ~ cq ¼ φ, then there exist angles φ1 and
p,~
 !   ! 
α, such that φ2  φ1  φ, α < π2, and ~ p ¼ 1; 0; 0 , ~ q ¼ cos φ; sin φ; 0 ,
 ! 
~
r ¼ cos φ1 cos α; sin φ1 cosα; sin α (see Figure 4.4).
  pffiffiffi
If α ¼ π2, then a ¼ 3.
 
y,!z , then we find that x ¼ 1, x cos φ + y sin φ ¼ 1 and x cos φ
If ~ a ¼ x, 1
cos α + y sin φ1 cos α + z sin α ¼ 1. Consequently, x ¼ 1, y ¼ tg φ2 , z¼
 φ
 φ
   φ

cos 2  cos α cos φ1  2 = sin α cos 2 .
  2
Then, we have to prove that cos φ2  cos α cos φ1  φ2  sin 2 α 1þ32cos φ.

Figure 4.4
z


r

О a
y
 
p j1 q

x
4.1 Application of Vectors for Proving Geometric and Trigonometric Inequalities 159

Figure 4.5

Figure 4.6 B¢ C¢

L
A¢ M0 M D¢

e2 L0

e3
B C

e1
K0
K
A D

   2
We have that cos φ2  cos φ1  φ2  1. Therefore, f ðtÞ ¼ cos φ2  t cos α 
   
max f cos φ2 ; f ð1Þ , at t 2 cos φ2 ; 1 .
   4  4
Note that f cos φ2 ¼ 1þ cos φ
4 sin α2  1þ32cos φ 4 sin α2  1þ32cos φ sin 2 α.
2
 2
It remains to prove that f ð1Þ ¼ cos φ2  cos α  1þ32cos φ sin 2 α, or
3ð1 þ cos φÞcos 2 α  4 cos φ2 cos α  2 cos φ  0.
 
We have that cos ~ cr ¼ cos α cos ðφ  φ Þ. Thus, it follows that
q,~ 1
pffiffi
π φ
cos α cos 4  cos α cos 2  cos α cos ðφ  φ1 Þ  3. Hence cos α  2 3 2. We have
2
h pffiffii
that t ¼ cos α 2 0; 2 3 2 . We deduce that gðtÞ ¼ 3ð1 þ cos φÞt2  4 cos φ2 t
  pffiffi 
2 cos φ  max g 2 3 2 ; gð0Þ .
 pffiffi pffiffi
Note that g(0) ¼  2 cos φ  0, and g 2 3 2 ¼ 83 ð1 þ cos φÞ  8 3 2 cos φ2 
pffiffi hpffiffi i
2 cos φ ¼ 43 cos 2 φ2  8 3 2 cos φ2 þ 2  0, because t ¼ cosφ2 2 22; 1 . Therefore,
160 4 Application of Vectors

pffiffi  pffiffi  pffiffi


φðtÞ ¼ 43 t2  8 3 2 t  2  max φ 22 ; φð1Þ  0, because φ 2
2
¼0 and
pffiffi
φð1Þ ¼ 103  3 < 0.
8 2

(b) Yes, it is possible (see Figure 4.5). First one has to construct “such” tetrahedron
and then add another 96 faces. One should take AB ¼ CD, AD ¼ DB ¼ BC ¼ AC,
where AB is large enough.
4.1.21. Let points K0, L0, M0 be the midpoints of edges AB, CC0 , and A0 D0 ,
! ! !
e1 ¼ KK00M
respectively. Let us denote ~ 0 e ¼ M0 L0 ,~
M0 ,~2
L0 M 0
M0 L0 e3 ¼ L0 M0 (see Figure 4.6).
Note that
 ! ! ! 
! ! e þ ! e3 ¼ KA þ AA0 þ A0 M ~
KM~e1 þ ML ~2 LK ~ e1 þ
 ! ! !   ! ! !
þ MD0 þ D0 C0 þ C0 L ~ e2 þ LC þ CB þ BK ~e3 ¼
! ! !  ! ! 
¼ AA0~e1 þD0 C0~
e2 þ CB~e3 þ KA~ e1 þ BK~e3 þ
 ! !   ! ! 
þ A0 M~ e1 þ MD0~ e2 þ C0 L~ e2 þ LC~e3 ¼
! ! ! ! !
¼ AA0~
e1 þD0 C0~ e2 þ CB~ e3 þ BA~ e1 þ BK ð~
e3 ~
e1 Þ
! 
!  !
þ þA0 D0~ e2 þ A0 M ð~
e1 ~e2 Þ þ C0 C~e3
!
þ C0 L ð~
e2 ~e3 Þ:
ð4:15Þ

! ! !
Because BK ð~ e3 ~e1 Þ ¼ 0, A0 M ð~ e2 Þ ¼ 0, C0 L ð~
e1 ~ e2 ~
e3 Þ ¼ 0 (see the proof
! ! !
of problem 2.4.18а), then from (4.15) it follows that, the sum KM~ e1 þ ML~ e2 þ LK
~
e3 is constant, this means that

! e þ ! ! ! ! !


KM ~1 ML~ e3 ¼ K 0 M0~
e2 þ LK ~ e1 þ M0 L0~
e2 þ L0 K 0~
e3
¼ K 0 M 0 þ M 0 L 0 þ L0 K 0 :
Thus,
rffiffiffi
! ! ! 3
KM þ ML þ LK  KM~ e1 þ ML~e2 þ LK ~
e3 ¼ K 0 M0 þ M0 L0 þ L0 K 0 ¼ 3 AB
2
pffiffiffi
¼ 1, 5 6AB:
4.1.22. Let G be the center of mass of tetrahedron ABCD, and O be the circumcenter.
Then, ma ¼ 43 AG, mb ¼ 43 BG, mc ¼ 43 CG, md ¼ 43 DG. We have that ma þ mb þ mc
4.1 Application of Vectors for Proving Geometric and Trigonometric Inequalities 161

pffiffiffiffiffiffiffiffiffiffiffiffiffiffiffiffiffiffiffiffiffiffiffiffiffiffiffiffiffiffiffiffiffiffiffiffiffiffiffiffiffiffiffiffiffiffiffiffiffiffiffiffi
þmd ¼ 43 ðAG þ BG þ CG þ DGÞ  83 AG2 þ BG2 þ CG2 þ DG2 , because R2 ¼
 ! !2 ! !
AO2 ¼ AG þ GO ¼ AG2 þ GO2 þ 2  GO  AG . We obtain that

! ! ! ! !
4R2 ¼ AG2 þ BG2 þ CG2 þ DG2 þ 4GO2 þ 2GO AG þ BG þ CG þ DG ¼
¼ AG2 þ BG2 þ CG2 þ DG2 þ 4GO2  AG2 þ BG2 þ CG2 þ DG2 :
pffiffiffiffiffiffiffiffiffiffiffiffiffiffiffiffiffiffiffiffiffiffiffiffiffiffiffiffiffiffiffiffiffiffiffiffiffiffiffiffiffiffiffiffiffiffiffiffiffiffiffiffi
Thus, ma þ mb þ mc þ md  83 AG2 þ BG2 þ CG2 þ DG2  16 3 R (see prob-
lem 4.1.13).
! ! !
4.1.23. Let CA ¼ a ~i, CB ¼ b ~j, CD ¼ x ~i þ y ~j þ z  ~ k, where ~i,~j, ~k are

! ! BD 
coordinate vectors and z 6¼ 0. We have that 
cos φ ¼ CA  ACBD
 
x
¼ qffiffiffiffiffiffiffiffiffiffiffiffiffiffiffiffiffiffiffiffiffiffiffiffiffiffiffiffiffiffiffiffiffiffiffiffi, where φ is the angle between lines AC and BD, as
x2 þ ðy  bÞ2 þ z2
pffiffiffiffiffiffiffiffiffiffiffiffiffiffi
x2 þy2 þz2 ! !
AB ¼
CD p ffiffiffiffiffiffiffiffiffi and 0 ¼ AD  BD ¼ xðx  aÞ þ yðy  bÞ þ z2 . We have to prove
a2 þb2
qffiffiffiffiffiffiffiffiffiffiffiffiffiffiffiffiffiffiffiffiffiffiffi
  qffiffiffiffiffiffiffiffiffiffiffiffiffiffiffiffiffiffiffiffiffiffiffiffiffiffiffiffiffiffiffiffiffiffiffiffiffiffiffiffiffiffiffiffiffiffiffiffiffiffiffiffiffiffiffiffiffiffiffiffiffiffiffiffiffiffiffiffiffiffiffiffiffiffiffiffiffiffi
 ffi qffiffiffiffiffiffiffiffiffiffiffiffiffiffiffiffiffiffiffiffiffiffiffi
 
that x2 a2 þ b2 < ðx2 þ y2 þ z2 Þ x2 þ y2 þ z2 þ b2  2by , or x2 a2 þ b2
qffiffiffiffiffiffiffiffiffiffiffiffiffiffiffiffiffiffiffiffiffiffiffiffiffiffiffiffiffiffiffiffiffiffiffiffiffiffiffiffiffiffiffiffiffiffiffiffi
 ffi
< ðax þ byÞ ax  by þ b2 , b2(x2 + y2) < b2(ax + by), 0 < z2.

4.1.24. (a) From the statement of the problem we have that


!
ðmi~
ei  mi1~
ei1 ÞAi Bi ¼ 0, i ¼ 1, 2, . . ., n. Consequently,

m1 A1 A2 þ m2 A2 A3 þ ::: þ mn1 An1 An þ mn An A1 ¼


! ! ! !
¼ m1 A1 A2~e1 þ m2 A2 A3~ e2 þ ::: þ mn1 An1 An~
en1 þ mn An A1~
en ¼
 ! ! !   ! ! ! 
¼ m1 A1 B1 þ B1 B2 þ B2 A2 ~ e1 þ m2 A2 B2 þ B2 B3 þ B3 A3 ~e2 þ þ :::
 ! ! !   ! ! ! 
þ mn1 An1 Bn1 þ Bn1 Bn þ Bn An ~ en1 þ þmn An Bn þ Bn B1 þ B1 A1 ~
en ¼
! ! !
¼ A1 B1 ðm1~e1  mn~en Þ þ A2 B2 ðm2~e2  m1~ e1 Þ þ ::: þ An Bn ðmn~en  mn1~
en1 Þ
! ! ! !
þ þm1 B1 B2~ e1 þ þ m2 B2 B3~ e2 þ ::: þ mn1 Bn1 Bn~ en1 þ mn Bn B1~
en ¼
! ! ! !
¼ m1 B1 B2~e1 þ m2 B2 B3~e2 þ ::: þ mn1 Bn1 Bn~ en1 þ mn Bn B1~ en 

 m1 B1 B2 þ m2 B2 B3 þ ::: þ mn1 Bn1 Bn þ mn Bn B1 :

Hence, we deduce that m1B1B2 + m2B2B3 + . . . + mn  1Bn  1Bn + mnBnB1 


m 1A1A2 + m 2A2A3 + . . . + m n  1A n  1An + m nAnA1.
(b) Let point M be the midpoint of edge C1C3 and point N be the midpoint of edge
C2C4. Let segment A1A4 be the common perpendicular of lines C3N and C2M,
that is, A1 2 C3N, A4 2 C2M, A1A4 ⊥ C2M, A1A4 ⊥ C3N. From the statement of
the problem it follows that points C1 and C3 are symmetric with respect to plane
162 4 Application of Vectors

C2C4M. Therefore, if point A3, symmetric to A1 with respect to plane C2C4M,


then A3 2 C1N, A3A4 ⊥ C2M, A3A4 ⊥ C1N.
Similarly, we obtain that, if point A2 is symmetric to A4 with respect to plane
C1C3N, then A2 2 C4M and A2A3 ⊥ C1N, A2A3 ⊥ C4M, A1A2 ⊥ C4M,
A1A2 ⊥ C3N.
!
A Aiþ1
Let~ei ¼ Aii Aiþ1 , i ¼ 1, 2, 3, 4; A5  A1. Because~ e3 ⊥C2 M and~ e4 ⊥C2 M, we have
that ~
e4 ~e3 ⊥C2 M. Also, A4A1 ¼ A4A3, A1A3 ⊥ C2C4M, C1C3 ⊥ C2C4M.
Consequently, A1A3||C1C3 and ~ e4 ~e3 ⊥A1 A3 . Hence ~
e4 ~ e3 ⊥C1 C3 . We have
obtained that ~ e4 ~ e3 ⊥C2 M, ~e4 ~e3 ⊥C1 C3 . Therefore, ~
e4 ~ e3 ⊥C1 C2 C3 . The
proof that other three conditions of problem 4.1.24a hold also true can be done
similarly. Thus, B1B2 + B2B3 + B3B4 + B4B1  A1A2 + A2A3 + A3A4 + A4A1. To
complete the solution we have to notice that points A1, A2, A3, A4 are on
segments C3N, C4M, C1N, C2M, respectively. Indeed, to prove, for example,
that point A1 is on segment C3N one has to consider projections of points A4, A1,
C3, N on a plane passing through point C2 and perpendicular to line C2M.
(c) Let points A1, A2, A3, A4, A5, A6 belong to the segments AC, A0 B, B0 C, A0 C0 , CD0 ,
DA0 , respectively, and

AA1 : AC ¼ BA2 : BA0 ¼ B0 A3 : B0 C ¼ C0 A4 : C0 A0 ¼ D0 A5 : D0 C


¼ DA6 : DA0 ¼ 1 : 3:

Now, it is not difficult to check that the conditions of problem 4.1.24a are
satisfied, where m1 ¼ m2 ¼ m3 ¼ m4 ¼ m5 ¼ m6 ¼ 1. Thus, it follows that

B1 B 2 þ B2 B 3 þ B3 B4 þ B4 B5 þ B5 B6 þ B6 B1
 A1 A2 þ A2 A3 þ A3 A4 þ A4 A5 þ A5 A6 þ A 6 A1 :

4.1.25. (a) Let us consider midpoints M, N, P, K, F, E of edges AD, BD, CD, AB,
AC, BC, respectively.
As OM ⊥ AD, OK ⊥ AB, OF ⊥ AC, then AO > AM, AO > AK, and AO > AF.
According to problem 7.1.38a tetrahedron AMKF does not contain point O. Then,
we obtain that point O is inside of the polyhedron with faces MPF, MFK, FKE,
PNE, MNK, FPE, MPN, NKE. For point X 6 O draw a plane α passing through
!
point O and perpendicular to vector OX . It is obvious, that at least one of the points
M, N, P, K, F, E and point X are on the different sides of plane α. If that is the point
! !
M, then OM  OX < 0.
! ! ! !
Let ~e1 ¼ OA ,~ e2 ¼ OB ,~e3 ¼ OC ,~ e4 ¼ OD and ~ e ¼~e1 þ~ e2 þ~ e3 þ~ e4 . We
OA OB OC OD
need to prove that
4.1 Application of Vectors for Proving Geometric and Trigonometric Inequalities 163

 
~
e < 2: ð4:16Þ

e 6¼ ~
Indeed, it is sufficient to prove inequality (4.16) in case ~ 0.
! ! !
Let OX ¼ ~ e, then as it was mentioned
  OM  OX < 0. Consequently,
ð~
e1 þ~ e < 0. We have that ~
e4 Þ~ e2~e  ~
e and ~ e  ~
e3~ e. Thus, ð~
e1 þ~
e2 þ
   2    
~
e3 þ~ e < 2~
e4 Þ~   
e . Hence ~ 
e < 2~   
e , then ~ 
e < 2. Note that

DA2 þ DB2 þ DC2 þ AB2 þ BC2 þ AC2 ¼ 


¼ AO2 ð~e1 ~e4 Þ2 þ ð~ e4 Þ2 þ ð~
e2 ~ e4 Þ2 þ ð~
e3 ~ e2 Þ2 þ ð~
e1 ~ e3 Þ2 þ ð~
e2 ~ e3 Þ2 ¼
e1 ~
¼ AO2 ð16  j~
e2 Þ > 12  AO2 :

Thus, DA2 + DB2 + DC2 + AB2 + BC2 + AC2 > 12  AO2.


(b) Note that

2OA  DA þ 2OA  DB þ 2OA  DC þ 2OA  AB þ 2OA  BC þ 2OA  AC >


> DA2 þ DB2 þ DC2 þ AB2 þ BC2 þ AC2 > 12  AO2

(see problem 4.1.25a).


Therefore, DA + DB + DC + AB + BC + AC > 6  OA.
Remark Given a polyhedron with n vertices and a point M inside of it. Let ~
ei be a
unit
 vector directed
 from point M to the i-th vertex of the polyhedron, then
~e1 þ~e2 þ ::: þ~en  < n  2.

Problems for Self-Study

4.1.26. Let ABCD be a quadrilateral circumscribed around a circle. Given that the
opposite sides AB and CD, BC and AD are on the lines intersecting at points M and
N. Prove that cos ∠A + cos ∠B + cos ∠C + cos ∠D + cos ∠M + cos ∠N  2.
4.1.27. Let ABC be an arbitrary triangle. Prove that for any equilateral triangle
 
A1B1C1 the inequality A1 A2 þ B1 B2 þ C1 C2  16 AB2 þ BC2 þ CA2  p2ffiffi3 SABC
holds true.
4.1.28. Given points A1, . . ., An and positive numbers m1, . . ., mn. For any point
X let us denote by f(X) the expression m1A1X + . . . + mnAnX. Given that point M is
on segment AB, such that BM AB ¼ α. Prove that f(M )  αf(A) + (1  α)f(B).

4.1.29. Given that pointsA1, A2, . . ., An are on the same sphere, m1, . . ., mn > 0 and
! !
G is such a point that m1 GA1 þ ::: þ mn GAn ¼ ~ 0. Let straight lines GA1, . . ., GAn
intersect this sphere (for the second time) at points B1, B2, . . . , Bn. Prove that
m1 GB1P þ ::: þ mn GBnP  m1 GA1P þ ::: þ mn GAnP , where 0  p  2.
164 4 Application of Vectors

4.1.30. Let O be the incenter of triangle ABC. Prove that ab + bc + ac  (AO + BO


+ CO)2.
Hint Prove that aAO2 + bBO2 + cCO2 ¼ abc (see problem 4.1.8b).
4.1.31. Let the medians of faces ABD, ACD, BCD drawn from vertex D of tetrahe-
dron ABCDform equal angles with edges AB, AC, BC, respectively. Prove that the
area of each of these faces is not greater than the sum of the areas of the other two
faces.
! !

DA þ DB ! 
Hint Let 2 ; AB ¼ α 6¼ 90 , then
 
 ! ! !  
SADB ¼ 12  AB  DA þ2 DB jctgαj ¼ 14 DA2  DB2 ctgα.
For α ¼ 90 see problem 1.1.14a (not only α ¼ 90 ).
4.1.32. Let in a tetrahedron ABCD the angles ADB, ADC, BDC be obtuse and the
lengths of edges AD, BD, CD be equal. Prove that ABC is an acute triangle.
! !
Hint Prove that CA  CB > 0.
4.1.33. Prove that the difference of the squares of the lengths of the adjacent sides of
a parallelogram is less than the product of its diagonals.
4.1.34. Let n points be inside of the unit sphere. Prove that the sum of the squares of
distances between all possible pairs of these points does not exceed n2.
4.1.35. (a) Consider a convex hexagon, such that the length of each of its sides is
more than 1. Will there always be in it a diagonal with the length more than 2?
(b) Consider a convex hexagon ABCDEF, such that the lengths of the diagonals AD,
BE, CFare greater than 2. Will it always have a side with the length more than 1?
Hint See problem 4.1.4a.
4.1.36. Let M be the intersection point of the diagonals of an inscribed quadrilateral,
N be the intersection point of its midlines and O be its circumcenter. Prove that
OM  ON (the midline is a segment connecting the midpoints of the opposite sides).
!
Hint Let ABCD be an inscribed quadrilateral, then ON ¼
 ! ! ! ! ! ! ! !
2
!2 OA þ OB þ OC þ OD .
4 OA þ OB þ OC þ OD , Prove that OM 
1
4

4.1.37. Prove that for any triangle ABC the following inequalities hold true:
(a) a2 þ b2 þ c2  9R2 ; cos 2α þ cos 2β þ cos 2γ  32,
(b) a3 þ b3 þ c3 þ 3abc  a2 b þ b2 a þ a2 c þ ac2 þ b2 c þ bc2 ; cos α þ cos β þ cos
γ  32,
(c) c2  a2 þ b2 þ R2 ; cos 2α þ cos 2β  cos 2γ  32.
4.1 Application of Vectors for Proving Geometric and Trigonometric Inequalities 165

4.1.38. Let α1, α2, . . ., α6 be (the linear angles of) the dihedral angles of a
tetrahedron. Prove that cosα1 + cos α2 + . . . + cos α6  2.
4.1.39. Use problem 4.1.8a for x ¼ y ¼ z ¼ 13 to prove problem 4.1.4b.
! ! ! ! !
Hint Consider points X, Y, Z, and M, such that XY ¼ BC þ CD  EF  FA ,
! ! ! ! ! ! ! !
YZ ¼ FA þ AB  DE  CD , and XM ¼ BC  EF .
4.1.40. The rectangular projection of a triangular pyramid to some plane has the
greatest possible area. Prove that this plane is parallel either to one of the sides of a
pyramid or to two skew edges of the pyramid.
4.1.41. (a) Given that in a tetrahedron ABCD the sum of the cosines of all plane
angles at vertex D does not exceed –1. Prove that for any point M, other than D,
holds true MA + MB + MC + MD > DA + DB + DC.
(b) Given that in a tetrahedron ABCD the sum of the cosines of all
plane angles at vertex D does not exceed –1. Prove that inside this
tetrahedron one can find a point M0, such that ∠AM0B ¼ ∠CM0D, ∠AM0C ¼
∠BM0D, ∠AM0D ¼ ∠BM0C.
Moreover, for any point M, other than M0, it holds true MA + MB + MC +
MD > M0A + M0B + M0C + M0D.
Hint See the proof of problem 4.1.17.
4.1.42. Let triangles A1A2A3 and B1B2B3 with orthocenters H1 and H2, respectively,
be inscribed in a circle of radius R. Prove that H1H2 < 4R + A1B1.
Hint Let G1 and G2 be the centroids of those triangles A1A2A3 and B1B2B3,
! 1  ! ! ! 
respectively. Prove that H1H2 ¼ 3G1G2 and G1 G2 ¼ 3 A1 B1 þ A2 B2 þ A3 B3 :
Chapter 5
Application of Trigonometric Inequalities

This chapter consists of five sections, that is, Sections 5.1, 5.2, 5.3, 5.4, and 5.5.
Many problems in geometry can be solved by applying trigonometry. In partic-
ular, many problems related to geometric inequalities can be deduced to trigono-
metric inequalities.
Section 5.1 mainly provides trigonometric inequalities concerning to angles of a
triangle. Let us emphasize few methods of proving such inequalities: note that, if
πβ πγ
α, β, γ are the angles of some triangle, then πα2 , 2 , 2 are the angles of some
acute triangle. Therefore, if some inequality holds true for angles α, β, γ of some
πβ πγ
triangle, then substituting these angles by πα 2 , 2 , 2 one can obtain a “new”
inequality for angles α, β, γ.
Moreover, if some inequality holds true for the angles of any acute triangle, then
from this inequality one can obtain a “new” inequality for the angles of any triangle.
In Section 5.1 many problems are proved using the maximal (minimal) values of
a quadratic polynomial. One of the crucial methods explained in this section is the
method based on the following statement: if the quadratic coefficient of a quadratic
function is positive (negative), then on any segment that function accepts its
maximal (minimal) value in one of the endpoints of the considered segment.
Section 5.2 selects such inequalities, concerning the angles of a triangle, that
hold true either only for acute triangles or only for obtuse triangles.
One of the most important methods of proving geometric inequalities is based on
modifications of trigonometric expressions. Therefore, Section 5.3 is devoted to
some important mathematical identities related to triangles.
Section 5.4 considers some trigonometric inequalities that are later on applied in
Section 5.5 in order to prove geometric inequalities.
Summarizing the above mentioned, Sections 5.1, 5.2, 5.3, and 5.4 provides
methods related to trigonometry in order to prove geometric inequalities. These
types of geometric inequalities are considered in Section 5.5.
Some problems in this chapter were inspired by [1, 2]. Nevertheless, even for
these problems the authors have mostly provided their own solutions.

© Springer International Publishing AG 2017 167


H. Sedrakyan, N. Sedrakyan, Geometric Inequalities, Problem Books
in Mathematics, DOI 10.1007/978-3-319-55080-0_5
168 5 Application of Trigonometric Inequalities

5.1 Inequalities for the Angles of a Triangle

Let α, β, and γ be the angles of some triangle. In the problems of this section it is
required to prove the following inequalities.
5.1.1. cos2 α þ cos2 β þ cos2 γ  34.
5.1.2. cosαcosβcosγ  18.
5.1.3. cos2α þ cos2β  cos2γ  32.
5.1.4. (a) 1 < cosα þ cosβ þ cosγ  32,
pffiffiffi 2  
(b) 3 cos α2 þ cos β2 þ cos 2γ  4 cos α2 þ cos β2 þ cos 2γ þ 2 cos α2 cos β2 cos 2γ .

5.1.5. 1 < sin α2 þ sin β2 þ sin 2γ  32.


pffiffi
5.1.6. sin α þ sin β þ sin γ  3 2 3.
pffiffi
5.1.7. cos α2 þ cos β2 þ cos 2γ  3 2 3.
pffiffiffi
5.1.8. (a) ctgα þ ctgβ þ ctgγ  3,
pffiffiffiffiffiffiffiffiffiffiffiffiffiffiffi pffiffiffiffiffiffiffiffiffiffiffiffiffiffiffi pffiffiffiffiffiffiffiffiffiffiffiffiffiffi
2α 1þ8cos 2 β 1þ8cos 2 γ
(b) 1þ8cos
sin α þ sin β þ sin γ  6.
p ffiffi

5.1.9. (a) tg α2 þ tg β2 þ tg 2γ  3,
 
(b) tg α2 tg β2 tg 2γ tg α2 þ tg β2 þ tg 2γ  13.
 
(c) ctg 2α2 þ ctg 2 β2 þ ctg 2 2γ  ctg α2 þ ctg β2 þ ctg 2γ ðctgα þ ctgβ þ ctgγ Þ.
pffiffiffi
5.1.10. ctg α2 þ ctg β2 þ ctg 2γ  3 3.
5.1.11. sin α2 sin β2 sin 2γ  18.
pffiffi
5.1.12. sin α  sin β  sin γ  3 8 3.
pffiffi
5.1.13. cos α2 cos β2 cos 2γ  3 8 3.
5.1.14. cos α cos β þ cos β cos γ þ cos γ cos α  34.
5.1.15. sin2α þ sin 2β þ sin 2γ  sin α þ sin β þ sin γ.
5.1.16. sin α þ sin β þ sin γ  cos α2 þ cos β2 þ cos 2γ .
5.1.17. ctgα þ ctgβ þ ctgγ  tg α2 þ tg β2 þ tg 2γ .
5.1.18. cos α þ cos β þ cos γ  sin α2 þ sin β2 þ sin 2γ .
5.1.19. ctg 2 α þ ctg 2 β þ ctg 2 γ  tg 2α2 þ tg 2 β2 þ tg 2 2γ .
5.1.20. (a) cos α  cosβ  cos γ  sin α2 sin β2 sin 2γ ,

(b) sin α  sin β  sin γ  cos α2 cos β2 cos 2γ ,


(c) cos α cos β cos γ  8sin 2α2 sin2 β2 sin2 2γ ,
5.1 Inequalities for the Angles of a Triangle 169

pffiffiffi
(d) sin α2 þ sin β2 þ sin 2γ  12  sin α2 sin β2 sin 2γ þ 3 cos α2 cos β2 cos 2γ .
(e) cos 2 αβ
4 þ cos
2 βγ
4 þ cos
2 γα
4  2ð cos α þ cos β þ cos γ Þ,
αβ βγ γα
(f) cos 2 þ cos 2 þ cos 2  p2ffiffi3 ð sin α þ sin β þ sin γ Þ,
pffiffiffi
(g) cos α þ cos β þ cos γ þ ctgα þ ctgβ þ ctgγ  1, 5 þ 3,
(h) (1  cos α)(1  cos β)(1  cos γ)  cos α cos β cos γ
(4  2 cos α  2 cos β  2 cos γ),
pffiffiffi
(i) sin 2α þ sin 2β þ sin 2γ  2 3ð cos α cos β þ cos β cos γ þ cos γ cos αÞ:
qffiffiffiffiffiffiffiffiffiffiffiffiffiffiffi
pffiffiffiffiffi
jαβj jβγ j jγαj
5.1.21. sin 2 þ sin 2 þ sin 2  71þ17 17
32 .

5.1.22. (a) 2xy cos α þ 2yzcosβ þ 2xz cos γ  x2 þ y2 þ z2, where α þ β þ γ ¼ π and
α, β, γ, x, y, z are arbitrary numbers.
cos α cos β cos γ
(b) þ sin
sin α1 β1 þ sin γ 1  ctg α1 þ ctg β1 þ ctg γ 1 , where α þ β þ γ ¼ π and
α1, β1, γ 1 are angles of some triangle.
(c) a2ctgα1 þ b2ctgβ1 þ c2ctgγ 1  4S, where a, b, c, S are respectively, the sides and
the area of some triangle, and α1, β1, γ 1 the angles of another triangle.
(d) (b2 þ c2)(1  cos φ) þ a2 cos φ  4S| sin φ|, where φ is any angle and S is the
area of the triangle with sides a, b, c.
     
(e) a2 b21 þ c21  a21 þ b2 a21 þ c21  b21 þ c2 a21 þ b21  c21  16SS1 , where a, b,
c, S are respectively, the sides and the area of some triangle, and a1, b1, c1, S1 of
another triangle.
(f) (xa2 þ yb2 þ zc2)2  16S2(xy þ yz þ zx), where x, y, z are arbitrary numbers and
S is the area of a triangle with sides a, b, c.
(g) k2tgα1 þ l2tgβ1 þ m2tgγ 1  S, where k, l, m are the distances from the circum-
center of triangle ABC to lines BC, AC, AB, respectively, S is the area of triangle
ABC, and α1, β1, γ 1 are the angles of some other acute triangle.
 2
5.1.23. cos 2α2 þ cos 2 β2 þ cos 2 2γ  sin α2 þ sin β2 þ sin 2γ .
πþα πþβ πþγ
5.1.24. sin 2α þ sin 2β þ sin 2γ  cos 8 þ cos 8 þ cos 8 .

5.1.25. cos 2 α þ cos 2 β þ cos γ > 34.


5.1.26. cos 2 αcos 2 β þ cos 2 βcos 2 γ þ cos 2 γcos 2 α  32 cos α cos β cos γ.
pffiffiffiffiffiffiffiffiffiffiffiffiffiffiffiffiffiffiffiffiffiffiffi
ffi pffiffiffiffiffiffiffiffiffiffiffiffiffiffiffiffiffiffiffiffiffiffiffi pffiffiffiffiffiffiffiffiffiffiffiffiffiffiffiffiffiffiffiffiffiffiffi
x2 þy2 þ2xy cos α y2 þz2 þ2yz cos β z2 þx2 þ2zx cos γ
5.1.27. sin α þ sin β þ sin γ  2x þ 2y þ 2z, where
x  0, y  0, z  0.
5.1.28. 3(cosα þ cos β þ cos γ)  2(sinα sin β þ sin β sin γ þ sin γ sin α).
5.1.29. sin α3  sin β3  sin 3γ  8sin 3π9  sin α2  sin β2  sin 2γ .
5.1.30. tg 2α2 þ tg 2 β2 þ tg 2 2γ  2, where γ  2arctg43.
5.1.31. sin α sin α1 þ sin β sin β1 þ sin γ sin γ 1  144 sin α2 sin β2 sin 2γ sin α21 sin β21 sin γ21 ,
where α, β, γ and α1, β1, γ 1 are angles of some triangles.
170 5 Application of Trigonometric Inequalities

5.1.32. Prove that

sin α sin α1 þ sin β sin β1 þ sin γ sin γ 1 


 2 þ ð cos α þ cos β þ cos γ  1Þð cos α1 þ cos β1 þ cos γ 1  1Þ,

where α, β, γ and α1, β1, γ 1 are the angles of some triangles.

Solutions

5.1.1. We have that


1 þ cos 2α 1 þ cos 2β
cos 2 α þ cos 2 β þ cos 2 γ ¼ þ þ cos 2 γ ¼
2 2
¼ 1 þ cos ðα þ βÞ cos ðα  βÞ þ cos 2 γ ¼
3 sin 2 ðα  βÞ
¼ 1  cos γ cos ðα  βÞ þ cos 2 γ ¼ þ
 2 3 4 4
cos ðαβÞ
þ cos γ  2  :
4
This ends the proof.
5.1.2. Since (see the proof of the problem 5.1.1)

cos 2 α þ cos 2 β þ cos 2 γ ¼


3
¼ 1  cos γ ð cos ðα  βÞ  cos γ Þ ¼ 1  2 cos α cos β cos γ  ,
4
1
then cos α  cos β  cos γ  .
8
This ends the proof.
5.1.3. Let us note that
3 1
 ð cos 2α þ cos 2β  cos2γ Þ ¼ 2cos 2 γ þ 2 cos γ cos ðα  βÞ þ ¼
2 2
1 
¼ ð2 cos γ þ cosðα  βÞÞ2 þ sin 2 ðα  βÞ  0:
2
This ends the proof.
5.1.4. (a) We have that
γ αβ γ
cos α þ cosβ þ cos γ ¼ 2 sin cos þ 1  2sin 2 ¼
2 2 2
α β γ
¼ 1 þ 4 sin sin sin > 1:
2 2 2
πβ πγ
If α, β, γ are the angles of a triangle, then πα
2 , 2 , 2 are also angles of some
acute triangle. Therefore, using the inequality of the problem 5.1.2, we deduce that
πβ πγ β γ
cos πα α
2 cos 2 cos 2  8 or sin 2 sin 2 sin 2  8, hence cos α þ cos β þ cos γ 
1 1

1 þ 4  8 ¼ 2.
1 3
5.1 Inequalities for the Angles of a Triangle 171

(b) We have that


α β γ αþβ βþγ γþα
cos þ cos þ cos ¼ 4 cos cos cos ,
2 2 2 4 4 4
α β γ βþγ αþγ αþβ
cos cos cos ¼ sin sin sin ¼
2 2 2 2 2 2
αþβ βþγ αþγ αþβ βþγ γþα
¼ 8 cos cos cos  sin sin sin :
4 4 4 4 4 4
We have to prove that
pffiffiffi αþβ βþγ γþα αþβ βþγ γþα
3 cos cos cos  1 þ sin sin sin ,
4 4 4 4 4 4

or
pffiffiffi    
3 α β γ 1 α β γ
cos þ cos þ cos 1þ sin þ sin þ sin  1 ,
4 2 2  2  4 2 2 2
α π  β π γ π 3
cos þ þ cos þ þ cos þ  ,
2 6 2 6 2 6 2

(see the problem 5.1.4a).


This ends the proof.
πα πβ πγ
5.1.5. From the problem 5.1.4a, we have that 1 < cos 2 þ cos 2 þ cos 2  32.
Hence, 1 < sin α2 þ sin β2 þ sin 2γ  32.
This ends the proof.
5.1.6. If x, y 2 [0, π], then sin x þ sin y ¼ 2 sin xþy xy
2 cos 2  2 sin xþy
2 .
Consequently,

π αþβ γ þ π3 αþβ
þ γþπ=3 π
sin α þ sin β þ sin γ þ sin  2 sin þ 2 sin  4 sin 2 2
¼ 4 sin :
3 2 2 2 3
pffiffi
Hence, we obtain that sin α þ sin β þ sin γ  3 sin π3 ¼ 3 2 3.
This ends the proof.
πβ πγ
5.1.7. Using the inequality of the
pffiffi
problem 5.1.6 for angles πα
2 , 2 , 2 , we deduce
α β γ
that cos 2 þ cos 2 þ cos 2  2 .
3 3

This ends the proof.


5.1.8. (a) Note that
sin γ cos γ 2 sin γ cos γ
ctgα þ ctgβ þ ctgγ ¼ þ ¼ þ 
sin α sin β sin γ cosðα  βÞ þ cos γ sin γ
0 1
2γ vffiffiffiffiffiffiffiffiffiffiffiffiffiffiffiffiffiffi
2 sin γ cos γ γ 1  tg 2 1@ 1 γA u u 1 γ pffiffiffi
 þ ¼ 2tg þ γ ¼ γ þ 3tg  t γ 3tg ¼ 3:
1 þ cos γ sin γ 2 2tg 2 tg 2 tg 2
2 2 2
172 5 Application of Trigonometric Inequalities

(b) We have that


pffiffiffiffiffiffiffiffiffiffiffiffiffiffiffiffiffiffiffiffiffiffi pffiffiffiffiffiffiffiffiffiffiffiffiffiffiffiffiffiffiffiffiffiffi pffiffiffiffiffiffiffiffiffiffiffiffiffiffiffiffiffiffiffiffiffi
1 þ 8cos 2 α 1 þ 8cos 2 β 1 þ 8cos 2 γ pffiffiffiffiffiffiffiffiffiffiffiffiffiffiffiffiffiffiffiffiffi pffiffiffiffiffiffiffiffiffiffiffiffiffiffiffiffiffiffiffiffiffi pffiffiffiffiffiffiffiffiffiffiffiffiffiffiffiffiffiffiffiffiffi
þ þ ¼ 1 þ 9ctg 2 α þ 1 þ 9ctg 2 β þ 1 þ 9ctg 2 γ 
sin α sin β sin γ
qffiffiffiffiffiffiffiffiffiffiffiffiffiffiffiffiffiffiffiffiffiffiffiffiffiffiffiffiffiffiffiffiffiffiffiffiffiffiffiffiffiffiffiffiffiffiffiffiffiffiffi
 9 þ 9ðctgα þ ctgβ þ ctgγ Þ2  6

(see the problem 1.2.5a and 5.1.8a).


This ends the proof.
πβ πγ
5.1.9. (a) Using the inequality of the problem p ffiffi
ffi 5.1.8a for angles πα 2 , 2 , 2p, ffiffiwe

πβ πγ
obtain that ctg πα
2 þ ctg 2 þ ctg 2  3. Therefore, tg α2 þ tg β2 þ tg 2γ  3.
 
(b) We have that tg α2 þ β2 ¼ ctg 2γ , consequently tg α2 tg β2 þ tg β2 tg 2γ þ tg 2γ tg α2 ¼ 1.
 
Hence 1  3 tg α2tg β2tg β2tg 2γ þ tg α2tg 2γ tg α2tg β2 þ tg α2tg 2γ tg β2tg 2γ , since (x þ y þ z)2
 3(xy þ yz þ zx).
 
(c) One needs to prove that tg α2  tg β2  tg 2γ ctg 2α2 þ ctg 2 β2 þ ctg 2 2γ  ctgαþ
ctgβ þ ctgγ, or equivalently
   
α β α γ α α β β γ β
1  tg  tg  tg  tg  ctg þ 1  tg  tg  tg  tg  ctg þ
2 2 2 2 2 2 2 2 2 2
  α β γ
α γ β γ γ 1 tg 1 tg 1 tg
þ 1  tg  tg  tg  tg  ctg   2 þ  2 þ 2
2 2 2 2 2 2tg α 2 β 2 γ 2 ,
2tg 2tg
2 2 2
 
1 α β γ α β γ
or  tg  tg  tg tg þ tg þ tg .
3 2 2 2 2 2 2
The last inequality follows from problem 5.1.9b.
This ends the proof.
 ðxþyÞ 2 sin ðxþyÞ
5.1.10. If x, y 2 0; π2 , then ctgx þ ctgy ¼ sin
sin x sin y  1 cos ðxþyÞ ¼ 2ctg 2 . Hence, it
xþy

follows that
γ π αþβþγ π
α β γ π αþβ þ þ
ctg þ ctg þ ctg þ ctg  2ctg þ 2ctg 2 6  4ctg 4 12 ¼ 4ctg π :
2 2 2 6 4 2 2 6
β γ
p ffiffi

Therefore, ctg α2 þ ctg 2 þ ctg 2  3ctg π6 ¼ 3 3.
This ends the proof.
πβ πγ
5.1.11. Using the inequality of the problem 5.1.2 for angles πα
2 , 2 , 2 , we obtain
that sin α2 sin β2 sin 2γ  18.
This ends the proof.
5.1.12. Note that
1 1  cos ðx þ yÞ
sin x  sin y ¼ ð cos ðx  yÞ  cos ðx þ yÞÞ  ¼
2 2
x þ y
¼ sin 2 :
2
5.1 Inequalities for the Angles of a Triangle 173

Therefore, we obtain that


αþβ γþπ3
π αþβ γ þ π3 þ π
sin α sin β sin γ sin  sin 2  sin 2  sin 4 2 2
¼ sin 4 :
3 2 2 2 3
pffiffi
This means that sin α sin β sin γ  sin 3π3 ¼ 3 8 3.
This ends the proof.
πα πβ πγ
5.1.13. Using the inequality ofpffiffithe problem 5.1.12 for angles 2 , 2 , 2 , we
obtain that cosα2 cos β2 cos 2γ  3 8 3.
This ends the proof.
5.1.14. Let α < π2, then

3
cos αcosβ þ cosβ cos γ þ cos γ cos α  ¼
4
βþγ βγ 1 3
¼ 2 cos α cos cos þ ð cos ðβ  γ Þ þ cos ðβ þ γ ÞÞ  
2 2 2 4 
α 1 3 1 α 2  α
 2 cos α sin þ ð1  cos αÞ  ¼  2 sin  1 4 sin þ 3  0:
2 2 4 4 2 2

This ends the proof.


5.1.15. We have that sin2α þ sin 2β ¼ 2 sin(α þ β) cos(α  β)  2 sin γ.
Similarly, sin2β þ sin 2γ  2 sin α and sin2α þ sin 2γ  2 sin β.
Summing up these three inequalities, we deduce that
sin2α þ sin 2β þ sin 2γ  sin α þ sin β þ sin γ.
This ends the proof.
πα πβ πγ
5.1.16. Using the inequality of the problem 5.1.15 for angles 2 , 2 , 2 , we
deduce that sin α þ sin β þ sin γ  cosα2 þ cos β2 þ cos 2γ .
This ends the proof.
5.1.17. We have that

sin ðα þ βÞ 2 sin γ 2 sin γ γ


ctgα þ ctgβ ¼ ¼  ¼ 2tg :
sin α sin β cos ðα  βÞ  cos ðα þ βÞ 1 þ cos γ 2

Similarly, we obtain that ctgβ þ ctgγ  2tg α2 and ctgγ þ ctgα  2tg β2.
Therefore, ctgα þ ctgβ þ ctgγ  tg α2 þ tg β2 þ tg 2γ .
This ends the proof.
5.1.18. Note that cos α þ cosβ ¼ 2 sin 2γ cos αβ
2  2 sin 2γ . Thus, it follows that
cosα þ cos β cosβ þ cos γ cos γ þ cosα
cos α þ cosβ þ cos γ ¼ þ þ 
2 2 2
γ α β
 sin þ sin þ sin :
2 2 2
This ends the proof.
174 5 Application of Trigonometric Inequalities

5.1.19. We have that ctg 2 α þ ctg 2 β  12 ðctgα þ ctgβÞ2  2tg 2 2γ (see the proof of
the problem 5.1.17). Hence ctg 2 α þ ctg 2 β þ ctg 2 γ  tg 2α2 þ tg 2 β2 þ tg 2 2γ .
This ends the proof.
5.1.20. (a) If the triangle is not acute angled, then inequality is correct, as
cos α cos β cos γ  0 < sin α2 sin β2 sin 2γq . ffiffiffiffiffiffiffiffiffiffiffiffiffiffiffiffiffiffiffi

π pffiffiffiffiffiffiffiffiffiffiffiffiffiffiffiffiffiffiffiffiffi
If x, y 2 0; 2 , then cos x cos y  1þ cos2ðxþyÞ ¼ cos xþy 2 .
But if the triangle is an acute triangle, then
pffiffiffiffiffiffiffiffiffiffiffiffiffiffiffiffiffiffiffiffiffiffipffiffiffiffiffiffiffiffiffiffiffiffiffiffiffiffiffiffiffiffiffipffiffiffiffiffiffiffiffiffiffiffiffiffiffiffiffiffiffiffiffiffi
cos α cos β cos γ ¼ cos α cos β cos β cos γ cos γ cos α 
αþβ βþγ αþγ γ α β
 cos cos cos ¼ sin sin sin :
2 2 2 2 2 2

(b) We have that


pffiffiffiffiffiffiffiffiffiffiffiffiffiffiffiffiffiffiffiffipffiffiffiffiffiffiffiffiffiffiffiffiffiffiffiffiffiffiffiffipffiffiffiffiffiffiffiffiffiffiffiffiffiffiffiffiffiffiffiffi
sin α sin β sin γ ¼ sin γ sin β sin α sin γ sin α sin β 
α β γ
 cos cos cos
2 2 2
(see the proof of the problem 5.1.12).
(c) Let α  β  γ, then we need to prove that ð cos ðα  βÞ þ cos γ Þ cos γ  4
 
γ 2
 
cos αβ 2γ
2  sin 2 sin 2, f cos 2
αβ
 0, where f ðtÞ ¼ ð2 cos γ  1Þt2 þ 4
γ γ γ
sin 3 2 t  cos 2 2 cos γ  2sin 4 2.
 
Since 0  cos αβ 2  1, then we have that f cos 2
αβ
 maxðf ð0Þ; f ð1ÞÞ.

We need to prove that, if f(0)  0, f(1)  0, then f cos αβ 2  0.
2γ 4γ
We have that f ð0Þ ¼ cos 2 cos γ  2sin 2  0, and f ð1Þ ¼ sin 2 2γ
 γ
2
2 sin 2  1  0.
(d) Let α  β  γ. We need to prove that
γ π  γ π 
sin α2 þ sin β2 þ sin 2γ  12 þ cos αβ γ

  2 cos 2 þ 6 þ sin 2 cos 2  6 , or


f cos αβ  0, where
4   γ π  γ π 
f ðtÞ ¼ 2 cos 2γ þ π6 t2  2 sin πγ γ γ
4  t þ2  sin 2  cos 2 þ 6 þ sin 2 cos 2  6 :
1

As 0  cos αβ 4  1, then f cos 4
αβ
 maxðf ð0Þ; f ð1ÞÞ.
 
We need to prove that, if f(0)  0 and f(1)  0, then we have that f cos αβ4  0.
We have that
1 γ γ π  γ γ π 
f ð0Þ ¼  sin  cos þ þ sin cos  ¼
2 2 2 6 2 2 6
p ffiffi
ffi p ffiffi
ffi 
1  π γ 3 3 γ
¼ sin γ   sin þ  cos  0,
2 6 2 2 2 2
since π6 < γ  π6  2γ  π6.
5.1 Inequalities for the Angles of a Triangle 175

Because
γ γ π   1 γ π  πγ
f ð1Þ ¼ sin
cos   1 þ þ cos þ  2 sin 
2 2 6 2 2 6 4
1  γ π  πγ πγ   πγ π  
 þ cos þ  2 sin ¼ 2 sin sin þ  1  0,
2 2 6 4 4 4 3
we have that f(1)  0.
(e) Let γ  β  α. One needs to prove that
 
3 1 αβ βγ γα
þ cos þ cos þ cos  cos α þ cos β þ cos γ:
4 4 2 2 2
 
γ 1 βα 1 βα π  3γ 3
2 sin  cos  cos cos þ cos γ   0:
2 4 2 2 4 4 4

Note that γ  π3. Therefore, 2 sin 2γ  14  34.


 
If f ðxÞ ¼ 2 2 sin 2γ  14 x2  12 x cos π3γ γ
4 þ cos γ  2 sin 2  2, then one needs to
1

prove that
 
f cos βα4  0, as 0  cos βα 4  1 and for x 2 [0, 1].

We have that f(x)  max( f(0), f(1)), then it is sufficient to prove that f(0)  0 and
f(1)  0.
Indeed, we have that

γ 1 1 γ γ 1 1 1
f ð0Þ ¼ cos γ  2 sin  ¼  2sin 2  2sin 2   2   2  < 0:
2 2 2 2 2 2 2 4
γ 1 π  3γ
f ð1Þ ¼ 2 sin  1  cos þ cos γ:
2 2 4

Let γ ¼ π  2φ, then


 2  
f ð1Þ ¼ 2 cos φ  1  12 sin 3φ
2  cos 2φ ¼ 12 sin φ2 2 sin φ2  1 4 sin φ2 þ 3  0,
as 0 < φ  π3 :

Remark For α þ β þ γ ¼ π the inequality of problem 5.1.20e may not hold true.
For example, if α ¼ π2 , β ¼ 5π
2 , γ ¼ 2π.

(f) Let α  β  γ. One needs to prove that

αβ αβ π  3γ 4 γ αβ 2


cos þ 2 cos cos  pffiffiffi cos cos þ pffiffiffi sin γ,
2 4 4 3 2 2 3
176 5 Application of Trigonometric Inequalities

or
 
4 γ αβ π  3γ αβ 2 4 γ
2 pffiffiffi cos  1 cos 2  2 cos cos þ pffiffiffi sin γ  pffiffiffi cos þ 1  0:
3 2 4 4 4 3 3 2
 
Let f ðxÞ ¼ 2 p4ffiffi cos γ  1 x2  2 cos π3γ
þ p2ffiffi3 sin γ  p4ffiffi3 cos 2γ þ 1.
3 2 4 x
Note that
    pffiffi
2 p4ffiffi3 cos 2γ  1  2 p4ffiffi3 cos π6  1 ¼ 2 and 22  cos αβ
4  1, as 0  αβ π
4  4.

One needs to prove that


 
αβ
f cos  0: ð5:1Þ
4

We have that
   pffiffiffi
αβ 2
f cos  max f ð1Þ; f :
4 2
pffiffi
Let us prove that f 22  0 and f(1)  0, then (5.1) holds true.
We have that
pffiffiffi
2 2 pffiffiffi π  3γ 2 π pffiffiffi π  3γ
f ¼ pffiffiffi sin γ  2 cos  pffiffiffi sin  2 cos ¼
2 3 4 3 3 4
pffiffiffi π  3γ pffiffiffi π pffiffiffi π
¼ 1  2 cos < 1  2 cos < 1  2 cos ¼ 0,
4 4 4
pffiffi
therefore f 22 < 0.
Now, let us prove that f(1)  0.
We need to prove that 1 þ 2 cos π3γ γ
4  3 cos 2 þ 3 sin γ.
p4ffiffi p2ffiffi

Consider the following function


π
gðγ Þ ¼ 1 þ 2 cos π3γ γ
4  3 cos 2  3 sin γ on 0; 3 .
p4ffiffi p2ffiffi

For 0 < γ  π3, we have that

0 3 π  3γ 2 γ 2 2 γ 2
g ðγ Þ ¼  sin þ pffiffiffi sin  pffiffiffi cos γ  pffiffiffi sin  pffiffiffi cos γ 
2 4 3 2 3 3 2 3
2 π 2 π
 pffiffiffi sin  pffiffiffi cos ¼ 0:
3 6 3 3
π 
Hence, gðγ Þ  g 3 ¼ 0.
Hence, it follows that f(1)  0.
5.1 Inequalities for the Angles of a Triangle 177

(g) Let max(α, β, γ) ¼ γ, then γ  π3.


We need to prove that
γ αβ sin γ 3 pffiffiffi
2 sin cos þ cos γ þ αβ γ
þ ctgγ  þ 3:
2 2 cos 2  sin 2
2 2 2

Consider the following function


 γ sin γ  γ i
f ðxÞ ¼ 2 sin x þ cos γ þ 2 γ þ ctgγ on sin ;1 :
2 x  sin 2 2 2

We have that
 2 
2γ 2 γ  
2γ 2 γ γ
0 γ x  sin 2  2x cos 2 γ 1  sin 2  2 sin 2 cos 2
f ðxÞ ¼ 2 sin   2 < 2 sin   2 ¼
2 x2  sin 2 γ 2
2 x2  sin 2 γ 2
3γ γ γ
cos  2 sin cos 3  1
¼ sin γ   2 2 2
2  sin γ   2  0:
x2  sin 2 2γ x2  sin 2 2γ


Thus, it follows that for x 2 sin 2γ ; 1 , we have that f 0 (x) < 0. Therefore, f(x) is a
decreasing function on sin 2γ ; 1 .  
αβ 3 pffiffiffi
Note that, it is sufficient to prove that f cos  þ 3.
  2 2
αβ  γ i αβ
As cos 2 sin ; 1 , then f cos  f ð1Þ.
2 2 2 
3 pffiffiffi αβ 3 pffiffiffi
Let us prove that f ð1Þ  þ 3, then f cos  þ 3.
2 2 2
γ sin γ
We have that f ð1Þ ¼ 2 sin þ cos γ þ þ ctgγ.
2 1  sin 2 2γ
Let γ ¼ π  2φ, then 0 < φ < π3 pffiffiffi and one needs to prove that
2 cos φ  cos 2φ þ 2ctgφ  ctg 2φ  32 þ 3, or
 
3 þ tg 2 φ pffiffiffi 1 2
 3  2 cos φ  :
2tgφ 2
pffiffiffi 2   2
3  tgφ  4tgφ cos φ  12 ,
pffiffiffi 2  2
3 cos φ  sin φ  4 cos φ sin φ cos φ  12 ,
2  2 pffiffiffi 2
ð3cos 2 φ  sin 2 φÞ  2 sin 2φ cos φ  12 3 cos φ þ sin φ ,
1 pffiffiffi 2
ð2 cos φ  1Þ2 ð2 cos φ þ 1Þ2  sin 2φð2 cos φ  1Þ2 3 cos φ þ sin φ :
2
178 5 Application of Trigonometric Inequalities

It is sufficient to prove that

1 pffiffiffi 2
ð2 cos φ þ 1Þ2  sin 2φ 3 cos φ þ sin φ :
2
pffiffiffi
The last inequality holds true, as (2 cos φ þ 1)2  4 and 1
2 sin 2φ 3 cos φþ
sin φÞ2  12  1  4 ¼ 2.
(h) Let α  β  γ, then we need to prove that

1  ð cosα þ cosβ þ cosγ Þ þ cosαcosβ þ ð cosα þ cosβÞcosγ 


 5cosαcosβ cosγ  2cosαcosβ cosγ ð cosα þ cosβ þ cosγ Þ,
αβ γ 1 αβ γ
1  2cos sin  cosγ þ ð cos ðα  βÞ  cosγ Þ þ 2cos sin cosγ 
2 2 2 2 2
 
5 αβ γ
 cosγ ð cos ðα  βÞ  cosγ Þ  ð cos ðα  βÞ  cosγ Þcosγ 2cos sin þ cosγ ,
2 2 2
 
αβ γ 1 α  β α  β γ
1  2cos sin  cosγ þ 2cos 2  1  cosγ þ 2cos sin cosγ 
2 2 2 2 2 2
     
5 αβ αβ αβ γ
 cosγ 2cos 2  1  cosγ  2cos2  1  cosγ cosγ 2cos sin þ cosγ :
2 2 2 2 2

Let us consider the following function


γ   γ 
f ðxÞ ¼ 4 cos γ sin x3 þ 1  5 cos γ þ 2cos 2 γ x2  2 sin 1 þ cos 2 γ x
2 2
cos 3 γ þ 1, 5cos 2 γ þ cos γ þ 0, 5

on [0; 1].  
We need to prove that f cos αβ2  0.
We have that 0 < γ  π3 , therefore

γ   γ 
f 0 ðxÞ ¼ 12 cos γ sin x2 þ 2 1  5 cos γ þ 2cos 2 γ x  2 sin 1 þ cos 2 γ ,
2 2
γ 
f 0 ð0Þ ¼ 2 sin 1 þ cos 2 γ < 0,
2
γ 
f 0 ð1Þ ¼ 2 sin 6 cos γ  1  cos 2 γ þ 2  10 cos γ þ 4cos 2 γ 
2
 6 cos γ  1  cos 2 γ þ 2  10 cos γ þ 4cos 2 γ ¼
¼ ð cos γ  1Þð3 cos γ  1Þ < 0:

As 0  x  1, then f0 (x)  max( f0 (0), f0 (1)) < 0.


As 0  cos αβ2  1, then
 
αβ γ   γ 
f cos  f ð1Þ ¼ 4 cos γ sin þ 1  5 cos γ þ 2cos 2 γ  2 sin 1 þ cos 2 γ 
2 2 2
 2
cos 3 γ þ 1, 5cos 2 γ þ cos γ þ 0, 5 ¼ 2ð1  cos γ Þ2 sin 2γ  0; 5  0:
5.1 Inequalities for the Angles of a Triangle 179

(i) Let α  β  γ. If γ ¼ π3, then α ¼ β ¼ π3 and


pffiffiffi
3 3 pffiffiffi
sin 2α þ sin 2β þ sin 2γ ¼ ¼ 2 3ð cos α cos β þ cos β cos γ þ cos γ cos αÞ:
2

If γ < π3, then


pffiffiffi
sin 2α þ sin 2β þ sin 2γ  2 3ð cos α cos β þ cos β cos γ þ cos γ cos αÞ ¼
pffiffiffi
¼ 2 sin ðα þ βÞ cos ðα  βÞ þ sin 2γ  3ð cos ðα  βÞ þ cos ðα þ βÞÞ
pffiffiffi αþβ αβ  pffiffiffi
4 3 cos γ cos cos ¼ 2 sin γ  3 cos ðα  βÞ
2 2
pffiffiffi γ αβ pffiffiffi  pffiffiffi αβ
4 3 cos γ sin cos þ sin 2γ þ 3 cos γ ¼ 2 2 sin γ  3 cos 2 
2 2 2
pffiffiffi γ αβ pffiffiffi pffiffiffi
4 3 cos γ sin cos þ sin 2γ þ 3 cos γ  2 sin γ þ 3:
2 2

Let us consider the following function


 pffiffiffi pffiffiffi γ pffiffiffi pffiffiffi
f ðxÞ ¼ 2 2 sin γ  3 x2  4 3 cos γ sin x þ sin 2γ þ 3 cos γ  2 sin γ þ 3
2

on [0; 1].  pffiffiffi


We have that 2 2 sin γ  3 < 0, and
pffiffiffi pffiffiffi pffiffiffi γ γ γ
f ð0Þ ¼ sin 2γ þ 3 cos γ  2 sin γ þ 3 ¼ 2 3cos 2  8 cos sin 3 ¼
2 2 2
   
γ pffiffiffi γ 3 3 γ
¼ 2 cos 3 cos  þ  4sin 3 > 0,
2 2 2 2 2
 pffiffiffi pffiffiffi γ pffiffiffi pffiffiffi
f ð1Þ ¼ 2 2 sin γ  3  4 3 cos γ sin þ sin 2γ þ 3 cos γ  2 sin γ þ 3 ¼
2
pffiffiffi pffiffiffi γ
¼ 2 sin γ þ sin 2γ  3ð1  cos γ Þ  4 3 cos γ sin ¼
2
γ γ pffiffiffi pffiffiffi γ
¼ 2 sin 4cos 3  2 3 cos γ  3 sin :
2 2 2
pffiffiffi pffiffiffi

Consider a function gðxÞ ¼ 4cos 3x2  2 3 cos x  3 sin 2x on 0; π3 :
As 0 < x < π3 , we have that
 pffiffiffi
0 x x pffiffiffi x 3
g ðxÞ ¼ cos 2 sin 2 3  3 cos  < 0,
2 2 2 2
pffiffiffi pffiffi
as sin 2x < 12 , 0 < 2 3  3 cos 2x < 23.
180 5 Application of Trigonometric Inequalities


Then, for 0 < x < π3 , it follows that gðxÞ > g π3 ¼ 0. Thus, it follows that
f ð1Þ ¼ 2 sin 2γ gðγ Þ > 0:
Note that, for 0 < x  1, f(x)  min( f(0), f(1)) > 0. Hence, we obtain that
pffiffiffi
sin 2αþ sin 2β þsin 2γ  2 3ð cos α cos β þ cos β cos γ þ cos γ cos αÞ ¼
αβ αβ
¼ f cos > 0, as 0 < cos  1:
2 2

This ends the proof.


5.1.21. Let α  β  γ. Let us denote α  β ¼ 2y, β  γ ¼ 2x. We have that x  0,
y  0 and π2 ¼ α2 þ β2 þ 2γ ¼ 32γ þ 2x þ y. Therefore, 2x þ y  π2 and x  π4.
Then

jα  β j jβ  γ j jγ  αj π  π 
sin þ sin þ sin  sin  2x þ sin x þ sin x ¼
2 2 2 2 2
pffiffiffiffiffiffiffiffiffiffiffiffiffiffiffiffiffiffiffiffi pffiffiffiffiffiffiffiffiffiffiffiffiffiffiffiffiffiffiffiffiffi
¼ sin x þ cos x þ cos 2x ¼ 1 þ sin 2x þ 1  sin 2x: 2

pffiffiffiffiffiffiffiffiffiffi pffiffiffiffiffiffiffiffiffiffiffiffi
Consider the function f ðtÞ ¼ 1 þ t þ 1  t2 on the interval [0; 1]. Since
pffiffiffiffiffiffi
f 0 ðtÞ ¼ 2p1ffiffiffiffiffi
1þt
ffi  pffiffiffiffiffiffiffi
t
1t2
¼ 2p 1t 2t
ffiffiffiffiffiffiffi
1t2
¼  2pffiffiffiffiffiffiffi4t2 þt1
pffiffiffiffiffiffi
1t2 ð 1tþ2tÞ
.
pffiffiffiffi  q ffiffiffiffiffiffiffiffiffiffi
pffiffiffiffiffi qffiffiffiffiffiffiffiffiffiffiffiffi
pffiffiffiffiffi
171
Consequently, max f ðtÞ ¼ f 8 ¼ 7þ 17
8 þ 23þ 17
32 .
½0;1
qffiffiffiffiffiffiffiffiffiffipffiffiffiffiffi qffiffiffiffiffiffiffiffiffiffiffiffi
pffiffiffiffiffi qffiffiffiffiffiffiffiffiffiffiffiffiffiffiffi
pffiffiffiffiffi
Thus, sin jαβ 2
j
þ sin jβγ j
2 þ sin jγαj
2  7þ 17
8 þ 23þ 17
32 ¼ 71þ17 17
32 .
pffiffiffiffi
Remark The given estimate is exact since at β  arcsin 1781, we have that
qffiffiffiffiffiffiffiffiffiffiffiffiffiffiffi
pffiffiffiffiffi
sin jαβ
2
j
þ sin jβγ j
2 þ sin jγαj
2  71þ17 17
32 .
Note that
sffiffiffiffiffiffiffiffiffiffiffiffiffiffiffiffiffiffiffiffiffiffiffiffi
pffiffiffiffiffiffi
jα  β j jβ  γ j jγ  αj 71 þ 17 17
sin þ sin þ sin < :
2 2 2 32

This ends the proof.


5.1.22. (a) We have that

x2 þ y2 þ z2  2xy cos α  2yzcosβ  2zx cos γ ¼


¼ ðx  ðy cos α þ z cos γ ÞÞ2 þ ðy sin α  z sin γ Þ2  0

(b) Let x, y, z > 0 and 2xy ¼ sin1α1 , 2yz ¼ sin1β , 2xz ¼ sin1 γ , this means that
1 1
sin β1 sin γ 1 sin α1
x ¼ pffiffiffiffiffiffiffiffiffiffiffiffiffiffiffiffiffiffiffiffiffiffiffiffiffiffi
ffi, y ¼ pffiffiffiffiffiffiffiffiffiffiffiffiffiffiffiffiffiffiffiffiffiffiffiffiffiffi
ffi, z ¼ pffiffiffiffiffiffiffiffiffiffiffiffiffiffiffiffiffiffiffiffiffiffiffiffiffiffi
ffi. Then from the
2 sin α1 sin β1 sin γ 1 2 sin α1 sin β1 sin γ 1 2 sin α1 sin β1 sin γ 1
problem 5.1.22a, it follows that
5.1 Inequalities for the Angles of a Triangle 181

cos α cos β cos γ


þ þ ¼ 2xy cos α þ 2yz cos β þ 2xz cos γ  x2 þ y2 þ z2 ¼
sin α1 sin β1 sin γ 1
 
1 sin β1 sin γ 1 sin α1
¼ þ þ ¼
2 sin α1 sin γ 1 sin α1 sin β1 sin γ 1 sin β1
1
¼ ððctg α1 þ ctg γ 1 Þ þ ðctg α1 þ ctg β1 Þ þ ðctg γ 1 þ ctg β1 ÞÞ
2
¼ ctg α1 þ ctg β1 þ ctg γ 1 :

(c) Let the angles and circumradius of the triangle with sides a, b, c be equal to u, v,
w and R, respectively. Then

1 1 1 1 
S ¼ R2 sin 2u þ R2 sin 2v þ R2 sin 2w ¼ a2 ctgu þ b2 ctgv þ c2 ctgw :
2 2 2 4
Therefore,

a2 ctg α1 þ b2 ctg β1 þ c2 ctg γ 1  4S ¼


¼ a2 ðctg α1  ctguÞ þ b2 ðctg β1  ctgvÞ þ c2 ðctg γ 1  ctgwÞ ¼
 
sin ðu  α1 Þ sin u sin ðv  β1 Þ sin v sin ðw  γ 1 Þ sin w
¼ 4R2 þ þ ¼
sin α1 sin β1 sin γ 1
 
cos ð2u  α1 Þ cos ð2v  β1 Þ cosð2w  γ 1 Þ
¼ 2R2 ctg α1  þ ctg β1  þ ctg γ 1  ¼
sin α1 sin β1 sin γ 1
 
cosα cosβ cosγ
¼ 2R2 ctg α1 þ ctg β1 þ ctg γ 1    0
sin α1 sin β1 cos γ 1

(see the problem 5.1.22b),


since α ¼ 2u  α1, β ¼ 2v  β1, γ ¼ 2w  γ 1 and α þ β þ γ ¼ π.
(d) It is sufficient to prove the inequality for 0  φ  2π. For φ ¼ 0 or π the
inequality is obvious. Using the inequality 5.1.22c, for the angles given
below, we obtain the required inequality:
If 0 < φ < π, then we take γ 1 ¼ β1 ¼ π2  φ2 , α1 ¼ φ.
If π < φ < 2π, then we take γ 1 ¼ β1 ¼ φπ 2 and α1 ¼ 2π  φ.
 
(e) Note that a2 b21 þ c21  a21 ¼ a2 2b1 c1 cos α1 ¼ 4S1 a2 ctg α1 . Therefore,
     
a2 b21 þ c21  a21 þ b2 a21 þ c21  b21 þ c2 a21 þ b21  c21 ¼
 2 
¼ 4S1 a ctg α1 þ b2 ctg β1 þ c2 ctg γ 1  16SS1 ,

(see the problem 5.1.22c).


182 5 Application of Trigonometric Inequalities

(f) If xy þ yz þ zx ¼ 0, then the proof is obvious.


If xy þ yz þ zx 6¼ 0, then without loss of generality one can assume that xy þ yz þ
zx ¼ 1 and x  0, y  0.
We need to prove that (xa2 þ yb2 þ zc2)2  16S2. 
Let x ¼ ctgα1, y ¼ ctgβ1, z ¼ ctgγ 1, where α1 , β1 2 0; π2 , γ 1 2 ð0; π Þ.
Then, we have that ctgα1ctgβ1 þ ctgβ1ctgγ 1 þ ctgγ 1ctgα1 ¼ 1. Therefore
α1 ctg β1
ctg γ 1 ¼ 1ctg
ctg α1 þctg β1 or ctgγ 1 ¼ ctg(π  α1  β1). Thus, it follows that α1, β1, γ 1 are
the angles of some triangle.
According to problem 5.1.22c, we obtain that xa2 þ yb2 þ zc2  4S.
Hence, we deduce that
 2
xa2 þ yb2 þ zc2  16S2

(g) Let points A1, B1, and C1 be the midpoints of sides BC, AC, and AB, respec-
tively. Using the problem 4.1.8а for x ¼ tgα1, y ¼ tgβ1, z ¼ tgγ 1, we obtain that
tg α tg β A B2 þtg β1 tg γ 1 B1 C21 þtg α1 tg γ 1 A1 C21
tg α1  OA21 þ tg β1  OB21 þ tg γ 1  OC21  1 1 1 1 tg α1 þtg β þtg γ ,
1 1
2 2 2
or k2 tg α1 þ l2 tg β1 þ m2 tg γ 1  ctg γ 1  c4 þ ctg α1  a4 þ ctg β1  b4 , as
tgα1 þ tgβ1 þ tgγ 1 ¼ tgα1tgβ1tgγ 1.

According to problem 5.1.22c, it follows that 14 a2 ctg α1 þ b2 ctg β1 þ
c2 ctg γ 1 Þ  S.
Therefore, k2tgα1 þ l2tgβ1 þ m2tgγ 1  S.
This ends the proof.
5.1.23. In any triangle there are angles α and β, such that either α2 , β2  π6 or α2 , β2  π6,
  
then 1  2 sin α2 1  2 sin β2  0. Thus, it follows that
 2
α β γ α β γ
cos 2 þ cos 2 þ cos 2  sin þ sin þ sin ¼
2 2 2 2 2 2
 
α β γ α β
¼ cos α þ cos β þ cos γ  2 sin sin  2 sin sin þ sin ¼
2 2 2 2 2
 
α β γ α β γ α β
¼ 1 þ 4 sin sin sin  2 sin sin  2 sin sin þ sin ¼
2 2 2 2 2 2 2 2
 
γ α β α β γ
¼ 1 þ sin 1  2 sin 1  2 sin  2 sin sin  sin ¼
2 2 2 2 2 2
 
αβ γ α β
¼ 1  cos þ sin 1  2 sin 1  2 sin  0,
2 2 2 2

(see the proof of the problem 5.1.4a). Hence, we obtain that


5.1 Inequalities for the Angles of a Triangle 183

 2
α β γ α β γ
cos 2 þ cos 2 þ cos 2  sin þ sin þ sin :
2 2 2 2 2 2

This ends the proof.


5.1.24. We have that (see the problem 5.1.15)

sin 2α þ sin 2β þ sin 2γ  sin α þ sin β þ sin γ: ð5:2Þ


πβ πγ
Using (5.2) for angles πα
2 , 2 , 2 , we obtain that

α β γ
sin α þ sin β þ sin γ  cos þ cos þ cos : ð5:3Þ
2 2 2
In the same way from (5.3), we deduce that
π α   π γ 
α β γ π β
cos þ cos þ cos  cos  þ cos  þ cos  : ð5:4Þ
2 2 2 4 4 4 4 4 4

From the last inequality, in the same way, it follows that


π  
α π β π γ  πþα πþβ πþγ
cos  þ cos  þ cos   cos þ cos þ cos :
4 4 4 4 4 4 8 8 8
ð5:5Þ
From inequalities (5.2)–(5.5), we deduce that
πþα πþβ πþγ
sin 2α þ sin 2β þ sin 2γ  cos þ cos þ cos :
8 8 8
This ends the proof.

5.1.25. If γ < 90 , then cos 2 α þ cos 2 β þ cos γ > cos 2 α þ cos 2 β þ cos 2 γ  34, (see
the problem 5.1.1).

In the case γ  90 , we have that

cos 2 α þ cos 2 β þ cos γ ¼ 1 þ cos ðα þ βÞ cos ðα  βÞ


 
3 1 2 3
þ cos γ  1 þ cos 2 ðα þ βÞ þ cos γ ¼¼ þ cos γ þ  :
4 2 4

Here the equality cannot hold true. Otherwise, we obtain that α þ β ¼ 90 and

γ ¼ 120 . This leads to a contradiction.
This ends the proof.
184 5 Application of Trigonometric Inequalities

5.1.26. We have that

cos 2 α cos 2 β þ cos 2 β cos 2 γ þ cos 2 γ cos 2 α ¼


qffiffiffiffiffiffiffiffiffiffiffiffiffiffiffiffiffiffiffiffiffiffiffiffiffiffiffiffiffiffiffiffiffiffiffiffiffiffiffiffiffiffiffiffiffiffiffiffiffiffiffiffiffiffiffiffiffiffiffiffiffiffiffiffiffiffiffiffiffiffiffiffiffiffiffiffiffiffiffiffiffiffiffiffiffiffiffiffiffiffiffiffiffiffiffiffiffiffiffiffiffiffiffiffi
¼ ðcos 2 α cos 2 βÞ2 þ ðcos 2 β cos 2 γ Þ2 þ ðcos 2 γ cos 2 αÞ2 þ

þ2cos 2 α cos 2 β cos 2 γ ðcos 2 α þ cos 2 β þ cos 2 γ Þ 


pffiffiffiffiffiffiffiffiffiffiffiffiffiffiffiffiffiffiffiffiffiffiffiffiffiffiffiffiffiffiffiffiffiffiffiffiffiffiffiffiffiffiffiffiffiffiffiffiffiffiffiffiffiffiffiffiffiffiffiffiffiffiffiffiffiffiffiffiffiffiffiffiffiffiffiffiffiffiffiffiffiffiffiffiffiffiffiffiffiffi 3
 3 cos 2 α cos 2 β cos 2 γ ðcos 2 α þ cos 2 β þ cos 2 γ Þ  cos α cos β cos γ:
2
Here, we have used the inequality x2 þ y2 þ z2  xy þ yz þ zx and the
problem 5.1.1.
This ends the proof.
5.1.27. Note that
qffiffiffiffiffiffiffiffiffiffiffiffiffiffiffiffiffiffiffiffiffiffiffiffiffiffiffiffiffiffiffiffiffiffiffiffiffiffiffiffiffiffiffiffiffiffiffiffiffiffiffiffiffiffiffiffiffiffiffiffiffiffiffiffiffiffiffiffiffiffiffiffiffiffiffi
pffiffiffiffiffiffiffiffiffiffiffiffiffiffiffiffiffiffiffiffiffiffiffiffiffiffiffiffiffiffiffiffiffiffiffiffiffi
x2 þ y2 þ 2xy cos α ðx sin γ þ y sin βÞ2 þ ðx cos γ  y cos βÞ2 sin γ sin β
¼ x þy :
sin α sin α sin α sin α
pffiffiffiffiffiffiffiffiffiffiffiffiffiffiffiffiffiffiffiffiffiffiffi
y2 þz2 þ2yz cos β α sin γ
Similarly, we get the inequalities:  y sin
sin β þ z sin β and
pffiffiffiffiffiffiffiffiffiffiffiffiffiffiffiffiffiffiffiffiffiffiffi sin β
z2 þx2 þ2zx cos γ β sin α
sin γ  z sin
sin γ þ x sin γ . By summing up these inequalities, we get
ffi pffiffiffiffiffiffiffiffiffiffiffiffiffiffiffiffiffiffiffiffiffiffiffiffiffiffiffiffiffiffiffiffiffiffiffiffiffi
pffiffiffiffiffiffiffiffiffiffiffiffiffiffiffiffiffiffiffiffiffiffiffiffiffiffiffiffiffiffiffiffiffiffiffiffiffiffi pffiffiffiffiffiffiffiffiffiffiffiffiffiffiffiffiffiffiffiffiffiffiffiffiffiffiffiffiffiffiffiffiffiffiffiffi
x2 þ y2 þ 2xy cos α þ y þ z þ 2yz cos β þ z þ x þ 2zx cos γ 
2 2 2 2

sin α sin β sin γ


     
sin γ sin α sin β sin α sin γ sin β
x þ þy þ þz þ  2x þ 2y þ 2z,
sin α sin γ sin α sin β sin β sin γ

as for a > 0, we have that a þ 1a  2.


This ends the proof.
5.1.28. Let α  β  γ, then we have that
2ð sin α sin β þ sin β sin γ þ sin γ sin αÞ  3ð cos α þ cos β þ cos γ Þ ¼
βþγ γβ γβ βþγ
¼ 4 sin α sin cos þ cos ðγ  βÞ  cos ðγ þ βÞ  3 cos α  6 cos cos ¼
2 2 2 2
γβ γβ  α α 
¼ 2cos 2 þ 2 cos 2 sin α cos  3 sin  2 cos α  1:
2 2 2 2
Since 0  γβ γ π
2 < 2 < 2, then 0 < cos
γβ
2  1.
 
Consider the function f ðxÞ ¼ 2x2 þ 2x 2 sin α cos α2  3 sin α2  2 cos α  1 in
the interval [0; 1]. Note that max f ðxÞ ¼ maxðf ð0Þ; f ð1ÞÞ. We need to prove that, if
 ½0;1 γβ
f(0)  0 and f(1)  0, then f cos 2  0.  
Indeed, we have that f(0) ¼  1  2 cos α < 0, 0 < α  π3 .
5.1 Inequalities for the Angles of a Triangle 185

   2  
α 3α α α α
f ð1Þ ¼ 1  2 cos α þ 2 sin þ sin  3 sin ¼  2 sin  1 2 sin þ 1  0:
2 2 2 2 2
This ends the proof.
5.1.29. We need to prove that, if 0 < x, y < π3, then

sin 2x sin 2y sin 2 ðx þ yÞ


 : ð5:6Þ
sin 3x sin 3y sin 2 3xþ3y
2

Let x  y, denote x  y ¼ α, x þ y ¼ β, then 0  α < β < 2π 3 . Therefore,


cos α2 > cos β2. We have to prove that cos 2α cos 2β
cos 3α cos 3β  1 cos 2β
1 cos 3β , or (cos2α  cos 2β)
(1  cos 3β)  (cos3α  cos 3β)(1  cos 2β).
Note that
ð cos 2α  cos 2βÞð1  cos 3βÞ  ð cos 3α  cos 3βÞð1  cos 2βÞ ¼
 
3α 3β
¼ 4 sin 2 βsin 2  sin 2 αsin 2 ¼
2 2
   
α β α β α β 3α 3β
¼ 8 sin sin 4 cos cos þ 1 cos  cos sin β sin þ sin α sin  0:
2 2 2 2 2 2 2 2

Using the inequality (5.6), we obtain that


 
 π
α β γ π 2 α β
2 γ
sin 2 sin 2 sin 2 sin 2 sin þ sin þ
6 6 6 18  6 6
  6γ 18π  
α β sin 3γ sin 3 π α β 2
sin 3 þ
sin 3 sin 3 18 sin 3 12 þ 12
2
6 6 6 12 36
 
4 α β γ π π
sin þ þ þ sin 4
12 12 12 36 9
 ¼ π:
α β γ π 4
þ þ þ sin
6
sin 4 3 12 12 12 36
2
Therefore, sin α3 sin β3 sin 3γ  8sin 3π9 sin α2 sin β2 sin 2γ .
This ends the proof.
 2
5.1.30. We have that tg 2α2 þ tg 2 β2 þ tg 2 2γ ¼ tg α2 þ tg β2 þ tg 2γ  2, (see the proof
of the problem 5.2.1b. Note that
αþγ αþγ
α γ 2 sin 2 sin
tg þ tg ¼ 2  2 ¼
2 2 cos γ  α þ cos γ þ α 2φ  ðγ þ α  2φÞ γþα
cos þ cos
2 2 2 2
γ þ α  2φ 2φ
¼ tg þ tg ,
2 2
2φðγþα2φÞ
since αγ
2  2  γα π
2 , where φ ¼ arctg3, φ > 4. Thus
4
186 5 Application of Trigonometric Inequalities

α β γ β γ þ α  2φ 4
tg þ tg þ tg  tg þ tg þ ¼
2 2 2 2 2 3
π 
2 sin φ 4 2 cos φ 4
¼  2  þ  þ ¼ 2:
β γ þ α  2φ  π  3 1 þ sin φ 3
cos  þ cos φ
2 2 2

Therefore, tg 2α2 þ tg 2 β2 þ tg 2 2γ  2.
This ends the proof.
5.1.31. We have that

sin α sin α1 þ sin β sin β1 þ sin γ sin γ 1


¼
α β γ α1 β γ
sin sin sin sin sin 1 sin 1
2 2 2 2 2 2
0 1
α α1 β β γ γ
cos cos cos cos 1 cos cos 1
B 2 2 2 2 2 2 C
¼ 4@ þ þ A
β β1 γ γ 1 sin α sin α1 sin γ sin γ 1 β β1 α α1
sin sin sin sin 2 2 2 2 sin sin sin sin
2 2 2 2 2 2 2 2
0
α α1 β β
B cos cos cos cos 1
 16B 2 2 þ 2 2
@ βþγ β1 þ γ 1 α þ γ  α1 þ γ 1  þ
1  cos 1  cos 1  cos 1  cos
2 2 2 2
1
γ γ
cos cos 1 C
þ  2
2 C ¼
βþα β 1 þ α1 A
1  cos 1  cos
2 2
 
π þ α π þ α1 π þ β π þ β1 π þ γ π þ γ1
¼ 16 tg tg þ tg tg þ tg tg 
4 4 4 4 4 4
rffiffiffiffiffiffiffiffiffiffiffiffiffiffiffiffiffiffiffiffiffiffiffiffiffiffiffiffiffiffiffiffiffiffiffiffiffiffiffiffiffiffiffiffiffiffiffiffiffiffiffiffiffiffiffiffiffiffiffiffiffiffiffiffiffiffiffiffiffiffiffiffiffiffiffiffiffiffiffiffiffiffiffiffiffiffiffiffiffiffiffiffiffiffiffiffiffiffiffi
π þ α π þ α1 π þ β π þ β 1 π þ γ π þ γ 1 p pffiffiffi pffiffiffi
ffiffiffiffiffiffiffiffiffiffiffiffiffiffiffiffiffiffiffiffi
 48 tg  48 3 3  3 3¼ 144
3 3
tg tg tg tg tg
4 4 4 4 4 4

Since πþα πþβ πþγ


4 þ 4 þ 4 ¼ π,
πþα1
4 þ πþβ πþγ 1
4 þ 4 ¼ π, (see the proof of the prob-
1

lems 5.2.1a and 5.2.9).


This ends the proof.
5.1.32. Without loss of generality, we can assume that α  β  γ. The expression
cosα1 þ cos β1 þ cos γ 1 is symmetric with respect to α1, β1, γ 1, and the value of the
0 0 0 0 0 0
0 sin0 α sin
expression
0
α1 þ sin β sin β1 þ sin γ sin γ 1 is maximal, when α1  β1  γ 1 ,
where α1 ; β1 ; γ 1 ¼ fα1 ; β1 ; γ 1 g. Therefore, without loss of generality, we can
assume that α1  β1  γ 1. pffiffiffiffiffiffiffiffiffiffiffiffiffiffiffiffiffiffiffiffiffiffiffiffiffiffiffiffiffiffiffiffiffiffiffiffiffiffiffiffiffiffiffiffiffiffiffiffiffiffiffiffiffiffiffiffiffiffiffiffiffiffiffiffiffi
Since sin α sin α1 þ sin β sin β1  ðsin 2 α þ sin 2 βÞðsin 2 α1 þ sin 2 β1 Þ, then it is
sufficient to prove that
pffiffiffiffiffiffiffiffiffiffiffiffiffiffiffiffiffiffiffiffiffiffiffiffiffiffiffipffiffiffiffiffiffiffiffiffiffiffiffiffiffiffiffiffiffiffiffiffiffiffiffiffiffiffiffiffiffiffi
sin 2 α þ sin 2 β sin 2 α1 þ sin 2 β1 þ sin γ sin γ 1 
ð5:7Þ
 2 þ ð cos α þ cos β þ cos γ  1Þð cos α1 þ cos β1 þ cos γ 1  1Þ:
5.1 Inequalities for the Angles of a Triangle 187

Note that

cos 2α þ cos 2β
sin 2 α þ sin 2 β ¼ 1  ¼ 1 þ cos γ cos ðα  βÞ
2
αβ
¼ 2 cos γcos 2 þ 1  cos γ
and 2

γ αβ
cos α þ cos β ¼ 2 sin cos :
2 2

We have to prove that


 
αβ
2 cos γcos 2 þ 1  cos γ ðsin 2 α1 þ sin 2 β1 Þ 
2

γ αβ
 2 þ 2 sin ð cos α1 þ cos β1 þ cos γ 1  1Þ cos þ
2 2
þð cos γ  1Þð cos α1 þ cos β1 þ cos γ 1  1Þ   sin γ sin γ 1 Þ2 :

The last inequality can be rewritten as:


 γ  αβ
2 cos γ ðsin 2 α1 þ sin 2 β1 Þ  4sin 2 ð cos α1 þ cos β1 þ cos γ 1  1Þ2 cos 2 þ
2 2
αβ
þB cos þ C  0:
2
ð5:8Þ

We need to prove that

sin 2 α1 þ sin 2 β1  ð cos α1 þ cos β1 þ cos γ 1  1Þ2 : ð5:9Þ

Indeed, we have that

sin 2 α1 þ sin 2 β1  ð cos α1 þ cos β1 þ cos γ 1  1Þ2 ¼ 1 þ cos γ 1 cos ðα1  β1 Þ


 2  
2 γ1 α1  β1
 2 sin γ21 cos α1 β
2
1
þ cos γ 1  1 ¼ 2 cos γ 1  4sin cos 2 þ
2 2
γ α1  β1
þ 4ð1  cos γ 1 Þ sin 1 cos þ cos γ 1 ð1  cos γ 1 Þ  0:
2 2
As γ 1  π3, then 2 cos γ 1  1  4sin 2 γ21 and 4ð1  cos γ 1 Þ sin γ21 cos α1 β1
2  0,
cosγ 1(1  cos γ 1)  0.
According to (5.9) and 2 cos γ  4sin 2 2γ , we obtain that
A ¼ 2 cos γ ðsin 2 α1 þ sin 2 β1 Þ  4sin 2 2γ ð cos α1 þ cos β1 þ cos γ 1  1Þ2  0.
188 5 Application of Trigonometric Inequalities

γ αþβ αβ
Let f(x) ¼ Ax2 þ  þ γC.
 Bx  Since sin 2 ¼ cos 2  cos 2  1 and A  0, then
max f ðxÞ ¼ max f sin 2 ; f ð1Þ ; thus, if we prove that the inequality (5.7)
½ sin 2γ ;1
holds true for cos αβ γ αβ
2 ¼ sin 2 and cos 2 ¼ 1, then it holds true (in the
general case).
If cos αβ γ
2 ¼ sin 2, then (5.7) has the following form:
pffiffiffiffiffiffiffiffiffiffiffiffiffiffiffiffiffiffiffiffipffiffiffiffiffiffiffiffiffiffiffiffiffiffiffiffiffiffiffiffiffiffiffiffiffiffiffiffiffiffiffiffiffiffiffiffiffiffiffiffiffiffiffiffiffiffiffi
1  cos 2 γ 1 þ cos γ 1 cos ðα1  β1 Þ þ sin γ sin γ 1  2

or
pffiffiffiffiffiffiffiffiffiffiffiffiffiffiffiffiffiffiffiffiffiffiffiffiffiffiffiffiffiffiffiffiffiffiffiffiffiffiffiffiffiffiffiffiffiffiffi
sin γ  1 þ cos γ 1 cos ðα1  β1 Þ þ sin γ sin γ 1  2:
pffiffi

pffiffiffiSince sin γ  3, cos(α1  β1)  1, then it is sufficient to prove that


pffiffiffiffiffiffiffiffiffiffiffiffiffiffiffiffiffiffiffiffiffi p2ffiffiffi
þ cos γ 1 þ p3ffiffiffi sin γ 1  4,
p3ffiffiffip1ffiffiffiffiffiffiffiffiffiffiffiffiffiffiffiffiffiffiffiffi pffiffiffi but the last inequality holds true, as

3 1 þ cos γ 1 þ 3 sin γ 1  6 þ 32 < 4.
If cos αβ 2 ¼ 1, then (5.7) has the form:
ffiffiffiffiffiffiffiffiffiffiffiffiffiffiffiffiffiffiffiffiffiffiffiffiffiffiffiffiffiffiffiffiffiffiffiffiffiffiffiffiffiffiffiffiffiffiffiffiffiffiffiffiffiffiffiffiffiffiffiffiffiffiffiffiffiffiffiffiffiffiffiffiffiffiffiffiffiffiffiffiffiffiffiffi
r
πγ π  γ
sin 2 þ sin 2 ðsin 2 α1 þ sin 2 β1 Þ þ sin γ sin γ 1 
2 2
 πγ πγ 
 2 þ cos þ cos þ cos γ  1 ð cos α1 þ cos β1 þ cos γ 1  1Þ:
2 2
ð5:10Þ

Similarly, to the proof given above in this case too it is sufficient to prove the
inequality (5.10) for cos α1 β π
2 ¼ 1. Then under the condition γ, γ 1  3 one has to
1

prove the inequality


pffiffiffiffiffiffiffiffiffiffiffiffiffiffiffiffiffiffiffiffiffiffiffiffiffiffiffiffiffiffiffiffiffiffiffiffiffiffiffiffiffiffiffiffiffiffiffi
ð1 þ cos γ Þð1 þ cos γ 1 Þ þ sin γ sin γ 1 
 γ  γ 
 2 þ 2 sin þ cos γ  1 2 sin 1 þ cos γ 1  1 ,
2 2
or
γ γ γ γ  γ  γ
2 cos cos 1 þ sin γ sin γ 1  2 þ 4 sin sin 1 1  sin 1 1  sin : ð5:11Þ
2 2 2 2 2 2

Indeed, we have that


γ γ γ γ  γ  γ γ  γ1
D ¼ 2 cos cos 1 þ sin γ sin γ 1  4 sin sin 1 1  sin 1 1  sin ¼ cos þ
2 2 2 2 2 2 2
γ þ γ1 γ γ  γ γ γ γ γ γ 
þ cos þ 4 sin sin 1 cos cos 1  1 þ sin 1 þ sin  sin 1 sin ¼
2 2 2 2 2 2 2 2 2
γ  γ1 γ þ γ1 γ γ  γ γ  γ þ γ1 γ  γ1 
¼ cos þ cos þ 4 sin sin 1 cos þ 1  1 þ 2 sin cos :
2 2 2 2 2 2 4 4
γ 
If cos 2 þ γ21  1 þ 2 sin γþγ 1
4 cos γγ 1
4  0, then D  cos γγ 1
2 þ cos γþγ 1
2  2.
5.1 Inequalities for the Angles of a Triangle 189

γ 
If cos 2 þ γ21  1 þ 2 sin γþγ 1
4 cos γγ 1
4 > 0, then

γ  γ1 γ þ γ1 γ γ  γ þ γ1 γ þ γ1 γ  γ1
D ¼ cos þ cos þ 4 sin sin 1 cos  1 þ 2 sin cos 
2 2 2 2 2 4 4
γ þ γ1 γ γ  γ þ γ1 γ þ γ1 
 1 þ cos þ 4 sin sin 1 cos  1 þ 2 sin 
2 2 2 2 4
γ þ γ1  γ þ γ1   γ þ γ1 γ þ γ1
 1 þ cos þ 2 1  cos cos  1 þ 2 sin ¼ D0 :
2 2 2 4
 2
Then D0 ¼ 2cos2 γ20 þ sin 2 γ 0  4sin 2 γ20 1  sin γ20 , where γ 0 ¼ γþγ
2 .
1

2 γ0
 
γ0 2
As 2  D0 ¼ 2sin 2 1  2 sin 2  0, then D0  2. Hence, D  2.
This ends the proof of (5.11).

Problems for Self-Study

Prove the inequalities 5.1.33–5.1.52. qffiffi


pffiffiffiffiffiffiffiffiffiffi pffiffiffiffiffiffiffiffiffiffi pffiffiffiffiffiffiffiffiffiffi
5.1.33. sin α þ sin β þ sin γ  3 4 34.
sin αþ sin βþ sin γ
5.1.34. sin α sin β sin γ  4.
pffiffi
5.1.35. 2  sin 3α þ sin 3β þ sin 3γ  3 2 3.
pffiffiffi
5.1.36. sin 2 α þ sin 2 β þ sin 2 γ  2 3 sin α sin β sin γ.
pffiffiffi
5.1.37. 3sin 2 α þ 3sin 2 β  sin 2 γ  2 3 sin α sin β sin γ.
5.1.38. 2 sin α sin β þ 2 sin β sin γ þ 2 sin γ sin α  sin 2 α  sin 2 β  sin 2 γ 
pffiffiffi
 2 3 sin α sin β sin γ:
qffiffiffiffiffiffiffiffiffiffiffiffiffiffiffiffiffiffiffiffiffiffiffiffiffiffiffiffiffiffiffi
5.1.39. sin α þ sin β þ sin γ  3 6 sin α2 sin β2 sin 2γ .

5.1.40. (sin2β þ sin2γ)(1  cos φ) þ sin2α cos φ  2 sin α sin β sin γ sin φ, where φ is
any angle.
5.1.41. sin2α þ sin2β þ sin2γ < 2(sinα sin β þ sin β sin γ þ sin γ sin α).
     
5.1.42. sin α2 þ β þ sin β2 þ γ þ sin 2γ þ α > sin α þ sin β þ sin γ.
5.1.43. (a) 1
sin α2 sin β2
þ 1
sin β2 sin 2γ
þ 1
sin α2 sin 2γ
 12,
pffiffiffi
cos α2 þ cos β þ cos 2γ  2 3,
1 1 1
(b)
2

sin α2 þ sin β þ sin 2γ  6,


1 1 1
(c)
2

sin α þ sin β þ sin γ  4 cos α cos β cos γ .


1 1 1 9
(d)
2 2 2

5.1.44. ctgα  ctgβ  ctgγ  tg α2  tg β2  tg 2γ .


190 5 Application of Trigonometric Inequalities

pffiffi
5.1.45. sin α  sin β  sin 2γ  2 9 3.
5.1.46. ctg2β  ctgαctgγ, if 2sin2β ¼ sin2α þ sin2γ.

5.1.47. (a) cos α cos β cos ðα  βÞ þ cos β cos γ cos ðβ  γ Þ þ cos γ cos α cos ðγ  αÞ 
 1  cos α cos β cos γ,
pffiffi
(b) sin α4 sin β4 sin 4γ þ cos α4 cos β4 cos 4γ  3 8 6,
α β γ
 α β β γ γ α

4  sin 2 þ sin 2 þ sin 2 k sin 2 sin 2 þ sin 2 sin 2 þ sin 2 sin2 , where k  3,
(c) 63k 2
β γ β β γ γ
(d) sin α2 þ sin 2 þ sin 2  k sin α2 sin 2 þ sin 2 sin 2 þ sin 2 sin α2  63k 4 , where 0 <
k  27,
 
(e) sin α2 þ sin β2 þ sin 2γ þ k sin α2 sin β2 þ sin β2 sin 2γ þ sin 2γ sin α2  sin α2 sin β2 sin 2γ 
 32 þ 5k
8 , where k  5,
4
 
(f) 7 k α β γ α β β γ γ α where
þ  sin þ sin þ sin þ k sin sin þ sin sin þ sin sin 
5 2 2 2 2 2 2 2 2 2 2
 
4 α β γ
 þ 2k sin sin sin
5 2 2 2
k  45.
qffiffiffiffiffiffiffiffiffiffiffiffiffiffiffiffiffiffiffiffiffiffiffiffiffiffiffiffiffiffiffiffiffiffiffiffiffiffiffiffiffiffiffiffiffiffiffiffiffiffiffiffiffiffiffiffiffiffiffiffiffiffiffiffiffiffiffiffiffiffiffiffiffiffiffiffiffiffiffiffiffiffiffiffiffiffiffiffi
5.1.48. (a) sin α þ sin β þ sin γ  15 4 þ cos ðα  βÞ þ cos ðβ  γ Þ þ cos ðγ  αÞ,

(b) 18  cos ðα  βÞ cos ðβ  γ Þ cos ðγ  αÞ  1,


pffiffi pffiffi
(c)  3 8 3  sin ðα  βÞ sin ðβ  γ Þ sin ðγ  αÞ  3 8 3.
Hint (c) If sin(β  γ)  0, then sin ðα  βÞ sin ðβ  γ Þ sin ðγ  αÞ  12 sin ðβ  γ Þ
ð1  cos ðβ  γ ÞÞ, while at sin(β  γ) < 0 we have sin ðα  βÞ sin ðβ  γ Þ
sin ðγ  αÞ  sin ð2γβÞ ð1 þ cos ðβ  γ ÞÞ.
5.1.49. (a) cos(α  β) cos(β  γ) cos(γ  α)  8 cos α cos βcosγ,
(b) cos2(α  β) þ cos2(β  γ) þ cos2(γ  α)  24 cos α cos βcosγ.
 
γ 3
5.1.50. cos 2 αβ
2 cos
2 βγ
2 cos
2 γα α β
2  8 sin 2 sin 2 sin 2 .

Hint See the problems 5.5.1b and 5.3.4.


 
5.1.51. 8 sin α2 sin β2 sin 2γ  min cos 2 αβ
2 ; cos
2 βγ
2 ; cos
2 γα
2 .

5.1.52. sin2α þ sin2β þ sin2γ  2 þ (cosα þ cos β þ cos γ  1)2.


5.1.53. Find the smallest value of the expression

cos ð2α þ βÞ þ cos ð2β þ γ Þþ


þ cos ð2γ þ αÞ  cos 2α  cos 2β  cos 2γ þ cos α þ cos β þ cos γ:
5.2 Inequalities for the Angles of Acute and Obtuse Triangles 191

5.2 Inequalities for the Angles of Acute


and Obtuse Triangles

Let α, β, and γ be the angles of some acute triangle. Prove the inequalities of the
problems 5.2.1–5.2.8.
pffiffiffi
5.2.1. (a) tgα þ tgβ þ tgγ  3 3,
(b) tg 2α2 þ tg 2 β2 þ tg 2 2γ < 2.
5.2.2. (a) tgα þ tgβ þ tgγ  ctg α2 þ ctg β2 þ ctg 2γ ,

(b) tgα  tgβ  tgγ  ctg α2 ctg β2 ctg 2γ .


5.2.3. (a) sinα þ sin β þ sin γ > 2,
(b) cos α2 cos β2 cos 2γ > 12.
5.2.4. cos(α  β) cos(β  γ) cos(γ  α)  8 cos α cos β cos γ.
5.2.5. (4 cos α þ 1)2 þ (4 cos β þ 1)2 þ (4 cos γ þ 1)2  27.
5.2.6. tg 2 α þ tg 2 β þ tg 2 γ  ctg 2α2 þ ctg 2 β2 þ ctg 2 2γ .
5.2.7. cos2α þ cos2β þ cos2γ  4cos2αcos2β þ 4cos2βcos2γ þ 4cos2γcos2α.
5.2.8. (a) sin α þ sin β þ sin γ  p2ffiffi3 ð cos α þ cos β þ cos γ Þ2 ,
pffiffiffi
(b) sin α þ sin β þ sin γ  2 3ð cos α cos β þ cos β cos γ þ cos γ cos αÞ,
(c) sin2α þ sin2β þ sin2γ  (cosα þ cos β þ cos γ)2.
Let α, β, and γ be the angles of some obtuse triangle. Prove the inequalities.
5.2.9. tgα þ tgβ þ tgγ < 0.
5.2.10. cos2α þ cos 2β  cos 2γ > 1, if γ > π2.
pffiffiffi
5.2.11. (a) sin α þ sin β þ sin γ < 1 þ 2,
pffiffiffiffiffiffiffiffiffiffi pffiffiffiffiffiffiffiffiffiffi pffiffiffiffiffiffiffiffiffiffi pffiffiffi
(b) sin α þ sin β þ sin γ < 1 þ 4 8,
(c) sin α sin β sin γ < 12.
pffiffi
5.2.12. (a) cos α2 cos β2 cos 2γ < 1þ4 2,
pffiffi
(b) sin α2 sin β2 sin 2γ < 241.
5.2.13. 1 þ cos α cos β cos γ > 2 sin α sin β sin γ.

Solutions
πβ1 πγ 1
5.2.1. (a) Note that, if α ¼ πα 2 , β ¼ 2 , γ ¼ 2 , then α1 þ β 1 þ γ 1 ¼ π and
1

α1, β1, γ 1 > 0. Hence, α1, β1, γ 1 are anglespof


ffiffiffi some triangle. By such replacement
we obtain that ctg α21 þ ctg β21 þ ctg γ21  3 3 (see the problem 5.1.10).
192 5 Application of Trigonometric Inequalities

π
Remark For α ¼ 2π
3 , β ¼ γ ¼ 6, we have that tgα þ tgβ þ tgγ ¼  3.
p1ffiffi

(b) Note that tg α2 tg β2 þ tg β2 tg 2γ þ tg α2 tg 2γ ¼ 1. Indeed,


    
α β γ β γ α β γ β γ β γ
tg tg þ tg þ tg tg ¼ tg tg þ 1  tg tg þ tg tg ¼
2 2 2 2 2 2 2 2 2 2 2 2
 
α α β γ β γ
¼ tg ctg 1  tg tg þ tg tg ¼ 1:
2 2 2 2 2 2

Then,

α β γ
tg 2 þ tg 2 þ tg 2 ¼
2 2 2
 
 2 α β β γ γ α
¼ tg α2 þ tg β2 þ tg 2γ  2 tg tg þ tg tg þ tg tg :
2 2 2 2 2 2

We need to prove that tg α2 þ tg β2 þ tg 2γ < 2. We have that


 
αþβ
2 sin
α β γ 2 γ
tg þ tg þ tg ¼ þ tg <
2 2 2 α  β α þ β 2
cos þ cos
2 2
 
αþβ
2 sin
2 γ
<    þ tg ,
90  α þ β  90 αþβ 2
cos þ cos
2 2
    αþβ
since 90  αþβ αβ
2 > 2 > 90 . This means that cos 90  2 < cos αβ
2 .
   
Thus tg α2 þ tg β2 þ tg 2γ < tg 45 þ tg αþβ90
2 þ tg  γ
2 < tg 45 þ tg 45 ¼ 2, since
  
αþβ90
tg 2 þ tg 2γ ¼ tg αþβ90 2
þγ
 1  tg αþβ90
2 tg 2γ .
 2
Therefore, tg 2α2 þ tg 2 β2 þ tg 2 2γ ¼ tg α2 þ tg β2 þ tg 2γ  2 < 2.
sin ðxþyÞ 2 sin ðxþyÞ
5.2.2. (a) If 0 < x, y < π2, then tgx þ tgy ¼ cos x cos y  1þ cos ðxþyÞ ¼ 2tg
xþy
2 .
Therefore,
 
1 αþβ βþγ αþγ γ α β
tgα þ tgβ þ tgγ  2tg þ 2tg þ 2tg ¼ ctg þ ctg þ ctg :
2 2 2 2 2 2 2

Remark If γ > π2, then tgα þ tgβ þ tgγ < 0 < ctg α2 þ ctg β2 þ ctg 2γ , (see the
problem 5.2.9).
(b) As tgα þ tgβ þ tgγ ¼ tgαtgβtgγ (see the proof of the problem 5.2.9) and
5.2 Inequalities for the Angles of Acute and Obtuse Triangles 193

α β γ πα πβ πγ πα πβ πγ


ctg þ ctg þ ctg ¼ tg þ tg þ tg ¼ tg tg tg ¼
2 2 2 2 2 2 2 2 2
α β γ
¼ ctg ctg ctg ,
2 2 2

then according to the problem 5.2.2a, we have that tgαtgβtgγ  ctg α2 ctg β2 ctg 2γ .

Remark If γ > π2, then tgαtgβtgγ < 0 < ctg α2 ctg β2 ctg 2γ ,
5.2.3. (a) Note that

sin α þ sin β þ sin γ > sin 2 α þ sin 2 β þ sin 2 γ ¼


¼ 2  cos ðα þ βÞ cos ðα  βÞ  cos 2 γ ¼ 2 þ 2 cos α cos β cos γ > 2:

pffiffi
π
Remark For γ ¼ 2π
3 , α ¼ β ¼ 6, we have that sin α þ sin β þ sin γ ¼ 1 þ 2
3
< 2:

(b) We have that sin α þ sin β þ sin γ ¼ 2 cos 2γ cos αβ


2 þ 2 sin 2γ cos 2γ ¼ 4
cos α2 cos β2 cos 2γ :
Therefore, according to the problem 5.2.3a, we deuce that cos α2 cos β2 cos 2γ > 12.
pffiffiffiffiffiffiffiffiffiffiffiffiffiffiffiffiffiffiffiffiffiffiffiffi
5.2.4. We have that sin α cosp  γ Þ ¼ 12 ð sin 2γ þ sin 2βÞ  sin 2β sin 2γ .
ðβffiffiffiffiffiffiffiffiffiffiffiffiffiffiffiffiffi
sin 2β sin 2γ
Therefore, cos ðβ  γ Þ  . Similarly, we deduce that cos ðα  βÞ 
pffiffiffiffiffiffiffiffiffiffiffiffiffiffiffiffiffiffi pffiffiffiffiffiffiffiffiffiffiffiffiffiffiffiffiffi
sin α
sin 2α sin 2β sin 2α sin 2γ
sin γ and cos ðγ  αÞ  sin β . By multiplying last three inequalities, we
obtain that cos(α  β) cos(β  γ) cos(γ  α)  8 cos α cos β cos γ.
Remark The inequality holds true for any triangle.
Indeed, let α  β < π2  γ. If γ  π2 þ α or γ  π2 þ β, then cos(α  β) cos(β  γ)
cos(γ  α)  0  8 cos α cos β cos γ. If π2 þ α < γ < π2 þ β, then 0 <  cos(γ  α) <
cos γ and 0 < cos(α  β) cos(β  γ)<cos(α  β) < 4 cos(β  α) þ 4 cos(α þ β) ¼
8 cos α cos β. Hence, cos(α  β) cos(β  γ) cos(γ  α) > 8 cos α cos β cos γ.
5.2.5. Note that
cos2α þ cos2β ¼ 1 þ cos(α þ β) cos(α  β) ¼ 1  cos γ cos(α  β)  1  cos γ.
Thus, it follows that

1 2     
cos 2 α þ cos 2 β þ cos 2 γ ¼ cos α þ cos 2 β þ cos 2 β þ cos 2 γ þ cos 2 γ þ cos 2 α 
2
1
 ð3  cos α  cos β cos γ Þ:
2

Therefore,

ð4 cos α þ 1Þ2 þ ð4 cos β þ 1Þ2 þ ð4 cos γ þ 1Þ2  27:


194 5 Application of Trigonometric Inequalities

2
5.2.6. Note that tg 2 α þ tg 2 β  ðtgαþtgβ
2
Þ
 2tg 2 αþβ 2γ
2 ¼ 2ctg 2 (see the proof of the
problem 5.2.2a). Thus,

tg 2 α þ tg 2 β þ tg 2 γ ¼
1  1  1  γ α β
¼ tg 2 α þ tg 2 β þ tg 2 β þ tg 2 γ þ tg 2 α þ tg 2 γ  ctg 2 þ ctg 2 þ ctg 2 :
2 2 2 2 2 2
Remark For angles β ¼ α, γ ¼ π  2α, where α is a sufficiently small number, the
inequality does not hold true.
5.2.7. We have that

4cos 2 αcos 2 β þ 4cos 2 βcos 2 γ þ 4cos 2 γcos 2 α  cos 2 α  cos 2 β  cos 2 γ ¼


¼ ð cos ðα  βÞ  cos γ Þ2 þ 4cos 2 γ ð1  cos γ cos ðα  βÞÞ  1 þ cos γ cos ðα  βÞ  cos 2 γ ¼
¼ cos 2 ðα  βÞ  ð4cos 3 γ þ cos γ Þcos ðα  βÞ þ 4cos 2 γ  1:

Consider a quadratic trinomial f(x) ¼ x2  (4cos3γ þ cos γ)x þ 4cos2γ  1.


Note that f(cosγ) ¼  4cos4γ þ 4cos2γ  1 ¼  (2cos2γ  1)2  0, f(1) ¼  4cos3
γ  cos γ þ 4cos2γ ¼  cos γ(2 cos γ  1)2  0. Therefore,
max f ðxÞ ¼ maxðf ð cos γ Þ; f ð1ÞÞ  0.
½ cos γ;1
As 0  |α  β|  γ, then cosγ  cos(α  β)  1, this means that

4cos 2 αcos 2 β þ 4cos 2 βcos 2 γ þ 4cos 2 γcos 2 α  cos 2 α  cos 2 β  cos 2 γ ¼


¼ f ð cos ðα  βÞÞ  0:

Remark For α ¼ β ¼ π6 , γ ¼ 2π
3 the inequality does not hold true.

5.2.8. (a) Note that

2
pffiffiffið cos α þ cos β þ cos γ Þ2  sin α  sin β  sin γ ¼
3
 2
2 γ αβ γ αβ 8 γ αβ
¼ pffiffiffi 2 sin cos þ cos γ  2 cos cos  sin γ ¼ pffiffiffisin 2 cos 2 þ
3 2 2 2 2 3 2 2
 
4 γ γ αβ 2
þ2 pffiffiffi sin cos γ  cos cos þ pffiffifficos 2 γ  sin γ:
3 2 2 2 3

Consider a quadratic trinomial


 
8 2γ 2 4 γ γ 2
f ðxÞ ¼ pffiffiffi sin x þ 2 pffiffiffi sin cos γ  cos x þ pffiffiffi cos 2 γ  sin γ:
3 2 3 2 2 3
Let γ  π3, then

2  γ 2 γ 2 3 γ  γ
f ð1Þ ¼ pffiffiffi 2 sin þ cos γ  2 cos  sin γ  pffiffiffi  2 sin þ cos γ  2 cos  sin γ,
3 2 2 3 2 2 2
5.2 Inequalities for the Angles of Acute and Obtuse Triangles 195

 2
since 2 sin 2γ þ cos γ ¼ 32  2 sin 2γ  12  32. Thus, it follows that
pffiffiffi γ  γ  γ π  π 
f ð1 Þ  3 2 sin þ cos γ  2 cos  sin γ ¼ 2 2 sin  þ sin γ ¼
2 2 2 6 3
γ π   γ π 
¼ 4 sin  1  cos   0: Therefore; f ð1Þ  0:
2 6 2 6
 
We need to prove that f cos 2γ ¼ p2ffiffi3 ð sin γ þ cos γ Þ2  ð sin γ þ cos γ Þ  1  0.
pffiffiffi  π
 pffiffiffi
Indeed, we have that 1 ¼ sin 2 γþ cos 2 γ  sin γ þ cos γ  2sin γ þ  2.
 γ pffiffiffi 2pffiffi2  4
Thus, f cos 2 ¼ ð sinγ þ cosγ Þ pffiffi3ð sinγ þ cosγ Þ1 1 2 pffiffi3 1 1<0.
2

Since 0  αβ  γ , then we have that cos γ  cos αβ  1, this means that
2 2 2 2

2
pffiffiffið cos α þ cos β þ cos γ  1Þ2  sin α  sin β  sin γ ¼
3  
αβ   γ 
¼ f cos  h max i f ðxÞ ¼ max f cos ; f ð1Þ  0:
2 γ 2
cos ; 1
2
Therefore,

2
sin α þ sin β þ sin γ  pffiffiffi ð cos α þ cos β þ cos γ Þ2 :
3

Remark For angles β ¼ α, γ ¼ π  2α, where α is a sufficiently small number the


inequality does not hold true.
 
(b) We have ðx þ y þ zÞ2  3xy  3yz  3xz ¼ 12 ðx  yÞ2 þ ðy  zÞ2 þ ðx  zÞ2  0.

Therefore, (x þ y þ z)2  3xy þ 3yz þ 3xz.


Thus, it follows that
2
sin α þ sin β þ sin γ  pffiffiffið cos α þ cos β þ cos γ Þ2 
3
pffiffiffi
 2 3ð cos α cos β þ cos β cos γ þ cos γ cos αÞ
(see the problem 5.2.8а).
This means that
pffiffiffi
sin α þ sin β þ sin γ  2 3ð cos α cos β þ cos β cos γ þ cos γ cos αÞ:

(c) Let max(α, β, γ) ¼ γ, then γ  π3 and cos 2γ  cos αβ


2  1.
196 5 Application of Trigonometric Inequalities

We have to prove that

αβ
ð4 cos γ  2Þcos 2
2
γ αβ
 4 sin cos γ cos þ 1  cos γ þ sin 2 γ  cos 2 γ  0: ð5:12Þ
2 2
   
Since 4 cos γ  2  0, then min f ðxÞ ¼ min f cos 2γ ; f ð1Þ , where f ðxÞ ¼
½ cos 2γ ;1
γ
ð4 cos γ  2Þx  4 sin 2 cos γx þ 1  cos γ þ sin 2 γ  cos 2 γ:
2
 2  
We have that f ð1Þ ¼ 2 sin 2γ  1 cos γ  0 and f cos 2γ ¼ 1  sin 2γ  0.
Thus, the inequality (5.12) holds true.
5.2.9. We have that

tgα þ tgβ þ tgγ ¼ tg ðα þ βÞð1  tgαtgβÞ þ tgγ ¼


 π π
¼ tgγ ð1  tgαtgβÞ þ tgγ ¼ tgαtgβtgγ < 0 0 < α; β < ; γ > :
2 2

5.2.10. We have that

cos 2α þ cos 2β  cos 2γ ¼


¼ 1 þ 2 cos ðα þ βÞ cos ðα  βÞ  2cos 2 γ ¼ 1  4 sin α sin β cos γ > 1:

5.2.11. (a) Let γ > π2, then α þ β < π2. Consequently,

sin α þ sin β þ sin γ ¼


αþβ αβ αþβ π pffiffiffi
¼ 2 sin cos þ sin γ  2 sin þ sin γ < 2 sin þ 1 ¼ 1 þ 2:
2 2 2 4
pffiffi pffiffiffi
Remark For α ¼ β ¼ γ ¼ π3, we have that sin α þ sin β þ sin γ ¼ 3 2 3 > 1 þ 2.
pffiffiffiffiffiffiffiffiffiffiffiffiffiffiffiffiffiffiffiffiffi
(b) Let γ > π2. Note that x þ y  2ðx2 þ y2 Þ. Therefore,

pffiffiffiffiffiffiffiffiffiffi pffiffiffiffiffiffiffiffiffiffi pffiffiffiffiffiffiffiffiffiffi qffiffiffiffiffiffiffiffiffiffiffiffiffiffiffiffiffiffiffiffiffiffiffiffiffiffiffiffiffiffiffiffiffiffiffiffiffiffiffiffiffiffiffiffiffiffiffiffiffiffi


pffiffiffiffiffiffiffiffiffiffi2 pffiffiffiffiffiffiffiffiffiffi 2 pffiffiffiffiffiffiffiffiffiffi
sin α þ sin β þ sin γ  2 sin α þ 2ð sin βÞ þ sin γ 
rffiffiffiffiffiffiffiffiffiffiffiffiffiffiffiffiffiffiffiffiffiffi
α þ β pffiffiffiffiffiffiffiffi pffiffiffiffiffiffiffiffiffi pffiffiffi pffiffiffi
 4 sin þ sinγ < 2 2 þ 1 ¼ 1 þ 4 8:
2
5.2 Inequalities for the Angles of Acute and Obtuse Triangles 197

(c) Let γ > π2, then


1
sin α  sin β  sin γ ¼ ð cos ðα  βÞ  cos ðα þ βÞÞ sin γ 
2
1 sin γ sin 2γ sin γ 1
 ð1 þ cos γ Þ sin γ ¼ þ < < :
2 2 4 2 2

Hence, sin α sin β sin γ < 12.


5.2.12. (a) Let γ > π2, then
 
α β γ 1 αβ αþβ γ 1 γ γ
cos cos cos ¼ cos þ cos cos  1 þ cos cos ¼
2 2 2 2 2 2 2 2 2 2
pffiffiffi pffiffiffi
1 γ 1 2 1 1þ 2
¼ cos þ sin γ < þ ¼ :
2 2 4 4 4 4
pffiffi pffiffi
Remark For α ¼ β ¼ γ ¼ π3, we have that cos α2  cos β2  cos 2γ ¼ 3 8 3 > 1þ4 2.

(b) Let γ > 90 , then

  
α β γ α γ þ β  90  45 45 
sin sin sin < sin sin sin 45  sin sin sin 45 ¼
2 2 2 2 pffiffiffi 2 2 2 
1  cos 45  21 β γ 1 γβ βþγ
¼ sin 45 ¼ , since sin sin ¼ cos  cos ,
2 4 2 2 2 2 2
   
sin γþβ90
2 sin 45 ¼ 12 cos α2  cos βþγ 2
        
sin α2 sin γþβ90
2 ¼ 12 cos 45  α  cos 45 , sin 2452 ¼ 12 1  cos 45 ,

cos(45  α)  1, and 0 < α2 < γβ π
2 < 2. Therefore, cos
γβ
2 < cos α2.
5.2.13. Let γ > π2, then

1 þ cos α cos β cos γ  2 sin α sin β sin γ ¼


1 
¼ 2 þ cos ðα  βÞð cos γ  2 sin γ Þ  cos 2 γ  2 sin γ cos γ 
2
1 
 2 þ cos γ  2 sin γ  cos 2 γ  2 sin γ cos γ ,
2

since cosγ  2 sin γ < 0.


Consider a quadratic trinomial f(x) ¼  x2 þ (1  2 sin γ)x þ 2  2 sin γ. Note
that f(0) ¼ 2  2 sin γ > 0 and f(1) ¼ 0. Hence,f(x) > 0, for all 1 < x < 0 . Thus
2 þ cos γ  2 sin γ  cos2γ  2 sin γ cos γ ¼ f(cosγ) > 0. Therefore, 1 þ cos α
cos β cos γ > 2 sin α sin β sin γ.
Remark For α ¼ β ¼ γ ¼ π3, we have that 1 þ cos α cos β cos γ < 2 sin α sin β sin γ.
198 5 Application of Trigonometric Inequalities

Problems for Self-Study

Let α, β, and γ be the angles of some acute triangle.


Prove the following inequalities.
5.2.14. sin2α > sin 2β > sin 2γ, if α < β < γ.
5.2.15. cos2α þ cos 2β  cos 2γ < 1.
5.2.16. 2 cos(α  β) cos(β  γ) cos(γ  α)  1 þ 8 cos α cos β cos γ.
5.2.17. 2 < sin 2 α þ sin 2 β þ sin 2 γ  32 þ 12 ð cos α þ cos β þ cos γ Þ.
5.2.18. (a) sinα þ sin β þ sin γ > cos α þ cos β þ cos γ,
pffiffiffiffiffiffiffiffiffiffi pffiffiffiffiffiffiffiffiffiffi pffiffiffiffiffiffiffiffiffiffi
(b) sin α þ sin β þ sin γ > 2.
pffiffiffiffiffi
5.2.19. tg n α þ tg n β þ tg n γ  3 3n , where n 2 N.
5
αþtg βþtg γ
5 5
5.2.20. tg tgαþtgβþtgγ 9.

5.2.21. 1  tg 2α2 þ tg 2 β2 þ tg 2 2γ < 2.


     
5.2.22. tgα þ tgβ þ tgγ  ctg π8 þ α8 þ ctg π8 þ β8 þ ctg π8 þ 8γ .
3
αþsin 3 βþsin 3 γ
pffiffiffi
sin βð sin αþ sin βþ sin γ Þ >
5.2.23. sin αsin 2.

Hint Prove that

sin 3 α þ sin 3 β þ sin 3 γ >


> ð sin α þ sin βÞ sin α sin β þ sin γ ðsin 2 α þ sin 2 βÞ  sin α sin βðsinα þ sin β þ 2 sin γ Þ:

5.2.24. ð cos α cos β þ cos α cos γ þ cos β cos γ Þ2 


 cos 2 αsin 2 γ þ cos 2 γsin 2 β þ cos 2 βsin 2 α:
Let α, β, and γ be the angles of some obtuse triangle.
Prove the following inequalities.
5.2.25. sin2α þ sin2β þ sin2γ < 2.
5.2.26. cosα cos β cos γ >  1.

5.3 Some Relations for a Triangle

Let α, β, γ be the angles of triangle ABC. Prove the following relations.


5.3.1. (a) sin α þ sin β þ sin γ ¼ Rp ,
5.3 Some Relations for a Triangle 199

(b) sin α sin β sin γ ¼ 2R


pr
2,

(c) sin 2α þ sin 2β þ sin 2γ ¼ 2pr


R2
.
5.3.2. ctg α2 þ ctg β2 þ ctg 2γ ¼ pr.
5.3.3. cos α2 cos β2 cos 2γ ¼ 4R
p
.
5.3.4. sin α2 sin β2 sin 2γ ¼ 4R
r
.
5.3.5. tg α2 tg β2 tg 2γ ¼ pr .
5.3.6. cos α þ cos β þ cos γ ¼ Rþr
R .
2
þb2 þc2 2
r2 4rR
5.3.7. ctgα þ ctgβ þ ctgγ ¼ a 4S ¼p 2pr .
2
r2 4rR
5.3.8. (a) sin 2 α þ sin 2 β þ sin 2 γ ¼ p 2R2
,

(b) cos 2 α þ cos 2 β þ cos 2 γ ¼ 6R p2Rþ4Rrþr


2 2 2
2 .
2
p2 þr2 þ4rR
5.3.9. (a) cos 2α þ cos 2β þ cos 2γ ¼ 3R R2
,
2
ð2RþrÞ2
(b) cos α cos β cos γ ¼ p 4R2
.

5.3.10. (a) sin α sin β þ sin β sin γ þ sin γ sin α ¼ p þ4Rrþr


2 2

4R2
,
2
þp2 4R2
(b) cos α cos β þ cos β cos γ þ cos γ cos α ¼ r 4R2
,
¼ pp2 þrð2Rþr
4R
2 2 2
(c) 1
cos α þ 1
cos β þ 1
cos γ Þ2
.
αβ βγ γα
¼ p þ2Rrþr
2 2
5.3.11. cos 2 cos 2 cos 2 8R2
.
5.3.12. tgα þ tgβ þ tgγ ¼ a2 þb2 þc
4S
2 8R2 ¼ 2
2pr
p ð2RþrÞ2
.

5.3.13. tgαtgβ þ tgβtgγ þ tgγtgα ¼ pp2 rð2Rþr


4Rr 2 2

Þ2
.

5.3.14. tg α2 þ tg β2 þ tg 2γ ¼ 4Rþr
p .

5.3.15. sin 3 α cos β cos γ þ sin 3 β cos αcos γ þ sin 3 γ cos α cos β ¼
pr  2
¼ 4 2R  p2 þ ð2R þ r Þ2 :
4R
βγ αγ αβ
5.3.16. sin 2 α cos 2 þ sin 2 β cos 2 þ sin 2 γ cos 2 ¼
2 2 2
ð2R þ r Þp þ 4Rr þ r
2 2 3
¼ :
8R3
200 5 Application of Trigonometric Inequalities

Solutions

5.3.1. (a) Taking into account the relation sin α þ sin β þ sin γ ¼ 2R
a
þ 2R
b
þ 2R
c
, we
obtain that sin α þ sin β þ sin γ ¼ R.
p

(b) Since sin α sin β sin γ ¼ 8R3 and 4R ¼ S ¼ pr, then sin α sin β sin γ ¼
abc abc pr
2R2
.

(c) Note that

sin 2α þ sin 2β þ sin 2γ ¼


¼ 2 sin ðα þ βÞ cos ðα  βÞ þ sin 2γ ¼ 2 sin γ ð cos ðα  βÞ  cos ðα þ βÞÞ
¼ 4 sin α sin β sin γ,

then sin 2α þ sin 2β þ sin 2γ ¼ 42R


pr
2 ¼
2pr
R2
(see the problem 5.3.1).
β γ
5.3.2. Taking into account that ctg α2 ¼ pa
r , ctg 2 ¼ r , ctg 2 ¼ r
pb pc
and p  a þ
α β γ
p  b þ p  c ¼ p, we deduce that ctg 2 þ ctg 2 þ ctg 2 ¼ r .
p

5.3.3. Note that

αþβ αβ γ γ
sin α þ sin β þ sin γ ¼ 2 sin cos þ 2 sin cos ¼
 2  2 2 2
γ αβ αþβ α β γ
¼ 2 cos cos þ cos ¼ 4 cos cos cos :
2 2 2 2 2 2

Therefore,

α β γ 1 p
cos cos cos ¼ ð sin α þ sin β þ sin γ Þ ¼ ,
2 2 2 4 4R

(see the problem 5.3.1a).


5.3.4. We have that

α β γ sin α sin β sin γ pr 2p r


sin sin sin ¼ α β γ
¼ 2: ¼ ,
2 2 2 8 cos 2 cos 2 cos 2 2R R 4R

(see the problem 5.3.1b and 5.3.3).


5.3 Some Relations for a Triangle 201

5.3.5. Since

α β γ sin α2 sin β2 sin 2γ r p r


tg tg tg ¼ ¼ : ¼
2 2 2 cos α2 cos β2 cos 2γ 4R 4R p

(problems 5.3.3 and 5.3.4).


5.3.6. Note that

αþβ αβ γ αβ γ


cos α þ cos β þ cos γ ¼ 2 cos cos þ cos γ ¼ 2 sin cos þ 1  2sin 2 ¼
2 2 2 2 2
 
γ αβ αþβ α β γ r Rþr
¼ 2 sin cos  cos þ 1 ¼ 1 þ 4 sin sin sin ¼ 1 þ ¼ ,
2 2 2 2 2 2 R R

(see the problem 5.3.4).


5.3.7. We have that

cos α cos β cos γ b2 þ c2  a2 a2 þ c2  b2 a2 þ b2  c2


ctgα þ ctgβ þ tgγ ¼ þ þ ¼ þ þ ¼
sin α sin β sin γ 2bc sin α 2ac sin β 2ab sin γ
b2 þ c2  a2 a2 þ c2  b2 a2 þ b2  c2 a2 þ b2 þ c2
¼ þ þ ¼ :
4S 4S 4S 4S
pffiffiffiffiffiffiffiffiffiffiffiffiffiffiffiffiffiffiffiffiffiffiffiffiffiffiffiffiffiffiffiffiffiffiffiffiffiffiffiffiffiffiffiffiffiffi
Taking into account that pr ¼ pðp  aÞðp  bÞðp  cÞ and abc ¼ 4SR,
we obtain that p2r2 ¼ p( p  a)( p  b)( p  c). Thus, it follows that pr2 ¼ p3  p2
(a þ b þ c) þ p(ab þ bc þ ac)  4SR ¼ p3  2p3 þ p(ab þ bc þ ac)  4pRr, this
means that ab þ bc þ ac ¼ p2 þ r2 þ 4Rr and
a2 þ b2 þ c2 ¼ (a þ b þ c)2  2(ab þ bc þ ac) ¼ 2p2  2r2  8Rr.
Thus,
a2 þ b2 þ c2 2p2  2r 2  8Rr p2  r 2  4Rr
¼ ¼ :
4S 4pr 2pr

5.3.8. (a) We have that


a2 þ b2 þ c2 p2  r 2  4Rr
sin 2 α þ sin 2 β þ sin 2 γ ¼ ¼ ,
4R2 2R2

(see the proof of the problem 5.3.7).


(b) Note that
  6R2  p2 þ r 2 þ 4Rr
cos 2 α þ cos 2 β þ cos 2 γ ¼ 3  sin 2 α þ sin 2 β þ sin 2 γ ¼ ,
2R2
(see the problem 5.3.8a).
202 5 Application of Trigonometric Inequalities

5.3.9. (a) We have that

cos 2α þ cos 2β þ cos 2γ ¼ cos 2 α þ cos 2 β þ cos 2 γ  ðsin 2 α þ sin 2 β þ sin 2 γ Þ ¼


6R2  p2 þ r 2 þ 4Rr p2  r2  4Rr 3R2  p2 þ r 2 þ 4Rr
¼  ¼ ,
2R2 2R2 R2

(see the problems 5.3.8a and b).


(b) Note that

cos 2α þ cos 2β þ cos 2γ ¼ 2 cos ðα þ βÞ cos ðα  βÞ þ 2cos 2 γ  1


¼ 1  4 cos α cos β cos γ:

Hence,
1 p2  ð2R þ r Þ2
cos α cos β cos γ ¼  ð1 þ cos 2α þ cos 2β þ cos 2γ Þ ¼ ,
4 4R2
(see the problem 5.3.9a).
5.3.10. (a) We have that

sin α sin β þ sin β sin γ þ sin γ sin α


1 
¼ ð sin α þ sin β þ sin γ Þ2  sin 2 α  sin 2 β  sin 2 γ ¼
2
 
1 p2 p2  r 2  4rR p2 þ r 2 þ 4Rr
¼  ¼
2 R2 2R2 4R2

(see the problems 5.3.1a and 5.3.8a).


(b) We have that
cos α cos β þ cos β cos γ þ cos γ cos α ¼
1 
¼  ð cos α þ cos β þ cos γ Þ2  cos 2 α  cos 2 β  cos 2 γ ¼
2
!
1 ðR þ r Þ2 6R2  p2 þ 4Rr þ r 2 p2  4R2 þ r 2
¼  ¼ ,
2 R2 2R2 4R2

(see the problems 5.3.6 and 5.3.8b).


(c) Note that

1 1 1 cos α cos β þ cos β cos γ þ cos α cos γ


þ þ ¼ ¼
cos α cos β cos γ cos α cos β cos γ
r 2 þ p2  4R2 p2  ð2R þ r Þ2 r 2 þ p2  4R2
¼ : ¼ ,
4R2 4R2 p2  ð2R þ r Þ2

(see the problems 5.3.10b and 5.3.9b).


5.3 Some Relations for a Triangle 203

5.3.11. Note that


 
αβ βγ γα 1 αγ α þ γ  2β γ α
cos cos cos ¼ cos þ cos cos ¼
2 2 2 2 2 2 2
1 αγ 1 α þ γ  2β γ α 1
¼ cos 2 þ cos cos ¼ ð1 þ cos ðγ  αÞ þ cos ðβ  γ Þ þ cos ðα  βÞÞ ¼
2 2 2 2 2 4
1  
¼ 1 þ cos γ cos α þ cos β cos γ þ cos α cos β þ sin αsinβ þ sinβ sinγ þ sin γ sinα ¼
4
 
1 r 2 þ p2  4R2 p2 þ 4Rr þ r 2 p2 þ r2 þ 2Rr
¼ 1þ þ ¼ ,
4 4R2 4R2 8R2

(see the problems 5.3.10a and 5.3.10b).


5.3.12. We have that

tgα þ tgβ þ tgγ ¼ tg ðα þ βÞð1  tgαtgβÞ þ tgγ ¼ tgγ ð1  tgαtgβÞ þ tgγ ¼


sin α sin β sin γ pr p2  ð2R þ r Þ2 2pr
¼ tgαtgβtgγ ¼ ¼ 2: ¼ ,
cos α cos β cos γ 2R 4R 2
p  ð2R þ r Þ2
2

(see the problems 5.3.1b and 5.3.9b).


Since a2 þ b2 þ c2  8R2 ¼ 2p2  2r2  8Rr  8R2 ¼ 2( p2  (2R þ r)2), (see the
proof of the problem 5.3.7) and S ¼ pr, then

2pr 4S
tgα þ tgβ þ tgγ ¼ ¼ :
p2  ð2R þ r Þ 2
a2 þ b þ c2  8R2
2

5.3.13. Note that

ctgα þ ctgβ þ ctgγ p2  r2  4rR


tgαtgβ þ tgβtgγ þ tgαtgγ ¼ ¼ tgαtgβtgγ ¼
ctgαctgβctgγ 2pr
p2  r2  4rR p2  r2  4rR 2pr p2  r2  4rR
¼ ðtgα þ tgβ þ tgγ Þ ¼ 2
¼ ,
2pr 2pr p2  ð2R þ rÞ p2  ð2R þ rÞ2

(see the problem 5.3.7 and the proof of the problem 5.3.12).
5.3.14. Note that

α β γ r r r
tg þ tg þ tg ¼ þ þ
2 2 2 pa pb pc
ab þ bc þ ac  p2 r 2 þ 4Rr r þ 4R
¼ r¼ r¼ ,
ðp  aÞðp  bÞðp  cÞ pr 2 p

(see the proof of the problem 5.3.7).


204 5 Application of Trigonometric Inequalities

5.3.15. We have that


sin 3 α cos β cos γ þ sin 3 βcosα cos γ þ sin 3 γcosα cos β ¼
¼ sin 3 αð cos ðβ þ γ Þ þ sin β sin γ Þ þ sin 3 βð cos ðα þ γ Þ þ sin α sin γ Þþ
þsin 3 γ ð cos ðα þ βÞ þ sin α sin βÞ ¼
1  cos 2α sin 2α 1  cos 2β sin 2β 1  cos 2γ sin 2γ
¼   þ
2 2 2 2 2 2
þ sin α sin β sin γ ðsin 2 α þ sin 2 β þ sin 2 γ Þ ¼
1 1
¼  ð sin 2α þ sin 2β þ sin 2γ Þ þ ð sin 4α þ sin 4β þ sin 4γ Þþ
4 8
þ sin α sin β sin γ ðsin 2 α þ sin 2 β þ sin 2 γ Þ ¼
1
¼  sin α sin β sin γ þ ð2 sin ð2α þ 2βÞ cos ð2α  2βÞ þ 2 sin 2γ cos 2γ Þþ
8
þ sin α sin β sin γ ðsin 2 α þ sin 2 β þ sin 2 γ Þ ¼
1
¼  sin α sin β sin γ þ ð sin 2γ cos ð2α  2βÞ þ sin 2γ cos ð2α þ 2βÞÞþ
4
þ sin α sin β sin γ ðsin 2 α þ sin 2 β þ sin 2 γ Þ ¼
1  
¼  sin α sin β sin γ  sin 2α sin 2β sin 2γ þ sin α sin β sin γ sin 2 α þ sin 2 β þ sin 2 γ ¼
2
¼ sin α sin β sin γ ðsin 2 α þ sin 2 β þ sin 2 γ  1  4cosα cos β cos γ Þ ¼
¼ sin α sin β sin γ ðsin 2 α þ sin 2 β þ sin 2 γ þ cos 2α þ cos 2β þ cos 2γ Þ ¼
pr 6R2  p2 þ 4Rr þ r 2
¼ sin α sin β sin γ ðcos 2 α þ cos 2 β þ cos 2 γ Þ ¼ 2 ¼
  2R 2R2
pr
¼ 4 2R2  p2 þ ð2R þ r Þ2 ,
4R

(see the proof of the problems 5.3.1c, 5.3.9b and the problems 5.3.1b, 5.3.8b).
5.3.16. We have that
βγ αγ α  β sin 2 α
sin 2 αcos 2 þ sin 2 βcos 2 þ sin 2 γcos 2 ¼ ð1 þ cos ðβ  γ ÞÞ þ
2 2 2 2
sin β
2
sin γ
2
þ ð1 þ cos ðα  γ ÞÞ þ ð1 þ cos ðα  βÞÞ ¼
2 2
1  ð1  cos 2 αÞ cos β cos γ ð1  cos 2 βÞ cos α cos γ
¼ sin 2 α þ sin 2 β þ sin 2 γ þ þ þ
2 2 2
ð1  cos 2 γ Þ cos α cos β sin α sin β sin γ
þ þ ð sin α þ sin β þ sin γ Þ ¼
2 2
sin 2 α þ sin 2 β þ sin 2 γ cos α cos β þ cos β cos γ þ cos γ cos α
¼ þ 
2 2
cosα cos β cos γ sin α sin β sin γ
 ðcosα þ cos β þ cos γ Þ þ ð sin α þ sin β þ sin γ Þ ¼
2 2
p2  r2  4rR r2 þ p2  4R2 p2  ð2R þ r Þ2 R þ r pr p
¼ 2
þ 2
 2
 þ 2 ¼
4R 8R 8R R 4R R
ð2R þ rÞp þ 4Rr þ r
2 2 3
¼
8R3
(see the problems 5.3.1a, 5.3.1b, 5.3.6, 5.3.8a, 5.3.9b, 5.3.10b).
5.4 Trigonometric Inequalities 205

Problems for Self-Study

Let α, β, γ be the angles of triangle ABC. Prove the following relations.


 2
5.3.17. cos1 2 α þ cos1 2 β þ cos1 2 γ ¼ 1 þ 4Rþr
p .
2 2 2

5.3.18. cos 2 α1cos 2 β þ cos 2 β1cos 2 γ þ cos 2 α1cos 2 γ ¼ 32R pþ8Rr


2
2 .
2 2 2 2 2 2

5.4 Trigonometric Inequalities


αþβ βþγ γþα
5.4.1. Prove that ctgα þ tgβ þ ctgγ  ctg 2 þ ctg 2 þ ctg 2 , where 0 < α,
β, γ < π2.
5.4.2. Prove that
(a) sin α cos β þ sin β cos γ þ sin γ cos α  32,
(b) sin 2 α cos β þ sin 2 β cos γ þ sin 2 γ cos α < 32.
pffiffiffi
5.4.3. Prove that cos α þ cos β þ cos γ  5, if sinα þ sin β þ sin γ  2.
qffiffiffiffiffiffiffiffiffiffiffiffiffiffiffiffiffiffiffiffiffiffiffiffiffiffiffiffiffiffiffi
 ffi
5.4.4. Prove that a sin α þ b sin β þ c sin γ  2 a2 þ b2 þ c2 , if cos2α þ cos2
β þ cos2γ ¼ 1.
5.4.5. Prove that ctg α21 þ ctg β21 þ ctg γ21 > ctg α2 þ ctg β2 þ ctg 2γ , if 0 < α1  α  β,
0 < γ 1  γ  β, β1 6¼ β and α1 þ β1 þ γ 1 ¼ α þ β þ γ ¼ π.
5.4.6. Prove that α1 þ α2 þ ::: þ αn < π2 ðn  1Þ, if 0 < αi < π2, i ¼ 1, 2, . . . , n and
cos2α1 þ cos2α2 þ . . . þ cos2αn > 1.
5.4.7. Prove that cos φ1 cos φ2  :::  cos φn  cos nπn, if φ1, φ2, . . . , φn > 0, and
φ1 þ . . . þ φn ¼ π
5.4.8. Prove that
(a) tgα þ tgβ þ tgγ  pffiffi3 cos α 2cos β cos γ, if 0  α, β, γ < π2,
pffiffi
(b) tgα þ tgβ þ tgγ þ tgδ  4 cos α cos3 β 3cos γ cos δ, if 0  α, β, γ, δ < π2.

5.4.9. Prove that sin2α þ sin 2β þ sin 2γ þ sin 2δ  16 sin α sin β sin γ sin δ, if
α, β, γ, δ > 0 and α þ β þ γ þ δ ¼ π.
pffiffiffiffiffiffiffiffiffiffiffiffiffiffiffiffiffiffiffiffiffiffiffiffiffiffiffiffiffiffiffiffiffiffiffiffiffiffiffiffiffiffiffiffiffiffiffiffiffiffiffiffiffiffiffiffiffiffi
5.4.10. Prove that ab sin α þ cd sin β  2 ðp  aÞðp  bÞðp  cÞðp  d Þ, if a, b, c,
d, p  a, p  b, p  c, p  d are positive numbers, where p ¼ aþbþcþd 2 and a2 þ b2
 2ab cos α ¼ c þ d  2cd cos β.
2 2
206 5 Application of Trigonometric Inequalities

5.4.11. Prove that


(a) 1 þ 4 cos α cos β cos γ  cos α þ cos β þ cos γ, if α, β, γ > 0 and α þ β þ γ ¼ π,
(b) cos α þ cos β þ cos γ  17 12 þ 3 cos α cos β cos γ, if
2
0 < α, β, γ < π2 and
α þ β þ γ ¼ π,
(c) 1 þ 4 sin α2 sin β2 sin 2γ  sin α2 þ sin β2 þ sin 2γ  17 α β γ
12 þ 3 sin 2 sin 2 sin 2, if 0 < α,
2

β, γ and α þ β þ γ ¼ π,
(d) sin α2 sin β2 þ sin β2 sin 2γ þ sin 2γ sin α2  12 þ 2 sin α2 sin β2 sin 2γ , where α, β, γ > 0
and α þ β þ γ ¼ π.
5.4.12. Prove that cosα þ cos β  1, if α, β > 0 and 2α þ β  π, 2β þ α  π.
sin ðβþλαÞ sin β
5.4.13. Prove that sin ðαþλβÞ  sin α, if 0 < α  β, α þ β < π, and 0  λ  1.

5.4.14. Prove that, if α1 þ . . . þ αn  βi þ . . . þ βn  π, α1, . . . , αn > 0 and


β1 cos βn cos α1 cos αn
β1, . . . , βn  0, i ¼ 1, . . . , n, then cos
sin α1 þ ::: þ sin αn  sin α1 þ ::: þ sin αn .

5.4.15. Prove that sin α1 þ sin α2 þ ::: þ sin αn  n sin αn, if 0  αi  π, i ¼ 1, . . . , n,


and α1 þ . . . þ αn ¼ α.
5.4.16. Prove that tg α1 þ tg α2 þ ::: þ tg αn  ntg α1 þα2 þ:::þα
n
n
, if 0  αi < π2,
i ¼ 1, 2, . . . , n.
5.4.17. Prove that 0  α  sin α  sin β  sin γ þ sin(α þ β) þ sin(α þ γ)  π, if
α þ β þ γ  π and α, β, γ > 0.
 
5.4.18. Prove that φ  min π6; βþγ
3 , if sin φ ¼ sin(α  φ) sin(β  φ) sin(γ  φ) and
3

0 < φ < γ  β  α, α þ β þ γ ¼ π.
5.4.19. Prove that

x1 x2 cos α1 þ x2 x3 cos α2 þ ::: þ xn1 xn cos αn1 þ xn x1 cos αn


π 
 cos x21 þ x22 þ ::: þ x2n , ð5:13Þ
n

where α1 þ α2 þ . . . þ αn ¼ π, for (a) n ¼ 3, (b) n ¼ 4, and (c) n ¼ 6.

Solutions
 
5.4.1. Note that if x, y 2 0; π2 , then

2 sin ðx þ yÞ 2 sin ðx þ yÞ xþy


ctgx þ ctgy ¼  ¼ 2ctg ,
cos ðx  yÞ  cos ðx þ yÞ 1  cos ðx þ yÞ 2

thus
5.4 Trigonometric Inequalities 207

1 1 1
ctgα þ ctgβ þ ctgγ ¼ ðctgα þ ctgβÞ þ ðctgβ þ ctgγ Þ þ ðctgγ þ ctgαÞ 
2 2 2
αþβ βþγ γþα
 ctg þ ctg þ ctg :
2 2 2

5.4.2 (a) Since ab  a þb


2 2
2 , then

sin 2 α þ cos 2 β sin 2 β þ cos 2 γ sin 2 γ þ cos 2 α


sin α cos β þ sin β cos γ þ sin γ cos α  þ þ ¼
2 2 2
ðsin 2 α þ cos 2 αÞ þ ðsin 2 β þ cos 2 βÞ þ ðsin 2 γ þ cos 2 γ Þ 3
¼ ¼ :
2 2

Remark Similarly, one can prove that

j sin α cos βj þ j sin β cos γ j þ j sin γ cos αj  32.


(b) We have that sin 2 α cos β þ sin 2 β cos γ þ sin 2 γ cos α  j sin α cos βjþ
j sin β cos γ j þ j sin γ cos αj  32.
The equality cannot hold true. Otherwise, the following conditions must be
fulfilled: |sin α| ¼ |cos β| ¼ 0 or 1, |sin β| ¼ |cos γ| ¼ 0 or 1, |sin γ| ¼ |cos α| ¼ 0 or 1.
This leads to a contradiction, as (sin2α þ cos2β) þ (sin2β þ cos2γ) þ (sin2γ þ cos
2
α) ¼ 3 is an odd number.
Therefore, sin 2 α cos β þ sin 2 β cos γ þ sin 2 γ cos α < 32.
5.4.3. Note that

ð cos α þ cos β þ cos γ Þ2 þ ð sin α þ sin β þ sin γ Þ2 ¼


¼ 3 þ 2 cos ðα  βÞ þ 2 cos ðβ  γ Þ þ 2 cos ðγ  αÞ  9

and since (sinα þ sin β þ sin γ)2  4, then (cosα


pffiffiffi þ cos β þ cos γ)  5.
2

Consequently, cos α þ cos β þ cos γ  5.


5.4.4. Note that sin2α þ sin2β þ sin2γ ¼ 1  cos2α þ 1  cos2β þ 1  cos2γ ¼ 2,
consequently, if a2 þ b2 þ c2 6¼ 0. Then,

a sin α b sin β c sin γ


ffi þ qffiffiffiffiffiffiffiffiffiffiffiffiffiffiffiffiffiffiffiffiffiffiffiffiffiffiffiffiffiffiffi
qffiffiffiffiffiffiffiffiffiffiffiffiffiffiffiffiffiffiffiffiffiffiffiffiffiffiffiffiffiffiffi
  ffi þ qffiffiffiffiffiffiffiffiffiffiffiffiffiffiffiffiffiffiffiffiffiffiffiffiffiffiffiffiffiffiffi
 ffi ¼
2 a2 þ b þ c 2 2
2 a2 þ b þ c2 2
2 a2 þ b2 þ c 2
a sin α b sin β c sin γ
¼ qffiffiffiffiffiffiffiffiffiffiffiffiffiffiffiffiffiffiffiffiffiffiffiffiffiffiffiffiffi
  pffiffi2ffi þ qffiffiffiffiffiffiffiffiffiffiffiffiffiffiffiffiffiffiffiffiffiffiffiffiffiffiffiffiffi
  pffiffi2ffi þ qffiffiffiffiffiffiffiffiffiffiffiffiffiffiffiffiffiffiffiffiffiffiffiffiffiffiffiffiffi
 pffiffi2ffi 
a2 þ b þ c2 2
a2 þ b þ c2 2
a2 þ b þ c 2
2
   
1 a 2
sin α
2
1 b 2
sin β 2
 þ þ þ
2 aþ b þ c
2 2 2 2 2 a þ b þ c
2 2 2 2
1 c2 sin 2 γ
þ þ ¼ 1,
2 a2 þ b2 þ c 2 2
208 5 Application of Trigonometric Inequalities

(see theqproof of the problem


ffiffiffiffiffiffiffiffiffiffiffiffiffiffiffiffiffiffiffiffiffiffiffiffiffiffiffiffiffiffiffi 5.4.2a), this means that a sin α þ b sin β þ c
 ffi
sin γ  2 a þ b þ c .
2 2 2
2
þ b2 þ c2 ¼ 0,ffi then a ¼ b ¼ c ¼ 0 and
Ifqaffiffiffiffiffiffiffiffiffiffiffiffiffiffiffiffiffiffiffiffiffiffiffiffiffiffiffiffiffiffiffi a sin α þ b sin β þ c sin γ ¼
 
0 ¼ 2 a2 þ b2 þ c 2 .

5.4.5. We need to prove that ctg α21 þ ctg β21 þ ctg γ21  ctg α21 þ ctg β1 þγ 1 γ
2 þ
ctg 2γ  ctg α2 þ ctg β2 þ ctg 2γ .
As α1  α and γ 1  γ, then β1  β (β1 ¼
6 β). Hence, β1 > β.
We have to prove that

β1 γ β þ γ1  γ γ
ctg þ ctg 1  ctg 1 þ ctg , ð5:14Þ
2 2 2 2

this means that

β1 þγ 1 β1 þγ 1
sin sin
β1 γ 1
2
β1 þγ 1
 β1 þγ 1 2γ
2
:
cos 2  cos 2 cos 2  cos β1 þγ
2
1

The last inequality holds true, as π2 > β1 γ


2 
1 β1 þγ 1 2γ
2 >  β1 γ π
2 > 2.
1

Let us prove the second inequality:

α1 β þ γ1  γ α β
ctg þ ctg 1  ctg þ ctg , ð5:15Þ
2 2 2 2

that is
πγ πγ
sin sin
α1 þγβ1 γ 1
2
πγ
 βα
2
πγ
:
cos 2  cos 2
cos 2  cos 2

This holds true, as π2 > β1 þγ 12α1 γ  βα 2 


α1 þγβ1 γ 1
2 > π2.
Note that in (5.14) and (5.15) the equality cannot simultaneously hold true.
Otherwise, γ 1 ¼ γ and α1 ¼ α, then β1 ¼ β. This leads to a contradiction. Thus,
ctg α21 þ ctg β21 þ ctg γ21 > ctg α2 þ ctg β2 þ ctg 2γ .
5.4.6. Consider the triangle in Figure 5.1.
a2 þa2 þ:::þa2n
We have that cos 2 α1 þ cos 2 α2 þ ::: þ cos 2 αn ¼ 1 2 4 > 1, from which it
follows that
a21 þ a22 þ ::: þ a2n > 4: ð5:16Þ

Now we shall arrange triangles in a way shown in Figure 5.2.

Figure 5.1
1 1
ai ai
ai
5.4 Trigonometric Inequalities 209

Figure 5.2

an-1 a2

p-2an-1 p-2a2

p-2an p-2a1 a1

an

Figure 5.3 an-1


b2
an
bn-2 a2
b1
а a1

Note that π  2α1 þ π  2α2 þ . . . þ π  2αn > π, or α1 þ α2 þ ::: þ αn < π n1 2 .


Indeed, if π  2α1 þ π  2α2 þ . . . þ π  2αn  π, then (see Figure 5.3)
a2  a21 þ b21  a21 þ a22 þ b22  :::  a21 þ ::: þ a2n2 þ b2n2 
 a21 þ ::: þ a2n2 þ a2n1 þ a2n . Hence, we obtain that 4  a21 þ a22 þ ::: þ a2n . This
leads to a contradiction with (5.16).
Remark When α1  0, α2 ¼ ::: ¼ αn  π2 and α1 þ α2 þ ::: þ αn  π n1
2 , cos α1 þ
2

cos α2 þ . . . þ cos αn  1.
2 2

5.4.7. Let 0 < φi  π2, i ¼ 1, . . . , n.


If φ1 ¼ φ2 ¼ . . . ¼ φn(1), then cos φ1 cos φ2  :::  cos φn ¼ cos nπn. If the condi-
tion (5.16) is not satisfied, then there exist such i, j, that φi < πn < φj , but then
 
 
    cos φi þ φj þ cos φi þ φj  2π
cos φi þ φj þ cos φj  φi n
cos φi cos φj ¼ < ¼
2 2
 π  π
¼ cos φi þ φj  cos ,
n n
210 5 Application of Trigonometric Inequalities

since π2 < φi  φj < φi þ φj  2π π


n < φj  φ i < 2 .
Thus, it follows that, if φi þ φj is replaced by n and φi þ φj  πn, then the product
π

cosφ1  . . .  cos φn increases and if we repeat this action not more than n  1 times,
we deduce that cos φ1  :::  cos φn < cos nπn.
If for some i we have that φi > π2, then at n  2, we obtain cos φ1  :::
cos φn < 0  cos nπn.
5.4.8. (a) Since
A ¼ cos α cos β cos γ ðtgα þ tgβ þ tgγ Þ ¼
¼ sin ðα þ βÞ cos γ þ cos α cos β sin γ,
qffiffiffiffiffiffiffiffiffiffiffiffiffiffiffiffiffiffiffiffiffiffiffiffiffiffiffiffiffiffiffiffiffiffiffiffiffiffi
 2  
then using the inequality α1 b1 þ α2 b2  a1 þ a22 b21 þ b22 , we obtain that
pffiffiffiffiffiffiffiffiffiffiffiffiffiffiffiffiffiffiffiffiffiffiffiffiffiffiffiffiffiffiffiffiffiffiffiffiffiffiffiffiffiffiffiffiffiffiffiffiffiffiffiffiffiffiffiffiffiffiffiffiffiffiffiffiffiffiffiffiffiffiffiffiffiffiffiffiffiffiffiffiffiffiffiffiffiffi
A ðsin 2 ðα þ βÞ þ cos 2 αcos 2 βÞðcos 2 γ þ sin 2 γ Þffi ¼
rffiffiffiffiffiffiffiffiffiffiffiffiffiffiffiffiffiffiffiffiffiffiffiffiffiffiffiffiffiffiffiffiffiffiffiffiffiffiffiffiffiffiffiffiffiffiffiffiffiffiffiffiffiffiffiffiffiffiffiffiffiffiffiffiffiffiffiffiffiffiffiffiffiffiffiffiffiffiffiffiffiffiffiffiffiffiffiffiffi
 1
¼ sin 2 ðα þ βÞ þ ð cos ðα þ βÞ þ cos ðα  βÞÞ2 
4
rffiffiffiffiffiffiffiffiffiffiffiffiffiffiffiffiffiffiffiffiffiffiffiffiffiffiffiffiffiffiffiffiffiffiffiffiffiffiffiffiffiffiffiffiffiffiffiffiffiffiffiffiffiffiffiffiffiffiffiffiffiffiffiffiffiffiffiffiffiffi sffiffiffiffiffiffiffiffiffiffiffiffiffiffiffiffiffiffiffiffiffiffiffiffiffiffiffiffiffiffiffiffiffiffiffiffiffiffiffiffiffiffiffiffiffiffiffiffiffiffiffiffiffi
 
1 4 3 1 2 2
 sin 2 ðα þ βÞ þ ðj cos ðα þ βÞj þ 1Þ2 ¼  j cos ðα þ βÞj   pffiffiffi:
4 3 4 3 3

Thus, cos α cos β cos γ ðtgα þ tgβ þ tgγ Þ  p2ffiffi3.


Hence, it follows that tgα þ tgβ þ tgγ  p2ffiffi3  cos α cos1 β cos γ.

(b) We have that

B ¼ cos α cos β cos γ cos δðtgα þ tgβ þ tgγ þ tgδÞ ¼


¼ sin ðα þ βÞ cos γ cos δ þ sin ðγ þ δÞ cos α cos β 
 2  2
 sin ðα þ βÞ cos γþ2 cos δ þ sin ðγ þ δÞ cos αþ2 cos β 
γþδ αþβ
 sin ðα þ βÞcos 2 þ sin ðγ þ δÞcos 2 ¼
2 2
αþβ γþδ αþβþγþδ
¼ 2 cos cos sin ¼
2 2 2
 
αþβþγþδ αþβγδ αþβþγþδ
¼ cos þ cos sin 
2 2 2
 
αþβþγþδ αþβþγþδ αþβþγþδ αþβþγþδ
 1 þ cos sin  2cos 2 sin ¼
2 2 4 2
αþβþγþδ αþβþγþδ
¼ 4cos 3 sin :
4 4


We have obtained that B  4cos3t sin t, where t ¼ αþβþγþδ 2 0; π2 .
 2 4
Since ab  aþb 2 , then
5.4 Trigonometric Inequalities 211

cos 2 t cos 2 t cos 2 t  


cos 6 tsin 2 t ¼ 27 1  cos 2 t 
 2 32 23 3 
4
3 þ 3 þ 3 þ1cos t
cos t cos t cos t 2 27
 27 4 ¼ 4:
4
pffiffi pffiffi pffiffi
Consequently, B  4 3423 ¼ 3 4 3, thus tgα þ tgβ þ tgγ þ tgδ  4 cos α cos3 β 3cos γ cos δ.

5.4.9. We have that

sin 2α þ sin 2β þ sin 2γ þ sin 2δ


¼
sin α sin β sin γsinδ
4 sin α sin β sin ðγ þ δÞ  sin ð2γ þ 2δÞ þ 4 sin γ sin δ sin ðα þ βÞ  sin ð2α þ 2βÞ
¼ ¼
sin α sin β sin γsinδ
4 sin α sin β sin ðγ þ δÞ þ 4 sin γ sin δ sin ðα þ βÞ
¼
sin α sin β sin γ sin δ
¼ 4ðctgα þ ctgβ þ ctgγ þ ctgδÞ,

(see the proof of the problem 5.3.1c).


Note that, if 0 < x, y and x þ y  π, then ctgx þ ctgy  2ctg xþy
2 .
Indeed, we have that

sin ðx þ yÞ 2 sin ðx þ yÞ 2 sin ðx þ yÞ xþy


ctgx þ ctgy ¼ ¼  ¼ 2ctg :
sin x sin y cos ðx  yÞ  cos ðx þ yÞ 1  cos ðx þ yÞ 2

Therefore,
αþβ γþδ αþβþγþδ
ctgα þ ctgβ þ ctgγ þ ctgδ  2ctg þ 2ctg  4ctg ¼ 4:
2 2 4

Thus, sin 2αþ sin 2βþ sin 2γþ sin 2δ


sin α sin β sin γ sin δ  16, this means that sin2α þ sin 2β þ sin 2γ þ
sin 2δ  16 sin α sin β sin γ sin δ.
5.4.10. We have that (ab sin α þ cd sin β)2 þ (ab cos α  cd cos β)2 ¼ a2b2 þ c2d2
 2abcd cos(α þ β), since ab cos α  cd cos β ¼ a þb c d 2
2 2 2
2 , then we deduce that
 2 2 2 2 2
ðab sin α þ cd sin βÞ2 ¼ a2 b2 þ c2 d 2  2abcd cos ðα þ βÞ  a þb c 2
d
¼
 2 2 2 2 2
¼ ðab þ cd Þ2  a þb c 2
d
 2abcd ð1 þ cos ðα þ βÞÞ ¼
  
a2 þ b2  c2  d2 a2 þ b2  c2  d 2 αþβ
¼ ab þ cd  ab þ cd þ  4abcd cos 2 ¼
2 2 2
ðc þ d Þ2  ða  bÞ2 ða þ bÞ2  ðc  d Þ2 αþβ
¼  4abcd cos 2 ¼
2 2 2
αþβ
¼ 4ðp  aÞðp  bÞðp  cÞðp  d Þ  4abcd cos 2  4ðp  aÞðp  bÞðp  cÞðp  dÞ:
2
pffiffiffiffiffiffiffiffiffiffiffiffiffiffiffiffiffiffiffiffiffiffiffiffiffiffiffiffiffiffiffiffiffiffiffiffiffiffiffiffiffiffiffiffiffiffiffiffiffiffiffiffiffiffiffiffiffiffi
Therefore, ab sin α þ cd sin β  2 ðp  aÞðp  bÞðp  cÞðp  dÞ.
212 5 Application of Trigonometric Inequalities

5.4.11. (a) According to the problem 5.1.20, it follows that 1 þ 4 sin α2


sin β2 sin 2γ  1 þ 4 cos α cos β cos γ and it remains to note that cos α þ cos β þ cos γ
¼ 1 þ 4 sin α2 sin β2 sin 2γ (see the proof of the problem 5.3.6).
(b) Using the problems 5.3.6 and 5.3.9b, we need to prove the inequality:
p2 ð2Rþr Þ2
Rþr
R  17
12 þ 3
2
4R2
, that is p2  32 R2 þ 10Rr þ r 2 .
According to the problem 5.5.23, we have that p2  2R2 þ 8Rr3 þ2 3r . Note that
2

2R þ 8Rr þ 3r  2 R þ 10Rr þ r , since 2R þ 8Rr þ 3r  2R þ 10Rr þ r 2
2 2 3 2 2 2 2

¼ 12 ðR  2r Þ2  0. Consequently, p2  32 R2 þ 10Rr þ r 2 .
πβ πγ
(c) Since πα
2 þ 2 þ 2 ¼ π, then according to the problems 5.4.11a and b, we
obtain that
πα πβ πγ πα πβ πγ
1 þ 4 cos cos cos  cos þ cos þ cos 
2 2 2 2 2 2
17 2 πα πβ πγ
 þ cos cos cos ,
12 3 2 2 2

or 1 þ 4 sin α2 sin β2 sin 2γ  sin α2 þ sin β2 þ sin 2γ  17 α β γ


12 þ 3 sin 2 sin 2 sin 2.
2

qffiffiffiffiffiffiffiffiffiffiffiffiffiffiffiffiffiffiffiffiffiffiffiffiffiffiffiffiffiffiffiffiffiffiffi  
(d) We have that 2 sin α2 sin β2 ¼ tg β2 sin αtg α2 sin β  12 tg β2 sin α þ sin βtgα2 ,
this means that

α β β γ α γ
2 sin sin þ 2 sin sin þ 2 sin sin 
2 2 2 2 2 2
1 α 1 β 1 γ
 tg ð sin β þ sin γ Þ þ tg ð sin α þ sin γ Þ þ tg ð sin α þ sin βÞ ¼
2 2 2 2 2 2
α βγ β αγ γ αβ
¼ sin cos þ sin cos þ sin cos ¼
2 2 2 2 2 2
α β γ
¼ cosα þ cos β þ cos γ ¼ 1 þ 4 sin sin sin ,
2 2 2

(see the proof of the problem 5.3.6).


Another proof of the problem can be obtained by the inequality of the problem
πβ πγ
5.2.8c for the triangle with angles πα
2 , 2 , 2 .

5.4.12. Since α < α þ β  π  α, then sin(α þ β)  sin α, analogously sin(α þ β) 


sin β.
It is clear that 0 < α, β < π2, consequently, sin(α þ β) ¼ sin α cos β þ sin β cos α 
sin(α þ β) cos β þ sin(α þ β) cos α.
Thus, it follows that cosα þ cos β  1.
5.4.13. If α þ λβ2 > π2, then β þ λα λβ π
2  α þ 2 > 2. Therefore, α þ λβ þ β þ λα > π, thus
sin α sin ðαþλβÞ sin ðβþλαÞ sin β
sin β  1  sin ðβþλαÞ or sin ðαþλβÞ  sin α.
5.4 Trigonometric Inequalities 213

   
If α þ λβ2  π2, then cos β þ λα λβ λβ λα
2  cos α þ 2 , as α þ 2  β þ 2 < β þ α < π.
We have that sin 2  sin 2 and sin β  sin α, consequently, at α þ λβ2  π2 the
λα λβ 1 1

following inequality holds true:


  λβ
 
λα
cos β þ λα cos α þ λβ2
2 sin 2 2
 2 sin 2
:
sin β sin α
sin ðβþλαÞ sin β sin ðαþλβÞ sin α sin ðβþλαÞ sin β
Hence, we deduce that sin β  sin α . Thus sin ðαþλβÞ  sin α.

5.4.14. We proceed to prove by mathematical induction.


β1 cos α1
For n ¼ 1we have that 0 < α1  β1  π, consequently, cossin α1  sin α1 .
At n ¼ 2 we have that α1, α2 > 0, β1, β2  0, and α1 þ α2  β1 þ β2  π.
We have to prove that

cos β1 cos β2 cos α1 cos α2


þ  þ :
sin α1 sin α2 sin α1 sin α2

Let α1 and α2 be constant numbers. Consider the expression cos sin α1 þ sin α2 , where
x1 cos x2

α1 > 0, α2 > 0, α1 þ α2  x1 þ x2  π, and x1, x2  0 and let this expression accept its
maximal value at x1 ¼ β1 and x2 ¼ β2.
Let α1  α2, then one can assume that β1  β2. Otherwise, we have that
β1 cos β2
cosβ1 < cos β2. Therefore, (cosβ1  cos β2)(sinα2  sin α1)  0 or cos sin α1 þ sin α2 
cos β2 cos β1
sin α1 þ sin α2 .
If β1 ¼ 0, then

cos β1 cos β2 cos α1 cos α2 1 cos ðα1 þ α2 Þ cos α1 cos α2


þ    þ   ¼
sin α1 sin α2 sin α1 sin α2 sin α1 sin α2 sin α1 sin α2
α1 α  α1
1
α1 2 sin 2 sin þ α2 sin sin ðα1 þ α2 Þ
¼ tg  2 ¼ 2  0:
2 sin α2 α1
cos sin α2
2
β1
If 0 < β1  β2, then with the decrease of the value of β1, the expression cos sin α1
cos β2 sin β1 sin β2
þ increases. Thus, β1 þ β2 ¼ α1 þ α2. We need to prove that sin α1 ¼ sin α2 .
sin α2
ðβ1 þxÞ cos ðβ2 xÞ
Indeed, note that the function f ðxÞ ¼ cossin α1 þ sin α2 accepts its maximal
value on the interval [β1; β2] at the point x ¼ 0. Therefore, according to the
sin β1 sin β2
Fermat’s theorem f 0 ð0Þ ¼  sin α1 þ sin α2 ¼ 0.

Lemma

If α1 , :::, αn , β1 , :::, βn > 0, α1 þ ::: þ αn ¼ β1 þ ::: þ βn


sin β1 sin βn
 π, and ¼ ::: ¼ ¼ λ, ð5:17Þ
sin α1 sin αn

then λ ¼ 1.
214 5 Application of Trigonometric Inequalities

We proceed the proof by contradiction argument. Let λ 6¼ 1, then one can assume
that λ < 1. Let α1  α2  . . .  αn, then it follows from (5.17) that
β1  β2  . . .  βn and β1 < α1 , :::, βn1 < αn1  π2.
Note that

sin ðβ1 þ β2 Þ ¼ λð sin α1 cos β2 þ sin α2 cos β1 Þ > λð sin α1 cos α2 þ sin α2 cos α1 Þ ¼
¼ λ sin ðα1 þ α2 Þ:

Analogously, we obtain that

sin ððβ1 þ β2 Þ þ β3 Þ ¼ sin ðβ1 þ β2 Þ cos β3 þ sin β3 cos ðβ1 þ β2 Þ >


> λð sin ðα1 þ α2 Þ cos β3 þ sin α3 cos ðβ1 þ β2 ÞÞ >
> λð sin ðα1 þ α2 Þ cos α3 þ sin α3 cos ðα1 þ α2 ÞÞ ¼ λ sin ðα1 þ α2 þ α3 Þ,
etc., sin(β1 þ ... þ βn  1) > λ sin(α1 þ ... þ αn  1).
sin βn
Hence, sin ðφ  βn Þ > sin αn sin ðφ  αn Þ, where φ ¼ α1 þ . . . þ αn ¼ β1 þ . . .
þ βn.
Thus, sinαn sin(φ  βn) > sin βn sin(φ  αn) or
sinαn sin φ cos βn  sin αn sin βn cos φ > sin βn sin φ cos αn  sin βn sin αn cos φ,
sinφ sin(αn  βn) > 0. Since 0 < φ  π, then φ 6¼ π and sinφ > 0, then
αn  βn > 0. This means that αn > βn.
We have obtained that α1 > β1, . . . , αn  1 > βn  1, αn > βn, then
α1 þ . . . þ αn > β1 þ . . . þ βn. This leads to a contradiction. Therefore, λ ¼ 1.
This ends the proof of the lemma.
Since sinα1 ¼ sin β1 and sinα2 ¼ sin β2, then α1 ¼ β1 and α2 ¼ β2, or α1 ¼ β1 and
π  α2 ¼ β2 (α1 þ α2 ¼ α1 þ π  α2, α2 ¼ π2, this means that α2 ¼ β2).
cos β1 cos β2 cos α1 cos α2
Consequently, þ ¼ þ .
sin α1 sin α2 sin α1 sin α2
Let now n  3 and the inequality holds true for n  1, we need to prove that it
holds true also for n. Let 0 < α1  α2  . . .  αn be constant numbers. Consider the
sin α1 þ ::: þ sin αn , where x1, x2, . . . , xn  0 and α1 þ . . . þ αn  x1 þ
x1 cos xn
expression cos
. . . þ xn  π and let this expression reach its greatest value at x1 ¼ β1, . . . , xn ¼ βn.
Then, β1  β2  . . .  βn (see the case of n ¼ 2). One can consider β1 > 0,
otherwise for n  1 we have that

cos β2 cos β3 cos βn cos ðα1 þ α2 Þ cos αn


þ þ ::: þ  þ ::: þ : ð5:18Þ
sin ðα1 þ α2 Þ sin α3 sin αn sin ðα1 þ α2 Þ sin αn

Note that

1 cos β2 cos α1 cos α2 cos β2 cos ðα1 þ α2 Þ


þ     : ð5:19Þ
sin α1 sin α2 sin α1 sin α2 sin ðα1 þ α2 Þ sin ðα1 þ α2 Þ
5.4 Trigonometric Inequalities 215

 
Indeed, since cos β2 1
sin α2  1
sin ðα1 þα2 Þ  1
sin α2  1
sin ðα1 þα2 Þ, (α2 þ (α1 þ α2) 
α1 þ α2 þ α3  π, sin (α1 þ α2)  sin α2); thus it is sufficient to prove that
1  cos α1 1  cos α2 1  cos ðα1 þ α2 Þ  
þ   0, or tg α21 þ tg α22  tg α21 þ α22  0,
sin
 α1 sin α2 sin ð α 1 þ α 2 Þ
 tg α21 þ tg α22 tg α21 tg α22
 0.
1  tg α21 tg α22
The last inequality holds true, since 0 < α21 , α22 < π4.
By summing up the inequalities (5.18) and (5.19), we deduce the inequality at n.
Thus, we have that 0 < β1  β2  . . .  βn. If the value of β1 decreases, then the
β1 cos βn
expression cossin α1 þ ::: þ sin αn increases. Thus, α1 þ . . . þ αn ¼ β1 þ . . . þ βn. Then
sin β1 sin βn
sin α1 ¼ ::: ¼ sin αn (see the case of n ¼ 2). Consequently, according to the lemma,
α1 ¼ β1, . . . , αn ¼ βn. This ends the proof of the lemma.
5.4.15. Since at 0  α1, α2  π we have that sin α1 þ sin α2 ¼
2 sin α1 þα
2
2
cos α1 α2
2  2 sin α1 þα2
2 , then according to the Jensen’s inequality, we
deduce that sin α1 þ sin α2 þ ::: þ sin αn  n sin αn.
5.4.16. As 0  α1 , α2 < π2, we have that

sin ðα1 þ α2 Þ 2 sin ðα1 þ α2 Þ


tg α1 þ tg α2 ¼ 1 
2 ð cos ð α1  α 2 Þ þ cos ð α1 þ α2 Þ Þ 1 þ cos ðα1 þ α2 Þ
α1 þ α2
¼ 2tg
2

then according to the Jensen’s inequality, we deduce that tg α1 þ tg α2 þ


::: þ tg αn  ntg α1 þα2 þ:::þα
n
n
.
5.4.17. Note that, if
x, y  0 and x þ y  π, then sin(x þ y) ¼ | sin(x þ y)|  | sin x|| cos y| þ | sin y|| cos x| 
sin x þ sin y, this means that

α  sin α  sin β  sin γ þ sin ðα þ βÞ þ sin ðα þ γ Þ 


 α  sin α  sin β  sin γ þ sin α þ sin β þ sin α þ sin γ ¼ α þ sin α
¼ α þ sin ðπ  αÞ   α þ π  α ¼ π:

Here, we use the inequality sinx  x at x  0 (see the remark of the proof of the
problem 7.1.87).
The proof of the second part is done as follows:

α  sin α þ sin ðα þ βÞ  sin β þ sin ðα þ γ Þ  sin γ 


 sin ðα þ βÞ 
αsin βþ sin ðαþ γÞ 
sin γ ¼
α α
¼ 2 sin cos þ β þ cos þγ
2 2 α  2
α α 
 2 sin cos π  þ γ þ cos þγ ¼ 0:
2 2 2
216 5 Application of Trigonometric Inequalities

5.4.18. We have that sin3φ ¼ sin(α  φ) sin(β  φ) sin(γ  φ). Therefore,

4sin 3 φ þ sin 3φ ¼ sin ðφ þ 2αÞ þ sin ðφ þ 2βÞ sin ðφ þ 2γ Þ,


3 sin φ  sin ðφ þ 2αÞ  sin ðφ þ β þ γ Þ,
2 sin φ  2 sin α cos ðφ þ αÞ  sin ðφ þ β þ γ Þ,
   
βþγ βþγ α α
2 sin cos φ þ  2 sin cos cos ðφ þ αÞ,
2 2 2 2
    
βþγ 3α α
 2 cos φ þ  sin þ φ  sin φ þ ,
2 2 2 ð5:20Þ
      
α α 3α α
sin φ  þ sin φ þ  sin þ φ  sin φ  ,
2 2 2 2
α  α
sin φ cos  sin α cos φ þ ,
2 2
α  α
sin φ  2 sin cos φ þ ,
2 2
2 sin φ  sin ðα þ φÞ:

From (5.20) we obtain that sin φ  12, and since 3φ < π, we deduce that φ  π6.
  π
If min π6; βþγ
3  ¼ 6, then the inequality holds true.
π βþγ
If min 6; 3 ¼ βþγ π
3 , then β þ γ  2 and we obtain from (5.20) that sin2φ <
2 sin φ  sin (α þ φ) ¼ sin (β þ γ  φ). Thus, 2φ  β þ γ  φ, or φ  βþγ
3 .

5.4.19. Note that

ða cos α þ b cos βÞ2  ða cos α þ b cos βÞ2 þ ða sin α  b sin βÞ2


¼ a2 þ b2 þ 2ab cos ðα þ βÞ: ð5:21Þ

(a) We have that

x1 x2 cos α1 þ x2 x3 cos α2 þ x3 x1 cos α3 ¼ x2 ðx1 cos α1 þ x3 cos α2 Þ þ x3 x1 cos α3 


x22 þ ðx1 cos α1 þ x3 cos α2 Þ2
 þ x3 x1 cos α3 
2
x2 þ x21 þ x23 þ 2x1 x3 cos ðα1 þ α2 Þ
 2 þ x3 x1 cos α3 ¼
2
π 
¼ cos x22 þ x21 þ x23 ,
3
 
consequently, x1 x2 cos α1 þ x2 x3 cos α2 þ x3 x1 cos α3  cos π3 x22 þ x21 þ x23 ,
(see also the proof of the problem 5.1.22а).
5.4 Trigonometric Inequalities 217

(b, c) We need to prove that, if the inequality (5.20) holds true for n ¼ k, then it
holds true also at n ¼ 2k, (k 2 N, k  2).
Indeed, let α1 þ α2 þ . . . þ α2k ¼ π, using the inequalities (5.21) and the
Cauchy-Bunyakovsky inequality, and also (5.20) for n ¼ k, we deduce that

x1 x2 cos α1 þ x2 x3 cos α2 þ ::: þ x2k1 x2k cos α2k1 þ x2k x1 cos α2k 
 x2 ðx1 cos α1 þ x3 cos α2 Þ þ ::: þ x2k ðx2k1 cos α2k1 þ x1 cos α2k Þ 

 x2  jx1 cos α1 þ x3 cos α2 j þ ::: þ jx2k j  jx2k1 cos α2k1 þ x1 cos α2k j 
pffiffiffiffiffiffiffiffiffiffiffiffiffiffiffiffiffiffiffiffiffiffiffiffiffiffiffiffiffiffiffiffiffiffiffiffiffiffiffiffiffiffiffiffiffiffiffiffiffiffiffiffiffiffi pffiffiffiffiffiffiffiffiffiffiffiffiffiffiffiffiffiffiffiffiffiffiffiffiffiffiffiffiffiffiffiffiffiffiffiffiffiffiffiffiffiffiffiffiffiffiffiffiffiffiffiffiffiffiffiffiffiffiffiffiffiffiffiffiffiffiffiffiffiffiffiffiffi
 x2  x21 þ x23 þ 2x1 x3 cos ðα1 þ α2 Þ þ ::: þ jx2k j  x22k1 þ x21 þ 2x1 x2k1 cos ðα2k1 þ α2k Þ 
qffiffiffiffiffiffiffiffiffiffiffiffiffiffiffiffiffiffiffiffiffiffiffiffiffiffiffiffiffiffiffiffiffiffiffiffiffiffiffiffiffiffiffiffiffiffiffiffiffiffiffiffiffiffiffiffiffiffiffiffiffiffiffiffiffiffiffiffiffiffiffiffiffiffiffiffiffiffiffiffiffiffiffiffiffiffiffiffiffiffiffiffiffiffiffiffiffiffiffiffiffiffiffiffiffiffiffiffiffiffiffiffiffiffiffiffiffiffiffiffiffiffiffiffiffiffiffiffiffiffiffiffiffiffiffiffiffiffiffiffiffiffiffiffiffiffiffiffiffiffiffiffiffiffiffiffiffiffiffiffiffiffiffiffiffiffiffiffiffiffiffiffiffiffiffiffiffiffiffiffi
 2  
 x2 þ ::: þ x22k 2x21 þ 2x23 þ ::: þ 2x22k1 þ 2x1 x3 cos ðα1 þ α2 Þ þ ::: þ 2x1 x2k1 cos ðα2k1 þ α2k Þ 
rffiffiffiffiffiffiffiffiffiffiffiffiffiffiffiffiffiffiffiffiffiffiffiffiffiffiffiffiffiffiffiffiffiffiffiffiffiffiffiffiffiffiffiffiffiffiffiffiffiffiffiffiffiffiffiffiffiffiffiffiffiffiffiffiffiffiffiffiffiffiffiffiffiffiffiffiffiffiffiffiffiffiffiffiffiffiffiffiffiffiffiffiffiffi

   π
 2 x22 þ ::: þ x22k x21 þ x23 þ ::: þ x22k1 1 þ cos
k
qffiffiffiffiffiffiffiffiffiffiffiffiffiffiffiffiffiffiffiffiffiffiffiffiffiffiffiffiffiffiffiffiffiffiffiffiffiffiffiffiffiffiffiffiffiffiffiffiffiffiffiffiffiffiffiffiffiffiffiffiffiffiffiffiffiffiffiffiffi ffi
π  2   
¼ 2 cos x2 þ ::: þ x22k x21 þ x23 þ ::: þ x22k1 
2k
π 2 
 cos x þ x22 þ ::: þ x22k :
2k 1

Since the inequality (5.20) holds true for n ¼ 2 and n ¼ 3, then according to the
proved statement it holds true also for n ¼ 4 and n ¼ 6.

Problems for Self-Study

Prove the inequalities.


   2  2
1 2
5.4.20. ðtgα þ tgβ þ tgγ Þ2  1 þ cos α þ 1þ 1
cos β þ 1þ 1
cos γ , if 0 < α, β,
π
γ < and α þ β þ γ ¼ π.
2
P  
5.4.21. cos αi  αj  n2.
1i<jn
   
5.4.22. sin 2 β sin α2 sin α þ β2 > sin 2 α sin β2 sin β þ α2 , if 0 < α < β and α þ β < π.
sin α sin β sin β sin γ sin γ sin α
5.4.23. sin 2 2γ
þ sin 2 α2 þ sin 2 β2
 9, where α, β, γ > 0 and α þ β þ γ  π.

sin α sin β 4 sin α sin β


Hint Prove that   1.
sin 2 2γ sin 2 γ
n1

5.4.24. tg α1 þ ::: þ tg αn  ðn1n2Þ where n  2, 0  αi < π2, i ¼


2 1
n 2
cos α1 ::: cos αn ,
1, . . . , n.
218 5 Application of Trigonometric Inequalities

Hint Let α1 þ:::þα


n
n
¼ α, if α1  α  α2, then

ðtg α1 þ ::: þ tg αn Þ cos α1  :::  cos αn ¼ sin ðα1 þ α2 Þ cos α3  :::  cos αn þ
1
þ ð cos ðα1 þ α2 Þ þ cos ðα2  α1 ÞÞðtg α3 þ ::: þ tg αn Þ cos α3  :::  cos αn 
2
1
 sin ðα þ ðα1 þ α2  αÞÞ cos α3  :::  cos αn þ ð cos ðα þ ðα1 þ α2  αÞÞþ
2
þ cos ðα  ðα1 þ α2  αÞÞÞðtg α3 þ ::: þ tg αn Þ cos α3  :::  cos αn ¼
¼ ðtgα þ tg ðα1 þ α2  αÞ þ tg α3 þ ::: þ tg αn Þ
cos α cos ðα1 þ α2  αÞ cos α3  :::  cos αn ,

since π2 < α1  α2  α  ðα1 þ α2  αÞ  α2  α1 < π2. Consequently, we obtain


that
qffiffiffiffiffiffiffiffiffiffiffiffiffiffiffiffiffiffiffiffiffiffiffiffiffiffiffiffiffi
cos 2 αn1
n1
cos α1  :::  cos αn ðtg α1 þ ::: þ tg αn Þ  cos n α  ntgα ¼ nðn  1Þ 2
n1 sin 2 α 
sffiffiffiffiffiffiffiffiffiffiffiffiffiffiffiffiffiffiffiffiffiffiffiffiffiffiffiffiffiffiffiffiffiffiffiffiffiffiffi
 2 nffi n1
n1 cos α
þ:::þ cos 2 α
þsin 2α ðn  1Þ 2
 nðn  1Þ 2 n1
n
n1
¼ n2 :
n2

5.4.25. Let π > α1, α2, . . . , α6 > 0 and α1 þ α2 þ . . . þ α6 ¼ 2π. Is the inequality
cosα1 þ . . . þ cos α6  3 correct?
Hint Take α1 ¼ α2 ¼ α3 ¼ π6, α4 ¼ α5 ¼ π4, and α6 ¼ π.
5.4.26. 1sinsinα αsinsinβ βsinsinγ sin δ
γ sin δ  12 ð sin 2α þ sin 2β þ sin 2γ þ sin 2δÞ, where α, β, γ,
1

δ > 0 and α þ β þ γ þ δ ¼ π.
n sin 2π
5.4.27. sin 2α1 þ ::: þ sin 2αn  n
sin α1  :::  sin αn , where α1  . . .  αn > 0
ð sin πnÞ
n

and α1 þ α2 þ . . . þ αn ¼ π.
5.4.28. sin2α þ sin 2β þ sin 2γ þ sin 2δ  4k þ 1(sin α sin β sin γ sin δ)k, where k 2 N,
α, β, γ, δ > 0, and α þ β þ γ þ δ ¼ π.
5.4.29. sin α2 þ sin β2 þ sin 2γ  2 sin α2 sin β2 þ 2 sin β2 sin 2γ þ 2 sin 2γ sin α2, where α, β,
γ > 0 and α þ β þ γ ¼ π.

5.4.30. p2ffiffi3 ð sin α þ sin β þ sin γ Þ  cos βα
2 þ cos
γβ
2 þ cos αγ
2  p2ffiffi3 cos α2 þ cos β2 þ
γ
cos 2Þ, if α, β, γ > 0 and α þ β þ γ ¼ π.

5.5 Using Trigonometric Inequalities for Proving


Geometric Inequalities

Prove the inequalities for the elements of triangle ABC (5.5.1–5.5.11).


5.5.1. (a) R  2r,
5.5 Using Trigonometric Inequalities for Proving Geometric Inequalities 219

(b) cos 2 βγ


2  R.
2r

pffiffiffi
5.5.2. (a) a2 þ b2 þ c2  4 3S,
pffiffiffi
(b) 3a2 þ 3b2  c2  4 3S,
2
(c) S  a3 þ bc
6.

5.5.3. a2 þ b2 þ c2 < 8R2, if triangle ABC is obtuse.


5.5.4. (a) a2 þ b2 þ c2 > 8R2, if triangle ABC is acute,
(b) b þ c > 2R þ 2r, if triangle ABC is acute.
5.5.5. (a) a2 þ b2 þ c2  8R2 þ 4r2,
2
(b) a2 þ b2 þ c2  8R2 þ 2Rr
Rr ,
(c) a2 þ b2 þ c2  24Rr  12r2,
(d) b2 þ c2 þ R2  a2.
5.5.6. p2  4R2 þ 4Rr þ 3r2.
5.5.7. (a) p > 2R þ r, if triangle ABC is acute,
(b) l2a a2 þ l2b b2 > l2c c2 , if triangle ABC is acute.
5.5.8. (a) p < 2R þ r, if triangle ABC is obtuse.
(b) ma þ mb þ mc  min( p þ 2R, 4R þ r).
pffiffiffi
5.5.9. (a) a2 þ b2 þ c2  ða  bÞ2  ðb  cÞ2  ða  cÞ2  4 3S.
pffiffiffi
(b) a2 þ b2 þ c2  12 ðja  bj þ jb  cj þ jc  ajÞ2  4 3S,
pffiffiffiffiffiffiffiffiffi
(c) aa0 þ bb0 þ cc0  4 3SS0 , where S0 is the area of triangle with sides a0 , b0 , c0 ,
(d) For any tetrahedron ABCD proves that
pffiffiffi
AC  BD þ AB  CD þ BC  AD  3S, where S is the surface area of tetrahe-
dron ABCD.

(e) Given that any edge of tetrahedron ABCD has an angle not smaller than 120 .
Prove that AC  BD þ BC  AD þ AB  CD > 2S, where S is the surface area of
tetrahedron ABCD.
(f) For a convex hexagon ABCDEF with area S prove that
pffiffiffi
ACðBD þ BF  DFÞ þ CEðBD þ DF  BFÞ þ AEðBF þ DF  BDÞ  2 3S:

pffiffiffi  
(g) a2 þ b2 þ c2  4 3S þ 3 ða  bÞ2 þ ðb  cÞ2 þ ða  cÞ2 :

5.5.10. (a) a3 þ b3 þ c3 þ 3abc  a2b þ b2a þ a2c þ c2a þ c2b þ b2c,


(b) p2  16Rr  5r2,  
(c) ððp  aÞðp  bÞ þ ðp  bÞðp  cÞ þ ðp  cÞðp  aÞÞ 1
a2 þ b12 þ c12  94.
220 5 Application of Trigonometric Inequalities

a2 b2 2 2 2 2
(d) þ bac2 þ cba2  9R2 , if triangle ABC is acute.
c2
pffiffiffi
5.5.11. la lb þ lb lc þ lc la  3 3S.
5.5.12. Point H is the orthocenter of the acute triangle ABC, and AD, BE, and CF are
altitudes of that triangle. Prove that
(a) HA2 þ HB2 þ HC2  4(HE2 þ HD2 þ HF2),
pffiffiffi
(b) AB þ BC þ AC  2 3ðHD þ HE þ HFÞ,
(c) (HD þ HE þ HF)2  AF2 þ BD2 þ CE2.
5.5.13. Given a tetrahedron ABCD, such that the edges AD, BD, CD are mutually
perpendicular and AD ¼ a, BD ¼ b, and CD ¼ c. Prove that the sum of the distances
from A, B, and C to the q
straight line l, passing
ffiffiffiffiffiffiffiffiffiffiffiffiffiffiffiffiffiffiffiffiffiffiffiffiffiffiffiffiffiffiffi through D and intersecting the face
 ffi
ABC, does not exceed 2 a þ b þ c . For which location of line l does the
2 2 2

equality hold true?


5.5.14. Let a circle with the center O be inscribed in triangle ABC. Given that the
tangents to the circle are drawn from the intersection points of the circle with rays
OA, OB, and OC. Prove that these tangents define a triangle, such that its perimeter
does not exceed the perimeter of the given triangle.

5.5.15. The central angle AOB is less than 90 . Find on the smaller arc AB a point M,
S 2 B2
such that the ratio SAA AMAþSBB B
has the minimal value. Here, MA1 ⊥ OA, A1 2 OA,
1 2 1 2
MB1 ⊥ OB, B1 2 OB, A2 is the intersection point of segments MA1 and AB, B2 is the
intersection point of segments MB1 and AB.
5.5.16. Given a point M inside of triangle ABC. Prove that SABC
ðABþBCþACÞ2
>
SAMC
ðAMþMCþACÞ2
, if ∠B  max(∠A, ∠C).

5.5.17. Prove that if the sides of the inscribed hexagon ABCDEF are, such that
AB ¼ BC, CD ¼ DE, EF ¼ FA, then the area of triangle ACE does not exceed the
area of triangle BDF.
5.5.18. The incircle of triangle ABC touches the sides of the triangle at points A1, B1,
2
and C1. Prove that A1 B21 þ B1 C21 þ A1 C21  p3 and 2pRr  A1 B1 þ B1 C1 þ A1 C1 ,
where p is the semiperimeter, and R and r are the circumradius and inradius of
triangle ABC, respectively.
5.5.19. Let ABC be an acute triangle with the circumcenter O and the circumradius
R. Let AO intersect the circumcircle of triangle OBC at point D, BO intersect the
circumcircle of triangle OCA at point E, and CO intersect the circumcircle of
triangle OAB at point F. Prove that
(a) OD  OE  OF  8R3,
AD
(b) OD þ OE
BE
þ OF
CF
 4, 5.
5.5 Using Trigonometric Inequalities for Proving Geometric Inequalities 221

5.5.20. Let M be a point inside a convex n-gon A1A2 . . . An. Denote the distances
MA1, MA2, . . . , MAn of the point from the vertices of the polygon by R1, R2, . . . ,
Rn, and the distances MP1, MP2, . . . , MPn from the straight lines A1A2, A2A3, . . . ,
AnA1 by d1, d2, . . . , dn. Prove that R1  R2  :::  Rn cos nπn  d1  d2  :::  d n .
5.5.21. Let the rectangle P1 be inscribed in the rectangle P2 with sides c and
d (c  d), where d is smaller than the greatest side of P1. Prove that sin 2α  dc,
where α is the angle between two straight lines containing any two of the sides of
the rectangles P1 and P2.
5.5.22. Prove that of all quadrilaterals with the given side lengths a, b, c, d the
greatest area has the one that can be inscribed in a circle.
5.5.23. Prove the inequality for the acute triangle it holds true p2  2R2 þ 8Rr þ 3r2.
5.5.24. Let a, b, c be the sides of triangle ABC, and S be its area. Prove that
(a) x
yþz a
4
þ xþz
y
b4 þ xþy
z
c4  8S2 ,
pffiffiffi
(b) x
yþz a
2
þ xþz
y
b2 þ xþy
z
c2  2 3S, if x þ y, x þ z, y þ z > 0.

5.5.25. Given points A1, A2, . . . , An on a unit circle, dividing the circle into n arcs.
Let P be the midpoint of the greatest of these n arcs. Prove that
(a) PA1  . . .  PAn  2, for n ¼ 2, 3, 4,
(b) There exist points A1, A2, . . . , An, such that PA1  PA2  . . .  PAn < 2, for n  5.
5.5.26. (a) Given points A1, A2, . . . , An on a unit circle. Prove that on this circle
exists a point P, such that PA1  PA2 . . .  PAn  2.
(b) Given points A1, A2, . . . , An on a plane and a circle with a center O. Prove that
on that circle exists a point P, such that PA1  PA2  :::  PAn 
1
2n1
ðOP þ OA1 ÞðOP þ OA2 Þ  :::  ðOP þ OAn Þ.
5.5.27. Let point P be inside of the convex quadrilateral ABCD. Prove that at least
one of angles ∠PAB, ∠PBC, ∠PCD, ∠PDA is not greater than π4.
5.5.28. (a) Let P be the intersection point of the cevians CD, BE of the triangle ABC
(D 2 AB, E 2 AC) and BD  CD. Prove that AD þ DP > AE þ EP.
(b) Let P be the intersection point of the cevians CD, BE of the isosceles triangle
ABC (AB ¼ AC) and AD < AE. Prove that AD þ DP > AE þ EP.
5.5.29. Let O be the intersection point of the diagonals AC and BD of the convex
quadrilateral ABCD. Let circles S1, S2, S3, S4 with centers O1, O2, O3, O4 be the
incircles of triangles AOB, BOC, COD, DOA, respectively. Prove that
(a) The sum of the diameters of the circles S1, S2, S3, S4 is less than or equal to
 pffiffiffi
2  2 ðAC þ BDÞ,
(b) O1O2 þ O2O3 þ O3O4 þ O4O1 < AC þ BD.
5.5.30. Let D be a point inside of angle ACB, such that ∠DAC ¼ ∠ DCB and
∠DBC ¼ ∠ DCA. Prove that CD  R, where R is the circumradius of triangle ABC.
222 5 Application of Trigonometric Inequalities

5.5.31. Let M be a point inside of the convex n-gon A1A2 . . . An.


Denoted by MAi ¼ Ri, ρ(M, AiAi þ 1) ¼ di, i ¼ 1, 2, . . . n, where An þ 1
A1.
Prove that, R1 þ R2 þ . . . þ Rn  cos πðd1 þd12 þ...þdn Þ.
n

5.5.32. Prove that


pffiffiffiffiffiffiffiffiffiffiffiffiffiffiffiffiffiffiffiffiffiffiffiffiffiffiffiffiffiffiffiffiffiffiffiffiffiffiffiffiffiffiffiffiffiffiffiffiffiffiffiffiffiffiffiffiffiffiffiffiffiffiffiffiffiffiffiffiffiffiffiffiffiffiffiffiffiffiffiffiffiffiffiffiffiffiffi 1
2 ðR1 þ R3 þ ::: þ R2k1 ÞðR2 þ R4 þ ::: þ R2k Þ  π ðd 1 þ d2 þ ::: þ d2k Þ,
cos 2k

(see the notations of the problem 5.5.31).


5.5.33. Prove the following inequalities for a convex 2n-gon A1A2 . . . A2n.
π 
(a) 4sin 2 A1 A2nþ1 þ A2 A2nþ2 þ ::: þ An A22n  ðA1 A2 þ Anþ1 Anþ2 Þ2 þ
2n
þðA2 A3 þ Anþ2 Anþ3 Þ2 þ ::: þ ðAn Anþ1 þ A2n A1 Þ2 ,
π 
(b) 4tg 2 B1 B2nþ1 þ B2 B2nþ2 þ ::: þ Bn B22n  ðA1 A2 þ Anþ1 Anþ2 Þ2 þ
2n
þðA2 A3 þ Anþ2 Anþ3 Þ2 þ ::: þ ðAn Anþ1 þ A2n A1 Þ2 ,
where points B1B2 . . . B2n are the midpoints of segments A1A2, A2A3, . . . ,
A2nA1 respectively.
5.5.34. Given two circles with radiuses r and R. Let the distance between the centers
of these circles be equal to d, (d > 0). Given that any of those circles touches three
sides of a convex quadrilateral with perimeter p (Figure 5.14).
5.5.35. Given two circles on a plane, such that any of those circles touches three
sides of a convex quadrilateral ABCD (see Figure 5.14). For which quadrilateral
ABCD expression pS2 is the greatest possible, where S is the area of quadrilateral
ABCD and p is its perimeter.
5.5.36. Let two circles have the radiuses r and R, and any of those two circles
touches three sides of a convex quadrilateral ABCD (see Figure 5.14). Prove that
S  32Rþr
pffiffiffiffi p2 , where S is the area of quadrilateral ABCD and p is its perimeter.
Rr

5.5.37. Given two circles with radiuses r and R not having any common interior
points. Given also that any of those circles touches three sides of a convex
quadrilateral with perimeter p. Prove that
qffiffiffiffiffiffiffiffiffiffiffiffiffiffiffiffiffiffiffiffiffiffiffiffiffiffiffiffiffiffiffiffiffiffiffiffiffiffiffiffiffiffiffiffiffiffiffiffiffiffiffiffiffiffiffiffiffiffi
pffiffiffiffiffi
9ðR þ r Þ þ 81ðR þ r Þ2  144ðR þ r Þ Rr
p :
2

5.5.38. Given that all angles of triangle ABC are greater π4 : Let C1, A1, B1 be points
on sides AB, BC, AC of triangle ABC, respectively. Consider the circumcircle of
triangle ABC and for any points M, N, belonging to the inner part of this circle,
denote the length of the chord passing through points M, N by X(M, N ). Prove that
(a) A1B1 þ B1C1 þ A1C1 > X(A1, B1),
(b) max(AB1 þ B1C1 þ AC1, A1B þ BC1 þ A1C1, A1B1 þ B1C þ A1C) > X(A1, B1).
5.5 Using Trigonometric Inequalities for Proving Geometric Inequalities 223

5.5.39. Let r be the inradius of triangle ABC and let M, M0 be given points inside of
triangle ABC, such that ∠MAB ¼ ∠ M0 AC and ∠MBA ¼ ∠ M0 BC.
Denote the distances da, db, dc and d0a , d0b , d0c from pointsM and M0 to lines BC,
AC, AB, respectively. Find the greatest possible value of the product da d b d c d0a d 0b d0c :
5.5.40. Inside of triangle ABC is given a point M. Let da, db, dc be the distances from
point M to lines BC, CA, AB and Ra, Rb, Rc be the distances from point M to vertices
A, B, C, respectively. Prove that
sffiffiffiffiffiffiffiffiffiffiffiffiffiffiffiffiffiffiffiffiffiffiffiffiffiffiffiffiffiffiffiffiffiffiffiffiffiffiffiffiffiffiffiffiffiffiffiffiffiffiffiffiffiffiffiffiffiffiffiffiffi
da db dc
maxðRa ; Rb ; Rc Þ  d2a þ d 2b þ d2c þ :
maxðda ; db ; d c Þ

5.5.41. Let the area of a convex pentagonABCDE be equal to S and the


circumradiuses of triangles ABC, BCD, CDE, DEA, EAB be equal to R1, R2, R3, R4,
R5. Prove that R41 þ R42 þ R43 þ R44 þ R45  5sin 24108 S2 :

Solutions

5.5.1. (a) Using the problems 5.3.6 and 5.1.4a, we deduce that Rþr R ¼ cos αþ
cos β þ cos γ  32. Therefore, R  2r.
(b) According to the problem 5.3.4, we need to prove that cos 2 βγ 2 
α β γ
 βγ

α 2
8 sin 2 sin 2 sin 2, or cos 2  2 sin 2  0.
pffiffiffi   
5.5.2. (a) Note that a2 þ b2 þ c2  4 3S ¼ p 2ffi ða  bÞ2 þ 4ab 1  cos γ  π3  0:
ffiffi
Thus, it follows that a2 þ b2 þ c2  4 3S..
pffiffiffi   
(b) Note that 3a2 þ 3b2  c2  4 3S ¼ 2ða  bÞ2 þ 4ab 1  sin γ  π6  0:
pffiffiffi
Therefore, 3a2 þ 3b2  c2  4 3S.
2 ðbcÞ2   
(c) Note that a3 þ bc6 S¼ 3 þ 5bc
6 1  sin α þ arccos5
3
 0:
2
Therefore, a3 þ bc
6  S.

5.5.3. According to the problem 5.2.9, it follows that tgα þ tgβ þ tgγ ¼ a2 þb2 þc
4S
2 8R2

< 0 (see the problem 5.3.12). Thus, we obtain that a þ b þ c < 8R .


2 2 2 2

5.5.4. (a) According to the problem 5.3.12, it follows that 4S


a2 þb2 þc2 8R2
¼
tgα þ tgβ þ tgγ > 0. Consequently, a2 þ b2 þ c2 > 8R2.
(b) Note that

b þ c  2R  2r ¼ a þ 2ðp  aÞ  2R  2r ¼
α  α α 1  r α α α
¼ 2R sin α þ 2rctg  2R  2r ¼ 2R cos  sin  sin cos  sin ¼
2 2 2 sin α R 2 2 2
α π   2
pffiffiffi β  γ pffiffiffi α π  β  γ pffiffiffi
¼ 2 2Rcos þ 2cos  2 sin þ > 0, as 2 cos > 2:
2 4 2 2 4 2
224 5 Application of Trigonometric Inequalities

Remark If ABC is a right angle triangle, then a2 þ b2 þ c2 ¼ 2c2 ¼ 8R2


5.5.5. If triangle ABC is non-acute, then a2 þ b2 þ c2  8R2 (see the problem 5.5.3).
2
Thus, a2 þ b2 þ c2 < 8R2 þ 4r2 and a2 þ b2 þ c2 < 8R2 þ 2Rr Rr .
If ABC is an acute triangle, then:
(a) According to the problem 5.2.2a, we have that a2 þb24pr
þc2 8R2
¼ tgα þ tgβ þ tgγ 
ctg α2 þ ctg β2 þ ctg 2γ ¼ pr (see the problems 5.3.12 and 5.3.2). Therefore, a2 þ b2
þ c2  8R2 þ 4r2.
(b) According to the problems 5.3.12 and 5.3.4, we need to prove that
2r2
tgαþtgβþtgγ  14 sin α sin β sin γ .
4S
2 2 2

As S ¼ pr ¼ Rr(sinα þ sin β þ sin γ) and tgα þ tgβ þ tgγ ¼ tgαtgβtgγ (see the
proof of the problem 5.3.12), then it remains to prove that
 2
sin α2 sin β2 sin 2γ
cos α cos β cos γ  1 α β γ
ðsee the proof of the problem 5:3:3Þ:
4  sin 2 sin 2 sin 2

We have to prove that


 2
α β γ α β γ 1
sin sin sin þ cos α cos β cos γ sin sin sin  cos α cos β cos γ  0,
2 2 2 2 2 2 4
or
  
2γ 2αβ 2γ αβ γ γ
 
γ 2
cos αβ
2  sin 2 sin þ 2 cos  cos cos  sin sin cos γ
 2 2 2 2 2 2
2αβ 2γ
 cos  cos cos γ  0:
2 2
ð5:22Þ
αβ
Let α  β  γ and x ¼ cos we need to prove that for 0  x  1, we have the
2 ,
inequality:
 2     
f ðxÞ ¼ x  sin 2γ sin 2 2γ þ 2 x2  cos 2 2γ x  sin 2γ sin 2γ cos γ  x2  cos 2 2γ
cos γ  0.  
Indeed, we have that f 0 ðxÞ ¼ 6 cos γ sin 2γ x2 þ 2 sin 2 2γ  cos γ  2 cos γsin 2 2γ
x  2sin 3 2γ  2cos 2 2γ sin 2γ cos γ. Note that, if f0 (x1) ¼ f0 (x2) ¼ 0. Then, according to
the Vieta’s theorem x1x2 < 0.
We need to prove that f0 (1) < 0. Indeed, we have that
γ  γ γ γ γ γ
f 0 ð1Þ ¼ 6cos γ sin þ 2 sin 2  cos γ  2 cos γsin2  2sin 3  2cos 2 sin cos γ ¼
2 2 2 2 2 2
 γ  γ γ  γ γ  γ 1
¼ 2 1  sin 2sin 3 sin  1 þ 2sin2 þ sin  1 < 0, since 0 < sin  ,
2 2 2 2 2 2 2
 
2sin 3 2γ sin 2γ  1 < 0 and 2sin 2 2γ þ sin 2γ  1  0.
5.5 Using Trigonometric Inequalities for Proving Geometric Inequalities 225

We have obtained that, for x 2 [0; 1], f0 (x) < 0. Therefore, for 0  x  1
 2  
f ðxÞ  f ð1Þ ¼ sin 4 2γ 2 sin 2γ  1  0. Hence, f cos αβ
2  0, this means that
(5.22) holds true.
(c) We have that a2 þ b2 þ c2 ¼ 2p2  2r2  8Rr (see the proof of problem 5.3.7),
then from the problem 5.5.10b we obtain that a2 þ b2 þ c2  24Rr  12r2.
(d) Note that b2 þ c2 þ R2  a2 ¼ R2((2 cos α  cos(β  γ))2 þ sin2(β  γ))  0.
Thus, it follows that b2 þ c2 þ R2  a2.
5.5.6. Since a2 þ b2 þ c2 ¼ 2p2  2r2  8Rr (see the proof of the problem 5.3.7),
then according to the problem 5.5.5, we have that 2p2  2r2  8Rr  8R2 þ 4r2.
Hence, p2  4R2 þ 4Rr þ 3r2.
5.5.7. (a) According to the problem 5.3.12, it follows that 2pr
p2 ð2RþrÞ2
¼ tgα þ tgβþ
tgγ > 0. Consequently, p > 2R þ r.
Second Solution Let maxðα; β; γ Þ ¼ γ < π2, we have that a sin 2γ þ b sin 2γ ¼ c cos φ
(see Figure 5.4).   
Note that φ ¼ β  90  2γ < 2γ . Therefore, ða þ bÞ sin 2γ > c  cos 2γ , thus
cð sin 2γ þ cos 2γ Þ
p> 2 sin 2γ
.
  
c γ γ 1 γ
p  2R  r ¼ p   ðp  cÞtg ¼ p 1  tg  c  tg >
 sin γ  2   2 sin
 γ 2
γ γ γ γ γ
c sin þ cos cos  sin c 1  2sin 2
> 2 2 2 2  2 ¼ 0:
γ γ 2 sin γ
2 sin cos
2 2

γ 
(b) We have that l2a a2 þ l2b b2  h2a a2 þ h2b b2 ¼ 2h2c c2 > l2c c2 , since hlcc ¼ sin þβ
pffiffi  
2
> 22 (135 > 2γ þ β > γþβ 2 > 45 ).

5.5.8. (a) According to the problems 5.3.12 and 5.2.9, it follows that 2pr
p2 ð2RþrÞ2
¼
tgα þ tgβ þ tgγ < 0, consequently, p < 2R þ r.

γ
Second Solution Let max(a, b, c) ¼ c, γ > 90 , then 2 > 450 . Thus, p  c < r,
consequently, p < c þ r < 2R þ r.

Figure 5.4 j

g/2
r

g/2 b
226 5 Application of Trigonometric Inequalities

Figure 5.5 C

R
A B
C1
R R
O

(b) If triangle ABC is not obtuse, then according to the Carnot’s theorem
ka þ kb þ kc ¼ R þ r (see the proof of problem 2.4.11). Then, it follows that
ma  R þ ka, mb  R þ kb, mc  R þ kc.
Therefore, ma þ mb þ mc  4R þ r  min( p þ 2R, 4R þ r).
If triangle ABC is obtuse (see Figure 5.5).
According to the problem 1.1.8a, we have that mc þ 2c < 2R, ma < b2 þ 2c, mb
< 2 þ 2c (see problem 1.1.7a).
a

Therefore, ma þ mb þ mc < p þ 2R < 4R þ r (see problem 5.5.8a).


5.5.9. (a) We have that
 
a2 þ b2 þ c2  ða  bÞ2  ðb  cÞ2  ða  cÞ2 ¼ 2ðab þ bc þ acÞ  a2 þ b2 þ c2 ¼
¼ 2ðp2 þ r 2 þ 4Rr Þ  ð2p2  2r 2  8Rr Þ ¼ 4r 2 þ 16Rr,

(see the proof of the problem 5.3.7). pffiffiffi


ppffiffi
According to the problems 5.1.9 and 5.3.14, we have that 4Rþr

3. Hence,
pffiffiffi
2 2 2
a þ b þ c  ða  bÞ  ðb  cÞ  ða  cÞ ¼ 4r ð4R þ r Þ  4 3pr ¼ 4 3S.
2 2 2

Second Solution Let max(α, β, γ) ¼ γ. pffiffi



If γ  120 , then c2 ¼ a2 þ b2  2ab cos γ  a2 þ b2 þ ab, S ¼ ab
2 sin γ  4 ab.
3

Therefore,

2ab þ 2bc þ 2ac  a2  b2  c2 ¼ 2ab þ 2cða þ bÞ  a2  b2  c2 >


pffiffiffi
> 2ab þ 2c2  a2  b2  c2  3ab  4 3S,
pffiffiffi
thus 2ab þ 2bc þ 2ac  a2  b2  c2 > 4 3S.

If γ < 120 , then inside of triangle ABC exists a point M, such that ∠AMB ¼

∠ BMC ¼ ∠ CMA ¼ 120 . pffiffiffiffiffiffiffiffiffiffiffiffiffiffiffiffiffiffiffiffiffiffiffiffi pffiffiffiffiffiffiffiffiffiffiffiffiffiffiffiffiffiffiffiffiffiffiffiffi
Let MA ¼ x, MBffi ¼ y, p
pffiffiffiffiffiffiffiffiffiffiffiffiffiffiffiffiffiffiffiffiffiffiffiffi MCffiffi ¼ z, then a ¼ y þ yz þ z , b ¼ x þ xz þ z ,
2 2 2 2

c ¼ x þ y þ xy S ¼ 4 ðxy þ yz þ zxÞ. Consequently,


2 2 3
5.5 Using Trigonometric Inequalities for Proving Geometric Inequalities 227

pffiffiffi pffiffiffiffiffiffiffiffiffiffiffiffiffiffiffiffiffiffiffiffiffiffiffiffiffipffiffiffiffiffiffiffiffiffiffiffiffiffiffiffiffiffiffiffiffiffiffiffiffi
2ab þ 2bc þ 2ac  a2  b2  c2  4 3S ¼ 2 x2 þ xy þ y2 y2 þ yz þ z2 þ
pffiffiffiffiffiffiffiffiffiffiffiffiffiffiffiffiffiffiffiffiffiffiffiffiffipffiffiffiffiffiffiffiffiffiffiffiffiffiffiffiffiffiffiffiffiffiffiffiffi pffiffiffiffiffiffiffiffiffiffiffiffiffiffiffiffiffiffiffiffiffiffiffiffipffiffiffiffiffiffiffiffiffiffiffiffiffiffiffiffiffiffiffiffiffiffiffiffi 
þ x2 þ xy þ y2 x2 þ xz þ z2 þ y2 þ yz þ z2 x2 þ xz þ z2  ðx þ y þ zÞ2 
rffiffiffiffiffiffiffiffiffiffiffiffiffiffiffiffiffiffiffiffiffiffiffiffiffiffiffiffiffiffiffiffiffiffiffiffi
pffiffi 2 rffiffiffiffiffiffiffiffiffiffiffiffiffiffiffiffiffiffiffiffiffiffiffiffiffiffiffiffiffiffiffiffiffiffiffiffi
 
x 2
 2 pffiffi 2
2 y þ 2 þ 23x y þ 2z þ 23z þ
rffiffiffiffiffiffiffiffiffiffiffiffiffiffiffiffiffiffiffiffiffiffiffiffiffiffiffiffiffiffiffiffiffiffiffi
pffiffi 2ffirffiffiffiffiffiffiffiffiffiffiffiffiffiffiffiffiffiffiffiffiffiffiffiffiffiffiffiffiffiffiffiffiffiffiffiffi
 
y 2
 2 pffiffi 2
þ x þ 2 þ 23y x þ 2z þ 23z þ
rffiffiffiffiffiffiffiffiffiffiffiffiffiffiffiffiffiffiffiffiffiffiffiffiffiffiffiffiffiffiffiffiffiffiffi !
  pffiffi 2 rffiffiffiffiffiffiffiffiffiffiffiffiffiffiffiffiffiffiffiffiffiffiffiffiffiffiffiffiffiffiffiffiffiffiffiffi
  pffiffi 2
y 2 x 2 2
þ zþ2 þ 2 3y
z þ 2 þ 2 x  ðx þ y þ zÞ 
3


x  z  3xz  y  z  3yz  y  x
2 yþ yþ þ þ xþ xþ þ þ zþ zþ þ
2 2 4 2 2 4 2 2

3xy
þ  ðx þ y þ zÞ2 ¼ 0,
4
pffiffiffi
thus 2ab þ 2bc þ 2ac  a2  b2  c2  4 3S.
Remark The inequality
pffiffiffiffiffiffiffiffiffiffiffiffiffiffiffiffiffiffiffiffiffiffiffiffiffipffiffiffiffiffiffiffiffiffiffiffiffiffiffiffiffiffiffiffiffiffiffiffiffi pffiffiffiffiffiffiffiffiffiffiffiffiffiffiffiffiffiffiffiffiffiffiffiffipffiffiffiffiffiffiffiffiffiffiffiffiffiffiffiffiffiffiffiffiffiffiffiffi
x2ffiffiffiffiffiffiffiffiffiffiffiffiffiffiffiffiffiffiffiffiffiffiffiffi
p þ yz þ z2 þ
þ xy þ y2 py2ffiffiffiffiffiffiffiffiffiffiffiffiffiffiffiffiffiffiffiffiffiffiffiffi ffi y2 þ yz þ z2 z2 þ zx þ x2 þ
þ z þ zx þ x x þ xy þ y  ðx þ y þ zÞ2
2 2 2 2

holds true for arbitrary values of x, y, z.


Third Solution We need to prove that 2ab þ 2bc þ 2ac  2ðOA þ OB þ OCÞ2 
pffiffiffi
a2 þ b2 þ c2 þ 4 3S, where  O is the incenter of triangle
 ABC.
We have that 1a þ 1b þ 1c a  OA2 þ b  OB2 þ c  OC2  ðOA þ OB þ OCÞ2 and
a  OA2 þ b  OB2 þ c  OC2 ¼ abc. Consequently, ab þ bc þ ac  (OA þ OB þ OC)2,
(see the proof of the problem 4.1.8b).
We need to prove that for any point M in plane p ABC
ffiffiffi the following inequality
holds true: 2ðMA þ MB þ MCÞ2  a2 þ b2 þ c2 þ 4 3S.
Let ∠BMC ¼ α1, ∠AMC ¼ β1, ∠AMB ¼ γ 1, then we have that
pffiffiffi pffiffiffi pffiffiffi
4 3ðSAMB þ SBMC þ SCMA Þ ¼ 2 3  MA  MB sin γ 1 þ 2 3  MB
pffiffiffi
 MC sin α1 þ 2 3  MA  MC sin γ 1 ð5:23Þ

Note that sin(x  30 )  1, consequently
pffiffiffi
2 3 sin x  2 cos x þ 4 ð5:24Þ
228 5 Application of Trigonometric Inequalities

According to (5.23) and (5.24), we deduce that


pffiffiffi
4 3S  2  MA  MB cos γ 1 þ 4  MA  MB þ 2  MB  MC cos α1 þ
þ4  MB  MC þ 2  MA  MC cos β1 þ 4  MA  MC ¼ 2ðMA þ MB þ MCÞ2
a2  b2  c2 ,

hence
pffiffiffi
2ðMA þ MB þ MCÞ2  a2 þ b2 þ c2 þ 4 3S: ð5:25Þ

Remark The equality in (5.25) holds true, only if ∠BMB ¼ ∠ CMA ¼ ∠ BMC ¼
 
120 . Hence, if point M0 is such that ∠BM0A ¼ ∠ CM0A ¼ ∠ BM0C ¼ 120 and
M=6
M0 , then MA þ MB þ MC > M0A þ M0B þ M0C.
(b) Let a  b  c and a ¼ x þ y, b ¼ y þ z, c ¼ x þ z, then y  x  z  0. One needs to
prove that ðx þ yÞ2 þ ðy þ zÞ2 þ ðx þ zÞ2  12 ððx  zÞþ ðy  zÞ þ ðy  xÞÞ2 
pffiffiffiffiffiffiffiffiffiffiffiffiffiffiffiffiffiffiffiffiffiffiffiffiffiffiffiffiffiffiffi pffiffiffiffiffiffiffiffiffiffiffiffiffiffiffiffiffiffiffiffiffiffiffiffiffiffiffiffiffiffiffi
4 3xyzðx þ y þ zÞ, or xðx þ y þ zÞ þ 3xyz  2 xðx þ y þ zÞ3yz.
pffiffiffiffiffi
Obviously the last inequality holds true, as u þ v  2 uv.
(c) We have that

0 0 0
 0 0 0
aa þ bb þ cc 2 sin α sin α þ sin β sin β þ sin γ sin γ
pffiffiffiffiffiffi0 ¼ p ffiffiffiffiffiffiffiffiffiffiffiffiffiffiffiffiffiffiffiffiffiffiffiffiffiffiffiffiffiffiffiffiffiffiffiffiffiffiffiffiffiffiffiffiffiffiffiffiffiffiffiffiffiffiffiffiffiffiffiffiffi
0
ffi 
SS sin α sin β sin γ sin α0 sin β sin γ 0
p ffiffiffiffiffiffiffiffiffiffiffiffiffiffiffiffiffiffiffiffiffiffiffiffiffiffiffiffiffiffiffiffiffiffiffiffiffiffiffiffiffiffiffiffiffiffiffiffiffiffiffiffiffiffiffiffiffiffiffiffiffi
0

3
sin α sin β sin γ sin α0 sin β sin γ 0
 6 pffiffiffiffiffiffiffiffiffiffiffiffiffiffiffiffiffiffiffiffiffiffiffiffiffiffiffiffiffiffiffiffiffiffiffiffiffiffiffiffiffiffiffiffiffiffiffiffiffiffiffiffiffiffiffiffiffiffiffiffiffi
0

sin α sin β sin γ sin α0 sin β sin γ 0
6
¼p ffi
ffiffiffiffiffiffiffiffiffiffiffiffiffiffiffiffiffiffiffiffiffiffiffiffiffiffiffiffiffiffiffiffiffiffiffiffiffiffiffiffiffiffiffiffiffiffiffiffiffiffiffiffiffiffiffiffiffiffiffiffiffi
0
6
sin α sin β sin γ sin α0 sin β sin γ 0
6 pffiffiffi
 rffiffiffiffiffiffiffiffiffiffiffiffiffiffi
 pffiffi2 ¼ 4 3,
6 3 3
8

0 0 0
pffiffiffiffiffiffiffiffi0ffi
(see problem 5.1.12). Thus, it follows that aa þ bb þ cc  4 3SS .
Remark It also holds true for the following inequality
 pffiffiffiffi pffiffiffiffi2   pffiffiffiffi pffiffiffiffi2   pffiffiffiffi pffiffiffiffi2 
0 0 0 0 0 0 0 0
a a  b  c þb b  a  c þc c  a0  b
pffiffiffiffiffiffiffiffi0ffi
4 3SS :
5.5 Using Trigonometric Inequalities for Proving Geometric Inequalities 229

(d) Let BC ¼ a, AB ¼ c, AC ¼ b, AD ¼ x, BD ¼ y, CD ¼ z.
Then, according to problem 5.5.9а, we have that

2ab þ 2bc þ 2ac  a2  b2  c2 þ 2bx þ 2bz þ 2xz  b2  x2  z2 þ 2ayþ


pffiffiffi
þ2az þ 2yz  y2  z2  a2 þ 2cx þ 2cy þ 2xy  c2  x2  y2  4 3S:

In order to prove this inequality, it is sufficient to prove that

2ax þ 2by þ 2cz  ab þ bc þ ac þ bx þ bz þ xz þ ay þ az þ yz þ cx þ cy þ xy


a2  b2  c2  x2  y2  z2 :

This is obvious, as it is equivalent to the following inequality

ða þ xÞ2 þ ðb þ yÞ2 þ ðc þ zÞ2  ða þ xÞðb þ yÞ þ ðb þ yÞðc þ zÞ


þ ðc þ zÞða þ xÞ:
 
(e) Let max(AB, BC, AC, AD, BD, CD) ¼ AB, then ∠ADB  120 , ∠ ACB  120 .

Note that ∠DAC < 120 .
 
Indeed, if ∠DAC  120 , then we deduce that ∠DAB þ ∠ BAC > ∠ DAC  120 ,
 
but ∠DAB < 60 , ∠ BAC < 60 .
This leads to a contradiction.

Without loss of generality one can assume that ∠ADC  120 .
Let AB ¼ c, BC ¼ a, AC ¼ b, AD ¼ x, BD ¼ y, CD ¼ z. We have that c2  a2 þ b2
þ ab and b2  x2 þ z2 þ xz, therefore
pffiffiffiffiffiffiffiffiffiffiffiffiffiffiffiffiffiffiffiffiffiffi pffiffiffiffiffiffiffiffiffiffiffiffiffiffiffiffiffiffiffiffiffiffiffiffi
AC  BD þ BC  AD þ AB  CD ¼ ax þ by þ cz  ax þ y x2 þ z2 þ xz þ z a2 þ b2 þ ab 
pffiffiffi pffiffiffi pffiffiffi pffiffiffi pffiffiffi pffiffiffi pffiffiffi
3 3 3 3 3 3 3
 ax þ ðx þ zÞy þ ða þ bÞz > ax þ xy þ zy þ az þ bz >
pffiffiffi 2 pffiffiffi 2 pffiffiffi pffiffiffi 2 pffiffiffi 2 pffiffiffi2 pffiffiffi2 pffiffiffi2
3 3 3 3 3 3 3 3
> aðx þ zÞ þ xy þ zy þ bz > ab þ xy þ zy þ bz 
2 2 2 2 2 2 2 2
 2ðABCÞ þ 2ðABDÞ þ 2ðBDCÞ þ 2ðADCÞ ¼ 2S:

Hence, we obtain that AC  BD þ BC  AD þ AB  CD > 2S.


(f) Let max(∠EAC, ∠ACE, ∠AEC) ¼ ∠ EAC.

pffiffiffiffiffiffiffiffiffiffiffiffiffiffiffiffiffiffiffiffiffiffiffiffiffiffiffiffiffiffiffiffiffiffiffiffiffiffiffiffiffiffiffiffi pffiffi
If ∠EAC  120 ,then CE  AC2 þ AE2 þ AC  AE  23 ðAC þ AEÞ:
Thus, it follows that

ACðBD þ BF  DFÞ þ CEðBD þ DF  BFÞ þ AEðBF þ DF  BDÞ >


pffiffiffi pffiffiffi
3 3
> ACðBD þ BF  DFÞ þ ðAC þ AEÞðBD þ DF  BFÞþ
2
pffiffiffi 2
3 pffiffiffi pffiffiffi pffiffiffi pffiffiffi
þ AEðBF þ DF  BDÞ ¼ 3AC  BD þ 3AE  DF  2 3SABCD þ 2 3SADEF
2pffiffiffi
¼ 2 3S:
230 5 Application of Trigonometric Inequalities

Hence, we obtain pffiffithat ffi ACðBD þ BF  DFÞ þ CEðBD þ DF  BFÞþ


AEðBF þ DF  BDÞ > 2 3S:

If ∠EAC < 120 , inside of triangle ACE exists a point T, such that ∠ATE ¼

∠ ATC ¼ ∠ CTE ¼ 120 .
Let AT ¼ x,ffi CT ¼ y,p
pffiffiffiffiffiffiffiffiffiffiffiffiffiffiffiffiffiffiffiffiffiffiffiffi ET ¼ z, BF ¼ m þ n, BD
ffiffiffiffiffiffiffiffiffiffiffiffiffiffiffiffiffiffiffiffiffiffiffiffi ¼ n þ k, DF ¼ m þ k, then AC ¼
pffiffiffiffiffiffiffiffiffiffiffiffiffiffiffiffiffiffiffiffiffiffiffiffi
x2 þ y2 þ xy, CE ¼ y2 þ z2 þ yz, AE ¼ x2 þ z2 þ xz and S ¼ SABTF þ SBCDT
þ SDEFT. Therefore
pffiffiffi pffiffiffi pffiffiffi pffiffiffi pffiffiffi pffiffiffi
2 3S  3xðm þ nÞ þ 3yðn þ kÞ þ 3zðm þ kÞ ¼ 3mðx þ zÞ þ 3nðx þ yÞ
pffiffiffi pffiffiffiffiffiffiffiffiffiffiffiffiffiffiffiffiffiffiffiffiffiffiffiffi pffiffiffiffiffiffiffiffiffiffiffiffiffiffiffiffiffiffiffiffiffiffiffiffiffi pffiffiffiffiffiffiffiffiffiffiffiffiffiffiffiffiffiffiffiffiffiffiffiffi
þ 3kðy þ zÞ  2m x2 þ z2 þ xz þ 2n x2 þ y2 þ xy þ 2k y2 þ z2 þ yz ¼
¼ ACðBD þ BF  DFÞ þ CEðBD þ DF  BFÞ þ AEðBF þ DF  BDÞ:

Thus, it follows that


pffiffiffi
ACðBD þ BF  DFÞ þ CEðBD þ DF  BFÞ þ AEðBF þ DF  BDÞ  2 3S:

See also the proof of problem 7.1.115b.


(g) Let a ¼ m þ n, b ¼ n þ k, c ¼ m þ k, then m > 0, n > 0, k > 0. One needs to prove
that
pffiffiffiffiffiffiffiffiffiffiffiffiffiffiffiffiffiffiffiffiffiffiffiffiffiffiffiffiffiffiffiffiffiffi
2mn þ 2nk þ 2mk  m2  n2  k2  3mnkðm þ n þ kÞ: ð5:26Þ

At first, we need the following lemma.


Lemma If m > 0, n > 0, k > 0, 2mn þ 2nk þ 2mk  m2  n2  k2 > 0, then for any
numbers x, y, z it holds true (2mn þ 2nk þ 2mk  m2  n2  k2)(myz þ nxz þ kxy) 
mnk(x þ y þ z)2.
In order to prove inequality (5.26), consider the case
2mn þ 2nk þ 2mk  m2  n2  k2  0. Hence (5.26) holds true.
In the second case 2mn þ 2nk þ 2mk  m2  n2  k2 > 0, according to the lemma
for x ¼ y ¼ z ¼ 1, we have that 2mn þ 2nk þ 2mk  m2  n2  k2  mþnþk 9mnk
:
p ffiffiffiffiffiffiffiffiffiffiffiffiffiffiffiffiffiffiffiffiffiffiffiffiffiffiffiffiffiffiffiffiffiffi
Note that mþnþk  3mnkðm þ n þ kÞ, therefore 2mn þ 2nk þ 2mk  m2  n2
9mnk
pffiffiffiffiffiffiffiffiffiffiffiffiffiffiffiffiffiffiffiffiffiffiffiffiffiffiffiffiffiffiffiffiffiffi
k2  3mnkðm þ n þ kÞ: Now, let us prove the lemma.
Let max(m, n, k) ¼ m and A ¼ 2mn þ 2nk þ 2mk  m2  n2  k2.
Thus, it follows that

A  4mk ¼ 2nk þ 2mn  m2  n2  k2  2mk ¼ ðm þ k  nÞ2 < 0:

One needs to prove that

mnkz2 þ ð2mnkðx þ yÞ  Aðmy þ nxÞÞz þ mnkðx þ yÞ2  Akxy  0,


5.5 Using Trigonometric Inequalities for Proving Geometric Inequalities 231

in order to prove this it is enough to prove the following inequality: D ¼ (2mnk(x þ


y)  A(my þ nx))2  4mnk(mnk(x þ y)2  Akxy)  0 or

n2 ðA  4mkÞx2 þ 2ðAmn þ 2mnkðk  m  nÞÞxy þ m2 ðA  4nkÞy2  0: ð5:27Þ

Note that A  4mk < 0 and


 
D ¼ 4ðAmn þ 2mnkðk  m  nÞÞ2  4ðA  4mkÞðA  4nkÞm2 n2 y2 ¼
 
¼ 16m2 n2 k2 y2 A þ ðk  m  nÞ2  4mn ¼ 0:

Therefore, the inequality (5.27) holds true.


See also the proof of problem 8.2.14.
5.5.10. (a) Note that

a3 þ b3 þ c3 þ 3abc  a2 b  b2 a  b2 c  c2 b  a2 c  c2 a ¼
8S2
¼ abc  ða þ b  cÞða  b þ cÞðb þ c  aÞ ¼ 4RS 
p
¼ 4SðR  2r Þ  0,

(see the problem 5.5.1a).


(b) Let a ¼ x þ y, b ¼ y þ z, c ¼ z þ x, then x ¼ p  b > 0, y ¼ p  c > 0, z ¼ p  a >
0 . Therefore,

1 3 
p2  ð16Rr  5r 2 Þ ¼ p  16Rpr þ 5r 2 p ¼
p
1 
¼ ðx þ y þ zÞ3  4ðx þ yÞðy þ zÞðx þ zÞ þ 5xyz ¼
p
1 
¼ x3 þ y3 þ z3 þ 3xyz  x2 y  y2 x  x2 z  z2 x  y2 z  z2 y ¼
p
1
¼ ðxyz  ðx þ y  zÞðx  y þ zÞðy þ z  xÞÞ,
p

(see the proof of the problem 5.5.10a).


If x, y, z are the lengths of the sides of a certain triangle, then according to
5.5.10a, p2  16Rr  5r2.
If x, y, z are not the lengths of the sides of a triangle, then xyz > 0  (x þ y  z)
(x  y þ z)(y þ z  x). Hence, p2 > 16Rr  5r2.
(c) We have that abc ¼ 4pRr and ab þ bc þ ac ¼ p2 þ r2 þ 4Rr (see the proof of
problem 5.3.7), thus
232 5 Application of Trigonometric Inequalities

 
1 1 1
ððp  aÞðp  bÞ þ ðp  bÞðp  cÞ þ ðp  cÞðp  aÞÞ 2 þ 2 þ 2 ¼
  a b c
ðab þ bc þ ac  p2 Þ ðab þ bc þ acÞ2  4abcp
¼
 a2 b2 c2 
2
ðr þ 4RÞ ðp þ r þ 4Rr Þ  16p2 R2 r
2 2
¼ :
16p2 R2 r

We have to prove that (4R þ r)(( p2 þ r2 þ 4rR)2  16p2R2r)  36p2R2r or


 
ð4R þ r Þp4  2r 34R2  r 2 p2 þ r 2 ð4R þ r Þ3  0: ð5:28Þ

Let f(x) ¼ (4R þ r)x2  2r(34R2  r2)x þ r2(4R þ r)3.


rð34R2 r2 Þ
According to the problems 5.5.1a and 5.5.10b, we have that xвер ¼ 4Rþr <
r34R2
4R ¼ 8, 5Rr < 16Rr  5r  p . Therefore, f( p )  f(16Rr  5r ) ¼ 4r (R  2r)
2 2 2 2 3 2

 0, thus (5.28) holds true.


This proof was provided by D. Harutyunyan, ninth grade.
(d) Note that
   2 
1 a2 b2 b2 c 2 c 2 a2 sin αsin 2 β sin 2 βsin 2 γ sin 2 γsin 2 α
þ 2 þ 2 ¼4 þ þ
R2 c 2 a b sin 2 γ sin 2 α sin 2 β
!
1 1 1
¼4 2
þ 2
þ ¼
ðctgβ þ ctgαÞ ðctgγ þ ctgβÞ ðctgα þ ctgγ Þ2
¼ 4ðctgαctgβ þ ctgβctgγ þ ctgγctgαÞ
!
1 1 1
þ þ  9,
ðctgβ þ ctgαÞ2 ðctgγ þ ctgβÞ2 ðctgα þ ctgγ Þ2

according to problem 5.5.10c.


5.5.11. Using the law of sines, we obtain that lb
sin γ ¼ a
sin ðβ2þαÞ
and la
sin γ ¼ b
sin ðα2þβÞ
.
2 sin γ 2 sin γ
Therefore, laSlb ¼ sin ðβ2þαÞ sin ðα2þβÞ
¼ cos γα γβ.
2 cos 2

We have to prove that

αβ αγ βγ


2 sin γ cos þ 2 sin β cos þ 2 sin α cos 
2 2 2
pffiffiffi αβ βγ γα
 3 3 cos cos cos :
2 2 2

Note that
5.5 Using Trigonometric Inequalities for Proving Geometric Inequalities 233

αβ αγ βγ


2 sin γ cos þ 2 sin β cos þ 2 sin α cos ¼
2 2 2
sffiffiffiffiffiffiffiffiffiffiffiffiffiffiffiffiffiffiffiffiffiffiffiffiffiffiffiffiffiffiffiffiffiffiffiffiffiffiffiffiffiffiffiffiffiffiffiffiffiffiffiffiffiffiffiffiffiffiffiffiffiffiffiffiffiffiffiffiffiffiffiffiffiffiffiffiffiffiffiffiffiffiffiffiffiffiffiffiffiffiffiffiffiffiffiffiffiffiffiffiffiffiffiffiffiffiffiffiffiffiffiffiffiffiffiffiffi
 
αβ 2γα 2βγ
4 sin γcos2 2 þ sin βcos2
þ sin αcos 2
þ
2 2 2
αβ αγ γβ γα
þ8 sin γ sin β cos cos þ 8 sin β sin α cos cos þ
2 2 rffiffiffiffiffiffiffiffiffiffiffiffiffiffiffiffiffiffiffiffiffiffiffiffiffiffiffiffiffiffiffiffiffiffiffiffiffiffiffiffiffiffiffiffiffiffiffiffiffiffiffi2
2
βγ βα ð2R þ r Þp2 þ 4Rr 2 þ r 3
þ8 sin α sin γ cos cos  þ ,
2 2 2R3
αβ βγ γα
þ8 cos cos cos ð sin α sin β þ sin β sin γ þ sin γ sin αÞ ¼
2 2 2
rffiffiffiffiffiffiffiffiffiffiffiffiffiffiffiffiffiffiffiffiffiffiffiffiffiffiffiffiffiffiffiffiffiffiffiffiffiffiffiffiffiffiffiffiffiffiffiffiffiffiffiffiffiffiffiffiffiffiffiffiffiffiffiffiffiffiffiffiffiffiffiffiffiffiffiffiffiffiffiffiffiffiffiffiffiffiffiffiffiffiffiffiffiffiffiffiffiffiffiffiffiffiffiffiffiffiffiffiffiffi

ð2R þ r Þp2 þ 4Rr 2 þ r 3 p2 þ 2Rr þ r 2 p2 þ 4Rr þ r 2
 þ8 ¼
2R3 8R2 4R2
qffiffiffiffiffiffiffiffiffiffiffiffiffiffiffiffiffiffiffiffiffiffiffiffiffiffiffiffiffiffiffiffiffiffiffiffiffiffiffiffiffiffiffiffiffiffiffiffiffiffiffiffiffiffiffiffiffiffiffiffiffiffiffiffiffiffiffiffiffiffiffiffiffiffiffiffiffiffiffiffi
 
p4 þ 4R2 þ 8Rr þ 2r 2 p2 þ ð4Rr þ r 2 Þ2
¼
2R2

(see the problems 5.3.16, 5.3.11, and 5.3.10a).


pffiffiffiffiffiffiffiffiffiffiffiffiffiffiffiffiffiffiffiffiffiffiffiffiffiffiffiffiffiffiffiffiffiffiffiffiffiffiffiffiffiffiffiffiffiffiffiffiffiffi
p4 þð4R2 þ8Rrþ2r 2 Þp2 þð4Rrþr2 Þ2 pffiffiffi 2
 3 3 p þ2Rrþr
2
We need to prove that 2R2 8R2
.
Denote m ¼ p þ 2Rr þ r , then according to the problems 5.5.6 and 5.5.10b, we
2 2

have that m1 ¼ 18Rr  4r2  m  4R2 þ 6Rr þ 4r2 ¼ m2.


Note that
qffiffiffiffiffiffiffiffiffiffiffiffiffiffiffiffiffiffiffiffiffiffiffiffiffiffiffiffiffiffiffiffiffiffiffiffiffiffiffiffiffiffiffiffiffiffiffiffiffiffiffiffiffiffiffiffiffiffiffiffiffiffiffiffiffiffiffiffiffiffiffiffiffiffiffiffiffiffiffiffi
  qffiffiffiffiffiffiffiffiffiffiffiffiffiffiffiffiffiffiffiffiffiffiffiffiffiffiffiffiffiffiffiffiffiffiffiffiffiffiffiffiffiffiffiffiffiffiffiffiffiffiffi
p4 þ 4R2 þ 8Rr þ 2r 2 p2 þ ð4Rr þ r 2 Þ2 ¼ ðp2 þ 4Rr þ r 2 Þ2 þ 4p2 R2 ¼
qffiffiffiffiffiffiffiffiffiffiffiffiffiffiffiffiffiffiffiffiffiffiffiffiffiffiffiffiffiffiffiffiffiffiffiffiffiffiffiffiffiffiffiffiffiffiffiffiffiffiffiffiffiffiffiffiffiffiffiffiffiffiffiffiffiffi
¼ ðm þ 2Rr Þ2 þ 4R2 ðm  2Rr  r 2 Þ:

It remains to prove that ðm þ 2Rr Þ2 þ 4R2 ðm  2Rr  r 2 Þ  27 2


16 m . That is
16 m þ 4RðR þ r Þm  8R r  0.
11 2 3

Consider a function f ðm1 Þ ¼ 11


16 m þ 4RðR þ r Þm  8R r on a segment [m1,
2 3

m2]. Since min f ðmÞ ¼ minðf ðm1 Þ; f ðm2 ÞÞ, then it remains to verify that f(m1) 
½m1 ;m2 
0 and f(m2)  0. Indeed, we have that
 
11
f ðm2 Þ ¼ ð4R2 þ 6Rr þ 4r 2 Þ 4RðR þ rÞ  ð4R2 þ 6Rr þ 4r 2 Þ  8R3 r ¼
16
1
¼ ð2R þ 3Rr þ 2r Þð10R  Rr  22r Þ  8R3 r 
2 2 2 2
4
1
 ð2R2 þ 3Rr þ 2r 2 Þð10R2  Rr  11RrÞ  8R3 r ¼
4
1 1
¼ ð2R2 þ 3Rr þ 2r 2 Þð5R  6rÞR  8R3 r  ð7Rr þ 2r 2 Þð5R  6rÞR  8R3 r ¼
2 2
Rr Rr
¼ ð19R  32Rr  12r Þ  ð6Rr  12r Þ  0: Therefore, f ðm2 Þ  0:
2 2 2
2 2
234 5 Application of Trigonometric Inequalities

We have that,
 
11 
f ðm1 Þ ¼ ð18Rr  4r Þ 4RðR þ r Þ 
2
9Rr  2r 2
 8R3 r ¼
8
ð9R  2r Þr  
¼ 32R2  67Rr þ 22r 2  8R3 r
4 
¼ 2Rr 32R2  67Rr þ 22r 2  8R3 r ¼
 
¼ 2Rr 28R2  67Rr þ 22r 2 ¼ 2Rr ðR  2r Þð28R  11r Þ  0:
Hence, f(m1)  0.

cos α
5.5.12. Note that HA ¼ AF

cos ð90 βÞ
¼ bsin β ¼ 2R cos α and HE ¼ HA sin(90  γ) ¼

2R cos α cos γ. In a similar way, we deduce that HB ¼ 2R cos β, HC ¼ 2R cos γ,


HD ¼ 2R cos β cos γ, HF ¼ 2R cos α cos β.
(a) According to the problem 5.2.7, we have that

4R2 cos 2 α þ 4R2 cos 2 β þ 4R2 cos 2 γ 


 16R2 cos 2 α cos 2 β þ 16R2 cos 2 β cos 2 γ þ 16R2 cos 2 γ cos 2 α:

Therefore, HA2 þ HB2 þ HC2  4(HE2 þ HD2 þ HF2).


(b) According to the problem 5.2.8b, we have that 2R sin α þ 2R sin β þ 2R sin γ
pffiffiffi
 2 3 ð2R cos α cos β þ 2R cos β cos γ þ 2R cos γ cos αÞ. Thus, AB þ BC þ
pffiffiffi
AC  2 3ðHD þ HE þ HFÞ.
(c) According to the problem 5.5.12b, we have that
1
ðHD þ HE þ HFÞ2  ðAF þ BD þ CE þ BF þ CD þ AEÞ2 
12
1 
 ðAF þ BD þ CEÞ2 þ ðBF þ CD þ AEÞ2 
6
1    
 3 AF2 þ BD2 þ CE2 þ 3 BF2 þ CD2 þ AE2 ¼ AF2 þ BD2 þ CE2 ,
6
since AF2  BF2 þ BD2  CD2 þ CE2  AE2 ¼ AH2  BH2 þ BH2  CH2 þ CH2
 AH ¼ 0.
2

5.5.13. Let the straight line l intersect lines DA, DB, and DC and form angles α, β, γ,
respectively. Then, cos2α þ cos2β þ cos2γ ¼ 1 (see the q proof of the problem
ffiffiffiffiffiffiffiffiffiffiffiffiffiffiffiffiffiffiffiffiffiffiffiffiffiffiffiffiffiffiffi 7.1.10).
 ffi
We have to prove that a sin α þ b sin β þ c sin γ  2 a þ b þ c 2 2 2 (see the
problem 5.4.4).
sin α sin β sin γ
The equality holds true, if and only if a ¼ b ¼ c , this means that
pffiffiffi pffiffiffi pffiffiffi
2a 2b 2c
sin α ¼ pffiffiffiffiffiffiffiffiffiffiffiffiffiffiffiffiffiffiffiffiffiffiffiffiffi , sinβ ¼ pffiffiffiffiffiffiffiffiffiffiffiffiffiffiffiffiffiffiffiffiffiffiffiffiffi , sinγ ¼ pffiffiffiffiffiffiffiffiffiffiffiffiffiffiffiffiffiffiffiffiffiffiffiffiffi ,
a þb þc
2 2 2 a þb þc
2 2 2 a þ b2 þ c 2
2
0 sffiffiffiffiffiffiffiffiffiffiffiffiffiffiffiffiffiffiffiffiffiffiffiffiffi sffiffiffiffiffiffiffiffiffiffiffiffiffiffiffiffiffiffiffiffiffiffiffiffiffi sffiffiffiffiffiffiffiffiffiffiffiffiffiffiffiffiffiffiffiffiffiffiffiffiffi1
B b þc a
2 2 2 a þc b
2 2 2
a2 þ b2  c2 C
@cos α ¼ , cosβ ¼ , cosγ ¼ A:
a2 þ b þ c 2 2
a2 þ b þ c2 2
a2 þ b2 þ c2
5.5 Using Trigonometric Inequalities for Proving Geometric Inequalities 235

Figure 5.6 A
A1 A2
M
a B2
b

O B1 B

Remark If max(a, b, c) ¼ a and a2 > b2 þ c2, then the equality cannot hold true.
5.5.14.
 Let ∠A ¼ α, ∠B ¼ β, ∠C ¼ γ and r be the inradius of triangle ABC. Then,
2r ctg α2 þ ctg β2 þ ctg 2γ and 2r ctg αþβ βþγ γþα
4 þ ctg 4 þ ctg 4 are the perimeters of
the given and obtained triangles, respectively.
According to the problem 5.4.1, we have that
   
α β γ αþβ βþγ γþα
2r ctg þ ctg þ ctg  2r ctg þ ctg þ ctg :
2 2 2 4 4 4

5.5.15. Consider Figure 5.6.


Let OA ¼ OB ¼ OM ¼ R, ∠AOM ¼ α, ∠ BOM ¼ β. Note that

R2
SOA1 MB1 ¼ SMOA1 þ SMOB1 ¼ ð sin 2α þ sin 2βÞ ¼
4
2
R R2
¼ sin ðα þ βÞ cos ðα  βÞ  sin ðα þ βÞ ¼ SAOB :
2 2

Therefore, SMA2 B2  SAA1 A2 þ SBB1 B2 , and the equality holds true for α ¼ β.
S 2 B2
Thus, SAA AMAþS BB B
 1 and the greatest value is reached, when M is the midpoint
1 2 1 2
of the arc AB
5.5.16. Let ∠A ¼ α, ∠ B ¼ β, ∠ C ¼ γ and ∠MAC ¼ α1, ∠ MCA ¼ γ 1, ∠ AMC ¼ β1
(see Figure 5.7).
Therefore, α1 < α  β, γ 1 < γ  β, and β1 > β, According to the problem 5.4.5, it
follows that ctg α21 þ ctg β21 þ ctg γ21 > ctg α2 þ ctg β2 þ ctg 2γ .
Þ2  α 
Note that ðABþBCþAC β γ
2

SABC ¼ 4p
pr ¼ r ¼ 4 ctg 2 þ ctg 2 þ ctg 2 , (see the problem
4p

5.3.2).  
Þ2
Similarly we obtain that ðAMþMCþAC SAMC ¼ 4 ctg α1
2 þ ctg β1
2 þ ctg γ1
2 .
2 2
Thus, ðAMþMCþAC
SAMC
Þ
> ðABþBCþAC
SABC
Þ
.
236 5 Application of Trigonometric Inequalities

Figure 5.7 B

b
M

b1

a a1 g1 g
A C

5.5.17. If point O is the circumcenter with the radius R of the given hexagon
ABCDEF and ∠COD ¼ ∠ DOE ¼ α, ∠AOB ¼ ∠ BOC ¼ β, ∠FOA ¼ ∠ EOF ¼ γ.

Then, α þ β þ γ ¼ 180 and using the problem 5.1.15, we obtain that

1 1
SACE ¼ R2 ð sin 2α þ sin 2β þ sin 2γ Þ  R2 ð sin α þ sin β þ sin γ Þ ¼
2 2
1 2
¼ R ð sin ðβ þ γ Þ þ sin ðα þ γ Þ þ sin ðα þ βÞÞ ¼ SBDF :
2

5.5.18. Let ∠A1OB1 ¼ 2γ, ∠A1OC1 ¼ 2β, and ∠B1OC1 ¼ 2α, where O is the incen-
ter of triangle ABC. Then, we have that 0 < α, β, γ < π2, α þ β þ γ ¼ π and
A1B1 þ B1C1 þ A1C1 ¼ 2r(sinα þ sin β þ sin γ), p ¼ r(tgαpffiffiffiþ tgβ þ tgγ). Therefore,
A1 B1 þ B1 C1 þ A1 C1 ¼ 2r ð sin α þ sin β þ sin γ Þ  3 3r  r(tgα þ tgβ þ tgγ) ¼
p (see the problems 5.1.6 and 5.2.1). We have that

2pr 2r 2 ðtgα þ tgβ þ tgγ Þ


¼ ¼ 8r ðtgα þ tgβ þ tgγ Þ cos α cos β cos γ ¼
R r ðtgα þ tgβÞ
2 sin ðπ  2γ Þ
¼ 8r sin α sin β sin γ ¼ 2r ð sin 2α þ sin 2β þ sin 2γ Þ:

R ¼ 2r ð sin 2α þ sin 2β þ sin 2γ Þ  2r ð sin α þ sin β þ sin γ Þ


Consequently, 2pr
¼ A1 B1 þ B1 C1 þ A1 C1 (see the problem 5.1.15).
5.5.19. Note that ∠COD ¼ π  2β, ∠ODC ¼ ∠OBC ¼ π2  α. Hence, ∠OCD ¼
π
2 þ β  γ.
R sin ðπ þβγ Þ
Using the law of sines, we obtain that OD ¼ sin 2πα ¼ R coscosðβγ Þ
α . Similarly,
ð2 Þ
we deduce that OE ¼ R coscosðγα
β
Þ
and OF ¼ R coscosðαβ
γ .
Þ

(a) We have that OD  OE  OF ¼ R3 cos ðαβÞcos ðβγÞcos ðγαÞ


cos αcos βcos γ  8R3 , (see the problem
5.2.4).
(b) Since
cos α
OD ¼ OD ¼ 1 þ OD ¼ 1 þ cos ðβγ Þ ¼ 1 þ 2 sin α cos ðβγ Þ ¼ 1 þ sin 2βþ sin 2γ .
AD AOþOD R sin 2α sin 2α
5.5 Using Trigonometric Inequalities for Proving Geometric Inequalities 237

Similarly, OE BE
¼ 1 þ sin 2αþ sin 2γ and OF ¼ 1 þ sin 2αþ sin 2β. Thus, we have to prove
sin 2β CF sin 2γ

that A ¼ sin 2βþ sin 2γ þ sin 2αþ sin 2γ þ sin 2αþ sin 2β  1, 5.
sin 2α sin 2β sin 2γ

Denote by x ¼ sin 2β þ sin 2γ, y ¼ sin 2α þ sin 2γ, z ¼ sin 2α þ sin 2β. It is clear
 y, z >0 and
that x,

sin 2α¼ yþzx
 2 ,
 sin 2β ¼ xþzy
2 , sin 2γ ¼ xþyz
2 . Thus,
A ¼ 2 y þ x þ 2 z þ x þ 2 z þ y  1, 5  1, 5, as for a > 0 we have that
1 x y 1 x z 1 y z

a þ 12  2.
5.5.20. Denote by φi ¼ 12 ∠Ai MAiþ1 , i ¼ 1, . . . , n, where An þ 1
A1. We have that

d 2i ¼ MAi  MAiþ1 cos ∠Ai MPi cos ∠Aiþ1 MPi ¼


cos ð∠Ai MPi þ cos ∠Aiþ1 MPi Þ þ cos ð∠Ai MPi  cos ∠Aiþ1 MPi Þ
¼ MAi  MAiþ1 
2
1 þ cos ð∠Ai MPi þ ∠Aiþ1 MPi Þ
 MAi  MAiþ1
2
 
∠Ai MPi þ ∠Aiþ1 MPi
¼ MAi  MAiþ1 cos 2 
2
∠Ai MPi þ ∠Aiþ1 MPi π
 MAi  MAiþ1 cos 2 φi , sinceφi  < :
2 2

Thus, we obtain that


pffiffiffiffiffiffiffiffiffiffiffiffiffiffiffiffiffiffiffiffiffi pffiffiffiffiffiffiffiffiffiffiffiffiffiffiffiffiffiffiffiffiffi pffiffiffiffiffiffiffiffiffiffiffiffiffiffiffiffiffiffiffiffiffi
d 1 d2 . . . dn  MA1  MA2  MA2  MA3  . . .  MAn  MA1  cos φ1  cos φ2  . . .  cos φn ¼
π
¼ R1  R2  . . .  Rn cos φ1  . . .  cos φn  cos nn R1  R2  . . .  Rn ðsee the problem 5:4:7Þ:

5.5.21. Let a and b(a  b) be the sides of the rectangle P1. We can assume that α is
the angle between the lines containing the greatest sides of the rectangles P1 and P2.
We have that d ¼ b cos α þ a sin α  c ¼ b sin α þ a cos α and b > b cos α þ a sin α.
Therefore, α  π4 and b > a 1sincosα α.
sin αþa cos α cos 2αðb sin αa cos αÞ
Thus, it follows that sin 2α  dc ¼ sin 2α  bb cos αþa sin α ¼ b cos αþa sin α 
a cos 2α
b cos αþa sin α  0.
Hence, we obtain that sin 2α  dc.
5.5.22. Let in a quadrilateral AB ¼ a, BC ¼ b, CD ¼ c, DA ¼ d and ∠ABC ¼ α,
∠ ADC ¼ β, p ¼ aþbþcþd
2 .
According to the problem 5.4.10, we have that

ab cd pffiffiffiffiffiffiffiffiffiffiffiffiffiffiffiffiffiffiffiffiffiffiffiffiffiffiffiffiffiffiffiffiffiffiffiffiffiffiffiffiffiffiffiffiffiffiffiffiffiffiffiffiffiffiffiffiffiffi
SABCD  sin α þ sin β  ðp  aÞðp  bÞðp  cÞðp  dÞ:
2 2
2
þb2 c2 d2
Note that 1 < a 2abþ2cd < 1. Thus, there exists an angle α 2 (0; π), such that
cos α ¼ a þb c2 d2
2 2
2abþ2cd .
238 5 Application of Trigonometric Inequalities

Let us construct a triangle r ABC with AB ¼ a,ffi BC ¼ b and ∠B ¼ α. Then,


ffiffiffiffiffiffiffiffiffiffiffiffiffiffiffiffiffiffiffiffiffiffiffiffiffiffiffiffiffiffiffiffiffiffi
pffiffiffiffiffiffiffiffiffiffiffiffiffiffiffiffiffiffiffiffiffiffiffiffiffiffiffiffiffiffiffiffiffiffiffiffiffiffiffi cd ð a Þ ð
2 þb2 þab c2 þd 2
Þ
AC ¼ a2 þ b2  2ab cos α ¼ abþcd . It is not difficult to verify
that AC < c þ d, AC þ c > d, AC þ d > c . Hence, there exists a quadrilateral
ABCD, so that AB ¼ a, BC ¼ b, CD ¼ c, AD ¼ d and ∠ABC ¼ α. Let ∠ADC ¼ β,
then AC2 ¼ a2 þ b2  2ab cos α ¼ c2 þ d2  2cd cos β ¼ c2 þ d2 þ 2cd cos α. Hence,
α þ β ¼ π and

1
SABCD ¼ ðab þ cd Þ sin α ¼
2
vffiffiffiffiffiffiffiffiffiffiffiffiffiffiffiffiffiffiffiffiffiffiffiffiffiffiffiffiffiffiffiffiffiffiffiffiffiffiffiffiffiffiffiffiffiffiffiffiffiffiffiffiffiffiffiffiffi
u  ! 2
1 u a2 þ b2  c 2  d 2 pffiffiffiffiffiffiffiffiffiffiffiffiffiffiffiffiffiffiffiffiffiffiffiffiffiffiffiffiffiffiffiffiffiffiffiffiffiffiffiffiffiffiffiffiffiffiffiffiffiffiffiffiffiffiffiffiffiffi
t
¼ ðab þ cd Þ 1  ¼ ðp  aÞðp  bÞðp  cÞðp  dÞ:
2 2ðab þ cd Þ

This ends the proof.


5.5.23. According to the problem 5.2.8c, we have that sin2α þ sin2β þ sin2
γ  (cosα þ cos β þ cos γ)2.
Þ2
Using the problems 5.3.8a and 5.3.6, we obtain that p r2R4Rr  ðRþr
2 2
2
R2
.
Thus, p  2R þ 8Rr þ 3r .
2 2 2

5.5.24. Denote by y þ z ¼ m, x þ z ¼ n, x þ y ¼ k, then x ¼ nþkm


2 , y¼
kþmn
2 ,
z¼ 2 .
mþnk

(a) One needs to prove that

nþk 4 kþm 4 mþn 4


a þ b þ c  a4  b4  c4  16S2 :
m n k

Note that
n  
nþk 4 kþm 4 mþn 4 m  k 4 m 4
a þ b þ c  a4  b4  c4 ¼¼ a4 þ b4 þ a þ c þ
m n k m n m k
  rffiffiffiffiffiffiffiffiffiffiffiffiffiffiffiffiffiffiffi rffiffiffiffiffiffiffiffiffiffiffiffiffiffiffiffiffiffi rffiffiffiffiffiffiffiffiffiffiffiffiffiffiffiffi
k m n 4 m 4 k 4 m 4 k 4 n 4
þ b4 þ c 4  a4  b4  c 4  2 a  b þ2 a  c þ2 b  c 
n k m n m k n k
a4  b4  c4 ¼ 2a2 b2 þ 2a2 c2 þ 2b2 c2  a4  b4  c4 ¼ 16S2 :

We have to prove that

nþk 2 kþm 2 mþn 2 pffiffiffi


a þ b þ c  a2  b2  c2  4 3S:
m n k
5.5 Using Trigonometric Inequalities for Proving Geometric Inequalities 239

Figure 5.8 A3 ·
2a2

A2 ·
2a1 · An
A1 ·

2a n

Note that

nþk 2 kþm 2 mþn 2


a þ b þ c  a2  b2  c2 ¼
m n k
n    
m 2 k 2 m 2 k 2 m 2
¼ a þ b þ
2
a þ c þ b þ c  a2  b2  c 2 
m n m k n k
rffiffiffiffiffiffiffiffiffiffiffiffiffiffiffiffiffiffiffi rffiffiffiffiffiffiffiffiffiffiffiffiffiffiffiffiffiffiffi rffiffiffiffiffiffiffiffiffiffiffiffiffiffiffiffiffi
n 2 m 2 k 2 m 2 k 2 n 2
2 a  b þ2 a  c þ2 b  c  a 2  b2  c 2 ¼
m n m k n k
pffiffiffi
¼ a2 þ b2 þ c2  ða  bÞ  ðb  cÞ  ða  cÞ2  4 3S
2 2

(see problem 5.5.9).


5.5.25. (a) Let A1 A2 ¼ 2α1 , A2 A3 ¼ 2α2 , . . ., An A1 ¼ 2αn (see Figure 5.8), where
α1 þ . . . þ αn ¼ π and let max(α1, α2, ..., αn) ¼ αn.
 2
For n ¼ 2, we have that PA1  PA2 ¼ 2 sin α22 and π > α2  π2. Therefore,
 2
PA1  PA2  2 sin π4 ¼ 2. It is clear that, the equality holds  true, only if α2 ¼ π2 .
2 α3 α3
For n ¼ 3, we have that PA1  PA2  PA3 ¼ 8sin 2 sin  α2 þ 2 ,πwhere α2  α1.
2 α3 þ2α2 α3 α3 α3 þ2α2
Note that PA1  PA2  PA3  8sin 6 sin α2 þ 2 , because 2 > 2  6 .
α3 þ2α2 π π
Denote by 6 ¼ α, then it is not difficult to prove that 4  α  6. Hence,
PA1  PA2  PA3  8sin2α sin 3α  4 sin α sin 3α ¼ 2(cos2α(1  2 cos 2α) þ 1)  2.
Obviously, PA1  PA2  PA3 ¼ 2, if and only if α3 ¼ α2 ¼ α1.
For, n ¼ 4 we need the following lemma.
Lemma Given points M, A, B, C, D on a unit circle (see Figure 5.9), so that
c ¼d
AB CD.
Then, MA  MD < MB  MC.
Indeed, let d
AB ¼ d c ¼ 2β, and MA
CD ¼ 2α, BC d ¼ 2γ. Then, we have that

MA  MD ¼ 4 sin γ sin ð2α þ β þ γ Þ ¼ 2ð cos ð2α þ βÞ  cos ð2α þ β þ 2γ ÞÞ <


< 2ð cos β  cos ð2α þ β þ 2γ ÞÞ:

As β < 2α þ β < π, thus MA  MD < 2(cosβ  cos(2α þ β þ 2γ)) ¼ 4 sin(α þ γ)


sin(α þ β þ γ) ¼ MB  MC.
240 5 Application of Trigonometric Inequalities

Figure 5.9 D ·
2a

C ·
2b · M
B · 2g
2a ·
A

2a2
A2 · A2 2a2
· A3 2a3
2a1 A3
2a3 ® A2¢ ·
2a3
· A4
A1 · A4º A3¢
·
A1 · a4
·
2a4 P a4 ·
P

Figure 5.10

2a2 2a2 2a
A2 · A2 · A¢¢
a -a A¢¢ 3
x
· A3 A¢2 4 2 · A3 2
A¢3
x
a -a
4 2 A¢2
® A¢3 ®
· A4 · A4 A4
A1 x
A1 · A1 · x A¢¢
4

· · A¢¢
P
1 P
P

Figure 5.11

Now we need to prove the problem for n ¼ 4. Let α1  α3. If α4  π2, then
    pffiffi4
PA1  PA2  PA3  PA4 ¼ 16sin 2 α24 sin α1 þ α24 sin α1 þ α2 þ α24 > 16 22 ¼ 4.
If α4 < π2, then the proof consists in moving points A1, A2, A3, A4 along the
circumference in such a way that PA1  PA2  PA3  PA4 decrease. If α4  α2 þ 2α3,
then 3α4 ¼ α4 þ α4 þ α4  α4 þ α1 þ α2 þ 2α3 > π.
Using the lemma we obtain that (Figure 5.10)
PA1  PA2  PA3  PA4 > PA1  PA0 2  PA0 3  PA4 > PA1  PA0 2  PA4 > 2 (PA0 3 > 1
and for n ¼ 3 the statement of the problem holds true).
Now, let α4 < α2 þ 2α3. Choose x, such that 2α4  2x ¼ 2α1  (α4  α2) þ 2x, x
¼ 3α4 α42 2α1 (see Figure 5.11).
5.5 Using Trigonometric Inequalities for Proving Geometric Inequalities 241

Using the lemma twice, we obtain that PA1  PA2  PA3  PA4  (PA1  PA0 2) 
(PA0 3  PA4)  (PA00 1  PA00 2)  (PA00 3  PA00 4).
00 00 00 00 00 00 00 00
Let A1 A2 ¼ A2 A3 ¼ A1 A4 ¼ 2α, as A3 A4  2α, then 2π  8α and 6α < 2π.
00 00 00 00
Therefore, π4  α < π3. We have to prove that PA1  PA2  PA3  PA ¼
16sin 2α2 sin 3α2 sin 2  2, or

2ð cos α  cos 2αÞð cos 2α  cos 3αÞ  1,


2 cos α cos 2α  2cos 2 2α  2 cos α cos 3α þ 2 cos 2α cos 3α  1,
cos α þ cos 3α  1  cos 4α  cos 2α  cos 4α þ cos α þ cos 5α  1,
2 cos α þ cos 3α þ cos 5α  2 þ cos 2α þ 2 cos 4α,
2 cos α þ 2 cos α cos 4α  cos 2αð4 cos 2α þ 1Þ,
4 cos αcos 2 2α  cos 2αð4 cos 2α þ 1Þ:

As π2  2α < 2π3 , then cos2α  0.


We have to prove that 4 cos α cos 2α  4 cos 2α þ 1; this means that 8cos3
α  8cos2α  4 cos α þ 3  0, (2 cos α  1)(4cos2α  2 cos α  3)  0. The last
inequality holds true since cos α > 12 and 4cos2α < 4 cos α < 2 cos α þ 3. The equal-
ity holds when the quadrilateral A1A2A3A4 is a square.
(b) Take the points, so that A1A2A3An is a square and PA4,    , PAn  1 < 1. This
leads to a contradiction, since PAn ¼ 2 sin π8 < 2 sin π6 ¼ 1. Then, we have that
PA1  PA2  . . .  PAn < PA1  PA2  PA3  PAn ¼ 2.
5.5.26. (a) Let points A1, A2, . . . , An and M be represented in the complex plane by
the numbers α1, α2, . . . αn and z, respectively. Then, MA1  MA2, . . . , MAn ¼ |
(z  α1)  ...  (z  αn)| and let f(z) ¼ (z  α1)  . . .  (z  αn) ¼ zn þ cn  1zn  1
2πj
þ . . . þ c1z þ c0, where |c0| ¼ |α1  ...  αn| ¼ 1 and wn ¼ c0. Denote by wj ¼ e n i ,
xj ¼ wwj, j ¼ 0, . . . , n  1, then |xj| ¼ |w|  |wj| ¼ 1 and

jf ð x0 Þj þ jf ðx1 Þj þ ::: þ jf ðxn1 Þj  jf ðx0 Þ þ f ðx1 Þ þ ::: þ f ðxn1 Þj ¼


X n Xn1 Xn1

¼ wn wjn þ cn1 wn1 wn1 j þ ::: þ c 1 w wj þ nc 0
j¼0 j¼0 j¼0

¼ jnwn þ 0 þ ::: þ 0 þ nc0 j ¼ 2njc0 j ¼ 2n:

Thus, exists a number i, such that |f(xi)|  2.

Second Solution
Lemma Let m be the smallest and M the greatest values of the trigonometric
polynomial of the n-th order g(t) ¼ λ0 þ λ1 cos t þ μ1 sin t þ . . . þ λn cos nt þ μn sin nt,
qffiffiffiffiffiffiffiffiffiffiffiffiffiffiffi
then M þ m  2 λ2n þ μ2n .
242 5 Application of Trigonometric Inequalities

qffiffiffiffiffiffiffiffiffiffiffiffiffiffiffi
Indeed, let gðtÞ ¼ f ðtÞ þ λ2n þ μ2n sin ðnt þ φÞ, where f(t) ¼ λ0 þ λ1 cos t þ μ1 sin t
qffiffiffiffiffiffiffiffiffiffiffiffiffiffiffi qffiffiffiffiffiffiffiffiffiffiffiffiffiffiffi
þ . . . þ λn  1 cos(n  1)t þ μn  1 sin(n  1)t, λ2n þ μ2n cos φ ¼ μn , λ2n þ μ2n sin φ
   
¼ λn , tk ¼ π2  φ þ 2πk =n, k ¼ 1, . . . , n and t0k ¼ 3π 2  φ þ 2πk =n. Since gðtk Þ
qffiffiffiffiffiffiffiffiffiffiffiffiffiffiffi     qffiffiffiffiffiffiffiffiffiffiffiffiffiffiffi
¼ f ðtk Þ þ λ2n þ μ2n  M and g t0k ¼ f t0k  λ2n þ μ2n  m, k ¼ 1, . . . , n, then
P
n qffiffiffiffiffiffiffiffiffiffiffiffiffiffiffi P n   qffiffiffiffiffiffiffiffiffiffiffiffiffiffiffi
1
n f ðtk Þ þ λ2n þ μ2n  M and 1n f t0k þ m  λ2n þ μ2n . Thus
k¼1 k¼1

qffiffiffiffiffiffiffiffiffiffiffiffiffiffiffi
1X n
1X n  
f ðtk Þ þ 2 λ2n þ μ2n  M þ m þ f t0k : ð5:29Þ
n k¼1 n k¼1

P
n n  
P
We need to prove that 1n f ðtk Þ ¼ λ0 ¼ 1n f t0k , then from (5.29) it follows
qffiffiffiffiffiffiffiffiffiffiffiffiffiffiffi k¼1 k¼1

that M þ m  2 λ2n þ μ2n .


sin
ðnþ1Þα
sin ðβþnα2 Þ
It is known that sin β þ sin ðβ þ αÞ þ . . . þ sin ðβ þ nαÞ ¼ 2
sin α2 and
sin 2 cos ðβþnα

ðnþ1Þα
cos β þ cos ðβ þ αÞ þ . . . þ cos ðβ þ nαÞ ¼ sin α .
2
Hence,
 0 π
1
 φ m þ 2πm
cos mt1 þ cos mt2 þ . . . þ cos mtn ¼ cos @ 2 Aþ
n
0 π  1 0π  1
 φ m þ 2πm 2πm  φ m þ 2πm 2πm
þ cos @ 2 þ A þ . . . þ cos @ 2 þ ðn  1ÞA ¼
n n n n
0π  1
   φ m þ 2πm 2πm
2πm
sin n sin @ 2 þ ðn  1ÞA
2n n 2n
¼ πm ¼ 0, for m ¼ 1, . . . ,n  1:
sin
n
P
n P
n P
n
Similarly, we need to prove that sin mtk ¼ 0, sin mt0 k ¼ 0, cos mt0 k ¼ 0,
k¼1 k¼1 k¼1
Pn n  0
P
for m ¼ 1, . . . , n  1. Therefore, 1n f ðtk Þ ¼ λ0 ¼ 1n f tk .
k¼1 k¼1
Let Ai(cos2αi, sin2αi), i ¼ 1, . . . , n and P(cos2t, sin2t). Then,
qffiffiffiffiffiffiffiffiffiffiffiffiffiffiffiffiffiffiffiffiffiffiffiffiffiffiffiffiffiffiffiffiffiffiffiffiffiffiffiffiffiffiffiffiffiffiffiffiffiffiffiffiffiffiffiffiffiffiffiffiffiffiffiffiffiffiffiffiffiffiffiffiffiffiffiffiffiffiffi pffiffiffiffiffiffiffiffiffiffiffiffiffiffiffiffiffiffiffiffiffiffiffiffiffiffiffiffiffiffiffiffiffiffiffiffiffiffiffi
PAi ¼ ð cos 2t  sin 2αi Þ2 þ ð sin 2t  cos 2αi Þ2 ¼ 2  2 cos ð2t  2αi Þ

¼ 2 sin ðt  αi Þ :

Thus, PA1  PA2    PAn ¼ 2|2n  1 sin(t  α1) sin(t  α2)    sin(t  αn)|.
5.5 Using Trigonometric Inequalities for Proving Geometric Inequalities 243

We need to prove, by mathematical induction, that 2n  1 sin(t  α1) sin


(t  α2)    sin(t  αn) is a trigonometric polynomial of the n-th order, where
λ2n þ μ2n ¼ 1.

Indeed, for n ¼1 we have that2 sin(t  α1) ¼  sin α1 cos t þ cos α1 sin t.
Thus, λ21 þ μ21 ¼ ð sin α1 Þ2 þ ð cos α1 Þ2 ¼ 1.
Let for n ¼ k, we have that 2k1 sin ðt α1 Þ sin ðt  α2 Þ   sin ðt  αk Þ ¼
1
kP
ðλi cos it þ μi sin itÞ þ ðλk cos kt þ μk sin ktÞ, where λ2k þ μ2k ¼ 1. Therefore,
i¼0

2k sin ðt  α1 Þ sin ðt  α2 Þ   sin ðt  αkþ1 Þ ¼


!
X
k1
¼ ðλi cos it þ μi sin itÞ ð2 cos αkþ1 sin t  2 sin αkþ1 cos tÞþ
i¼0

þðλk cos kt þ μk sin ktÞð2 cos αkþ1 sin t  2 sin αkþ1 cos tÞ ¼
¼ ðλk sin αkþ1  μk cos αkþ1 Þ cos ðk þ 1Þt
þðμk sin αkþ1 þ λk cos αkþ1 Þ sin ðk þ 1Þtþ
X
k
þ ðai cos it þ bi sin itÞ:
i¼0

It remains to prove that λ2kþ1 þ μ2kþ1 ¼ 1; this means that

ðλk sin αkþ1  μk cos αkþ1 Þ2 þ ðμk sin αkþ1 þ λk cos αkþ1 Þ2 ¼
 
¼ λ2k þ μ2k ðsin 2 αkþ1 þ cos 2 αkþ1 Þ ¼ 1:

Let
 
max 2n1 sin ðt  α1 Þ sin ðt  α2 Þ   sin ðt  αn Þ ¼
½0;2π 

¼ 2n1 sin ðt0  α1 Þ sin ðt0  α2 Þ   sin ðt0  αn Þ ¼ M

and
 
min 2n1 sin ðt  α1 Þ sin ðt  α2 Þ   sin ðt  αn Þ ¼
½0;2π 

¼ 2n1 sin ðt1  α1 Þ sin ðt1  α2 Þ   sin ðt1  αn Þ ¼ m:

Consequently, according to the lemma, M  1 or m  1. Thus, |M|  1 or |m|  1.


Hence, one of points P(cos2t0, sin2t0) and P(cos2t1, sin2t1) satisfies the condition
PA1  PA2  . . .  PAn  2.
Remark One can prove that, if points A1, A2, . . . , An (n  3) are not the vertices of
the regular n-gon, then exists a point Pon the circle, so that PA1  PA2  . . . 
PAn > 2.
244 5 Application of Trigonometric Inequalities

(b) The proof by mathematical induction.


For n ¼ 1, the inequality obviously holds true.
Let us consider two cases:
(1) At least one of points Ai coincides with point O. Let An
0, then by mathe-
matical induction there exists a point on the circle (with the center O), so that

PA1  PA2  :::  PAn1  PAn ¼ PA1  PA2  :::  PAn1 


ðOP þ OA1 Þ  :::  ðOP þ OAn1 Þ
PO  PO ¼
2n2
ðOP þ OA1 Þ  :::  ðOP þ OAn1 ÞðOP þ OAn Þ
¼
2n2
ðOP þ OA1 Þ  :::  ðOP þ OAn1 ÞðOP þ OAn Þ
> :
2n1

(2) None of points Ai coincides with point O.


Let Bi be the intersection point of ray OAi with the given circle and let point Ci be
diametrically opposite to point Bi.
We need to prove that OPþOA PAi
i
 OPþOB
PBi
i
, i ¼ 1, 2, . . . n, where P is a point on a
given circle. If P coincides with point Bi or Ci, then the proof is obvious. Let P not
coincide with any of points Bi and Ci. Then we have that OPþOA PAi
i
¼
sin ∠PCi Ai
Ai Ci ¼ sin ∠Ai PCi  sin ∠Pi Ci Ai ¼ Bi Ci ¼ OPþOBi .
PAi PBi PBi

Let us consider a homothety with a center O, such that the image of the given
circle is a unit circle. Let M0 be the image of point M by this homothety. Then,
according to the problem 5.5.26a there exists a point P0 , such that P0 B0 1  . . . 
0 0
P0 B0 i
P0 B0 n  2, as OPþOB PBi
i
¼ OPP0 þOB
Bi
0 ¼
i 2 . Therefore, it follows that OPþOB1  :::
PB1

OPþOBn  2n ¼ 2n1 . Thus, OPþOA1  :::  OPþOAn  OPþOB1  :::  OPþOBn  2n1 or PA1
PBn 2 1 PA1 PAn PB1 PBn 1

:::  PAn  2n1 ðOP þ OA1 Þ  :::  ðOP þ OAn Þ.


1

This ends the proof.


5.5.27. Denote ∠PAB ¼ α1, ∠ PBC ¼ β1, ∠ PCD ¼ γ 1, ∠ PDA ¼ δ1 and ∠PAD ¼
α2, ∠ PBA ¼ β2, ∠ PCB ¼ γ 2, ∠ PDC ¼ δ2, we have to prove that
minðα1 ; β1 ; γ 1 ; δ1 Þ  π4.
Let minðα1 ; β1 ; γ 1 ; δ1 Þ > π4, then according to the law of sines 1 ¼ PB
PA 
sin α1 sin β1 sin γ 1 sin δ1
PB  PC  PD ¼ sin β  sin γ  sin δ2  sin α2 . Therefore,
PC PD PA
2 2

pffiffiffiffiffiffiffiffiffiffiffiffiffiffiffiffiffiffiffiffiffiffiffiffiffiffiffiffiffiffiffiffiffiffiffiffiffiffiffiffiffiffiffiffiffiffiffiffiffiffiffiffiffiffiffiffiffiffiffiffiffiffiffiffiffiffiffiffiffiffiffiffiffiffiffiffiffiffiffiffiffiffiffiffiffiffiffiffiffiffiffiffi
sin α1 sin β1 sin γ 1 sin δ1 ¼ sin α1 sin β1 sin γ 1 sin δ1 sin α2 sin β2 sin γ 2 sin δ2 
α1 þ β1 γ þ δ1 α2 þ β2 γ þ δ2
 sin sin 1 sin sin 2 
2 2 2 2
α1 þ β1 þ γ 1 þ δ1 2 α2 þ β2 þ γ 2 þ δ2
 sin 2 sin 
4 4
α 1 þ β þ γ þ δ1 þ α2 þ β þ γ þ δ 2 π
 sin 4 1 1 2 2
¼ sin 4 ,
8 4

since for x, y, 2 [0; π], we have that xþy


2 2 ½0; π  and 0  sin x sin y  sin 2 .
2 xþy
5.5 Using Trigonometric Inequalities for Proving Geometric Inequalities 245

 4
We find that sin α1 sin β1 sin γ 1 sin δ1  sin π4 and minðα1 ; β1 ; γ 1 ; δ1 Þ > π4.
Thus, maxðα1 ; β1 ; γ 1 ; δ1 Þ > 4 . Let point P be inside triangle OCB, where O is the

intersection point of the diagonals AC and BD, then α1 þ β1 < π, β1 þ γ 1 < π.


Therefore, maxðα1 ; β1 ; γ 1 ; δ1 Þ ¼ δ1 > 3π 4.
Note that maxðα2 ; β2 ; γ 2 ; δ2 Þ  minðα1 ; β1 ; γ 1 ; δ1 Þ > π4. To prove this, one has to
consider a segment with the length max(PA, PB, PC, PD) and use that in a triangle
where the longer side is opposite to the larger angle.
Let E be the intersection point of the straight lines PC and AB. Through points
P and C draw a circle touching the half-line EB at point B0 , then β01 ¼ ∠PB0 C  β1
and ∠PCB0 ¼ γ 02 ¼ ∠PB0 A ¼ β02 .
Since α2 þ δ1 < π, δ1 þ δ2 < π, then maxðβ2 ; γ 2 Þ ¼ maxðα2 ; β2 ; γ 2 ; δ2 Þ > π4.
Hence, δ1 þ α2 þ δ2 < π. Otherwise, we have that 2π > ðα1 þ β1 þ
π
γ 1 Þ þ ðα2 þ δ1 þ δ2 Þ þ maxðβ2 ; γ 2 Þ > 3π 4 þ π þ 4 ¼ 2π. This leads to a contradic-
tion. Thus, the half-lines DC and AP intersect (at point F).
Let us draw through points P and A a circle touching the half-line FC at point D0 ,
then δ01 ¼ ∠PD0 A  ∠PDA ¼ δ1 and α02 ¼ ∠PAD0 ¼ ∠PD0 C ¼ δ02 .
Note that point D0 is not between points C and F. Otherwise, the circumcircle of
triangle APD contains point D0 and hence also point C, but then γ 1 > ∠ACP >
π π
δ1 > 3π 4 . This leads to a contradiction, as ðα1 þ β 1 þ γ 1 þ δ1 > 4 þ 4 þ 4 þ 4 Þ.
3π 3π
0 0 0 0
Since the obtained quadrilateral  0 0 AB0 CD 0
 contains  segments
0 0
PA,
 PB , PC PD ,
then one can  prove that max α2 ; β2 ; γ 2 ; δ2 min απ 1 ; βπ1 ; γ 1π;δ1 . πThen, we have
that 2π ¼ α1 þ β01 þ γ 1 þ δ01 þ 2α02 þ 2β02 > 3π 4 þ 4 þ 4 þ 4 þ 24 ¼ 2π, but this
is not possible. Thus, ðα1 ; β1 ; γ 1 ; δ1 Þ  π4.
Second Solution Let AB ¼ a, BC ¼ b, CD ¼ c, DA ¼ d, PA ¼ x, PB ¼ y, PC ¼ z,
and PD ¼ t.
Then, we have that

1 1 1 1
SABCD ¼ ax sin α1 þ by sin β1 þ cz sin γ 1 þ dt sin δ1 ð5:30Þ
2 2 2 2

According to the law of cosines, we obtain that y2 ¼ a2 þ x2  2ax cos α1, z2 ¼ y2


þ b2  2yb cos β1, t2 ¼ c2 þ z2  2cz cos γ 1, and x2 ¼ d2 þ t2  2dt cos δ1.
Therefore,

a2 þ b2 þ c2 þ d2 ¼ 2ax cos α1 þ 2by cos β1 þ 2cz cos γ 1 þ 2dt cos δ1 ð5:31Þ

a þb 2
c þd
2 2 2
Note that SABCD ¼ SABC þ SACD  ab 2 þ 2  4 þ 4 . From the last inequal-
cd

ities, using (5.30) and (5.31), we deduce that ax sin α1 þ by sin β1 þ


cz sin γ 1 þ dt sin δ1  ax cos α1 þ by cos β1 þ cz cos γ 1 þ dt cos δ1.
Hence, there exists such α, that α 2 {α1, β1, γ 1, δ1} and sinα  cos α, then α  π4
(see also the problem 7.1.71c).
246 5 Application of Trigonometric Inequalities

Figure 5.12 A

E
D
M P K
N

B C

5.5.28. (a) At first, note that it is sufficient to prove the problem for BD ¼ CD.
Indeed, if BD > CD, then consider on segment BD a point B0 , such that
B D ¼ CD. Let line B0 P intersect segment EC at point E0 . Since B0 D ¼ CD, then
0

for triangle B0 AC, we have that

AD þ DP > AE0 þ E0 P ¼¼ AE þ EE0 þ E0 P > AE þ EP:

Thus, AD þ DP > AE þ EP.


Now, we need to prove the problem for BD ¼ CD. Let ∠DBP ¼ α, ∠PBC ¼ β,
∠DCA ¼ γ. Let us draw through point P a segment MK, parallel to line BC
(Figure 5.12).
Let N be a point on ray EC, such that EN ¼ EP. We have that
∠EPN ¼ ∠ENP ¼ ∠EBCþ∠ECB 2 ¼αþγ
2 þ β > β ¼ ∠EPK. Thus, point N is on ray KC.
Let ∠PMN ¼ x, ∠ PNM ¼ y. We have that ∠AMN þ ∠ MNA ¼ ∠ ABC þ
∠ BCA, consequently, x þ y ¼ αþγ 2 . Note that, AM ¼ AD þ DM ¼ AD þ DP and
AN ¼ AE þ EN ¼ AE þ EP and we have to prove that AM > AN; this means that αþγ 2
þβ þ y ¼ ∠MNA > ∠MAN ¼ α þ β þ x or y > α2.
We proceed the proof by contradiction argument. Let y  α2.
Let ∠BAP ¼ u, ∠CAP ¼ v, then according to the Ceva’s theorem, we have that
for triangles ABC  and AMN: sinβ sin u sin γ ¼ sin α sin v sin(α þ β) and
sin x sin u sin αþγ 2 þ β ¼ sin y sin ðα þ βÞ sin v. Therefore,
sin β sin γ
sin y ¼ sin ðαþγþβÞ sin α.
sin x
2
sin ðαþγ
2 yÞ αþγ αþγ α π sin β sin γ
Since sin y ¼ sin y ¼ sin 2 ctgy  cos 2 and 0 < y  2 < 2. sin αþγþβ sin α ¼
sin x
ð2 Þ
γ
sin 2 ctgy  cos 2  sin 2 ctg 2  cos 2 ¼ sin sin α. Thus, 2 sin β cos 2γ  2
αþγ αþγ αþγ α αþγ 2

      2
sin αþγ α γ
2 þ β cos 2 or sin β  2  sin β þ 2 þ α .
γ

This leads to a contradiction, as π2 < β  2γ < β þ α þ 2γ ¼ ∠ABCþ∠BCA


2 < π2.
(b) Let A0 be a point on side AC, such that AA0 ¼ AD and let D0 be the intersection
point of segments CD and BA0 , then BD0 ¼ D0 C. Hence, according to the
problem 5.5.28a, we obtain for triangle A0 BC that A0 D0 þ D0 P > A0 E þ EP.
Thus, DD0 þ D0 P > A0 E þ EP or AD þ DP > AE þ EP, since AD ¼ AA0 and
DD0 ¼ A0 D0 .
5.5 Using Trigonometric Inequalities for Proving Geometric Inequalities 247

5.5.29. (a) Let ∠AOB ¼ α, then according to the law of cosines, we have that
qffiffiffiffiffiffiffiffiffiffiffiffiffiffiffiffiffiffiffiffiffiffiffiffiffiffiffiffiffiffiffiffiffiffiffiffiffiffiffiffiffiffiffiffiffiffiffiffiffiffiffiffiffiffiffiffiffiffiffiffiffiffiffiffiffiffiffiffiffiffiffiffi
AB ¼ ðAO þ BOÞ2  2AO  BOð1 þ cos αÞ 
rffiffiffiffiffiffiffiffiffiffiffiffiffiffiffiffiffiffiffiffiffiffiffiffiffiffiffiffiffiffiffiffiffiffiffiffiffiffiffiffiffiffiffiffiffiffiffiffiffiffiffiffiffiffiffiffiffiffiffiffiffiffiffiffiffiffiffiffiffiffiffiffiffiffiffiffiffiffiffiffi
1 α
 ðAO þ BOÞ2  ðAO þ BOÞ2 ð1 þ cos αÞ ¼ ðAO þ BOÞ sin :
2 2

Similarly, we obtain that CD  ðCO þ ODÞ sin α2.


Since,
α α
2r 1 þ 2r 3 ¼ ðAO þ BO  ABÞtg þ ðCO þ DO  CDÞtg ¼
2 2
α  α α
¼ ðAC þ BD  ðAB þ CDÞÞtg  ðAC þ BDÞ  1  sin tg
2 2 2
 α
 α
and similarly we deduce  that 2rα2 þα2r 4  ðAC þαBD
 Þ α1  cos 2 ctg 2. Then,
2r 1 þ 2r 2 þ 2r 3 þ 2r 4  1  sin 2 tg 2 þ 1  cos 2 ctg 2 ðAC þ BDÞ, where
r1, r2, r3, r4 are the radiuses of the circles S1, S2, S3, S4, respectively.  
In order to end the p ffiffi
ffi proof, it remains to prove that 1  sin α2 tg α2 þ
 
1  cos α2 ctg α2  2  2.    
Let t ¼ sin α2 þ cos α2, then t > 0 and 1  sin α2 tg α2 þ 1  cos α2 ctg α2 ¼
t3 3tþ2 t2 þt2
pffiffiffi  pffiffiffi pffiffiffi 
t2 1 ¼ tþ1  2  2, i.e., t  2 t þ 2 2  1  0. The last inequality
p ffiffiffi
holds true, as t 1 α α
2
2 ¼ sin 2 cos 2 ¼ 2 sin α  2, thus t 
1 1
2.
(b) According to the problem 1.1.4a, we have that

2r 1 þ 2r 3 2r 2 þ 2r 4
O1 O2 þ O2 O3 þ O3 O4 þ O4 O1 < 2ðO1 O3 þ O2 O4 Þ ¼ α þ α 
sin cos
0 1 2 2
α α
1  sin 1  cos 2
B 2 2C
 ðAC þ BDÞ@ α þ α A ¼ t þ 1ðAC þ BDÞ < AC þ BD,
cos sin
2 2

as sin α2 cos α2 ¼ t 1
2
2 > 0, this means that t > 1. This ends the proof.
5.5.30. Let us denote AC ¼ b, CB ¼ a, ∠DAC ¼ α, ∠ DBC ¼ β, then ∠ACB ¼ α þ β
pffiffiffiffiffiffiffiffiffiffiffiffiffiffiffiffiffiffiffiffiffiffiffiffiffiffiffiffiffiffiffi
a2 þb2 2ab cos ðαþβÞ
and R ¼ 2 sin ðαþβÞ .
Using the law of sines for triangles ΔADC and ΔBDC, we obtain that
CD2 ¼ absinsin2 ðα sin β
αþβÞ .
We have that

a2 þ b2  2ab cos ðα þ βÞ  2abð1  cos ðα þ βÞÞ ¼


¼ 2abð2 sin α  sin β þ 1  cos ðα  βÞÞ  2ab  2 sin α  sin β ¼ 4ab sin α  sin β:

Thus, a2 þ b2  2ab cos(α þ β)  4ab sin α  sin β or R2  CD2. Hence, we


deduce that R  CD. This ends the proof.
248 5 Application of Trigonometric Inequalities

5.5.31. Let us denote by φi ¼ 12 ∠Ai MAiþ1 , i ¼ 1, . . . , n. We have that, d2i ¼


MAi  MAiþ1 cos ∠Ai MPi  cos ∠Aiþ1 MPi , where Pi 2 AiAi þ 1 and MPi ⊥ AiAi þ 1.
Hence,
1 þ cos ð∠Ai MPi þ ∠Aiþ1 MPi Þ
d 2i ¼ Ri Riþ1 cos ∠Ai MPi  cos ∠Aiþ1 MPi  Ri Riþ1  ¼
2
∠Ai MPi þ ∠Aiþ1 MPi
¼ Ri Riþ1 cos 2  Ri Riþ1 cos 2 φi ,
2

as φi  ∠Ai MPi þ∠A


2
iþ1 MPi
< π2. Consequently
pffiffiffiffiffiffiffiffiffiffi pffiffiffiffiffiffiffiffiffiffi
d1 þ d2 þ . . . þ dn  R1 R2 cos φ1 þ R2 R3 cos φ2 þ . . .
pffiffiffiffiffiffiffiffiffiffi
þ Rn R1 cos φn : ð5:32Þ

It is clear that φ1 þ φ2 þ . . . þ φn ¼ π and if we need to prove that


pffiffiffiffiffiffiffiffiffiffi pffiffiffiffiffiffiffiffiffiffi pffiffiffiffiffiffiffiffiffiffi
R1 R2 cos φ1 þ R2 R3 cos φ2 þ . . . þ Rn R1 cos φn
π
 cos ðR1 þ R2 þ . . . þ Rn Þ, ð5:33Þ
n

then from (5.32) and (5.33) we obtain that

1
R1 þ R 2 þ . . . þ R n 
cos πnðd1 þ d2 þ . . . þ dn Þ:

2(a2, b2), . . . ,p
Consider on the coordinate plane points B1(a1, 0), pBffiffiffiffiffi Bnffiffiffiffiffi
(an, bn),
Bn þ 1(a , 0)
p1ffiffiffiffiffi (see Figure 5.13), such that OB1 ¼ R 1 , OB 2 ¼ R 2, . . . ,
OBn ¼ Rn , ∠B1OB2 ¼ φ1, ∠B2OB3 ¼ φ2, . . . , ∠ BnOBn þ 1 ¼ φn.
pffiffiffiffiffiffiffiffiffiffiffiffiffi OB2 þOB2iþ1 Bi B2iþ1
According to the law of cosines Ri Riþ1 cos φi ¼ i 2 .

Figure 5.13 y

B3

B2
Bn
j2
jn j1
Bn+1 O B1 x
5.5 Using Trigonometric Inequalities for Proving Geometric Inequalities 249

 
Therefore, the inequality (5.33) can be rewritten, as 2 OB21 þ OB22 þ . . . þ OB2n
B1 B22  B2 B23  . . .  Bn B2nþ1  2 cos πn OB21 þ . . . þ OB2n , or

a1 a2 þ a2 a3 þ . . . þ an1 an  an a1 þ b2 b3 þ b3 b4 þ . . . þ bn1 bn 
π  ð5:34Þ
 cos a21 þ . . . þ a2n þ b22 þ . . . þ b2n :
n
If we prove that for any numbers
π 
x1 x2 þ x2 x3 þ . . . þ xn1 xn  xn x1  cos x21 þ x22 þ . . . þ x2n , ð5:35Þ
n

then we can use the inequality (5.35) for x1 ¼ a1, . . . , xn ¼ an and x1 ¼ 0,


x2 ¼ b2, . . . , xn ¼ bn, in order to prove the inequality (5.34).
Thus, we have to prove that for n  3 and any numbers x1, x2x3, . . . , xn it holds true
π 2 
A ¼ 2 cos x þ x22 þ . . . þ x2n  2x1 x2  2x2 x3  , . . . ,  2xn1 xn þ 2xn x1  0:
n 1
ð5:36Þ
One can easily verify that
 
1 2π π π 2
A¼ x sin  x2 sin þ xn sin þ
π 2π 1 n n n
sin sin
n n 
1 3π 2π π 2
þ x sin  x3 sin þ xn sin þ ...
2π 3π 2 n n n
sin sin
n n  
1 ðn  1Þπ ðn  2Þπ π 2
þ xn2 sin  xn1 sin þ xn sin :
ðn  2Þπ ðn  1Þπ n n n
sin sin
n n
ð5:37Þ

Therefore, the inequality (5.36) holds true. To prove (5.37), it remains to note that
0 1
πB 1 1 1 C
sin 2 B þ þ .. . þ C¼
n@ π 2π 2π 3π ðn  2Þπ ðn  1Þπ A
sin sin sin sin sin sin
n n n n n n
0      1
2π π 3π 2π ðn  1Þπ ðn  2Þπ
sin  sin  sin 
πB n n n n n n C
¼ sin B þ þ ... þ C¼
n@ π 2π 2π 3π ðn  2Þπ ðn  1Þπ A
sin sin sin sin sin sin
n n n n n n
 
π π 2π 2π 3π ðn  2Þπ ðn  1Þπ
¼ sin ctg  ctg þ ctg  ctg þ .. . þ ctg  ctg ¼
n n n n n n n
 
π π ðn  1Þπ π
¼ sin ctg  ctg ¼ 2 cos :
n n n n
This ends the proof.
250 5 Application of Trigonometric Inequalities

5.5.32. We have that (a cos α þ b cos β)2  (a cos α þ b cos β)2 þ (a sin α  b sin β)2
¼ a2 þ b2 þ 2ab cos(α þ β) (see the solution of the problem 5.5.31), thus
pffiffiffiffiffiffiffiffiffiffi pffiffiffiffiffiffiffiffiffiffi pffiffiffiffiffiffiffiffiffiffiffiffiffiffiffiffiffiffi pffiffiffiffiffiffiffiffiffiffiffiffi
d 1 þ d 2 þ ::: þ d 2k  R1 R2 cos φ1 þ R2 R3 cos φ2 þ ::: þ R2k1 R2k cos φ2k1 þ R2k R1 cos φ2k 
pffiffiffiffiffipffiffiffiffiffi pffiffiffiffiffi  pffiffiffiffiffiffiffipffiffiffiffiffiffiffiffiffiffiffi pffiffiffiffiffi 
 R2 R1 cos φ1 þ R3 cos φ2 þ ::: þ R2k R2k1 cos φ2k1 þ R1 cos φ2k 
rffiffiffiffiffiffiffiffiffiffiffiffiffiffiffiffiffiffiffiffiffiffiffiffiffiffiffiffiffiffiffiffiffiffiffiffiffiffiffiffiffiffiffiffiffiffiffiffiffiffiffiffiffiffiffiffiffiffiffiffiffiffiffiffiffiffiffiffiffiffiffiffiffiffiffiffiffiffiffiffiffiffiffiffiffiffiffiffiffiffiffiffiffiffiffiffiffiffiffiffiffiffiffiffiffiffiffiffiffiffiffiffiffiffiffiffiffiffiffiffiffiffiffiffiffiffiffiffiffiffiffiffiffiffiffiffiffiffiffiffiffiffiffiffiffiffiffiffiffiffiffiffiffiffiffiffiffiffiffiffiffiffiffiffiffiffiffiffiffiffiffiffiffiffiffiffiffiffi
pffiffiffiffiffi ffi
pffiffiffiffiffi 2 pffiffiffiffiffiffiffiffiffiffiffi pffiffiffiffiffi 2 
 ðR2 þ R4 þ ::: þ R2k Þ R1 cos φ1 þ R3 cos φ2 þ ::: þ R2k1 cos φ2k1 þ R1 cos φ2k 
qffiffiffiffiffiffiffiffiffiffiffiffiffiffiffiffiffiffiffiffiffiffiffiffiffiffiffiffiffiffiffiffiffiffiffiffiffiffiffiffiffiffiffiffiffiffiffiffiffiffiffiffiffiffiffiffiffiffiffiffiffiffiffiffiffiffiffiffiffiffiffiffiffiffiffiffiffiffiffiffiffiffiffiffiffiffiffiffiffiffiffiffiffiffiffiffiffiffiffiffiffiffiffiffiffiffiffiffiffiffiffiffiffiffiffiffiffiffiffiffiffiffiffiffiffiffiffiffiffiffiffiffiffiffiffiffiffiffiffiffiffiffiffiffiffiffiffiffiffiffiffiffiffiffiffiffiffiffiffiffiffiffiffiffiffiffiffiffiffiffiffiffiffiffiffiffiffiffiffiffiffiffiffiffiffiffiffiffiffiffiffiffiffiffiffiffiffiffiffi
 pffiffiffiffiffiffiffiffiffiffi pffiffiffiffiffiffiffiffiffiffiffiffiffiffiffiffi ffi
 ðR2 þ R4 þ ::: þ R2k Þ R1 þ R3 þ 2 R1 R3 cos ðφ1 þ φ2 Þ þ ::: þ R2k1 þ R1 þ 2 R2k1 R1 cos ðφ2k1 þ φ2k Þ 
rffiffiffiffiffiffiffiffiffiffiffiffiffiffiffiffiffiffiffiffiffiffiffiffiffiffiffiffiffiffiffiffiffiffiffiffiffiffiffiffiffiffiffiffiffiffiffiffiffiffiffiffiffiffiffiffiffiffiffiffiffiffiffiffiffiffiffiffiffiffiffiffiffiffiffiffiffiffiffiffiffiffiffiffiffiffiffiffiffiffiffiffiffiffiffiffiffiffiffiffiffiffiffiffiffiffiffiffiffiffiffiffiffiffiffiffiffiffiffiffiffiffiffiffiffiffiffiffiffiffiffiffiffiffiffiffiffiffiffiffiffiffiffiffiffiffiffiffiffiffi
 ffi
π
 ðR2 þ R4 þ ::: þ R2k Þ 2ðR1 þ R3 þ ::: þ R2k1 Þ þ 2ðR1 þ R3 þ ::: þ R2k1 Þ cos 
k
π pffiffiffiffiffiffiffiffiffiffiffiffiffiffiffiffiffiffiffiffiffiffiffiffiffiffiffiffiffiffiffiffiffiffiffiffiffiffiffiffiffiffiffiffiffiffiffiffiffiffiffiffiffiffiffiffiffiffiffiffiffiffiffiffiffiffiffiffiffiffiffiffiffiffiffiffiffiffiffiffiffiffi
 2cos ðR2 þ R4 þ ::: þ R2k ÞðR1 þ R3 þ ::: þ R2k1 Þ:
2k

This ends the proof.


! ! ~ ! ~ ~2
5.5.33. 
 nþ1 ¼ d 1 ,A2 Anþ2 ¼ d 2 , :::, An A2n
(a) Let A1 A  ¼ d n , di ¼ Ri , i ¼ 1, . . . , n,
~d d d
d1 , ~
d 2 ¼ φ1 , :::, ~
d n1 , ~
dn ¼ φn1 and ~
d n , ~
d1 ¼ φn , note that
φ1 þ φ2 þ . . . þ φn ¼ π.
! ! ! !
We have that ~ d1 þ Anþ1 Anþ2  d 2 þ A2 A1 ¼ ~ 0. Thus, it follows that ~ d1  d2

! ! ! 2
¼ A1 A2  Anþ1 Anþ2  A1 A2 þ Anþ1 Anþ2 ; therefore, ~ d1  d 2  ðA1 A2 þ Anþ1
pffiffiffiffiffiffiffiffiffiffi
Anþ2 Þ2 or R1  2 R1 R2 cos φ1 þ R2  ðA1 A2 þ Anþ1 Anþ2 Þ2 .
Similarly, we obtain that
pffiffiffiffiffiffiffiffiffiffi
R2 R3ffi cos φ2 þ R3  ðA2 A3 þ Anþ2 Anþ3 Þ2 , and so on recurrently,
R2 p2ffiffiffiffiffiffiffiffiffiffiffiffiffiffi
2
Rn1  2 Rn1 Rn cos φn1 þ Rn  ðAn1 An þ A2n1 A2n Þ .
! ! ! ! ! !
As ~ d 1 þ Anþ1 An þ d n þ A2n A1 ¼ ~ 0, then ~d1 þ dn ¼ A1 A2n  Anþ1 An  An
pffiffiffiffiffiffiffiffiffiffi
Anþ1 þ A2n A1 ; therefore. R1 þ 2 R1 Rn cos ðπ  φn Þ þ Rn  ðAn Anþ1 þ A2n A1 Þ2 .
Summing up these inequalities, we deduce that
pffiffiffiffiffiffiffiffiffiffi pffiffiffiffiffiffiffiffiffiffi pffiffiffiffiffiffiffiffiffiffi 
2ðR1 þ R2 þ ::: þ Rn Þ  2 R1 R2 cos φ1 þ R2 R3 cos φ2 þ ::: þ Rn R1 cos φn 
 ðA1 A2 þ Anþ1 Anþ2 Þ2 þ ðA2 A3 þ Anþ2 Anþ3 Þ2 þ ðAn Anþ1 þ A2n A1 Þ2 :
ð5:38Þ
We have
pffiffiffiffiffiffiffiffiffiffi pffiffiffiffiffiffiffiffiffiffi pffiffiffiffiffiffiffiffiffiffi
R1 R2 cos φ1 þ R2 R3 cos φ2 þ ::: þ Rn R1 cos φn
π
 cos ðR1 þ R2 þ ::: þ Rn Þ ð5:39Þ
n

(see the proof of problem 5.5.31); thus from (5.38) and (5.39) it follows that
 π
2 1  cos ðR1 þ R2 þ ::: þ Rn Þ 
n
 ðA1 A2 þ Anþ1 Anþ2 Þ2 þ ðA2 A3 þ Anþ2 Anþ3 Þ2 þ ::: þ ðAn Anþ1 þ A2n A1 Þ2
5.5 Using Trigonometric Inequalities for Proving Geometric Inequalities 251

or
π 2
4sin 2 A1 A2nþ1 þ A2 A2nþ2 þ ::: þ An A22n  ðA1 A2 þ Anþ1 Anþ2 Þ2 þ
2n
þðA2 A3 þ Anþ2 Anþ3 Þ2 þ ::: þ ðAn Anþ1 þ A2n A1 Þ2 :

Remark This estimate is an optimal one, as for a regular 2n-gon the equality
holds true.
! ! pffiffiffiffiffiffiffiffiffiffi
(a) We have that 2B1 Bnþ1 ¼ ~ d1 þ d2 , therefore 4B1 B2nþ1 ¼ R1 þ 2 R1 R2 cos φ1 þ
R2 .
pffiffiffiffiffiffiffiffiffiffi
Similarly, we obtain that 4B2 B2nþ2 ¼ R2 þ 2 R2 R3 cos φ2 þ R3 , and so on
p ffiffiffiffiffiffiffiffiffiffiffiffiffiffi

recurrently, 4Bn1 B22n1 ¼ Rn1 þ 2 Rn1 Rn cos φn1 þ Rn .
! ! pffiffiffiffiffiffiffiffiffiffi
We have that 2Bn B2n ¼ ~ d n  d 1 , hence 4Bn B22n ¼ Rn þ 2 Rn R1 cos φn þ R1 .
Summing up these n inequalities, we obtain that
 
nþ1 þ B2 Bnþ2 þ ::: þ n B2n ffi ¼ 2ðR1 þ Rp
2þ ::: þ Rn Þþ
2 2 2
4 B1p
Bffiffiffiffiffiffiffiffiffiffi pBffiffiffiffiffiffiffiffiffiffiffiffiffiffi ffiffiffiffiffiffiffiffiffiffi
þ2 R1 R2 cos φ1 þ ::: þ Rn1 Rn cos φn1 þ Rn R1 cos φn :

From the proof of problem 5.5.33а, it follows that


   π
4 B1 B2nþ1 þ B2 B2nþ2 þ ::: þ Bn B22n  2 1 þ cos ðR1 þ R2 þ ::: þ Rn Þ 
n
π 
 ctg 2 ðA1 A2 þ Anþ1 Anþ2 Þ2 þ ðA2 A3 þ Anþ2 Anþ3 Þ2 þ ::: þ ðAn Anþ1 þ A2n A1 Þ2 :
2n

We deduce that
π 
4tg 2 B1 B2nþ1 þ B2 B2nþ2 þ ::: þ Bn B22n 
2n
 ðA1 A2 þ Anþ1 Anþ2 Þ2 þ ðA2 A3 þ Anþ2 Anþ3 Þ2 þ ::: þ ðAn Anþ1 þ A2n A1 Þ2 :

5.5.34. Let any of the given circles touch three sides of a convex quadrilateral
ABCD. Let us find out for which quadrilateral ABCD its perimeter is the smallest.
Consider another two circles, such that the first circle touches lines BC, AD, AB
and the second circle touches lines BC, AD, CD (Figure 5.14).

Figure 5.14
252 5 Application of Trigonometric Inequalities

Let O1O2D0 ¼ 2α.


One can prove that MA0 ¼ AB, ND0 ¼ CD and p ¼ 2AB þ 2CD þ 2A0D0.
Therefore, p ¼ 2MN. Note that p is the smallest, if MN is the smallest.
As MN ¼ O3O4 sin 2α, then MN is the smallest, if O3O4 is the smallest.
Note that O3O4 is the smallest, if the circles with centers O3 and O4 touch the
circles with centers O1 and O2, respectively.
Therefore, the perimeter of quadrilateral ABCD is the smallest, if AB ⊥ O1O2
and CD ⊥ O1O2.
Hence, we obtain that it is sufficient to prove the following inequality: p  2d
2
þ4r þ 4R þ 3ðRr d
Þ
, for quadrilateral ABCD, such that sides AB and CD are
perpendicular to line O1O2.
Þ2
Note that, if R ¼ r, then p ¼ 2d þ 8R ¼ 2d þ 4r þ 4R þ 3ðRr
d :
If R > r, then AB ¼ 2rtgα, CD ¼ 2Rctgα, A0 D0 ¼ ðR  r Þtg 2α, d ¼ cos
Rr
2α :
Thus, we need to prove that

Rr 3
2rtgα þ 2Rctgα þ ðR  r Þtg 2α  þ 2r þ 2R þ ðR  r Þ cos 2α, ð5:40Þ
cos 2α 2

where 0 < α < π4 :


The inequality (5.40) can be rewritten as
 
2R 2r ð cos α  sin αÞ2
 ð cos α  sin αÞ  ðR  r Þ þ
sin α cos α cos 2α ð5:41Þ
3  
þ ðR  r Þ cos 2 α  sin 2 α :
2

Note that
cosα  sin α > 0 and sin 2R
α  cos α > sin α :
2r 2R2r

Therefore, in order to prove (5.41) it is sufficient to prove that


sin α > cos αþ sin α þ 2 ð cos α þ sin αÞ, or cos αð3cos  1Þ > 2 sin α:
2 1 3 2 1

The last inequality holds true, as cosα > sin α and ð3cos α  1Þ > 12 sin α:
2

Remark One can prove that


pffiffiffi
(a) p  2 6jR  r j þ 4R þ r:
2
(b) If d ¼ R þ r, then p  6R þ 6r þ 3ðRr Þ
Rþr :

5.5.35. Consider Figure 5.14.


Let us prove that the value of pS2 is the greatest possible one, if AB ⊥ O1O2 and
CD ⊥ O1O2. Indeed, we have that

S AB  r þ CD  R þ A0 D0 ðR þ r Þ ðAB þ CD þ A0 D0 ÞðR þ r Þ  AB  R  CD  r
¼ ¼ ¼
p2 p2 p2
R þ r MA0  R þ ND0  r R þ r 2rtgα  R þ 2Rctgα  r R þ r 4Rr
¼  2
  2
¼  2 :
2p p 2p p 2p p sin 2α
ð5:42Þ
5.5 Using Trigonometric Inequalities for Proving Geometric Inequalities 253

Let quadrilateral A1B1C1D1 be such that any of the given two circles touches its
three sides and A1B1 ⊥ O1O2 and C1D1 ⊥ O1O2.
Let the area of quadrilateral A1B1C1D1 be equal to S1 and its perimeter be equal
to p1.
We have that

S1 Rþr 4Rr
¼  2 : ð5:43Þ
p1 2 2p1 p1 sin 2α

From (5.42) and (5.43), it follows that it is sufficient to prove the following
inequality,

Rþr 4Rr Rþr 4Rr


 2   2 : ð5:44Þ
2p p sin 2α 2p1 p1 sin 2α

According to the proof of problem 5.5.34, we have that p  p1. Therefore, in


order to prove (5.44), it is sufficient to prove that,
 
4Rr 1 1 Rþr
þ  : ð5:45Þ
sin 2α p1 p 2

As p  p1, then in order to prove (5.45), it is sufficient to prove that


4Rr
 p2  Rþr
sin 2α 2 , or p1 ðR þ r Þ  sin 2α.
16Rr
1
If R ¼ r, then p1 ¼ 8R þ 2d. Therefore, it follows that p18R.
If R > r, then we have that 0<α<π/4, and one needs to prove that,

ð2rtgα þ 2Rctgα þ ðR  r Þtg 2αÞ sin 2αðR þ r Þ  8Rr,


R2  r 2
ð2rsin 2 þ 2Rcos 2 αÞðR þ r Þ þ tg 2α sin 2α  4Rr,
2
R r
2 2  
2ðR þ r Þ2 cos 2 α þ tg 2α sin 2α  2 R2  r 2 sin 2 α  4Rr:
2

The last inequality holds true as 2(R þ r)2cos2α > (R þ r)2  4Rr and
tg2α sin 2α  4sin2α.
5.5.36. According to the proof of problem 5.5.35 (see 5.42), we have that

S Rþr 1 4Rr 1 ðR þ r Þ2
     sin 2α:
p2 2 p sin 2α p2 64Rr

Let the distance between the centers of those circles be equal to d.


If 0 < d  R þ r, then we have that

Rr Rr
cos 2α ¼  :
d Rþr
254 5 Application of Trigonometric Inequalities

Thus, it follows that


pffiffiffiffiffi
2 Rr
sin 2α  :
Rþr

Hence, we deduce that S  32Rþrpffiffiffiffi p2 .


Rr
If d > R þ r, then according to problem 7.1.109b, we have that

p2 p2 Rþr
S <  pffiffiffiffiffi p2 :
18 16 32 Rr

Therefore, we obtain that

Rþr 2
S< pffiffiffiffiffi p :
32 Rr

This ends the proof.


5.5.37. According to the proof of problem 5.5.34, it is sufficient to prove the given
inequality for quadrilateral ABCD, such that AB ⊥ O1O2 and CD ⊥ O1O2.
According to problem 7.1.109а, we have that pS2  18
1
. Moreover, according to the
proof of problem 5.5.35, we obtain that p2 ¼ 2p  p2 sin 2α  18
S Rþr 4Rr 1
.
On the other hand, we have that cos 2α ¼ d  Rþr.
Rr Rr
pffiffiffiffi
We deduce that sin 2α  2RþrRr
.
Therefore, we obtain that
pffiffiffiffiffi
R þ r 2 Rr ðR þ r Þ 1
  :
2p p2 18

Thus, it follows that


qffiffiffiffiffiffiffiffiffiffiffiffiffiffiffiffiffiffiffiffiffiffiffiffiffiffiffiffiffiffiffiffiffiffiffiffiffiffiffiffiffiffiffiffiffiffiffiffiffiffiffiffiffiffiffiffiffiffi
pffiffiffiffiffi
9ð R þ r Þ þ 81ðR þ r Þ2  144ðR þ r Þ Rr
p ,
2

(see problem 7.1.109a).


5.5.38. (а) According to the proof of problem 1.2.7, we have that

A1 B1 þ B1 C1 þ A1 C1  a cos α þ b cos β þ c cos γ


¼ Rð sin 2α þ sin 2β þ sin 2γ Þ: ð5:46Þ

As α, β, γ > π4 , then α, β, γ < π2. According to problem 4.1.2.1f, we obtain that


5.5 Using Trigonometric Inequalities for Proving Geometric Inequalities 255

sin 2α þ sin 2β þ sin 2γ ¼ sin ðπ  2αÞ þ sin ðπ  2βÞ þ sin ðπ  2γ Þ >


2 2 2
> ðπ  2αÞ þ ðπ  2βÞ þ ðπ  2γ Þ ¼ 2:
π π π
ð5:47Þ

Hence, from (5.46) and (5.47), we deduce that A1B1 þ B1C1 þ A1C1 > 2R  X(A1, B1).
Therefore, A1B1 þ B1C1 þ A1C1 > X(A1, B1).
(b) Note that PAC1 B1 þ PBA1 C1 þ PCA1 B1 ¼ PABC þ PA1 B1 C1 , and PABC > 4R, (see
problem 5.2.3а). According to the proof of problem 5.5.39а, we have that
PA1 B1 C1 > 2R. Therefore, PAC1 B1 þ PBA1 C1 þ PCA1 B1 > 6R, hence
maxðPAC1 B1 ; PBA1 C1 ; PCA1 B1 Þ > 2R > XðA1 ; B1 Þ.
Thus, it follows that maxðPAC1 B1 ; PBA1 C1 ; PCA1 B1 Þ > XðA1 ; B1 Þ:
This ends the proof.
5.5.39. At first, let us prove the following lemma.
ρðX;QRÞ
Lemma If inside of angle PQR is chosen a point X, then ρðX;QPÞ ¼ sin ∠
PQR  ctg ∠PQX  cos ∠PQR:
ρðX;QRÞ QX sin ∠RQX sin ð∠PQR∠PQXÞ
Proof of the lemma Indeed, we have that ρðX;QPÞ ¼ QX sin ∠PQX ¼ sin ∠PQX
ρðX;QRÞ
¼ sin ∠PQR  ctg ∠PQX  cos ∠PQR, therefore ρðX;QPÞ ¼ sin ∠PQR  ctg ∠PQX
 cos ∠PQR:
This ends the proof of the lemma.
According to the lemma, we have that

dc d0
¼ sin ∠Actg∠MAC  cos ∠A ¼ sin ∠Actg∠M0 AB  cos ∠A ¼ b0 :
db dc

Thus, it follows that d c d0c ¼ d b d0b : Similarly, we deduce that dc d 0c ¼ da d0a ,


therefore d a d 0a ¼ d b d0b :
d0
Hence ddab ¼ db0 , and according to the lemma, we obtain that sin ∠ Cctg ∠
a
MCA  cos ∠ C ¼ sin ∠ Cctg ∠ M0 CB  cos ∠ C.
Therefore, ∠MCA ¼ ∠ M0 CB.
Without loss of generality one can assume that ∠C  ∠ B  ∠ A.
Let us prove that dc d0c  r 2 , where r is the inradius of triangle ABC.
Let ∠A ¼ α, ∠ B ¼ β, ∠ MAB ¼ φ, ∠ MBA ¼ ψ.
Note that

AB AB AB
dc ¼ , d0c ¼ ,r ¼
ctgφ þ ctgψ ctg ðα  φÞ þ ctg ðβ  ψ Þ ctg 2 þ ctg β2,
α

then by Cauchy-Bunyakovsky inequality, it follows that


256 5 Application of Trigonometric Inequalities

AB2
d c d0c ¼
ðctgφ þ ctgψ Þðctg ðα  φÞ þ ctg ðβ  ψ ÞÞ
AB2
 pffiffiffiffiffiffiffiffiffiffiffiffiffiffiffiffiffiffiffiffiffiffiffiffiffiffiffiffiffiffi pffiffiffiffiffiffiffiffiffiffiffiffiffiffiffiffiffiffiffiffiffiffiffiffiffiffiffiffiffiffiffi2 :
ctgφctgðα  φÞ þ ctgψctgðβ  ψ Þ
As
cos α
ctgφctgðα  φÞ ¼ ctgφctgðα  φÞ  1 þ 1 ¼ þ1¼
sin φ sin ðα  φÞ
2 cos α 2 cos α α
¼ þ1 þ 1 ¼ ctg 2 ,
cos ðα  2φÞ  cos α 1  cos α 2

then

AB2
dc d0c   2 ¼ r :
2

ctg α2 þ ctg β2
 3
Then da d b d c d0a d 0b d0c ¼ dc d0c  r 6 , and if point M is the incenter of triangle
ABC, we obtain that da db d c d0a d0b d 0c ¼ r 6 :
Therefore, the greatest possible value of expression d a d b dc d 0a d0b d0c is equal to r6.
ðR2 OM2 ÞðR2 OM0 2 Þ
Remark One can prove that da d 0a ¼ 4R2
, where O is the circumcen-
ter of triangle ABC and R is its circumradius.
5.5.40. Let R be a positive root of
 
x3  d2a þ d2b þ d2c x  2d a d b dc ¼ 0, ð5:48Þ

and α, β, γ be acute angles, such that cos α ¼ dRa , cos β ¼ dRb , cos γ ¼ dRc .
Then, from (5.48) it follows that

cos 2 α þ cos 2 β þ cos 2 γ þ 2 cos α cos β cos γ ¼ 1:

From the last equation, we deduce that


pffiffiffiffiffiffiffiffiffiffiffiffiffiffiffiffiffiffiffiffiffiffiffiffiffiffiffiffiffiffiffiffiffiffiffiffiffiffiffiffiffiffiffiffiffiffiffiffiffiffiffiffiffiffiffiffiffiffiffiffiffiffiffiffiffiffiffi
cos γ ¼  cos α cos β þ cos 2 αcos 2 β  cos 2 α  cos 2 β þ 1 ¼
¼  cos α cos β þ sin α sin β ¼ cos ðπ  ðα þ βÞÞ:

Thus, it follows that α þ β þ γ ¼ π.


Obviously, one of the following inequalities holds true α  ∠ A, β 
∠ B, γ  ∠ C. If α  ∠ A, then
5.5 Using Trigonometric Inequalities for Proving Geometric Inequalities 257

qffiffiffiffiffiffiffiffiffiffiffiffiffiffiffiffiffiffiffiffiffiffiffiffiffiffiffiffiffiffiffiffiffiffiffiffiffiffiffiffiffiffiffi qffiffiffiffiffiffiffiffiffiffiffiffiffiffiffiffiffiffiffiffiffiffiffiffiffiffiffiffiffiffiffiffiffiffiffiffiffiffiffiffiffiffiffiffiffiffiffi
d2b þ d2c þ 2d b dc cos α d 2b þ d2c þ 2db d c cos ∠A
R¼  ¼ Ra
sin α sin ∠A
 maxðRa ; Rb ; Rc Þ:
Similarly, if β  ∠ B or if γ  ∠ C, then R  max(Ra, Rb, Rc).
Let min(α, β, γ) ¼ α, we have that α  π3. Therefore cos α  12 and
pffiffiffiffiffiffiffiffiffiffiffiffiffiffiffiffiffiffiffiffiffiffiffiffiffiffiffiffiffiffiffiffiffiffiffiffiffiffiffiffiffiffiffiffiffiffiffiffiffiffiffiffiffiffiffiffiffiffiffiffiffiffiffiffiffiffiffiffiffiffiffiffiffiffiffiffiffiffiffiffiffiffiffiffiffiffiffi
maxðRa ; Rb ; Rc Þ  R ¼ R cos 2 α þ cos 2 β þ cos 2 γ þ 2 cos α cos β cos γ 
rffiffiffiffiffiffiffiffiffiffiffiffiffiffiffiffiffiffiffiffiffiffiffiffiffiffiffiffiffiffiffiffiffiffiffiffiffiffiffiffiffiffiffiffiffiffiffiffiffiffiffiffiffiffiffiffiffiffiffiffiffi
pffiffiffiffiffiffiffiffiffiffiffiffiffiffiffiffiffiffiffiffiffiffiffiffiffiffiffiffiffiffiffiffiffiffiffiffiffiffiffiffiffiffiffiffiffiffiffiffiffiffiffiffiffiffiffiffiffiffiffiffiffiffiffiffiffiffiffiffiffiffiffiffiffiffi da db dc
 R cos 2 α þ cos 2 β þ cos 2 γ þ cos β cos γ ¼ d2a þ d 2b þ d2c þ :
maxðda ; d b ; d c Þ
qffiffiffiffiffiffiffiffiffiffiffiffiffiffiffiffiffiffiffiffiffiffiffiffiffiffiffiffiffiffiffiffiffiffiffiffiffiffiffiffiffiffiffiffiffiffiffiffiffiffiffi
Hence, maxðRa ; Rb ; Rc Þ  d 2a þ d2b þ d 2c þ maxdðaddab;ddbc;dc Þ:
This ends the proof.
5.5.41. We are going to use the following lemmas.
Lemma 1 Let the area S of a convex n-gon A1A2 . . . Ansatisfy to the following
inequality 4S  AnA2  R1 þ A1A3  R2 þ . . . þ An  1A1  Rn, where Riis the
circumradius of triangle Ai  1AiAi þ 1, i ¼ 1, 2, . . . , n, A0
An, An þ 1
A1.
Let Mi be the midpoints of AiAi þ 1, for i ¼ 1, 2, . . . , n.
For any i consider a quadrilateral, created by segments AiMi and AiMi  1, and
also by perpendiculars to these segments drawn from points Mi and Mi  1, respec-
tively. Let us prove that, these n quadrilateral cover given n-gon.
Indeed, let P be a point inside of n-gon. Let PAk be the smallest of the distances
PA1, PA2, . . . , PAn. We have that PAk  PAk þ 1и PAk  PAk  1, therefore point
P is inside of n-gon and any of the two semiplanes containing Ak are bounded by the
midpoints of perpendiculars to AkAk þ 1 and AkAk  1, this means that in k-th quad-
rilateral. In order to end the proof, it is sufficient to note that the area of i-th
quadrilateral is not greater than 12  Ai12Aiþ1  Ri :
According to the assumptions of our problem, it follows that

4S  2R21 sin ∠A1 þ 2R22 sin ∠A2 þ ::: þ 2R25 sin ∠A5 :

By Cauchy-Bunyakovsky inequality, we obtain that


qffiffiffiffiffiffiffiffiffiffiffiffiffiffiffiffiffiffiffiffiffiffiffiffiffiffiffiffiffiffiffiffiffiffiffiffiffiffiffiffiffiffiffiffiffiffiffiffiffiffiffiffiffiffiffiffiffiffiffiffiffiffiffiffiffiffiffiffiffiffiffiffiffiffiffiffiffiffiffiffiffiffi

2S  R41 þ ::: þ R45 ðsin 2 ∠A1 þ ::: þ sin 2 ∠A5 Þ
qffiffiffiffiffiffiffiffiffiffiffiffiffiffiffiffiffiffiffiffiffiffiffiffiffiffiffiffiffiffiffiffiffiffiffiffiffiffiffiffiffiffiffiffiffiffiffiffiffi
 
 5sin 2 1080 R41 þ ::: þ R45 ,

therefore R41 þ R42 þ R43 þ R44 þ R45  5sin 24108 S2 :


In this inequality we have used the following lemma.
Lemma 2 If α1, α2, . . . , α5are the angles of a convex pentagon, then sin2α1 þ . . .

þ sin2α5  5sin2108 .
258 5 Application of Trigonometric Inequalities

The given sum does not depend on the order of the angles; thus without loss of
generality one can assume that α1  α2  . . .  α5.
 
If α1 ¼ 108 , then α2 ¼ . . . ¼ α5 ¼ 108 , and the inequality becomes an equality.
  
If α1 < 108 , then α5 > 108 . Note that α1 þ α5 < 270 .
  
Otherwise, if α1 þ α5  270 , then α2 þ α3 þ α4  270 . Therefore α2  90 , and
 
α1  90 . Hence, we deduce that α5  180 . This leads to a contradiction.
Hence, we obtain that
  
sin 2 108 þ sin 2 α1 þ α5  108  sin 2 α1  sin 2 α5 ¼
   
¼ 2 cos ðα1 þ α5 Þ sin α1  108 sin α5  108 > 0:
 
This means that substituting α1 by 108 and α5 by α1 þ α5  108 makes the sum
of the squares of sines greater.
Repeating these steps several times, one can make all the angles to be equal to

108 .
This ends the proof.

Problems for Self-Study

Prove the inequalities for the elements of triangle ABC (5.5.33–5.5.37).


5.5.42. 1 þ Rr  sin α2 þ sin β2 þ sin 2γ  17
12 þ 6R.
r

5.5.43. sin α2  sin β2 þ sin β2  sin 2γ þ sin 2γ  sin α2  12 þ 2R


r
.
5.5.44. (a) m2a þ m2b þ m2c  27 2
4 R ,
pffiffiffiffiffiffiffiffiffiffiffiffiffiffiffiffiffiffiffi
(b) 3 3r ðR þ r Þ  ma þ mb þ mc  92 R,
pffiffiffi
pffiffiffiffiffiffiffiffiffi
(c) ma þ mb þ mc  3 3S, 2 2 2 pffiffiffi 
(d) 8ðma mb þ mb mc þ mc ma Þ  3 a þb2 þc þ 6 3S ,
(e) 2R(ma þ mb þ mc)  p2 þ r2 þ 4Rr,
pffiffiffi
(f) 4 3S  aþbþc
9abc
qffiffiffiffi
(g) ha  la  2rR,
2
(h) 10r  2rR  ha þ hb þ hc  2ðrþR Þ
2
R ,
(i) lalb þ lblc þ lcla  13Rr þ r .
2

5.5.45. a cos α þ b cos β þ c cos γ  p.


5.5.46. 13, 5Rr2  lalblc  16Rr2.
5.5.47. Let ABC be an arbitrary triangle and I be its incenter. Denote the intersection
points of the straight lines AI, BI, CI by A0 , B0 , C0 with the circumcircle of the
0 0 0
5 ðR  2r Þ  IA þ IB þ IC  IA  IB  IC
4
triangle ABC. Prove that
 2ðR  2r Þ.
5.5 Using Trigonometric Inequalities for Proving Geometric Inequalities 259

! ! !
5.5.48. Let point O be inside of triangle ABC, such that OK þ OM þ ON ¼ ~ 0,
where K, M, N are the feet of the perpendiculars drawn from point O to sides AB,
ABþBCþCA  2 3.
1 ffiffi
BC, AC. Prove that OKþOMþON p

5.5.49. Let I be the incenter of triangle ABC, and R and r be, respectively, the
circumradius and the inradius of triangle ABC. Prove that
(a) R3  IA  IB  IC,
p ffiffiffiffiffiffiffiffiffiffi
(b) 3 4Rr 2  IA þ IB þ IC  2R þ 2r.
3

Hint Prove that IA  IB  IC ¼ 4Rr2.


5.5.50. Prove that for the triangle with angles α, β, γ and circumradius R the
inequality tg α2 þ tg β2 þ tg 2γ  9R
2
4S holds true, where S is the area of the triangle.

5.5.51. Let ABC be a triangle, such that β < π2 , γ < π2. Prove that R þ r < bþc
2 , if and
π
only if α < 2.
5.5.52. Let O be the intersection point of bisectors AA1, BB1, CC1 of the triangle
ABC. Prove that 8  AAO  BO  CO  4R
1 O B 1 O C1 O r , where R and r are, respectively, the
circumradius and the inradius of triangle ABC.
5.5.53. Prove that 36r1r2  a1a2 þ b1b2 þ c1c2  8R1R2 þ 4r1r2, where Ri and ri are,
respectively, the circumradius and the inradius of the triangles with sides ai, bi, ci
(i ¼ 1, 2).
Hint See the problems 5.1.31 and 5.1.32.
5.5.54. Let triangle ABC be cut of the paper. Bend triangle ABC around the segment
passing through A, so that the area of obtained figure is minimal, if:
(a) ∠C  ∠ B  3 ∠ C,
 
(b) ∠A ¼ 40 , ∠B ¼ 125 .
Hint Prove that triangle ABC should be bent (a) around bisector AD, (b) around
segment AX, where X 2 [BC] and ∠XAC ¼ ∠ C.
5.5.55. Let D, E, F be points on sides BC, CA, AB of triangle ABC, respectively.
Prove that
(a) PDEF  min(PAEF, PBDF, PCDE),
(b) rDEF  min(rAEF, rBDF, rCDE),
where PXYZ and rXYZ are the perimeter and the inradius of triangle XYZ,
respectively.
Chapter 6
Inequalities for Radiuses

This chapter consists of Sections 6.1 and 6.2. In Section 6.1, selected problems
related to the inequalities with radiuses of circles are provided. Perhaps, the most
well known among them is the following one. Prove that R  2r, where R and r are
the circumradius and inradius of triangle ABC, respectively.
One of the proofs of this problem is obtained by Euler’s formula,
pffiffiffiffiffiffiffiffiffiffiffiffiffiffiffiffiffiffiffi
d ¼ R2  2Rr , where d is the distance between the circumcenter and incenter
of triangle ABC.
One of the methods to prove that R  2r and many other inequalities is the
following statement: if triangle ABC is circumscribed to a circle with radius r and
points M, N, K are three points not on the same line, belonging to lines AB, BC, AC,
then the circumradius of triangle MNK is not less than r. In the proofs of many
problems of this paragraph is used the following statement, if a circle does not have
any points outside of triangle ABC, then the radius of that circle is not greater than
the inradius of triangle ABC.
Most of the problems provided in Section 6.1 can be proved using trigonometry.
In Section 6.2, problems related to polygons, such that all of their vertices are
chosen at the nodes of the integer lattice are considered. Among those problems, we
would like to emphasize the following one, as it has a wide range of applications: if
the vertices of a triangle are at the nodes of the integer lattice, then the area of the
triangle is not less than 0.5.
Some problems in this chapter were inspired by [4, 7, 15]. Nevertheless, even for
these problems the authors have mostly provided their own solutions.

© Springer International Publishing AG 2017 261


H. Sedrakyan, N. Sedrakyan, Geometric Inequalities, Problem Books
in Mathematics, DOI 10.1007/978-3-319-55080-0_6
262 6 Inequalities for Radiuses

6.1 Inequalities for Radiuses of Circles

6.1.1. Prove that for the triangle with sides a, b, and c, the inequality 2uvR  (u + v)2
r holds true, where 2u ¼ min (a + b  c, b + c  a, c + a  b) and 2v ¼ max (a
+ b  c, b + c  a, c + a  b).
6.1.2 One of the inner angles of the triangle is equal to α. Prove that
2R 1
(a)   ,
r sin α2 1  sin α2
(b) R  2r.
6.1.3. Consider three circles pairwise not having any common internal points
intersecting pairwise, such that their centers are on one straight line. Prove that, if
the fourth circle is tangent to all three circles its radius cannot be less than the
radiuses of these three circles.
6.1.4. (a) Circle S1 touches sides AC, AB of triangle ABC, circle S2 touches sides
BC, AB, and S1, S2 touch each other externally. Prove that the sum of the radiuses of
these circles is greater than the radius of the incircle S of triangle ABC.
(b) Given a triangle ABC. Circles with radiuses r1, r2, and r3 are such that each
touches the other two circles and two sides of the triangle. Prove that r1 + r2 > r3.
6.1.5. Let two circles touch each other externally and touch the circle with the
radius R internally on diametrically opposite points. Given a circle with radius R1
touching all three circles, prove that R1  R3 .
6.1.6. Let M be the midpoints of side BC of triangle ABC, r1 and r2 be the radiuses
of the incircles of triangles ABM and ACM, respectively. Prove that r1 < 2r2.
6.1.7. A rectangle is cut into rectangles. Prove that the sum of the circumradiuses of
all these rectangles is not less than the circumradius of the initial rectangle.
6.1.8. (a) Prove that, if the polygon circumscribed around a circle with radius r is
broken into convex circumscribed polygons, then the sum r1 + . . . + rn of the
inradiuses of these polygons is not less than r.
(b) Prove that, if a circle with radius r0 is inside of the polygon circumscribed
around a circle with radius r, then r0  r.
6.1.9. Let point M belong to the diameter AB ¼ 2R of some half-circle.
From point M are drawn segments MC and MD, such that
α ¼ ∠AMC ¼ ∠BMD  90 and points C, D are on that half-circle. Given that
the circle with radius r1 touches the half-circle and segments CM, AM. Given
also that the circle with radius r2 touches the half-circle and segments DM, MB.
Prove that
pffiffiffi 
(a) r 1 þ r 2  2 2  1 R,
α
(b) r 1 þ r 2  2 sin 2
.
1 þ sin α2R
6.1 Inequalities for Radiuses of Circles 263

6.1.10. A circle is inscribed into triangle A1B1C1. Let the vertices of triangle
Ai + 1Bi + 1Ci + 1 be the intersection points of the bisectors of triangle AiBiCi with
the given circle, i ¼ 1, 2, . . ., and ri be the inradius of triangle AiBiCi. Prove that
(a) rn + 1  rn, for n ¼ 1, 2, . . .,
(b) 2rn  rn  1 + rn + 1, for n ¼ 3, 4, . . ..
6.1.11. Given an acute triangle ABC and a point M on the line AB.
Prove that R1 + R2 > R, where R1, R2, R are the circumradiuses of triangles ACM,
BCM, ABC, respectively.
6.1.12. (a) Let ABCD be a convex quadrilateral, RA, RB, RC, and RD the
circumradiuses of triangles DAB, ABC, BCD, and CDA, respectively. Prove that
RA + RC > RB + RD, if and only if ∠A + ∠C > ∠B + ∠D.
(b) Let ABCD be a convex quadrilateral, rA, rB, rC, and rD the inradiuses of the
triangle DAB, ABC, BCD, and CDA, respectively. Prove that rA + rC > rB + rD, if
and only if ∠A + ∠C < ∠B + ∠D.
6.1.13. Let points C1, B1, and A1 be chosen, respectively, on sides AB, AC, and BC
of the equilateral triangle ABC with side length 2. What is the greatest possible
value of the sum of the inradiuses of triangles AB1C1, A1BC1, A1B1C ?
6.1.14. Let the inradius and the circumradius of the regular n-gon be equal to rn and
 2
Rn, respectively. Prove that Rrnn  Rrnþ1
nþ1
.

6.1.15. Let r be the inradius of triangle ABC, rA be the radius of its excircle,
touching sides AB, AC, (rB and rC are defined similarly).
2
Prove that (a) rA + rB + rC  r, (b) r A r B þ r B r C þ r C r A  r3 .
6.1.16. Prove that R  3r, where R is the radius of the circumsphere of the tetrahe-
dron ABCD, and r is the radius of its insphere.

Solutions

6.1.1. Let a  b  c, then 2u ¼ a + b  c and 2v ¼ b + c  a. We have to prove


aþbc bþca abc S
that 2  R  b2 r. We have R¼ , r ¼ , and
2 2
pffiffiffiffiffiffiffiffiffiffiffiffiffiffiffiffiffiffiffiffiffiffiffiffiffiffiffiffiffiffiffiffiffiffiffiffiffiffiffiffiffiffiffiffiffiffi 4S p
S ¼ pð p  aÞ ð p  bÞ ð p  c Þ .
Thus, we have to prove that ac  b(a + c  b), or equivalently, (b  c)(a  b)  0.
The last inequality obviously holds true. This ends the proof.
β γ
6.1.2. (a) We have a ¼ rctg þ rctg ¼ 2R sin α, consequently
2 2
264 6 Inequalities for Radiuses

 
β γ β γ
þ
2R ctg 2 þ ctg 2
sin
2 2 1
¼ ¼ ¼ ¼
r sin α β γ α β γ
sin α sin sin 2 sin sin sin
2 2 2 2 2
1 1 1
¼    ¼ α α:
α βγ βþγ α βþγ sin 1  sin
sin cos  cos sin 1  cos 2 2
2 2 2 2 2
 
(b) Note that 0 < sin α2 1  sin α2  14. According to problem 6.1.2a, we obtain
r  4, or R  2r. This ends the proof.
that 2R
6.1.3. Denote the centers and the radiuses of these circles (see Figure 6.1) by O, O1,
O2, O3 and r, r1, r2, r3, respectively.
We have that OO1 + OO3  O1O3 and r + r1  OO1, r + r3  OO3,
O1O3  r1 + 2r2 + r3. Therefore, r + r1 + r + r3  r1 + 2r2 + r3, or r  r2.
6.1.4. (a) Since ∠O1OO2 > 90 , we have that OM < O12O2 ¼ r1 þr
2 , where M is the
2

r1 þr2
midpoint of segment O1O2 (see Figure 6.2). Hence, r  OM þ 2 < r 1 þ r 2 .
Remark If circles S1 and S2 do not touch each other, then 2r  O1O2 + r1 + r2. The
equality holds true, only if these circles coincide.
(b) According to problems 6.1.4a and 6.1.8b, we have that r1 + r2 > r > r3.

Figure 6.1

O1 O2 O3

Figure 6.2 C

O
O2
O1 M r

A B
6.1 Inequalities for Radiuses of Circles 265

Figure 6.3

R1
R-x+R1
x R-R1

R-x x

6.1.5. We have that x < R (see Figure 6.3).


Proof by contradiction argument. Assume that R1 > R3 .
By Stuart’s theorem, we have that

ðR  R1 Þ2 R ¼ ðR1 þ xÞ2 x þ ðR þ R1  xÞ2 ðR  xÞ  ðR  xÞxR:

Therefore,
 2  2
4R3 R 4R
> ðR  R1 Þ2 R > þx xþ  x ðR  xÞ  ðR  xÞxR:
9 3 3

Hence, we deduce that 0 > (R  2x)2. This leads to contradiction.


6.1.6. Let r be the inradius of triangle ABC. It is clear that r1 < r (see problem
6.1.8b). By homothety with center B and ratio 2, the image of triangle ABM
contains triangle ABC. Therefore, r < 2r1. Thus, 2r < r 1 < r.
Similarly, we can prove that 2r < r 2 < r. Hence, r1 < 2r2.
6.1.7. We have that (see Figure 6.4),

2R ¼ AB ¼ AA1 þ A1 A2 þ . . . þ Ak B  2R1 þ 2R2 þ . . . þ 2Rkþ1


 2R1 þ . . . þ 2Rn :

Therefore, R  R1 + . . . + Rn.
6.1.8. (a) Let the semiperimeters of the polygons circumscribed around the circle
with radiuses r1, r2, . . . , rn, r are equal to p1, p2, . . . , pn, p, respectively.
We have that S ¼ S1 + . . . + Sn and pi < p (see problem 2.1.1). Consequently,
r 1 þ . . . þ r n ¼ Sp1 þ . . . þ Spn > S1 þ...þS
p
n
¼ Sp ¼ r.
1 n
266 6 Inequalities for Radiuses

Figure 6.4 B

Ak 2Rk+1
2R2
A3
A2
A1 2R3

2R1
А

(b) Let the sides of the polygons circumscribed around the circle with radius r be
equal to a1, a2, . . . , an, and the area to S. Then, S  a12r0 þ a22r0 þ . . . þ an2r0 .
Hence, r 0  a1 þ...þa
2S
n
¼ r and the equality holds true, if and only if the circle with
radius r0 is the incircle of the given polygon.
6.1.9. Let O1, O2, and O be circles with radiuses r1, r2, and R, respectively. Consider
a point symmetric to point O1 with respect to the straight line AB.
Let that point is O01 . Since ∠O01 MA ¼ ∠O1 MA ¼ ∠AMC 2 and
∠BMD 0
∠O2 MB ¼ 2 , ∠AMC ¼ ∠BMD, we have that ∠O1 MA ¼ ∠O2 MB. This
means that points O01 , M, O2 are on the same line. According to the triangle
inequality, we have that O01 O2  O01 O þ OO2 .
(a) Note that O01 O ¼ O1 O ¼ R  r 1 , OO2 ¼ R  r2 and ∠O1MA ¼ ∠O2MB  45 ,
pffiffiffi pffiffiffi
thus O1 M ¼ sin ∠O
r1
1 MA  2 r 1 and O 2 M ¼ sin
r2
∠O 2 MB  2r 2 .
pffiffiffi pffiffiffi
To end the proof we have to use the obtained inequalities: 2r 1 þ 2r 2  O1
M þ O2 M ¼ O01 M þ O2 M ¼ O01 O2  O01 O þ OO2 ¼ R  r 1 þ R  r 2 :
pffiffiffi 
ffiffi ¼ 2 2  1 R.
Therefore, r 1 þ r 2  p2R
2þ1

(b) We have that

r1 r2 0 0 0
αþ α ¼ O1 M þ O2 M ¼ O1 M þ O2 M ¼ O1 O2  O1 O þ OO2 ¼
sin sin
2 2
¼ O1 O þ O2 O ¼ R  r 1 þ R  r 2 :
α
Thus, it follows that r 1 þ r 2  2 sin sin α2þ1R.
2

6.1.10. Let us deduce (recall) the formula expressing the inradius of the triangle
through its angles and the circumradius. Let α, β, γ be the angles of triangle ABC
and I, R be the incenter and the circumradius, respectively (see Figure 6.5).
6.1 Inequalities for Radiuses of Circles 267

Figure 6.5 B

a/2 g/2
A C

We have that
γ
α AC sin α 2R sin β γ α
r ¼ AI sin ¼ 2
2 α þ γ sin 2 ¼ β
sin sin ¼
2 2
sin cos
2  2 
α β γ αβ αþβ γ
¼ 4R sin sin sin ¼ 2R cos  cos sin ¼
 2 2 2 2 2 2 
αþγβ βþγα αþβþγ αþβγ
¼ R sin þ sin  sin þ sin ¼
2 2 2 2
¼ Rð cos α þ cos β þ cos γ  1Þ:

Without loss of generality, one can take R ¼ 1. If ρ is the inradius of the triangle
with the angles αþβ βþγ γþα
2 , 2 , 2 , then

αþβ βþγ γþα


ρ  r¼ cos þ cos þ cos  cos α  cos β  cos γ ¼
 2 2 2  
αþβ 1 βþγ 1
¼ cos  ð cos α þ cos βÞ þ cos  ð cos β þ cos γ Þ þ
 2 2  2 2
ð6:1Þ
γþα 1
þ cos  ð cos γ þ cos αÞ ¼
2 2
αþβ 2αβ βþγ 2βγ αþγ 2αγ
¼ 2 cos sin þ 2cos sin þ 2 cos sin :
2 4 2 4 2 4

Let us now continue the proof of our problem.


The part (a) follows immediately from (6.1), since all summands in it are
nonnegative and from triangle AiBiCi with the angles αi, βi, γ i by a transition
described in the problem we get a triangle with the angles equal to αi þβ βi þγ i
2 , 2 ,
i γ i þαi
2 .

(b) Since for any initial triangle the second triangle is acute-angled, we consider an
acute triangle with angles α, β, γ. Hence, we get a triangle with the angles
βþγ π α γþα π β αþβ π γ
2 ¼ 2  2, 2 ¼ 2  2, 2 ¼ 2 2 .

This triangle, in turn, will transform into a triangle with the angles equal to
π
2  βþγ π α π γþα π β π αþβ π
4 ¼ 4 þ 4 , 2  4 ¼ 4 þ 4 , 2  4 ¼ 4 þ 4.
γ
268 6 Inequalities for Radiuses

Let r2, r3, r4 be the corresponding inradiuses of these triangles and


for simplicity let us assume that α  β  γ < π2. According to (6.1), we have that
r 3  r 2 ¼ 2 sin α2 sin 2 βγ þ 2 sin β2 sin 2 γα þ 2 sin 2γ sin 2 βα, and r4  r3 ¼
π α 2 βγ 4 π β  24γα π γ4 2 βα
2 sin 4  4 sin 8 þ 2 sin 4  4 sin 8 þ 2 sin 4  4 sin 8 .
The last equality can be obtained from the previous one substituting α, β, γ by
βþγ γþα αþβ
2 , 2 , 2 and by a simple transformation.
We have that
 
2βγ α 2 β  γ 1 π α 
2r 3  r 2  r 4 ¼ ðr 3  r 2 Þ  ðr 4  r 3 Þ ¼ 8sin sin cos  sin  þ
8 2 8 4 4 4
  
2γα β 2γα 1 π β
þ 8sin sin cos  sin  þ
8 2 8 4 4 4
 
βα γ β  α 1 π γ 
þ 8sin 2 sin cos 2  sin  :
8 2 8 4 4 4

If α  π6, then each summand in the right-hand side is nonnegative,

α βγ 1 π α  α π 1 π α 
sin cos 2  sin  > sin cos 2  sin  
2 8 4 4 4  21 16π 4 4 4
π π 1 π π π
 sin cos 2  sin  ¼ 2sin  sin >0
12 16 4 4 24 4 12 24

Thus, it follows that, 2r3  r2  r4  0.


Let α < π6, then π3 < β  γ < π2, and as β  α > π6, γ  α > π6, 0  γ  β < π6, we
obtain that
 
γβ α β  γ 1 π α 
2r 3  r 2  r 4 > 8sin 2 sin cos 2  sin  þ
8 2 8 4  4  4
π β γα 1 π β
þ8sin 2 sin cos 2  sin  þ
48 2 8 4 4 4 
π γ β  α 1 π γ 
þ8sin 2 sin cos 2  sin  >
48 2 8 4 4 4 
2γβ 1 π 2 π π 2γα 1 π
> 8sin  sin þ 8sin sin cos  sin þ
8
 4 4 48 6 8 46
π π β  α 1 π π 1 π
þ8sin 2 sin cos 2  sin > 8sin 2  sin þ
48   6 8 4 6  48 4 4
2 π 1 2γα 2βα 1
þ8sin cos þ cos  ¼
48 2 8 8 4
0 1
γα βα pffiffiffi
πB 1 þ cos 1 þ cos
¼ 8sin 2 @ 4 þ 4  1  2C A¼
48 4 4 4 8
 pffiffiffi 
π 2 2 1 γα 1 βα
¼ 8sin 2 þ cos þ cos > 0,
48 8 4 4 4 4

Therefore, 2r3  r2  r4 > 0. This ends the proof.


6.1 Inequalities for Radiuses of Circles 269

Remark If α ¼ 0, β ¼ 0, γ ¼ π, then

α βγ β γα γ βα


2 sin sin 2 þ 2 sin sin 2 þ 2 sin sin 2 <
2  4  2 4  2 4
π α βγ π β γα
< 2 sin  sin 2 þ 2 sin  sin 2
4 4 8 4 4 8
π γ  βα
þ 2 sin  sin 2 :
4 4 8

6.1.11. Note that R1  AC


2 and R2  2 .
BC

Therefore, R1 þ R2  2 > AOþBO


ACþBC
2 ¼ R, where O is the circumcenter of
triangle ABC (see problem 1.1.8a).
6.1.12. (a) Let O be the intersection point of segments AC and BD. It is clear that
one of the angles ∠AOB and ∠BOC is not less than 90 , and we may consider that
∠BOC  90 (otherwise A and C can be interchanged).
Let us denote ∠DAO ¼ α, ∠CBO ¼ β, ∠BCO ¼ γ and ∠ADO ¼ δ, then the
angles α, β, γ, δ are acute. By the law of sines we have that,
2RD sin α ¼ CD ¼ 2RC sin β and 2RA sin δ ¼ AB ¼ 2RB sin γ. Therefore,
γ sin α
RA þ RC ¼ RB sinsin δ þ R D sin β .
If ∠A + ∠C > ∠B + ∠D, we have that ∠A + ∠C > 180 . This means that point
A is inside of the circumcircle of triangle BCD. Hence, it follows that δ < γ and
δ sin β
β < α. Thus, RA þ RC > RB sin sin δ þ RD sin β ¼ RB þ RD .
If ∠A + ∠C < ∠B + ∠D, then ∠B + ∠D > ∠A + ∠C. Hence, RB + RD > RA +
RC or RA + RC < RB + RD. Therefore, if RA + RC > RB + RD, then ∠A + ∠C < ∠B +
∠D does not hold true.
For ∠A + ∠C ¼ ∠B + ∠D, we have that ABCD is an inscribed quadrilateral, but
then RA ¼ RB ¼ RC ¼ RD. Thus, RA + RC ¼ RB + RD. This leads to contradiction.
Thus, it follows that ∠A + ∠C > ∠B + ∠D.
(b) We need to prove that, if ∠B + ∠D > 180 , then rA + rC > rB + rD.
Denote by OA, OB, OC, and OD the incenters of triangles DAB, ABC, BCD, and
CDA, respectively.
At first, let us prove the following statements.
1. OAOBOCOD is a convex quadrilateral.
Note that

1 1
∠ABOA ¼ ∠ABD < ∠ABC ¼ ∠ABOB and ∠BAOB < ∠BAOA :
2 2

Therefore, segments AOB and BOA intersect. Denote their intersection point by
M (Figure 6.6) and the intersection point of segments BOC and COB by N.
270 6 Inequalities for Radiuses

Figure 6.6 B

OB
N
M

OA OC
A C

We have that

 1
∠AOB B ¼ 90 þ ∠ACB: ð6:2Þ
2

Therefore, ∠AOBB > 90 . Similarly, ∠BOBC > 90 and ∠AOBC > 90 , this
means that point OB is inside triangle BMN. Thus, it is also inside triangle OABOC.
Exactly in the same way, we can prove that point OD is inside triangle OADOC.
Since the quadrilateral OABOCD is convex, we obtain that the quadrilateral
OAOBOCOD is convex too.
2. If ∠B + ∠D ¼ 180 , then ∠OAOBOC ¼ 90 .
Indeed, since ∠B + ∠D ¼ 180 , then one can circumscribe a circle around the
quadrilateral ABCD. Therefore, ∠ADB ¼ ∠ACB. Taking this into consideration and
by (6.2) we deduce that ∠AOAB ¼ ∠AOBB. This means that, points A, B, OB, and OA
 
are on the same circle. Thus, ∠BOB OA ¼ 180  ∠BAOA ¼ 180  12 ∠BAD. Simi-

larly, we obtain that ∠BOB OC ¼ 180  12 ∠BCD. Hence, it follows that
 
∠BOB OA þ ∠BOB OC ¼ 360  12 ð∠A þ ∠CÞ ¼ 270 . Thus, ∠OAOBOC ¼ 90 .
Corollary If ∠B + ∠D ¼ 180 , then OAOBOCOD is a rectangle.
3. If ∠B + ∠D > 180 , then ∠OAOBOC > 90 .
Since ∠B + ∠D > 180 , then point D is inside the circumcircle of the triangle
ABC. Let the straight line BD intersects that circle at point D0 (Figure 6.7).
Denote the incenters of triangles ABD0 and CBD0 by O0A and O0C , respectively. As
∠BAO0 A > ∠BAOA, then point OA is on segment O0A B.
6.1 Inequalities for Radiuses of Circles 271

Figure 6.7 B

OB
OC
OA C
A O¢A O¢C

Similarly, point OC is on segment O0C B. By the statement 2, we have that


 
∠O0A OB O0C ¼ 90 . Therefore, ∠OA OB OC > ∠O0A OB O0C ¼ 90 .
Corollary

If ∠B þ ∠D > 180 , then OB OD < OA OC : ð6:3Þ

Indeed, we have that ∠OAOBOC > 90 and ∠OAODOC > 90 . Therefore, points
OB and OD are inside a circle with diameter OAOC. Thus, OBOD < OAOC.
Let the incircle of triangle DAB touches side BD at point A1 (points B1, C1 and
D1 are defined similarly).
4. Prove that A1C1 ¼ B1D1.
We have that,

AD þ BD  AB BD þ CD  BC
A1 C1 ¼ jDA1  DC1 j ¼  ¼

2 2
ð6:4Þ
AD þ BC  AB  CD
¼ ¼ B1 D 1 :

2

Let us now continue the proof of the problem.


Note that,

OA O2C ¼ A1 C21 þ ðr A þ r C Þ2 and OB O2D ¼ B1 D21 þ ðr B þ r D Þ2 : ð6:5Þ

If ∠B + ∠D > 180 , then from (6.3), (6.4), and (6.5) we deduce that
rA + rC > rB + rD.
If ∠B + ∠D ¼ 180 , then using the corollaries of the statements 2, 3, and 4, we
obtain that rA + rC ¼ rB + rD.
If ∠B + ∠D < 180 , then ∠A + ∠C > 180 . Therefore, rB + rD > rA + rC.
272 6 Inequalities for Radiuses

Let rA + rC > rB + r D, then as we have already proven, ∠A + ∠C > 180 or


∠A + ∠C ¼ 180 are impossible. This means that ∠A + ∠C < 180 . Hence, we
obtain that ∠A + ∠C < ∠B + ∠D.
Let ∠A + ∠C < ∠B + ∠D, then ∠A + ∠C < 180 . Therefore, rA + rC > rB + rD.
6.1.13. Let r1, r2, r3 be the inradiuses of triangles AB1C1, BA1C1, CA1B1, respec-
tively. Note that
 
π π π ∠AC1 B1 ∠AB1 C1
6 ¼ 2r 1 ctg þ 2r 2 ctg þ 2r 3 ctg þ r 1 ctg þ ctg þ
6 6 6   2 2 
∠BC1 A1 ∠BA1 C1 ∠CA1 B1 ∠CB1 A1
þ r 2 ctg þ ctg þ r 3 ctg þ ctg :
2 2 2 2
ð6:6Þ

xþy 

According to inequality ctgx þ ctgy  2ctg , where x, y 2 0; π2 (see the


2
proof of problem 6.1.10). From (6.6) we deduce that 6  2r 1 ctg π6 þ 2r 2 ctg π6 þ 2r 3
ctg π6 þ 2r 1 ctg π6 þ 2r 2 ctg π6 þ 2r 3 ctg π6  4ctg π6 ðr 1 þ r 2 þ r 3 Þ. Thus, it follows that
pffiffi
r 1 þ r 2 þ r 3  23.
If A1, B1, C1 are the midpoints of sides BC, AC, AB, respectively, then
pffiffi
r 1 þ r 2 þ r 3 ¼ 23.
pffiffi
Thus, the greatest value of the sum of radiuses r1, r2, r3 is equal to 23.
6.1.14. We have that Rrnn ¼ cos πn. We have to prove that cos πn  cos 2 nþ1 π
, for n ¼ 3,
2 π 2π π
p ffiffi
ffi π
4, 5, . . ., or sin nþ1  2sin 2n, sin nþ1  2 sin 2n.
sin α sin ðα  βÞ αβ α
If 0 < β < α < π2, then ¼ þ cos ðα  βÞ < þ1¼ .
sin β tgβ β β
π
pffiffiffi
π π 2 2
Also, for n  3, nþ1  π4. Hence, sin 4
π . Therefore, sin  .
π nþ1 nþ1
sin  π4
n þ 1 nþ1
π
nþ1 pffi
Then, for n  4 we have that sin 2 2
, as (4  π)n  (4  π)4 > π.
π
sin 2n > πnþ1pffi
> 2
2n
π
While, for n ¼ 3, we have that sin nþ1
π .
π
sin 2n ¼ sin sin π4¼pffiffi2
π
pffiffiffi π
6

Thus, for n  3, we obtain that sin nþ1  2 sin 2n .

6.1.15. (a) We have that sin ∠A


2 ¼ rþrA , according to problem 5.1.5, it follows
rrA

sin ∠A ∠B ∠C
2 þ sin 2 þ sin 2  2. Therefore,
3
rþrA þ rþrB þrþrC  2. Hence, rþrA þ
rrA rrB rrC 3 r
  
rþrB þ rþr C  4. Note that 9  rþr þ rþr þ rþrr C rþrr þ r þ r  94 
r r 9 r r A rþr B rþr C
A B
3rþrA þrB þrC
r .
Thus rA + rB + rC  r.
6.1 Inequalities for Radiuses of Circles 273

1 sin ∠A
(b) r A ¼ r 1þ sin ∠A2 ¼ r  tg 2 π∠A
4 . Therefore,
2

r A r Bþ rB rC þ rC rA ¼ 
2 π  ∠A 2 π  ∠B 2 π  ∠B 2 π  ∠C 2 π  ∠C 2 π  ∠A
¼ r tg2
tg þ tg tg þ tg tg 
4 4 4 4 4 4
 
r2 π  ∠A π  ∠B π  ∠B π  ∠C π  ∠C π  ∠A 2
 tg tg þ tg tg þ tg tg ¼
3 4  4 4  4 4  4
r2 π  ∠B π  ∠A π  ∠C π  ∠A π  ∠C
¼ tg tg þ 1  tg tg þ
3 4 4 4 4 4

π∠A 2 r2
þtg π∠C 4 tg 4 ¼ :
3

6.1.16. Let the center O of the insphere of the tetrahedron ABCD be inside of
it. Then, V ¼ 13 SBDC  ρðA; BDCÞ  13 SBDC  ðR þ ρðO; BDCÞÞ.
We need to obtain similar inequalities for other three faces of the tetrahedron.
Summing up all these four inequalities, we deduce that 4V  13 SR þ V, and as
V ¼ 13 SR, then it follows that R  3r.
Now, let points O and A are not at the same side of the plane BCD. Then, ρ(A,
BCD)  R. Let the plane passing through the center of the insphere of the tetrahe-
dron ABCD and parallel to the plane BCD intersects the edges AB, AC, AD at points
B0 , C0 , and D0 , respectively.
Denote the circumcenters of triangles B0 C0 D0 and BCD by O0 and O1, respec-
tively. As O0 D0 > 2r and ρðρA;BDC Þr O0 D0
ðA;BDCÞ ¼ O1 D > O1 D, then 1 > ρðA;BDCÞ þ O1 D  R þ R .
2r r 2r r 2r

Thus, it follows that R > 3r.


This ends the proof.

Problems for Self-Study

6.1.17. Prove that for any triangle ABC


(a) r  ρa þ ρb þ ρc  34 R, where ρa, ρb, ρc are the radiuses of three circles,
inscribed into the segments cut of the circumcircle K of triangle ABC by the
sides of triangle BC, CA, AB (circles touching the circumcircle and one of the
sides of the triangle at the midpoint (see Figure 6.8)).
(b) 4r  pa , pb , pc  43 ðR þ r Þ, 16
3 r  pa pb þ pb pc þ pc pa  3 Rr, 27 r  pa pb pc 
2 8 64 3
32 2
27 Rr , where pa, pb, pc are the radiuses of three circles touching two sides of the
triangle (not their extensions) and the circumcircle of the triangle (see
Figure 6.9).
274 6 Inequalities for Radiuses

Figure 6.8 A

rb
rc

B C
ra

Figure 6.9 A

pb
pc

pa

B C

Hint
(b) Prove that pa ¼ pðbcr
paÞ.

6.1.18. Let a quadrilateral be inscribed into a unit square, such that its sides are the
hypotenuses of the triangles. Given that in each triangle is inscribed a circle. Prove
pffiffiffi
that the sum of the radiuses of these circles does not exceed 2  2 and is equal to
pffiffiffi
2  2, only if the sides of the inscribed quadrilateral are parallel to the diagonals
of the square.
Hint See problem 1.1.10.
6.1.19. Prove that the inradius of any face of the tetrahedron is greater than the
radius of its insphere.
6.1.20. Prove that for any triangle ABC
6.2 Integer Lattice 275

(a) 1r ¼ r1a þ r1b þ r1c  R2 ,


pffiffiffiffiffiffiffiffi pffiffiffiffiffiffiffiffi pffiffiffiffiffiffiffiffi
(b) 9r  r a r b þ r b r c þ r c r a  r þ 4R,
(c) r 2a þ r 2b þ r 2c  27r 2 ,
(d) 4R < ra + rb + rc  4, 5R.


n  R  r, where R is the circumradius and r is the inradius


6.1.21. Prove that cos 180
of any convex n-gon.
6.1.22. Given a point B on segment AC. Points M and N are such that AM ¼ MB,
BN ¼ NC and MN||AC. Let r, r1 and r2 be, respectively, the inradiuses of triangles
MBN, AMB, and BNC. Prove that, if r1 < r2, then r1 < r < r2.
Hint Prove that 2SMBN ¼ SAMB + SBNC and 2PMBN ¼ PAMB + PBNC.

6.2 Integer Lattice

6.2.1. An integer lattice is the set of all points (knots) that have integer coordinates
(in a rectangular coordinate system). Prove that, if the vertices of a parallelogram
coincide with the lattice points of an integer lattice and inside of the parallelogram
or on its border there are other lattice points, then the area of such parallelogram is
greater than 1.
6.2.2. The vertices of a convex pentagon are in the integer lattice. Prove that, the
area of the pentagon is not less than 2.5.
6.2.3. (a) The vertices of the quadrilateral ABCD are in the integer lattice. Given
that the angles A and C of the quadrilateral are equal, and the angles B and D are not
equal. Prove that |AB  BC  CD  DA|  1.
(b) Vertices of the inscribed quadrilateral ABCD are in the integer lattice. Given that
ABCD is neither a trapezoid nor a rectangle. Prove that |AC  AD  BC  BD|  1.
6.2.4. Given on a plane a circle with the radius r and with the center at the origin of
the rectangular coordinate system. Let δ(r) be the distance from the nearest point
with the integer coordinates to that circle. Prove that δðr Þ < p2ffir. (The distance
between the point on a plane and a circle is defined as follows: draw a line through
the given point and the center of the circle, then find the distance between the point
and the nearest intersection point of the drawn line with the circle).
6.2.5. Given a figure, with the area smaller than 1, on a grid plane having a grid size
(length) equal to 1. Prove that, it is possible to translate this figure on the grid plane,
such that none of the grid points (knots) is inside of it.
6.2.6. (a) Vertices of an acute triangle ABC are in the integer lattice. Given that
pffiffiffiffiffi
minðAB; BC; ACÞ  65. Prove that SABC  30.
276 6 Inequalities for Radiuses

(b) Vertices of an acute triangle ABC are in the integer lattice. Given that min(AB,
BC, AC)  2000. Prove that inside of that triangle there is a lattice point, such
that either the difference or the sum of its coordinates is divisible by 2000.
6.2.7. Given on a coordinate plane a square S with dimensions n  n. Prove that, for
any disposition of the square it covers not more than (n + 1)2 integer points (both
x and y coordinates are integer numbers), where n 2 N.
6.2.8. Prove that any convex integer polygon (a polygon with the integer coordinates
of all vertices) with area S can be placed into a integer parallelogram with area 4S.
6.2.9. Let each point of a plane with the integer coordinates be the center of a disk
1
with the radius 1000. Prove that
(a) there exists an equilateral triangle, such that all the three vertices are in different
disks,
(b) the side of any equilateral triangle, such that all vertices are in different disks, is
greater than 100.
6.2.10. Consider on a plane nonself-intersecting polygons, such that their all sides
are expressed by integers, and angles are either right or are equal to 270 . Let S be
the area of one of these polygons. Prove that 2n  1  S  n2, if all polygons have
the same given perimeter equal to 4n.
6.2.11. Let the vertices of a triangle be in the knots of the integer lattice. Given that
inside of the triangle there are n lattice points, where n 2 N. What is the greatest
number of the lattice points that can be on the boundaries of the triangle (vertices
included)?

Solutions

6.2.1. We need to prove that, if the vertices of the triangle coincide with the lattice
points of the integer lattice, then the area of the triangle is not smaller than 12.
Indeed, any triangle can be built up to a rectangle by adding rectangular triangles
(Figure 6.10).
As the area of the obtained rectangle is integer, and the area of every right
triangle has a form n2, where n 2 N, then SABC  12.
Let the vertices of the parallelogram ABCD coincide with the lattice points and
the lattice point M be inside or on the boundary of the parallelogram ABCD.
As SABCD ¼ SAMB + SBMC + SCMD + SDMA and three of the numbers SAMB, SBMC,
SCMD, SDMA are not smaller than 12, then SABCD  1.5.
Remark One can prove that SABC ¼ c þ b2  1, where c is the number of the lattice
points inside of triangle ABC, аnd b is the number of the lattice points on the
boundary of triangle ABC.
6.2 Integer Lattice 277

Figure 6.10 B
y

C
C

B
A

Figure 6.11 B
M C
A

D
E

(Hint. Check that the proved statement holds true for a rectangle, with the side
passing along the lines of the grid and for right triangle, with the legs passing along
the lines of the grid).
6.2.2. It is not difficult to note that, if the vertices of the pentagon are in the lattice
points, then for some two vertices A and B, the corresponding coordinates have the
same parity. Hence, the midpoint of segment AB is a lattice point. This means that,
if the vertices of convex pentagon ABCDE are in the lattice points, then there exists
a lattice point M, inside or on the boundary of pentagon ABCDE (see Figure 6.11).
In the first case, we have that
SABCDE ¼ SAMB þ SBMC þ SCMD þ SDME þ SMEA  5  12 ¼ 2:5 (see the proof of
problem 6.2.1), and in the second case, we obtain that SABCDE  2.
Since the pentagon MBCDE (see Figure 6.11) satisfies the conditions of the problem,
then, as we have proven, SMBCDE  2. Thus, SABCDE ¼ SAME + SMBCDE  0.5 + 2 ¼ 2.5.
6.2.3. (a) Consider a parallelogram ABA0 D (Figure 6.12).
Since points A, B, and D are the lattice points, then point A0 is also a lattice point.
It is clear that points C and A0 differ (∠B 6¼ ∠D). We have that ∠BCD ¼ ∠BA0 D.
Hence, points B, C, A0 , and D are on one circle. According to problem 1.2.9a, we
have that BA0  CD ¼ CB  A0 D + CA0  BD or BC  A0 D ¼ BA0  CD + CA0  BD. Thus, |
AB  BC  CD  AD| ¼ CA0  BD  1, as CA0  1 and BD  1.
278 6 Inequalities for Radiuses

Figure 6.12 C

B
D

(b) Note that ∠CBD ¼ ∠CAD and let ∠CAD ¼ α. We have that, jAC  AD
BC  BDj ¼ sin2 α jSACD  SBCD j  2jSACD  SBCD j. If SACD ¼ SBCD, then
AB k CD. This leads to contradiction. Hence, jSACD  SBCD j  12 (see the proof
of problem 6.2.1). Therefore, |AC  AD  BC  BD|  1.
6.2.4. It is clear that δðr Þ  12, and if 12 < p2ffir. Then, this ends the proof.
 qffiffiffiffiffiffiffiffiffiffiffiffiffiffiffiffiffi 
2
Let r  16. Note that the lattice point A ½r ; r  ½r  þ 1 is outside that
2

 qffiffiffiffiffiffiffiffiffiffiffiffiffiffiffiffiffi 2 qffiffiffiffiffiffiffiffiffiffiffiffiffiffiffiffiffi2
2 2 2
circle. Indeed, ½r  þ r  ½r  þ 1 > ½r  þ
2 r 2  ½r 2 ¼ r 2 . Hence,
OA > r, where O is the origin of the coordinate system. Obviously, δ(r)  OA  r.
We need to estimate the value of OA  r. We have that,
2
r2 OA2 r2
OAþr <
OA  r ¼ OA 2r .
Note that
qffiffiffiffiffiffiffiffiffiffiffiffiffiffiffiffi qffiffiffiffiffiffiffiffiffiffiffiffiffiffiffiffi 2 qffiffiffiffiffiffiffiffiffiffiffiffiffiffiffiffi2
OA  r ¼ 1 þ 2
2 2
r 2  ½r 2 þ r 2  ½r 2  r 2  ½r 2 
qffiffiffiffiffiffiffiffiffiffiffiffiffiffiffiffi qffiffiffiffiffiffiffiffiffiffiffiffiffiffiffiffi pffiffiffiffiffiffiffiffiffiffiffiffiffiffiffiffiffiffiffiffiffiffiffi pffiffiffiffiffi pffiffi
1þ2 r 2  ½r 2  1 þ 2 r 2  ½r 2 ¼ 1 þ 2 fr gðr þ ½r Þ < 1 þ 2 2r < 1 þ 3 r :

pffi pffiffi
Hence, OA  r < 1þ3 r
. Since for r  16 we have that 1 < r , then
pffi 2r
OA  r < 1þ3 p2ffi p2ffi p2ffi
2r < r. Therefore OA  r < r, and hence δðr Þ < r.
r

6.2.5. Let the points of this figure are red. We attach the figure to the transparent
grid paper, cut the paper into the cells and put them in a pile, translating them
parallelly, but without turning, such that one cell remains in its place. The red points
cannot cover completely the cell since the area of the red parts is smaller than that of
the cell. Now, let us recall how the figure was located on the grid paper, and shift
parallelly only the grid paper such that one of its lattice points gets into a not red
point from a pile. Therefore, we obtain the required translation (location) of the
figure.
6.2 Integer Lattice 279

B «mid.vert»
B «mid.vert»
N

m

b
a h
a
A b h-1 M
A M
«mid.horiz» n¢ n «mid.horiz» n
K
m C
C «mid.vert»
a b
Figure 6.13

6.2.6. Consider three vertical lines of the integer lattice passing through the vertices
A, B, C. One of these lines is between the other two. A vertex of the triangle is called
a “mid-vertical,” if it belongs to this line. Similarly, we define the “mid-horizontal”
vertex of triangle AB C. Since AB C is an acute triangle, then none of its vertices can
be simultaneously “mid-vertical” and “mid-horizontal,”
Consider two cases (see Figure 6.13a, b).
(a) If in the case I (Figure 6.13a), h  6. Since h2 + m2  65, then it follows that m,
n  6. Hence α  45 , β  45 . Therefore, ∠A  90 .
This leads to a contradiction.
Hence, h  7, and since m + n  9, then SABC  31.5.
In case II (Figure 6.13b), one can assume that a  b. If a  7, then
SABC ¼ ab 2 þ 2 þ 2  2 þ 2 þ 2 ¼ 31:5.
an bm 77 71 71

Let a  6 and b0 is the minimal positive integer, such that a2 þ b20  65.
It is clear that a  b0  b. Let n0 be a minimal nonnegative integer, such that
b20 þ n20  65. The vertex C is on the marked part of the plane (Figure 6.14), hence
   
bðbo aÞ bðbo aþ1Þ
S  min ab 2 þ 2 þ ;
ano ab
2 2 þ 2 þ a1
2 ¼ min bbo þan
2
o bbo þaþb
; 2 .
Hence, we deduce that at a  3, bo  8 and SABC  32.  
2 ; 2 ¼
For a ¼ 4, we obtain that bo ¼ 7, no ¼ 4, thus SABC  min 65 60
30.

For a ¼ 5, we obtain that bo ¼ 7, no ¼ 4, so that SABC  min 2 ; 2 ¼ 30:5.
69 61

For a ¼ 6, b  7, we have that SABC  422 þ 2 þ 2 > 30.


72 61

For a ¼ 6, b ¼ 6, we have that SABC  362 þ 2 ¼ 30.


64

(b) In the case of Figure 6.13b, we have that a2 + b2  20002. Since a  b, then
(a + b)2 > b(a + b)  20002. Hence, a + b  2001. This means that on the broken
280 6 Inequalities for Radiuses

Figure 6.14 B

b
b0-a
A a
a

line AMB there are not less than 2000 lattice points (apart from A and B), then
the sum of the coordinates of these lattice points accepts the values S, S + 1, . . . ,
S + 1999. Hence, one of them is divisible by 2000 (for other disposition of the
broken line AMB it can be necessary to consider the difference of the
coordinates).
In the case of Figure 6.13a, either α < 45 , or β < 45 .
Let α < 45 , then h > m and h + m  2001 (see the proof in the case of
Figure 6.13b). Since m0 ¼ h1 h m > m  1 and m 2
0
= N, then inside of the triangle
there are at least h + m  2 lattice points (on the broken line AMN), for h + m  2002
the proof of the problem is straightforward. While, for h + m ¼ 2001, if m > 1, then
h  1999 and h2 + m2 < h(h + m) 1999  2001 < 20002.
This leads to contradiction.
If h + m ¼ 2001 and m ¼ 1, then h ¼ 2000, n  1999 and n0 ¼ h1 h n > 1. Thus, the
number of lattice points located on segment AM is equal to 1999. Let us consider
one more lattice point on segment MK (n0 > 1), then the difference of the coordi-
nates of one of these lattice points (their total number is 2000) is divisible by 2000.
This ends the proof.
6.2.7. Let M be the set of all lattice points covered by square S, and H be the convex
envelope of the set M. Since H is contained in S, then its area cannot exceed the area
of square S. Therefore, SH  n2.
By Pick’s theorem (see the remark to the proof of problem 6.2.1) SH ¼ c þ
b
2  1  n2 and c þ b2  n2 þ 1.
According to problem 2.1.1, the perimeter H cannot exceed the perimeter of S.
Thus, it follows that PH  4n.
It is clear that, the distance between no two lattice points can be less than 1. Hence,
the number of the lattice points on the boundary H cannot be more than 4n. Thus, we
 
obtain that b  4n. Therefore, c þ b ¼ c þ b2 þ b2  n2 þ 1 þ 2n ¼ ðn þ 1Þ2 .
6.2 Integer Lattice 281

C2 C C1

A B

D2 D D1

Figure 6.15

6.2.8. Let A and B be the vertices of the given polygon, such that the distance
between them is the greatest, and C,D be the most distant vertices of the polygon
from the straight line AB and are located at different sides of this line.
Construct parallelograms ABC1C, ABCC2, ABD1D, and ABDD2 (Figure 6.15).
We need to prove that C1C2D2D1 is the required parallelogram.
Indeed, since C1C2 ¼ 2AB ¼ D1D2 and C1C2 k AB k D1D2, then quadrilateral
C1C2D2D1 is a parallelogram. It is clear that points C1, D1, D2, C2 are integer
lattice points (as A, B, C, D are lattice points).
We have that SC1 C2 D2 D1 ¼ 4ðSABC þ SABD Þ  4S. Finally, the given polygon is
located in the union of the trapezoids ABC1C2 and ABD1D2. Otherwise, if the vertex
E of the polygon is in the strip with boundaries AB and C1C2, but not inside the
trapezoid ABC1C2 (Figure 6.15), then SACE > SABC and ρ(E, AB) > ρ(C, AB).
This leads to contradiction.
pffiffiffi
6.2.9. (a) Note that there exist such natural numbers m and n, that 3n  m < 1000
1
.
Indeed, (2;1) is a solution of the equation

x2  3y2 ¼ 1: ð6:7Þ

If (x0, y0) is a solution of this equation, then (2x0 + 3y0; x0 + 2y0) is also a solution.
Consequently, if the Eq. 6.7 has a solution (m, n), where m, n 2 N m, n > 1000, then
pffiffiffi  pffiffiffi 
3n  m ¼ pffiffi 1 < 1 < 1 and triangle with vertices (0; 0); (2n; 0); n; 3n
3nþm m 1000
satisfies the conditions of the problem.
(b) Proof by contradiction argument. Let A, B, C be the centers of three circles
containing the vertices of equilateral triangle with the side length no greater than
100.
 
Then, AB2  BC2 ¼ jAB  BCj  jAB þ BCj  1000 4
 2  100 þ 1000
4
< 1 (see
problem 1.2.1). Therefore, AB ¼ BC. Similarly, we obtain that AB ¼ AC. This leads
pffiffi
to contradiction, as SABC ¼ 43 AB2 must be rational (see the proof of problem 6.2.1).
6.2.10. From the condition of the problem, it follows that, one can assume that the
sides of the polygon are on the lines of the integer lattice, and vertices are in the
282 6 Inequalities for Radiuses

lattice points. It is known that S ¼ c þ b2  1, where c is the number of the lattice


points inside the polygon and b the number of the lattice points on its boundaries.
Since, c  0, b ¼ 4n, then S  2n  1.
Let us consider the smallest rectangle containing the polygon with area S, the
sides (passing) on the lines of the integer lattice . Let k and m be the sides of the
constructed rectangle. It is clear that 2k + 2m  4n, consequently,
 2
S  mk  mþk 2  n2 . Thus, it follows that S  n2. This ends the proof.
6.2.11. We need to prove that the maximal number of the lattice points that can be
on the boundaries of the triangle (vertices included) is equal to 9 for n ¼ 1 and to 2n
+ 6, for n  2 (see Figure 6.16)
Let A, B, C be the lattice points and the number of the lattice points inside
triangle ABC is equal to n, while those on sides AB, BC, and CA (and different from
the vertices of triangle ABC ) are k, l, and p, respectively, where k  l  p.
~ ¼ ZT,
Note that if X, Y, Z are the lattice points, then point T, where XY ~ is also a
lattice point.
Let l  1 and the lattice points A1, A2, . . . , Al are on side BC, then from the
aforesaid it follows that BA1 ¼ A1A2 ¼ . . . ¼ Al  1Al ¼ AlC. Similarly, if points B1,
B2, . . . , Bp are on side CA, then CB1 ¼ B1B2 ¼ . . . ¼ Bp  1Bp ¼ BpA (B0 C). Let
CADB be a parallelogram, then we deduce that (see Figure 6.17)

A
B

1 2…n
B C A C
n=1 n³2
Figure 6.16

Figure 6.17 D

A1

A2

A
Bp
Al

B2 B
1 C
6.2 Integer Lattice 283

Figure 6.18 B

E F

U M V

A C

lp  2n þ k: ð6:8Þ

For k ¼ 0, we have that k + l + p ¼ l + p ¼ lp + 1  (l  1)( p  1)  lp + 1  2n + 1.


Hence, k + l + p + 3 < 2n + 6.
For k  1, we have that (l  k)( p  k)  0, consequently

lp þ k2 2n
kþlþpþ3 þkþ3 þ 2k þ 4: ð6:9Þ
k k

If n ¼ 1, then from (6.8), we obtain that k2  lp  2 + k. Therefore, k ¼ 1 or k ¼ 2,


then from (6.9), we deduce that l + p + k + 3  9.
If n > 1, then from (6.8) we obtain that k(k  1)  2n. Thus k  n, we deduce that
l þ p þ k þ 3  2n k þ 2k þ 4  2n þ 6.
Case l ¼ 0 remains to be considered. In this case, we have that k ¼ 0. Let EF be
the midline of triangle ABC (see Figure 6.18), and point M is the closest lattice point
to side AC.
Then, point M belongs to the trapezoid AEFC. Otherwise, point B0 , where B0 2 BM
and BM ¼ MB0 is a lattice point and is closer to AC than point M. Let UV k AC and
CB1 ¼ a, then the number of the lattice points on segment UV is not greater than n, and
UV < (n + 1)a. Thus, it follows that AC ¼ 2EF  2UV < (2n + 2)a. On the other hand,
AC ¼ ( p + 1)a, this means that p < 2n + 1. Therefore, l + p + k + 3 ¼ p + 3 < 2n + 6.

Problems for Self-Study

6.2.12. What is the least perimeter of a convex 32-gon, such that its all vertices are
in the knots of a unit grid paper?
6.2.13. A nonempty set of the lattice points is marked on the integer lattice. Given
also the final set of nonzero vectors with the integer coordinates. Given that, if one
takes the origin of all given vectors at any of the marked lattice points, then among
their endpoints there are more marked lattice points than unmarked ones. Prove that
the number of the marked lattice points is infinite.
284 6 Inequalities for Radiuses

6.2.14. Prove that, if points A, B, C coincide with the knots of the integer lattice and
 pffiffiffi 2  pffiffiffi 2
AC  2AB þ AC  2BC > 0, then (AB + BC)2  4AB  BC sin ∠ABC  1.
Hint For points A(x; y), B(0; 0) and C(u; v), we have that AB  BC sin ∠ABC ¼
|uy  vx|.
6.2.15. Vertices of the convex 2n-gon are in the knots of a unit integer lattice. Prove
that the area of 2n-gon is not smaller than (a) nðn1Þ n3
2 , (b) 100.

Hint Firstly, prove the statement of the problem for centrally symmetric polygons.
6.2.16. A triangle is placed on a coordinate plane, so that its translations by vectors
with integer coordinates do not overlap. Find the greatest possible area of such
triangle.
Hint See problem 3.1.38.
6.2.17. The number of the lattice points inside of the convex figure with area S and
semiperimeter p is n. Prove that n > S  p.
Chapter 7
Miscellaneous Inequalities

In this chapter we consider problems that can be proved either by the methods
described in the previous chapters or by some other methods introduced in this
chapter. For example, complex numbers, the method of coordinates and application
of geometric transformations are used in order to prove some inequalities.
In Section 7.1, we consider some problems related to combinatorial geometry
and provide many inequalities related to the parts of a triangle. Perhaps, the most
important advice to deal with such problems is the following one: in order to prove
inequalities related to the parts of a triangle, in some cases one needs to make a
substitution a ¼ m þ n, b ¼ m þ k, c ¼ n þ k, where m > 0, n > 0, k > 0.
Some problems related to the parts of a triangle can be proved considering the
triangle made of the medians of the given triangle. Every inequality related to this
new triangle holds true also for the given triangle.
In this section we have selected the most useful inequalities of different themes
related to geometric inequalities. This is the reason why Section 7.1 is much longer
than the other sections.
In order to prove any new inequality that is not included in Section 7.1, one can
find a similar inequality in Section 7.1 and use its method of proof.
In this chapter many inequalities related to different spaces are included, such
that in their proofs the following properties of a polyhedral angle are used:
1. The sum of all planar angles of a convex polyhedral angle is less than 360 .
2. The sum of all planar angles of a convex polyhedral angle is greater than its any
planar angle multiplied by 2.
Some problems in this chapter were inspired by [1–6, 9, 11–16]. Nevertheless,
even for these problems the authors have mostly provided their own solutions.

© Springer International Publishing AG 2017 285


H. Sedrakyan, N. Sedrakyan, Geometric Inequalities, Problem Books
in Mathematics, DOI 10.1007/978-3-319-55080-0_7
286 7 Miscellaneous Inequalities

7.1 Miscellaneous Inequalities

7.1.1. Given a point M inside of triangle ABC. Straight lines AM, BM and CM
intersect the sides of triangle ABC at points A1, B1 and C1, respectively. Prove that
(a) MA1 þ MB1 þ MC1  max (AA1, BB1, CC1),
(b) MA1 þ MB1 þ MC1 < MA þ MB þ MC,
(c) MA21 þ MB21 þ MC21 < MA2 þ MB2 þ MC2 ,
(d)AAM
1M
þ BBM1M
þ CCM 1M
 6,
(e) A1 M  B1 M  C1 M  8,
AM BM CM

(f) AAM  BM þ BBM


1 M B1 M
 CM þ CCM
1 M C1 M
AM
1 M A1 M
 12
(g) MA  MA1 þ MB  MB1 þ MC  MC1  2MB1  MA1 þ 2MC1  MB1 þ 2MA1  MC1,
(h) AAM1M
þ BBM1M
þ CCM 1M
 32,
pffiffiffiffiffiffiffiffiffiffiffiffiffiffiffiffiffiffiffiffiffiffi pffiffiffiffiffiffiffiffiffiffiffiffiffiffiffiffiffiffiffiffiffiffi pffiffiffiffiffiffiffiffiffiffiffiffiffiffiffiffiffiffiffiffiffiffi
(i) MA þ MB þ MC  2 MA1  MB1 þ MB1  MC1 þ MC1  MA1 ,
pffiffiffiffiffiffiffiffiffiffiffiffiffiffiffiffi pffiffiffiffiffiffiffiffiffiffiffiffiffiffiffiffi pffiffiffiffiffiffiffiffiffiffiffiffiffiffiffiffiffiffi
(j) MAþMBþMCþ2ðMA1þMB1þMC1 Þ2 MAMBþ MBMC þ MC  MA ,
qffiffiffiffiffiffiffiffiffiffiffiffiffiffiffiffiffiffiffiffiffiffiffiffiffiffiffi
(k) MA2 þMB2 þMC2 2ðMA1 MB1 þMB1 MC1 þMC1 MA1 Þþ6 3 MA21 MB21 MC21 ,
(l) MA  MB þ MB MC þ MC  MA  MA1  MB1 þ MB1  MC1 þ MC1  MA1 þ
qffiffiffiffiffiffiffiffiffiffiffiffiffiffiffiffiffiffiffiffiffiffiffiffiffiffiffi
9 3 MA21 MB21 MC21
(m) 2ða1 MA þ b1 MB þ c1 MCÞ2  a2 ðb21 þ c21  a21 Þ þ b2 ða21 þ c21  b21 Þþ c2 ða21 þ
b21  c21 Þ þ 16SS1 , where SABC ¼ S and S1 is the area of the triangle with
vertices A1, B1, C1 and sides a1, b1, c1.
7.1.2. (a) Prove that the inradius r of the right triangle is less than one-fourth of the
hypotenuse.
(b) Prove that the inradius r of an isosceles triangle is less than one-third of the
lateral side.
7.1.3. Prove that for non-obtuse triangle
min(ha, hb, hc)  r þ R  max (ha, hb, hc).
7.1.4. Given a triangle ABC and bisectors AA1, BB1, CC1 of the internal angles,
prove that
(a) A1 B1 þ B1 C1 þ A1 C1  12 ðAB þ BC þ CAÞ,
(b) A1 B1 2 þ B1 C1 2 þ A1 C1 2  12
1
ðAB þ BC þ CAÞ2 .
7.1.5. Let D, E, F be points on sides BC, CA, AB of triangle ABC, respectively and
points P, Q, R are (the second) intersection points of AD, BE, CF, respectively, with
the circumcircle of triangle ABC. Prove that ADPD þ QE þ RF  9 and find when does
BE CF

the equality hold true.


7.1.6. Given that the side lengths of the convex hexagon ABCDEF satisfy the
conditions AB ¼ BC, CD ¼ DE, EF ¼ FA, prove that BC
BE þ DA þ FC  2.
DE FA 3
7.1 Miscellaneous Inequalities 287

7.1.7. Let a, b and c, d, respectively be the sides of two triangles such that
a < c  d < b. Prove that the first rectangle can be placed inside the second one, if
and only if (b2  a2)2  (bd  ac)2 þ (bc  ad)2.
7.1.8. For triangle ABC, prove that w 2 2
uþv a b þ vþw
u
b2 c2 þ uþw
v
a2 c2  8S2 , where
u, v, w > 0.
7.1.9. Given two similarly oriented triangles ABC and O on a plane, such that
∠A ¼ ∠A1, ∠B ¼ ∠B1, prove that AA1  BC  BB1  CA þ CC1  AB.
7.1.10. Place in a cube a circle of the greatest possible radius.
7.1.11. (a) Given three points inside of a unit cube, prove that one can
find two
pffiffiffi points among these points, such that the distance between them is less
than 2.
(b) Given eight points inside of a unit cube, prove that one can find two points among
these points, such that the distance between them is less than 1.
(c) Is it possible to place seven points inside of a cube with an edge 2000, so that the
distance between any two of them is not greater than 2001?
7.1.12. Let us fix the position of unit square S on a plane. What is the maximal
number of non-intersecting pairs of unit squares that can be arranged in a plane, so
that they touch the given square, but do not intersect with it? (Two squares are
considered intersecting, if their common part is a polygon).
7.1.13. Prove that for an acute-angled triangle, ma mb mc > 58 abc holds true.
7.1.14. Given a point M inside of triangle ABC and A1, B1, C1 as the midpoints of
sides BC, AC, AB respectively, prove that
(a) max(MA, MB, MC)  2 min (MA1, MB1, MC1),
(b) min(MA, MB, MC)  2 max (MA1, MB1, MC1),
(c) min(∠A, ∠B, ∠C) < ∠MAB þ ∠MBC þ ∠MCA < π  min (∠A, ∠B, ∠C),
(d) for a point M inside of the rectangle ABCD, ∠MAB þ ∠MBC þ ∠MCD þ ∠MDA
> minð∠BAC; ∠DACÞ þ π2.
7.1.15. Given a point M inside of triangle ABC and A1, B1, C1 as the feet of
the altitudes of ABC,
pffiffiprove
ffi that MA2 þ MB2 þ MC2  MA21 þ MB21 þ MC21 , if
maxðAB; BC; ACÞ  2minðAB; BC; ACÞ.
7.1.16. Given a point M inside of triangle ABC and A1, B1, C1 as the feet
of the perpendiculars drawn from point M to lines BC, AC, AB respectively,
prove that
pffiffiffi
(a) MA þ MB þ MC < 2 2ðMA1 þ MB1 þ MC1 Þ, if ∠A, ∠B,  ∠C  45 ,
0

(b) MA þ MB þ MC < max sin1 α þ sin1 β; sin1 β þ sin1 γ; sin1 γ þ sin1 α ðMA1 þMB1 þ MC1 Þ,
where α, β and γ are the angles of triangle ABC.
288 7 Miscellaneous Inequalities

7.1.17. Prove that the sum of the distances of the points inside of the tetrahedron
from its vertices is less than the perimeter of the tetrahedron.
7.1.18. (a) Prove that, among 21 distances between the pairs of seven different
points on a plane, no number can occur more than 12 times.
(b) What is the maximal number of occurrence of the same number among
15 distances of the pairs of six different points on a plane?
7.1.19. Given that the sides of the articulated quadrilateral ABCD are equal (con-
sequently) to a, b, c, d, what is the greatest possible value of the sum of the midlines
of such quadrilateral? (The midline is a segment connecting the midpoints of the
opposite sides).
7.1.20. (a) Let ABCD be a convex quadrilateral, such that its sides and diagonals do
not exceed 1. Prove that the perimeter of the quadrilateral ABCD does not exceed
π
2 þ 4 sin 12.
(b) Let points A, B, C and D be in a space such that no more than one of the distances
AB, AC, AD, BC, BD, CD is greater than 1. Prove that the sum of these six
pffiffiffi
distances is smaller than or equal to 5 þ 3.
(c) Given that the lengths of five edges of a tetrahedron are smaller than or equal to
1, prove that its volume is smaller than or equal to 18.
7.1.21. Given positive numbers m1, m2, . . ., mn and points A1, A2, . . ., An (n  2) on
P
n
a plane. For any line l on the plane let us define ρðlÞ ¼ mi ρðAi ; lÞ. Let M be the
i¼1
set of lines AiAj (i 6¼ j). Prove that if line l does not belong to M, then
min
0
ρðl0 Þ < ρðlÞ.
l 2M

7.1.22. Let the quadrilateral ABCD be circumscribed around a circle and M, N be


the intersection points of lines AB and CD, AC and BD, respectively. Prove that, if
 
max(AB, BC, CD, AD) ¼ AD, then 90  ∠AND  90 þ 12 ∠AMD.
7.1.23. Given a tetrahedron ABCD inside of a unit cube, prove that AB  CD  d < 2,
where d is the distance between the straight lines AB and CD.
7.1.24. Given a tetrahedron ABCD inside of a parallelepiped, prove that the volume
of ABCD is less than V3 , where V is the volume of the parallelepiped.
7.1.25. Let the straight line intersect sides AB and BC of triangle ABC at points
M and K respectively. Given that the area of triangle MBK is equal to the area of the
MBþBK
quadrilateral AMKC, prove that AMþCAþKC > 13.
7.1.26. Let the triangular section of a cube touch the insphere of the cube. Prove that
the area of this section is less than the half of the area of the face of the cube.
7.1.27. (a) Prove that if the length of each of the bisectors of the triangle is greater
than 1, then its area is greater than p1ffiffi3.
7.1 Miscellaneous Inequalities 289

(b) Prove that if the length of each of the bisectors of the triangle is less than 1, then
its area is less than p1ffiffi3.
(c) Let points A1, B1, C1 be, respectively, on sides BC, AC and AB of the triangle
ABC. Given that AA1  1, BB1  1, CC1  1, prove that the area of triangle ABC
is smaller than or equal to p1ffiffi3.

7.1.28. Find the greatest value of the number k, such that the inequality a2 þ b2 þ c2
> k(a þ b þ c)2 holds true for any obtuse triangle.
7.1.29. For triangle ABC prove the following inequalities:
(a) a2 þ b2 þ c2 þ 4abc < 12, if a þ b þ c ¼ 1,
(b) a(2a2  b2  c2) þ b(2b2  c2  a2) þ c(2c2  a2  b2)  0,
(c) a2b(a  b) þ b2c(b  c) þ c2a(c  a)  0,
pffiffiffi pffiffiffi pffiffiffi qffiffiffiffiffiffiffiffiffiffiffiffiffiffiffiffiffiffiffiffiffiffiffiffiffiffiffiffiffiffiffiffiffiffiffiffiffiffiffiffiffiffiffiffiffiffiffiffi
 
(d) aða þ c  bÞ þ bða þ b  cÞ þ cðb þ c  aÞ  ða þ b þ cÞ a2 þ b2 þ c2 ,
pffiffiffiffiffiffiffiffiffiffiffiffiffiffiffiffiffiffiffi pffiffiffiffiffiffiffiffiffiffiffiffiffiffiffiffiffiffiffi pffiffiffiffiffiffiffiffiffiffiffiffiffiffiffiffiffiffiffi pffiffiffi pffiffiffi pffiffiffi
(e) a þ b  c þ b þ c  a þ a þ c  b  a þ b þ c,
(f) a2 þ b2 þ c2 þ 2abc < 2, where a þ b þ c ¼ 2,
(g) 2 ab þ bc þ ac  ac þ bc þ ba þ 3,
 
(h) ab þ bc þ ac  ac  bc  ba < 1,
(i) a2pq þ b2qr þ c2pr  0, where p þ q þ r ¼ 0,
(j) a2 ð2b þ 2c  aÞ þ b2 ð2c þ 2a  bÞ þ c2 ð2a þ 2b  cÞ  9abc,
(k) abþbcþca
ðaþbþcÞ2
> 14,
ðaþbÞðbþcÞðaþcÞ
(l) ðaþbþcÞ3
> 14,
(m) a þ b þ c < (a þ b þ c)(ab þ bc þ ac),
3 3 3

(n) 32  bþc
a
þ cþa
b
þ aþbc
< 2,
pffiffiffiffiffiffiffiffiffiffiffiffiffiffiffiffiffiffiffiffiffiffiffiffi
ffi pffiffiffiffiffiffiffiffiffiffiffiffiffiffiffiffiffiffiffiffiffiffiffiffiffi pffiffiffiffiffiffiffiffiffiffiffiffiffiffiffiffiffiffiffiffiffiffiffiffiffi
(o) a þ b þ c  b þ c2  a2 þ a2 þ c2  b2 þ a2 þ b2  c2 , where ABC
2

is not an obtuse triangle,


qffiffiffiffiffiffiffiffiffiffiffi qffiffiffiffiffiffiffiffiffiffiffi qffiffiffiffiffiffiffiffiffiffiffi
(p) aþbcabc
þ abþcabc
þ abc
 a þ b þ c,
ab bc ca  1 bþca
(q)  þ þ  < .
aþb bþc cþa 8

7.1.30. Let A, B, C be given points on a plane. Prove that for any point X of that
plane, m(ABC)  m(ABX) þ m(AXC) þ m(XBC), where for points P, Q, R on a plane
we denote by m(PQR) the smallest of the lengths of the altitudes of triangle PQR
(if points P, Q, R belong to the same line, then m(PQR) ¼ 0).
7.1.31. Given a triangle ABC and its incenter I and A0 , B0 , C0 be the intersection
points of the bisectors of angles CAB, ABC, BCA with sides BC, CA,
ABrespectively, prove that 14 < AA
AI
0 
BB0  CC0  27.
BI CI 8

7.1.32. Given a plane π, a point P on that plane and a point Q outside plane π, find
all points R in plane π, so that the ratio QPþPR
QR has the greatest value.

7.1.33. Given on a plane a finite number of strips with the sum of widths equal to
11, and a circle with a radius 1, prove that each of these strips can be displaced
parallelly to itself so that together they cover the circle.
290 7 Miscellaneous Inequalities

ffiffi the right triangle ABC can be covered by two unit circles, prove
7.1.34. Givenpthat
that SABC  3 2 3.
7.1.35. Prove that
(a) in a trihedral angle, each plane angle is less than the sum of two other flat
angles,
(b) in a convex polyhedral angle, the sum of all face angles is less than 360 ,
(c) in a polyhedral angle, each face angle is less than the sum of all other face
angles,
(d) ∠AMB þ ∠BMC þ ∠CMD > 180 , if point M is inside of the tetrahedron
ABCD,
(e) ∠AMB þ ∠BMC þ ∠AMC þ ∠AMD þ ∠BMD þ ∠CMD > 3π,
(f) the sum of all dihedral angles of the trihedral angle is greater than π,
(g) the sum of all dihedral angles of a tetrahedron is between 2π and 3π,
4 < ∠A1 AC1 þ ∠BAC1 þ ∠DAC1 < π, if ABCDA1B1C1D1 is a rectangular
(h) 3π
parallelepiped.
7.1.36. (а) Prove that if the cube with an edge b does not have points outside the
cube with an edge a and does not contain its center of symmetry, then b < a2.
(b) Given a rectangular coordinate system in a space and a cube with an edge a.
Given also that all coordinates of any point belonging to this cube are
non-negative, prove that, there is a point belonging to the cube, such that all
its coordinates are not less than a,
(c) Cubes with edges a and b do not have common points and are inside of the cube
with an edge c. Prove that a þ b < c.
7.1.37. For triangle ABC, prove the following inequalities:

R  a þ b þ c,
p ha hb hc
(a)
a þ b þ c  2r,
ma mb mc p
(b)
R  a þ b þ c 
p ra rb rc p
(c) ,
pffiffi2r
a þ b þ c  2p,ffiffiffi
ma mb mc 3 3
(d)
ma þ mb þ mc  2 3,
a b c
(e)
ha þ hb þ hc  1 þ r ,
ma mb mc R
(f)
4Rþr3
(g) ma mb mc  3 ,
(h) l2a þ l2b þ l2c  p2 ,
pffiffiffi
(i) m2a þ m2b þ m2c  ðma  mb Þ2  ðmb  mc Þ2  ðma  mc Þ2  3 3S,
haþ hb þ hc 3,
ra rb rc
(j)
 
(k) 3 raa þ rbb þ rcc  4 raa þ rbb þ rcc ,
2
(l) r 2a þ r 2b þ r 2c  27R
4 ,
(m) ha þ hb þ hc  9r,
(n) m2a þ m2b þ m2c  27 2
4 R ,
7.1 Miscellaneous Inequalities 291

(o) m1a þ m1b þ m1c  R2 ,


(p) r a þ r b þ r c  412 R,
pffiffiffi 
(q) l1a þ l1b þ l1c < 2 1a þ 1b þ 1c , if ABC is an acute triangle,
(r) la  hc, if a  b  c,
bc
(s) 2R  la  2pbcffiffiffiffiffi
ffi,
2Rr
(t) ma þ mb þ mc  a þb2Rþc ,
2 2 2

(u) mmb ma c þ mma mb c þ mma mc b  1r ,


2 2 2
(v) 2r  ða  bÞ þ ðb  cÞ þ ðc  aÞ ,
p2 R2r
(w) mamb þ mbmc þ mcma  lara þ lbrb þ lcrc,
(x) la þ lb < 43 ða þ bÞ,
(y) 4acmamc  16S2 þ (b2  a2)(c2  b2),
pffiffi
ma mb þ mb mc þ mc ma  S ,
1 1 1 3
(z)
ma b þ mb a  3S,
(aa) 1 1 p2ffiffi

(bb) S  a2abm a mb
2 þb2 þc2 ,

(cc) mlaa þ mlbb þ mlcc > 1,


(dd) ha þ hb þ hc  2R þ 5r,
(ee) ha þ hb þ hc > 2R þ 4r, if ABC is an acute triangle,
pffiffiffiffiffiffiffiffiffiffiffiffiffiffiffiffiffiffiffiffiffiffiffiffiffiffi
(ff) la þ lb þ lc  32 ab þ bc þ ac.
7.1.38. For any tetrahedron ABCD and any point M inside of it, prove that
(a) MA  MB  MC  MD  81MA1  MB1  MC1  MD1, where A1, B1, C1, D1 are the
intersection points of the straight lines AM, BM, CM, DM with the opposite
faces of the tetrahedron ABCD.
(b) at least one of segments AM, BM, CM is less than the corresponding segment
AD, BD, CD,
(c) DM < max (AD, BD, CD),
(d) AM þ BM þ CM þ DM < AB þ BC þ CD þ AD þ BD þ AC,
(e) AM þ BM þ CMþ DM << max (AB þ AC þ AD, BA þ BC þ BD,
CA þ CB þ CD, DA þ DB þ DC).
7.1.39. Given points A, B, A0 , B0 on a line l, and a point P outside that line. Given
also that ∠APB ¼ ∠A0P B0 and ∠PA0 B0 ¼ ∠PB0 A0 , prove that
(a) AB  A0 B0,
(b) |PA  PB|  AB  A0 B0.
7.1.40. Given a triangle ABC, such that AB > AC. The bisector of angle ∠A
intersects side BC at point D. Let P be a point on segment AD, and Q,R the
intersection points of BP and CP with sides AC and AB, respectively. Prove that,
PB  PC > RB  QC > 0.
7.1.41. Let P be any point inside of the equilateral triangle ABC. Prove that
(a) |∠PAB  ∠PAC|  |∠PBC  ∠PCB|,
292 7 Miscellaneous Inequalities


j∠PAB  ∠PACj j∠PAB  ∠PACj
j∠PAB  ∠PACj  arcsin 2 sin  
(b) 2 2
 j∠PBC  ∠PCBj:
0
7.1.42. Consider two triangles ABC and A0 B0 C0 , such that ∠A  90 and ∠A  90 .
Let ha and h0a be the altitudes drawn to sides a and a0 . Prove that, 1 0  1 0 þ cc1 0 ,
ha ha bb
where a, b, c and a0 , b0 , c0 are the sides of triangles ABC and A0 B0 C0 , respectively.
7.1.43. Given a square ABCD and P and Q are points on segments BC and CD
respectively. Let E and F be the intersection points of line PQ with the extension of
segments AB and AD respectively. Prove that π  ∠PAQ þ ∠ECF < 5π 4.

7.1.44. Let the straight line l be perpendicular to plane P. Three spheres touch each
other, so that each of them touches plane P and the straight line l.
Given that the radius of the largest sphere
pffiffiffi is equal to 1, the radius of the smallest
sphere is equal to r, prove that r  3  2 2.
7.1.45. Given an equilateral triangle DAC and a point B on ray CD such that
∠BAC ¼ 70 . Let point E be on segment AB, so that ∠ECA ¼ 55 and point K be
the midpoint of segment ED. Without using a computer, calculator or protractor,
show that 60 > ∠AKC > 57, 5 .
7.1.46. (a) Let M be the centroid of triangle ABC. Let AB touch the circumcircle of
triangle AMC. Prove that sin ∠CAM þ sin ∠CBM  p2ffiffi3.

(b) Let M be the centroid of triangle ABC. Prove that sin ∠CAMþ sin ∠CBM  p2ffiffi3.

7.1.47. Given on a plane points M, N and triangle ABC, prove that


AMAN
ABAC þ BMBN
BABC þ CACB  1.
CMCN

7.1.48. Given distinct points A1 , A2 , :::, An on a plane and arbitrary points B1,
B2, . . . , Bn  1 (n  2), prove that
B1 A1 B2 A1 :::Bn1 A1
A2 A1 A3 A1 :::An A1 þ BA1 A1 A2 B 2 A2 :::Bn1 A2
2 A3 A2 :::An A2
þ ::: þ BA11 AAnn B 2 An :::Bn1 An
A2 An :::An1 An  1.

7.1.49. Given distinct points A1 , A2 , :::, An on a plane and arbitrary points B1, . . . ,
Bn  2 (n  3), prove that BA1 A2 A1 :::B n2 A1
1 :::An A1
þ :::þ BA1 A1 An1 B2 An1 :::Bn2 An1
n1 A2 An1 :::An An1
 BA11 AAnn B 2 An :::Bn2 An
A2 An :::An1 An .

7.1.50. Let M be an arbitrary point inside of triangle ABC with the semiperimeter p.
Prove that, AM sin ∠BMC þ BM sin ∠AMC þ CM sin ∠AMB  p and that the
equality holds true, if and only if M coincides with its incenter.
7.1.51. Let equilateral triangles MNP and M1 N 1 P1 be inscribed in triangle ABC
such that M, M1 2 BC, N, N1 2 AB, P, P1 2 AC, (M ≢ M1 , N ≢ N 1 , P ≢ P1). Prove that
one can construct a triangle with segments MM1, NN1 and PP1.
7.1.52. Given a unit square ABCD and points P and Q on its sides BC and CD
respectively so that ∠PAQ  45 . Let E, F be the intersection points
pffiffiof
ffi the straight
line PQ with lines AB, AD respectively. Prove that, AE þ AF  2 2.
7.1.53. Let a convex polygon A1A2 . . . An be inscribed in a circle with the radius R.
Let A be a point on that circle, distinct from the vertices of the polygon. Denote by
7.1 Miscellaneous Inequalities 293

ai ¼ AAi and let bi be the distance from point A to line Ai Aiþ1 , i ¼ 1, . . . , n, where
a2 a2 a2
An þ 1  A1. Prove that, b11 þ b22 þ ::: þ bnn  2nR.

7.1.54. (a) Let points M and N be chosen on side AC of triangle Q0 such that
∠ABM ¼ ∠MBN ¼ ∠NBC. Prove that AN  CM < 4  AM  NC.
(b) Let points M and N be chosen on sides AB and AC of triangle ABC respectively.
Prove that BN þ MN þ CM  2BC, if max(∠A, ∠B, ∠C)  120 and
∠A  ∠B, ∠C.
7.1.55. Let O be the intersection point of the diagonals of the convex quadrilateral
ABCD. Given that AB ¼ a, BC ¼ b, CD ¼ c, DA ¼ d, and ra, rb, rc, rd are the
inradiuses of triangles AOB, BOC, COD, DOA respectively, prove that
(a) a
þ Scc > Sbb þ Sdd and r1a þ r1c > r1b þ r1d , if a þ c > b þ d, where Sa ¼ SAOB, Sb ¼
Sa
SBOC, Sc ¼ SCOD, Sd ¼ SDOA,
(b) ∠IdIaIb þ ∠IdIcIb > π, if a þ c > b þ d, where Ia, Ib, Ic, Id are the incenters of
triangles AOB, BOC, COD, DOA, respectively.

7.1.56. Given a convex polygon A1A2 . . . An and points B1, B2, . . . , Bn on its sides
B1 2 A1 A2 ; B2 2 A2 A3 ; :::; Bn 2 An A1 ; Bi ≢ Aj . Given also points C1 , C2 , :::, Cn on
segments A1B1, A2B2, . . . , AnBn respectively so that ∠BnB1B2 ¼ ∠CnC1C2 ¼ β1,
∠B1B2B3 ¼ ∠C1C2C3 ¼ β2, . . ., ∠Bn  1BnB1 ¼ ∠Cn  1CnC1 ¼ βn, prove that
B1C1 sin β1 þ B2C2 sin β2 þ . . . þ Bn  1Cn  1 sin βn  1 > BnCn sin βn.
7.1.57. Let the real number λ >  1. Let a, b, c be the sides of a triangle. Prove that
ðpþλaÞðpþλbÞðpþλcÞ
ðpaÞðpbÞðpcÞ ð2λ þ 3Þ3 , where p ¼ aþbþc
2 .

7.1.58. Let a, b, c be the sides of a triangle ABC, and P be any point inside of
it. Let also AP intersect the circumcircle of triangle BPC (for the second time) at
point A0 . Points B0 and C0 are pffiffiffiffiffi defined
pffiffiffiffiffi similarly. Prove that
pffiffiffiffiffi
AB0 þ B0 C þ CA0 þ A0 B þ BC0 þ C0 A  2 ab þ 2 bc þ 2 ac.
7.1.59. Let O be the incenter of a triangle ABC and D, E be the midpoints of
segments AB, AC, respectively. Let K, L be the intersection points of segments BO
and CO with DE respectively. Prove that AO þ BO þ CO > BC þ KL.
7.1.60. For a non-obtuse triangle, prove that
h2a h2 h2
(a) þ b2 þ c2c  94,
pffiffiffi b
a2
(b) 3maxðha ; hb ; hc Þ  p.
7.1.61. Let M be the midpoint of segment AB of triangle ABC and line l contains
the bisector ofpangle ACB. Prove that the distance from point M to line l does not
ffiffiffiffiffiffiffiffiffiffiffiffiffiffiffiffiffi
exceed aþb
2  p ð p  cÞ, where p ¼ aþbþc
2 .
pffiffiffi pffiffiffiffiffiffiffiffiffiffiffiffiffiffiffiffiffi pffiffiffiffiffiffiffiffiffiffiffiffiffiffiffiffiffi
7.1.62. Given a triangle, prove that mc  3p  pðp  aÞ  pðp  bÞ.
7.1.63. Let the bisectors of the internal angles A and C of triangle ABC intersect
with the median BM at points E and F respectively. Prove that AE < CF if BC > AB.
294 7 Miscellaneous Inequalities

7.1.64. Let E and F be the intersection points of the altitudes of the internal
angles A and C of triangle ABC with the median BM, respectively. Prove that
AE < CF if BC > AB.
7.1.65. Consider a triangle ABC, such that BC > AB. Let points E and F be chosen
on the median BM such that ∠BAE ¼ ∠BCF. Prove that
(a) AE < CF,
(b) BE < BF, if points E and B are different.
7.1.66. Given a square ABCD. Let E be a point on ray AB before point B and F be
a point on ray AD before point D such that EF ¼ 2AB. Let P and Q be the
intersection points of EF with BC and R respectively. Prove that
(a) the APQ is an acute triangle,
(b) ∠PAQ  450.
7.1.67. Let the circumradius of triangle ABC be equal to 1, O be its center, P be a
point inside of the triangle such that OP ¼ x. Prove that
AP  BP  CP  (1 þ x)2(1  x) and that equality holds true only if P  O.
7.1.68. Let A, B, C be points on a circle. Let P, Q, R be the midpoints of the arcs BC,
CA, AB respectively. Given that segments AP, BQ, CR intersect sides BC, CA, AB at
points L, M, N respectively, prove that ALPL þ QM þ RN  9.
BM CN

For which triangle ABC does the equality hold true?


7.1.69. (a) Let A1A2 . . . An be a regular n-gon, inscribed in a unit circle with the
Pn
center O. Given a point M on ray OA1 outside of the n-gon, prove that MA 1
i
> MO
n
.
i¼1

(b) Let A1A2 . . . An be a regular polygon and let M be a point inside of it. Prove that

2π π
sin ∠A1 MA2 þ sin ∠A2 MA3 þ ::: þ sin ∠An MA1 > sin þ ðn  2Þ sin :
n n

7.1.70. Let point M be chosen inside of angle BAC and the straight line l drawn
through it be such that the foot of the perpendicular drawn from point A to line l is
symmetric to point M with respect to the midpoint of segment UV, where UV is
segment of the line cut by the sides of the angle. Let the segment PQ pass through
point M, where points P and Q are on the sides of angle BAC. Prove that PQ  UV.
 3
7.1.71. (a) Given a point M inside of triangle ABC, prove that αβγ  π6 , where
α ¼ ∠MAB, β ¼ ∠MBC, γ ¼ ∠MCA.
(b) Given a point M inside of the convex polygon A1 A2 :::An , let αi ¼ ∠MAiAi þ 1,
 n
i ¼ 1, 2, . . . , n, An þ 1  A1. Does the inequality α1 α2  :::  αn  πðn2
2n
Þ
hold
true for n  4?
(c) Given a point M inside of the convex polygon A1 A2 :::An , let αi ¼ ∠MAiAi þ 1,
i ¼ 1, 2, . . . , n, An þ 1  A1. Prove that minðα1 ; α2 ; :::; αn Þ  π ðn2
2n .
Þ
7.1 Miscellaneous Inequalities 295

7.1.72. Let 2n-gon A1A2 . . . A2n be inscribed in a unit circle with the center O.
Prove that


!  1
 A1 A þ A 3 A4 þ ::: þ A2n1 A2n  2 sin 2 ∠A1 OA2 þ ∠A3 OA4 þ ::: þ ∠A2n
2
1OA2n Þ.
7.1.73. Let O be the intersection point of the diagonals of the inscribed quadrilateral
ABCD. Prove that CDAB
þ CD
AB þ AD þ BC  OC þ OA þ OD þ OB .
BC AD OA OC OB OD

7.1.74. Let the incircle of triangle ABC touch its sides BC and AC at points A0
and B0 respectively. Given that point L is the midpoint of segment A0 B0 , prove
that angle ALB is obtuse.
7.1.75. Given an acute triangle with the smallest side cand angle γ opposite to
it. Given that it is possible to paint the triangle in two colors, so that the distance
between any two points having the same color is not greater than c, prove that
γ  36 .
7.1.76. Given a point M inside of triangle ABC and da, db, dc denote the distances
from point M to lines BC, CA, AB, and Ra, Rb, Rc denote the distances from point
M to vertices A, B, C respectively. Prove that
(a) daRa þ dbRb þ dcRc  2(dadb þ dbdc þ dcda),
(b) Ra Rb þ Rb Rc þ Rc Ra  2ðda Ra þ d b Rb þ d c Rc Þ,
(c) Ra þ Rb þ Rc  2(da þ db þ dc), 
(d) 1
da db þ db1dc þ dc1da  2 þ db1Rb þ dc1Rc ,
1
d Ra
a 
(e) da1Ra þ db1Rb þ dc1Rc  2 Ra1Rb þ Rb1Rc þ Rc1Ra ,
 
(f) d1a þ d1b þ d1c  2 R1a þ R1b þ R1c .

7.1.77. Let r be the radius of the insphere of the tetrahedron ABCD. Prove that
r < 2ðABCD
ABþCDÞ.

7.1.78. Let ABCD be a tetrahedron such that ∠ADB ¼ ∠ADC ¼ ∠BDC ¼ 90 .
Given that r and R are the radiuses of the insphere and circumsphere of the
tetrahedron ABCD respectively, prove that
(a) DA þ BD þ DC  2R þ 6r,
(b) ðR þ r ÞðR  3r Þ  OO21 , where O and O1 are the centers of the spheres with the
radiuses r and R, respectively.
7.1.79. Let M be an arbitrary point in the plane of triangle ABC. Given that
R and r are the circumradius and the inradius of triangle ABC respectively and
point M be at a distance d from the circumcenter of triangle ABC, prove that
MA  MB  MC  2rjR2  d2j.
7.1.80. Given the circles ω(O, r) and ω1(O, r1), where r1 > r and let n-gon A1 A2 :::An
be inscribed in the circle ω and rays A1 A2 , A2 A3 , :::, An1 An , An A1 intersect the
circumference of ω1 at points B1, B2, . . . , Bn1, Bn, respectively. Prove that
296 7 Miscellaneous Inequalities

(a) B1 B2 þ B2 B3 þ ::: þ Bn1 Bn þ Bn B1  rr1 ðA1 A2 þ A2 A3 þ ::: þ An1 An þ An A1 Þ,


r2
(b) SB1 B2 :::Bn  r12 SA1 A2 :::An , for n ¼ 3, 4.
7.1.81. Consider on a plane circles C1(O1, r1), . . . , Cn(On, rn). Given that any
straight line on the plane intersects not more than two of these circles, prove that
P ri þrj nðn1Þ
Oi Oj  2 sin πn, where n  3.
1i<jn

7.1.82. Let points M, L, K be on side BC of triangle ABC (in the order B, M, L, K, C)


such that BM ¼ ML ¼ LK ¼ KC. Given that ∠ACB ¼ ∠MAB, prove that
(a) ∠KAL > 1, 5 ∠CAK,
(b) the factor 1.5 in (a) is the greatest possible.
7.1.83. Let B1, B2, . . . , Bn be n unit spheres, where n  3, arranged so that each
sphere touches externally other two spheres and let C1, C2, . . . , Cn be n tangential
points of these spheres. Let P be a point outside of all these spheres. Denote by ti
the length of the tangent drawn from point P to the Bi sphere (1  i  n). Prove that
the product of ti is not greater than the product of the distances PCi.
7.1.84. Prove that among all convex n-gons located inside of the given circle, the
inscribed regular n-gon has the greatest (a) area, and (b) perimeter.
7.1.85. Prove that among all convex n-gons containing the given circle, the
circumscribed regular n-gon has the smallest (a) area, and (b) perimeter.
7.1.86. Prove that, among all convex n-gons with the given perimeter, the regular
n-gon has the greatest area.
π π
7.1.87. Let M be a convex n-gon. Prove that 2ntg2n  H  P  2n sin 2n  D, where
1
H, P, D are respectively, the width, perimeter and diameter of the n-gon.
7.1.88. Let to any non-vanishing vector ~ V i on the plane, i ¼ 1, 2, . . . , n, be brought
into correspondence a strip p
 i , with the boundaries perpendicular to vector ~ V i , and
the width Wi is less than ~ V i . Prove that for any point O of that plane there exist
numbers a1, . . . , an 2 {0; 1} such that point M does not belong to any of these

!
strips, where OM ¼ a1 ~ V 1 þ a2 ~ V 2 þ ::: þ an ~
Vn.
7.1.89. Let A1(x1, y1), A2(x2, y2), . . . , An(xn, yn) be different points on the coordinate
n y1 þy2 þ:::þyn
plane, n  2 and M x1 þx2 þ:::þx
n ; n is their center of mass. Denote by C the
center of a circle with the minimum radius r, containing all points A1, A2, . . . , An.
Denote by d the distance between points M and C. Prove that
r  n .
d n2

7.1.90. Let a cube be sectioned by a plane, such that this plane passes through one of
the vertices of the cube and the section is a pentagon. Prove that the ratio of some of
the sides of the pentagon is greater than (a) 1,7; (b) 1,83; and (c) 1,84.

1
H ¼ min l(M ), D ¼ max l(M ) (See notations in the solution of the problem 4.1b.)
7.1 Miscellaneous Inequalities 297

7.1.91. Prove that if each of the plane angles at the vertex of the pyramid is not less
than 60 , then the sum of the lengths of all its edges is not greater than the perimeter
of the base.
7.1.92. Given points A1, B1, C1 respectively on the edges SA, SB and SC of the
tetrahedron SABC with a volume V, and points A2, B2, C2 on sides B1C1, C1A1, A1B1
of triangle A1B1C1 respectively. Let V1, V , Vffi 3 pbe
p2ffiffiffiffiffi ffiffiffiffiffiffithepvolumes
ffiffiffiffiffiffi pffiffiffiffi of tetrahedrons
AA1B2C2, BB1C2A2, CC1A2B2, prove that, 3 V 1 þ 3 V 2 þ 3 V 3  3 V .
7.1.93. Let R be the circumradius of the quadrilateral ABCD and S be its area. Prove
that
(a) (AB þ BC)(CD þ AD)  4R  AC,
(b) ðAB  CD þ BC  ADÞ2 ðAB  BC þ CD  ADÞ 
R2(AB þ BC)(BC þ CD)(CD þ AD)(AD þ AB).
(c) AB þ BC þ CD þ DA < AC þ BD þ 2R,
pffiffiffi 3
(d) 2RS  ðabcd Þ4 .
7.1.94. Let in the acute triangle ABC, bisector AD, the median BM and the altitude
CH be concurrent. Prove that angle BAC is greater than 45 .
7.1.95. (a) Let O, A, B, C, D be distinct points. Prove that ∠AOB  ∠COD þ
∠BOC  ∠AOD  0, 5 ∠AOC  ∠BOD.
(b) Let to each two points A, B on the plane, by a certain rule, is brought into
correspondences the number kAB k  0, such that three conditions are satisfied:
1. kABk ¼ 0, if and only if points A and B coincide,
2. kABk ¼ k BAk, for any points A and B,
3. kACk  k ABk þ k BCk, for any points A, B and C.
Prove that, for any points A, B, C, D on the plane

kABk  kCDk þ kBCk  k ADk 0, 5 kACk  kBDk : ð1Þ

(c) Prove that the number 0.5 in inequality (1) of 7.1.95b cannot be increased.
7.1.96. Let O be the intersection point of segments AA1, BB1, CC1 (A1 2 BC,
B1 2 AC, C1 2 AB), where O is the circumcenter of triangle ABC. Prove that,
(a) OA1 þ OB1 þ OC1  1, 5R,
(b) OA1 1 þ OB1 1 þ OC
1
1
 R6 .
7.1.97. Given a point M inside of a triangle with sides a, b, c, where a  b, c, and da,
db, dc denote the distances of point M from the lines containing the sides of the
triangle. Prove that
pffiffiffiffiffi pffiffiffiffiffi pffiffiffiffiffi qffiffiffiffiffiffiffiffiffi
pffiffi ffi
(a) d a þ db þ dc  3 2 3 a,
pffiffiffiffiffi pffiffiffiffiffi pffiffiffiffiffi pffiffiffiffiffiffiffiffiffiffiffiffi
(b) d a þ db þ dc  1, 92a, if triangle with sides a, b, c is obtuse.
298 7 Miscellaneous Inequalities

7.1.98. Let the convex pentagon ABCDE be inscribed in a unit circle and AB ¼ a,
BC ¼ b, CD ¼ c, DE ¼ d, AE ¼ 2. Prove that
a2 þ b2 þ c2 þ d2 þ abc þ bcd < 4.
7.1.99. (a) Vectors ~ a2 , :::,~
a1 ,~ an are on a plane such that their lengths are smaller
than or equal to 1. Prove  that  pone
ffiffiffi can choose the signs in the sum
~
c ¼ ~a1 ~ an so that ~
a2  :::  ~ c   2.
(b) Let ~ a2 , :::,~
a1 ,~ an be vectors with the lengths smaller than or equal 1. Prove that
  pffiffiffi
one can choose the signs in the sum ~ c ¼ ~a1  ~ an so that ~
a2  :::  ~ c   7.
     
(c) Vectors ~ a1 ,~ 
an are on a plane such that ~
a2 , :::,~ 
a1 þ ~  
a2 þ ::: þ ~ 
an  1. Prove
that there exist positive integers 1  i1 < i2 < . . . < ik  n, such that
  pffiffi
~ai1 þ ~ai2 þ ::: þ ~ aik   82.
7.1.100. Given a right-angled triangle ABC, with ∠ACB ¼ 90 . Let A1 2 BC,
B1 2 AC, C1 2 AB, and triangle A1B1C1 be right-angled. What is the smallest
possible value of the length of the hypotenuse of triangle A1B1C1?
7.1.101. Given an arbitrary point O on the base ABC of the triangular pyramid
SABC, such that all plane angles at vertex S are not greater than 60 . Prove that at
least one of angles SAO, SBO and SCO is not less than 60 .
7.1.102. Let ABC be a triangle with the semiperimeter p and inradius r. Half-circles
with diameters BC, CA, AB are drawn outside of triangle ABC. Given that  the circle
pffiffi
touching all these three half circles has a radius t, prove that 2 < t  2 þ 1  23 r.
p p

7.1.103. Given distinct points A1, A2, . . . , An on a plane, where n  2 and


D ¼ max Ai Aj , d ¼ min Ai Aj . Prove that
1i<jn 1i<jn
pffiffiffi
(a) there exists an equilateral triangle with sides equal to 3D containing all these
points,
(b) there exists a regular hexagon with sides equal to pDffiffi3 containing all these points,
(c) there exists a circle with a radius pDffiffi3 containing all these points,
pffiffi pffiffiffi
(d) D > 23 ð n  1Þd,
pffiffiffi
(e) D  3d, for n ¼ 6.
7.1.104. Given a convex n-gon. The length of any segment, connecting the vertex of
that polygon with a certain point of its boundary and dividing its area into two
isometric parts is smaller than or equal to 1. Prove that the area of the n-gon
pffiffi
(a) is smaller than or equal to 33, if n ¼ 3,
(b) is less than 23, if n ¼ 4,
(c) is less than n4 sin πn, if n  5.
7.1.105. In a given triangle each median is greater than any of its altitudes. Prove
that the smallest angle of that triangle is less than 5 300 .
7.1 Miscellaneous Inequalities 299

7.1.106. Given on a plane an acute triangle ABC and the straight line l. Let u, v,
w be, respectively, the distances from points A, B and C to line l.
Prove that u2tgα þ v2tgβ þ w2tgγ  S, where ∠A ¼ α, ∠B ¼ β, ∠C ¼ γ and S is
the area of triangle ABC.
7.1.107. Given a point M and a triangle ABC with sides a, b, c. Prove that
(a) MA þ MB þ MC  pffiffiffiffiffiffiffiffiffiffiffiffiffiffiffiffiffiffiffiffiffiffiffiffi
3abc
,
2 2 a b þb c þc a
2 2 2 2

(b) MA þ MB þ MC  6r.
7.1.108. In a non-isosceles triangle ABC, let O, I, H be the circumcenter, incenter
and orthocenter, respectively. Prove that ∠OIH > 135 .
7.1.109. (a) Let the inscribed circle of triangle ABC touch sides AB, BC, AC at
points C1, A1, B1 respectively. Prove that segments AA1, BB1, CC1 intersect at one
point (denote it by M ). Prove that AM  p2ffiffi3 AB1 :

(b) Let ABCDEF be a convex hexagon with area S. Prove that


pffiffiffi
ACðBD þ BF  DFÞ þ CEðBD þ DF  BFÞ þ AEðBF þ DF  BDÞ  2 3S.
(c) Three circles externally touch each other at points X, Y, Z.
Let us multiply the radiuses of these circles by p2ffiffi3, keeping the centers fixed, and
consider these new circles. Prove that any point of triangle XYZ is covered at
least by one of the new circles.
(d) Given triangle ABC with sides a, b, c and point M, prove that

MA þ MB MB þ MC MC þ MA pffiffiffi R
þ þ 2 3 :
pc pa pb r

7.1.110. (a) Let A1A2 . . . An be a convex polygon. Point P is chosen inside of this
polygon, such that its projections P1, P2, . . . , Pn on lines A1A2, . . . , AnA1 respec-
tively lie on the sides of the polygon. Prove that for arbitrary points X1, X2, . . . , Xn
on sides A1A2, . . . , AnA1 respectively,

X 1 X2 Xn X 1
max ; :::;  1:
P1 P2 Pn P1

(b) Let A1A2 . . . An be a cyclic convex polygon whose circumcenter is strictly in its
interior. Let B1, B2, . . . , Bn be arbitrary points on sides A1A2, . . . , AnA1 respec-
tively other than the vertices. Prove that

B 1 B2 B 2 B3 B n B1
þ þ ::: þ > 1:
A 1 A3 A 2 A4 A n A2
300 7 Miscellaneous Inequalities

7.1.111. Let MNP be an equilateral triangle inscribed in triangle ABC such that
M 2 BC, N 2 AC, P 2 AB. Point I is not inside of triangle MNP, where I is the
incenter of triangle ABC. Prove that ∠A > 120 .
7.1.112. Consider triangle ABC with inradius r. Let M and M0 be two points inside
of triangle, such that ∠MAB ¼ ∠M0 AC and ∠MBA ¼ ∠M0 BC. Denote by da, db, dc
and da0 , db0 , dc0 the distances from M and M0 to sides BC, CA, AB respectively. Prove
that dadbdcda0 db0 dc0  r6.
7.1.113. (a) Given convex hexagon ABCDEF with AB k DE, BC k EF, CD k FA. The
distance between lines AB and DE is equal to the distance between lines BC and EF,
and to the distance between lines CD and FA. Prove that the sum AD þ BE þ CF
does not exceed the perimeter of hexagon ABCDEF.
(b) Let ABCDEF be a cyclic hexagon, such that AC ¼CE ¼ EA.  Given that diag-
onals AD, BE, CF intersect at one point, prove that DE
EF  CD
BC < 1.

7.2 Solutions

2MA1 BC sin ∠AA1 C


1

AA1 ¼ 12AA1 BC sin ∠AA1 C ¼ SABC . In the same way, one can prove
MA1 SMBC
7.1.1. (a) Note that
BB1 ¼ SABC and CC1 ¼ SABC . Consequently, AA1 þ BB1 þ CC1 ¼ 1, or 1 ¼ AA1 þ
SMAC
that MB 1 MC1 SMAB MA1 MB1 MC1 MA1

BB1 þ CC1  maxðAA1 ; BB1 ; CC1 Þ þ maxðAA1 ; BB1 ; CC1 Þ þ maxðAA1 ; BB1 ; CC1 Þ. Hence, we deduce
MB1 MC1 MA1 MB1 MC1

that MA1 þ MB1 þ MC1  max (AA1, BB1, CC1).


Remark 1 max(AA1, BB1, CC1) < max (AB, BC, AC), consequently, the inequality
MA1 þ MB1 þ MC1 < max (AB, BC, AC) holds true.
Remark 2 MA1 þ MB1 þ MC1  min (AA1, BB1, CC1).
(b), (c) Let max(AA1, BB1, CC1) ¼ AA1, then according to the problem 7.1.1a
MA  MB1 þ MC1. Note that MA1  max (MB, MC). Therefore,
MA þ MB þ MC > MA þ max (MB, MC)  MB1 þ MC1 þ MA1,
 
MA2 þ MB2 þ MC2 > MA2 þ max MB2 ; MC2  ðMC1 þ MB1 Þ2 þ
MA21 > MA21 þ MB21 þ MC21 :
AA1 þ BB1 þ CC1 ¼ 1 (see the proof of the problem 7.1.1a). Also
(d) We have that MA 1 MB1 MC1

 
AA1 BB1 CC1 MA1 MB1 MC1
þ þ þ þ ¼
MA1 MB1 MC1 AA1 BB1 CC1
  
AA1 MB1 BB1 MA1 AA1 MC1 CC1 MA1 BB1 MC1 CC1 MB1
¼3þ þ þ þ þ þ 
BB1 MA1 AA1 MB1 CC1 MA1 AA1 MC1 CC1 MB1 BB1 MC1
AA1 BB1 CC1 MA MB MC
 3 þ 2 þ 2 þ 2 ¼ 9, hence þ þ  9: Thus, þ þ 6
MA1 MB1 MC1 MA1 MB1 MC1
7.2 Solutions 301

SMAB þSMCB MB SMAB þSMCB MC SMCB þSMAC


AA1 þ
(e) We have that MA SMAC , MB1 ¼ SMAC , MC1 ¼ (see the proof of
1
SMAB
the problem 7.1.1а). Hence, it follows that
pffiffiffiffiffiffiffiffiffiffiffiffiffiffiffiffiffiffiffiffiffiffiffi pffiffiffiffiffiffiffiffiffiffiffiffiffiffiffiffiffiffiffiffiffiffiffi pffiffiffiffiffiffiffiffiffiffiffiffiffiffiffiffiffiffiffiffiffiffiffi
MA MB MC 2 SMAB  SMAC 2 SMAB  SMCB 2 SMCB  SMAC
     ¼ 8:
MA1 MB1 MC1 SMBC SMAC SMAB

(f) According to the Cauchy’s inequality and the problem 7.1.1e, we have that
sffiffiffiffiffiffiffiffiffiffiffiffiffiffiffiffiffiffiffiffiffiffiffiffiffiffiffiffiffiffiffiffiffiffiffiffiffiffiffiffiffi
 ffi
AM BM BM CM CM AM 3 AM BM CM 2 p 3
ffiffiffiffiffi
 þ  þ  3    3 82 ¼ 12:
A1 M B1 M B1 M C1 M C1 M A1 M A1 M B1 M C1 M

(g) We have that

MA1 MB1 MC1


1¼ þ þ ¼
MA þ MA1 MB þ MB1 MC þ MC1
MA21 MB21 MC21
¼ þ þ 
MA1 ðMA þ MA1 Þ MB1 ðMB þ MB1 Þ MC1 ðMC þ MC1 Þ
ðMA1 þ MB1 þ MC1 Þ2
 :
MA1 ðMA þ MA1 Þ þ MB1 ðMB þ MB1 Þ þ MC1 ðMC þ MC1 Þ

Therefore, MA  MA1 þ MB  MB1 þ MC  MC1  2MA1  MB1 þ 2MB1  MC1 þ


2MC1 MA1.
(h) We have that,

A1 M B1 M C1 M SMBC SMAC SMAB 3


þ þ ¼ þ þ 
AM BM CM SMAB þ SMAC SMAB þ SMBC SMAC þ SMBC 2

(see the proof of the problem 5.5.19b)


(i) We have that,

MA1 MB1 MC1


1¼ þ þ
MA þ MA1 MB þ MB1 MC þ MC1
pffiffiffiffiffiffiffiffiffi pffiffiffiffiffiffiffiffiffi pffiffiffiffiffiffiffiffiffiffi 2
MA1 þ MB1 þ MC1
 ,
MA þ MB þ MC þ MA1 þ MB1 þ MC1

consequently
pffiffiffiffiffiffiffiffiffiffiffiffiffiffiffiffiffiffiffiffiffiffi pffiffiffiffiffiffiffiffiffiffiffiffiffiffiffiffiffiffiffiffiffiffi pffiffiffiffiffiffiffiffiffiffiffiffiffiffiffiffiffiffiffiffiffiffi
MA þ MB þ MC  2 MA1  MB1 þ MB1  MC1 þ MC1  MA1 :
302 7 Miscellaneous Inequalities

(j) We have that

pffiffiffiffiffiffiffi pffiffiffiffiffiffiffi pffiffiffiffiffiffiffiffi2


MA MB MC MA þ MB þ MC
2¼ þ þ  :
MA þ MA1 MB þ MB1 MC þ MC1 MA þ MB þ MC þ MA1 þ MB1 þ MC1

Thus, it follows that

MA þ MB þ MC þ 2ðMA1 þ MB1 þ MC1 Þ


pffiffiffiffiffiffiffiffiffiffiffiffiffiffiffiffiffiffi pffiffiffiffiffiffiffiffiffiffiffiffiffiffiffiffiffiffi pffiffiffiffiffiffiffiffiffiffiffiffiffiffiffiffiffiffi
 2 MA  MB þ MB  MC þ MC  MA :

(k), (l) Denote by SMAB ¼ SC, SMBC ¼ SA, SMAC ¼ SB. We have that, MA ¼
SB þSC SA þSC SA þSB
SA  MA1 , MB ¼ SB  MB1 , MC ¼ SC  MC1 , consequently

  2
S2B 2 S2A 2 SC S2A
MA þ MB þ MC ¼
2 2
MA þ MB þ 2 MA1 þ 2 MC1 þ
2 2 2
S2A 1 S2B 1 SA SC
 2 2 
SC SB SB SC SA SC SA S B
þ 2 MB1 þ 2 MC1 þ 2
2 2
MA1 þ 2 MB1 þ 2 MC1 
2 2 2
SB SC S2A SB SC
qffiffiffiffiffiffiffiffiffiffiffiffiffiffiffiffiffiffiffiffiffiffiffiffiffiffiffiffiffiffiffiffiffiffi
 2ðMA1  MB1 þ MB1  MC1 þ MC1  MA1 Þ þ 6 3 MA21  MB21  MC21 :

We have that,

MA  MB þ MB  MC þ MC  MA ¼ MA1  MB1 þ MB1  MC1 þ MC1  MA1 þ


 2
SC SC SC S2
þ MA1  MB1 þ MA1  MB1 þ MA1  MB1 þ A MB1  MC1 þ
SA SB SB SA SB SC

SA SA S2 SB SB
þ MB1  MC1 þ MB1  MC1 þ B MC1  MA1 þ MC1  MA1 þ MC1  MA1 
SB SC SA SC SA SC
qffiffiffiffiffiffiffiffiffiffiffiffiffiffiffiffiffiffiffiffiffiffiffiffiffiffiffiffiffiffiffiffiffi
 MA1  MB1 þ MB1  MC1 þ MC1  MA1 þ 9 MA61  MB61  MC61 ¼
9

qffiffiffiffiffiffiffiffiffiffiffiffiffiffiffiffiffiffiffiffiffiffiffiffiffiffiffiffiffiffiffiffiffi
¼ MA1  MB1 þ MB1  MC1 þ MC1  MA1 þ 9 3 MA21  MB21  MC21 :

(m) Let ∠BMC ¼ α2, ∠AMC ¼ β2 and ∠AMB ¼ γ 2. We have that

2S  2SMAB þ 2SMBC þ 2SMAC  MA  MB sin γ 2 þ MC  MB sin α2 þ MA  MC sin β2


ð7:2Þ
7.2 Solutions 303

(point M can be inside or outside triangle ABC). Since, sin x sin y  1 þ cos x cos y,
then from (7.2) we deduce that

1 þ cos γ 1 cos γ 2 1 þ cos α1 cos α2 1 þ cos β1 cos β2


4S  2MA  MB þ 2MC  MB þ 2MA  MC ¼
sin γ 1 sin α1 sin β1
2MA  MB 2MC  MB 2MA  MC  2 
¼ þ þ þ MA þ MB2  c2 ctg γ 1 þ
sin γ 1 sin α1 sin β1
   
þ MB2 þ MC2  a2 ctg α1 þ þ MA2 þ MC2  b2 ctg β1 ,

where α1, β1, γ 1 are the angles of the triangle with sides a1, b1, c1. Consequently,

2MA  MB 2MC  MB
4S þ a2 ctg α1 þ b2 ctg β1 þ c2 ctg γ 1  þ þ
sin γ 1 sin α1
2MA  MC
þ þ MA2 ðctg γ 1 þ ctg β1 Þ þ MB2 ðctg α1 þ ctg γ 1 Þ
sin β1
2MA  MB 2MC  MB 2MA  MC
þ MC2 ðctg α1 þ ctg β1 Þ ¼ þ þ
sin γ 1 sin α1 sin β1
MA2 sin α1 MB2 sin β1 MC2 sin γ 1 ða1 MA þ b1 MB þ c1 MCÞ2
þ þ þ ¼ :
sin β1 sin γ 1 sin α1 sin γ 1 sin α1 sin β1 2S1

Thus, it follows that


 
8SS1 þ 2S1 a2 ctg α1 þ b2 ctg β1 þ c2 ctg γ 1  ða1 MA þ b1 MB þ c1 MCÞ2 :

To complete the proof, it remains to note that


 
4S1 a2 ctg α1 þ b2 ctg β1 þ c2 ctg γ 1 ¼ a2  2b1 c1 cos α1 þ b2  2a1 c1 cos β1 þ
c2  2a1 b1 cos γ 1 ¼
     
¼ a2 b21 þ c21  a21 þ b2 a21 þ c21  b21 þ c2 a21 þ b21  c21 :

Remark If point M0 is, such that ∠BM0C ¼ 180  α1, ∠AM0C ¼ 180  β2,
∠AM0B ¼ 180  γ 1, then a1MA þ b1MB þ c1MC > a1M0A þ b1M0B þ c1M0C,
where M ≢ M0 (see also problem 4.1.17).
7.1.2. (a) Let us complete triangle up to a rectangular (Figure 7.1).
Then, EF  BD or 4r < EF  BD. Hence, we obtain that r < BD 4 .

2 ¼ k, then k > 1. We have that


(b) Let AB ¼ BC. Denote AB : AC BO
OH ¼ k (see
Figure 7.2).
2 2
þ ðr þ rkÞ2 . Therefore, AB2 ¼ r 2 ðkþ1 Þ k
2
Hence, AB2 ¼ AB k2 k2 1
> 9r 2 , as k2(k þ 1)2
 9(k2  1) ¼ (k2  3)2 þ 2k2(k  1) > 0.
304 7 Miscellaneous Inequalities

Figure 7.1

B C

O2 F

E O1
A D

Figure 7.2 B

A H C

7.1.3. By Carnot’s theorem, we have that

aka bkb ckc aka þ bkb þ ckc


r þ R ¼ ka þ kb þ kc ¼ þ þ  ¼
a b c minða; b; cÞ
2S
¼ ¼ maxðha ; hb ; hc Þ,
minða; b; cÞ
aka bkb ckc aka þ bkb þ ckc 2S
r þ R ¼ ka þ kb þ kc ¼ þ þ  ¼
a b c maxða; b; cÞ maxða; b; cÞ
¼ minðha ; hb ; hc Þ

(see the proof of problem 2.4.11).


7.1.4. (a) Denote BC ¼ a, CA ¼ b, AB ¼ c and ∠C ¼ γ. By triangle angle bisector
proportionality theorem, one can easily obtain that CA1 ¼ bþc
ab
, CB1 ¼ aþc
ab
. Note that
A1B1 can be found by using the cosine law for triangles A1CB1 and ABC,
7.2 Solutions 305

sffiffiffiffiffiffiffiffiffiffiffiffiffiffiffiffiffiffiffiffiffiffiffiffiffiffiffiffiffiffiffiffiffiffiffiffiffiffiffiffiffiffiffiffiffiffiffiffiffiffiffiffiffiffiffiffiffiffiffiffiffiffiffiffiffiffiffiffiffiffiffiffiffiffiffiffiffiffiffiffi
 2  2 ðabÞ2
A1 B1 ¼ ab
þ aþc ab
2 cos γ ¼
bþc ða þ c Þðb þ c Þ
sffiffiffiffiffiffiffiffiffiffiffiffiffiffiffiffiffiffiffiffiffiffiffiffiffiffiffiffiffiffiffiffiffiffiffiffiffiffiffiffiffiffiffiffiffiffiffiffiffiffiffiffiffiffiffiffiffiffiffiffiffiffiffiffiffiffiffiffiffiffiffiffiffiffiffiffiffiffiffiffiffiffiffiffiffiffiffiffiffiffiffiffiffiffiffi
 2  2 ðabÞ2 a 2 þ b 2  c2
¼ ab
þ aþc ab
2 ¼
bþc ða þ c Þðb þ c Þ 2ab
vffiffiffiffiffiffiffiffiffiffiffiffiffiffiffiffiffiffiffiffiffiffiffiffiffiffiffiffiffiffiffiffiffiffiffiffiffiffiffiffiffiffiffiffiffiffiffiffiffiffiffiffiffiffiffiffiffiffiffiffiffiffiffiffiffiffiffiffiffiffiffiffiffiffiffiffiffiffiffiffiffiffiffiffiffiffiffiffiffiffiffiffiffiffiffiffiffiffiffiffiffiffiffiffiffiffiffiffiffiffiffiffiffiffiffiffiffiffiffiffiffiffiffiffiffiffiffiffiffiffiffiffiffiffiffiffiffiffiffiffiffiffiffiffiffiffiffiffiffiffiffiffiffiffiffiffiffiffiffiffiffiffiffiffiffiffi
!ffi
u
u 2 aba2 þ b2  þ 2cða þ b þ cÞab  a2 þ b2  c2 ðab þ cða þ b þ cÞÞ
¼ t ðaþcab ÞðbþcÞ ab
sffiffiffiffiffiffiffiffiffiffiffiffiffiffiffiffiffiffiffiffiffiffiffiffiffiffiffiffiffiffiffiffiffiffiffiffiffiffiffiffiffiffiffiffiffiffiffiffiffiffiffiffiffiffiffiffiffiffiffiffiffiffiffiffiffiffiffiffiffiffiffiffiffiffiffiffiffiffiffiffiffiffiffiffiffiffiffiffiffiffiffiffiffiffiffiffiffiffiffiffiffiffiffiffi sffiffiffiffiffiffiffiffiffiffiffiffiffiffiffiffiffiffiffiffiffiffiffiffiffiffiffiffiffi
 2 c2 ða þ cÞðb þ cÞ  cða  bÞ2 ða þ b þ cÞ abc2
¼ ab
ðaþcÞðbþcÞ  
ab ða þ cÞðb þ cÞ
sffiffiffiffiffiffiffiffiffiffiffiffiffiffiffiffiffiffiffiffiffiffiffiffiffiffi rp
ffiffiffiffiffiffiffiffiffiffiffi
ffiffiffiffiffi pffiffiffiffiffi aþb
abc2 abc ab þ c þ c a þ b þ 2c
 pffiffiffiffiffi pffiffiffiffiffi ¼   2 ¼ :
2 ac  2 bc 4 4 4 8

Similarly, we deduce that B1 C1  bþcþ2a8 and A1 C1  aþcþ2b


8 . Therefore,
A1 B1 þ B1 C1 þ A1 C1  4aþ4bþ4c
8 ¼ 2 ðAB þ BC þ ACÞ.
1

(b) We have that


pffiffiffiffiffi pffiffiffiffiffi pffiffiffiffiffi
2 2c ab b ac a bc 2
ð A 1 B1 Þ þ ð B1 C 1 Þ þ ð A1 C 1 Þ  þ þ 
4 4 4
cða þ bÞ bða þ cÞ aðb þ cÞ 1
 þ þ  ða þ b þ cÞ2
8 8 8 12

(see the proof of problem 7.1.4a).


AD
7.1.5. We need to prove that DP is the smallest, if AD is the bisector of angle A.
Indeed, we have that

AD AD2 AD2 AD AD sin β sin γ


¼ ¼ ¼  ¼  ¼
DP DP  AD BD  DC BD DC sin ∠BAD sin ∠CAD
2 sin β sin γ 2 sin β sin γ sin β sin γ AD21
¼  ¼ a ¼ ,
cos ð∠BAD  ∠CADÞ  cos a 1  cos a sin 2 D1 B  D1 C
2

where ∠BAD1 ¼ ∠CAD1, D1 2 BC.


bca2 AD2
It is known that BD1 ¼ bþc
ac
, CD1 ¼ bþc
ba
and AD21 ¼ bc  ðbþc Þ2
. Hence, D1 BD11 C ¼
ðbþcÞ2 a2 ðbþcÞ2 a2 2
 ðaþcbÞ2 b
2

DP 
. Therefore, AD BE
a2 a2 . Similarly, we obtain that EQ and
ðaþbÞ2 c2
RF 
CF
c2 . By summing up these three inequalities, we deduce that
306 7 Miscellaneous Inequalities


AD BE CF ðb þ cÞ2 ða þ cÞ2 ða þ bÞ2 1 bþc aþb aþc 2
þ þ  þ 2
þ  3  þ þ 3¼
DP EQ RF a2 b c2 3 a c b
  
1 b a a c c b 2 1
¼ þ þ þ þ þ  3  ð2 þ 2 þ 2Þ2  3 ¼ 9,
3 a b c a b c 3

as x2 þ y2 þ z2  13 ðx þ y þ zÞ2 and x þ 1x  2 (x > 0). Thus, AD


DP þ EQ þ RF  9.
BE CF

The equality holds true only for an equilateral triangle ABC, with AD, BE, CF
being the bisectors of angles A, B, C respectively.
7.1.6. Let AC ¼ a, CE ¼ b, AE ¼ c. According to problem 1.1.14a, we get that
AB  CE þ BC  AE  AC  BE. This means that BC
BE  bþc. Similarly, we deduce that
a

DA  aþc and FC  aþb. By summing up these three inequalities, we obtain that


DE b FA c

BE þ DA þ FC  bþc þ aþc þ aþb  2 (see the proof of problem 5.5.19b). Therefore,


BC DE FA a b c 3

BE þ DA þ FC  2.
BC DE FA 3

7.1.7. Let the rectangle with sides a and b be inside of the rectangle with sides c and
d. Consider a rectangle with sides c1 and d1, such that its sides are parallel to the
sides of the rectangle with sides c and d. Moreover, the rectangle with sides a and
b is inscribed in it (see Figure 7.3).
It is clear that c1  c and d1  d. Hence, one can assume that the centers of these
rectangles coincide (prove that the center O of the rectangle with sides a and b is the
center of rectangle with sides c1 and d1). Let us consider a circle with the center
pffiffiffiffiffiffiffiffiffi
a2 þb2
O and the radius 2 (see Figure 7.4).

Figure 7.3 d
d1
b
c O
O1 c1
a

Figure 7.4

A d
7.2 Solutions 307

Figure 7.5
b

a cos α + b sin α
a
α
b cos α + a sin α

We have that a  AB (see the proof of problem 3.1.34), which means that
0 sffiffiffiffiffiffiffiffiffiffiffiffiffiffiffiffiffiffiffiffiffiffiffiffiffi12 0 sffiffiffiffiffiffiffiffiffiffiffiffiffiffiffiffiffiffiffiffiffiffiffiffiffi12
d a2 þ b2 c 2 A c a2 þ b2 d 2 A
a2  @   þ@   ,
2 4 4 2 4 4
 ffi 2
qffiffiffiffiffiffiffiffiffiffiffiffiffiffiffiffiffiffiffiffiffiffiffiffiffiffiffiffiffiffiffiffiffiffiffiffiffiffiffiffiffiffiffiffiffiffiffiffiffiffiffiffiffiffiffiffiffiffiffiffiffiffiffiffiffiffiffiffiffiffiffiffiffiffiffiffiffiffi

2 2
  
cd  a þb  a þb c þd þc d
2 2 2 2 2 2 2
 4a2 b2 ,

(cd  2ab)2  (a2 þ b2)2  (a2 þ b2)(c2 þ d2) þ c2d2, (b2  a2)2  (bd  ac)2
þ (bc  ad)2.
Let a < c  d < b and (b2  a2)2  (bd  ac)2þ (bc   ad) . Denote by
2
π
bd  ac ¼ R cos α, bc  ad ¼ R sin α, where R > 0, α 2 0; 2 . Then, from the condi-
tion (bd  ac)2 þ (bc  ad)2  (b2  a2)2, we deduce that R  b2  a2. Note that,
d ¼ Rðb cosb2αþa
a2
sin αÞ
, c ¼ Rða cosb2αþb
a2
sin αÞ
. Hence, d  b cos α þ a sin α,
c  a cos α þ b sin α. Thus, the rectangle with sides b cos α þ a sin α,
a cos α þ b sin α can be placed inside of the rectangle with sides c and d. Then it
can also contain the rectangle with sides a and b (see Figure 7.5).
Second Solution (G. Nersisyan). Without loss of generality, one can assume that
two adjacent vertices of the rectangle with sides a and b are on the sides of the
second rectangle. In that case, the following conditions must be fulfilled.

a cos α þ b sin α  c,
ð7:3Þ
b cos α þ a sin α  d:

Note that sin α  0 and cos α  0. Consider the following inequalities:



ax þ by  c,
ð7:4Þ
bx þ ay  d,
c c
< 1, > 1, ð7:5Þ
b a
d d
< 1, > 1: ð7:6Þ
b a
The set of the inequalities in (7.3) has a solution, if and only if the solution of
equations ax þ by ¼ c, bx þ ay ¼ d is outside the circle (see Figure 7.6).
Hence, (b2  a2)2  (bc  ad)2 þ (bd  ac)2. This ends the proof.
308 7 Miscellaneous Inequalities

Figure 7.6 y

d/a

bx+ay=d

c/a x
1 ax+by=c

7.1.8. Let a  b  c. Denote u þ v ¼ x, v þ w ¼ y, and u þ w ¼ z, Then, w ¼ yþzx


2 ,
u¼ 2 ,v¼ 2 .
xþzy xþyz

If ab  ac þ bc, we have that



w 2 2 u 2 2 v 2 2 4w u v
ab þ b c þ a c > 4S 2
þ þ ¼
uþv  vþw uþw  u þv vþw uþw
y x 2z x 1 z y
¼ 4S2 2 þ þ þ þ þ  3  4S2 ð2 þ 2 þ 1  3Þ ¼ 8S2 :
x 2y x 2z 2 y z

a b þ vþw
w 2 2
Thus, uþv u
b2 c2 þ uþw
v
a2 c2 > 8S2 .
If ab < ac þ bc, we have that
w 2 2 u 2 2 v 2 2
a b þ bc þ a c ¼
uþv vþw uþw
 2 2   2 2
ya b xb2 c2 za2 b2 xa2 c2 zb c ya2 c2 1 
¼ þ þ þ þ þ  a2 b2 þ b2 c2 þ a2 c2 
2x 2y 2x 2z 2y 2z 2
1 
 ab2 c þ a2 bc þ abc2  a2 b2 þ b2 c2 þ a2 c2 :
2

We need to prove that 2ab2c þ 2a2bc þ 2abc2  16S2 þ a2b2 þ b2c2 þ a2c2, that is

2ab2 c þ 2a2 bc þ 2abc2  ða þ b þ cÞða þ b  cÞða þ c  bÞðb þ c  aÞ þ a2 b2


þ b2 c 2 þ a 2 c 2 ,
  
2ab2 c þ 2a2 bc þ 2abc2  ða þ bÞ2  c2 c2  ða  bÞ2 þ a2 b2 þ b2 c2 þ a2 c2 ,
   2
2abcða þ b þ cÞ  c2 ða þ bÞ2 þ ða  bÞ2  c4  a2  b2 þ a2 b2
þ b2 c2 þ a2 c2 ,
7.2 Solutions 309

a4 þ b4 þ c4 þ 2abcða þ b þ cÞ  3a2 b2  3b2 c2  3a2 c2  0,


 2
c4  ðac þ bc  abÞ2 þ a2  b2  2c2 ða  bÞ2  0,
 2    
c þ ac þ bc  ab c2  ac  bc þ ab þ ða  bÞ2 ða þ bÞ2  2c2  0,
   
ðc  aÞðc  bÞ c2 þ ac þ bc  ab þ ða  bÞ2 ða þ bÞ2  2c2  0:

The obtained inequality holds true, since (c  a)(c  b)  0, c2 þ ac þ bc >


ac þ bc > ab, (a þ b)2  4b2 > 2c2.
7.1.9. Let complex numbers a, b, c and a1, b1, c1 correspond to the vertices of
triangles ABC and A1B1C1 in a complex plane. According to the assumptions of
the problem, triangles ABC and A1B1C1 are similar and have the same orientation,
hence

b1  a1 b  a
¼ : ð∗ Þ
c 1  a1 c  a

We need to prove that there exist complex numbers z and v such that
8
< vðz  aÞ ¼ a1  a,
> ð7:7Þ
vðz  bÞ ¼ b1  b, ð7:8Þ
>
:
vðz  cÞ ¼ c1  c: ð7:9Þ

1 b1
From (7.7) and (7.8), it follows that v ¼ aba þ 1, while from (7.7) and (7.9) we
a1 c1 1 b1 1 c1
obtain that v ¼ ca þ 1. From (*), we have that aba þ 1 ¼ aca þ 1.
If v ¼ 0, then from (7.7) and (7.8), we have that a1 ¼ a, b1 ¼ b, but from the
assumptions of the problem, we have that c1 ¼ c. Thus, it follows that

AA1  BC ¼ 0 ¼ BB1  CA þ CC1  AB:

If v 6¼ 0, then z ¼ a1va þ a. Let the complex number z corresponds to point M.


Then |a1  a| ¼ |v||z  a|, |b1  b| ¼ |v||z  b|, |c1  c| ¼ |v||z  c|, that is AA1 ¼ |v| 
MA, BB1 ¼ |v|  MB and CC1 ¼ |v|  MC. Using the inequality of the problem
1.1.14a, for points M, A, B, C, we obtain that MA  BC  MB  AC þ MC  AB.
Hence, |v|MA  BC  |v|MB  AC þ |v|MC  AB, or AA1  BC  BB1  AC þ CC1  AB.
7.1.10. Let the length of the edge of the cube ABCDA1B1C1D1 be equal to a.
Consider the midpoints P, Q, R, S, T, U of the edges AB, BC, CC1, C1D1, D1A1,
A1A, respectively. Since segments RU, SP and QT pass through the center O of the
cube and QP k RU k ST, then points P, Q, R, S, T, U belong to the same plane.
We have that, pffiffi
QR ¼ RS ¼ ST ¼ TU ¼ UP ¼ PQ ¼ OQ ¼ OR ¼ OS ¼ OT ¼
OU ¼ OP ¼pffiffia 2 p2
ffiffi hexagon PQRSTU is regular. The inradius of the hexagon
,ffiffithuspthe
is equal to 2  2 ¼ 46 a. We need to prove that if a circle
a 2 3
pffiffi
with the radius r can be
placed inside of the cube, then it is impossible that r > 4 a.
6
310 7 Miscellaneous Inequalities

Figure 7.7
P

O1 P1
M a h

pffiffi
Let O1 be the center of that circle and the radius r > 6
4 a.
 
Consider a plane Π containing that circle. Denote by α ¼ Π d , AA1 B1 B ,
   
β¼ Π d, BB1 C1 C , γ ¼ Π d, ABCD . Note that cos2α þ cos2β þ cos2γ ¼ 1.
 
Indeed, let B(0; 0; 0), A(a; 0; 0), C(0; a; 0), B1(0; 0; a),~ nðu; v; wÞ and ~
n⊥Π,~ n ¼ 1,
then cos 2 α þ cos 2 β þ cos 2 γ ¼ ð~ e1  ~
nÞ2 þ ð~e2 ~
nÞ2 þ ð~
e3  ~
nÞ2 ¼ u2 þ v2 þ w2 ¼
1, where ~
e1 ð0; 1; 0Þ,~
e2 ð1; 0; 0Þ,~
e3 ð0; 0; 1Þ. Since Π d, DD1 C1 C ¼ α, then one can
assume that the distance of O1 from the plane Π1  AA1Bq a
1Bffiffi is not greater than 2.

Therefore, sin α ¼ ρðρOðO1 ;Π


1 ;Π1 Þ
\ΠÞ ¼ O1 M  2O1 M  2r < 2 6a ¼
h a a affi
p 2
3 (see Figure 7.7).
qffiffi 1 4

Thus, sin α < 23. Hence, cos α > p1ffiffi3. Similarly, we obtain that cos β > p1ffiffi3 and
cos γ > p1ffiffi. Then, cos2α þ cos2β þ cos2γ > 1. This leads to a contradiction.
3

7.1.11. (a) Let points Ai(xi; yi; zi), i ¼ 1, 2, 3 be given inside of the cube with vertices
(0; 0; 0), (0; 1; 0), (0; 0; 1), (1; 0; 0), . . . , (1; 1; 1), then xi, yi, zi 2 (0; 1), i ¼ 1, 2, 3.
We have that,

A1 A2 2 þ A2 A3 2 þ A3 A1 2  2
 2  2
 2max xi  xj þ 2max yi  yj þ 2max zi  zj < 6:
pffiffiffi
Hence, it follows that minðA1 A2 ; A2 A3 ; A3 A1 Þ < 2.
(b) Let d be the smallest number, such that eight cubes with edges d, placed in each
of the vertices of the cube, contain all given points (see Figure 7.8).
Then, it is clear that d  12. Thus, each of these cubes contains only one of these
points; otherwise the distance between two points in the same cube is less than
pffiffiffi pffiffi
3d  23. In this case the problem would have been proven. Since d is minimal,
then at least one of the points is on the surface of one of the cubes. Thus, some two points
M and N from eight given points are in a rectangular parallelepiped with dimensions
qffiffiffiffiffiffiffiffiffiffiffiffiffiffiffiffiffiffiffiffiffiffiffiffiffiffiffiffiffiffiffiffiffiffiffiffiffi pffiffiffiffiffiffiffiffiffiffiffiffiffiffiffiffiffiffiffiffiffiffiffiffiffiffiffiffi
d d (1  d). Then, MN  d2 þ d 2 þ ð1  dÞ2 ¼ 1  dð2  3dÞ < 1.

(c) Yes, it is possible; an example is presented in Figure 7.9.


7.2 Solutions 311

Figure 7.8

N M

Figure 7.9 128

64
506

2000
64 506

64

Figure 7.10

7.1.12. Figure 7.10 gives an example with eight adjoining squares.


Denote by A and B the centers of two squares touching the fixed square S with
the center O. Let OA ¼ a, OB ¼ b and AB ¼ c. We have that
pffiffiffi pffiffi
OApffiffi¼ a  ONþ KA  12 þ 12 ¼ 1 and a, b  2. Also, a ¼ OA  OM þ MA  22
pffiffiffi
þ 22 ¼ 2 (see Figure 7.11).
Applying the law of cosines to triangle OAB, we obtain that c2 ¼ a2 þ b2
 2ab cos α. Hence, cos α ¼ a þb c2
2 2
2ab , where α ¼ ∠AOB.
Since c  1, then cos α  a þb 1
2 2
2ab .
a2 þb2 1
We need to prove that 2ab  34, or equivalently, 4a2  6ab þ 4b2pffiffiffi4  0.

Indeed, consider a function f(x) ¼ 4x2  6xbpþ 4b2  4on the interval 1; 2 . We
  ffiffiffi
have that f ðaÞ  maxpffiffi f ðxÞ ¼ max f ð1Þ; f 2 .
½1; 2

312 7 Miscellaneous Inequalities

Figure 7.11

M А
N K
O

pffiffiffi
Note that, f(1) ¼ 4b2  6b¼ 4b(b   1, 5) < 0, as 1  b  2 < 1, 5 and
pffiffiffi pffiffiffi pffiffi  pffiffiffi pffiffi pffiffiffi
f 2 ¼ 4b2  6 2b þ 4 ¼ 4 b  22 b  2  0, as 22 < 1  b  2.
Thus, f(a) ¼ 4a2  6ab þ 4b2  4  0.
We have
 obtained that angle α is, such that cos α  34 ¼ cos α0 , where
π
α0 2 0; 2 . We have that cos 3α0 ¼ 4cos 3 α0  3 cos α0 ¼ 16 9
< 12. Hence,

3α0 > 3 . Thus, α  α0 > 9 , it follows that the 360 -layout of segments connecting
2π 2π

the centers of the adjoining squares with point O cannot contain such nine angles α

9 ¼ 40 . Therefore, the greatest number of the adjoining squares is
greater than 2π
equal to eight.
7.1.13. Since the triangle is acute, then a2, b2, c2 are the sides of some triangle and
consequently, there exist numbers m, n, k > 0 such that a2 ¼ m þ n, b2 ¼ n þ k, c2
¼ k þ m. It remains to prove that (4m þ n þ k)(4n þ k þ m)(4k þ m þ n) > 25
(m þ n)(n þ k)(k þ m), or 12mnk þ m3 þ n3 þ k3 > m2k þ k2m þ n2m þ m2n þ k2
n þ n2k.
We need to prove that 3mnk þ m3 þ n3 þ k3  m2k þ k2m þ n2m þ m2n þ k2n þ n
2
k, or mnk  (m þ n  k)(k þ m  n)(n þ k  m).
If m, n . , k are not the sides of some triangle, then mnk > 0  (m þ n  k)
(k þ m  n)(n þ k  m).
If m, n . , k are the sides of some triangle, then

ðm þ n  kÞðk þ m  nÞðn þ k  mÞ ¼
pffiffiffiffiffiffiffiffiffiffiffiffiffiffiffiffiffiffiffiffiffiffiffiffiffiffiffiffiffiffiffiffiffiffiffiffiffiffiffiffiffiffiffiffiffiffiffi pffiffiffiffiffiffiffiffiffiffiffiffiffiffiffiffiffiffiffiffiffiffiffiffiffiffiffiffiffiffiffiffiffiffiffiffiffiffiffiffiffiffiffiffiffiffiffi pffiffiffiffiffiffiffiffiffiffiffiffiffiffiffiffiffiffiffiffiffiffiffiffiffiffiffiffiffiffiffiffiffiffiffiffiffiffiffiffiffiffiffiffiffiffiffi
¼ ðm þ n  kÞðk þ m  nÞ  ðk þ m  nÞðn þ k  mÞ  ðm þ n  kÞðn þ k  mÞ 
mþnkþkþmn kþmnþnþkm mþnkþnþkm
   ¼ mnk:
2 2 2

7.1.14. (a), (b) Let max(∠AMB, ∠BMC, ∠AMC) ¼ ∠BMC and min
(∠AMB, ∠BMC, ∠AMC) ¼ ∠AMB. As ∠AMB þ ∠BMC þ ∠AMC ¼ 360 , then
∠BMC  120 , ∠AMB  120 . Consider parallelograms CMBA2 and AMBC2 (see
Figure 7.12).
Note that ∠MBA2  60 . Thus, 2MA1 ¼ MA2  max (MB, BA2) ¼ max (MB, MC).
Hence, 2 min (MA1, MB1, MC1)  2MA1  max (MB, MC)  max (MA, MB, MC).
Therefore, 2 min (MA1, MB1, MC1)  max (MA, MB, MC).
As ∠AMB  120 , then ∠MBC2  60 . Therefore, we deduce that
2MC1 ¼ MC2  min (MB, BC2) ¼ min (MB, MA). Hence, it follows that 2 max
(MA1, MB1, MC1)  2MC1  min (MB, MA)  min (MA, MB, MC). Thus, 2 max
(MA1, MB1, MC1)  min (MA, MB, MC).
7.2 Solutions 313

Figure 7.12 B

M
C2 C1 A1 A2

A C

(c) Suppose that ∠MAB þ ∠MBC þ ∠MCA  min (∠A, ∠B, ∠C), then ∠MAB <
∠B  ∠MBC ¼ ∠MBA, hence MB < MA. Similarly, we obtain that
∠MBC < ∠C  ∠MCA ¼ ∠MCB. Thus, MC < MB, and ∠MCA <
∠A  ∠MAB ¼ ∠MAC. Hence, MA < MC.
Thus, we have obtained that MA < MC < MB < MA. This leads to a contradic-
tion. Hence, ∠MAB þ ∠MBC þ ∠MCA > min (∠A, ∠B, ∠C). Therefore, it fol-
lows that ∠A  ∠MAB þ ∠B  ∠MBC þ ∠C  ∠MCA > min (∠A,∠B,∠C).
Hence, ∠MAB þ ∠MBC þ ∠MCA < π  min (∠A,∠B,∠C).
(d) Let us draw the diagonals AC and BD dividing the quadrilateral into four triangles
AOB, BOC, COD, DOA. Point M is in one of these triangles. (If point M is on the
common border of two triangles, then one can consider it being in both triangles.)
If point M is in triangle COD, then according to problem 7.1.14c, we have that
∠MBC þ ∠MCD þ∠MDB > min (∠BDC,∠DBC) ¼ φ, where φ ¼ min (∠BAC,
∠DAC). Consequently, we deduce that

∠MAB þ ∠MBC þ ∠MCD þ ∠MDA  ∠OAB þ ∠MBC þ ∠MCD þ ∠MDB þ ∠ODA ¼


π π
¼ þ ∠MBC þ ∠MCD þ ∠MDB > þ φ:
2 2

If point M is in triangle AOD, then by making a change of variables A ! D,


B ! A, C ! B, D ! C, we obtain that M 2 ΔCOD. This ends the proof as min
(∠BAC,∠DAC) ¼ min (∠ABD,∠CBD).
The proof for the other cases can be done similarly.
 2  2
7.1.15. Let max(AB, BC, AC) ¼ AC, then AB2 þ BC2  p ACffiffi
2
þ p ACffiffi
2
¼ AC2 .
Thus, ABC is a non-obtuse triangle. We need to prove that the inequality holds
true for the vertices of triangle ABC.
For vertex C, we have to prove that a2 þ b2  h2c þ a2 cos 2 γ þ b2 cos 2 γ; this is
ða sin γ Þ2 þ ðb sin γ Þ2  h2c , or h2a þ h2b  h2c (let min(AB, BC, AC) ¼ AB).
2
4S2 4S2 4S2
Indeed, we have that h2a þ h2b ¼ 4S a2 þ b2  2c2 þ 2c2  hc .
2

It remains to note that, if M(x; y) satisfies the condition


MA2 þ MB2 þ MC2  MA21 þ MB21 þ MC21 , then it is equivalent to that the num-
bers x and y satisfy the inequality a1x þ b1y þ c1  0 for some constant numbers a1,
b1, c1. Thus, the inequality holds true for any points inside triangle ABC.
314 7 Miscellaneous Inequalities

7.1.16. (a) We have that

MB1 þ MC1 ¼ MAð sin ∠MAC þ sin ∠MABÞ


α ∠MAC  ∠MAB
¼ 2MA  sin cos >
2 2 pffiffiffi
α α 2
> 2MA  sin cos ¼ MA  sin α  MA,
2 2 2
pffiffi
since 90  α  45 . We have
pffiffi
obtained that MB 1 þ MC 1
pffiffi
> 2
2 MA. Similarly, we
deduce that MB1 þ MA1 > 2 MC and MA1 þ MC1 > 2 MB. Summing up these
2 2

three inequalities, we deduce that


pffiffiffi
MA þ MB þ MC < 2 2ðMA1 þ MB1 þ MC1 Þ:

(b) Note that, if M  A, then



AA1 AA1 1 1
MA þ MB þ MC ¼ AB þ AC ¼ þ ¼ MA1 þ 
sin β sin γ sin β sin γ

1 1 1 1 1 1
 max þ ; þ ; þ  MA1 ¼
sin α sin β sin β sin γ sin γ sin α

1 1 1 1 1 1
¼ max þ ; þ ; þ  ðMA1 þ MB1 þ MC1 Þ:
sin α sin β sin β sin γ sin γ sin α
ð7:10Þ

Similarly, one can prove the inequality (7.10) for vertices B and C.
If points M1 and M2 satisfy the condition (7.10) and M 2 [M1M2], with MMM 1
1 M2
¼λ
(0 < λ < 1), then we have that

! 
!
! 
AM ¼  AM  ¼ λAM 2 þ ð1  λÞAM 1   λAM2 þ ð1  λÞAM1 ð7:11Þ

and ρ(M, BC) ¼ ρ(M2, BC)  λ þ (1  λ)  ρ(M1, BC).


As, at least for one of the vertices of ABC the condition (7.11) is an inequality,
then

AM þ BM þ CM  λðAM2 þ BM2 þ CM2 Þ þ ð1  λÞðAM1 þ BM1 þ CM1 Þ 



1 1 1 1 1 1 
 max þ ; þ ; þ ρðM2 ;BCÞ  λ þ ð1  λÞ  ρðM1 ;BCÞþ
sin α sin β sin β sin γ sin γ sin α

þρðM2 ; ABÞ  λ þ ð1  λÞ  ρðM1 ;ABÞ þ ρðM2 ;ACÞ  λ þ ð1  λÞ  ρðM1 ;ACÞ ¼

1 1 1 1 1 1
¼ max þ ; þ ; þ ðρðM;BCÞ þ ρðM; ABÞ þ ρðM;ACÞÞ:
sin α sin β sin β sin γ sin γ sin α

We have obtained that the inequality holds true for point M.


This ends the proof.
7.2 Solutions 315

7.1.17. Given a point M inside of the tetrahedron ABCD, we have to prove that
MA þ MB þ MC þ MD < AB þ BC þ CD þ AC þ AD þ BD ¼ p.
Denote by f(M ) ¼ MA þ MB þ MC þ MD. Let us prove that set of points M, such
that f(M ) < p, is a convex figure.
Indeed, let f(M1) < p, f(M2) < p, M 2 [M1M2] and MMM 1
1 M2
¼ λ. Then, f(M ) < λf
(M2) þ (1  λ)f(M1) < λp þ (1  λ)p ¼ p (see the proof of problem 7.1.16b). Note
that, f(A) ¼ AB þ AC þ AD < p and similarly f(B) < p, f(C) < p, f(D) < p. Thus, for
any point M of the tetrahedron f(M ) < p.
See also the proof of problem 7.1.38e.
7.1.18. (a) Let the distance a between pairs of points A1, A2, . . . , A7 on the plane
occur at most k times. Denote by ni the number of the segments with lengths a,
drawn from point Ai. Without loss of generality, one can assume that
n1  n 2  . . .  n 7.
Note that n1 þ n2  9; otherwise on the circles with centers A1 and A2 and radius
a will be located at least n1 þ (n2  2)  8 of seven points. This leads to a
contradiction.
It is clear that the distance between any two of points A1, A2, A3, A4 is not equal to
a. Let AiAj 6¼ a (i 6¼ j). In that case, on the circles with centers Ai, Aj and radius a are
located at least ni þ nj  2 points. Since, Ai, Aj are not on those circles,
ni þ nj  2 þ 2  7. Hence, n3 þ n4  ni þ nj  7. Thus, we deduce that n4  3. Con-
sequently, n7  n6  n5  3.
We have that k ¼ 12 ððn1 þ n2 Þ þ ðn3 þ n4 Þ þ n5 þ n6 þ n7 Þ  12 ð9 þ 7 þ 3þ
3 þ 3Þ. Hence it follows that k  12.
In a regular hexagon with a side a, the distance a for the vertices and center
occurs 12 times. Thus, k ¼ 12.
(b) In a similar way, as for the proof of problem 7.1.18a, one can obtain that
k  12 ð8 þ 6 þ 3 þ 3Þ ¼ 10. However, it is easy to check that the cases n1 ¼ 5,
n2 ¼ n3 ¼ . . . ¼ n6 ¼ 3 and n1 ¼ n2 ¼ 4, n3 ¼ . . . ¼ n6 ¼ 3 are not possible.
(In the last case for points Ai, the only point not located on the circle with a
radius a and center A1, ni  2).
Answer 9. The example is presented in Figure 7.13.
7.1.19. Let in a quadrilateral ABCD we have that AB ¼ a, BC ¼ b, CD ¼ c, DA ¼ d,
the midline KM ¼ m and the distance between the midpoints of the diagonals
PQ ¼ x. As PKQM is a parallelogram with sides a/2 and c/2 (K is on BC, M is on
2 2
AD), then m2 ¼ KM2 ¼ 2PK 2 þ 2KQ2  PQ2 ¼ a2 þ c2  x2 .

Figure 7.13
316 7 Miscellaneous Inequalities

Figure 7.14 M

D D⬘
C⬘
C
A B

2 2
Similarly, if n is the second midline, then n2 ¼ b2 þ d2  x2 . Thus, each midline
accepts its greatest value when x has the smallest value. Therefore, in that case the
sum of the midlines also accepts the greatest value.  
 However,  by the triangle inequality, PQ ¼ x  12 c  a. Similarly, x 
1 
2 db .
For the simplicity, assume that |d  b|  |c   a|. In this case, the greatest value of
the sum of the midlines is reached if, x ¼ 12 d  b, this means that the required
quadrilateral is a trapezoid with bases b,d and lateral sides a,c (or a parallelogram).
2 þ
bþd
qThe sum of midlines
ffiffiffiffiffiffiffiffiffiffiffiffiffiffiffiffiffiffiffiffiffiffiffiffiffiffiffiffiffiffiffiffiffiffiffiffiffiffiffiffiffi
ffi of such trapezoid is equal to
1
2 2a2 þ 2c2  ðd  bÞ2 .

7.1.20. (a) Let us consider two cases.


1. If max (AB, BC, CD, DA, AC, BD) ¼ max (AB, BC, CD, DA).
Let max (AB, BC, CD, DA) ¼ AB, then one can assume that points C and D are
inside of the figure AMB (Figure 7.14).
In the case, if ray BD intersects the arc BM, we have that the quadrilateral
ABCD is inside of the quadrilateral ABC0 D0 . Hence, it follows that AB þ BC þ
CD þ DA  AB þ BC0 þ C0 D0 þ D0 A (see the problem 2.1.1).
Let AB ¼ R, ∠C0 AD0 ¼ 2α and ∠C0 AB ¼ 2β, then 2α þ 2β  π3 and C0 D0 þ
C0 B ¼ 2R(sin α þ sin β)4R sin αþβ π
2  4R sin 12. Therefore, AB þ BC þ CD þ
αþβ π
DA  2R þ 4R sin 2  2 þ 4 sin 12 . Similarly, one can consider the case
when ray AC interests the arc AM.
It remains to consider the case presented in Figure 7.15.
Since ∠BAD0  π3 and ∠ABC0  π3, then minð∠AD0 C0 ; ∠BC0 D0 Þ  2π 3.
Let ∠AD0 C0  2π 3 . We have that AB þ BC þ CD þ DA  AB þ BC 0
þ C0 D0 þ
0 0 0 0 0
D A. If we prove that AD þ D C  AM þ MC , then

AB þ BC þ CD þ DA  AB þ BC0 þ C0 D0 þ D0 A  AB þ BC0 þ C0 M þ MA 
π π
 2R þ 4R sin  2 þ 4 sin :
12 12
0 0
Indeed, since ∠AD0 M ¼ 5π 0 0 0
6 and ∠AD C  3 , then ∠AD K  ∠C D M.

7.2 Solutions 317

Figure 7.15 T
M

D⬘ C⬘
D C
K

A B

Figure 7.16 C⬘
A

K N D⬘ M T

A⬘

While ∠AMK < π6 < ∠C0 MT, as ∠AMC0 ¼ ∠AC0 M < π2.
We have that ∠AD0 K > ∠AMK.
Let A0 be the point symmetric to A with respect to line KT (Figure 7.16).
Then from the conditions ∠AD0 K  ∠C0 D0 T and ∠AMK < ∠C0 MT it follows
that, points N, D0 , M are on line KT, as it is shown in Figure 7.15. According to
problem 2.1.1, we have that A0 C0 þ A0 D0 þ C0 D0  A0 C0 þ A0 M þ MC0 . Hence,
we obtain that AD0 þ C0 D0  AM þ MC0 .
2. If max(AB, BC, CD, DA, AC, BD) > max (AB, BC, CD, DA).
Let max(AB, BC, CD, DA, AC, BD) ¼ AC and l be the perpendicular bisector
of segment AC. One can assume that points B and D are in the same half-plane
with the boundary l (see Figure 7.17a).
Indeed, otherwise we have for the quadrilateral ABCD0 that AD0 ¼ CD,
CD0 ¼ AD and BD0 < BN þ ND0 ¼ BD.
Now, let l1 || AC, l2 || AC and l3 || BD (see Figure 7.17b) and points B0 and D0

!0
!0
are, such that B0 2 l1, D0 2 l2, BB ¼ DD and max(AB0 , B0 C, CD0 , D0 A) ¼ AC.
Since max(CD0, CB0) > AC, then we can assume that D0 2 [DD0].
Now it is clear that the quadrilateral AB0 CD0 is convex and that max(AB0 , B0 C, CD0 ,
D0 A, AC, B0 D0 ) ¼ max (AB0 , B0 C, CD0 , D0 A) ¼ AC  1. Moreover, AB þ BC þ
0 0
CD þ DA  AB0 þ B0 C þ CD0 þ D0 A. According to the case (a), it follows AB þ B
0 0
π π
Cþ CD þ D A  2 þ 4 sin 12 . Hence, AB þ BC þ CD þ DA  2þ 4 sin 12 .
This solution was proposed by D. Harutyunyan and A. Kalantaryan, Ninth grade.
(b) Let, apart from BD, all other distances are not greater than 1. According to
pffiffiffiffiffiffiffiffiffiffiffiffiffi pffiffiffiffiffiffiffiffiffiffiffiffiffi
problem 4.1.3, if AC ¼ c, then BD  4  c2 . Hence AC þ BD  c þ 4  c2 .
π
pffiffiffi c ¼2 sin πα,
Let
 pffiffiffi 0π απ 6. Then
where pffiffiffi AC þ BD  2 sin α þ 2 cos α ¼
2 2 sin α þ 4  2 2 sin 6 þ 4 ¼ 1 þ 3. pffiffiffi
Thus, we obtain that AB þ BC þ AD þ CD þ AC þ BD  5 þ 3.
318 7 Miscellaneous Inequalities

l l3 l
B B0 B¢ B l1

A C A C

l2
D¢ D D0 D¢ D

a b
Figure 7.17

(c) Let in a tetrahedron SABC we have that SA, SB, AB, AC, BC  1 and AB ¼ a.
Draw the altitudes SE and CF of triangles A SB and A CB, respectively. Since
pffiffiffiffiffiffiffiffiffiffiffiffiffiffiffiffiffiffiffiffiffiffi
AF þ FB  a, then AF  a
or FB  a
. Let AF  a
, then CF ¼ AC 2
 AF2
pffiffiffiffiffiffiffiffiffiffiffiffiffiffiffiffiffi qffiffiffiffiffiffiffiffiffiffiffi2ffi 2 2 2 qffiffiffiffiffiffiffiffiffiffiffiffi
 1  AF2  1  a4 . Similarly, we deduce that SE  1  a4 .
2

We have that V ¼ 13 SABC SH, where SH is the altitude


 of the tetrahedron SABC.
2
Hence, V ¼ 13  ABCF
2  SH  6 CF  SE  6 a 1  4
a 1 a
 18, as a3  4a þ 3 ¼ (a  1)
(a2 þ a  3)  0 (a  1, a2 þ a  3  2  3 < 0).
Note that if SB ¼ SA ¼ AB ¼ AC ¼ BC ¼ 1 and (SAB) ⊥ (ABC), then V ¼ 18.
7.1.21. If line l does not contain any of the given points, then one can move it
parallel to itself, so that the ρ(l ) does not increase and it approaches to one of the
given points. Draw a straight line through one of the given points and start turning it
around that point. Let φ be the turning angle, 0  φ < π. Now, if φk, k ¼ 1, . . . , m
(m  n  1), be the values of φ, at which the line passes (sequentially) through
the remaining points of our set, then ρ(l ) can be expressed as ρðlÞ ¼
Pm  
ak  sin ðφ  φk Þ, ak > 0.
k¼1
But on each of the intervals [φk, φk þ 1] the considered function, being a sum of
convex (upward) functions, is itself a convex function, consequently, cannot reach
the minimal value inside that interval.
7.1.22. From triangles AND, BNC, ABN, CND, according to the law of cosines, we
obtain that

AD2 þ BC2  AB2  CD2 ¼ AN 2 þ ND2  2AN  ND  cos αþ


þ BN 2 þ CN 2  2BN  CN  cos α  AN 2  BN 2
 2AN  BN  cos α  CN 2  ND2 
 2CN  ND  cos α ¼ 2AC  BD  cos α,

where α ¼ ∠AND.
7.2 Solutions 319

Figure 7.18 M

B B1 C

B2 C2
А1 N
C1

A2 D2

A D1 D

Since AD þ BC ¼ AB þ CD, then

2AC  BD  cos α ¼
¼ AD2 þ BC2  AB2  CD2 ¼
¼ 2ðAD  ABÞ  ðAD  CDÞ  0:

Therefore, ∠AND ¼ α  90 .


We need to prove that the center O of the incircle of the quadrilateral ABCD is
_ _
inside of triangle AND. Note that DD1  BB1, hence D1 D2 C1  A1 B2 B1 .
_ _
Thus, A2 D2 C2  A2 B2 C2 (see Figure 7.18).
_ _
Similarly, we deduce that B2 A2 D2  B2 C2 D2 . Consequently, point O is inside
of triangle AND. Hence, ∠AND  ∠AOD ¼ 90 þ 12 ∠AMD.
7.1.23. Denote the midpoints of segments AB and CD by M and N, respectively. We
need to prove that AB2 þ CD2 þ 2MN2 < 6. Let A0 , B0 , C0 , D0 , M0 , N0 be the points,
symmetric to points A, B, C, D, M, N with respect to the center of the cube. Consider
the parallelograms D0 CDC0 , ABA0 B0 and MNM0 N0 . Since in a parallelogram the sum
of the squares of the adjacent sides does not exceed the square of the largest diagonal,
0 0 0 0
then CD2 þ NN 2 < 3 and AB2 þ MM 2 < 3. Therefore, CD2 þ AB2 þ NN 2 þ MM 2
0 2
< 6. Thus, CD þ AB þ 2MN þ 2M N < 6.
2 2 2

Hence, CD2 þ AB2 þ 2MN2 < 6. Since, for x, y, z  0, we have that


pffiffiffiffiffiffiffi 1 p ffiffiffi p ffiffiffi pffiffip ffiffiffi p ffiffiffi2 pffiffiffi pffiffi2 pffiffi pffiffiffi2 
x þ y þ z  3 3 xyz ¼ 3
xþ 3 yþ 3 z 3
x 3 y þ 3 y 3 z þ 3 z 3 x  0:
2
Thus, we deduce that
pffiffiffiffiffiffiffi
x þ y þ z  3 3 xyz: ð7:12Þ
320 7 Miscellaneous Inequalities

p According to the inequality (7.12), it follows CD2 þ AB2 þ 2MN 2 


ffiffiffiffiffiffiffiffiffiffiffiffiffiffiffiffiffiffiffiffiffiffiffiffiffiffiffiffiffiffiffiffiffiffiffi
3 CD  AB  2MN 2 :
3 2 2
p ffiffiffiffiffiffiffiffiffiffiffiffiffiffiffiffiffiffiffiffiffiffiffiffiffiffiffiffiffiffiffiffiffiffiffi p ffiffiffiffiffiffiffiffiffiffiffiffiffiffiffiffiffiffiffiffiffiffiffiffiffiffiffi
Hence, we obtain that 6 > 3 CD2  AB2  2MN 2  3 2AB2  CD2 d 2 .
3 3

Therefore, AB  CD  d < 2. This ends the proof.


7.1.24. Given a tetrahedron ABCD inside of the parallelepiped with the volume V.
Draw through point A a line l not parallel to plane BCD. Let A1 and A2 be the
intersection points of the straight line l with faces of the parallelepiped. Then,
V ABCD < maxðV A1 BCD ; V A2 BCD Þ.
Let maxðV A1 BCD ; V A2 BCD Þ ¼ V A1 BCD . Let us draw through point A1 a line
intersecting the edges of the parallelepiped at points A3 and A4. Therefore,
V A1 BCD  maxðV A3 BCD ; V A4 BCD Þ ¼ V A3 BCD . If point A3 belongs to the edge A5A6
of the parallelepiped, then V A3 BCD  maxðV A5 BCD ; V A6 BCD Þ. Thus, we have proven
that there exists a vertex A0 of the parallelepiped, such that V ABCD < V A0 BCD .
Similarly, one can prove that there exist vertices B0, C0, D0 of the parallelepiped,
such that

V ABCD < V A0 BCD  V A0 B0 CD  V A0 B0 C0 D  V A0 B0 C0 D0 :

It remains to note that V A0 B0 C0 D0 ¼ V3 or V6 .


7.1.25. Let A1 and C1 be the midpoints of sides BC and AB, respectively. Note that
point K belongs to segment A1C; otherwise we deduce that SBMK < SABA1 ¼ 12 SABC .
This leads to a contradiction. Consequently, BK  BC 2.
Similarly, one can prove that BM  AB2 . Hence, it follows that

MB þ BK AB
þ BC ABþBC
1
 AB 2 2
> ABþBC 2 ¼ :
AM þ CA þ KC 2 þ CA þ BC
2 2 þ AB þ BC 3

7.1.26. Let ABC be a triangular section of the cube with the edge 2 and the center D
(1;1;1) (see Figure 7.19).

Figure 7.19 z

О B
A y

x
7.2 Solutions 321

Denote by OA ¼ a, OB ¼ b, OC ¼ c, then the equation of plane ABC is


x
aþ by þ cz ¼ 1, аnd ρ(D, ABC) ¼ 1. It is known that S2ABC ¼ S2AOB þ S2BOC þ S2AOC
j1aþffiffiffiffiffiffiffiffiffiffiffiffiffi
bþ c1j
1 1
and ρðD; ABCÞ ¼ p .
1
þ 2þ
1 1
a2 b c2

We have to prove that, if 0 < a, b, c  2 and 2(a þ b þ c)  2(ab þ bc þ ca) þ


abc ¼ 0, then a2b2 þ b2c2 þ c2a2 < 16.
Note that

a2 b2 þ b 2 c 2 þ c 2 a2 ¼  2
¼ ðab þ bc þ caÞ2  2abcða þ b þ cÞ ¼ a þ b þ c þ abc2  2abcða þ b þ cÞ ¼
 
abc 2
¼ aþbþc 2 :
 
Hence, we have to prove that a þ b þ c  abc 
2 < 4.
We have that a þ b þ c  2  3a  2 > 0 (let min(a, b, c) ¼ a).
abc abc

Since 8  2(a þ b þ c) ¼ (2  a)(2  b)(2  c)  0, then a þ b þ c  4. Hence,


a þ b þ c  abc 2 < 4. Note that the area of the total surface of the tetrahedron
OABC is equal to 12 a þ b þ c  abc 2 þ 2 þ 2 þ 2 ¼ a þ b þ c  4.
ab bc ac

Thus, we have obtained that the area of total surface of the tetrahedron OABC is
not greater than the area of the face of the cube.
7.1.27. (а) Let α  60 , then according to problem 3.1.6a
α 1 1
SABC  l2a tg  pffiffiffi l2a > pffiffiffi :
2 3 3

(b) Let minða; b; cÞ ¼ c < p2ffiffi3, then SABC ¼ ch2c  cl2c < p1ffiffi3.

If minða; b; cÞ  p2ffiffi3 and max(a, b, c) ¼ a, then α  60 . Let point B1 on side BC


be such that ∠BAB1 ¼ 60 . Then ρ(B, AB1) ¼ AB  sin 60  1. Since lb  ρ(B,
AB1)  1, then lb  1. This leads to a contradiction.
(c) See the proof of problem 7.1.27b.
pffiffiffi 2 pffiffiffi
pffiffiffiAnswer
7.1.28. 2 2  1 . Let a ¼ b and c ¼ 2a þ 1n, where n 2 N and
 
2  2 a > 1n. Then for n ! 1, from the inequality a2 þ b2 þ c2 > k(a þ b þ c)2,
pffiffiffi 2
we deduce that k  2 2  1 .
pffiffiffi 2
We need to prove that a2 þ b2 þ c2 > 2 2  1 ða þ b þ cÞ2 .
pffiffiffi 2
Let max(a,
pffiffiffi b, c) ¼ c, a 2
þ b 2
¼ 1 and k 0 ¼ 2 2  1 , then c > 1 and
a þ b  2. Consider a function f(x) ¼ apffiffiþ b þ x  k0(a þ b þ x)2 on the interval
2 2 2

(1, þ1). Note that xb ¼ 1k k0


0
ða þ bÞ  1k
2k 0
0
< 1. Hence, it follows that
pffiffiffi 2
f ðcÞ > f ð1Þ ¼ 2  k0 ða þ b þ 1Þ2  2  k0 2 þ 1 ¼ 0:
pffiffiffi 2
Thus, a2 þ b2 þ c2 > 2 2  1 ða þ b þ cÞ2 .
322 7 Miscellaneous Inequalities

7.1.29. (a) From the condition a þ b þ c ¼ 1, according to the triangle inequality,


we deduce that a, b, c < 12. Denote by a ¼ 12  k, then k > 0 and c ¼ 12 þ k  b . We
have to prove that 2k(k  b)(1  2b) < 0. Note that c ¼ 12 þ k  b < 12, consequently
k  b < 0.
(b) Note that
     
a 2a2  b2  c2 þ b 2b2  a2  c2 þ c 2c2  a2  b2 ¼
         
¼ a a2  b2 þ aða2  c2 Þ þ b b2  a2 þ b b2  c2 þ c c2  a2 þ c c2  b2 ¼
¼ ða þ bÞða  bÞ2 þ ða þ cÞða  cÞ2 þ ðb þ cÞðb  cÞ2  0
Hence, a(2a2  b2  c2) þ b(2b2  a2  c2) þ c(2c2  a2  b2)  0.
Remark The inequality holds true, only if a, b, c  0.
(c)Denote by a ¼ m þ n, b ¼ n þ k, c ¼ m þ k. Then, m ¼ p  b, n ¼ p  c, k ¼ p  a,
hence m, n, k > 0. We have that

a2 bða  bÞ þ b2 cðb  cÞ þ c2 aðc  aÞ


¼ km3 þ mn3 þ nk3  k2 mn  m2 nk  n2 mk ¼
¼ kmðm  nÞ2 þ mnðn  kÞ2 þ nkðk  mÞ2  0

Consequently, a2b(a  b) þ b2c(b  c) þ c2a(c  a)  0.


(d) Note that
pffiffiffi pffiffiffi pffiffiffi
aða þ c  bÞ þ bða þ b  cÞ þ cðb þ c  aÞ ¼
pffiffiffiffiffiffiffiffiffiffiffiffiffiffiffiffiffiffiffiffiffiffiffiffi pffiffiffiffiffiffiffiffiffiffiffiffiffiffiffiffiffi pffiffiffiffiffiffiffiffiffiffiffiffiffiffiffiffiffiffiffiffiffiffiffiffi pffiffiffiffiffiffiffiffiffiffiffiffiffiffiffiffiffi pffiffiffiffiffiffiffiffiffiffiffiffiffiffiffiffiffiffiffiffiffiffiffi pffiffiffiffiffiffiffiffiffiffiffiffiffiffiffiffiffi
¼ aða þ c  bÞ  a þ c  b þ bða þ b  cÞ  a þ b  c þ cðb þ c  aÞ  b þ c  a ¼


!
pffiffiffiffiffiffiffiffiffiffiffiffiffiffiffiffiffiffiffiffiffiffiffiffi pffiffiffiffiffiffiffiffiffiffiffiffiffiffiffiffiffiffiffiffiffiffiffiffi pffiffiffiffiffiffiffiffiffiffiffiffiffiffiffiffiffiffiffiffiffiffiffi  

!
pffiffiffiffiffiffiffiffiffiffiffiffiffiffiffiffiffi pffiffiffiffiffiffiffiffiffiffiffiffiffiffiffiffiffi pffiffiffiffiffiffiffiffiffiffiffiffiffiffiffiffiffi
¼ aða þ c  bÞ; bða þ b  cÞ; cðb þ c  aÞ  a þ c  b; a þ b  c; b þ c  a 



!  

! 
 pffiffiffiffiffiffiffiffiffiffiffiffiffiffiffiffiffiffiffiffiffiffiffiffi pffiffiffiffiffiffiffiffiffiffiffiffiffiffiffiffiffiffiffiffiffiffiffiffi pffiffiffiffiffiffiffiffiffiffiffiffiffiffiffiffiffiffiffiffiffiffiffi   pffiffiffiffiffiffiffiffiffiffiffiffiffiffiffiffiffi pffiffiffiffiffiffiffiffiffiffiffiffiffiffiffiffiffi pffiffiffiffiffiffiffiffiffiffiffiffiffiffiffiffiffi 

 aða þ c  bÞ; bða þ b  cÞ; cðb þ c  aÞ    a þ c  b; a þ b  c; b þ c  a  ¼
 
pffiffiffiffiffiffiffiffiffiffiffiffiffiffiffiffiffiffiffiffiffiffiffiffiffiffiffiffiffiffiffiffiffiffiffiffiffiffiffiffiffiffiffiffiffiffiffiffiffiffiffiffiffiffiffiffiffiffiffiffiffiffiffiffiffiffiffiffiffiffiffiffiffiffiffiffiffiffiffi pffiffiffiffiffiffiffiffiffiffiffiffiffiffiffiffiffiffiffiffiffiffiffiffiffiffiffiffiffiffiffiffiffiffiffiffiffiffiffiffiffiffiffiffiffiffiffiffiffiffiffiffiffiffiffiffiffiffiffiffiffi
¼ aða þ c  bÞ þ bða þ b  cÞ þ cðb þ c  aÞ  a þ c  b þ a þ b  c þ b þ c  a ¼
qffiffiffiffiffiffiffiffiffiffiffiffiffiffiffiffiffiffiffiffiffiffiffiffiffiffiffiffiffiffiffiffiffiffiffiffiffiffiffiffiffiffiffiffiffiffiffiffi
 
¼ ða þ b þ cÞ a2 þ b2 þ c2 :
pffiffiffiffiffiffiffiffiffiffiffiffiffiffiffiffiffiffiffi pffiffiffiffiffiffiffiffiffiffiffiffiffiffiffiffiffiffiffi qffiffiffiffiffiffiffiffiffiffiffiffiffiffiffiffiffiffiffiffiffiffiffi pffiffiffi
(e) Note that a þ b  c þ b þ c  a  2 aþbcþbþca 2 ¼ 2 b. Similarly, we
pffiffiffiffiffiffiffiffiffiffiffiffiffiffiffiffiffiffiffi pffiffiffiffiffiffiffiffiffiffiffiffiffiffiffiffiffiffiffi pffiffiffi pffiffiffiffiffiffiffiffiffiffiffiffiffiffiffiffiffiffiffi
obtain that aþbcþ aþcb2 a and b þ c  aþ
pffiffiffiffiffiffiffiffiffiffiffiffiffiffiffiffiffiffiffi pffiffiffi pffiffiffiffiffiffiffiffiffiffiffiffiffiffiffiffiffiffiffi
a þ c  b  2 c. Summing up these three inequalities, we get a þ b  cþ
pffiffiffiffiffiffiffiffiffiffiffiffiffiffiffiffiffiffiffi pffiffiffiffiffiffiffiffiffiffiffiffiffiffiffiffiffiffiffi pffiffiffi pffiffiffi pffiffiffi
b þ c  a þ a þ c  b  a þ b þ c.
 2  2  2
(f) According to problem 7.1.29a, a2 þ b2 þ 2c þ 4a2  b2  2c < 12, it follows that
a2 þ b2 þ c2 þ 2abc < 2.
(g) Let max(a, b, c) ¼ b, we have to prove that f(b) ¼ (2a  c)b2 þ (2c2  a2  3ac)
b þ ac(2a  c)  0.
7.2 Solutions 323

Note that f(a) ¼ a(a  c)2  0, f(c) ¼ c(a  c)2  0 and f(a þ c) ¼ a3 þ c(a  c)2
> 0.
If 2a  c  0, f(b)  min ( f(c), f(a þ c))  0, as c  b < a þ c.
If 2a  c > 0 and c  a, we have that bϐep ¼ a 2þ3ac2c
2 2
ð2acÞ  c.
Hence, f(b)  f(c)  0.
If a  c > 0, we have that bϐep ¼ a 2þ3ac2c
2 2
ð2acÞ < a. Hence, f(b)  f(a)  0.

Second Solution Using the notations a ¼ m þ n, b ¼ n þ k, c ¼ m þ k, the given


inequality can be rewritten in the form m3 þ n3 þ k3 þ m2n þ k2m þ n2k  2(m2k þ
n2m þ k2n). The last inequality holds true, since m3 þ k2m  2m2k, n3 þ m2n 
2n2m, k3 þ n2k  2k2n.
 
(h) Note that ab þ bc þ ac  ac  bc  ba ¼ jabj jbcj jcaj
c  a  b < 1, since ja  bj < c,
jb  cj < a, jc  aj < b.
(i) Note that one of the numbers pq, qr, pr is non-negative. Let pq  0, then
 
a2 pq þ b2 qr þ c2 rp ¼ a2 pq  b2 q þ c2 p ðp þ qÞ ¼
¼ ðb þ c  aÞðb þ c þ aÞpq  ðbq  cpÞ2  0:

(j) Note that

a2 ð2b þ 2c  aÞ þ b2 ð2c þ 2a  bÞ þ c2 ð2a þ 2b  cÞ ¼


     
¼ b 2a2 þ 2c2  b2 þ a 2b2 þ 2c2  a2 þ c 2a2 þ 2b2  c2 ¼ 4m2a a þ 4m2b b þ 4m2c c ¼
   2  2 
2
¼ 9 23ma a þ 23mb b þ 23mc c  9abc

(see problem 4.1.8b).


(k) Note that

ða þ b þ cÞ2 ¼ a2 þ b2 þ c2 þ 2ab þ 2bc þ 2ac <


< aðb þ cÞ þ bða þ cÞ þ cða þ bÞ þ 2ab þ 2bc þ 2ac
¼ 4ab þ 4bc þ 4ac

Hence abþbcþac
ðaþbþcÞ2
> 14.
(l) Note that

ða þ b þ cÞ3 ¼ ða þ bÞ3 þ 3ða þ bÞ2 c þ 3ða þ bÞc2 þ c3 <


< ða þ bÞ3 þ 3ða þ bÞ2 c þ 3ða þ bÞc2
 
þ ða þ bÞc2 ¼ ða þ bÞ ða þ bÞ2 þ 3ða þ bÞc þ 4c2 ¼
 
¼ ða þ bÞ ða  bÞ2 þ 4ab þ 3ða þ bÞc þ 4c2 :
324 7 Miscellaneous Inequalities

Since |a  b| < c and |a  b| < a þ b, then (a  b)2 < c(a þ b). Thus, it follows
that
 
ða þ b þ cÞ3 < ða þ bÞ ða  bÞ2 þ 4ab þ 3ða þ bÞc þ 4c2 <
< ða þ bÞðcða þ bÞ þ 4ab þ 3ða þ bÞc þ 4c2 Þ ¼ 4ða þ bÞða þ cÞðb þ cÞ:

Hence ðaþbðaþbþc
ÞðaþcÞðbþcÞ
Þ3
> 14.

(m) a3 þ b3 þ c3 ¼ a2  a þ b2  b þ c2  c < a2 ðb þ cÞ þ b2 ða þ cÞ þ c2 ðb þ aÞ <


<(a þ b þ c)(ab þ bc þ ac).
2
 2ððabþbcþac
aþbþcÞ
2 2 2
a
(n) bþc þ aþc
b
þ aþb
c
¼ abþac
a
þ abþcb
b
þ acþbc
c
Þ  2.
3

Let maxða; b; cÞ ¼ c, then


a b c a b c c
þ þ  þ þ ¼1þ < 2:
bþc aþc aþb bþa aþb aþb aþb
pffiffiffiffiffiffiffiffiffiffiffiffiffiffiffiffiffiffiffiffiffiffiffiffiffi pffiffiffiffiffiffiffiffiffiffiffiffiffiffiffiffiffiffiffiffiffiffiffiffiffi2 
(o) We have that b2 þ c2  a2 þ a2 þ c2  b2  2 b2 þ c2  a2 þ c2 þ
pffiffiffiffiffiffiffiffiffiffiffiffiffiffiffiffiffiffiffiffiffiffiffiffiffi pffiffiffiffiffiffiffiffiffiffiffiffiffiffiffiffiffiffiffiffiffiffiffiffiffi
a2  b2 Þ ¼ 4c2 . Consequently b2 þ c2  a2 þ a2 þ c2  b2  2c.
pffiffiffiffiffiffiffiffiffiffiffiffiffiffiffiffiffiffiffiffiffiffiffiffiffi pffiffiffiffiffiffiffiffiffiffiffiffiffiffiffiffiffiffiffiffiffiffiffiffiffi
Similarly, we get that b2 þ c2  a2 þ a2 þ b2  c2  2b and
pffiffiffiffiffiffiffiffiffiffiffiffiffiffiffiffiffiffiffiffiffiffiffiffiffi pffiffiffiffiffiffiffiffiffiffiffiffiffiffiffiffiffiffiffiffiffiffiffiffiffi
a þ b  c þ a þ c  b  2a.
2 2 2 2 2 2

Summing
p upffi pthe
ffiffiffiffiffiffiffiffiffiffiffiffiffiffiffiffiffiffiffiffiffiffiffiffi last ffi three
ffiffiffiffiffiffiffiffiffiffiffiffiffiffiffiffiffiffiffiffiffiffiffiffi inequalities,
pffiffiffiffiffiffiffiffiffiffiffiffiffiffiffiffiffiffiffiffiffiffiffiffi
ffi we deduce that
b þ c  a þ a þ c  b þ a þ b  c  a þ b þ c.
2 2 2 2 2 2 2 2 2

abc
(p) We need to prove that aþbc þ abþc
abc
þ bþca
abc
 a2 þ b2 þ c2 . Indeed, without
loss of generality one can assume that a  b  c. Then, we have that

abc abc abc


 a2 þ  b2 þ  c2 ¼
bþca aþcb aþbc
aða  bÞða  cÞ bðb  aÞðb  cÞ cðc  aÞðc  bÞ
¼ þ þ 
bþca aþcb aþbc
aða  bÞða  cÞ bðb  aÞðb  cÞ bða  bÞðb  cÞ bðb  aÞðb  cÞ
 þ  þ 
bþca aþcb bþca aþcb
bða  bÞðb  cÞ bðb  aÞðb  cÞ
 þ ¼ 0:
aþcb aþcb

Note that
7.2 Solutions 325

qffiffiffiffiffiffiffiffiffiffiffi qffiffiffiffiffiffiffiffiffiffiffi qffiffiffiffiffiffiffiffiffiffiffi2 abc abc abc


abc
þ abc
þ abc
¼ þ þ þ
aþbc
0
abþc bþca a þ b  c a  b þ c b1
þca
B 1 1 1 C
þ2abc@qffiffiffiffiffiffiffiffiffiffiffiffiffiffiffiffiffiffiffiffiffiffiffiffiffiffiffi þ qffiffiffiffiffiffiffiffiffiffiffiffiffiffiffiffiffiffiffiffiffiffiffiffiffiffiffi þ qffiffiffiffiffiffiffiffiffiffiffiffiffiffiffiffiffiffiffiffiffiffiffiffiffiffiffiA 
2 2 2
c 2  ð a  bÞ b2  ðc  aÞ a2  ðb  cÞ

1 1 1
 a2 þ b2 þ c2 þ 2abc þ þ ¼ ða þ b þ c Þ2 ,
a b c
qffiffiffiffiffiffiffiffiffiffiffi qffiffiffiffiffiffiffiffiffiffiffi qffiffiffiffiffiffiffiffiffiffiffi
Thus abc
aþbc þ abc
abþc þ abc
bþca  a þ b þ c.

(q) We have that


   
 a  b b  c c  a  a  b b  а а  c c  a
   
 a þ b þ b þ c þ c þ a ¼  a þ b þ b þ c þ b þ c þ c þ a ¼
   
ða  bÞðc  aÞ ðа  cÞða  bÞ ða  bÞðc  aÞðc  bÞ
¼  þ  ¼  <
ða þ bÞðb þ cÞ ðb þ cÞðc þ aÞ  ða þ bÞðb þ cÞðc þ aÞ
cba abc 1
<  pffiffiffiffiffi pffiffiffiffiffi pffiffiffiffiffi ¼ :
ða þ bÞðb þ cÞðc þ aÞ 2 ab  2 bc  2 ca 8

7.1.30. At first, let us prove the following lemmas.


Lemma 1 If max(AB, BC, AC, AX, BX, CX) ¼ max (AB, BC, AC), then m(ABC)  m
(ABX) þ m(ACX) þ m(BCX).
Indeed, we have that SABC  SABX þ SBCX þ SACX  amð2ABXÞ þ amð2ACXÞ þ
amðBCXÞ
2 , where a ¼ max (AB, BC, AC). Hence, mðABCÞ ¼ 2SaABC  mðABXÞþ
mðACXÞ þ mðBCXÞ.
Lemma 2 If point C is inside of triangle ABX, then m(ABC)  m(ABX).
Consider two cases.
(a) max(AB, BC, AC, AX, BX, CX) ¼ max (AB, BC, AC) ¼ a. Then from the inequal-
ity SABC  SABX , we obtain that amð2ABCÞ  SABX  amð2ABXÞ
Thus m(ABC)  m(ABX).
(b) maxðAB; BC; AC; AX; BX; CXÞ ¼ AX. Then the perpendicular drawn from ver-
tex B to the straight line AX intersects ray AC. Hence, m(ABC)  m(ABX).
If the convex envelope of points A, B, C, X is a quadrilateral, then by lemma
2, we have that
m(ABO) þ m(ACO) þ m(BCO)  m(ABX) þ m(ACX) þ m(BCX), while by using
lemma 1, we deduce that m(ABO) þ m(ACO) þ m(BCO)  m(ABC), where O is the
intersection point of the diagonals of the quadrilateral with vertices A, B, C, X.
If the convex envelope of points A, B, C, X is a triangle ABC, then from lemma
1, the proof of the problem follows straight away.
326 7 Miscellaneous Inequalities

If the convex envelope of points A, B, C, X is a triangle ABX, then m(ABC)  m


(ABX)  m(ABX) þ m(ACX) þ m(BCX).
If the convex envelope of points A, B, C, X is a segment, then m(ABC) ¼ 0 ¼ m
(ABX) þ m(ACX) þ m(BCX).
7.1.31. Using the property of the bisector of the triangle, we obtain that AB0 ¼ aþc
bc

0 ¼
b . Consequently BB0 ¼ aþbþc. Similarly, we get that AA0 ¼ aþbþc and
BI aþc BI aþc AI bþc
and IB
CI
CC0
¼ aþbþc. Then,
aþb

AI BI CI ða þ cÞðb þ cÞða þ bÞ 1
  ¼ > :
AA0 BB0 CC0 ða þ b þ cÞ3 4

(see the problem 7.1.29l ). We have that


!3
AI BI CI aþc
þ aþbþc
bþc
þ aþbþc
aþb
8
   aþbþc
¼ :
AA0 BB0 CC0 3 27

xþyþz3
See the proof of problem 7.1.23, for x, y, z > 0, we have that xyz  3 .
7.1.32. Draw through point Q a perpendicular QQ0 to plane π and consider a circle
ω(Q0, Q0R) on plane π and let R0 be a point, such that for any point R of the
circumference ω we have PR  PR0 (see Figure 7.20).
QR  QR0 .
Then, QPþPR QPþPR0

Consider Figure 7.21, where PM0 ¼ PR0.


sin ∠QR0 M0
Then QPþPR
QR
0
¼ QM
QR ¼
0
sin α  sin1 α. Hence the value of QPþPR
QR is the greatest if
0 0 2 2

∠QR0 Q0 ¼ 90  α2. When points Q0 and P coincide, the ratio QPþPR QR reaches its
greatest value at any point R belonging to the circumference of ω0(Q0, QQ0).
7.1.33. Let us take on the plane a point O and draw lines l1, . . . , ln parallel to the
borders of the given strips. Let us choose a coordinate system with the point of
origin at O. Consider those lines of the set l1, . . . , ln having points in the
quarter I. Without loss of generality, one can assume that the strips corresponding
to these lines are such that the sum of their widths is not less than 11
2 . Now, consider
line l that is a bisector of the second quarter (Figure 7.22) and angles α1, α2, . . . , αk,
formed by the considered lines with line l.

Figure 7.20 Q

w R p
R0 Q0 P
7.2 Solutions 327

Figure 7.21 Q

a
R0 Q0 P

a
2

M0

Figure 7.22 y lk

l2

l1

O x
a1 a2 ak

Figure 7.23
C

P1 P2 Pk
A a1 a2 ak B l

Let us arrange on the plane the strips Π1, Π2, . . . , Πk corresponding to lines l1,
l2, . . . , lk in a way shown on Figure 7.23. Then, it is clear that triangle ABC is
π π
entirely covered by the strips Π1, . . . , Πk, AB  11
2 and α1  4 , π  αk  4. Hence,
328 7 Miscellaneous Inequalities

for the inradius r of triangle ABC, the condition r ¼ ctg α1 þctg


AB
παk  pffiffi
AB
2 2 4 ð 21Þ>1
2ctg π811
holds true. Therefore, r > 1.

!
To complete the proof one has to make a parallel translation by a vector OO 1 ,
where O is the incenter of triangle ABC, and O1 is the center of the given circle.
pffiffi
7.1.34. Let ∠C ¼ 90 . If AB  2, then SABC ¼ AB2  hc  hc  m c ¼ 2  1 < 2 .
AB 3 3

Let ω1 and ω2 be given circles. If AB > 2, then one can assume that A, C 2 ω1.
Let us choose a point C1 on segment CB, such that AC1 ¼ 2. Then,pffiffiffiffiffiffiffiffi
it is clear that B,
C1 2 ω2. Hence, it follows that BC1  2, and SABC  ð2þxÞ2 4x , where CC1 ¼ x.
2

Note that
pffiffiffiffiffiffiffiffiffiffiffiffiffi
ð2 þ xÞ 4  x 2
¼
2
pffiffiffi rffiffiffiffiffiffiffiffiffiffiffiffiffiffiffiffiffiffiffiffiffiffiffiffiffiffiffiffiffiffiffiffiffiffiffiffiffiffiffiffiffiffiffiffiffiffiffiffiffiffiffiffiffi pffiffiffi sffiffiffiffiffiffiffiffiffiffiffiffiffiffiffiffiffiffiffiffiffiffiffiffiffiffiffiffiffiffiffiffiffiffiffiffiffiffiffiffiffiffiffiffiffiffiffiffiffiffiffiffiffiffiffiffiffiffiffiffiffi
2þx 2þx 2 2þx 2
3 3 2þx 2þx 2þx 3 3 3 þ 3 þ2x
¼   ð2  xÞ   3 
2 3 3 3 2 2 2
pffiffiffi sffiffiffiffiffiffiffiffiffiffiffiffiffiffiffiffiffiffiffiffiffiffiffiffiffiffiffiffiffiffiffiffiffiffiffiffiffiffiffiffiffiffiffiffiffiffiffiffiffiffiffiffiffiffi
2þx 2þx 2þx 4
ffi pffiffiffi
3 3 þ þ þ 2  x 3 3
 3 3 3
¼ :
2 4 2
pffiffi
Hence, SABC  3 2 3.
7.1.35. (a) Let us display on edges of the trihedral angle equal segments SA, SB, SC,
where S is the vertex of the trihedral angle. Let point O be the foot of the
perpendicular drawn from point S to plane ABC, Then, OA ¼ OB ¼ OC.
Take on segment SC1 point O1, so that O1C1 ¼ C1O, where C1 is the midpoint of
segment AB. Then, we have that ∠AOB ¼ ∠AO1B ¼ ∠ASB þ ∠O1AS þ
∠O1BS > ∠ASB. Consequently, ∠ASB þ ∠BSC þ ∠ASC < ∠AOB þ ∠BOC þ
∠AOC  360 , thus

∠ASB þ ∠BSC þ ∠ASC < 360 : ð7:13Þ

Let ASC be the largest plane angle in the trihedral angle SABC. Using (7.13) we
obtain that for the trihedral angle SAB0C, where B0 is the point, symmetric to point
B with respect to point S
180  ∠ASB þ 180  ∠BSC þ ∠ASC < 360 , hence ∠ASC < ∠ASB þ ∠BSC.
(b) Let us produce a plane crossing all sides of the given polyhedral angle with a
vertex S. Let the resulting section be a convex n-gon A1A2, . . . , An.
According to problem 7.1.35a, we have that

180 ðn  2Þ ¼ ∠An A1 A2 þ ∠A1 A2 A3 þ ::: þ ∠An1 An A1 <
< ð∠An A1 S þ ∠A2 A1 SÞ þ ð∠A1 A2 S þ ∠A3 A2 SÞ þ ::: þ ð∠An1 An S þ ∠A1 An SÞ ¼

¼ ð∠An A1 S þ ∠A1 An SÞ þ ::: þ ð∠A2 A1 S þ ∠A1 A2 SÞ ¼ 180 n  ∠A1 SA2  :::  ∠An SA1 :

Therefore, ∠A1SA2 þ ∠A2SA3 þ . . . þ ∠AnSA1 < 360 .


7.2 Solutions 329

Figure 7.24 D

A B
D1
D2

(c) Let AnSA1 be the largest plane angle in the n-hedral angle SA1A2 . . . An.
According to the problem 7.1.35a, we have that
∠A1SA2 þ ∠A2SA3 þ . . . þ ∠An 1SAn> ∠A1SA3þ ∠A3SA4 þ . . . þ ∠An  1SAn
 ∠A1SA4 þ ∠A4SA5 þ . . . þ ∠An  1SAn  . . .  ∠A1SAn, hence
∠A1SA2 þ ∠A2SA3 þ . . . þ ∠An  1SAn > ∠A1SAn.
(d) We have to prove that ∠AMB þ ∠BMC > ∠CMD1 (see Figure 7.24).
We have that

∠AMB þ ∠BMC ¼ ∠AMB þ ∠BMD2 þ ∠D2 MC > ∠AMD2 þ ∠D2 MC ¼


¼ ∠AMD1 þ ∠D1 MD2 þ ∠D2 MC > ∠AMD1 þ ∠CMD1 > ∠CMD1 :

Hence, ∠AMB þ ∠BMC > ∠CMD1.


(e) According to the solution of problem 7.1.35d, we have that ∠AMB þ ∠BMC
> > ∠AMD1 þ ∠CMD1 ¼ π  ∠AMD þ π  ∠CMD. Consequently, ∠AMB þ
∠BMC þ ∠AMD þ ∠CMD > 2π. Similarly, one can prove the inequali-
ties ∠AMC þ ∠BMD þ ∠AMB þ ∠CMD > 2π, ∠AMD þ ∠BMC þ ∠BMD þ
∠AMC > 2π.
Summing up the last three inequalities we obtain the required inequality.
(f) Let O be any point inside of the trihedral angle with the dihedral angles α, β, γ.
Draw through point O rays OA, OB, OC perpendicular to the faces of the
trihedral angle. According to problem 7.1.35b, we have that π  α þ
π  β þ π  γ < 2π, consequently α þ β þ γ > π.
(g) Let O be any point inside of the tetrahedron with the dihedral angles α, β, γ,
α1, β1, γ 1. Let rays OA, OB, OC, OD be perpendicular to the faces of the tetrahe-
dron. Then, point O is inside of the tetrahedronа ABCD, since one can choose

!
!
!
!
points A1, B1, C1, D1 on those rays, such that OA 1 þ OB 1 þ OC 1 þ OD 1 ¼ ~ 0.
According to problem 7.1.35e, it follows that π  α þ π  β þ π 
γ þ π  α1 þ π  β1 þ π  γ 1 > 3π. Therefore, α þ β þ γ þ α1 þ β1 þ γ 1 < 3π.
330 7 Miscellaneous Inequalities

Figure 7.25

a g

b1
a1
g1

According to problem 7.1.35f, it follows that α þ β þ γ > π, α þ β1 þ γ 1 > π,


β þ α1 þ γ 1 > π, γ þ α1 þ β1 > π. Summing up all these inequalities, we deduce
that α þ β þ γ þ α1 þ β1 þ γ 1 > 2π (see Figure 7.25).
(h) Let O and O1 be the midpoints of segments AC1 and BD respectively. Then,
∠BOC1 ¼ 2 ∠BAC1, ∠DOC1 ¼ 2 ∠DAC1.
∠BOD ¼ 2 ∠BOO1 ¼ 2 ∠AOO1 ¼ 2 ∠A1AC1. According to problem
7.1.35b ∠BOC1 þ ∠DOC1 þ ∠BOD < 2π. Consequently ∠BAC1 þ ∠A1AC1 þ
∠DAC1 < π.
However, according to problem 7.1.35a

∠BAC1 þ ∠A1 AC1 þ ∠DAC1 ¼


1 1 1
¼ ð∠BAC1 þ ∠A1 AC1 Þ þ ð∠A1 AC1 þ ∠DAC1 Þ þ ð∠BAC1 þ ∠DAC1 Þ >
2 2 2
π π π 3π
> þ þ ¼ :
4 4 4 4

7.1.36. (a) Denote the cubes with edges a and b by Φa and Φb respectively, and the
cube, symmetric to the cube Φb with respect to the center of the cube Φa by Φb0 .
It is not difficult to prove that the cube Φb and the center of the cube Φa are on
different sides of plane α, containing one of the faces of the cube Φb.
Consider two following cases:
(a) α is parallel to one of the edges of the cube Φa,
(b) α is not parallel to any of the edges of the cubeΦa.
Note that the cubes Φb and Φb0 do not have any common points. Indeed, if points
M 2 Φb and M 2 Φb0 , then point M0 , symmetric to point M with respect to the center
of the cube Φa, also belongs to Φb. Consequently, the center of the cube Φa, that is
the midpoint of segment MM0 , belongs to the cube Φb.
This leads to a contradiction.
In the case (a) the proof is obtained if one considers the projection of the cubes
Φb and Φb0 on the face of the cube Φa, that is perpendicular to plane α.
7.2 Solutions 331

It is clear that these projections do not have any common points and are
rectangles with sides b and b(sin φ þ cos φ), where φ is the angle between the
planes containing one of the faces of the cubes Φa and Φb.
Since sin φ þ cos φ  sin2φ þ cos2φ ¼ 1, then these projections contain squares
with sides b. Then, according to the problem 2.1.6b, we have that 2b < a. Thus, b < a2.
Let us continue now the case (b). Let the cube Φb be inside of a triangular
pyramid ABCD (see the proof of problem 7.1.36c), where A is the vertex of the cube
Φa, and the edges of the cube Φa, drawn from vertex A, belong to the rays AB, AC,
AD, with points B, C, D belonging to plane α.
Let us consider now a plane (B0 C0 D0 ), parallel to the plane (BCD) and containing
one of the faces of the cube Φb, with B0 2 AB, C0 2 AC, D0 2 AD. Using problem
2.1.10, we obtain that the cube Φb can be placed inside of the pyramid ABCD, so
that one of its edges belongs to, let us say, segment B0 C0 .
Now we will consider the sections of the tetrahedron ABCD, obtained by the
intersections with planes β and β0 , containing the faces of the cube Φb and
perpendicular to the straight line B0 C0 . Since AD ⊥ (ABC), then AD ⊥ B0 C0 . We
have that B0 C0 ⊥ β, B0 C0 ⊥ β0 , thus AD k β, AD k β0 , or AD belongs to one of planes β
and β0 . Note that these sections are right-angled triangles, and using problem 2.1.5,
we obtain that the cube Φb is in a triangular pyramid ABCD, with the face ABC
containing one of the faces of the cube Φb.
Repeating this reasoning once again, we obtain that the cube Φb is inside of the
triangular pyramid ABCD and point A is one of the vertices of the cube Φb. It is
clear that b < a2, since the center of the cube Φa is not inside of the pyramid ABCD.
(b) Denote the cube with the edge a by Φa. Let b, c, d be the smallest non-negative
numbers, such that by the parallel translation, given by the relations x0 ¼ x  b,
y0 ¼ y  c, z0 ¼ z  d, the cube Φa transforms into the cube Φa0 , having vertices
M1, M2, M3 belonging to the coordinate planes Oxy, Oyz and Oxz, respectively.
Now, consider a cube Φ2a with sides 2a, with one of the vertices at the point of
the origin and three edges on the positive semiaxes Ox, Oy, Oz. Note that the cube
Φa0 does not have points outside the cube Φ2a.
Indeed, if M(x, y, z) 2 Φa0 and M 2 = Φ2a, then without loss of generality, we can
assume that x > 2a. Consequently pffiffiffi MM 2 > 2a. However, on the other hand, M2,
M 2 Φa0 . Therefore, MM2  3a. This leads to a contradiction. Thus, the cube Φa0
does not have points outside the cube Φ2a.
If M00 (a, a, a) 2
= Φa0 , then since M00 is the center of the cube Φ2a and according to
problem 7.1.36а, it follows that a < 2a 2 . This leads to a contradiction.
Thus, M00 2 Φa0 . Hence all coordinates of point M0(a þ b, a þ c, a þ d ) are not
less than a and it belongs to the cube Φa.
(c) Denote the cubes with edges a, b and c by Φa, Φb and Φc respectively. Since the
cubes Φa and Φb do not have common points, then there exists a plane α, such
that these cubes are on the different sides of that plane. Since the cubes Φa and
Φb are on different sides of plane α, then consequently the cube Φc has vertices
on the both sides of plane α. Let A be the most distant vertex of Φc from plane α
and is in the half-space with a boundary α containing the cube Φa.
332 7 Miscellaneous Inequalities

Vertex B of the cube Φc is defined similarly.


Note that the ray with a vertex at A, containing one of the edges of the cube Φc is
either parallel to plane α or intersects it. If the chosen coordinate system with the
origin at point A is such that the positive semiaxes Ox, Oy, Oz contain the edges of
the cube Φc, then according to problem 7.1.36b, there exists a point M0(x0, y0, z0),
belonging to the cube Φa and x0  a, y0  a, z0  a.
In that case, if the cube with the edge a, with one of the vertices at point A and its
three edges are on the positive semiaxes Ox, Oy, Oz, then it is in the half-space with
a boundary α, containing point A.
From the aforesaid, it follows that one can assume that each of the cubes Φa and
Φb, that do not have common points, has three edges on the edges of the cube Φc.
The following cases are possible.
(a) AB is the edge of Φc, then a þ b < c.
pffiffiffi pffiffiffi pffiffiffi
(b) AB is a diagonal of one of the faces of the cube Φc, then 2a þ 2b < 2c.
Therefore, a þ b < c.
pffiffiffi pffiffiffi pffiffiffi
(c) AB is a diagonal of the cube Φc, then 3a þ 3b < 3c. Consequently,
a þ b < c.
Thus, in all cases, we deduce that a þ b < c.
7.1.37. (a) We have that

ha hb hc aha bhb chc ðabÞ2 þ ðbcÞ2 þ ðacÞ2


þ þ ¼ 2 þ 2 þ 2 ¼ 2S  
a b c a b c ðabcÞ2
ðabÞðbcÞ þ ðabÞðacÞ þ ðbcÞðacÞ 4Sp p
 2S  ¼ ¼ :
ðabcÞ2 abc R

(b) Since maa þ mbb þ mcc ¼ m2S


a ha
þ m2S
b hb
þ m2S
c hc
, then we have to prove that
maha þ mbhb þ mchc  p2.
Let max(α, β, γ) ¼ γ. If γ > π2, then
  c a
c b
ma ha þ mb hb þ mc hc < þ bþ þ a
2 2 2 2
c cb ca ab a2 þ b2
þ hc < þ þ þ ¼ p2 :
2 2 2 2 2

If γ  π2, then we need to prove that maha  p( p  a), mbhb  p( p  b) and


mchc  p( p  c). We have that
  c a
c b
ma ha þ mb hb þ mc hc < þ bþ þ a
2 2 2 2
c cb ca ab a2 þ b2
þ hc < þ þ þ ¼ p2 :
2 2 2 2 2
7.2 Solutions 333

If γ  π2, then we need to prove that maha  p( p  a), mbhb  p( p  b) and


mchc  p( p  c). We have that
  S2
p2 ðp  aÞ2  m2a h2a ¼ p2 ðp  aÞ2  ðb  cÞ2 þ ðb þ cÞ2  a2  2 ¼
a !
ðb  cÞ2 ð p  bÞ ð p  c Þ
¼ pð p  aÞ pð p  aÞ  ðp  bÞðp  cÞ  4pðp  aÞ ¼
a2 a2
 !
ð a þ c  bÞ ð a þ b  c Þ ðb  c Þ2
¼ pðp  aÞ pðp  aÞ 1   ðp  bÞðp  cÞ ¼
a2 a2
ðb  cÞ2 pðp  aÞðb  cÞ2  2 
¼ pð p  aÞ 2
ð p ð p  a Þ  ð p  b Þð p  c Þ Þ ¼ 2
b þ c 2  a2 ¼
a 2a
pðp  aÞðb  cÞ2
¼ bc cos α  0:
a2
Hence maha  p( p  a). Similarly, we obtain that mbhb  p( p  b) and
mchc  p( p  c). Summing up these three inequalities, we obtain that
maha þ mbhb þ mchc  p2.
(c) It is known that r a ¼ pa
S
, r b ¼ pb
S
, r c ¼ pc
S
. Hence,

ra rb rc
þ þ ¼
a b c 
S S S p ðpbÞðpcÞ ðpaÞðpcÞ ðpbÞðpaÞ
¼ þ þ ¼ þ þ 
að
paÞ bðpbÞ cðpcÞ S a b c
p 1 1 1 pp p
 ðpbþpcÞ2 þ ðpaþpcÞ2 þ ðpbþpcÞ2 ¼  ¼ :
S 4a 4b 4c S 2 2r

Since raa þ rbb þ rcc ¼ aðpa


S
Þ þ bðpbÞ þ cðpcÞ and
S S p
R ¼ 4Sp
abc, then we have to prove
that pa þ pb þ pc  4p. Note that
bc ac ab

bc ac ab ðpaþpcÞðpaþpbÞ ðpbþpcÞðpaþpbÞ
þ þ ¼ þ þ
pa pb pc pa pb
ðpbþpcÞðpaþpcÞ ðpaÞðpbÞ ðpbÞðpcÞ ðpcÞðpaÞ
þ ¼ þ þ þ3p
pc pc pa pb
sffiffiffiffiffiffiffiffiffiffiffiffiffiffiffiffiffiffiffiffiffiffiffiffiffiffiffiffiffiffiffiffiffiffiffiffiffiffiffiffiffiffiffiffiffiffiffiffiffiffiffiffiffiffi sffiffiffiffiffiffiffiffiffiffiffiffiffiffiffiffiffiffiffiffiffiffiffiffiffiffiffiffiffiffiffiffiffiffiffiffiffiffiffiffiffiffiffiffiffiffiffiffiffiffiffiffiffiffi
ðpaÞðpbÞ ðpbÞðpcÞ ðpaÞðpbÞ ðpcÞðpaÞ
  þ  þ
pc pa pc pb
sffiffiffiffiffiffiffiffiffiffiffiffiffiffiffiffiffiffiffiffiffiffiffiffiffiffiffiffiffiffiffiffiffiffiffiffiffiffiffiffiffiffiffiffiffiffiffiffiffiffiffiffiffi
ðpbÞðpcÞ ðpcÞðpaÞ
þ  þ3p¼4p,
pa pb

since x2 þ y2 þ z2  xy þ yz þ zx.
334 7 Miscellaneous Inequalities

(d) We have that


m mb mc 2
a
þ þ ¼
a b c
     
2 2 2 2 2 2
ma m b c þ mb mc a þ ma mc b
ma 2 mb 2 mc 2 9 3 3 3 3 3 3
¼ þ 2 þ 2 þ  
a2 b c 2 abc
ma 2 mb 2 mc 2 9
 þ 2 þ 2 þ
a2 b c 2

(see problem 4.1.8c).  2  2  2  


2 2 2 2 2 2
Since maa2 þ mbb2 þ mcc2 ¼ 14 2 ba2 þ ab2 þ 2 bc2 þ bc2 þ 2 ac2 þ ac2  3  94, then
m  pffiffi
mc 2
a
a
þ mb
b þ c  9
4 þ 9
2 ¼ 27
4 . Thus, ma
a þ mb
b þ mc
c  3 3
2 .

(e) Since the triangle with sides ma, mb, mc has medians with the lengths 34 a, 34 b, 34 c
(see Figure 7.26).
3 3 3 pffiffi
a b c
Then, according to problem 7.1.37d, it follows that m4 a þ m4 b þ m4 c  3 2 3. Hence,
pffiffi

ma þ mb þ mc  2 3 .
a b c

(f) Let max(α, β, γ) ¼ γ. If γ  π2, then we have that ma  R þ ka, mb  R þ kb,


mc  R þ kc (See the proof of problem 2.4.11). Hence,

ma mb mc 1 1 1 ka kb kc
þ þ R þ þ þ þ þ ¼
ha hb hc h a hb hc a b c
 aka bkb ckc aka bkb ckc
þ þ
a b c Rp 2 ¼ R þ 1:
R þ þ þ 2 þ 2 þ 2 ¼ þ 2 2
2S 2S 2S aha bhb chc S S r
2 2 2

If γ > π2, then ma < bþc


2 , mb < 2 and mc < 2. Consequently,
aþc c

Figure 7.26

a/2
ma 2
mc
ma 3 3
2 3
mb
3
7.2 Solutions 335

ma mb mc b þ c a þ c c b a c
þ þ < þ þ ¼ þ þ ¼
ha hb hc 2ha 2hb 2hc 2ha 2hb 2r
ab1 c 2R c
¼ þ ¼ þ :
2S 2r c 2r

c þ 2r < 1 þ r , or (2R  c)(c  2r) > 0.


c R
We need to prove that 2R
This leads to a contradiction since 2R > c > ha > 2r.
(g) According to problem 5.5.8b, we have that ma þ mb þ mc  min
( p þ 2R, 4R þ r)  4R þ r.
pffiffiffiffiffiffiffiffiffiffiffiffiffiffiffiffi
Thus, it follows that ma þ mb þ mc  4R þ r. Therefore 3 3 ma mb mc  ma þ
mb þ mc  4R þ r. Hence, we deduce that

4R þ r 3
ma mb mc  :
3

(h) It is known that l2a ¼ ðbþc


4bc
Þ2
pðp  aÞ. Hence l2a  pðp  aÞ. Similarly, we get that
l2b  pðp  bÞ, l2c  pðp  cÞ and summing up these three inequalities, we obtain
that l2a þ l2b þ l2c  p2 .
(i) Since the triangle with sides 23 ma , 23 mb , 23 mc has an area S3 (see the proof of
problem 7.1.37e), then according to problem 5.5.9, it follows that
 2  2  2  2  2
2 2 2 2 2 2 2
ma þ mb þ mc  ma  mb  mb  mc 
3 3 3 3 3 3 3
 2
2 2 pffiffiffi S
ma  mc  4 3  :
3 3 3

pffiffiffi
Hence, ma 2 þ mb 2 þ mc 2  ðma  mb Þ2  ðmb  mc Þ2  ðma  mc Þ2  3 3S.
Remark Using the fact that the triangle with sides 23 ma , 23 mb , 23 mc has the medians
a b c
2,2,2and the area S3, one can propose «new» problems arising from this problem.
For example, according to problem 4.1.8c, we have that ma mb c þ mb mc a
þmc ma b  94 abc, then the inequality abmc þ bcma þ acmb  4mambmc holds true.
(j) We have that

ra rb rc S S S 1 a b c
þ þ ¼ þ þ ¼ þ þ ¼
ha hb hc ðp  aÞha ðp  bÞhb ðp  cÞhc 2 p  a p  b p  c

1 pbþpc paþpc paþpb
¼ þ þ ¼
2 pa pb pc
  
1 pb pa 1 pb pc 1 pa pc
¼ þ þ þ þ þ  1 þ 1 þ 1 ¼ 3:
2 pa pb 2 pc pb 2 pc pa
336 7 Miscellaneous Inequalities

Figure 7.27

B
b/2 OA
ra
A1 g/2

A C

  α
(k) Note that BC ¼ CA1 þ A1B¼r a tg β2 þ tg 2γ ¼ r a cos 2
cos β2 cos 2γ
(see Figure 7.27).
α β
Hence, a
ra ¼ cos 2
cos 2 cos 2γ
β . Similarly, we get that b
rb ¼ cos 2
cos α2 cos 2γ
,
γ
c
rc ¼ cos 2
cos β2 cos α2
.
We have to prove that
 
2α 2β 2γ 2α 2β 2β 2γ 2α 2γ
3 cos þ cos þ cos  4 cos cos þ cos cos þ cos cos ,
2 2 2 2 2 2 2 2 2

or

3 cos α þ cos β þ cos γ
3 þ 
2 2
 ð1 þ cos αÞð1 þ cos βÞ þ ð1 þ cos βÞð1 þ cos γ Þ þ ð1 þ cos αÞð1 þ cos γ Þ,

this means that 3  cos α þ cos β þ cos γ þ 2(cos α cos β þ cos β cos γ þ cos γ cos α).
The last inequality is obtained according to problems 5.1.4a and 5.1.14.
β γ
a cos cos
(l) We have that ra ¼ 2 2
(see the proof of problem 7.1.37k).
cos α2 ¼ 4R sin α2 cos β2 cos 2γ
Similarly, r b ¼ 4R sin β2 cos α2 cos 2γ and r c ¼ 4R sin 2γ cos α2 cos β2. Therefore,

r 2a þ r 2b þ r 2c ¼
2α 2β 2γ 2β 2α 2γ 2γ 2α 2β
¼ 16R sin cos cos þ sin cos cos þ sin cos cos
2
¼
2 2 2 2 2 2 2 2 2
¼ 2R2 ðð1  cos αÞð1 þ cos βÞð1 þ cos γ Þ þ ð1  cos βÞð1 þ cos αÞð1 þ cos γ Þþ
þð1  cos γ Þð1 þ cos αÞð1 þ cos βÞÞ ¼ 2R2 ð3 þ cos α þ cos β þ cos γ
 cos α cos β  cos β cos γ  cos γ cos α  3 cos α cos β cos ! γÞ ¼
2
R þ r r 2
þ p 2
 4R 2
p 2
 ð 2R þ r Þ
¼ 2R2 3 þ  3 ¼ ð4R þ r Þ2  2p2
R 4R2 4R2

(see problems 5.3.6, 5.3.10b and 5.3.9b).


7.2 Solutions 337

According to problem 5.5.6, p2  4R2 þ 4Rr þ 3r2. Hence, r 2a þ r 2b þ r 2c  8


R2  5r 2  27 2
4 R (see problem 5.5.1a), thus r 2a þ r 2b þ r 2c  27 R2 .
   4   
(m) We have that ðha þ hb þ hc Þ h1a þ h1b þ h1c ¼ 3 þ hhab þ hhbc þ hhbc þ hhbc þ
 
hc þ ha  9 and h1a þ h1b þ h1c ¼ 2S þ 2S þ 2S ¼ S=p ¼ 1r . Hence, ha þ hb þ
ha hc a b c 1

hc  9r.
 
(n) We have that m2a þ m2b þ m2c ¼ 34 a2 þ b2 þ c2 ¼ 3R2 ðsin 2 α þ sin 2 β þ sin 2 γ Þ
 27 2
4 R (see problem 5.1.1).
(o) Since (x þ y þ z)2  3x2 þ 3y2 þ 3z2, according to problem 7.1.37n ma þ mb þ
qffiffiffiffiffiffiffiffiffiffiffiffiffiffiffiffiffiffiffiffiffiffiffiffiffiffiffiffiffiffiffiffiffiffiffi
 
mc  3 m2a þ m2b þ m2c  4, 5R. Consequently, m1a þ m1b þ m1c  ma þm9b þmc 
¼ R2 . Hence, m1a þ m1b þ m1c  R2 .
9
4, 5R
(p) We need to prove that ra þ rb þ rc ¼ 4R þ r. Indeed,

ra þ rb þ rc ¼
S S S S ð ð p  a Þ ð p  b Þ þ ð p  b Þð p  c Þ þ ð p  c Þ ð p  a ÞÞ
¼ þ þ ¼ ¼
pa pb pc ð p  a Þ ð p  b Þð p  c Þ
Sðab þ bc þ ac  p2 Þ Sðr2 þ 4rRÞ
¼ ¼ ,
ð p  a Þ ð p  b Þð p  c Þ ð p  a Þ ð p  b Þð p  c Þ

(see the proof of problem 5.3.7).


r a þ r b þ r c ¼ ðpaSrÞððpb rþ4RÞ S ðrþ4RÞ
2
Since ÞðpcÞ ¼ pðpaÞðpbÞðpcÞ ¼ r þ 4R, then
according to problem 5.5.1a, ra þ rb þ rc  4, 5R.
pffiffi
2bc cos α
(q) Since S ¼ 12 bc sin α ¼ 12 bla sin α2 þ 12 cla sin α2, then la ¼ bþc 2 > bþc 2bc
.
   
Consequently, l1a < p1ffiffi2 1b þ 1c . Similarly, we obtain that l1b < p1ffiffi2 1a þ 1c and
 
lc < 2 a þ bp. ffiffiffi Summing up these three inequalities, we deduce that
1 p1ffiffi 1 1
 
la þ lb þ lc < 2 1a þ 1b þ 1c .
1 1 1

pffiffiffiffiffiffiffiffiffiffiffiffiffiffiffiffiffi
(r) We have that la  pðp  aÞ (see the proof of problem 7.1.37h). Hence,
pffiffiffiffiffiffiffiffiffiffiffiffiffiffiffiffiffiffiffiffiffiffiffiffiffiffiffiffi
2 pðpaÞðpbÞðpcÞ
hc ¼ 2S c ¼ c  2la ðpb
c
Þ
 la .
(s) Let point A1 be on segment BC and straight line AA1 intersect the circumcircle of
triangle ABC (for the second time) at point A2. Then, AA1  A1A2 ¼ BA1  A1C and
A1A2 ¼ AA2  AA1  2R  AA1. Consequently, BA1  A1C  AA1(2R  AA1).
Thus,

AA21 þ BA1  A1 C ∗
AA1  ð Þ
2R

One can easily prove that, if AA1 ¼ la, AA21 þ BA1  A1 C ¼ bc, then la  2R
bc
.
338 7 Miscellaneous Inequalities

Figure 7.28 A

B1

B A1 C

A⬘

pffiffiffiffi pffiffiffiffiffiffiffiffiffiffiffiffiffiffiffiffiffi
We have that la ¼ 2bþcbc pðp  aÞ, 2Rr ¼ abc 2S  P ¼ aþbþc to prove the inequal-
S abc
qffiffiffiffiffiffiffiffiffiffiffiffiffiffiffi
pffiffiffiffiffi ffi p ffiffiffiffiffiffiffiffiffiffiffiffiffiffiffiffiffi pffiffiffi
ity la  2pbcffiffiffiffiffi
2Rr
ffi ¼ 12 bc
a  2p , we have to prove that 4 a ð p  a Þ  ðb þ c Þ 2,
or (b þ c  2a)2  0.
(t) According to (*) (see the proof of problem 7.1.37s), when AA1 ¼ ma, we obtain
2 2 2 2
m 2 þa m2 þm2 þm2 þa þb þc
¼ a þb2Rþc .
2 2 2
that ma  a2R 4 . Hence, ma þ mb þ mc  a b 2R c 4 4 4

0
(u) Let GA1 ¼ A1A (see Figure 7.28), then using the inequality in problem 7.1.37t
4
ðm2 þm2 þm2 Þ
for triangle BGA0 , we deduce that a2 þ b2 þ 2c  9 2maa 2mbb2mc c .
3 3 3
2SBGA0

Note that SBGA0 ¼ 13 SABC , consequently mmb ma c þ mma mb c þ mma mc b  SABC


P
¼ 1r .
m2 þm2 þm2
Remark We have that, mmb ma c þ mma mb c þ mma mc b ¼ ama mbb mc c  ma mb þm b mc þmc ma
ma mb mc ¼ m1a þ
mb þ mc  R (see problem 7.1.37o). Hence, mb mc þ ma mc þ ma mb  R.
1 1 2 ma mb mc 2

(v) We shall prove that if a, b, c  0, then (a þ b)(b þ c)(a þ c)(a þ b þ c)2  24abc
(a2 þ b2 þ c2).
Indeed, without loss of generality, we can assume that a þ b þ c ¼ 1 and c  13.
Then, we have to prove that (1  a)(1  b)(1  c)  24abc(1  2ab  2bc 
2ac), or

ab þ ða þ bÞc  abc  24abcð1  2ab  2cða þ bÞÞ: ð7:14Þ


Let f(x) ¼ 48cx þ (1  25c þ 48c (1  c))x þ c(1  c), then (7.14) is equivalent
2 2

to the inequality f(ab)  0.


Þ2 Þ2
Note that 0  ab  ðaþb 4 ¼ ð1c
4 ¼ x0 .
If 1  25c þ 48c2(1  c)  0 or f(x) does not have roots, then it is clear that
f(ab)  0.
Let 1  25c þ 48c2(1  c) < 0 and x1 and x2 be its roots. Then by Viètta’s
theorem x1 > 0, x2 > 0 (in the case of c ¼ 1, a ¼ b ¼ 0 and then f(ab) ¼ 0). Note
that f ðx0 Þ ¼ 14 ð3c  1Þ2 ð2c  1Þ2  0. Consequently, x0  x1  x2 or x0  x2  x1 .
7.2 Solutions 339

4
In the case of x0  x2  x1, we have that x20  x1 x2 . Hence, ð1c Þ
16  48 .
1c

Therefore, ð1  cÞ3  13. This leads to a contradiction as ð1  cÞ3 


23
3 ¼ 27
8
< 13.
Thus, ab  x0  x1  x2. It follows that f(ab)  0.
For numbers p  a > 0, p  b > 0, p  c > 0, we have that

ððp  aÞ þ ðp  bÞÞððp  bÞ þ ðp  cÞÞððp  aÞ þ ðp  cÞÞ ððp  aÞ þ ðp  bÞ þ ðp  cÞÞ2 


 
 24ðp  aÞðp  bÞðp  cÞ ðp  aÞ2 þ ðp  bÞ2 þ ðp  cÞ2 :

Consequently,
 
abcp2  24ðp  aÞðp  bÞðp  cÞ ðp  aÞ2 þ ðp  bÞ2 þ ðp  cÞ2 ,
S2  
4RSp2  24  ðp  aÞ2 þ ðp  bÞ2 þ ðp  cÞ2 ,
p
 
Rp  2r  3 ðp  aÞ2 þ ðp  bÞ2 þ ðp  cÞ2 ,
2


Rp2  2r ðp  a þ p  b þ p  cÞ2 þ ððp  aÞ  ðp  bÞÞ2 þ

þððp  bÞ  ðp  cÞÞ2 þ ððp  aÞ  ðp  cÞÞ2 :

2 2 2
2r  p  ða  bÞ þ ðb  cÞ þ ðc  aÞ .
Hence, we obtain that R2r 2

pffiffiffiffiffiffiffiffiffiffiffiffiffiffiffiffiffi pffiffiffiffiffiffiffiffi
(w) We have that la  pðp  aÞ ¼ r b r c (see the proof of problem 7.1.37h). Note
2
 ðbþc4Þ a ¼ pðp  aÞ ¼ r b r c . Hence,
2
that m2a ¼ 2b þ2c a2
2 2
4
pffiffiffiffiffiffiffiffi pffiffiffiffiffiffiffiffi pffiffiffiffiffiffiffiffi pffiffiffiffiffiffiffiffi pffiffiffiffiffiffiffiffi pffiffiffiffiffiffiffiffi
ma mb þ mb mc þ mc ma  r b r c  r c r a þ r a r c  r b r a þ r a r b  r b r c ¼
pffiffiffiffiffiffiffiffi pffiffiffiffiffiffiffiffi pffiffiffiffiffiffiffiffi
¼ r c  r a r b þ r a r b r c þ r b r a r c  r c lc þ r a la þ r b lb :

(x) See the proof of problem 7.1.37q,


 2
2bc 2ac b a2
la þ lb < þ ¼ 2b þ 2a  2 þ 
bþc aþc bþc aþc
2 2
2ð b þ aÞ 2ð b þ aÞ 4
 2ð b þ aÞ  < 2ð b þ aÞ  ¼ ða þ bÞ:
a þ b þ 2c 3ða þ bÞ 3
(y) We have that
qffiffiffiffiffiffiffiffiffiffiffiffiffiffiffiffiffiffiffiffiffiffiffiffiffiffiffiffiffiffiffiffiffiffiffiffiffiffiffiffiffiffiffiffiffiffiffiffiffiffiffiffiffiffiffiffiffiffiffiffiffiffiffiffiffiffiffi
 
4acma mc ¼ ac 2b2 þ 2c2  a2 2a2 þ 2b2  c2 ¼
qffiffiffiffiffiffiffiffiffiffiffiffiffiffiffiffiffiffiffiffiffiffiffiffiffiffiffiffiffiffiffiffiffiffiffiffiffiffiffiffiffiffiffiffiffiffiffiffiffiffiffiffiffiffiffiffiffiffiffiffiffiffiffiffiffiffiffiffiffiffiffiffiffiffiffiffiffiffiffi
  
¼ ac 2 2b2  a2  c2 a2 þ b2 þ c2 þ 9a2 c2

and 16S2 þ (b2  a2)(c2  b2) ¼ (2b2  a2  c2)(a2 þ c2  b2) þ 3a2c2.


Denote x ¼ 2b2  a2  c2, y ¼ a2 þ c2, z ¼ b2, then we have to prove that
pffiffiffiffiffiffiffiffiffiffiffiffiffiffiffiffiffiffiffiffiffiffiffiffiffiffiffiffiffiffiffiffiffiffiffi
ac 2xðy þ zÞ þ 9a2 c2  xðy  zÞ þ 3a2 c2 (if x(y  z) þ 3a2c2 < 0, then the
340 7 Miscellaneous Inequalities

inequality is evident) or 2a2c2x(y þ z)  x2(y  z)2 þ 6x(y  z)a2c2, 2a2c2x


(4z  2y)  x2(y  z)2.
Since 2z  y ¼ x, then 4a2c2x2  x2(y  z)2, x2(2ac  y þ z)(2ac þ y  z)  0, x
2
(b þ a  c)(b þ c  a)(a þ c  b)(a þ b þ c)  0.
pffiffi
(z) According to problem 5.1.6, we have that 2R a
þ 2R
b
þ 2R
c
 3 2 3. Therefore,
pffiffiffi
a þ b þ c  3 3abc 4S . Then, according to the remark of the problem 7.1.37i, it
follows from the last inequality that

2 2 2 pffiffiffi
2 2 2 pffiffiffi 3ma  3mb  3mc 1 1 1 3
ma þ mb þ mc  3 3 : Thus, þ þ  :
3 3 3 S ma mb mb mc mc ma S
4
3

(aa) See the second proof of problem 7.1.46b.


(ab) See the second proof of problem 7.1.46b.
(ac) Let a  b  c, then max(ma, mb, mc) ¼ ma, the median drawn from vertex
A intersects the bisector drawn from vertex B at point E, аnd I is the incenter
of triangle ABC. Then, mlaa þ mlbb þ mlcc > la þlmbaþlc > BEþEIþIA
ma  BEþEA
ma > 1.
(ad) We have that

ha þ hb þ hc ¼ 2Rð sin α sin β þ sin β sin γ þ sin α sin γ Þ ¼


p2 þ 4Rr þ r 2 4R2 þ 8Rr þ 4r 2
¼ 2R    2R þ 5r
4R2 2R
(see problems 5.3.10а, 5.5.6 and 5.5.1a).
(ae) See the proof of problem 7.1.37ad. We have that ha þ hb þ hc ¼ p þ4Rrþr
2 2

2R
2
> ð2RþrÞ 2R
þ4Rrþr 2
> 2R þ 4r (see problem 5.5.7a).
(af) We have that (see the proof of problem 7.1.37q)
pffiffiffiffiffi pffiffiffiffiffi pffiffiffiffiffi
pffiffiffiffiffi 2 bc α pffiffiffiffiffi 2 ac β pffiffiffiffiffi 2 ab γ
la þ lb þ lc ¼ bc  cos þ ac  cos þ ab  cos 
b þ c 2 a þ c 2 a þ b 2
sffiffiffiffiffiffiffiffiffiffiffiffiffiffiffiffiffiffiffiffiffiffiffiffiffiffiffiffiffiffiffiffiffiffiffiffiffiffiffiffiffiffiffiffiffiffiffiffiffiffiffiffiffiffiffiffiffiffiffiffiffiffiffiffiffiffiffiffiffiffiffiffiffiffiffiffiffiffiffiffiffiffiffiffiffiffiffiffiffiffiffiffiffiffiffiffiffiffiffiffiffiffiffiffiffiffiffiffiffiffiffiffiffiffiffiffiffiffiffiffiffiffiffi
pffiffiffiffiffiffiffiffiffiffiffiffiffiffiffiffiffiffiffiffiffiffiffiffiffi 2bc 2ac 2ab
 ab þ bc þ ac ð1 þ cos αÞ þ ð1 þ cos βÞ þ ð1 þ cos γ Þ ¼
ðb þ c Þ2 ða þ cÞ2 ða þ b Þ2
sffiffiffiffiffiffiffiffiffiffiffiffiffiffiffiffiffiffiffiffiffiffiffiffiffiffiffiffiffiffiffiffiffiffiffiffiffiffiffiffiffiffiffiffiffiffiffiffiffiffiffiffiffiffiffiffiffiffiffiffiffiffiffiffiffi
pffiffiffiffiffiffiffiffiffiffiffiffiffiffiffiffiffiffiffiffiffiffiffiffiffi a2 b2 c2
¼ ab þ bc þ ac  3    
ðb þ cÞ ða þ cÞ ða þ cÞ2
2 2
sffiffiffiffiffiffiffiffiffiffiffiffiffiffiffiffiffiffiffiffiffiffiffiffiffiffiffiffiffiffiffiffiffiffiffiffiffiffiffiffiffiffiffiffiffiffiffiffiffiffiffiffiffiffiffiffiffiffiffi
 2
pffiffiffiffiffiffiffiffiffiffiffiffiffiffiffiffiffiffiffiffiffiffiffiffiffi 1 a b c 3pffiffiffiffiffiffiffiffiffiffiffiffiffiffiffiffiffiffiffiffiffiffiffiffiffi
 ab þ bc þ ac 3  þ þ  ab þ bc þ ac
3 bþc aþc aþc 2

(see the proof of problem 5.5.19b).


7.1.38. (a) Denote by xA ¼ MA MA
1
, xB ¼ MB
MB
1
, xC ¼ MC MC
1
and xD ¼ MDMD
1
. The ratio of the
altitudes AP and MU of the tetrahedrons ABCD and MBCD is equal to 1 þ xA : 1.
Hence, VVMBCD
ABCD
¼ 1þx
1
A
. Similarly, VVMACD
ABCD
¼ 1þx1
, V MABD ¼ 1þx
B V ABCD
1
C
and VVMABC
ABCD
¼ 1þx1
D
. Apart
from that, since VMBCD þ VMACD þ VMABD þ VMABC ¼ VABCD, then 1þx 1
A
þ 1þxB þ
1

1þxC þ 1þxD ¼ 1. xA ¼ 1þxB þ 1þxC þ 1þxD 


1 1 1þxA 1þxA 1þxA
Thus, it follows that
7.2 Solutions 341

qffiffiffiffiffiffiffiffiffiffiffiffiffiffiffiffiffiffiffiffiffiffiffiffiffiffiffiffiffiffiffi
ð1þxA Þ 3 pffiffiffiffiffiffiffi
x þ y þ z  3 3 xyz, for x, y, z  0. Similarly, xB 
3
3 1þxB Þð1þxC Þð1þxD Þ, as
qðffiffiffiffiffiffiffiffiffiffiffiffiffiffiffiffiffiffiffiffiffiffiffiffiffiffiffiffiffiffiffi
3
q ffiffiffiffiffiffiffiffiffiffiffiffiffiffiffiffiffiffiffiffiffiffiffiffiffiffiffiffiffiffiffi qffiffiffiffiffiffiffiffiffiffiffiffiffiffiffiffiffiffiffiffiffiffiffiffiffiffiffiffiffiffi

ð1þxC Þ3 ð1þxD Þ3
3 ð1þxA Þðð1þx BÞ
 
3 3 3
1þxC Þð1þxD Þ , x C 3 ð1þxA Þð1þxB Þð1þxD Þ , xD 3 ð1þxA Þð1þxB Þð1þxC Þ.
Multiplying these four inequalities, we deduce that xAxBxCxD  81. Hence, it
follows that MA  MB  MC  MD  81MA1  MB1  MC1  MD1.
(b) Consider a plane α, passing through point M and perpendicular to segment DM.
Then it is clear that plane α intersects at least one of the edges DA, DB, DC,
otherwise we obtain that point M is outside of the tetrahedron ABCD. Let DB be
that edge, then we have that ∠BMD > π2. Consequently, ∠BMD > ∠MDB.
Thus, it follows that BD > BM.
(c) Let the straight line DM and the face ABC intersect at point D1. Let lines AD1
and BC intersect at point D2. Then, we have that DM < DD1 < max (AD, DD2)
and DD2 < max (BD, CD).
Thus, it follows that DM < max (AD, BD, CD).
(d) According to the problem 7.1.38a, without loss of generality, we can assume
DM  3D1M. Consider a plane passing through point M and parallel to plane
ABC. Let A2, B2, C2 be the intersection points of that plane with the edges AD,
BD, CD, respectively. Let also max(A2D, B2D, C2D) ¼ A2D, then according to
the triangle inequality and the problem 7.1.38c), we deduce that

AM þ BM þ CM þ DM < AA2 þ A2 M þ BB2 þ B2 M þ CC2 þ C2 M þ A2 D 


BD CD BD CD
 AD þ þ þ A2 M þ B2 M þ C2 M < AD þ þ þ A2 B2 þ B2 C 2 þ A2 C 2 <
4 4 4 4
BD CD
< AD þ þ þ AB þ BC þ AC
4 4

(see problem 1.1.8c). Consequently,


AM þ BM þ CM þ DM < AB þ BC þ CD þ AD þ BD þ AC (see problem
7.1.38e).
(e) We need to prove that, if for any distinct points M1 and M2 point M is the inner
point of segment M1M2, then f(M ) < max ( f(M1), f(M2)), where f(X) ¼
AX þ BX þ CX þ DX.
Indeed, let MM11MM2 ¼ MM11MA0 ¼ α, where M0 is the point of segment AM1 (if M1  A,
then we take M0 on segment AM2). Then, we obtain that AM  AM0 þ M0M ¼
(1  α)AM1 þ αAM2, the equality holds true, if and only if A is on the straight line
M1M2.
Then at least one of the vertices of the tetrahedron ABCD does not belong to the
straight line M1M2. Thus it follows that

f ðMÞ < ð1  αÞf ðM1 Þ þ αf ðM2 Þ  maxðf ðM1 Þ; f ðM2 ÞÞ: ð7:15Þ
342 7 Miscellaneous Inequalities

Figure 7.29 P0 P l1

A A1 B1 B l

P P

a-j
j a j

l l
A H B A B H
а) b)

Figure 7.30

According to the inequality (7.4) (see the notations of problem 7.1.38c, it


follows f(M ) < max ( f(D), f(D1)) < max ( f(D), f(A), f(D2)) < max ( f(D), f(A), f(B),
f(C)), thus f(M ) < max ( f(D), f(A), f(B), f(C)). This ends the proof.
7.1.39. (a) Draw through point P a line l1 parallel of the straight line l (Figure 7.29).
Take on the straight line l1 a point P0, such that P0A ¼ P0B. Then the straight line
_
l1 touches the circumcircle of triangle AP0B. We have that ∠AP0 B ¼ AB
2 > ∠APB.
Consequently, one can take points A1 and B1 on segment AB such that
∠A1P0B1 ¼ ∠APB and A1P0 ¼ B1P0. Hence, AB  A1B1 ¼ A0B0.
(b) Let PH ⊥ l and PA  PB (Figure 7.30).
Consider Figure 7.30a (in the case of Figure 7.30b, the proof is similar).
Denote by ∠APB ¼ α, PH ¼ h, ∠BPH ¼ φ. We have that ∠APH ¼ α 
φ  φ ¼ ∠BPH.
Then, we have to prove that cos ðhαφÞ  cosh φ  hðtg ðα  φÞ þ tgφÞ  2htgα2, that
 
2 sin α2  φ sin α2 2 sin α2 cos α2 2 sin α2
is,   , or
cosðα  φÞcos φ cos ðα  φÞ cos φ cos α2
α α α
sin  φ cos  cos 2  cos ðα  φÞ cos φ,

2  2
α 2
α  α  π α 
sin  φ cos  sin 2  φ ¼ sin  φ cos  þφ
2 2 2 2 2 2
The last inequality holds true since 0  α2  φ < π2, 0  α2 < π2  α2 þ φ  π2.
Remark If PA 6¼ PB, then |PA  PB| > AB  A0 B0.
7.1.40. Consider points C0 and Q0 symmetric to points C and Q with respect to the
straight line AD (Figure 7.31).
7.2 Solutions 343

Figure 7.31 A

Q⬘
Q R

P C⬘

C D B

Note that ΔPQC ¼ ΔPQ0 C0 , hence ∠PC0 B ¼ π  ∠PCQ > ∠PBC0 . Thus, it
follows that PB > PC0 ¼ PC.
Since ∠BPR ¼ ∠CPQ ¼ ∠C0 PQ0 , then ∠RPQ0 ¼ ∠BPC0 .
0
According to problem 7.1.39b, we have that PB  PC0 > BC0  B0 C0 , where B0
0 0 0
, C0 2 AB and ∠B0PC0 0 ¼ ∠BPC0 , ∠PB0 C0 ¼ ∠PC 0 B0 . Moreover, according to
problem 7.1.39a, it follows that Q0 R  B0 C00 . Hence PB  PC0 > BC0  Q0 R. This
means that PB  PC > BR  QC since BC0  Q0 R ¼ BR  Q0 C0 ¼ BR  QC.
We have to prove that BR > QC. We have that ρ(P, AC) ¼ ρ(P, AB)
BR
Hence QC ¼ SSPQC
PBR
¼ PQPC
PRPB
. Since AB > AC, then hc < hb. Note that hc ¼ CPþPR
PR 
ρðP; ABÞ and hb ¼ PBþPQ PQ  ρðP; ACÞ. Thus,
CPþPR
PR < PBþPQ
PQ , consequently
CP  PQ < PR  PB. Therefore, BR > QC.
7.1.41. (a) If P belongs to the altitude AH, then |∠PAB  ∠PAC| ¼
0 ¼ |∠PBC  ∠PCB|. Otherwise consider points P0 and P symmetric to each
other with respect to the straight line AH (Figure 7.32).
Note that |∠PAB  ∠PAC| ¼ ∠PAP0 and |∠PBC  ∠PCB| ¼ ∠PCP0 ¼
∠PRP0 . Hence, we have to prove that RE  AE.
_
Since ∠BP0 C þ ∠BRC ¼ 180 , ∠BN0 C þ ∠BSC ¼ 180 and ∠BP0 C ¼ BRC2
> ∠BN 0 C, then ∠BRC < ∠BSC. Thus, it follows that RE > SE.
We need to prove that SE  AE. This means that SN0  N0 A ¼ NN0 .
_ 
_ _
Let ∠NBN0 ¼ α. Since α  60 and SBN 0 ¼ 180  α, N 0 QN ¼ 2α, then SBN 0 
_
N 0 QN. Consequently, SN0  NN0 .
(a) Let ∠PBC ¼ β  ∠PCB ¼ γ, then PC  PB. Therefore P is inside of the triangle
ABH, where AH is the altitude of triangle ABC.
Consequently, ∠PAC ¼ 30 þ α, ∠PAB ¼ 30  α, where 0  α  30 .
We have to prove that β  γ  arcsin (2 sin α)  α  2α, or sin(β  γ þ α) 
2 sin α  sin 3α.
Note that sin3α  sin α ¼ 2 sin α cos 2α  sin α, since 0  2α  60 .
We have to prove that sin(β  γ) cos α þ cos (β  γ) sin α  2 sin α, or
tgα  2sincosðβγ Þ
ðβγ Þ.
344 7 Miscellaneous Inequalities

Figure 7.32

According to the law of cosines, it follows that


 
sin ð30 þαÞ sin ð60 βÞ sin γ
1¼ PC
PA  PA
PB  PB
PC ¼ 
sin ð60 γ Þ
 
sin ð30 αÞ
 sin β.
 
sin ð30 αÞ sin ð60 βÞ sin γ
Consequently 
sin ð30 þαÞ
¼
sin ð60 γ Þsinβ


pffiffi pffiffi
ffiffi ¼ pffiffi ð 3 cos β sin βÞ sin γ
Thus, 1p3 tgα
. Hence, we deduce that tgα ¼
1þ 3tgα ð 3 cos γ sin γÞ sin β
sin ðβγ Þ
pffiffi .
3 sin ðβþγ Þ2 sin β sin γ
sin ðβγ Þ pffiffiffi
We have to prove that pffiffi3 sin ðβþγ Þ2 β γ
 2sincosðβγ Þ
ðβγ Þ or 2  cos ðβ  γ Þ  3
pffiffi sin sin
sin ðβ þ γ Þ  2 sin β sin γ, 1  2 sin ðβ þ γ Þ þ 2 cos ðβ þ γ Þ, 1  sin (β þ γ þ 30 ).
3 1 0

h h0 0 h h0 0 0
0 þ
cc0 ¼ sin γ sin γ þ sin β sin β  sin γ cos β þ cos γ
a a a a
7.1.42. We have that bb
sin β0 ¼ sin ðγ þ β0 Þ  1, as γ 0  90  β0 and β  90  γ. Thus, ha1h0  bb1 0 þ cc1 0 .
a

7.1.43. Denote by ∠PAB ¼ α, ∠QAD ¼ β, ∠BCE ¼ γ, ∠DCF ¼ δ and BP ¼ b,


DQ ¼ c, DF ¼ a, BE ¼ d (see Figure 7.33).
Note that ∠PAQ þ ∠ECF ¼ π2  ðα þ βÞ þ ðγ þ δÞ þ π2 ¼ π þ γ þ δ  α  β,
hence we have to prove that 0  γ þ δ  α  β < π4.
7.2 Solutions 345

Figure 7.33
E
d
P g C
B b

d
Q

a c

b
A D a F

 
Since γ þ δ, α þ β 2 0; π2 , then π2 < γ þ δ  α  β < π2.
Thus, it is sufficient to prove that 0  tg(γ þ δ  α  β) < 1.
Let AB ¼ 1, we have that

tg ðγ þ δÞ  tg ðα þ βÞ aþd
 bþc
tg ðγ þ δ  α  βÞ ¼ ¼ 1adaþd 1bc :
1 þ tg ðγ þ δÞtg ðα þ βÞ 1 þ 1ad  1bc
bþc

Since ΔDQF ΔBEP, then ad ¼ bc.


Similarly, we have that ΔDQF ΔPQC and ΔBEP ΔPQC. Thus, one can
easily deduce that a ¼ cð1c
1bÞ
and d ¼ bð1b
1cÞ
.
ðaþdbcÞð1bcÞ
We have that tg ðγ þ δ  α  βÞ ¼ ð1bcÞ2 þðaþdÞðbþcÞ and a þ d  b  c ¼
ðcbÞ2
ð1cÞð1bÞ 0.
Therefore, it follows that tg(γ þ δ  α  β)  0.
aþdÞðbþcÞ, since 1  bc < 1 and (1  bc) > 0.
Note that tg ðγ þ δ  α  βÞ < ðaþdbc 2

aþdÞðbþcÞ < ðaþdÞðbþcÞ ¼ bþc  1. Thus, we obtain that tg


If b þ c  1, then ðaþdbc aþd 1

(γ þ δ  α  β) < 1.
If b þ c < 1, then a þ d ¼ cð1c1bÞ
þ bð1b
1cÞ
< 1c
c
þ 1b
b
< 1cb
c
þ 1cb
b
, hence
a þ d  (a þ d)(c þ b) < c þ b. Consequently, tg ðγ þ δ  α  βÞ < ðaþdÞðbþcÞ < 1.
aþdbc

Thus, tg(γ þ δ  α  β) < 1. This ends the proof.


7.1.44. Denote the radius of the middle sphere by R. Let l \ P ¼ O, and A, B, C be
the points at which the spheres with radiuses 1, R, r touch plane P (Figure 7.34),
respectively. pffiffi pffiffiffi pffiffiffiffiffi
We have that OA ¼ 1, OB ¼ R, OC ¼ r and AC ¼ 2 r , AB ¼ 2 R, BC ¼ 2 Rr .
According
pffiffi to the triangle p inequality,
ffiffiffi it follows that OC þ AC  OA, thus
1  r þ 2 r . Hence, r  3  2 2p . ffiffiffi pffiffiffi
Note that the equality r ¼ 3  2 2 is possible. Indeed, at r ¼ 3  2 2 we have
that OC þ AC ¼ OA, thus C lies on segment OA (Figure 7.35).
346 7 Miscellaneous Inequalities

Figure 7.34 A

C O B

Figure 7.35 A 1-r C r


O

2 Rr
2 R R

Figure 7.36 C

D A
K
B E

Using Stewart’s theorem, we get that 4Rr ¼ 4Rr þ R2(1  r)  r(1  r) or


pffiffiffi pffiffiffi pffiffiffi 2 pffiffiffi
R ¼ 2  1. Since 1 > R ¼ 2  1 > 2  1 ¼ r and pffiffiffiffiffiR þ 2 R > rþ
pffiffi
2 r ¼ 1, then triangle AOB exists, which means that BC ¼ 2 Rr and thus there
exist three spheres satisfying the conditions of the problem.
7.1.45. Note that (see Figure 7.36) ∠CEA ¼ 180  70  55 ¼ 55 . Hence,
EA ¼ CA ¼ AD. Consequently ∠KAD ¼ 5 , ∠KAC ¼ 65 . In triangle ACE , we
have that CE > AC; thus CE > CD. Let ∠DCN ¼ ∠ECN (Figure 7.37). Then,
 
NE ¼ CE < 1. Hence ∠KCE < 2, 5 ¼ ∠NCE and ∠KCA > ∠ECA ¼ 55 .
DN DC
 
Consequently, ∠AKC ¼ 115  ∠KCA > 57, 5 and ∠AKC ¼ 115 

∠KCA < 60 .
7.1.46. (a) Let SABC ¼ S, then SAMC ¼ SBMA ¼ SCMB ¼ S3.
We have that ∠MAB ¼ ∠MCA; consequently ΔAMK ΔACK, where K is the
qffiffiffiffiffiffiffiffiffiffiffiffiffiffiffiffiffiffi
2b2 þ2c2 a2
intersection point of lines CM and AB. Thus, mac ¼ mcb, since ma ¼ 4
qffiffiffiffiffiffiffiffiffiffiffiffiffiffiffiffiffiffi pffiffi pffiffi
2a2 þ2b2 c2
and mc ¼ 4 , we obtain that 2c ¼ a þ b and ma ¼ 2 , mb ¼ 2 .
2 2 2 3b 3a
7.2 Solutions 347

Figure 7.37 C

D
N
E

Note that

2SAMC 2SCMB S S
sin ∠CAM þ sin ∠CBM ¼ þ ¼ þ ¼
   AC BM  BC m2 a b 2 mb a
AM
1 a b 1 a b 1 a þb
¼ þ sin γ ¼ pffiffiffi þ sin γ ¼ pffiffiffi  sin γ:
2 ma mb 3 b a 3 ab

Since 2c2 ¼ a2 þ b2, then according to the law of cosines, we deduce that
a2 þb2
ab ¼ 4 cos γ. Thus, sin ∠CAM þ sin ∠CBM ¼ p2ffiffi3 sin 2γ. Hence, sin ∠CAMþ
sin ∠CBM  p2ffiffi3.

(b) Consider two circles passing through points M and C and touching the straight
line AB at points B1 and A1, where point B1 belongs to ray KB, and K is the point
of intersection of lines CM and AB.
Since KB21 ¼ KM  KC ¼ KA21 and MK CM
¼ 2, then point M is the orthocenter of
triangle CA1B1. We have that ∠CBM  ∠CB1M and ∠CAM  ∠CA1M.
If ∠CB1M > 90 , then let us consider points E and N on segment A1M, such that
ME ¼ EN ¼ 18 MA1 . Then, NP k B1C, where P is the intersection point of lines B1M
and A1C.
We have that ∠MPN ¼ ∠MB1C > 90 ; consequently point P is inside of the
circle with a diameter MN. Hence EP < 12 MN. According to the law of sines,
sin ∠MA1 C ¼ EP sinA1∠A
E
1 PE
 AEP
1E
< 2A
MN
1E
¼ 17. Hence, it follows that sin ∠CBM þ sin
∠CAM  1þ sin ∠CA1 M < 87 < p2ffiffi3, since ∠CAM  ∠CA1M < 90 .
It remains to consider the case when ∠CB1M  90 and ∠CA1M  90 .
According to problem 7.1.46a, it follows that sin ∠CAM þ sin ∠CBM
 sin ∠CA1 M þ sin ∠CB1 M  p2ffiffi3.

Second Solution We have that sin ∠CAM þ sin ∠CBM ¼ mSa b þ mSb a.
abma mb
We have to prove that S  pffiffi pffiffi .
2
3
b  m a þ 2
3
a  m b
x2 þ y2 pffiffi pffiffi
Since xy  , then 23 b  ma þ 23 a  mb 
 2 
1 3 2 1 3 2 a2 þ b2 þ c2
 b þ m2a þ a þ m2b ¼ : Hence, it is sufficient to
2 4 2 4 2
2 þb2 þc2 , or (a þ b þ c ) sin γ  4mamb.
prove that S  a2abm a mb 2 2 2
348 7 Miscellaneous Inequalities

qffiffiffiffiffiffiffiffiffiffiffiffiffiffiffiffiffiffiffiffiffiffiffiffiffiffiffiffiffiffiffiffiffiffiffiffiffiffiffiffiffiffiffiffiffiffiffiffiffiffiffiffiffiffiffiffiffiffiffiffiffiffiffiffiffiffiffiffiffiffiffiffiffiffiffiffiffiffiffi
   ffi
We have that 4ma mb ¼ 2 2c2  a2  b2 c2 þ a2 þ b2 þ 9a2 b2 .
Denote by c2 ¼ x, ab cos γ ¼ y. Then we have to prove that
qffiffiffiffiffiffiffiffiffiffiffiffiffiffiffiffiffiffiffiffiffiffiffiffiffiffiffiffiffiffiffiffiffiffiffiffiffiffiffiffiffiffiffiffiffiffiffiffiffiffiffi

9y2
2ðx þ yÞ sin γ  2ðx  2yÞð2x þ 2yÞ þ cos 2 γ , or

4cos 2 γx2  4xyð1 þ 2sin 2 γ Þ þ 98coscosγsin


2 2
2γ y  0,
2γ 2

 pffiffiffiffiffiffiffiffiffiffiffiffiffiffiffiffiffiffiffiffiffiffiffiffiffiffi 2
98cos 2 γsin 2 2γ
2 cos γx  cos γ y  0, since

     2
9  8cos 2 γ  sin 2 2γ ¼ 9  8 1  sin 2 γ  4sin 2 γ 1  sin 2 γ ¼ 1 þ 2sin 2 γ :

7.1.47. Let the numbers a, b, c and m, n correspond in the complex plane to the
vertices of triangle ABC and points M, N respectively. Then, we have to prove that
jmajjnaj jmbjjnbj jmcjjncj
jbajjcaj þ jabjjcbj þ jacjjbcj  1.
Note that

ðm  aÞðn  aÞ ðm  bÞðn  bÞ ðm  cÞðn  cÞ


þ þ ¼
ðb  aÞðc  aÞ ða  bÞðc  bÞ ða  cÞðb  cÞ
mnðc  b þ a  c þ b  aÞ  mðaðc  bÞ þ bða  cÞ þ cðb  aÞÞ
¼
ða  bÞðb  cÞðc  aÞ
nðaðc  bÞ þ bða  cÞ þ cðb  aÞÞ þ a2 ðc  bÞ þ b2 ða  cÞ þ c2 ðb  aÞ
¼
ða  bÞðb  cÞðc  aÞ
 2 
a2 ðc  bÞ þ b2 ða  cÞ þ c2 ðb  aÞ a  b2 c  abða  bÞ  c2 ða  bÞ
¼ ¼ ¼
ða  bÞðb  cÞðc  aÞ ða  bÞðb  cÞðc  aÞ
ac þ bc  ab  c2
¼ ¼ 1:
ð b  c Þ ð c  aÞ

Consequently, according to the triangle inequality,


 
ðm  aÞðn  aÞ ðm  bÞðn  bÞ ðm  cÞðn  cÞ
1 ¼  þ þ 
ð b  aÞ ð c  aÞ ð a  bÞ ð c  bÞ ð a  c Þ ð b  c Þ 
   
ðm  aÞðn  aÞ ðm  bÞðn  bÞ ðm  cÞðn  cÞ
  þ  þ  
ðb  aÞðc  aÞ ða  bÞðc  bÞ   ða  cÞðb  cÞ 
     
ðm  aÞðn  aÞ ðm  bÞðn  bÞ ðm  cÞðn  cÞ
  þ þ ¼
ðb  aÞðc  aÞ   ða  bÞðc  bÞ   ða  cÞðb  cÞ 
jm  ajjn  aj jm  bjjn  bj jm  cjjn  cj
¼ þ þ ,
jb  ajjc  aj ja  bjjc  bj ja  cjjb  cj

thus
7.2 Solutions 349

jm  ajjn  aj jm  bjjn  bj jm  cjjn  cj


þ þ  1:
jb  ajjc  aj ja  bjjc  bj ja  cjjb  cj

Remark If points M and N coincide, then we obtain the inequality of


problem 4.1.8b.
7.1.48. Let the numbers a1, a2, . . . , an and b1, b2, . . . , bn  1 correspond in the
complex plane to points A1, A2, . . . , An and B1, B2, . . . , Bn  1 respectively.
By mathematical induction we need to prove that

ðb1  a1 Þðb2  a1 Þ  :::  ðbn1  a1 Þ ðb1  a2 Þðb2  a2 Þ  :::  ðbn1  a2 Þ


þ þ :::
ða2  a1 Þða3  a1 Þ  :::  ðan  a1 Þ ða1  a2 Þða3  a2 Þ  :::  ðan  a2 Þ
ðb1  an Þðb2  an Þ  :::  ðbn1  an Þ
þ ¼1
ða1  an Þða2  an Þ  :::  ðan1  an Þ

Indeed, for n ¼ 2, we have that ba12 a b1 a2


a1 þ a1 a2 ¼ 1.
1

Let for n ¼ k the statement holds true. Let us prove that the statement holds true
also for n ¼ k þ 1.
Consider the expression pðzÞ ¼ ððab21a
a1 Þ:::ðbk1 a1 Þðza1 Þ
1 Þ:::ðak a1 Þðakþ1 a1 Þ
þ :::þ
ðb1 akþ1 Þ:::ðbk1 akþ1 Þðzakþ1 Þ
ða1 akþ1 Þ:::ðak1 akþ1 Þðak akþ1 Þ.
It has a form of Az þ B, for the value z ¼ a1. We have that
pða1 Þ ¼ ððba13 a 2 Þ:::ðbk1 a2 Þ ðb1 akþ1 Þ:::ðbk1 akþ1 Þ
a2 Þ:::ðakþ1 a2 Þ þ ::: þ ða2 akþ1 Þ:::ðak akþ1 Þ ¼ 1, since for numbers a2,
a3, . . . , ak þ 1 and b1, . . . , bk  1 the statement holds true (n ¼ k). Similarly, we
obtain that p(a2) ¼ 1, Thus Aa1 þ B ¼ 1 and Aa2 þ B ¼ 1.
Hence, A ¼ 0 and B ¼ 1, then p(bk) ¼ 1.
For the numbers z1, . . . , zn one can easily prove that |z1| þ . . . þ |zn|  |
z1 þ ... þ zn| (see the proof of problem 7.1.47); hence from the equality (*), we
obtain the given inequality.
7.1.49. Let numbers a1, a2, . . . , an and b1, b2, . . . , bn  2 correspond in the complex
plane to points A1, . . . , An and B1, . . . , Bn  2 respectively.
By mathematical induction we need to prove that

ðb1  a1 Þðb2  a1 Þ  :::  ðbn2  a1 Þ ðb1  an Þðb2  an Þ  :::  ðbn2  an Þ


þ ::: þ ¼ 0 ð∗ Þ
ða2  a1 Þða3  a1 Þ  :::  ðan  a1 Þ ða1  an Þða2  an Þ  :::  ðan1  an Þ

Indeed, for n ¼ 3, we have that

b1  a1 b1  a2 b1  a3
þ þ ¼
ða2  a1 Þða3  a1 Þ ða1  a2 Þða3  a2 Þ ða1  a3 Þða2  a3 Þ
b1 ða3  a2 þ a1  a3 þ a2  a1 Þ  a1 ða3  a2 Þ  a2 ða1  a3 Þ  a3 ða2  a1 Þ
¼ ¼ 0:
ða1  a2 Þða2  a3 Þða3  a1 Þ

Let for n ¼ k (*) holds true.


350 7 Miscellaneous Inequalities

We need to prove that (*) holds true also for n ¼ k þ 1.


Consider the expression

ðb1  a1 Þ  :::  ðbk2  a1 Þðz  a1 Þ


pð z Þ ¼ þ :::
ða2  a1 Þ  :::  ðakþ1  a1 Þ
ðb1  akþ1 Þ  :::  ðbk2  akþ1 Þðz  akþ1 Þ
þ
ða1  akþ1 Þ  :::  ðak  akþ1 Þ

We have that p(z) ¼ Az þ B and

ðb1  a1 Þ  :::  ðbk2  a1 Þ ðb1  ak Þ  :::  ðbk2  ak Þ


pðakþ1 Þ ¼ þ ::: þ ¼ 0 ðn ¼ kÞ:
ða2  a1 Þ  :::  ðak  a1 Þ ða1  ak Þ  :::  ðak1  ak Þ

Similarly, we obtain that p(ak) ¼ 0.


Hence, it follows that A ¼ 0, B ¼ 0.
Thus, it follows that p(bk  1) ¼ 0.
Therefore, we have that
 
B1 An  B2 An  :::  Bn2 An ðb1  an Þðb2  an Þ  :::  ðbn2  an Þ
¼ ¼
A1 An  A2 An  :::  An1 An ða1  an Þða2  an Þ  :::  ðan1  an Þ
 
ðb1  a1 Þ  :::  ðbn2  a1 Þ ðb1  an1 Þ  :::  ðbn2  an1 Þ 
¼  þ ::: þ 
ða2  a1 Þ  :::  ðan  a1 Þ ða1  an1 Þ  :::  ðan2  an1 Þðan  an1 Þ
   
ðb1  a1 Þ  :::  ðbn2  a1 Þ
 þ ::: þ ðb1  an1 Þ  :::  ðbn2  an1 Þ ¼
 
   
ða2  a1 Þ  :::  ðan  a1 Þ ða1  an1 Þ  :::  ðan  an1 Þ 
B1 A1  :::  Bn2 A1 B1 An1  :::  Bn2 An1
¼ þ ::: þ :
A2 A1  :::  An A1 A1 An1  :::  An An1

This ends the proof.


7.1.50. Let M1, A1, B1, C1 be such points that their H images are points M, A, B, C
(Figure 7.38).
(see the proof of problem 4.1.8c).

B
B1

H - image M
M1

A1 C1
A C

Figure 7.38
7.2 Solutions 351

We have that

MA ¼ M1 B1  M1 C1 , MB ¼ M1 A1  M1 C1 , MC ¼ M1 A1  M1 B1 , AB ¼ M1 C1  A1 B1 ,
BC ¼ B1 C1  M1 A1 , AC ¼ M1 B1  A1 C1

(such points exist, since ∠A1MB1 ¼ ∠AMB,


qffiffiffiffiffiffiffiffiffiffiffi qffiffiffiffiffiffiffiffiffiffiffi∠B1M1C1 q
¼ ffiffiffiffiffiffiffiffiffiffiffi
∠B MC, ∠A1M1C1 ¼
∠AMC and A1 M1 ¼ MA , B1 M1 ¼
MBMC
MB , C1 M1 ¼
MAMC MAMB
MC ).
We have to prove that

M1 A1  M1 C1 sin∠A1 M1 C1 þ M1 B1  M1 C1 sin∠B1 M1 C1 þ M1 A1  M1 B1 sin∠A1 M1 B1 


1
 ðM1 A1  B1 C1 þ M1 B1  A1 C1 þ M1 C1  A1 B1 Þ:
2

Note that

M1 A1  M1 C1 sin ∠A1 M1 C1 þ M1 B1  M1 C1 sin ∠B1 M1 C1 þ M1 A1  M1 B1 sin ∠A1 M1 B1 ¼


A 1 M 1  B 1 C1 B 1 M 1  A 1 C1 C 1 M 1  A 1 B 1
¼ 2SA1 B1 C1 ¼ SA1 B1 M1 C1 þ SA1 B1 C1 M1 þ SA1 M1 B1 C1  þ þ ,
2 2 2

where the equality holds true, if A1M1 ⊥ B1C1, B1M1 ⊥ A1C1, C1M1 ⊥ A1B1. Then,
∠MAB ¼ ∠M1B1A1 ¼ ∠M1 C1A1 ¼ ∠MAC. Similarly, we obtain that
∠MBA ¼ ∠MBC and ∠MCB ¼ ∠MCA. This means that point M coincides with
the incenter of triangle ABC.
This ends the proof.
7.1.51. There are two possible cases (Figure 7.39a, b).

!

!
!
!
!
!
Let us construct vectors OX ¼ MM1 , OY ¼ NN 1 and OZ ¼ PP 1 (Figure 7.40).

We need to prove that triangle XYZ is equilateral. Denote by R60 ~a the image of
a obtained by rotation by an angle of 60 . Note that
the vector ~

N b
M1 B
N1 b M
N1 M1
N

M
a g a g
A P P1 C A P P1 C
а) b)

Figure 7.39
352 7 Miscellaneous Inequalities

Figure 7.40 X
Y

1800-g

1800-b
O Z O a Z 1800-b
g
1800-a

Y X
a b


! 

!
! 
!



! 
R60 XY ¼ R60 OY  OX ¼ ¼ R60 NN 1  MM1 ¼


!

!  
! 

!
!

!
¼ R60 NM  N 1 M1 ¼ ¼ R60 NM  R60 N 1 M 1 ¼ PM  P1 M1 ¼

!

!
!
!
!
¼ PP 1  MM1 ¼ OZ  OX ¼ XZ ,

!
!
hence R60 XY ¼ XZ . Consequently, triangle XYZ is equilateral.
In case (a), according to problem 1.1.11а, one can construct a triangle using
segments OX ¼ MM1, OY ¼ NN1 and OZ ¼ PP1.
In case (b), according to problem 1.2.9, one cannot construct a triangle using
segments OX, OY and OZ only if points O, X, Y, Z are on the same circumference,
and in the last case, we obtain that α ¼ β ¼ γ ¼ 60 . Let K and K1 be the centers of
circumcircles of triangles MNP and M1N1P1 respectively. Since β ¼ 60 , NKM d ¼

120 and NK ¼ KM, then point K is on the bisector of the angle B. Similarly, we need
to prove that point K is on the bisector of angle A. Hence point K is the incenter of
triangle ABC. The same is true also for point K1, because points K and K1 coincide.
But then we have the result from the case (a). This leads to a contradiction.
Remark See problem 7.1.9.
7.1.52. According to problem 7.1.43, it follows that ∠ECF  135 .
Therefore γ þ δ  45 (see the solution of this
pffiffiproblem).
ffi We have that AE þ AF
γþδ π
¼ 2 þ tgγ þ tgδ  2 þ 2tg 2  2 þ 2tg 8 ¼ 2 2 (see the proof of problem
5.2.2а).
a2i
7.1.53. Note that bi¼ ai  b1i ¼ sin ∠Aaiiþ1 Ai A ¼ 2R  aaiþ1i (an þ 1 ¼ a1), i ¼ 1, 2, . . . , n.
 ai 
a2 a2
Hence, b11 þ ::: þ bnn ¼ 2R aa12 þ aa23 þ ::: þ aan1 þ aan1 . According to Cauchy’s inequal-
qffiffiffiffiffiffiffiffiffiffiffiffiffiffiffiffiffiffiffiffiffiffiffiffiffiffiffiffiffiffiffiffiffiffiffiffiffiffiffiffiffi
n
a21 a22
ity, a1
a2 þ a2
a3 þ ::: þ an1
an þ an
a1  n a2  a3  :::  an  a1 ¼ n. Thus,
n a1 a2 an1 an
b1 þ b2 þ :::þ
a2n
bn  2nR.
AB sin φ
7.1.54. (a) Let ∠ABM ¼ ∠MBN ¼ ∠NBC ¼ φ. We have that MC AM
¼ SSMBC
ABM
¼ BC sin 2φ.
BC sin φ
Similarly, we deduce that CN
AN ¼ AB sin 2φ . Thus, 4AMNC
ANCM ¼ 1
cos 2 φ > 1. Hence,
4AM  NC > AN  CM.
7.2 Solutions 353

(b) Denote the points symmetric to points B and C with respect to the straight lines
AC and AB by B1 and C1 respectively. We have that BN ¼ B1N and CM ¼ C1M.
Consequently BN þ MN þ CM ¼ B1N þ MN þ C1M  B1C1.
We need to prove that B1C1  2BC, then BN þ MN þ CM  2BC.
Let AB ¼ c, AC ¼ b, BC ¼ a, ∠A ¼ α, ∠B ¼ β, ∠C ¼ γ, and R be the
circumradius of triangle ABC. Then cos ∠B1AC1 ¼ cos 3α and AB1 ¼ c, AC1 ¼ b.
According to the law of cosines, we have that b2 þ c2  a2 ¼ 2bc cos α; then
(b þ c2  a2)sin2α ¼ bc sin α sin 2α.
2
Therefore, 4(b2 þ c2  a2)sin2α ¼ 2bc
cos α  2bc cos 3α, Thus
 
4 b2 þ c2  a2 sin 2 α  3a2 ¼ B1 C21  4a2 ð7:16Þ
pffiffi pffiffiffi
Since a ¼ 2R sin α and maxðb; cÞ ¼ 2Rmaxð sin β; sin γ Þ  2R 23 ¼ R 3, we
have that b2 þ c2  a2  3R2  0. Then, from (7.2), we deduce that B1C1  2a.
7.1.55. Denote by OA ¼ x, OB ¼ y, OC ¼ z, OD ¼ t and ∠AOB ¼ α. Note that

Sa x Sd Sb y Sa
¼ ¼ , ¼ ¼ ð7:17Þ
Sb z Sc Sc t Sd
According to (7.3), we deduce that

Sa Sc ¼ Sb Sd ð7:18Þ

and

1 1 1 1 1 xþyþa zþtþc yþzþb xþtþd
þ   ¼ þ   ¼
ra rc rb rd 2 Sa Sc Sb Sd
   
1 x z y t z x t y a c b d
¼  þ  þ  þ  þ þ   ¼
2 Sa Sb Sa Sd Sc Sd Sc Sb Sa Sc Sb Sd

1 a c b d
¼ 0þ0þ0þ0þ þ  
2 Sa Sc Sb Sd

Hence, we obtain that



1 1 1 1 1 a c b d
þ   ¼ þ   ð7:19Þ
r a r c r b r d 2 Sa Sc Sb Sd

We need to prove that


 2  2
a c b d ða þ c Þ2 ðb þ d Þ2
þ  þ ¼  ð7:20Þ
Sa Sc Sb Sd Sa Sc Sb Sd

Then from (7.4), (7.5), and (7.6), we obtain the proof of the case (a).
By using the law of cosines, we deduce that
354 7 Miscellaneous Inequalities

a2 ¼ x2 þ y2  4Sa ctgα,
b2 ¼ y2 þ z2 þ 4Sb ctgα,
ð7:21Þ
c2 ¼ z2 þ t2  4Sc ctgα,
d2 ¼ x2 þ t2 þ 4Sd ctgα

Consequently,

ð4Sb þ 4Sd þ 4Sa þ 4Sc Þctgα ¼ b2 þ d 2  a2  c2 ð7:22Þ

We have that
 2  2
Sa Sc a
Sa þ Scc  Sb Sd b
Sb þ Sdd ¼

a2 c2 b2 d2
¼  Sc þ  Sa   Sd  Sb þ 2ac  2bd ¼
Sa Sc Sb Sd
x2 þ y2  4Sa ctgα z2 þ t2  4Sc ctgα y2 þ z2 þ 4Sb ctgα
¼ Sc þ Sa  Sd 
Sa Sc Sb
 2 2  2 2
x2 þ t2 þ 4Sd ctgα x Sc z2 S2d y Sc t2 S2b
 Sb þ 2ac  2bd ¼  þ  þ
Sd Sa Sc Sd Sb Sa Sc Sb Sd
 2 2 2 2
z Sa x2 S2b t Sa y2 S2d
þ  þ   ð4Sc þ 4Sa þ 4Sd þ 4Sb Þctgα þ 2ac  2bd ¼
Sa Sc Sb Sd Sa Sc Sb Sd
¼ 0 þ 0 þ 0 þ 0 þ a2 þ c2  b2  d 2 þ 2ac  2bd ¼ ða þ cÞ2  ðb þ d Þ2
  22
We obtain that Sa Sc  Sb Sd Sbb þ Sdd ¼ ða þ cÞ2  ðb þ dÞ2 .
a
Sa þ Scc
 2  2
Þ2 ðbþdÞ2
According to (7.4), we have that Saa þ Scc  Sbb þ Sdd ¼ ðaþc
Sa Sc  Sb Sd .

(b) Denote by OA1 ¼ x1, OB1 ¼ y1, OC1 ¼ z1 and OD1 ¼ t1 (Figure 7.41).
α π α
Since 2 ¼ ∠I a OB1 ¼ ∠I c OD1 and 2  2 ¼ ∠I b OC1 ¼ ∠I d OA1 , then
ΔIaOB1 ΔIcOD1 ΔIbOC1 ΔIdOA1. Thus,

Figure 7.41 b C
B Ib
B1 rb
ra C1
a A1 O rc Ic c
Ia
rd D1
A Id

D
7.2 Solutions 355

t1 r c
x 1 y1 ¼ r a r d , y1 z1 ¼ r a r b , ¼ ð7:23Þ
y1 r a

We have that a þ c > b þ d and x1 þ z1 ¼ xþyþzþtbd 2 , y1 þ t1 ¼ xþyþzþtac


2 .
Hence, x1 þ z1 > y1 þ t1.
Consequently, according to (7.12) and r1a þ r1c > r1b þ r1d (see problem 7.1.55a),
   
we obtain that rayrd þ rya rb > y1 þ rrac y1 ¼ r c y1 r1c þ r1a > r c y1 r1b þ r1d . Thus,
qffiffiffiffiffiffiffiffiffi 1 1

y1 < ra rb rd
rc .
b OI d
OI a OI c ¼ y  t1 ¼ r c y2 > 1. Hence OIb  OId > OIa  OIc.
Note that OI rb rd ra rb rd
1 1
If we draw a circle through points Ia, Ib, Id, then according to the last inequality,
point Ic is inside of that circle. Thus ∠IdIaIb þ ∠IdIcIb > π.
7.1.56. Since ∠D1C1D2 ¼ ∠D1B1D2, then points D1, C1, B1 , and D2 (Figure 7.42)
are located on the same circumference. Hence, ∠C1D1B1 ¼ ∠C1D2B1 ¼ ∠C2D2B2.
Similarly, we find that ∠C2D2B2 ¼ ∠C2D3B2 and so on.
Denote by φ ¼ ∠C1D1B1 ¼ ∠C1D2B1 ¼ ∠C2D2B2 ¼ ∠C2D3B2 ¼ . . . ¼
∠CnD1Bn.
According to the law of sines, we have that Dsin1 Dβ2 ¼ 2R1 ¼ Bsin
1 C1
φ, where R1 is the
1
circumradius of the quadrilateral D1C1B1D2. Consequently, B1C1 sin β1 ¼
D1D2 sin φ. Similarly, we obtain that B2C2 sin β2 ¼ D3D2 sin φ and,
BnCn sin βn ¼ DnD1 sin φ.
Thus, we have that

A3 C3 B3

B2 D3
C2

D2

A2

B1
C1
D1 Dn

A1 Bn Cn An

Figure 7.42
356 7 Miscellaneous Inequalities

B1 C1 sin β1 þ ::: þ Bn1 Cn1 sin βn1 ¼ D1 D2 sin φ þ D2 D3 sin φ þ ::: þ Dn1 Dn sin φ >
> D1 Dn sin φ ¼ Bn Cn sin βn :

Hence, B1C1 sin β1 þ . . . þ Bn  1Cn  1 sin βn  1 > BnCn sin βn.


Remark See problems 7.1.9 and 7.1.51.
7.1.57. Note that, if a1, a2, a3, b1, b2, b3 > 0, then according to the Cauchy’s
inequality
rffiffiffiffiffiffiffiffiffiffiffiffiffiffiffiffiffiffiffiffiffiffiffiffiffiffiffiffiffiffiffiffiffiffiffiffiffiffiffiffiffiffiffiffiffiffiffiffiffiffiffi rffiffiffiffiffiffiffiffiffiffiffiffiffiffiffiffiffiffiffiffiffiffiffiffiffiffiffiffiffiffiffiffiffiffiffiffiffiffiffiffiffiffiffiffiffiffiffiffiffiffiffi
a1 a2 a3 b1 b2 b3
  þ   
3 3

a1 þ b1 a2 þ b2 a3 þ b3 a1 þ b1 a2 þ b2 a3 þ b3
a1 a2 a3 b1 ba2 ba3
þ þ þ þ
a þ b1 a2 þ b2 a3 þ b3 a1 þ b1 a2 þ b2 a3 þ b3
 1 þ ¼ 1:
3 3
Hence,
p ffiffiffiffiffiffiffiffiffiffiffiffiffiffiffiffiffiffiffiffiffiffiffiffiffiffiffiffiffiffiffiffiffiffiffiffiffiffiffiffiffiffiffiffiffiffiffiffiffiffiffiffiffiffiffi pffiffiffiffiffiffiffiffiffiffiffiffiffi pffiffiffiffiffiffiffiffiffiffiffiffiffi
3
ða1 þ b1 Þða2 þ b2 Þða3 þ b3 Þ 3 a1 a2 a3 þ 3 b1 b2 b3 : ð7:24Þ

According to the inequality (7.24), we have that


sffiffiffiffiffiffiffiffiffiffiffiffiffiffiffiffiffiffiffiffiffiffiffiffiffiffiffiffiffiffiffiffiffiffiffiffiffiffiffiffiffiffiffiffiffiffiffiffiffiffiffiffiffiffiffiffiffiffiffi
  ffi
p þ λa p þ λb p þ λc
¼
3

pa pb pc


s ffiffiffiffiffiffiffiffiffiffiffiffiffiffiffiffiffiffiffiffiffiffiffiffiffiffiffiffiffiffiffiffiffiffiffiffiffiffiffiffiffiffiffiffiffiffiffiffiffiffiffiffiffiffiffiffiffiffiffiffiffiffiffiffiffiffiffiffiffiffiffiffiffiffiffiffiffiffiffiffiffiffiffiffiffiffiffiffiffiffiffiffiffiffiffiffiffiffiffiffiffiffiffiffiffiffiffiffiffiffiffiffiffiffiffiffiffiffiffiffi
 
a b c
¼ 1 þ ðλ þ 1Þ 1 þ ðλ þ 1Þ 1 þ ðλ þ 1Þ 
3

pa pb pc


rffiffiffiffiffiffiffiffiffiffiffiffiffiffiffiffiffiffiffiffiffiffiffiffiffiffiffiffiffiffiffiffiffiffiffiffiffiffiffiffiffiffiffi
abc
 1 þ ðλ þ 1Þ 3 ¼
ðp  aÞðp  bÞðp  cÞ
sffiffiffiffiffiffiffiffiffiffiffiffiffiffiffiffiffiffiffiffiffiffiffiffiffiffiffiffiffiffiffiffiffiffiffiffiffiffiffiffiffiffiffiffiffiffiffiffiffiffiffiffiffiffiffiffiffiffiffiffiffiffiffiffiffiffiffiffiffiffiffiffiffiffiffiffiffiffiffiffiffiffiffiffiffiffiffiffiffiffiffiffi
3 ðp  b þ p  cÞðp  a þ p  cÞðp  a þ p  bÞ
¼ 1 þ ðλ þ 1Þ 
ðp  aÞðp  bÞðp  cÞ
sffiffiffiffiffiffiffiffiffiffiffiffiffiffiffiffiffiffiffiffiffiffiffiffiffiffiffiffiffiffiffiffiffiffiffiffiffiffiffiffiffiffiffiffiffiffiffiffiffiffiffiffiffiffiffiffiffiffiffiffiffiffiffiffiffiffiffiffiffiffiffiffiffiffiffiffiffiffiffiffiffiffiffiffiffiffiffiffiffiffiffiffiffiffiffiffiffiffiffiffiffiffiffiffiffiffiffiffiffiffiffiffiffiffi
pffiffiffiffiffiffiffiffiffiffiffiffiffiffiffiffiffiffiffiffiffiffiffiffiffiffiffiffiffi pffiffiffiffiffiffiffiffiffiffiffiffiffiffiffiffiffiffiffiffiffiffiffiffiffiffiffiffiffi pffiffiffiffiffiffiffiffiffiffiffiffiffiffiffiffiffiffiffiffiffiffiffiffiffiffiffiffiffi
3 2 ðp  bÞðp  cÞ  2 ðp  aÞðp  cÞ  2 ðp  aÞðp  bÞ
 1 þ ðλ þ 1Þ ¼ 2λ þ 3:
ðp  aÞðp  bÞðp  cÞ

Thus, ðpþλa ÞðpþλbÞðpþλcÞ 3


ðpaÞðpbÞðpcÞ  ð2λ þ 3Þ .

7.1.58. Denote by ∠PBC ¼ α1, ∠APC ¼ β1, ∠APB ¼ γ 1. Let R1 be the radius of the
circumference passing through points P, B, A0 and C. According to the law of sines,
sin γ 1 sin β1
we have that BA0 þ A0 C ¼ 2R1 ð sin ðπ  γ 1 Þ þ sin ðπ  β1 ÞÞ ¼ a sin α1 þ a sin α1 .
sin γ 1 sin α1
Similarly, we deuce that AB0 þ B0 C ¼ b sin β þ b sin β and
1 1
α1 sin β1
A0 C þ C0 B ¼ c sin
sin γ þ c sin γ . Hence, it follows that
1 1
7.2 Solutions 357

Figure 7.43 A

L K E
O

B C

AB0 þ B0 C þ CA0 þ A0 B þ BC0 þ C0 A ¼


  
sin γ 1 sin α1 sin β1 sin α1 sin γ 1 sin β1
¼ a þc þ a þb þ b þc 
sin α1 sin γ 1 sin α1 sin β1 sin β1 sin γ 1
rffiffiffiffiffiffiffiffiffiffiffiffiffiffiffiffiffiffiffiffiffiffiffiffiffiffiffiffiffiffiffiffiffiffi rffiffiffiffiffiffiffiffiffiffiffiffiffiffiffiffiffiffiffiffiffiffiffiffiffiffiffiffiffiffiffiffiffiffi rffiffiffiffiffiffiffiffiffiffiffiffiffiffiffiffiffiffiffiffiffiffiffiffiffiffiffiffiffiffiffiffiffi
sin γ 1 sin α1 sin β1 sin α1 sin γ 1 sin β1
2 a c þ2 a b þ2 b c ¼
sin α1 sin γ 1 sin α1 sin β1 sin β1 sin γ 1
pffiffiffiffiffi pffiffiffiffiffi pffiffiffiffiffi
¼ 2 ac þ 2 ab þ 2 bc:
pffiffiffiffiffi pffiffiffiffiffi pffiffiffiffiffi
Thus, AB0 þ B0 C þ CA0 þ A0 B þ BC0 þ C0 A  2 ac þ 2 ab þ 2 bc.
7.1.59. Since DE k BC and ∠DBK ¼ ∠KBC, then ∠DKB ¼ ∠KBC ¼ ∠DBK (see
Figure 7.43).
Hence DK ¼ BD ¼ AD. Consequently ∠AKB ¼ 90 . Similarly, we obtain that
∠ALC ¼ 90 . Consequently, points K and L are on the circumference with a

diameter AO. Thus, AO  LK . (Note that ∠LOK ¼ ∠BOC ¼ 90 þ ∠A 2 ; this
means that AO > LK.) According to the triangle inequality, BO þ CO > BC;
hence AO þ BO þ CO > LK þ BC.
7.1.60. (a) For a non-obtuse triangle, we have that a ¼ ha(ctgβ þ ctgγ),
b ¼ hb(ctgα þ ctgγ), c ¼ hc(ctgα þ ctgβ), where ctgα, ctgβ, ctgγ  0.
Note that ctgαctgβ þ ctgβctgγ þ ctgγctgα ¼ 1. Indeed, we have that
ctgγ ¼ ctg ðα þ βÞ ¼ 1ctgαctgβ
ctgαþctgβ , or ctgαctgβ þ ctgβctgγ þ ctgγctgα ¼ 1.
Denote by ctgα ¼ x, ctgβ ¼ y, ctgγ ¼ z, then we need to prove that, if x, y, z  0
and x þ y > 0,y þ z > 0, x þ z > 0, then
!
1 1 1 9
ðxy þ yz þ zxÞ 2
þ 2
þ 2
 : ð7:25Þ
ðx þ y Þ ðy þ zÞ ðx þ zÞ 4

The last inequality is equivalent to the following inequality.

2xyzðxðx  yÞðx  zÞ þ yðy  zÞðy  xÞ þ zðz  xÞðz  yÞÞ þ xyðx2 þ xy þ y2 Þðx  yÞ2 þ
2
þyzðy2 þ yz þ z2 Þðy  zÞ2 þ xzðx2 þ xz þ z2 Þðx  zÞ2 þ 3xyðx2  y2 Þ þ
2 2
þ3yzðy2  z2 Þ þ 3xzðx2  z2 Þ  0:

It remains to prove that x(x  y)(x  z) þ y(y  z)(y  x) þ z(z  x)(z  y)  0.


358 7 Miscellaneous Inequalities

Indeed, let min(x, y, z) ¼ z  0, then x(x  y)(x  z) þ y(y  z)(y  x) þ z(z  x)


(z  y) ¼ (x  y)2(x þ y  z) þ z(z  x)(z  y)  0, since (x  y)2  0, x þ y  z  0,
z  0, (z  x)(z  y)  0.
As

h2a h2b h2c 1 1 1


þ þ ¼ þ þ ¼
a2 b2 c2 ðctgα þ ctgβÞ2 ðctgβ þ ctgγ Þ2 ðctgγ þ ctgαÞ2

1 1
¼ ðctgαctgβ þ ctgβctgγ þ ctgγctgαÞ 2
þ þ
ðctgα þ ctgβÞ ðctgβ þ ctgγ Þ2
! !
1 1 1 1 9
þ ¼ ðxy þ yz þ xzÞ þ þ  :
ðctgγ þ ctgαÞ2 ðx þ yÞ2 ðy þ zÞ2 ðx þ zÞ2 4

h2 h2 h2
Therefore, aa2 þ bb2 þ c2c  94.
Remark Another proof of the inequality (7.25) can be obtained from
problem 5.5.10c.
pffiffiffi
(b) Let a  b  c, then we have to prove that 2 3S  pa, or 3( p  a)(a2 
(b  c)2)  pa2, 3(b þ c  a)(a2  (b  c)2)  (a þ b þ c)a2.
Let a ¼ 1, c ¼ b þ x, then p
from
ffiffi
ffi conditions a2 þ b2  c2, we obtain that 1  x2
þ 2bx  x2 þ 2x. Hence, x  2  1.
We have to prove that 3(2b þ x  1)(1  x2)  1 þ 2b þ x, b(4  6x2) 
4  2x  3x2 þ 3x3.
We have that b(4  6x2)  4  6x2  4  2x  3x2 þ 3x3.
Let a < b. (If a ¼ b, then this distance is equal to zero, and
7.1.61.pffiffiffiffiffiffiffiffiffiffiffiffiffiffiffiffiffi
aþb
2  pðp  cÞ > aþb 2 
pþpc
2 ¼ 0.)
γ
þða sin γ Þ
We have that ρðM; lÞ ¼ b sin 2 2 2 (see Figure 7.44).

Figure 7.44 y
C

g g
2 2
a b

B
х
M
l
A
7.2 Solutions 359

qffiffiffiffiffiffiffiffiffiffiffiffi qffiffiffiffiffiffiffiffiffiffiffiffiffiffiffiffiffiffi
1 cos γ ðpaÞðpbÞ
Therefore, ρðM; lÞ ¼ ba
2 2 ¼ ¼ ba
2 ab . Note that
rffiffiffiffiffiffiffiffiffiffiffiffiffiffiffiffiffiffiffiffiffiffiffiffiffiffiffiffiffi rffiffiffiffiffiffiffiffiffiffiffiffiffiffiffiffiffiffiffiffiffiffiffiffiffiffiffiffiffiffiffiffiffiffiffiffiffiffiffiffiffiffiffiffiffiffiffiffiffiffiffiffiffiffiffiffiffiffiffiffiffiffiffiffiffiffiffiffiffiffiffiffiffiffi
ðb  aÞ ðp  aÞðp  bÞ 1   p2  pða þ bÞ þ ab
¼ ða þ bÞ2  4ab ¼
2 ab 2 ab
vffiffiffiffiffiffiffiffiffiffiffiffiffiffiffiffiffiffiffiffiffiffiffiffiffiffiffiffiffiffiffiffiffiffiffiffiffiffiffiffiffiffiffiffiffiffiffiffiffiffiffiffiffiffiffiffiffiffiffiffiffiffiffiffiffiffiffiffiffiffiffiffiffiffiffiffiffiffiffiffiffiffiffiffiffiffiffiffiffiffiffiffiffiffiffiffiffiffiffiffiffiffiffiffiffiffiffiffiffiffiffiffiffiffiffiffiffiffiffiffiffiffiffiffiffiffiffiffiffiffiffi
!
u
1u ða þ b Þ 2
ð p 2
þ p ð a þ b Þ Þ
¼ tða þ bÞ þ 4pða þ bÞ  4p2 
2
þ 4ab 
2 ab
qffiffiffiffiffiffiffiffiffiffiffiffiffiffiffiffiffiffiffiffiffiffiffiffiffiffiffiffiffiffiffiffiffiffiffiffiffiffiffiffiffiffiffiffiffiffiffiffiffiffiffiffiffiffiffiffiffiffiffiffiffiffiffiffiffiffiffiffiffiffiffiffiffiffiffiffiffiffiffiffiffiffiffiffiffiffiffiffiffiffiffiffiffiffiffiffiffiffiffiffiffiffiffiffiffiffiffiffi
pffiffiffiffiffiffiffiffiffiffiffiffiffiffiffiffiffiffiffiffiffiffiffiffiffiffiffi
1
 ða þ bÞ2 þ 4pða þ bÞ  4p2  4ða þ bÞ pða þ bÞ  p2 ¼
2
a þ b pffiffiffiffiffiffiffiffiffiffiffiffiffiffiffiffiffiffiffiffiffiffiffiffiffiffiffi2 a þ b pffiffiffiffiffiffiffiffiffiffiffiffiffiffiffiffiffi
¼  pð a þ bÞ  p ¼  pð p  c Þ :
2 2
pffiffiffiffi
Remark The equality holds true when cos 2γ ¼ 2aþb ab
.
qffiffiffiffiffiffiffiffiffiffiffiffiffiffiffiffiffiffi qffiffiffiffiffiffiffiffiffiffiffiffiffiffiffiffiffiffiffiffiffiffiffiffiffiffiffiffiffiffiffiffiffiffiffiffiffiffiffiffiffiffiffiffiffiffi2ffi
7.1.62. We have that mc ¼ 2a þ2b c2
¼ pðp  cÞ þ ððpaÞ4ðpbÞÞ and we have
2 2
4
rffiffiffiffiffiffiffiffiffiffiffiffiffiffiffiffiffiffiffiffiffiffiffiffiffiffiffiffiffiffiffiffiffiffiffiffiffiffiffiffiffiffiffiffiffiffiffiffiffiffiffiffiffiffiffiffiffiffiffiffiffiffiffiffiffiffiffiffiffiffiffiffiffiffiffiffiffiffiffiffiffiffiffiffiffi
 2 qffiffiffiffiffiffi qffiffiffiffiffiffi pffiffiffiffi
to prove that A ¼ p þ4
pc 1 pa
p  p
pb
þ pa p þ p 
pb
3.
qffiffiffiffiffiffi qffiffiffiffiffiffi qffiffiffiffiffiffi qffiffiffiffiffiffi
Denote by pa
p ¼ x þ t, pb
p ¼ x  t ; then 2x ¼ p þ
pa pb
p
rffiffiffiffiffiffiffiffiffiffiffiffiffiffiffiffiffiffiffiffiffiffiffiffiffiffi
  qffiffiffiffi pffiffiffi pffiffi
 2 p þ p ¼ 2c
pa pb
p < 2. Consequently, 0 < x < 22 and

rffiffiffiffiffiffiffiffiffiffiffiffiffiffiffiffiffiffiffiffiffiffiffiffiffiffiffiffiffiffiffiffiffiffiffiffiffiffiffiffiffiffiffiffiffiffiffiffiffiffiffiffiffiffiffiffiffiffiffiffiffiffiffiffiffiffiffiffiffiffiffiffiffiffiffiffiffiffiffiffiffiffiffiffiffiffiffiffiffiffiffiffiffiffiffiffi
1 2
A ¼ 1  ðx þ tÞ2  ðx  tÞ2 þ ðx þ tÞ2  ðx  tÞ2 þ 2x ¼
4
pffiffiffiffiffiffiffiffiffiffiffiffiffiffiffiffiffiffiffiffiffiffiffiffiffiffiffiffiffiffiffiffiffiffiffiffiffiffiffiffiffiffiffiffiffi pffiffiffiffiffiffiffiffiffiffiffiffiffiffiffi pffiffiffi
¼ 1  2x  2ð1  2x Þt þ 2x  1  2x2 þ 2x  3,
2 2 2

pffiffiffi 2
since 1  2x2  3  2x .
pffiffiffi 2
This means that 3x  1  0.
BE
7.1.63. Figure 7.45 is correct since EM ¼ AM
AB
< AM
BC
¼ CM
BC
¼ FM
BF BE
or EM þ 1 < FM
BF
þ 1,
FM < EM.

Figure 7.45 B

E
I
F

A M C
360 7 Miscellaneous Inequalities

Consider triangle AIC. According to Menelaus’s theorem, it follows that


AE
IE FC  MA ¼ 1, or
IF CM
CF ¼ IF. We have that
AE IE
∠EFI ¼ ∠C
2 þ ∠FBC and
∠IEF ¼ 2 þ ∠ABE. Consequently, ∠EFI < 2 < ∠Bþ∠A
∠A ∠Bþ∠C
2 < ∠IEF.
Thus, EI < IF, hence AE < CF. This ends the proof.
7.1.64. From the statement of the problem it follows that triangle ABC is either
acute or right. Let triangle ABC be acute and point N be the midpoint of segment
AB, then MN ¼ BC 2 > 2 ¼ NB, ∠ABM > ∠BMN ¼ ∠MBC.
AB

Figure 7.46 is correct since ∠ABH ¼ 90  ∠A < ∠B 2 < ∠ABM.


We have to prove that HF > HE (see the proof of problem 7.1.63), or
∠HEF ¼ 90  ∠B þ ∠ABM > 90  ∠B þ ∠MBC ¼ ∠HFE is correct.
If triangle ABC is right-angled, then if ∠B ¼ 90 , we have that
AE ¼ AB < BC ¼ CF, while if ∠A ¼ 90 , we deduce that CF ¼ CM ¼ AM > AE,
since ∠AEM ¼ 90  ∠MBC > ∠C þ ∠MBC ¼ ∠AFE.
7.1.65. Consider three cases (see Figure 7.47, I, II, III).
(I) For triangle AKC and line BM, according to Menelaus’s theorem, we obtain
that EK  FC  MA ¼ 1, CF
AE FK CM AE
¼ KE
KF < 1, since ∠EFK ¼ ∠FCB þ ∠MBC <
∠EAB þ ∠MBA ¼ ∠KEF.
(see the proof of problem 7.1.64).
(II) Since ∠A > ∠C, then ∠EAC > ∠ECA. Hence, CF > EA.

Figure 7.46 B

E
N H F

A M C

B B
B
E
K E=F
F F К
E

A M C A M C A M C
I II III

Figure 7.47
7.2 Solutions 361

(III) We have that AE


EK  FK
FC  MA ¼ 1. Consequently,
CM AE
CF ¼ KE
EF < 1, since
∠EFK < ∠KEF.
∠BAE sin ∠BCF sin ∠BCF
(b) We have that BE ¼ AE sin sin ∠ABE ¼ AE sin ∠ABE < AE sin ∠FBC, since ∠FBC <
 sin ∠BCF ∠BCF
∠ABE < 180  ∠FBC. Hence, BE < AE sin ∠FBC < CF sinsin ∠FBC ¼ BF
(see problem 7.1.65a). Thus, BE < BF.
7.1.66. Denote by AB ¼ a, ∠EFA ¼ α, ∠BAP ¼ β, ∠DAQ ¼ γ, then we have to
prove that

∠EPA > 90 , ð7:26Þ

∠FQA > 90 , and β þ γ  45 .


(а) To prove (7.26), we have to prove that AB  BE > BP2. This means that point P is
inside of the circle with a diameter AE.
2
¼ ðð2aaðsin αaÞctgαÞ
¼ 1 þ cos αsinsin2 α2α1 < 1.
2
BP
We have that ABBE 2a sin αaÞ
Similarly, we need to prove that ∠FQA > 90 .
(b) We have to prove that tg(β þ γ)  1, that is tgβ þ tgγ þ tgβtgγ  1.
We have that tgβ ¼ BP
AB ¼ ð2 sin α  1Þctgα and tgγ ¼ (2 cos α  1)tgα.
2
Therefore, tgγ þ tgβ þ tgγtgβ ¼ 1  sincosα cos

α  1.

7.1.67. Let point P be inside triangle ABO (Figure 7.48).


_ _
Note that if P ≢ O, then P1 B < B1 P2, for which it is sufficient to draw a diameter
_ _
parallel to BB1. Similarly, P1 A < A1 P2, consequently ∠PBA1 > ∠BA1P.
Hence, it follows that PA1 > PB.
We have that (1  x)(1 þ x) ¼ PA  PA1 > PA  PB and PC  PO þ OC ¼ 1 þ x.
Thus, PA  PB  PC < (1 þ x)2(1  x).
If P  O, then PA  PB  PB ¼ 1 and x ¼ 0. Hence, AP  BP  CP ¼ (1 þ x)2(1  x).
2 2
LP ¼ LPAL ¼ BLLC ¼
7.1.68. We have that AL , since ΔBAL ΔAPC. Thus,
AL AL ABACBLLC
BLLC
AC ¼ ALþLP, or AL ¼ AB  AC  AL  LP ¼ AB  AC  BL  LC. Consequently,
AL AB 2

AL
LP ¼ ABAC
BLLC  1  ðBLþLC
4ABAC
Þ2
 1 ¼ 4ABAC
BC2
 1.

Figure 7.48 B A1
P1
P

A C
O

P2
B1
362 7 Miscellaneous Inequalities

BM
Similarly, we deduce that QM  4ABBC
AC2
 1 and CN
RN  AB2  1.
4ACBC

Summing up the above inequalities, we obtain that



AL BM CN AB  AC AB  BC AC  BC
þ þ 4 þ þ 3
LP QM RN BC2 AC2 AB2
rffiffiffiffiffiffiffiffiffiffiffiffiffiffiffiffiffiffiffiffiffiffiffiffiffiffiffiffiffiffiffiffiffiffiffiffiffiffiffiffiffiffiffiffiffiffiffiffiffiffiffiffiffiffi
3 AB  AC AB  BC AC  BC
 12    3 ¼ 9:
BC2 AC2 AB2

LP þ QM þ RN  9.
Hence AL BM CN

Note that the equality holds true, only if BL ¼ LC, AM ¼ CM, AN ¼ BN and
ABAC
BC2
¼ ABBC
AC2
¼ ACBC
AB2
. This means that, triangle ABC is equilateral.
See also problem 7.1.5.
2π ðn1Þ
7.1.69. (a) Let z1 ¼ 1, z2 ¼ cos 2π n þ i sin n , :::, zn ¼ cos

n þ i sin 2π ðn1
n
Þ
cor-
respond in the complex plane to the vertices of the regular n-gon A1A2 . . . An. Then,
the numbers z1, . . . , zn are the roots of the polynomial zn  1so that zn  1 ¼
(z  z1) . . . (z  zn). For points M(z) we have that z 2 R and z > 1; consequently,

MA1  MA2  :::  MAn ¼ jz  z1 jjz  z2 j  :::  jz  zn j ¼ jðz  z1 Þðz  z2 Þ  :::  ðz  zn Þj ¼


¼ jzn  1j ¼ zn  1 ¼ MOn  1:

P
n qffiffiffiffiffiffiffiffiffiffiffiffiffiffiffiffiffiffiffiffiffiffiffiffiffiffiffiffiffiffiffiffiffiffi
According to Cauchy’s inequality, we have that 1
MAi  n n MA
1
1
 MA 1
2
 :::  MA 1
n
.
i¼1
P
n P
n
Thus 1
MAi  n pn ffiffiffiffiffiffiffiffiffiffiffiffiffiffiffiffiffi
1
MA1 :::MAn
ffi ¼ pn ffiffiffiffiffiffiffiffiffiffiffi
n
MOn 1
> n
MO, consequently 1
MAi > MO
n
.
i¼1 i¼1

(b) We have that ∠Ai MAiþ1 ¼ αi > [Ai2Aiþ1 ¼ πn , i ¼ 1, 2, :::, n, Anþ1  A1 and
α1 þ α2 þ . . . þ αn ¼ 2π.
Let us prove that, if πn < αi < π, i ¼ 1, 2, :::, n and α1 þ α2 þ . . . þ αn ¼ 2π, then
π
sin α1 þ sin α2 þ ::: þ sin αn > sin 2π n þ ðn  2Þ sin n :
π π
Note that, if α > n , β > n and α þ β < 2π, then

π  π
sin α þ sin β > sin þ sin α þ β  :
n n

Let αn ¼ max (α1, α2, ..., αn). Thus, it follows that sinα1 þ sin α2 þ :::þ
sin αn1 > ðn  2Þ sin πn þ sin α1 þ α2 þ ::: þ αn1  ðn  2Þπn : 
It is sufficient to note that sin α1 þ α2 þ ::: þ αn1  ðn  2Þπn þ sin αn  sin
n þ sin π ¼ sin n : sin α1 þ sin α2 þ :::þ
2π 2π
Hence, we obtain that
sin αn > sin n þ ðn  2Þ sin πn :

This ends the proof.


 
7.1.70. Denote ∠NAV ¼ α, ∠NAB ¼ β, ∠PMU ¼ φ. It is clear that α, β, φ, 2 0; π2
(Figure 7.49a, b).
7.2 Solutions 363

B
B P
P1 U P1
U

P
M M

j
N j N
b Q1 b
a a Q V
A V Q A Q1
l l
a b

Figure 7.49

Consider Figure 7.49a. If QQ1  PP1, then PQ  P1Q1  UV. Hence, PQ  UV.
Applying the law of sines to triangles VQQ1 and UPP1 , we obtain that

QQ1 VQ1 sin α P1 U sin β VQ1 sin α cos ðβ  φÞ VM sin α cos ðβ  φÞ


¼ : ¼  ¼  ¼
PP1 cos ðα þ φÞ cos ðβ  φÞ P1 U cos ðα þ φÞ sin β MU cos ðα þ φÞ sin β
UN sin α cos ðβ  φÞ ANtgβ sin α cos ðβ  φÞ cos ðβ  φÞ  cos α
¼  ¼  ¼ ¼
VN cos ðα þ φÞ sin β ANtgα cos ðα þ φÞ sin β cos β  cos ðα þ φÞ
cos α cos β cos φ þ sin β sin φ cos α
¼ > 1,
cos α cos β cos φ  sin α sin φ cos β

since cos(α þ φ) > 0 and sin φ sin (α þ β) > 0.


Let us consider Figure 7.49b and prove that QQ1  PP1, then PQ  P1Q1  UV.
Indeed, in the same way as in the first case, we deduce that

QQ1 VQ1 sin α P1 U sin β cos α cos ðβ þ φÞ


¼ : ¼  ¼
PP1 cos ðα  φÞ cos ðβ þ φÞ cos ðα  φÞ cos β
cos α cos β cos φ  sin β sin φ cos α
¼ < 1,
cos α cos β cos φ þ sin α sin φ cos β

as sin φ sin (α þ β) > 0 and cos(α  φ) > 0.


7.1.71. (a) First we need to prove that there exists a triangle ABC and a point
M inside of it, such that αβγ has the greatest value.
Consider the set X of all possible products αβγ. Since, α, β, γ < π, then αβγ < π 3.
We have obtained that the set X is bounded from above; hence it has a supremum.
Let sup X ¼ a. If a 2 X, then this ends the proof of the statement.
Suppose that a 2 X. Then there exists a sequence of triangles AnBnCn and points
Mn (Mn 2 ΔAnBnCn) such that lim ðαn βn γ n Þ ¼ a, where αn ¼ ∠MnAnBn, βn ¼
n!1
∠MnBnCn,γ n ¼ ∠MnCnAn.
364 7 Miscellaneous Inequalities

0 0 0
  α0n¼ ∠M
 0Let 0
 n An Cn , βn ¼ ∠Mn Bn A, γ n ¼ ∠Mn Cn Bn . Since the ðαn Þ, ðβn Þ, ðγ n Þ,
αn , βn , γ n sequences are bounded, then without loss of Generality, one can
assume that they are convergent. Let lim αn ¼ α, lim βn ¼ β, lim γ n ¼ γ,
n!1 n!1 n!1
lim α0 ¼ α0 , lim β0n ¼ β0 lim γ 0n ¼ γ 0 . Then, we have that α þ α0 þ β þ β0 þ γ þ γ 0
n!1 n  n!1 n!1 
¼ lim αn þ α0n þ βn þ β0n þ γ n þ γ 0n ¼ π and a ¼ lim ðαn βn γ n Þ ¼ αβγ, hence
n!1 n!1
α, β, γ > 0.
According to the law of sines we have that,

Mn An Mn Bn Mn Cn sin β0n sin γ 0n sin α0n


1¼   ¼    Consequently;
Mn Bn Mn Cn Mn An sin αn sin βn sin γ n
 
sin αsin β sin γ ¼ lim ð sin αn sin βn sin γ n Þ ¼ lim sin α0n sin β0n sin γ 0n ¼ sin α0 sin β0 sin γ 0 :
n!1 n!1

Hence,

sin α sin β sin γ ¼ sin α0 sin β0 sin γ 0 ð7:27Þ

Thus α0 , β0 , γ 0 > 0.
Now, consider triangle ABC and point M inside of it, such that ∠MAB ¼ α,
∠MBA ¼ β0 , ∠MAC ¼ α0 , ∠MBC ¼ β, then ∠ACB ¼ γ þ γ 0 .
00
Let ∠MCB ¼ γ , then
 0 0
sin α sin β sin γ þ γ 0  γ 0 ¼ sin α0 sin β0 sin γ 0 ð7:28Þ

sin γ sin ðγþγ 0 γ 0 0 Þ


From (7.27) and (7.28), we obtain that sin γ0 ¼ sin γ 0 0
,
00
Thus sin(γ þ γ 0 )ctgγ 0  cos (γ þ γ 0 ) ¼ sin (γ þ γ 0 )ctgγ  cos (γ þ γ 0 )
00
Hence γ 0 ¼ γ .
We have obtained that a 2 X. This leads to a contradiction.
This ends the proof.
Thus, there exists a triangle ABC and a point M inside of it, such that the product
αβγ has the greatest value. We need to prove that α0 ¼ β0 ¼ γ 0 .
Indeed, let for example α0 6¼ β0 , then let us draw through points B and M a circle
touching ray KA at point A0, where K is the intersection point of the straight lines
BM and AC. Since, α0 6¼ β0 , then points A and A0 are different. Thus α < α0, where
α0 ¼ ∠MA0B, hence a ¼ αβγ < α0βγ.
This leads to a contradiction.
Now by using the following lemma, we can prove that α ¼ β ¼ γ.
Lemma If point M given inside of triangle ABC is, such that α ¼ ∠MAB 6¼
∠MBC ¼ β and α þ β þ 2 ∠MBA < π, then there exists such a point D on the
straight line BC, that point M is again inside of triangle ADC and α1β1 > αβ,
where α1 ¼ ∠MAD, β1 ¼ ∠MDC.
Let D be any point of line BC (M 2 △ ADC). Suppose that α1β1  αβ.
Then α ¼ β. This leads to a contradiction.
7.2 Solutions 365

Indeed, let ∠DAB ¼ x, ∠DMB ¼ y. Note  that x ! 0 , y ! 0,


Therefore, lim xy ¼ lim sinx x  sin
sin y 
x sin y
y ¼ lim sin x
sin y.
x!0 x!0 x!0
Let ∠ABM ¼ φ. According to the law of sines,

sin x sin x sin β sin ðβ þ φÞ


¼  
sin y sin ðβ þ φÞ sin y sin β
BD MD sin ðβ þ φÞ MD sin ðβ þ φÞ
¼   ¼  :
AD BD sin β AD sin β

Consequently

x sin x MB sin ðβ þ φÞ sin α sin ðβ þ φÞ


lim ¼ lim ¼  ¼ : ð7:29Þ
x!0 y x!0 sin y AB sin β sin ðα þ φÞ sin β

If point D is inside of segment BC (M 2 ΔADC), then α1β1 ¼ (α  x)(β þ y)  αβ.


α α
Consequently, βþy  xy; hence lim βþy  lim xy, thus
x!0 x!0

α x
 lim : ð7:30Þ
β n!0 y

If point D is outside of segment BC(M 2 ΔADC), then α1β1 ¼ (α þ x)


(β  y)  αβ. Consequently, xy  αþx
β , thus

x α
lim  ð7:31Þ
x!0 y β
sin α sin ðβþφÞ
From (7.29), (7.30), and (7.31), we obtain that αβ ¼ lim xy ¼ sin ðαþφÞ sin β.
x!0
Thus α sinsinðαþφ
α
Þ
¼ β sinsinðβþφ
β .
Þ

Let α > β, then since α þ φ þ β þ φ < π, we obtain

sin ðβ þ φÞ < sin ðα þ φÞ ð7:32Þ


 
We have that sin(α  β) < α  β and β < tgβ β < π2 ; consequently

sin α sin ðα  βÞ αβ α


¼ þ cos ðα  βÞ < þ1¼ ð7:33Þ
sin β tgβ β β

According to (7.32) and (7.33), we deduce that α sinsinðαþφ


α
Þ
> β sinsinðβþφ
β .
Þ

This leads to a contradiction and ends the proof of the lemma.


Since α0 ¼ β0 ¼ γ 0 ¼ φ, α ¼ β ¼ γ, then according to (7.27) sin3α ¼ sin3φ, conse-
 3
quently, α ¼ β ¼ γ ¼ φ ¼ π6 and a ¼ αβγ ¼ π6 .
 4
Remark If given a point M is inside of triangle ABC, then αβγ ðα þ β þ γ Þ  3 π6 ,
where α ¼ ∠MAB, β ¼ ∠MBC, γ ¼ ∠MCA.
366 7 Miscellaneous Inequalities

π 4
Hint Similar to the solution of problem 7.1.71a, one can prove that α2 βγ  6 .
(b) For any n  4 we need to give an example of an n- gon and a point M inside of it,
 n
such that α1 α2  :::  αn > πðn2
2n
Þ
.

Consider triangle ABC and point pffiffi


M inside of it such p that
ffiffi
∠MCA ¼ ∠MCB ¼ ∠MBC ¼ ∠MBA ¼ 82π , then ∠MAC ¼ ∠MAB ¼ π2  42π .
Since ∠MAB 6¼ ∠MBC, ∠MAB þ ∠MBC þ 2 ∠MBA < π, then from the lemma
 pffiffiD on line BC such that point M is inside of
above it follows that thereexistspaffiffi point
triangle ADC and γ 1 β1 > 2  4  82π , where γ 1 ¼ ∠MAD, β1 ¼ ∠MDC.
π 2π

Let point D1 be on side AC and close to point A and point A2, located on the half-
line MD1, close to D1, so that the quadrilateral A1A2A3A4 is convex. Here A1  C,
A3  A, A4  D  andpffiffi apart  from that
pffiffi  pffiffi pffiffi 3
the inequality αp2ffiffiγ 1 β1 ¼
α2 α3 α4 > π  π2  42π π
2  4
2π 2π
8 ¼ 2π
64 holds true. Since α 1 > 2π
8 , then
pffiffi 3 pffiffi  4

α1 α2 α3 α4 > 64  82π ¼ π4 .
Let n  5 and A1A2 . . . An  1 be a regular (n  1)-gon with the center M. Take
point A0n on side A1An  1 and point An on the half-line MA0 n outside of the polygon
A1A2 . . . An  1, so that the polygon A1A2 . . . An  1An is convex. Then,
 n1
α1 α2  :::  αn1 > π2ððn1
n3Þ
Þ . Evidently, for any given angle α < π  π2ððn1
n3Þ
Þ , one
can take point A0n close to point A1 and point An close to point A0n , such that αn > α.
 n1
Thus, point An can be chosen, such that π2ððn1
nþ1Þ π ðn3Þ
Þ 2ðn1Þ < α1 α2  :::  αn . We
need to prove that for n  5 the following inequality holds true
 n1  n
π ðnþ1Þ π ðn3Þ π ðn2Þ
2ðn1Þ 2ðn1Þ > 2n , or
 n
n2  3n n3
> ð7:34Þ
n2  3n þ 2 nþ1
 n  n
n2 3n
According to Bernoulli’s inequality, we have that n2 3nþ2 ¼ 1  n2 3nþ2
2

 1  n2 3nþ2
2n
> 1  nþ1
4
at n  7,
Hence for n  7, (7.34) holds true.
Let us check that (7.34) holds true for n ¼ 5 and n ¼ 6.
 5
For n ¼ 5 , we have to check that 56 > 13, or 3125 > 2592.
 9 6 3
For n ¼ 6, we have to check that 10 > 7, or 3720087 > 3000000.
(c) At first, let us prove the following lemma.
Lemma If given α1, . . . , αn, β1, . . . , βn > 0, such that α1 þ . . . þ αn þ
β1 þ . . . þ βn ¼ π(n  2) and αi þ βi < π αi þ βi þ 1 < π, i ¼ 1, 2, . . . , n, βn þ 1 ¼ β1.
Moreover, if

sin α1  . . .  sin αn ¼ sin β1  . . .  sin βn , ð7:35Þ


7.2 Solutions 367

then there exists a convex polygon A1 . . . Anand point M inside of it, so that
∠MAiAi þ 1 ¼ αi, ∠MAiAi  1 ¼ βi, i ¼ 1, 2, . . . , n, where An þ 1  A1, A0  An.
Indeed, consider triangles ΔMAiAi þ 1i ¼ 1, 2, . . . , n  1, such that ∠MAiAi þ 1 ¼
αi, i ¼ 1, . . . , n  1, ∠MAiAi  1 ¼ βii ¼ 2, 3, . . . , n, with points Ai  1 and Ai þ 1 being
on different sides of the straight line MAi, i ¼ 2, . . . , n  1.
Note that ∠A1MAn ¼ 2π  ((n  1)π  (α1 þ β2)  ...  (αn  1 þ βn)) ¼ π 
(αn þ β1) and ∠MA1An þ ∠MAnA1 ¼ αn þ β1. Let ∠MA1An ¼ β , ∠MAnA1 ¼ α.
According to the law of sines, it follows that 1 ¼ MAMA2  MA3  :::
1 MA2
sin β2 sin β3 sin βn sin β
MA1 ¼ sin α1  sin α2  :::  sin αn1  sin α. From the last relation and (7.35) we obtain
MAn

sin β sin β1
that sin α ¼ sin αn and as αn þ β1 ¼ α þ β, then sin(α þ β)ctgα  cos (α þ β) ¼ sin
(α þ β)ctgαn  cos (α þ β).
Therefore, α ¼ αn and β ¼ β1.
Hence, the polygon A1A2 . . . An is convex and point M is inside of the polygon
A1A2 . . . An, since there exist close enough points to Ai on the AiM (i ¼ 1, 2, ..., n),
such that these points are inside of the polygon. This ends the proof of the lemma.
Let minðα1 ; :::; αn Þ > π ðn2 Þ
2n , then there exists a number ε > 0, such that

π ðn  2Þ
αi  þ ε, i ¼ 1, :::, n: ð7:36Þ
2n

Consider the set X of all possible products α1  . . .  αn, such that the condition
(7.36) holds true. Let sup X ¼ a.
In the case a 2 X, let α1  . . .  αn ¼ a. Then as it was shown in (a),
β1 ¼ . . . ¼ βn ¼ φ. We have that αi  π ðn2 Þ
2n þ ε, i ¼ 1, :::, n and

sin n φ ¼ sin α1 ::: sin αn ¼


pffiffiffiffiffiffiffiffiffiffiffiffiffiffiffiffiffiffiffiffiffiffiffiffiffiffiffiffiffiffiffiffiffiffiffiffiffiffiffi n
¼ sin α1 ::: sin αn sin n φ  sin α1 þ:::þ sin α2n n þ sin φþ:::þ sin φ

α 1 þ ::: þ α n þ nφ π ð n  2 Þ
 sin n ¼ sin n ,
2n 2n

and since φ þ αi < π, we have φ  πðn2 Þ


2n . If max(MA1, ..., MAn) ¼ MAi, then from
triangle MAiAi þ 1, we obtain that φ  αi, consequently, π ðn2 Þ
2n  αi . This leads to a
contradiction.
If a2 X, then we can deduce that there exist (see the proof of problem 7.1.71a)
numbers α1, . . . , αn > 0, β1, . . . , βn  0 such that α1 þ . . . þ αn þ β1 þ . . . þ
βn ¼ π(n  2), α1  . . .  αn ¼ a, and αi þ βi þ 1  π, i ¼ 1, . . . , n (βn þ 1 ¼ β1),
αi þ βi  π, i ¼ 1, . . . , n. Apart from that, we also have that sin α1  . . . 
sin αn ¼ sin β1  . . .  sin βn, αi  πðn2Þ
2n þ ε, i ¼ 1, . . . , n .
Let αi 6¼ π, i ¼ 1, . . . , n, then from the condition sin α1  . . .  sin αn ¼
sin β1  . . .  sin βn, we deduce that β1, . . . , βn > 0.
It is clear that there exists a number i, such that αi þ βi ¼ π or αi þ βi þ 1 ¼ π.
Otherwise, according to the lemma, we deduce that a 2 X.
368 7 Miscellaneous Inequalities

If αi þ βi ¼ π for some i, then consider the set α1, β1, α2, β2, . . . , αi  1,
βi  1, αi þ 1, βi þ 1, , . . . , αn, βn. If αi þ βi þ 1 ¼ π, then consider the set of angles
α1, β1, . . . , αi  1, βi  1, αi þ 1, βi, αi þ 2, βi þ 2, . . . , αn, βn. Excluding all such pairs,
we obtain a set of angles α1 , β1 , :::, αk , βk satisfying the conditions of the lemma with
n > k  3, because the number of those pairs is not greater than n  3. By mathe-
matical induction, we obtain that (problem 2.4.10c is the case of n ¼ 3 ), if k < n,
then minðα1 ; :::; αk Þ  π ðk2 Þ
2k < 2n ,
π ðn2Þ
as α1 , :::, αk 2 fα1 ; :::; αn g. Then,
minðα1 ; :::; αn Þ  minðα1 ; :::; αk Þ < π ðn2 2n
Þ
. This leads to a contradiction.
Let α1 ¼ π, then β1 ¼ 0 and α2 þ . . . þ αn  1 þ αn þ β2 þ . . . þ
βn  1 þ βn ¼ π(n  3). Therefore, it follows that n > 3.
Hence there exists a convex polygon A01 :::A0n and a point M inside of it, such that
α1  π  ε, α02 , :::, α0n  π ðn2
0 Þ
2n þ ε (ε can be chosen initially, such that π  ε >
π ðn2Þ 0 0 0 0 0
2n þ ε), where αi ¼ ∠MA i Aiþ1 , i ¼ 1, . . . , n, Anþ1  A1 .
If point M is inside of the polygon A02 :::A0n , then either
 0  π ðn3Þ πðn2Þ
min α2 ; :::; αn1  2ðn1Þ < 2n . This leads to a contradiction, or ∠MA0 n A02 
0

π ðn3Þ
2ðn1Þ < π ðn2 Þ 0 0 π ðn2Þ 0 π ðn2Þ
2n . Then, ∠MA n A1 < 2n þ ε which means that αn < 2n þ ε. This
leads also to a contradiction.
0
If point M is in triangle A0n A01 A02 , then αn < ε.
This leads to a contradiction, as α0n  πðn2 Þ
2n þ ε.
This ends the proof.
  


!  

!
7.1.72. We have that, A1 A2 þ A3 A4 þ ::: þ A2n1 A2n  ¼ A2 A3  A4 A5  ::: 
 

! 

! 
A2n A1 j ¼ A2 A3 þ A4 A5 þ ::: þ A2n A1  and
 
1 1
sin ð∠A1 OA2 þ ::: þ ∠A2n1 OA2n Þ ¼ sin 180  ð∠A2 OA3 þ ::: þ ∠A2n OA1 Þ ¼
2  2
1
¼ sin ð∠A2 OA3 þ ::: þ ∠A2n OA1 Þ :
2

Then, one can assume that ∠A1OA2 þ . . . þ ∠A2n  1OA2n  π.


We shall prove the inequality by induction. At n ¼ 1 , we get an evident equality.
Note that on the arc A2n3 A2n2 A2n exist such points A02n3 and A02n2 , that

!
A02n3 A0 2n2 ¼ A2n3 A2n2 þ A2n1 A2n (see Figure 7.50).

Figure 7.50 A2n-2


A2n-1

a  
  b b
a +b
B
A2n-3 A2n
7.2 Solutions 369

Let A be a point on the arc A2n  2A2n  1A2n such that A2n  2A ¼ A2n  1A2n. Then,
∠A2n  3A2n  2B < ∠A2n  3A2n  2A. Therefore, A02n3 A02n2 ¼ A2n3 B < A2n3 A ;
thus ∠A02n3 OA0 2n2 < ∠A2n3 OA2n2 þ ∠A2n1 OA2n . For n > 1 , we deduce that
  

! 


!  

! 
A1 A2 þ ::: þ A2n3 A2n2 þ A2n1 A2n  ¼ A1 A2 þ ::: þ A02n3 A0 2n2  

1 
 sin ∠A1 OA2 þ ::: þ ∠A02n3 OA0 2n2 <
2
1
< sin ð∠A1 OA2 þ ::: þ ∠A2n3 OA2n2 þ ∠A2n1 OA2n Þ :
2

7.1.73. Note that ΔAOB ΔCOD and ΔAOD ΔBOC; Thus CD AB


¼ OD
AO BC
and AD ¼ OD
CO
,
BO  OD ¼ OC  OA.
OD2
AO
We have to prove that OD þ OD
AO þ OD þ OC  OC þ OA þ OD2 þ AOOC or OD 
CO OD OA OC OCOA

(OA þ OC)(OD2 þ AO  OC)  OA2  OD2 þ OD2  OC2 þ OA2  OC2 þ OD4.
Indeed, we have that

   2
þOAOC
2
ðOD  OA þ OD  OCÞ OD2 þ OA  OC  ODOAþODOCþOD
2 ¼
  2
2
þOAOC
¼ 4  ODOAþODOCþOD
4 

OD2  OA2 þ OD2  OC2 þ OA2  OC2 þ OD4


4 ¼
4
¼ OD2  OA2 þ OD2  OC2 þ OA2  OC2 þ OD4 :

7.1.74. Let O be the incenter of triangle ABC and lines AO,BO ∠Aintersect lineA0 B0 at
00 00 00 
0 
points A and B respectively. Note that ∠AA B ¼ 180  2 þ 90 þ ∠C 2 ¼ ∠B2 ;
0 00 00
consequently the quadrilateral OA A B is inscribed, thus ∠AA B ¼ ∠OA B ¼ 90 .0
00
Similarly, one can prove that ∠AB B ¼ 90 .
00 00 00 00 00 00

Since ∠A OB ¼ ∠AOB ¼ 90 þ ∠C 2 and OL ⊥ A B , then L is on segment A B .
Thus, L is inside of the circle with a diameter AB, hence ∠ALB > 90 .
7.1.75. Consider triangle ABC with the lengths of sides AB ¼ c, AC ¼ b, BC ¼ a,
with a  b  c. Denote by α, β and γ the angles at vertices A, B and C, respectively.
Let B1 be a point on side AC, such that ∠B1BC ¼ γ.
From the assumptions of the problem, it follows that some two of points B, B1, C
have the same colour.
If B and C have the same colour, then a  c. Consequently, a ¼ b ¼ c and
γ ¼ 60 > 36 .
If B and B1 or C and B1 are of the same colour, then BB1 ¼ B1C  c. If B1
coincides with point A, then β ¼ γ and 2γ > 90 , γ > 45 .
If points B1 and A are different, then in triangle BAB1, c ¼ AB  BB1 holds true.
Consequently α ¼ ∠BAB1  ∠BB1A ¼ 2γ. Then, it follows that 180 ¼ α þ β þ
γ  2α þ γ  5γ. Thus, we obtain that γ  36 .
370 7 Miscellaneous Inequalities

Figure 7.51 B
А0

C0

A B0 C

7.1.76. Let us introduce the following definition.


A set of the positive numbers (a, b, c,Ra, Rb, Rc,da, db, dc) is called a describer, if
there exists a triangle ABC and a point M inside of it, such that BC ¼ a, AC ¼ b,
AB ¼ c, MA ¼ Ra, MB ¼ Rb, MC ¼ Rc,MA1 ¼ da, MA1 ¼ da, MB1 ¼ db, MC1 ¼ dc,
where MA1 ⊥ BC, MB1 ⊥ AC, MC1 ⊥ AB (Figure 7.51).
At first, let us prove the following lemmas.
Lemma 1 If (a, b, c,Ra, Rb, Rc,da, db, dc) is a describer, then prove that
a bRb cRc db dc da dc da db
( aR
2R , 2R , 2R , d a , d b , d c , Ra , Rb , Rc is also a describer , where R is the
circumradius of the triangle with sides a, b, c.
Proof of Lemma 1 Note that point M is inside of triangle A1B1C1 (Figure 7.51)
and that one can describe a circle around the quadrilateral AC1MB1 . According to
the law of sines, we have that B1 C1 ¼ Ra sin ∠A ¼ aR 2R . We have that
a

∠MC1A2 ¼ ∠MAB1.
Hence, ΔMC1A2 ΔMAB1, we deduce that MA dc ¼ Ra (MA2 ⊥ B1C1). Hence,
2 db

MA2 ¼ dRb da c . This ends the proof of the lemma 1.


Lemma 2 If (a, b, c,Ra, Rb, Rc,da, db, dc) is a describer, then prove that
(aRa, bRb, cRc, RbRc, RaRc, RaRb, daRa, dbRb, dcRc) is also a describer.
Proof of Lemma 2 Multiply the sides of triangles MAB, MBC and MAC by Rc, Ra
and Rb, respectively and combine these three triangles into one (Figure 7.52). This
ends the proof of lemma 2.
(a) Let ha, hb, hc be the altitudes drawn from vertices A, B, C of triangle ABC,
respectively. We have that dhaa þ dhbb þ dhcc ¼ SSMBC
ABC
þ SSMAC
ABC
þ SSMAB
ABC
¼ 1. On the other
hand, ha  Ra þ da, hb  Rb þ db, hc  Rc þ dc. Hence, Radþd
a
a
þ Rbdþd
b
b
þ Rcdþd
c
c
 1.
2
Using the inequality b11 þ b22 þ b33  ðab11þa2 þa3 Þ
a2 a2 a2
þb2 þb3 , where b1, b2, b3 > 0, we obtain

d2a d2b d2c ðd a þ d b þ d c Þ2


1 þ þ  :
Ra da þ d 2a Rb d b þ d2b Rc dc þ d2c Ra d a þ d2a þ Rb d b þ d2b þ Rc dc þ d2c
7.2 Solutions 371

Figure 7.52
b1

bк b2

b3

Hence, Rada þ Rbdb þ Rcdc  2(dadb þ dbdc þ dcda). Now, if we write the
a bRb cRc db dc da dc da db
inequality (a) for the describer ( aR
2R , 2R , 2R , da, db, dc, Ra , Rb , Rc ), we obtain
the inequality (b). So, the inequality (b) follows from (a) and lemma 1.
Inequality (c) follows from (b) and lemma 2, inequality (d) follows from (c) and
lemma 1, inequality (e) follows from (d) and lemma 1, inequality (f) follows from
(e) and lemma .
Thus, we have proven that a ) b ) c ) d ) e ) f. Note that, as (a) follows from
(f) and lemma 1, we obtain that a , b , c , d , e , f.
7.1.77. Let PQRS be the section of the tetrahedron ABCD passing through the center
O of the sphere with the radius r and parallel to lines AB and CD. Since the circle
with the radius r is in the parallelogram PQRS, then if PQ ¼ x  CD (PQ k CD,
QR k AB, 0 < x < 1), we obtain that QR ¼ (1  x)AB and PQ > 2r, RQ > 2r.
Therefore, x > CD 2r
, 1  x > AB 2r
. Thus, 1 > CD 2r
þ AB
2r
. Hence, r < 2ðABCD
ABþCDÞ.
a b c 
7.1.78. (a) Let DA ¼ a, DB ¼ b, DC ¼ c. Note that point O1 2; 2; 2 is equidistant
from
q points D(0;ffi 0; 0), A(a; 0; 0), B(0; b; 0), C(0; 0; c), consequently R¼
ffiffiffiffiffiffiffiffiffiffiffiffiffiffiffiffiffiffiffiffiffiffiffiffiffiffiffiffiffiffiffiffiffiffiffi

a 2
b2 c2
þ 2 þ 2 . We have that V ABCD ¼ 6 and V ABCD ¼ 3 Sn  r ¼ abc 1
2 pffiffiffiffiffiffiffiffiffiffiffiffiffiffiffiffiffiffiffiffiffiffiffiffiffiffiffiffiffiffiffiffiffiffiffiffiffiffi
1
3 r ab
2 þ bc
2 þ ac
2 þ 2 a b þb c þa c
1 2 2 2 2 2 2

Thus r ¼ pabc
ffiffiffiffiffiffiffiffiffiffiffiffiffiffiffiffiffiffiffiffiffiffiffiffi
abþbcþacþ a2 b2 þb2 c2 þa2 c2
Let a ¼ 1x , b ¼ 1y , c ¼ 1z , then we have to prove that
sffiffiffiffiffiffiffiffiffiffiffiffiffiffiffiffiffiffiffiffiffiffiffiffi
1 1 1 1 1 1 6  1x  1y  1z
þ þ  þ þ þ qffiffiffiffiffiffiffiffiffiffiffiffiffiffiffiffiffiffiffiffiffiffiffiffiffiffiffiffiffiffiffiffiffiffiffiffiffiffiffiffiffiffi
x y z x2 y2 z2 1  1 þ 1  1 þ 1  1 þ 1  1 þ 1  1 þ 1  1 ,
x y y z x z x2 y2 y2 z2 x2 z2
372 7 Miscellaneous Inequalities

or
 
2 1y þ 1y  1z þ 1x  1z
1
x 6
qffiffiffiffiffiffiffiffiffiffiffiffiffiffiffiffiffiffiffiffiffi  pffiffiffiffiffiffiffiffiffiffiffiffiffiffiffiffiffiffiffiffiffiffiffiffi ,
x þ y þ z þ x2 þ y 2 þ z2
xþyþzþ x 2 þ y 2 þ z2
1 1 1 1 1 1

or
pffiffiffiffiffiffiffiffiffiffiffiffiffiffiffiffiffiffiffiffiffiffiffiffiffiffiffiffiffiffiffiffiffiffiffiffi
x þ y þ z xy þ yz þ xz þ x2 y2 þ y2 z2 þ z2 x2
 pffiffiffiffiffiffiffiffiffiffiffiffiffiffiffiffiffiffiffiffiffiffiffiffi :
3 x þ y þ z þ x2 þ y 2 þ z2

Without loss of generality, one can assume that x þ y þ z ¼ 1 and min(x, y, z) ¼ z,


then z  13. Let xy þ yz þ xz ¼ p, xyz ¼ q.
pffiffiffiffiffiffiffiffiffiffiffiffiffiffi pffiffiffiffiffiffiffiffiffiffiffiffiffiffiffi
It remains to prove that 13 þ 13 1  2p  p þ p2  2q.
We have that (x þ y þ z)2  3(xy þ yz þ xz). Therefore, p  13. Thus we have to
 pffiffiffiffiffiffiffiffiffiffiffiffiffiffi 2 pffiffiffiffiffiffiffiffiffiffiffiffiffiffi1 
prove that 13 þ 13 1  2p  p  p2  2q, or 28p 9 þ 2q þ 3 1  2p 3  p  0.
2

It is sufficient to prove that 19 þ q  49 p  0.


Indeed, we have that

1 4 1 4 1 4 4
þ q  p ¼ þ xyz  ðzðx þ yÞ þ xyÞ ¼ þ xy z   zð1  zÞ 
9 9 9 9 9 9 9
    2
1 xþy 2 4 4 1 1z 9z  4 4
 þ z  zð1  zÞ ¼ þ   zð1  zÞ ¼
9 2 9 9 9 2 9 9
1 3  zð3z  1Þ2
¼ 9z  6z2 þ z ¼  0,
36 36
1 4
hence þ q  p  0.
9 9

a b c
(b) Note that O(r, r, r) and O1 ; ; ; , consequently
2 2 2

 2  2  2
ðR þ r ÞðR  3r Þ  OO21 ¼ R2  2Rr  3r 2  r  a2  r  b2  r  2c ¼
¼ r ða þ b þ cÞ  2Rr  6r 2 ¼ r ða þ b þ c  2R  6r Þ  0

(see problem 7.1.78а).


7.1.79. Let A1 and A2 be the intersection points, respectively, of the straight lines
AM and AО with circumcircle of triangle ΔABC. Here О is the center of the
circumcircle of triangle ABC. Let us choose on the half-lines AB, AA1, AA2, AC
and AM, respectively, points B0 , A01 , A02 , C0 and M0 , such that AB0 ¼ AB
1
, AA1 0 ¼ AA1 1 ,
AA2 0 ¼ AA1 2 , AC0 ¼ AC
1
, and AM0 ¼ AM 1
.
7.2 Solutions 373

Note that points A01 , A02 are on the straight line B0 C0 and B0 C0 ¼ ABACMA MABC
,
0
0 0 0 0 0
M C ¼ AMACAB, B M ¼ ABAMAC , and A1 M ¼ AMAA1 , ∠AA2 A1 ¼ ∠AA1A2 ¼ 90
MCAB BMAC A1 M 0 0

(see the proof of problem 1.2.9а). Thus, it follows that SM ¼ AM2 


AC2  AB2  SB0 M0 C0 , where SM is the area of the triangle with sides MA  BC,
MB  AC, MC  AB.
Let H 2 B0 C0 and M0 H ⊥ B0 C0 , then M0 H||AA2. Thus, ΔA01 M0 H ΔAA1 A2 .
0 A1 0 M 0 0
AA1 ¼ AA2 . Hence, M H ¼ 2AMR. Then, we deduce that SB M C ¼
A1 M
Therefore, M H 0 0 0

1 0 0 0
2 B C  M H ¼ 2ABAC  2AMR ¼ 2 and SM ¼ AM  AC  AB  SB0 M0 C0 ¼
BC A1 M BCA1 MMA 2 2 2
 2 4RABACAM

SABC  A1 M  MA ¼ SABC  R  d .  2

Let points A0, B0, C0 be the feet of the perpendiculars drawn from point  M to
lines BC, CA, AB, respectively. Then, SA0 B0 C0 ¼ 4R1 2  SM ¼ S4R  2
2  R  d
ABC 2
(see the
proof of lemma 1 of problem 7.1.76).
If points A0, B0, C0 are on the same straight line, then one can prove that d ¼ R.
Hence, it follows that MA  MB  MC  0 ¼ 2r|R2  d2|.
Let R0 be the circumradius of triangle A0B0C0, then  R0  r2 (see Figure 7.53).
Thus, we have that A0 B0 B4R 0 C0 A0 C0
0
¼ S A 0 B 0 C0 ¼ SABC  2
4R2
 R  d . Hence,

MA  BC MB  AC MC  AB SABC  2 
  ¼ 2
R  d2   4R0 :
2R 2R 2R 4R

Therefore, it follows that MA  MB  MC ¼ 2R0|R2  d2|  2r|R2  d2|.


7.1.80. (a) According to problem 1.1.14а, we obtain for points O, A2, B1, B2, that
r 1  B1A2 þ r  B1B2  r 1  A2B2. Similarly, r1  B2A3 þ r  B2B3  r1  A3B3, . . .,
r 1  BnA1 þ r  BnB1  r 1  A 1B1.
Summing up these inequalities we obtain that

r ðB1 B2 þ B2 B3 þ ::: þ Bn1 Bn þ Bn B1 Þ 


 r 1 ðA1 B1  B1 A2 Þ þ r 1 ðA2 B2  B2 A3 Þ þ ::: þ r 1 ðAn Bn  A1 Bn Þ:

aij

Oi O Oj

Ci aji Cj

Figure 7.53
374 7 Miscellaneous Inequalities

Therefore, B1 B2 þ B2 B3 þ ::: þ Bn1 Bn þ Bn B1  rr1 ðA1 A2 þ A2 A3 þ :::þ


An1 An þ An A1 Þ.
(b) Note that A1 B1  B1 A2 ¼ A2 B2  B2 A3 ¼ ::: ¼ An Bn  Bn A1 ¼ r 21  r 2 and S1 ¼
SA1 B1 Bn ¼ 12 A1 Bn  A1 B1 sin ∠A1 , S2 ¼ 12 A2 B1  A2 B2 sin ∠A2 ¼ SA2 B1 B2 , ::: ,
Sn ¼ SAn Bn1 Bn ¼ 12 An Bn1  An Bn sin ∠An .
Multiplying these inequalities, we deduce that

1 2 
2 n
S1 S2  :::  Sn ¼ n r1  r sin ∠A1  sin ∠A2  :::  sin ∠An 
2

For n ¼ 3 or n ¼ 4, we need to prove that

nr 2 p ffiffiffiffiffiffiffiffiffiffiffiffiffiffiffiffiffiffiffiffiffiffiffiffiffiffiffiffiffiffiffiffiffiffiffiffiffiffiffiffiffiffiffi
SA1 A2 :::An   n sin ∠A1  :::  sin ∠An : ð7:37Þ
2

For n ¼ 3, we have that

A1 A2  A2 A3  A3 A1
SA1 A2 A3 ¼ ¼ 2r 2 sin ∠A1  sin ∠A2  ∠: sin A3 
4r
3r 2 p ffiffiffiffiffiffiffiffiffiffiffiffiffiffiffiffiffiffiffiffiffiffiffiffiffiffiffiffiffiffiffiffiffiffiffiffiffiffiffiffiffiffiffiffiffiffiffiffi
 3
sin ∠A1 sin ∠A2 sin ∠A3
2

(see problem 5.1.12).


For n ¼ 4, we have that

1 pffiffiffiffiffiffiffiffiffiffiffiffiffiffiffiffiffiffiffiffiffiffiffiffiffiffiffiffiffiffiffiffi
SA1 A2 A3 A4  A1 A3  A2 A4 ¼ 2r 2 sin ∠A2 sin ∠A1  2r 2 sin ∠A2 sin ∠A1 ¼
2
pffiffiffiffiffiffiffiffiffiffiffiffiffiffiffiffiffiffiffiffiffiffiffiffiffiffiffiffiffiffiffiffiffiffiffiffiffiffiffiffiffiffiffiffiffiffiffiffiffiffiffiffiffiffiffiffiffiffiffiffiffiffiffiffiffi
¼ 2r 2 4 sin ∠A1 sin ∠A2 sin ∠A3 ∠: sin A4 :

For n  5 the inequality (7.37) may be wrong.


We have that SB1 B2 :::Bn ¼ SA1 A2 :::An þ S1 þ S2 þ ::: þ Sn . According to the arith-
metic mean-geometric mean of the inequality, we deduce that
 
p ffiffiffiffiffiffiffiffiffiffiffiffiffiffiffiffiffiffiffiffiffiffiffiffiffi n r 21  r 2 p ffiffiffiffiffiffiffiffiffiffiffiffiffiffiffiffiffiffiffiffiffiffiffiffiffiffiffiffiffiffiffiffiffiffiffiffiffiffiffiffi
SB1 B2 :::Bn  SA1 A2 :::An þ n S1  S2  :::  Sn ¼ SA1 A2 :::An þ
n
 n sin ∠A1  :::  sin ∠An :
2

According to (7.37), we have that (for n ¼ 3 or n ¼ 4) SB1 B2 :::Bn 


r 2 r 2 r2
SA1 A2 :::An þ 1r2 SA1 A2 :::An ¼ r12  SA1 A2 :::An .
r2
Remark For n  5 we do not know, whether the inequality SB1 B2 :::Bn  r12 SA1 A2 :::An
holds true or not.
7.1.81. Note that at any i 6¼ j the circles Ci and Cj do not have common points.
Indeed, if the circles Ci and Cj have a common point A, then let us draw through
point A a line, intersecting the circle Ck, where k 6¼ i, k 6¼ j, but this is in a
contradiction with the assumptions of the problem.
7.2 Solutions 375

Figure 7.54
Bm Ak
Bk
Om Cr Ok

tm Am

tk

Figure 7.55
Ai+1

Ai
w

li

Pi

Consider the smallest convex polygon, containing the centers of the circles Ci
(i ¼ 1, 2, ..., n). Denote by β1, β2, . . ., βk the external angles (Figure 7.54) of the
polygon, and by Cij the union of two arcs of the circles Ci such that if point M 2 Cij,
then the tangent to the circle Ci drawn from point M intersects (has a common point
with) the circle Cj.
Note that from the assumptions of the problem follows that Cij \ Cil ¼ ∅, if j 6¼ l.
PDenote by  2αij the radian measure of the arcs Cij, then
2 αij þ αji  2πn  ðβ1 þ β2 þ ::: þ βk Þ ¼ 2π ðn  1Þ. Let OiOj ¼ dij.
1i<jn
Note that ∠OiAOj ¼ ∠OiBOj ¼ 90 (see Figure 7.55).
Therefore, points Oi, A, Oj, B are on the same circle, thus
α α þα
∠Oi Oj A ¼ ∠Oi BA ¼ 2ij , ∠Oi OA ¼ ij 2 ji . It is not difficult to note that
r i þrj
sin ∠Oi OA ¼ dij , and according to Jensen’s inequality, it follows that
P αij þαji P αij þαji
P ri þrj P αij þαji nðn1Þ
2
nðn1Þ π
2

dij ¼ sin 2  2 sin nðn1Þ  2 sin n, since nðn1Þ


1i<jn 1i<jn

1i<jn 1i<jn 2 2

 πn (n  3). Thus, we have obtained that

X r i þ r j nð n  1Þ π
 sin :
1i<jn
dij 2 n
376 7 Miscellaneous Inequalities

P
Remark For r1 ¼ r2 ¼ . . . ¼ rn ¼ 1the inequality 1
dij < πðn1
4
Þ
holds true, as
1i<jn
sin πn < πn.
7.1.82. Let ∠CAK ¼ α, ∠KAL ¼ β and CK ¼ KL ¼ LM ¼ MB ¼ a. Note that
ΔMAB ΔACB. Consequently AB 4a ¼ AB ¼ AC . Hence, we deduce that AB ¼ 2a ¼ LB
a AM

and AC ¼ 2  AM. Note that if ∠LAB ¼ φ, then ∠ALB ¼ φ, ∠AKB ¼


φ  β, ∠ACB ¼ φ  β  α , and ∠LAM ¼ φ  (φ  β  α) ¼ α þ β.
sin α
For triangle AMC, we have that 12 ¼ KM CK
¼ AMAC sin ðαþ2βÞ.
Hence, it follows that sin(α þ 2β) ¼ 4 sin α.
Note that ∠CAM ¼ 2α þ 2β < π and sin α  14.
Therefore, sin α  12 sin π6 < sin 12
π
. Thus, α < 12 π
.
π
If β  1, 5α, then α þ 2β  4α < 3. Hence, it follows that 4 sin α ¼ sin
(α þ 2β)  sin 4α ¼ 4 sin α cos α cos 2α < 4 sin α.
This leads to a contradiction. Thus, we deduce that β > 1, 5α.
Let α ! 0, α þ 2β < π2 and sin(α þ 2β) ¼ 4 sin α(α, β > 0).
Then, α þ 2β ! 0 and αþ2β αþ2β 4 sin α
α ¼ sin ðαþ2βÞ  α ! 4, α ! 1, 5.
β

(To construct ΔABC, first construct ΔCAL).


7.1.83. Note that ΔOmAmCr ¼ ΔOkAkCr (see Figure 7.56).
Hence, AmCr ¼ CrAk ¼ xr. We have that t2m ¼ PCr  ðPCr  xr Þ and
tk ¼ PCr  ðPCr þ xr Þ. Therefore,
2
tm tk ¼ PCr  PC2r  x2r  PC4r . Thus,
2 2 2

PC2r  tm tk . Multiplying all these inequalities, we obtain that PC1  PC2  . . . 


PCn  t1  t2  . . .  tn.
7.1.84. Let the convex polygon A1A2 . . . An be inside of the circle ω. Take any point
M inside of the polygon A1A2 . . . An. Let ray MAi intersects the circle ω at point Bi
(i ¼ 1, 2, ..., n). It is clear that the polygon A1A2 . . . An is inside of the polygon
B1B2 . . . Bn, thus SA1 A2 :::An  SB1 B2 :::Bn and PA1 A2 :::An  PB1 B2 :::Bn (see problem 2.1.1).
Let point O be the center of the circle ω and ∠B2OB1 ¼ α1, ∠B3OB2 ¼ α2, . . .,
∠BnOBn  1 ¼ αn  1, ∠B1OBn ¼ αn. If point O is inside or is on one of the sides of
the polygon B1B2 . . . Bn, then we  obtain that SB1 B2 :::Bn ¼ 12 R2 ð sin α1 þ
sin α2 þ ::: þ sin αn Þ, PB1 B2 :::Bn ¼ 2R sin 2 þ ::: þ sin α2n , where R is the radius
α1

Figure 7.56 B1 C1 B2

a2 C2
a1 a3
a4 B3

B4
7.2 Solutions 377

of the circle ω. According to problem 5.4.15, it follows that SB1 B2 :::Bn 


π
2 R n  sin n and PB1 B2 :::Bn  2Rn sin n.
1 2 2π

If point O is outside of the polygon B1B2 . . . Bn and max(α1, ..., αn  1, αn) ¼ αn,
then αn ¼ α1 þ . . . þ αn  1 < π. Thus, we deduce that

1
SB1 B2 :::Bn < R2 ð sin α1 þ sin α2 þ ::: þ sin αn Þ
2
1 α1 þ ::: þ αn 1 2 2π
 R2 n sin  R n sin ,
2 n 2 n

Since α1 þ:::þα
n
n
< 2π π
n  2 (n  4).
For n ¼ 3, we have that SB1 B2 B3 < 12 R2 ð sin α1 þ sin α2 Þ < R2 < 12 R2  3 sin 2π
3.
In the same way, we obtain that
 α1 αn  α1 þ ::: þ αn π
PB1 B2 :::Bn ¼ 2R sin þ ::: þ sin  2Rn sin < 2Rn sin :
2 2 2n n
π
We have obtained that SA1 A2 :::An  12 R2 n  sin 2π
n and PA1 A2 :::An  2Rn sin n. To end
the proof it remains to note that the area and the perimeter of the regular n-gon,
π
inscribed into the circle with the radius R, are equal to 12 R2 n  sin 2π n and 2Rn sin n
respectively.
7.1.85. Let the convex polygon A1A2 . . . An contains the circle ω. Let the half-plane
Πi contains the circle ω, has a boundary li||AiAi þ 1 (An þ 1  A1), i ¼ 1, 2, . . . , n, line
li is tangent to the circle ω, and Πi is in the half-plane with a boundary AiAi þ 1,
containing the polygon (see Figure 7.57).
If Π1 \ Π2 \ . . . \ Πn ¼ B1B2 . . . Bn, then obviously B1B2 . . . Bn is circum-
scribed around the circle ω and B1B2 . . . Bn is in the polygon A1 . . . An. Thus,
SA1 A2 :::An  SB1 B2 :::Bn and PA1 A2 :::An  PB1 B2 :::Bn (see problem 2.1.1). We deduce
 that

(see Figure 7.58) PB1 B2 :::Bn ¼ r ðtg α1 þ tg α2 þ ::: þ tg α2n Þ, where αi 2 0; π2 ,
α1 þ α2 þ . . . þ α2n ¼ 2π and r is the radius of the circle ω. According to problem
5.4.16, it follows that PB1 B2 :::Bn ¼ r ðtg α1 þ tg α2 þ ::: þ tg α2n Þ  2nr  tg 2π 2n ¼
2nr  tg πn. Similarly, we obtain that SB1 B2 :::Bn ¼ 12 PB1 B2 :::Bn  r  nr 2 tg πn.
We have obtained that SA1 A2 :::An  nr 2 tg πn and PA1 A2 :::An  2nrtgπn. To end the
proof it remains to note that the area and the perimeter of the regular n-gon,
circumscribed around the circle with the radius R, are equal to nr 2 tg πn and 2nrtgπn
respectively.

Figure 7.57
378 7 Miscellaneous Inequalities

Figure 7.58 A3
A2
A4

A1

An

Figure 7.59
y

B C

a x
0 A

Figure 7.60
A2
An
A1 D Ak
C0
w C Ak+1

7.1.86. This proof is based on the fact of the existence, among all possible n-gon
with a given perimeter, of a n-gon with a maximal area.
First of all, note that the n-gon with a given perimeter that has a maximal area
must be convex. Otherwise, one can construct a n-gon with the same perimeter, but
with the greater area (see Figure 7.59).
Now, let us do the second remark about the properties of the required n-gon.
If (n  4) any four consecutive vertices lie on one circle, then the n-gon is an
inscribed polygon.
Indeed, it is sufficient to change only vertices A2 and A3 and make use of the fact,
that the articulated quadrilateral A1A2A3A4 (see Figure 7.60) has a maximal area,
when the quadrilateral A1A2A3A4 is inscribed (see problem 5.5.22).
7.2 Solutions 379

We need to prove that all sides of the required n-gon are equal. Otherwise,
without loss of generality, one can assume that A1A2 ¼ a þ x, A2A3 ¼ a  x, x 6¼ 0.
A1A3 ¼ 2b, then ffi by qHeron’s
Then, qifffiffiffiffiffiffiffiffiffiffiffiffiffiffiffiffiffiffiffiffiffiffiffiffiffiffiffiffiffiffiffiffiffiffiffiffiffiffiffiffiffiffiffiffiffiffiffi formula, we have that
ffiffiffiffiffiffiffiffiffiffiffiffiffiffiffiffiffiffiffiffiffiffiffiffiffiffiffi
 2    2
SA1 A2 A3 ¼ ða þ bÞða  bÞ b  x < 2 a  b  b . This leads to a contradic-
2 2

tion, as SA1 A2 :::An < SA1 A02 A3 :::An , where A1 A02 ¼ A02 A3 ¼ a. This ends the proof.
7.1.87. Consider instead of the given n-gon M a centrally symmetric convex 2m-gon
and denote it by M0 , such that the projections of M and M0 on any straight line are
the same. Such 2m-gon can be constructed (see the proof of problem 2.2.1b).
Since H ¼ min l(M ) ¼ min l(M0 ) and D ¼ max l(M ) ¼ l(M0 ), and polygons M,M0
have equal perimeters, then it is sufficient to prove the problem for the polygon M0 .
The polygon M0 contains a circle with the diameter H and it is contained in the
circle with the diameter D. Applying to it the problems 7.1.84b and 7.1.85b, we
deduce that
π π
2mtg  H  P  2m sin  D:
2m 2m

Since 2m  2n, then m  n and to complete the proof, we have to prove the
sin π π
tg π π
following inequalities sin 2mπ  2mπ and tg 2mπ  2mπ .
2n 2n 2n 2n
These inequalities hold true (see the solution of problem 7.1.71a). Moreover, if
0 < β  α < π2, then
ðαβÞ
tgα
tgβ ¼ tg ðα  βÞ cos
cos α sin β þ 1  tg ðα  βÞ 
1
sin β þ 1  αβ α
β þ 1 ¼ β.

Remark In the proof of the problem, we have used the following inequalities. If
0 < α < π2, then sin α < α < tgα. It follows from the inequality SAOB < SsectAOB <
SAOC (see Figure 7.61).
7.1.88. Let us choose in the strip pi a point Mi, equidistant from the boundaries of

!
the strip pi, where i ¼ 1, . . . , n and define bi ¼ ~ V i  OMi .
 2
Consider the numbers a1 ~
V 1 þ a2 ~
V 2 þ ::: þ an ~
V n  2a1 b1  :::  2an bn ,
where ai 2 {0, 1}, i ¼ 1, 2, . . . , n, in total 2n numbers. There are maximal numbers

Figure 7.61 Q D R
C

P E

B
T

S A
380 7 Miscellaneous Inequalities

among these numbers, let these maximums are reached at the values a1, . . . , an,
then
 2
a1 ~
V 1 þ a2 ~
V 2 þ ::: þ an~V n  2a1 b1  :::  2an bn 
 2
 a1 ~ V 1 þ ::: þ ai1 ~ V i þ aiþ1~
V i1 þ ð1  ai Þ~ V iþ1 þ ::: þ an~
Vn 
2a1 b1  :::  2ai1 bi1  2ð1  ai Þbi  2aiþ1 biþ1  :::  2an bn ,

where i ¼ 1, 2, . . . , 2n.
 2  2
Consequently a1 ~ V 1 þ ::: þ an ~V n  a1 ~ V þ ::: þ an~
V 1 þ ::: þ ð1  ai Þ~ Vn
   i
 2bi ð2ai  1Þ. ~ V i ð2ai  1Þ 2 a1 ~ V 1 þ ::: þ an ~ V n  ð2ai  1Þ~V i  2bi ð2ai  1Þ.
  
~ ~ ~
2ð2ai  1Þ a1 V 1 þ ::: þ an V n V i  bi  V i . ~ 2

!
!   
  j~V i jW i 

! 
Hence, OM  ~ V i  OMi  ~ V i   12 ~
V i > 2 , this means that MMi  ~
2
Vi >
j~V i j
2  Wi.
Therefore, point M is not inside of the strip pi, where i ¼ 1, 2, . . . n.
7.1.89. Lemma. Let n  2 and A1, A2, . . . , An are different points on the plane and
ω(C, r) is the circle with the smallest radius containing points A1, . . . , An. Then, on
the circle ω there are either two points Ai and Aj, such that AiAj ¼ 2r, or three points
Ai, Aj, Ak, so that AiAjAk is a non-obtuse triangle.
Indeed, the case CAi < r, i ¼ 1, 2, . . . , n leads to a contradiction since the circle
ω0(C, r0), where r 0 ¼ max CAi , contains all points Ai. However, r0 < r.
1in
The case CA1 ¼ r, CAi < r, i ¼ 2, 3, . . . , n leads also to a contradiction, since the
circle ω0(C0, r0), (where r 0 ¼ rþr
2 , r 1 ¼ max CAi , and C0 is a point of segment CA1,
1
2in
such that C0A1 ¼ r0), contains all points Ai.
As r 0 ¼ rþr 2 < r and C0 Ai  CC0 þ CAi  2 þ r 1 ¼ r 0 , i ¼ 2, . . . n, the last
1 rr1

statement holds true.


The case on Figure 7.62, where CA1 ¼ r, . . . , CAk ¼ r and CAi < r, i ¼ k þ 1, . . .
n, k  2, leads also to a contradiction, since the circle ω0(C0, r0) (where C0 is a point
of segment CD, such that C0 C < rr 2 , where
1
r 1 ¼ max CAi , and
  kþ1in
r 0 ¼ max C0 A1 ; :::; C0 Ak ; 2 ), contains all points Ai and r0 < r.
rþr 1

Indeed, as ∠CDAi  90 , i ¼ 1, . . . , k, then ∠CC0Ai > ∠CDAi  90 ,


i ¼ 1, . . . , k. Hence, C0Ai < r, i ¼ 1, . . . , k and C0 Ai  CC0 þ
CAi < rr 2
1
þ r 1 ¼ rþr1
2 < r, i ¼ k þ 1, . . . n. This ends the proof of the lemma.
Without loss of generality, one can assume that C(0;
qffiffiffiffiffiffiffiffiffiffiffiffiffiffiffiffiffiffiffiffiffiffiffiffiffiffiffiffiffiffiffiffiffiffiffiffiffiffiffiffiffiffiffiffiffiffiffiffiffiffiffiffiffiffiffiffiffiffiffiffiffiffiffiffiffiffiffiffiffiffiffiffiffiffiffi 0) and r ¼ 1. Then, we have
to prove that ðx1 þ x2 þ ::: þ xn Þ2 þ ðy1 þ ::: þ yn Þ2  n  2.
If A1, A2 are such that A1A2 ¼ 2, then x1 þ x2 ¼ y1 þ y2 ¼ 0. Hence,
7.2 Solutions 381

Figure 7.62 N

Q D
R
C
M
P E
B

K S

F A

qffiffiffiffiffiffiffiffiffiffiffiffiffiffiffiffiffiffiffiffiffiffiffiffiffiffiffiffiffiffiffiffiffiffiffiffiffiffiffiffiffiffiffiffiffiffiffiffiffiffiffiffiffiffiffiffiffiffiffiffiffiffiffiffiffi qffiffiffiffiffiffiffiffiffiffiffiffiffiffiffiffiffiffiffiffiffiffiffiffiffiffiffiffiffiffiffiffiffiffiffiffiffiffiffiffiffiffiffiffiffiffiffiffiffiffiffiffiffiffiffiffiffiffiffiffiffiffiffiffiffi
ðx1 þ ::: þ xn Þ2 þ ðy1 þ ::: þ yn Þ2 ¼ ðx3 þ ::: þ xn Þ2 þ ðy3 þ ::: þ yn Þ2 
qffiffiffiffiffiffiffiffiffiffiffiffiffiffiffiffiffiffiffiffiffiffiffiffiffiffiffiffiffiffiffiffiffiffiffiffiffiffiffiffiffiffiffiffiffiffiffiffiffiffiffiffiffiffiffiffiffiffiffiffiffiffi
 ffi pffiffiffiffiffiffiffiffiffiffiffiffiffiffiffiffiffiffiffiffiffiffiffiffiffiffiffiffiffi
 ðn  2Þ x23 þ y23 þ ::: þ x2n þ y2n  ðn  2Þðn  2Þ ¼ n  2:

If points A1, A2, A3 are, such that x21 þ y21 ¼ x22 þ y22 ¼ x23 þ y23 ¼ 1 and ΔA1A2A3
is non-obtuse, then (x1 þ x2 þ x3)2 þ (y1 þ y2 þ y3)2  1, since according to
problem 5.5.4a, we obtain that (x1  x2)2 þ (y1  y2)2 þ . . . þ (y2  y3)2  8 and
(x1 þ x2 þ x3)2 þ (y1 þ y2 þ y3)2 ¼ 9  (x1  x2)2  . . .  (y2  y3)2.
Consequently,
qffiffiffiffiffiffiffiffiffiffiffiffiffiffiffiffiffiffiffiffiffiffiffiffiffiffiffiffiffiffiffiffiffiffiffiffiffiffiffiffiffiffiffiffiffiffiffiffiffiffiffiffiffiffiffiffiffiffiffiffiffi
ðx1 þ ::: þ xn Þ2 þ ðy1 þ ::: þ yn Þ2 
rffiffiffiffiffiffiffiffiffiffiffiffiffiffiffiffiffiffiffiffiffiffiffiffiffiffiffiffiffiffiffiffiffiffiffiffiffiffiffiffiffiffiffiffiffiffiffiffiffiffiffiffiffiffiffiffiffiffiffiffiffiffiffiffiffiffiffiffiffiffiffiffiffiffiffiffiffiffiffiffiffiffiffiffiffiffiffiffiffiffiffiffiffiffiffiffiffiffiffiffiffiffiffiffiffiffiffiffiffiffiffiffiffiffiffiffiffiffiffiffiffiffiffiffiffiffiffiffiffiffiffiffiffiffi
 
 ðn  2Þ ðx1 þ x2 þ x3 Þ2 þ x24 þ ::: þ x2n þ ðy1 þ y2 þ y3 Þ2 þ y24 þ ::: þ y2n  n  2:

7.1.90. (a) We shall prove that for any pentagonal


pffiffi
section of the cube the ratio of
some of its two sides is greater than 1 þ 22. We proceed the proof by contradiction
argument.
Let AB ¼ a, AE ¼ b, BC ¼ b1, CD ¼ c1, PQ ¼ c, DE ¼ a1. We have that aa1  λ,
pffiffi
b1  λ, where λ ¼ 1 þ 2 .
b 2

Note that ΔBPC ΔETA and ΔDRE ΔASB (see Figure 7.63).
Therefore, PCc
¼ bb1  λ¸ DR
c
¼ aa1  λ.
Hence, we deduce that PC cλ, DR cλ.  
Thus, it follows that CQ  c 1  1λ , DQ  c 1  1λ .
From
pffiffiffi triangle
  CQD, according to the Pythagorean theorem, it follows that
c1  2c 1  1λ .
Thus, we obtain that ca1 > cc1  pffiffi2 111 ¼ λ.
ð λÞ
382 7 Miscellaneous Inequalities

Figure 7.63 N

Q D D¢ S1

C E¢
M¢ M
F1 P E
B

K S

F A

Figure 7.64 C

g g
a b
2 2
B
A

Hence, we obtain that ca1 > λ. This leads to a contradiction (see our assumption).
pffiffi

ffiffi problem it remains to note that 1 þ 2 > 1, 7.


2
To end the proof ofpthe
The estimate 1 þ 22 is exact, since one can pffiffi
take a section, with the sides
“slightly” different from the numbers c, c, cλ, cλ, 2cðλλ1Þ ¼ cλ.
(b) We proceed the proof by contradiction argument. Let a  b (see the notations of
the part [a]). Note that ΔBMN ¼ ΔAES (see Figure 7.64).
pffiffiffiffiffiffiffiffiffiffiffiffiffiffiffi
Thus ffi MK þ ES ¼ MK þ MN ¼ KN > c, this means that
pffiffiffiffiffiffiffiffiffiffiffiffiffiffi a2  c 2 þ
b  c > c.
2 2
pffiffiffiffiffiffiffiffiffiffiffiffiffiffiffi
Hence, we obtain that 2 a2  c2 > c, consequently
pffiffiffi
a 5
> ð7:38Þ
c 2
pffiffiffi
Let λ ¼ 1, 83. Since ca1  λ, thena  c1 λ  2cðλ  1Þ(see the proof of the part [a]).
pffiffiffi
Similarly, we deduce that b  2cðλ  1Þ. It is not p difficult
ffiffiffi to note that, if we
take a section ABCDE, such that AB ¼ AE ¼ 2 c ð λ  1Þ, then, CD ¼
pffiffiffi
2c 2  pffiffiffiffiffiffiffiffiffiffiffiffiffiffiffi
1
 c1 . ?From the last inequality and (7.38), it follows that
2λ 4λþ1
2
7.2 Solutions 383

pffiffiffiffiffi
a a 10
 pffiffiffi  >  > λ,
c1
2c 2  pffiffiffiffiffiffiffiffiffiffiffiffiffiffiffi
1
4 2  pffiffiffiffiffiffiffiffiffiffiffiffiffiffiffi
1
2
2λ 4λþ1 2
2λ 4λþ1

as
!  
1 1 1
4λ 2  pffiffiffiffiffiffiffiffiffiffiffiffiffiffiffiffiffiffiffiffiffiffiffiffiffiffi ¼ 7, 32 2  pffiffiffiffiffiffiffiffiffiffiffiffiffiffi < 7, 32 2  ¼
2λ2  4λ þ 1 pffiffiffiffiffi 0, 3778 0, 615
¼ 7, 32ð2  1; 626Þ < 3 < 10:

Thus, we obtain that ca1 > λ. This leads to a contradiction (see our assumption).
(c) Let a  b (see the notations of the part [a]).
At first, let us prove the following lemmas.
pffiffiffi
Lemma 1 Prove that the equation 2x4  6x2  2x þ 1 ¼ 0 has one root λ0in 2,

1 , with λ0 2 (1, 84; 1, 85).
ffi ¼  2x  6x  2x þ 1 ¼ 2(x
4 2 2
pffiffif(x)
Indeed, it is sufficient to note that the function
 2) þ 2(x  0, 5)  7, 5 is increasing in
2 2
2; 1 and that f(1, 84) ¼ 
0, 06902528, f(1, 85) ¼ 0, 1920125.
Lemma 2 Prove that a ¼ b, if and only if a1 ¼ b1.
Indeed, we phave thatffi PB þ BF
ffiffiffiffiffiffiffiffiffiffiffiffiffiffi ¼ c ¼ffiRE þ ES (see
pffiffiffiffiffiffiffiffiffiffiffiffiffiffi Figure ffi 7.64).pOn
pffiffiffiffiffiffiffiffiffiffiffiffiffiffi theffi other
ffiffiffiffiffiffiffiffiffiffiffiffiffiffi
hand, PB ¼ b b  c , BF ¼ a  c , RE ¼ a a  c , ES ¼ b  c2
b1 2 2 2 2 a1 2 2 2

Consequently
 
a1 pffiffiffiffiffiffiffiffiffiffiffiffiffiffi
ffi b1 pffiffiffiffiffiffiffiffiffiffiffiffiffiffi

1 a2  c2 ¼ 1 b2  c2 ð7:39Þ
a b

If a ¼ b, then from (7.39) it follows that 1  aa1 ¼ 1  bb1 , this means that a1 ¼ b1.
However, if a1 ¼ b1, but a 6¼ b, then a > b. On the other hand, from (7.39), it
follows that 1  aa1 < 1  bb1 . Hence, we deduce that b > a. This leads to the
contradiction and ends the proof of lemma 2.
Remark If a > b, then a1 > b1.
Indeed, from (7.39), we obtain that 1  aa1 < 1  bb1 . Thus, a1
b1 > ab > 1. Hence,
a1 > b 1.
Lemma 3 Prove that, if a1 ¼ b1 ¼ c1, then aa1 ¼ λ0 (see the notations of lemma 1).
Indeed, if a1 ¼ b1 ¼ c1, then according to lemma 2 we have that a ¼ b, and
according to the problem 7.1.90, it follows that λ ¼ aa1 > 1, 7. On the other hand,
1 pffiffiffiffiffiffiffiffiffiffiffiffiffiffiffi pffiffiffi  
λþ1 a2  c2 ¼ c. From triangle CQD, we obtain that 2c 1  1λ ¼ c1 ¼ aλ.
1þð1þ1λÞ
2
2
Consequently, 2ðλ  1Þ2 ¼ ac2 ¼ . Hence, we deduce that 2λ4  6λ2
ð1þλÞ
1 2

 2λ þ 1 ¼ 0, and according to lemma 1, we have that λ ¼ λ0.


384 7 Miscellaneous Inequalities

Lemma 4 Prove that, if a ¼ b, then minðba1 ;c1 Þ  λ0 (see the notations of lemma 1).
Let us introduce the following notations, a1 ¼ b1 ¼ aλ, c1 ¼ λa1 .
If we prove that λ  λ0 or λ1  λ0, then lemma 4 would be proven. Let

1 < λ < λ0 , λ1 < λ0 , ð7:40Þ


pffiffiffiffiffiffiffiffiffiffiffiffiffiffiffi  pffiffiffi
then we have that a2  c2 1 þ 1λ ¼ c and λa1 ¼ 2c ðλ1Þ
λ .
1þð1þ1Þ
2

Consequently, 2λ21 ¼  λ 2 . Now from the last equality and (7.40), it follows
1 12
λ  2
1þð1þ1λÞ
2 1þ 1þλ1
that 2λ20 > 2λ21 ¼  2 > 
0
2
1 12
λ 1 12
λ
0
 2  2
Hence 2λ20 1 1
λ20
> 1 þ 1 þ λ10 , or equivalently, 2λ40  6λ20  2λ0 þ 1 > 0.
This leads to a contradiction and ends the proof of the lemma 4. According to the
remark of lemma 2, we have that min (a, b, a1, b1, c1) ¼ min (b1, c1). Let
λ ¼ minðba1 ;c1 Þ.
If a ¼ b, then according to lemma 4, it follows that λ  λ0.
If a > b, then ∠PMA ¼ ∠FAB > ∠SAE ¼ ∠ANP. Thus, PM < PN (see
Figure 7.65).
Let us draw from point C a line parallel to the straight line F1S1 and consider the
pentagonal section AB0 CD0 E0 , with the sides equal to a0 , b01 , c01 , a01 , b0 , where
AB0 ¼ a0 , B0 C ¼ b01 , CD0 ¼ c01 , D0 E0 ¼ a01 , E0 A ¼ b0 . Note that a0 ¼ b0 , b01 > b1 ,
0 0
c01 > c1 , a > a01 , consequently, λ ¼ minðba1 ;c1 Þ > minðab1 ;c1 Þ > min ab0 ;c0 ¼ λ0 .
ð 1 1Þ
According to lemma 4, we have that λ0  λ0. Consequently, λ > λ0.
Thus, we have proven that in all cases λ  λ0 > 1, 84 (see lemma 1).
This ends the proof of the part [c].
From lemma 3 it follows that the number λ0 ¼ 1, 84267036 . . . is an exact
estimate.
This ends the proof.
7.1.91. Note that, if in triangle ABC, AC ¼ b, BC ¼ a, AB ¼ c and ∠C ¼ γ, then c
 ða þ bÞ sin 2γ (see Figure 7.66).

Figure 7.65 B

C
A

D

7.2 Solutions 385

a b a b c b
e e f

k
d f a k
c d
c d

a b c

Figure 7.66

Therefore, if γ  60 , then c  aþb


2 .
For the pyramid SA1A2 . . . An we have that

2A1 A2  SA1 þ SA2 , :::, 2An A1  SAn þ SA1 :

Summing up all obtained inequalities, we obtain that

A1 A2 þ A2 A3 þ ::: þ An A1  SA1 þ SA2 þ ::: þ SAn

This ends the proof.


V AA1 B1 C1 V ASB1 C1
7.1.92. Note that V AAVB1 ¼ AA11CB21  AA11 CB12 , V ASB1 C1 ¼ AA
SA ,
1
V ¼ SB
SB  SC , consequently
1 SC1
1 1 C1
V1
V ¼ AA11 CB21  AA11 CB12  SA 
AA1 SB1
SB  SC
SC . Thus, it follows that
1

rffiffiffiffiffiffi AA1 A1 B2 SB1 A1 C2 SC1 A1 C2 SC1


3 V1 þ  þ  1  SA
SA þ A1 C1  SB þ A1 B1 SC
1 A1 B2 SB1
 SA A1 C1 SB A1 B1 SC ¼ :
V 3 3

Similarly, one can prove the following inequalities


rffiffiffiffiffiffi rffiffiffiffiffiffi
3 V2 1  SB
SB þ B1 C1  SA þ A1 B1  SC
1 B1 A2 SA1 B1 C2 SC1
3 V3 1  SC
SC þ A1 C1  SB þ C1 B1  SA
1 C1 B2 SB1 C1 A2 SA1
 and  :
V 3 V 3
qffiffiffiffi qffiffiffiffi qffiffiffiffi
Summing up the last three inequalities, we obtain that 3 VV1 þ 3 VV2 þ 3 VV3  1, or
pffiffiffiffiffiffi pffiffiffiffiffiffi pffiffiffiffiffiffi pffiffiffiffi
equivalently, 3 V 1 þ 3 V 2 þ 3 V 3  3 V .
This ends the proof.
7.1.93. (a) Let DD0 ||AC (see Figure 7.67). Then according to Ptolemy’s theorem, we
obtain that AB  CD0 þ BC  AD0 ¼ AC  BD0 , AB  CD þ BC  AD ¼ AC  BD.
Note that CD ¼ AD0 , CD0 ¼ AD. Hence, it follows that (AB þ BC)(CD þ AD) ¼
AC(BD þ BD0 )  4R  AC.
386 7 Miscellaneous Inequalities

Figure 7.67
a

b c
db M dc
y z
da
y x z
a

Figure 7.68 C

A1
B1

M
E

A C1 B

(b) Let AB ¼ a, BC ¼ b, CD ¼ c, AD ¼ d, AC ¼ e, BD ¼ f (see Figure 7.68).


We have that

R2 ð a þ bÞ ð b þ c Þ ð c þ d Þ ð d þ aÞ ¼
¼ ððac þ bdÞ þ ad þ bcÞððac þ bdÞ þ cd þ abÞR2 ¼ pffiffiffiffiffi
¼ ðef þ ekÞðef þ fkÞR2 ¼ R2 ef ðf þ kÞðe þ kÞ  4R2 efk ef  e2 f 2  fk ¼
¼ ðac þ bd Þ2 ðab þ cd Þ,

as 2R  e, 2R  f.
(c) If we prove that e þ f þ k > a þ b þ c þ d (see the notations of the proof of
problem 7.1.93b), then e þ f þ 2R  e þ f þ k > a þ b þ c þ d.
Note that
 
ðe þ f þ kÞ2  ða þ b þ c þ dÞ2 ¼ e2 þ f 2 þ k2  a2 þ b2 þ c2 þ d 2 ¼
ðab þ cd Þðac þ bdÞ ðab þ cdÞðad þ bcÞ ðad þ bcÞðac þ bd Þ
¼ þ þ  a 2  b 2  c2  d 2 ¼
ad þ bc ac þ bd ab þ cd
abcd ða þ b þ c  dÞða þ b  c þ d Þða  b þ c þ d Þða þ b þ c þ dÞ
¼ >0
ðad þ bcÞðac þ bd Þðab þ cd Þ

holds true.
Therefore, we obtain that e þ f þ k > a þ b þ c þ d.
7.2 Solutions 387

(d) We have that S ¼ 12 ef  2R


k
, consequently ð4RSÞ2 ¼ ef  fk  ek ¼ ðac þ bdÞ
qffiffiffiffiffiffiffiffiffiffiffiffiffiffiffiffi
ðad þ bcÞðab þ cd Þ  8 ðabcd Þ3 .
pffiffiffi 3
Hence, it follows that 2RS  ðabcd Þ4 .
This ends the proof.
7.1.94. According to Ceva’s theorem, we have that AH HB  DC ¼ 1.
BD

Consequently, HB  AC ¼ 1, or AC  HB  BC ¼ 1.
AH AB AH BC AB
sin γ
Hence, cos α  cos1 β  sin α ¼ 1.
Thus, tgα ¼ sin α þ cos α  tgβ.
If α  45 , then β > 45 . Consequently, tgβ > 1. Hence we deduce that
tgα ¼ sin α þ cos αtgβ > sin α þ cos α > sin2α þ cos2α ¼ 1, thus α > 45 . This
leads to a contradiction, with the assumption that α  45 .
Therefore, it follows that α > 45 .
This ends the proof.
7.1.95. (a) At first, we need to prove that, if a, b, c, d  0, then 2ac þ 2bd  min (a þ
b, c þ d ) min (a þ d, b þ c).
Let min(a, b, c, d ) ¼ d.
Note that bd  d2, a(cd)d(cb). Therefore, 2ac þ 2bd  (c þ d)(a þ d ) 
min (a þ b, c þ d)  min (a þ d, b þ c).
Thus, it follows that

∠AOB  ∠COD þ ∠BOC  ∠AOD  0, 5minð∠AOB þ ∠BOC; ∠COD þ ∠AODÞ  


minð∠AOB þ ∠AOD; ∠BOC þ ∠CODÞ  0, 5  ∠AOC  ∠BOD

(see problem 7.1.35a).


(b) We have that kAB k  k CD k þ k BC k  k AD k  0, 5 min (kAB k þ k BCk,
kCD k þ k ADk)  min (kAB k þ k ADk, kBC k þ k CDk) (see the proof of
the problem 7.1.95a).
According to the condition (3) of the problem we have that kAB k þ
k AD k  k BDk and kBC k þ k CD k  k BDk, consequently min(kAB k þ
k ADk, kBC k þ k CDk)  k BDk.
Similarly, we obtain that min(kAB k þ k BCk, kCD k þ k ADk)  k ACk.
Hence, kAB k  k CD k þ k BC k  k AD k  0, 5 k AC k  k BDk.
(B) Let point O does not belong to plane α. For any points A and B on plane α ,
define kAB k ¼ ∠AOB. Then, it is not difficult to verify that the conditions (1),
(2), and (3) of the problem are satisfied (see problem 7.1.35a).
Let C0 > 0 be such that the inequality kAB k  k CD k þ
k BC k  k AD k  C0 k AC k  k BDk holds true for any points A, B, C and D on
plane α. Let O0 be the projection of point O on plane α and AnBnCnDn be a square
with the center O0 and O0 An ¼ n.
388 7 Miscellaneous Inequalities

We have that kAnBn k  k CnDn k þ k BnCn k  k AnDn k  C0 k AnCn k 


k BnDnk. Since lim k An Bn k¼ lim k Bn Cn k¼ lim k Cn Dn k¼ lim k An Dn k¼
n!1 n!1 n!1 n!1
π
2 and lim k An Cn k¼ lim k Bn Dn k¼ π, then obtain that π2  π2 þ π2  π2  C0 π  π.
n!1 n!1
Therefore, it follows that C0  12.
This leads to a contradiction.
7.1.96. (a) Note that OA1
AA1 þ OB
BB1 þ CC1 ¼ SABC þ SABC þ SABC ¼ 1. Consequently 2 ¼
1 OC1 SBOC SAOC SAOB

2
R
þ RþOB
R
þ RþOC
R
 3R2 þRðOA9RþOB ; hence OA1 þ OB1 þ OC1  1, 5R.
RþOA1 1 1 1 1 þOC1 Þ

(b) We have that 1 ¼ 1þ1 R þ 1þ1 R þ 1þ1 R  3þ 9


.
OA1 þOB1 þOC1
R R R
OA1 OB1 OC1

Therefore, we obtain that OA1 1 þ OB1 1 þ OC


1
1
 R6 .
7.1.97. Note that d2Sb ¼ xþyþz
y
, d2Sa ¼ xþyþz
x
,d2Sc ¼ xþyþz
z
(see Figure 7.69).
b a c
Therefore, it follows that
rffiffiffiffiffiffiffiffiffiffiffiffiffiffiffiffiffiffiffiffiffiffi rffiffiffiffiffiffiffiffiffiffiffiffiffiffiffiffiffiffiffiffiffiffi rffiffiffiffiffiffiffiffiffiffiffiffiffiffiffiffiffiffiffiffiffiffi
pffiffiffiffiffi pffiffiffiffiffi pffiffiffiffiffi pffiffiffiffiffi 1 x 1 y 1 z
d a þ db þ dc ¼ 2S  þ  þ  
a x þ y þ z b x þ y þ z c x þ yþz ffi
sffiffiffiffiffiffiffiffiffiffiffiffiffiffiffiffiffiffiffiffiffiffiffiffiffiffiffiffiffiffiffiffiffiffiffiffiffiffiffiffiffiffiffiffiffiffiffiffiffiffiffiffiffiffiffiffiffiffiffiffiffiffiffiffiffiffiffiffiffiffiffiffiffiffiffiffiffiffiffiffiffiffiffiffiffiffiffiffiffi
 
pffiffiffiffiffi 1 1 1 x y z
 2S þ þ þ þ
a b c xþyþz xþyþz xþyþz
rffiffiffiffiffiffiffiffiffiffiffiffiffiffiffiffiffiffiffiffiffiffiffiffiffiffiffiffiffiffiffiffiffiffiffi
2S
¼ ðab þ bc þ acÞ ¼
abc
rffiffiffiffiffiffiffiffiffiffiffiffiffiffiffiffiffiffiffiffiffiffiffiffi
ab þ bc þ ac
¼ :
2R
pffiffiffiffiffi pffiffiffiffiffi pffiffiffiffiffi qffiffiffiffiffiffiffiffiffiffiffiffiffiffi ffi
Consequently, d a þ db þ dc  a þb2Rþc .
2 2 2

If q > 90 , then b2 þ c2 < a2. Hence,


α ffiffiffiffiffiffiffiffiffiffiffiffiffiffiffiffiffiffiffiffiffi ab þ bc þ ac ¼ aðb þ cÞþ
 2  b2 þc2 pffiffiffi 1 2
bc  a 2 b þ c2 þ 2 < 2 þ 2 a < 1, 92a . 2

pffiffiffiffiffi pffiffiffiffiffi pffiffiffiffiffi qffiffiffiffiffiffiffiffiffiffiffiffiffiffiffiffiffiffiffiffiffiffiffiffiffiffiffiffiffiffiffiffiffiffiffi pffiffiffiffiffiffiffiffiffiffiffiffi


Therefore, da þ d b þ dc < 1, 922R a2
 1, 92a.
pffiffiffi
If α  90 , then 60  α  90 . Consequently, a  3R and since
a þb þc
2 2 2
2R ¼ 2R ð sin 2
α þ sin 2
β þ sin 2
γ Þ, then according to the inequality 5.1.1, we
have that

Figure 7.69
C

O b M
b M0
b
A B
7.2 Solutions 389

pffiffiffiffiffi pffiffiffiffiffi pffiffiffiffiffi pffiffiffiffiffiffiffiffiffiffiffiffiffiffiffiffiffiffiffiffiffiffiffiffiffiffiffiffiffiffiffiffiffiffiffiffiffiffiffiffiffiffiffiffiffiffiffiffiffiffi qffiffiffiffiffiffi qffiffiffiffiffiffiffiffiffiffiffiffiffiffiffiffiffiffi


pffiffi pffiffiffi qffiffiffiffiffiffiffiffiffiffiffi
pffiffi
da þ d b þ dc  2Rðsin 2 α þ sin 2 β þ sin 2 γ Þ  92 R ¼ 3 2 3  3R  3 2 3  a.

Remark For the triangle with angles α ¼ π2 , β ¼ γ ¼ π4 and a point M on the altitude
pffiffiffi pffiffiffiffiffi pffiffiffiffiffi pffiffiffiffiffi qffiffiffiffiffiffiffiffiffiffiffiffiffiffiffiffiffiffiffiffiffi
pffiffiffi 1
AM
AH, where MH ¼ 2 2, we have that d a þ db þ d c ¼ 2 þ 2 a.

7.1.98. Since AE ¼ 2, then AE is a diameter of the circle. Consequently, 4 ¼ AE2


¼ AC2 þ CE2 ¼ a2 þ b2 þ 2ab cos α þ c2 þ d2 þ 2cd sin α, where ∠AEC ¼ α.
Hence, 4 ¼ a2 þ b2 þ c2 þ d2 þ ab  CE þ cd  AC. Since ∠ABC, ∠CDE > 90 ,
then CE > CD ¼ c, AC > BC ¼ b. Therefore, 4 > a2 þ b2 þ c2 þ d2 þ abc þ bcd.
7.1.99. (a) The proof is shown by mathematical induction. Since pffiffiffi
ð~
a1 þ ~
a2 Þ2 þ ð~
a1  ~ a2 Þ2 ¼ 2~a1 2 þ 2~a2 2  4, then minðj~ a2 j; j~
a1 þ ~ a1  ~
a2 j Þ  2.
Hence, the statement holds true for n ¼ 2.
For n  3, we have that, if ~ a1 ,~
a2 ,~
a3 are non-vanishing vectors, then two of the
vectors ~a1 ,  ~
a2 ,  ~ a3 form an angle not exceeding 60 . Thus, the difference of
those two vectors has a length not exceeding 1. If even one of ~ a1 ,~
a2 ,~a3 is a
non-vanishing vector, then the last statement is also correct. Consequently, by
replacing these two vectors by their difference we obtain n  1 vector, satisfying
the conditions of the problem.
This ends the proof.
(b) At first, we need to prove that, if there are 15 non-vanishing vectors in the space,
then among them one can find two vectors, such that the angle between them is
less than 60 . Move the starting points of all 15 vectors to the same point and
consider a unit sphere with the center at that point.
Consider 15 cones, such that their altitudes contain the given vectors and the
bases of the cones lie on the surface of the sphere and form “caps.” Let the angles of
the axial sections of p these
ffiffi cones be equal to 60 , then the area of one “cap” is equal
 pffiffiffi  pffiffiffi
to 2πRH ¼ 2π 1  23 ¼ π 2  3 . We need to prove that 15π 2  3 > 4π
pffiffiffi
or 26 > 15 3, 676 > 675.
It follows that, among these “caps” there are two overlapping, then the angle
between corresponding vector is less than 60 .
Let n  7. Consider all possible vectors  2 ~ c1 ¼ ~ a1 þ~a2 þ ::: þ ~ an , :::,
~
c2n ¼ ~ an , then ~
a1  :::  ~ c1 2 þ ::: þ~ c 2 2 n ¼ 2n ~a1 þ ::: þ ~ an 2 . Consequently,
there
 exists a number  i, such that ci j2  j~
2n j~ c1 j2 þ ::: þ j~c2n j2 ¼
p ffiffi
ffi p ffiffi

2n j~ an j2  n2n . Hence, j~
a1 j2 þ ::: þ j~ c i j  n  7.
We proceed the proof of the problem by mathematical induction. At n  7, this
ends the proof of the statement. If among the vectors~ a1 , :::,~
an is a zero-vector, then
we obtain that the statement needs to be proven for n  1 vectors. This leads to a
contradiction.
If there is no zero-vector among~ a1 , :::,~
an vectors, then since~ a1 , :::,~
a8 vectors are
non-vanishing, then among 16 vectors ~ a8 ,  ~
a1 , :::,~ a1 , :::,  ~
a8 there are two
vectors, forming angle not exceeding 60 . Therefore, the length of the difference
390 7 Miscellaneous Inequalities

of these vectors does not exceed 1. Consequently, replacing these two vectors by
their difference we obtain n  1 vectors, satisfying the conditions of the problem,
such that the statement holds true.
(c) Let us obtain the vectors ~
a1 ðx1 ; y1 Þ, :::,~
an ðxn ; yn Þ.
Let M1 ¼ {(x, y)| x  0, y  0}, M2 ¼ {(x, y)| x  0, y  0}, M3 ¼ {(x, y)| x  0,
y  0}, M4 ¼ {(x, y)| x  0, y 0}. As (xi, yi) 2 M1 [ M2 [ M3 [ Mk, then there are
ai1 j þ ::: þ j~
numbers i1, . . . , ik, such that xij ; yij 2 Mp j ¼ 1, . . . , k and j~ aik j  14.
Consequently,
qffiffiffiffiffiffiffiffiffiffiffiffiffiffiffiffiffiffiffiffiffiffiffiffiffiffiffiffiffiffiffiffiffiffiffiffiffiffiffiffiffiffiffiffiffiffiffiffiffiffiffiffiffiffiffiffiffiffiffiffiffiffiffiffi
 2  
1 
j~ aik j ¼
ai1 þ ::: þ~ ðxi1 þ ::: þ xik Þ2 þ yi1 þ ::: þ yik  pffiffiffi jxi1 þ ::: þ xik j þ yi1 þ ::: þ yik  ¼
2
1      1 qffiffiffiffiffiffiffiffiffiffiffiffiffiffiffiffiffiffiffi qffiffiffiffiffiffiffiffiffiffiffiffiffiffiffiffiffiffiffi
¼ pffiffiffi jxi1 j þ yi1  þ ::: þ jxik j þ yik   pffiffiffi xi1 2 þ yi1 2 þ ::: þ xik 2 þ yik 2 ¼
2 2
pffiffiffi
1 2
¼ pffiffiffiðj~
ai1 j þ ::: þ j~aik jÞ  :
2 8
pffiffi
Remark If the vectors ~
a1p,ffiffi:::,~
an are given in the space, then the number 8
2
must be
3
replaced by the number 24 .
7.1.100. (1) Let ∠A1C1B1 ¼ 90 . Note that points C and C1 lie on the circle with the
diameter A1B1. Consequently, A1 B1  CC1  hc ¼ AB 2 sin 2β, where β ¼ ∠ABC.

(2) Let ∠A1B1C1 ¼ 90 and a circle with the diameter A1B1 intersects segment A1C1
at point E (see Figure 7.70).
Take a point M on the arc B1E, such that ∠A1MC1 ¼ 180  β. This is possible
since ∠A1B1C1 ¼ 90 < 180  β < 180 ¼ ∠A1EC1. As ∠A1MC1 þ ∠A1BC1 ¼
180 , then points A1, B, C1, M lie on the same circle with a radius R.
By the law of sines, we have that

A1 C1 ¼ 2R sin β  BM sin β ð7:41Þ


Note that ∠B1MC1 ¼ 360  ∠B1MA1  ∠A1MC1 ¼ 90 þ β ¼ 180  ∠BAC.
Hence, points A, B1, M, C1 lie on the same circle.

Figure 7.70 A

1200

S
C⬘ O⬘ B⬘
7.2 Solutions 391

Figure 7.71 T

T⬘

Since ∠AMC ¼ ∠AMB1 þ ∠CMB1 ¼ ∠AC1B1 þ ∠CA1B1 ¼ 90 þ β, then M is


on the arc with the endpoints A and C (Figure 7.71). Thus, BM þ
MO  BO ¼ BM0 þ M0O.
Consequently,
sffiffiffiffiffiffiffiffiffiffiffiffiffiffiffiffiffiffiffiffiffiffiffiffiffiffiffiffiffiffiffiffiffiffiffiffiffiffiffiffiffiffiffiffiffiffiffiffiffiffiffi
AB2 2 AB
BM  BM0 ¼ BO  OM0 ¼ AB2 þ tg β  tgβ ð7:42Þ
4 2

According to (7.2) and (7.3), we deduce that A1 C1 


pffiffiffiffiffiffiffiffiffiffiffiffiffiffiffiffiffiffi
AB
2 4 þ tg 2 β  tgβ sin β.

∠B1A1C1 ¼ 90
pIfffiffiffiffiffiffiffiffiffiffiffiffiffiffiffiffiffiffiffi  , similarly we obtain that B1 C1  AB
2

4 þ ctg β  ctgβ cos β.
2
pffiffiffiffiffiffiffiffiffiffiffiffiffiffiffiffiffiffi 
Let β  45 , we need to prove that 4 þ tg 2 β  tgβ sin β 
pffiffiffiffiffiffiffiffiffiffiffiffiffiffiffiffiffiffiffiffi 
4 þ ctg 2 β  ctgβ cos β  sin 2β.
pffiffiffiffiffiffiffiffiffiffiffiffiffiffiffiffiffiffiffiffi
The last inequality holds true, as 4 þ ctg 2 β  ctgβ þ 2 sin β or 4cos2
β  4 cos β. pffiffiffiffiffiffiffiffiffiffiffiffiffiffiffiffiffiffi  pffiffiffiffiffiffiffiffiffiffiffiffiffiffiffiffiffiffiffiffi
We have to prove that 4 þ tg 2 β  tgβ tgβ  4 þ ctg 2 β  ctgβ, or
pffiffiffiffiffiffiffiffiffiffiffiffiffiffiffiffiffiffiffiffi  pffiffiffiffiffiffiffiffiffiffiffiffiffiffiffiffiffiffi 3ð1tg 2 βÞ
4 þ ctg 2 β þ ctgβ tgβ  4 þ tg 2 β þ tgβ, 1  tgβ  pffiffiffiffiffiffiffiffiffiffiffi pffiffiffiffiffiffiffiffiffiffiffiffiffi.
4þtg 2 βþ 4tg 2 βþ1
pffiffiffiffiffiffiffiffiffiffiffiffiffiffiffiffiffiffi pffiffiffiffiffiffiffiffiffiffiffiffiffiffiffiffiffiffiffiffi
Therefore, we need p to ffiffiffiffiffiffiffiffiffiffiffiffiffiffiffiffiffi
prove that ffi 4 þ tg 2 β þpffiffiffiffiffiffiffiffiffiffiffiffiffiffiffiffiffiffiffiffi
4tg 2 β þ 1  3 þ 3tgβ. Note
that, this holds true, as 4 þ tg β < 2 þ tgβ and 4tg 2 β þ 1 < 2tgβ þ 1.
2

One can easily prove that, if M0A1 ⊥ BC, M0C1 ⊥ AB, M0B1 ⊥ AC, then
pffiffiffiffiffiffiffiffiffiffiffiffiffiffiffiffiffiffi
∠A1B1C1 ¼ 90 and A1 C1 ¼ AB 2 4 þ tg 2 β  tgβ sin β. Hence, the smallest
possible value for  the hypotenuse
pffiffiffiffiffiffiffiffiffiffiffiffiffiffiffiffiffi of triangle A1B1C 1 is equal to

AB
2 4 þ tg 2 β  tgβ sin β.

7.1.101. We need to prove that, if ∠SAC < 60 , ∠SAB < 60 , then ∠SAO < 60 .
Consider a cone with the altitude AS (see Figure 7.72). Let the angle of the axial
section of the cone be equal to 120 and lines AB, AC, AO intersect the plane of the
392 7 Miscellaneous Inequalities

i
i
3 k 3

2 α2 2 k
α1 αk
1 1
O

Figure 7.72

base of the cone at points B0 , C0 , O0 , respectively. Then, points B0 and C0 belong to


the base of the cone, and consequently O0 also belongs to the base of the cone, then
∠SAO ¼ ∠SAO0 < 60 .
We proceed the proof by contradiction argument. Assume that, either ∠SAC or
∠SAB  60 . Let ∠SAC  60 , consequently ∠SCA < 60 . Hence, ∠SCB  60
and ∠SBC < 60 , ∠SBA  60 and ∠SAB < 60 . Then, we obtain that SC > SA,
SB > SC and SA > SB. Therefore, SC > SA > SB > SC. This leads to a contradiction.
7.1.102. Let O be the center of the circle with the radius t and points F,E be the
midpoints of sides AB,AC, respectively. Without loss of generality, one can assume
that O 2
= EF. Consequently, OE þ OF > EF or t  AC 2 þ t  2 > 2 . Hence, t > 2.
AB BC p

Let m, n, k, x > 0. Note that the only solution of the equation


pffiffiffiffiffiffiffiffiffiffiffiffiffiffiffiffiffiffiffiffiffiffiffiffiffiffiffiffiffiffiffiffi pffiffiffiffiffiffiffiffiffiffiffiffiffiffiffiffiffiffiffiffiffiffiffiffiffiffiffiffiffiffiffiffi pffiffiffiffiffiffiffiffiffiffiffiffiffiffiffiffiffiffiffiffiffiffiffiffiffiffiffiffiffiffiffi
mnkðm þ n þ kÞ ¼ mnxðm þ n þ xÞ þ mkxðm þ k þ xÞ
pffiffiffiffiffiffiffiffiffiffiffiffiffiffiffiffiffiffiffiffiffiffiffiffiffiffiffiffiffi
þ nkxðn þ k þ xÞ ð7:43Þ

mnk
is the number x0 ¼ pffiffiffiffiffiffiffiffiffiffiffiffiffiffiffiffiffiffiffiffiffiffiffiffiffiffiffiffiffiffiffiffi.
mn þ nk þ mk þ 2 mnkðm þ n þ kÞ
It is clear that the equation (7.4) has not more than one solution and
qffiffiffiffiffiffiffiffiffiffiffiffiffiffiffiffiffiffiffiffiffiffiffiffiffiffiffiffiffiffiffiffiffiffiffiffiffiffiffiffiffiffiffiffiffiffiffiffiffiffiffiffiffiffiffiffiffiffiffiffiffiffiffiffiffiffiffiffi
pffiffiffi pffiffiffiffiffiffiffiffiffiffiffiffiffiffiffiffiffiffiffiffiffiffiffiffiffiffiffiffiffiffi2 pffiffiffi
pffiffiffiffiffiffiffiffiffiffiffiffiffiffiffiffiffiffiffiffiffiffiffiffiffiffiffiffiffiffiffiffiffiffiffiffi mn kðm þ nÞ þ mnðm þ k þ nÞ k
mnx0 ðm þ n þ x0 Þ ¼ pffiffiffiffiffiffiffiffiffiffiffiffiffiffiffiffiffiffiffiffiffiffiffiffiffiffiffiffiffiffiffiffi ¼
mn þ nk þ mk þ 2 mnkðm þ n þ kÞ
 pffiffiffi pffiffiffiffiffiffiffiffiffiffiffiffiffiffiffiffiffiffiffiffiffiffiffiffiffiffiffiffiffiffipffiffiffi
mn kðm þ nÞ þ mn mnðm þ k þ nÞ k
¼ pffiffiffiffiffiffiffiffiffiffiffiffiffiffiffiffiffiffiffiffiffiffiffiffiffiffiffiffiffiffiffiffi
mn þ nk þ mk þ 2 mnkðm þ k þ nÞ
pffiffiffiffiffiffiffiffiffiffiffiffiffiffiffiffiffiffiffiffiffiffiffiffiffiffiffiffiffiffiffiffi
mnkðm þ nÞ þ mn mnkðm þ k þ nÞ
¼ pffiffiffiffiffiffiffiffiffiffiffiffiffiffiffiffiffiffiffiffiffiffiffiffiffiffiffiffiffiffiffiffi ,
mn þ nk þ mk þ 2 mnkðm þ k þ nÞ
thus
7.2 Solutions 393

pffiffiffiffiffiffiffiffiffiffiffiffiffiffiffiffiffiffiffiffiffiffiffiffiffiffiffiffiffiffiffiffiffiffi pffiffiffiffiffiffiffiffiffiffiffiffiffiffiffiffiffiffiffiffiffiffiffiffiffiffiffiffiffiffiffiffiffi pffiffiffiffiffiffiffiffiffiffiffiffiffiffiffiffiffiffiffiffiffiffiffiffiffiffiffiffiffiffiffi


mnxðm þ n þ x0 Þ þ mkxðm þ k þ x0 Þ þ nkxðn þ k þ x0 Þ ¼
pffiffiffiffiffiffiffiffiffiffiffiffiffiffiffiffiffiffiffiffiffiffiffiffiffiffiffiffiffiffiffiffi
mnkð2m þ 2n þ 2kÞ þ ðmn þ nk þ mkÞ mnkðm þ k þ nÞ
¼ pffiffiffiffiffiffiffiffiffiffiffiffiffiffiffiffiffiffiffiffiffiffiffiffiffiffiffiffiffiffiffiffi
mn þ nk þ mk þ 2 mnkðm þ k þ nÞ
pffiffiffiffiffiffiffiffiffiffiffiffiffiffiffiffiffiffiffiffiffiffiffiffiffiffiffiffiffiffiffiffi
¼ mnkðm þ n þ kÞ:

Let D be the midpoint of side BC. Denote by DE ¼ m þ n, FD ¼ m þ k,


FE ¼ n þ k, then OD ¼ t  (n þ k) ¼ x þ m, where x ¼ t  (m þ n þ k) > 0,
OE ¼ x þ n, OF ¼ x þ k. One can easily prove that point O is inside of triangle
DEF(see the proof of problem 1.1.4a).
Therefore, SDEF ¼ SDOE þ SDOF þ SEOF; hence
x¼ pffiffiffiffiffiffiffiffiffiffiffiffiffiffiffiffiffiffiffiffi. If we take into account that mn þ nk þ mk 
mnk
mnþnkþmkþ2 mnkðmþnþkÞ
pffiffiffipffiffiffiffiffiffiffiffiffiffiffiffiffiffiffiffiffiffiffiffiffiffiffiffiffiffiffiffiffiffiffiffi  pffiffiffiqffiffiffiffiffiffiffiffiffiffiffi
3 mnkðm þ n þ kÞ, then it follows that x  2  3 mþnþk mnk
¼
 p ffiffiffiffiffiffiffiffiffiffiffiffiffiffiffiffiffiffiffi
pffiffiffi mnkðmþnþkÞ  ffi pffiffiffi r  pffiffi 
2 3 mþnþk ¼ 2  3  2. Hence, t  p2  1  23 r.

7.1.103. (a) Let T be an equilateral triangle containing all points A1, . . . , An, such
that each side of the triangle contains at least one of these points. Denote by T0
another equilateral triangle with the same properties as of T (Figure 7.73), sides of
T and T0 are parallel.
Note that, the sum of the distances between the parallel sides of these triangles
(see the proof of problem 7.1.76a) is equal to the sum of the lengths of the altitudes
of these triangles. On the other hand, it does not exceed 3D. Hence, it follows that
3D
the altitude of at least onepof
ffiffiffi the triangles does not exceed 2 , and the side of that
triangle does not exceed 3D.
(b) Here, we use that the length of the side of the equilateral triangle T is a
continuous function of its direction.
Consequently, the difference of the sides of triangles T and T0 by the rotation of
the side of triangle T by angle π changes the sign. Hence, there exists a direction of
the side of triangle T, such that triangles T and T0 have equal sides.

Figure 7.73 i i+1 E B


D
αi αi

O
M

A
C
394 7 Miscellaneous Inequalities

Intersection of such triangles is a centrally-symmetric “hexagon,” the distances


between its parallel sides do not exceed D. Consequently, that hexagon can be
placed inside of the centrally-symmetric hexagon with angles 2π 3 , with the distances
between the parallel sides equal to D. Then, it is a regular hexagon and the length of
its side is equal to pDffiffi3.

(c) This result follows


  (b). We provide also another proof. Let n  3. Consider
from
the circles ωi Ai ; pffiffi , i ¼ 1, 2, . . . , n. Note that any three of these circles have a
D
3
common point.
Indeed, if points Ai, Aj, Ak lie on the same straight line, such that point Aj belongs
to segment AiAk, then the midpoint of that segment belongs to the circles with the
centers Ai, Aj, Ak, as D2 < pDffiffi3.
If points Ai, Aj, Ak are the vertices of some acute triangle, then the center O of the
circumcircle of that triangle belongs to the circles with the centers Ai, Aj, Ak, as one
can assume that ∠Ai Aj Ak  π3. Hence, we obtain that

Ai Ak D
OAi ¼ OAj ¼ OAk ¼ 
2 sin ∠Ai Aj Ak 2 sin π3 ¼ pDffiffi3 :

If ∠Ai Aj Ak  π2, then the required point is the midpoint of segment AiAk.
Since any three of the circles ω1, ω2, . . . , ωn have a common point, then
according to Helly’s theorem,  allthese circles have a common point. Let O be
that point, then the circle ω O; pDffiffi3 contains all these points A1, A2, . . . , An.
 
(d) According to problem (c), there exists a circle ω O; pDffiffi3 containing all points
 
A1, A2, . . . , An. Then the circles ωi Ai ; d2 i ¼ 1, 2, . . . n do not have common
 
internal points and are contained in the circle ω0 O; pDffiffi3 þ d2 . Consequently, the
sum of the areas of the circles ω1, . . . , ωn is less than the area of the circle ω0 .
2
 2 pffiffi pffiffiffi
Therefore n  π  d4 < π pDffiffi3 þ d2 , hence D > 23 ð n  1Þd.
 
(e) According to the problem (c), there exists a circle ωi O; pDffiffi3 containing all
points A1, A2, . . . , A6. If point O coincides with point Ai, then for j 6¼ i, we have
that d  Aj Ai ¼ Aj O  pDffiffi3.

If O does not coincide with any of points A1, A2, . . . , A6, then there exist points
Ai and Aj, such that ∠Ai OAj  π3. Consequently, d  Ai Aj  max Ai O; Aj O  pDffiffi3.

7.1.104. (a) Let G be the centroid of triangle ABC. We have that AG, BG, GG  23,
since each median divides the triangle into two triangles of the equal area. Note that
7.2 Solutions 395

maxð∠AGB; ∠BGC; ∠AGCÞ  2π 3 .pffiffi Letpffiffi ∠AGB  3 , then



SABC ¼ 3SAGB ¼
3  2 AG  BG sin ∠AGB  2  3  3  2 ¼ 3 . For the pequilateral
1 3 2 2 3 3
ffiffi triangle with the
3
median equal to 1, we have that the area is equal to 3 .
(b) Let O be the intersection point of the diagonals of the convex quadrilateral
ABCD and BO  OD, CO  AO.
If AB k DC, then OD
BO
¼ CO
AO
, consequently BO ¼ OD and AO ¼ OC.
Therefore, the quadrilateral ABCD is a parallelogram.
Then, AC  1, BD  1, consequently SABCD  12 AC  BD  12 < 23.
If AB=k DC, then let M be the intersection point of rays BA and CD, with
BC ¼ b, ∠ABC ¼ β and ∠DCB ¼ γ.
b2
We have that S ¼ SABCD < SAMC ¼ 12  ctgβþctgα , consequently

b2
Sðctgα þ ctgβÞ < : ð7:44Þ
2

Let A1 and D1 be points on sides AB and CD, respectively, such that SBA1 C ¼ S2
and SBD1 C ¼ S2. Then, we have that CA1  1 and BD1  1, hence
S2 cos β
 2 S2
CA21 ¼ b2 þ b2 sin 2 β  2S sin β ¼ b  bctgβ
S
þ b2  1. Similarly, we obtain that
S2
 2
b2
þ b  bctgγ  1. Consequently, according to (7.5), it follows that
S

 2 
2S2 1 S 2S2 1 b 2 2S2 9b2
2 þ 2b  ð ctgβ þ ctgγ Þ > þ 2b  ¼ 2 þ  3S:
b2 2 b b2 2 2 b 8

Hence, we deduce that S < 23.


For the quadrilateral ABCD, where AB ¼ BC ¼ CD ¼ pffiffiffiffiffiffiffiffiffiffiffiffiffiffi 1
pffiffi and
pffiffiffi 3814 7
∠B ¼ ∠C ¼ β, with cos β ¼ 3  7, onepcan easily verify that the assumptions
ffiffiffiffiffiffiffiffiffiffiffiffiffiffi
pffiffi
20þ14 7
of the problem are satisfied, and that S ¼ 12 ¼ 0, 6293764:::.
(c) Let us call the median the segment connecting the vertex of n-gon with the point
on its boundary, dividing its area S into two equal parts.
Let us draw the medians of the polygon and note that any two of them either
coincide or intersect. Otherwise, they divide n-gon into three parts, such that two of
them have the area S2.
Let n-gon have k medians (3  k  n). Let us draw through the center O of the
circle ω diameters parallel to these medians (Figure 7.74) and enumerate these
diameters (in the clockwise direction) by the numbers 1, 2, . . . k.
Two medians are called adjacent, if they are parallel to the diameters with
numbers i and i þ 1(1  i  k  1) or k and 1.
396 7 Miscellaneous Inequalities

Figure 7.74 B
A M C0⬘ C
A0⬘
A0 M0 C0

O ω

Figure 7.75


!
!
Note that, if the medians AB and CD are adjacent, then points B and D (A and C)
belong to the same side of the n-gon. Otherwise, there is one more diameter
between the diameters with numbers i and i þ 1 (Figure 7.75).

!
!
Therefore the adjacent medians AB and CD form a “butterfly.” This means a
figure composed of triangles AMC and BMD, where M is the intersection point of
the medians. Note that SAMC ¼ SBMD, then the area of the “butterfly” is equal to
SAMC þ SBMD.
Since (AM þ MC) þ (BM þ MD) ¼ AB þ CD  2, then AM þ MC  1 or
BM þ MD  1. Let AM þ MC  1, then SAMC þ SBMD ¼ 2SAMC ¼ AM  MC  sin αi
Þ2
 ðAMþMC
4 sin αi  sin4 αi .
One can prove by mathematical induction, that the “butterflies” formed by the
medians 1 and 2, 2 and 3, . . . , i  1 and i cover the shaded part of the n-gon
(Figure 7.73). Consequently, all “butterflies” cover all of the n-gon. Hence, it
follows that S  14 ð sin α1 þ sin α2 þ ::: þ sin αk Þ  14 sin α1 þα2 þ:::þα
k
k
¼ 4k sin πk
π π
 4 sin n. This means that S  4 sin n (see problems 5.4.15 and 8.2.1e). Note that
n n

S ¼ n4 sin πn, if k ¼ n, all medians are concurrent and are divided by the intersection
point into two equal parts. This leads to a contradiction. Hence, we obtain that
S < n4 sin πn.
Remark If each chord dividing the area of a planar convex figure into two equal
parts has a length not exceeding 1, then the area of that figure does not exceed π4.
7.2 Solutions 397

Figure 7.76

7.1.105. Let in triangle ABC we have that AB  BC  AC and point M is the


midpoint of segment AC. Since BM > 2SAB ABC
¼ 4SAB
ABM
¼ 2BM sin ∠ABM, then
sin ∠ABM < 2. Consequently, ∠ABM < 30 or ∠ABM > 150 .
1 

If ∠ABM < 30 , then consider a parallelogram ABCD. In triangle BCD we have
that BC  AB ¼ CD, consequently ∠CBM  ∠CDB ¼ ∠ABM. Hence, ∠ABC 
2 ∠ABM < 60 . This leads to a contradiction.
We have obtained that ∠ABM > 150 . Consider the circumcircle of triangle
ABM. Take on that circle points A0 and M0, such that ∠A0BM0 ¼ 150 and AM||
A0M0. Let A0M0 ¼ M0C0 (see Figure 7.76), since AM||A0M0 and A0M0 ¼ M0C0, then
we have that A00 M > MC0 0 .
Therefore CM ¼ AM > A00 M > MC0 0 , consequently ∠ACB < ∠AC0 0B ¼
∠A0C0B.
Let B0 lie on the arc A0BM0 and C0B0 be a tangent to the circle ω. Then
∠A0C0B  ∠A0C0B0, hence ∠ACB < ∠A0C0B0. If we prove that ∠A0C0B0 <
5 300 , then this ends the proof of the problem.
pffiffiffi    qffiffi
Note that B0 C0 ¼ 2A0 M0 and cos 30 þ ∠A0 C0 B0 ¼ 23. Let us prove that,
qffiffi
if 0 < α < 90 and cos α ¼ 23, then α < 35 300 .
  
We have that cos 2α ¼ 13, а cos 180  4α ¼ 79. Now, if we prove that

cos 38 > 79, then as α < 45 and cos(180  4α) < cos 38 , it follows that
180  4α > 38 .
This means that α < 35 300 .
Let cos φ ¼ 79 and 00 < φ < 90 , then cos  3φ4 ¼ 729 < 729 ¼ 9.
329 324 4


Since φ < 60 and cos 180  3φ > 9, then if we prove that 49 >
 
sin 24 ¼ cos 66 , we deduce that 180  3φ < 66 , or φ > 38 . Hence,

9 ¼ cos φ < cos 38 .
7


It remains to prove that sin 24 < 49. Let sin β ¼ 49 andp0ffiffiffiffiffiffiffiffiffiffiffiffi < βffi < 90 . Since
pffiffi
β < 30 , then it is sufficient to prove that sin72
pffiffiffi

< sin 3β, or p4 ffiffiffi < 729.
10þ2 5 716

Note that 162 2
< 415
pffiffiffiffiffiffiffiffiffiffiffiffi  83, hence 162 5 < 415, or 81 10 þ 2 5 < 1225.
pffiffiffi
10þ2 5
Therefore, 4 < 3536 < 729. This ends the proof.
716

7.1.106. Without loss of generality, one can assume that AH ¼ 1 and the equation of
line l has the form x cos φ þ y sin φ  ρ ¼ 0.
398 7 Miscellaneous Inequalities

Note that, if B(ctgβ, 0), C(ctgγ, 0), A(0; 1), then u ¼ jsin φ  ρj,
v ¼ j cos φctgβ  ρj, w ¼ jcosφctgγ  ρj, а 2S ¼ ctgβ þ ctgα.
We have to prove that (sin φ  ρ)2tgα þ (cos φ  ctgβ þ ρ)2tgβ þ
(cos φ  ctgγ  ρ) tgγ  ctgα þ ctgβ, or
2

ðtgβ þ tgγ  tg ðβ þ γ ÞÞρ2 þ 2 sin φtgðβ þ γ Þρ þ cos 2 φðctgβ þ ctgγ Þ


tgβ þ tgγ
sin 2 φtgðβ þ γ Þ  :
tgβtgγ

Since tgβ, tgγ > 0 and tgβ  tgγ > 1, then we have to prove that tg2βtg2γ  ρ2
 2ρ sin φtgβtgγ þ sin2φ  0, or (ρtgβtgγ  sin φ)2  0, which is evident.
Note that the equality holds true, if and only if the straight line l passes through
the orthocenter of triangle ABC.
7.1.107. (a) Let max(∠A, ∠B, ∠C) ¼ ∠A ¼ α.
If α  120 , then according to problem 1.2.8, we have that MA þ MB þ
MC  b þ c.
We need to prove that b þ c  pffiffiffiffiffiffiffiffiffiffiffiffiffiffiffiffiffiffiffiffiffiffiffiffi
3abc
or
2 2 a b þb c þa c
2 2 2 2

   
a2 ðb þ cÞ2 b2 þ c2  9b2 c2 þ ðb þ cÞ2 b2 c2  0 ð7:45Þ

Since 0  a2  (b þ c)2, then it is sufficient to prove the inequality (7.6) for a2


¼ 0 and a2 ¼ (b þ c)2.
If a2 ¼ 0, then the proof is evident in (7.6), and if a2 ¼ (b þ c)2, then the
inequality (7.6) is obtained from the inequalities b2 þ c2  2bc and (b þ c)2  4bc.
If α < 120 , then there exists a point T inside of triangle ABC, such that
∠ATB ¼ ∠BTC ¼ ∠ATC ¼ 120 (seeqffiffiffiffiffiffiffiffiffiffiffiffiffiffiffiffiffiffiffiffiffiffiffiffiffiffiffiffiffiffiffiffiffiffiffiffiffiffiffiffiffiffiffiffiffiffiffiffiffiffiffiffiffiffi
the proof of problem 1.2.8), and

MA þ MB þ MC  TA þ TB þ TC ¼ b2 þ c2  2bc cos ð60 þ αÞ.
We need to prove that TA þ TB þ TC  pffiffiffiffiffiffiffiffiffiffiffiffiffiffiffiffiffiffiffiffiffiffiffiffi
3abc
or
2 2 a b þb c þc a
2 2 2 2

1 1 1 9
þ 2þ 2  2   0: ð7:46Þ
b 2 c b þ c  2bc cos α b þ c  2bc cos ð60 þ αÞ
2 2

Consider the function f ðxÞ ¼ qx2 þ x2q1cos α  x2q cos9 60 þα on [2q; 1), where
ð Þ
q ¼ bc. Since f 0 ðxÞ ¼ ðx2q cos αqÞðx2q cos αþqÞ
þ 9
 2 and x  2q  q þ
q2 ðx2q cos αÞ2 ðx2q cos ð60 þαÞÞ
2q cos α (60  α < 120 ), then for x  2q, we have that f0 (x) > 0.
Hence, it follows that f(x)  f(2q).
We need to prove that f(2q)  0 or 4 þ 1 cos 1
α  1 cos ð60 þαÞ.
9

The last inequality can be reduced to the inequality.


(sin(30 þ α)  1)(4 sin (30 þ α)  1)  0. Note that, this inequality holds true,
as sin(30 þ α)  1 and 30 þ α 2 [90 ; 150 ). Hence, 4 sin (30 þ α) > 2.
Since b2 þ c2  2bc ¼ 2q, then f(b2 þ c2)  f(2q)  0. Hence, f(b2 þ c2)  0.
Thus, it follows the inequality (7.10).
7.2 Solutions 399

(b) According to problem 7.1.107a, we have that MA þ MB þ MC  pffiffiffiffiffiffiffiffiffiffiffiffiffiffiffiffiffiffiffiffiffiffiffiffi


3abc
.
2 2 a b þb c þc a
2 2 2 2

We need to prove that pffiffiffiffiffiffiffiffiffiffiffiffiffiffiffiffiffiffiffiffiffiffiffiffi


3abc
 6r, or
2 2 a b þb c þc a
2 2 2 2

1 1 1 1
2
þ 2þ 2  2,
a b c 4r
h2a h2b h2c 1
þ þ  :
a2 ha b hb c2 h2c 4r 2
2 2 2

S2
h2a þ h2b þ h2c  ¼ p2 :
r2

The last inequality holds true, as


h2a þ h2b þ h2c  l2a þ l2b þ l2c  p2 (see the problem 7.1.37h).
This ends the proof.

7.1.108. Assume that ∠A > ∠B > ∠C. Let us prove that ∠OIH > 180  12 ∠A:
 
As ∠A < 90 , then 180  12 ∠A > 135 and therefore∠OIH > 135 . Let us draw
altitudes AA1 and CC1 in triangle ABC (Figure 7.77).
As ∠OAB ¼ 90  ∠C > 90  ∠B ¼ ∠BAA1 and ∠OCA ¼ 90  ∠B > 90 
∠A ¼ ∠ACC1, then point O is inside of triangle A1HC. Note that ∠BAA1 ¼
∠OAC ¼ 90  ∠B and ∠ACC1 ¼ ∠OCB ¼ 90  ∠A.
Hence, ∠HAI ¼ ∠IAO and ∠HCI ¼ ∠ICO. Let lines AI and CI intersect seg-
ment HO at points E and F, respectively. Let C2 be the midpoint of side AB and K be
intersection point of lines OC2 and AI.

Figure 7.77
400 7 Miscellaneous Inequalities

EO ¼ AO < AO ¼ CO ¼ FO, and lines CI, OC2 intersect on the


As HE AH HC HC HF

circumcircle of triangle ABC, then point I is on segment EK.


Note that HC < 2  CO, therefore HC
CO ¼ FO < 2 ¼ GO. Thus, it follows that point
HF HG

G is on segment FO.
 
(a) We have that ∠GIH > ∠FIE ¼ ∠AIC ¼ 90 þ 12 ∠B > 112, 5 , ∠PNA ¼
[X1 B1 [B1 X 
2  [X21 B1 < 30 , ∠B > 45 .
 
(b) If ∠B  60 ,then ∠AIC ¼ 90 þ 12 ∠B  180  ∠B ¼ ∠AHC: Note that point
I is inside of the circle passing through points A, H, C. Then, it follows that
∠AIH > ∠ACH ¼ 90  ∠A and ∠OIH ¼ ∠OIA þ ∠AIH > ∠OKA þ ∠AIH
 
¼ 90 þ 12 ∠A þ ∠AIH > 180  12 ∠A:
 
We deduce that ∠OIH > 180  12 ∠A: If ∠B < 60 ,then ∠AIC ¼ 90 þ
2 ∠B > 2∠B ¼ ∠AOC: Hence, point I is inside of the circle passing through points
1

A, O, C. Note that ∠IOA < 12 ∠C: Then,

  1
∠OIA ¼ 180  ∠IAO  ∠IOA ¼ 180  ð∠B  ∠CÞ  ∠IOA >
2
 1 1  1
> 180  ð∠B  ∠CÞ  ∠C ¼ 180  ∠B:
2 2 2
  
Thus, ∠OIA > 180  12 ∠B and ∠OIH > ∠OIA > 180  12 ∠B > 180  12∠A:

We obtain that, ∠OIH > 180  12 ∠A:
Remark This estimate is an exact one, as for ∠A ¼ 90 , ∠B ! 90 we have that
 
∠IHO ¼ 12 ð∠B  ∠CÞ ! 45 , ∠IOH ! 0 and thus ∠OIH ! 135 .
7.1.109. (a) Note that AC1 ¼ AB1, BC1 ¼ BA1, CA1 ¼ CB1, therefore
AC1
BC1  CA 1

BA1 CB1
AB1 ¼ 1: According to Ceva’s theorem, it follows that segments AA1,
BB1 and CC1 intersect at one point. Point M is called the Gergonne Point.
Let AC1 ¼ AB1 ¼ m, BC1 ¼ BA1 ¼ n, CA1 ¼ CB1 ¼ k.
According to Stewart’s theorem, from triangle ABC, we obtain that
2
Þ2 nnkðnþkÞ
AA1 2 ¼ ðmþnÞ kþðmþk nþk :
Note that BC1 þ CB1 ¼ SMBC þ SSMBC
AC1 AB1 SMAC MAB
¼ SSMAMACC þS
þSMA B ¼ SMA C ¼ MA1 ,
MAB SMAC MA
as SSMA
MAC
C
¼
1 1 1 1
SMAB
SMA1 B ¼ MA
MA
1
:
MA1 ¼ n þ k , therefore MA ¼ mkþmnþnk AA1 : Hence, we deduce
MA m m mkþmn
We obtain that
m3 ðkþnÞðmkþmnþ4nkÞ
that MA ¼ 2
ðmkþmnþnkÞ2
:
One needs to prove that MA  43 AB1 2 , or
2
mðkþnÞðmkþmnþ4nkÞ
ðmkþmnþnkÞ2
 43 :
The last inequality holds true, as it is equivalent to (mn þ mk  2nk)2  0.
7.2 Solutions 401

Figure 7.78

(b) Let point M be the Gergonne Point of triangle BDF, then from problem
7.1.109а, it follows BM  p1ffiffi3 ðBF þ BD  DFÞ, DM  p1ffiffi3 ðDF þ BD  BFÞ
и FM  p1ffiffi ðBF þ DF  BDÞ:
3

Note that

AC  BM CE  DM
S ¼ SABCM þ SCDEM þ SAFEM  þ
2 2
AE  FM 1
þ  pffiffiffiACðBF þ BD  DFÞ
2 2 3
1 1
þ pffiffiffiCEðDF þ BD  BFÞ þ pffiffiffiAEðBF þ DF  BDÞ:
2 3 2 3

Thus, it follows pthat


ffiffiffi ACðBF þ BD  DFÞ þ CEðDF þ BD  BFÞþ
AEðBF þ DF  BDÞ  2 3S:
(c) Let A, B, C be the centers of those circles (Figure 7.78).
As AY ¼ AZ, BX ¼ BZ, CX ¼ CY, then we obtain that AY ¼ AZ ¼ ABþACBC 2 , BX
¼ BZ ¼ BCþABAC
2 , CX ¼ CY ¼ BCþACAB
2 : Thus, it follows that the incircle of
triangle ABC touches sides BC, AC, AB at points X, Y, Z, respectively.
Let point M be the Gergonne Point of triangle ABC, then from problem 7.1.109а,
it follows that the circles with centers A, B, C and radiuses p2ffiffi3 AY, p2ffiffi3 BZ, p2ffiffi3 CX cover
triangles MZY, MXZ, MXY, respectively.
Therefore, these circles cover triangle XYZ too (point M is inside of triangle
XYZ).
(d) Let N be the Gergonne Point of triangle ABC, then from problem 7.1.115а, it
follows that NA  p2ffiffi3 ðp  aÞ, NB  p2ffiffi3 ðp  bÞ, NC  p2ffiffi3 ðp  cÞ:

p2 ðpaÞМA ffi
p2 ðpbÞМB
According to problem 7.1.47а, we have that 3
bc þ ac þ
3


p2 ðpcÞМC
3
 MANA
bc þ ac þ ab  1:
MBNB MCNC
Therefore, pc þ pa þ pb
MAþMB MBþMC MCþMA
pffiffiffi
ab
 2 3  Rr :
402 7 Miscellaneous Inequalities

7.1.110. (a) Let Pn þ 1  P1, Xn þ 1  X1, An þ 1  A1.


Note that, if point M is inside of polygon X1X2 . . . Xn, then we have that

ð∠X1 A2 X2 þ ∠X1 MX2 Þ þ ::: þ ð∠Xn A1 X1 þ ∠Xn MX1 Þ ¼


¼ ð∠X1 A2 X2 þ ::: þ ∠Xn A1 X1 Þ þ ð∠X1 MX2 þ ::: þ ∠Xn MX1 Þ
¼ ðn  2Þπ þ 2π ¼ nπ:

Therefore, the exists a number i, such that ∠Xi Aiþ1 Xiþ1 þ ∠Xi MXiþ1  πn n ¼ π:
Hence, point M is inside of circle passing through points Xi, Ai þ 1, Xi þ 1.
Thus, it follows that there exists a number i, such that point P is inside of circle
passing through points Xi, Ai þ 1, Xi þ 1.
Then, we have that

Xi Xiþ1
PAiþ1  ,
2 sin ∠Xi Aiþ1 Xiþ1

and

Pi Piþ1
PAiþ1 ¼ , ∠Xi Aiþ1 Xiþ1 ¼ ∠Pi Aiþ1 Piþ1 :
2 sin ∠Pi Aiþ1 Piþ1

Therefore, PiPi þ 1  XiXi þ n


1. o
Hence, we obtain that max XP11 XP22 ; XP22 XP33 ; :::; XPnn XP11  1:

(b) Let ∠A1 ¼ α1, ∠A2 ¼ α2, . . . , ∠An ¼ αn and segment B10 B20 be the projection
of segment B1B2 on the line perpendicular to the bisector of angle A1A2A3.
We have that,
  α    α 
B1 B2  B1 0 B2 0 ¼ B1 0 A2 þ A2 B2 0 ¼ B1 A2 cos 90 
2 2
þ A2 B2 cos 90  ¼
2 2
α 2 B 1 A2 þ A2 B2
¼ ðB1 A2 þ A2 B2 Þsin > sin α2
2 2
B1 A2 þ A2 B 2
¼ A1 A3 :
4R
þA2 B2
Therefore BA11 BA23 > B1 A24R , where R is the circumradius of polygon A1A2 . . . An.
þA3 B3 þA1 B1
Similarly, we deduce that A2 A4 > B2 A34R
B2 B3
, :::, BAnn BA12 > Bn A14R :
Summing up these inequalities, we obtain that

B1 B2 B 2 B3 Bn B1 B1 A 2 þ A2 B 2 B2 A3 þ A3 B3 Bn A1 þ A1 B1
þ þ ::: þ > þ þ ::: þ ¼
A1 A3 A 2 A4 An A2 4R 4R 4R
A1 A2 þ A2 A3 þ ::: þ An A1
¼ :
4R

Hence, we obtain that BA11 BA23 þ BA22 BA34 þ ::: þ BAnn BA12 > A1 A2 þA2 A4R
3 þ:::þAn A1
:
7.2 Solutions 403

Figure 7.79

Figure 7.80

According to problem 1.1.7b and problem 1.1.8b, one can prove that
A1A2 þ A2A3 þ . . . þ AnA1 > 4R, therefore BA11 BA23 þ BA22 BA34 þ ::: þ BAnn BA12 > 1:

7.1.111. Let points I and N be on the different sides of line PM. Without loss of
generality, one can assume that point N is on segment B1C, where B1 is the point at
which the incircle of triangle ABC touches side AC.
The following cases (Figures 7.79 and 7.80) are possible.
(a) Point P is on segment C1B (Figure 7.79), where C1 is the point at which the
incircle of triangle ABC touches side AB.

Note that 60 ¼ ∠MPN ¼ [XY[X 2
1 Y1
, therefore [XY  120 .
[X1 B1 [B1 X 
We have that ∠PNA ¼ 2  [X21 B1 < 30 , as [Y1BY < 180 .

Similarly, we deduce that ∠APN < 30 .
Thus, it follows that ∠A ¼ 180  ∠PNA  ∠APN > 120 .
(b) Point P is on segment AC1 (Figure 7.80).
We have that ∠ANP < 30 , note that ∠NPI > 60 and ∠IPB > ∠NPI > 60 .
Thus, it follows that ∠BPN > 120 . Therefore ∠APN < 60 , hence
∠A ¼ 180  ∠APN  ∠ANP > 90 .
Let point K is on segment AB1 and the incircle of triangle ABC touches segment

AB1. Note that ∠IAC1 ¼ 12 ∠BAC > 45 , thus ∠IAC1 > ∠AIC1. Therefore r ¼ IC1
> AC1 ¼ APþPKþAK
2 > PK, then r > PK.
Let us draw, through point I , a line parallel to line PM, and note that PM > 2r.
Using the triangle inequality, we obtain that
404 7 Miscellaneous Inequalities

KN  PN  PK ¼ PM  PK > 2r  PK > PK.


Hence, KN > PK.
We deduce that, ∠KPN > ∠PNK. Thus, it follows that

∠KPB ¼ 2∠KPI > 120 þ 2∠KPN:

Therefore, ∠APN < 60  ∠KPN < 60  ∠PNK.


Hence, we obtain that ∠A ¼ 180  ∠PNA  ∠APN > 120 .
This ends the proof.
7.1.112. Let a triangle with area S and perimeter P contain these circles with
radius R.
At first, let us prove the following lemmas.
Lemma 1 There exists a triangle with area S1and perimeter P1, such that any side
of this triangle touches at least one of the circles and S  S1, P  P1.
Indeed, for this one needs to translate (moving parallel) the lines containing the
sides of triangle ABC with area S and perimeter P, such that any of them touches at
least one of the circles (Figure 7.81). Note that triangle A1B1C1 is the required
triangle, as S1 ¼ SA1 B1 C1  S and P1  P.
Lemma 2 There exists a triangle with area S2and perimeter P2, such that one of its
sides touches both circles and any of the other two sides touches at least one of the
circles. Moreover, S2  S1 and P2  P1.
Indeed, according to lemma 1 we have that any of the sides of triangle A1B1C1
touches at least one of the circles.
We proceed the proof by contradiction arguement. Assume that none of the sides
of triangle A1B1C1 touches simultaneously both circles.
Let centers O1 and O2 of those circles be connected by a segment. Now, let us
rotate this fixed figure such that one of the sides, for example A1C1 touches both
circles (Figure 7.82).
We have that ∠O1 O2 O2 0 ¼ ∠O1 O2 0 O2 < π2  ∠O1 NC1 ¼ ∠O1 O2 C2 , then
∠O2 0 O2 M > ∠O2 0 O2 N > π2 : Therefore, O20 K > O2M ¼ R. This means that the
circle with center O20 is inside of triangle A1B1C1.
In order to end the proof of lemma 1, let us note that one can translate (moving
parallel) side B1C1 and decrease the area and the perimeter of the triangle.

Figure 7.81
7.2 Solutions 405

Figure 7.82

Figure 7.83

Let S2 ¼ (A2B2C2), P2 ¼ A2B2 þ B2C2 þ A2C2, then S1  S2 and P1  P2.


Lemma 3 There exists an equilateral triangle with area S3and perimeter P3, such
that its base touches both circles and any of the other two sides touches at least one
of the circles. Moreover, S2  S3 and P2  P3.
Indeed, let line l be the common tangent of those circles (Figure 7.83).
Denote by A10 , B20 and C20 , respectively, the symmetric points of points A1, B2
and C2 with respect to line l respectively. Let points A0, B0, C0 be the midpoints of
segments C20 A1, B2B20 and A10 C2,respectively. As C20 A1 ¼ A10 C2, then
A0C0 ¼ A1C2.
We have that S2 ¼ (A0B0C0) ¼ S0 and p2  p0, where p0 is the perimeter of
triangle A0B0C0. Moreover, it is obvious that the circles are in triangle A0B0C0
(Figure 7.83).
Consider Figure 7.84. Let area of equilateral triangle A3B0C3 is equal to S3, аnd
its perimeter is equal to p3.
Then, we have that S2 ¼ S0  S3 and p2  p0  p3. Therefore, S2  S3 and p2  p3.
This ends the proof of lemma 1.
Lemma 4 Let an isosceles triangle, with angles 45 , 45 , 90 and area S4, be such
that its bases touches both circles and any of the other two sides touches at least one
of the circles. Then, S3  S4.
Let ∠A3C3B0 ¼ 2α, note that A3C3 ¼ 2R(1 þ ctgα), B0H ¼ R(1 þ ctg(45  α)),
where B0H is the altitude of triangle A3C3B0.
406 7 Miscellaneous Inequalities

Figure 7.84

Therefore, S3 ¼ 2R ð1þctgα Þctgα


2

ctgα1 : Thus, it follows that the equation 2R2ctg2α þ


(2R  S3)ctgα þ S3 ¼ 0 must have a solution. We deduce that D ¼ (2R2  S3)2
2

p8R
ffiffiffi 2S3  0. On the
2
other
p ffiffi
ffi hand, S3 > 2πR2. Hence, we obtain that S3  6R2 þ
2R2
4 2R : If S3 ¼ 6R2 þ 4 2R2 , then D ¼ 0. Thus, it follows that ctgα ¼ S34R
pffiffiffi 2

¼ 1 þ 2: Therefore, α ¼ π8 :
We have that S  S1  S2  S3  S4,hence S  S4. This means that the smallest
ffiffiffi of a triangle containing two tangent circles with radius R is
possible value ofparea
equal to 6R2 þ 4 2R2 :
2
α
We have that p3 ¼ 2Rð1 þ ctgαÞ þ 2Rð1 þ ctgαÞ  cos1 2α ¼ 4Rctg ctgα1 , therefore the
equation 4Rctg α  p3ctgα þ p3 ¼ 0 must have a solution. Thus, it follows that
2

D ¼ p32  16p3R  0. We deduce that, p3  16R.


p3
Moreover, p3 ¼ 16R, if ctgα ¼ 8R ¼ 2:
We obtain that, the smallest possible value of perimeter of a triangle containing
two tangent circles with radiuses R is equal to 16R.
7.1.113. (a) At first, let us prove the following lemma.
Lemma Given a convex hexagon ABCDEF, such that ABjjDE, BCjjEF, CD||FA.
Given that the distance between lines AB,DE is equal to the distance between lines
BC,EF and is equal to the distance between lines CD, FA. Prove that AD2 ¼ (AB þ
DE)(FA þ CD).
Note that point D is equidistant from lines AB and FA.
Then, we have that ∠BAD ¼ ∠FAD.
As AB||DE,CD||FA, then ∠BAD ¼ ∠FAD ¼ ∠ADE ¼ ∠ADC.
Similarly, we obtain that ∠ABE ¼ ∠CBE ¼ ∠FEB ¼ ∠DEB and
∠BCF ¼ ∠DCF ¼ ∠CFA ¼ ∠CFE.
Let ∠BAD ¼ α, ∠ABE ¼ β and ∠BCF ¼ γ. As the sum of all interior angles of a
convex hexagon is equal to 720 , then it follows that α þ β þ γ ¼ 180 .
Let us consider parallelogram ADEK.
We have that BK ¼ BA þ AK ¼ BA þ DE, KE ¼ AD, ∠AKE ¼ ∠BAD ¼ α,
∠KBE ¼ ∠ABE ¼ β, ∠BEK ¼ γ.
7.3 Problems for Self-Study 407

Then, the triangles with sides AD, AB þ DE, BE and FA þ CD, AD, CF are sim-
ilar. Тhus, it follows that AD2 ¼ (AB þ DE)(FA þ CD).pffiffiffiffiffiffiffiffiffiffiffiffiffiffiffiffiffiffiffiffiffiffiffiffiffiffiffiffiffiffiffiffiffiffiffiffiffiffiffiffiffiffiffiffi
According to the lemma, it follows that AD ¼ ðAB þ DEÞðFA þ CDÞ 
ABþDEþFAþCD
2 .
Similarly, we deduce that BE  ABþBCþEFþED
2 , CF  BCþCDþEFþFA
2 .
Summing up the last three inequalities, we obtain that

AD þ BE þ CF  AB þ BC þ CD þ DF þ EF þ FA:

(b) Let diagonals AD, BE and CF intersect at point M.


Note that ∠MFE ¼ ∠CFE ¼ ∠CAE ¼ 60 .
Similarly, we deduce that ∠MDE ¼ 60 , ∠CDM ¼ 60 , ∠CBM ¼ 60 .
Let ∠BMC ¼ α, ∠CMD ¼ β, ∠DME ¼ γ, then ∠FME ¼ ∠BMC ¼ α.
γ ¼ sin 60 ¼ sin α :
DE ME EF
According to the law of sines, we have that sin
sin γ
EF ¼ sin α :
Thus, it follows that DE
sin β
Similarly, we deduce that BC ¼ sin
CD
α:
As α þ β þ γ ¼ π, then by triangle  inequality,  we obtain that
jsin β  sin γj < sin α. Therefore, it follows that DE
EF  CD
BC < 1:
Remark Let ABCDEF be a cyclic  hexagon. Given
 that diagonals AD, BE and CF
intersect at one point. Prove that DEAE
EF  CDAC
BC < CE:

7.3 Problems for Self-Study

7.1.114. Prove the following inequalities for triangle ABC.


1 1 1 4
(a) 2
þ 2
þ 2
 ,
ðma þ mb  mc Þ ðma  mb þ mc Þ ðma þ mb þ mc Þ 3R2
m2a m2b m2c 9
(b) þ þ 2  ,
a2 b2 c 4
9 ma mb þ mb mc þ mc ma 5
(c) < < ,
20 ab þ bc þ ac 4
a2 b 2
c2 pffiffiffi
(d) þ þ  3 3R,
a þ b þ c a  b þ c a þ b  c
ra rb rc R
(e) þ þ   1.
rb þ rc rc þ ra ra þ rb r
7.1.115. The parallelogram ABCD lies in the base of the pyramid SABCD. A plane
passing through vertex A and the midpoint K of the edge SC intersects the edges
SB and SD at points M and N. Prove that, 13  VV1  38, where V, V1 are the volumes
of the pyramids SABCD, SAMKN, respectively.
408 7 Miscellaneous Inequalities

7.1.116. Let the lateral edge of a regular triangular pyramid has a length a. Prove
3
that the volume of such a pyramid does not exceed a6 .
7.1.117. Prove that the ratio of the volume V of the regular n-gon pyramid to the
volume V1 of its insphere, satisfies the following inequality:

V tg ðπ=nÞ
 :
2V 1 π=n
pffiffi
r2
7.1.118. Prove that the inequality m2a þm2b
 358
holds true for the right-angled
triangle, where ma, mb are the lengths of the medians drawn to the legs, r is the
circumradius of the triangle.
rffiffiffiffiffiffiffiffiffiffiffiffiffiffiffiffiffiffiffiffiffiffiffiffiffiffiffiffiffiffiffiffiffiffiffiffiffiffiffi
 
7.1.119. Prove the inequality 2SR2  13 h1a þ h1b þ h1c for triangle ABC, where S is the
3

area of the triangle, R is the circumradius, ha, hb, hc are its altitudes.
7.1.120. Given a point M inside of angle AOB. Let M1 and M2 be its projections on
the sides of the angle. Prove that SOM1 MM2  12 OM2 sin ∠AOB.
7.1.121. Let a, b, c be the sides of triangle ABC, r, and R be its inradius and
circumradius, respectively. Prove that

1 1 1 1 1 2 1 1 1 1
 þ þ  2þ 2þ 2  2:
2Rr 3 a b c a b c 4r

7.1.122. Given a point O inside of triangle ABC, x, y, z are the distances of the points
O to the sides of the triangle. Theq circumradius
ffiffiffi of triangle ABC is R.
pffiffiffi pffiffiffi pffiffi
Prove that x þ y þ z  3 2 . R

  
7.1.123. Prove that m2a þ m2b þ m2c h2a þ h2b þ h2c  27S2 , where ma, mb, mc are
the medians, ha, hb, hc are the altitudes, and S is the area of triangle ABC.
7.1.124. The median BM of triangle ABC forms an angle φ with side BC . Prove that
pffiffi
2 23 cos φ
ctg ∠A  sin φ .

7.1.125. Let a radius drawn in the half-circle with the diameter AB divide it into two
sectors. Given that circles are inscribed into each of these sectors and M, N arepffiffithe
tangential points of these circles with the diameter AB, prove that MN  88 2 R,
where AB ¼ 2R.
7.1.126. Given a triangle ABC, prove that hαa þ hβb þ hγc  32, where ha, hb, hc are the
altitudes of ABC, α, β, γ are the distances of the feet of the bisectors of angles A, B, C
from the sides of the triangle.
7.3 Problems for Self-Study 409

7.1.127. Lateral edges of a triangular pyramid are mutually perpendicular.Prove


2
that S1 þ S2 þ S3  9h2 , where S1, S2, S3 are the areas of lateral facets and h is the
altitude of the pyramid.
7.1.128. Given triangle P1P2P3 and a point P inside of the triangle. The straight
lines P1P, P2P, P3P intersect the opposite sides at points Q1, Q2, Q3 respectively.
P1 P P2 P P3 P
Prove that at least one of the numbers PQ , PQ , PQ is less than or is equal to
1 2 3
2 and at least one of these numbers is greater than or equal to 2.
7.1.129. Prove that r 2 þ r 2a þ r 2b þ r 2c > 4S, where r, ra, rb, rc are, respectively, the
radiuses of the incircle and excircles of triangle ABC, S is the area of the triangle.
7.1.130. For triangle ABC prove that
rffiffiffiffiffiffi
1 1 1 9 R
þ þ  :
α β γ π 2r

7.1.131. In a rectangular sheet of paper, n rectangular holes were cut, and sides of
the holes are parallel to the edges of a sheet. What is the least guaranteed number of
the rectangular parts, such that this sheet with the holes is possible to cut? (Show
that, in all cases it is possible to cut the sheet into the number of parts found by you,
but in some cases, it is impossible to cut it into smaller number of parts).
Hint The proof by mathematical induction, if n rectangular holes were cut in a
polygon, such that all internal angles are equal to either 90 or 270 , with sides of
the holes parallel to sides of that polygon, then that sheet with the holes can be cut
into 3n þ k þ 1 rectangular parts, where k is the number of internal angles equal to
270 . Separately, consider the case, if n ¼ 0. To prove these statements one has to
consider the rectangular, with the left vertical side being the leftmost and that angle
of 270 from the angles of the polygon, is the one with the leftmost vertex.
7.1.132. Triangle ABC is cut into finite number of triangles, such that in each
triangle there is an angle greater than 120 . Prove that max(∠A, ∠B, ∠C) > 120 .
7.1.133. Is it possible to cut any convex polygon into a finite number of non-convex
quadrilaterals?
7.1.134. What is the minimal number of non-overlapping tetrahedrons that a cube
can be divided into?
7.1.135. Given a point M inside of the square ABCD, prove that
∠MAB þ ∠MBC þ ∠MCD þ ∠MDA > 3π 4.

Hint See problem 7.1.14d.


7.1.136. Given on a plane n vectors the length of each being equal to 1. The sum of
all these vectors is a zero-vector. Prove that these vectors can be enumerated in such
a way, that for all k ¼ 1, 2, . . . , n the following
pffiffi
condition holds true: the sum of
initial k vectors has a length not greater than 25.
410 7 Miscellaneous Inequalities

7.1.137. Given several acute triangles. Let a new triangle be constructed, from the
sides of the given triangles, by the following rule: the smallest side of it is equal to
the sum of the smallest sides of the triangles, the length of the middle side is equal
to the sum of the middle sides of the triangles, and the largest side is equal to the
sum of the largest sides of the triangles. Prove
pffiffiffi that the cosine of the largest angle of
the obtained triangle is greater than 1  2.
Hint Consider the triangles with sides ai  bi  ci, i ¼ 1, 2, . . . , n. Then, we have
pffiffiffi
to prove that ðb1  a1 þ ::: þ bn  an Þ2 þ 2 2ða1 þ ::: þ an Þðb1 þ ::: þ bn Þ 
ðc1 þ ::: þ cn Þ2 .
rffiffiffiffiffiffiffiffiffiffiffiffiffiffiffiffiffiffiffiffiffiffiffiffiffiffiffiffiffiffiffiffiffiffiffiffiffiffiffi
  pffiffiffi   2  
Prove that ðbi  ai Þ bj  aj þ 2 ai bj þ aj bi  bi þ a2i b2j þ a2j .

7.1.138. Given a point M inside of triangle ABC (see the notations of problem
7.1.76), prove that
(a) ha
daþ dhbb þ dhcc  9,
(b) hahbhc  27dadbdc,
(c) (ha  da)(hb  db)(hc  dc)  8dadbdc,
(d) aRa  bdb þ cdc,
(e) aRa  cdb þ bdc,
(f) aRa Rb Rc  bdb R2b þ cd c R2c ,
(g) Ra Rb Rc  2r R
ðda þ d b Þðdb þ d c Þðd c þ d a Þ,
(h) Ra Rb þ Rb Rc þ Rc Ra  2R r ðd a d b þ d b d c þ d c d a Þ,
2
þ dbc da þ dca db  4RðRRa þR b þRc Þ
2
a2 2
(i) db dc a Rb Rc
,
(j) aRa þ bRb þ cRc  2(ada þ bdb þ cdc),
(k) aRada þ bRbdb þ cRcdc   2(adbdc þ bdadc þ cdadb),
(l) ða þ b þ cÞRa Rb Rc  2 ada R2a þ bd b R2b þ cd c R2c ,
(m) R2 R2 R2  ðd Ra þ db Rb Þðdb Rb þ d c Rc Þðdc Rc þ da Ra Þ,
paffiffiffiffiffib c pffiffiffiffiffi a p ffiffiffiffiffi pffiffiffipffiffiffiffiffi pffiffiffiffiffi pffiffiffiffiffi
(n) Ra þ Rb þ R c  2 d a þ d b þ d c ,
 
(o) R2a þ R2b þ R2c > 2 d2a þ d2b þ d2c ,
a 2 þ b 2 þ c2  R ,
Ra Rb Rc 1
(p)
R2 R2 R2
(q) bca þ cab þ abc  1,
(r) RR0  RaRbRc, where R0 is the circumradius of the triangle with sides aRa, bRb,
cRc.
Hint See the proof of problems 7.1.47, 7.1.76, 7.1.79, and 7.1.97.
7.1.139. Prove that, for triangle ABC, the inequality m2a  14 ð4b þ 4c  5aÞ a holds
true, where BC ¼ a, AC ¼ b, AB ¼ c, ma is the median drawn to side a. For which
triangle the equality holds true?
7.1.140. Prove the inequality r  161
ðh1 þ h2 þ h3 þ h4 Þ for a triangular pyramid,
where r is the inradius and hi are the altitudes of the pyramid (i ¼ 1, 2, 3, 4).
7.3 Problems for Self-Study 411

7.1.141. The angle is divided by the rays drawn from its vertex into 2n þ 1 equal
angles. These angles cut on some straight line 2n þ 1 segments. Prove that the sum
of the lengths of the first, third, . . ., (2n þ 1)-th segments is greater than the sum of
the remaining segments.
Hint Draw a perpendicular from the vertex of the angle to the line.
7.1.142. For triangle ABC (see the notations of problem 7.1.76), prove that
  
α β γ da þ db 2 db þ dc 2 dc þ da 2
R2a sin 2 þ R2b sin 2 þ R2c sin 2  þ þ :
2 2 2 2 2 2

b þd c α
Hint Prove that Ra  dsin α cos 2.

7.1.143. For an acute triangle ABC (see the notations of problem 7.1.76), prove that
d2a þ d2b þ d2c  R2a sin 2α2 þ R2b sin 2 β2 þ R2c sin 2 2γ .
d2b þd2c 2
xþsin 2 ðαxÞ
Hint Prove that 2 ¼ R2a sin 2  R2a sin 2α2.
7.1.144. Given a regular n-gon A1A2 . . . An inscribed into a unit circle and a point
π
M on the minor arc A1An, prove that 2ctg 2n  MA1 þ MA2 þ ::: þ MAn  sin2 π .
2n

Hint Let M0 be the midpoint of the minor arc A1An and M is on the minor arc A1M0,
then according to problem 1.1.6b, we have that A1Ak þ 1 þ A1An  k 
Ak þ 1M þ An  kM  Ak þ 1M0 þ An  kM0. Consequently, it follows that

A1 A2 þ A1 A3 þ ::: þ A1 An  MA1 þ MA2 þ ::: þ MAn


 M0 A1 þ M0 A2 þ ::: þ M0 An :

7.1.145. A quadrilateral has a circumcribed


pffiffiffi circle with radius R and an inscribed
circle with radius r. Prove that R  2r.
pffiffiffiffiffiffiffiffiffiffi
Hint Prove that r 2 ¼ ðaþcabcd
Þðbþd Þ  4 abcd and see problem 7.1.93d).
1
Chapter 8
Some Applications of Geometric Inequalities

This chapter consists of two sections, that is, Sections 8.1 and 8.2.
Section 8.1 is devoted to geometric problems that can be solved applying
geometric inequalities. In the proofs of many problems of this chapter are used
the following statements about a triangle.
(a) Greater side of a triangle is opposite the greater angle.
(b) Greater angle of a triangle is opposite the greater side.
In some problems of this paragraph, one needs to understand when does the
equality holds true in the corresponding geometric inequality.
In many proofs of Section 8.1 is used the following statement: if points M and
N are on the same side of segment AB and point M is inside of triangle ANB, then
∠AMB < ∠ ANB.
In Section 8.2 is shown how one can prove algebraic inequalities using geomet-
ric inequalities. In this paragraph are selected such inequalities that can be proved
by comparing areas (volumes) of two geometric figures.
In order to prove some inequalities in Section 8.2 is used very useful substitution
x ¼ tgα, y ¼ tgβ, z ¼ tgγ, where α, β, γ are the angles of some triangle (see problem
8.2.9). On the other hand, in order to deal with many geometric inequalities and to
transform them into algebraic inequalities, often is used the method of coordinates.
Some problems in this chapter were inspired by [1, 13]. Nevertheless, even for
these problems, the authors have mostly provided their own solutions.

8.1 Application of Geometric Inequalities for Solving


Geometric problems

8.1.1. Given a point M inside of square ABCD, such that ∠MBC ¼ ∠ MCB ¼ 15 .
Prove that triangle AMD is equilateral.

© Springer International Publishing AG 2017 413


H. Sedrakyan, N. Sedrakyan, Geometric Inequalities, Problem Books
in Mathematics, DOI 10.1007/978-3-319-55080-0_8
414 8 Some Applications of Geometric Inequalities

8.1.2. Given a point D on side AC of triangle ABC, such that ∠ABD ¼ 90 ,
pffiffiffi
∠DBC ¼ 30 . Find segment AD, if AB ¼ 3, CD ¼ 1.
8.1.3. Given points E and F on sides AB and BC of triangle ABC, respectively, such
that AE ¼ EF ¼ BF. Find angle AEC, if ∠A ¼ 75 , ∠B ¼ 10 .
8.1.4. Given that in a convex quadrilateral AB ¼ BC and ∠ABD ¼ 65 ,
∠CBD ¼ 35 , ∠ADC ¼ 130 . Find the unknown angles of quadrilateral ABCD.
8.1.5. Given a triangle ABC, such that ∠B ¼ ∠ A + 30 and AB ¼ 2BC. Find the
angles of triangle ABC.
8.1.6. Given a point M inside of triangle ABC, so that ∠MAB ¼ ∠ MCB ¼ 20 ,
∠MAC ¼ 80 , ∠MCA ¼ 30 . Find the value of angle ∠AMB.
8.1.7. Given a point M inside of triangle ABC, so that ∠MAB ¼ ∠ MBC ¼ 20 ,
∠MBA ¼ 10 , ∠MAC ¼ 80 . Find the value of angle ∠AMC.
8.1.8. The circumcenter of triangle ABC is inside of the incircle of that triangle.
Find the angles of triangle ABC, if la ¼ R.
8.1.9. In triangle ABC, the bisectors of angles A and C intersect with the median BM
at points E and F, respectively. Given that AE ¼ CF. Prove that AB ¼ BC.
8.1.10. Prove that a convex quadrilateral ABCD is circumscribed, if and only if
quadrilateral MNPQ is inscribed, where M, N and P, Q are the tangential points of the
incircles of triangles ABC and ACD with sides AB, BC and CD, AD, respectively.
8.1.11. Let in a convex quadrilateral ABCD diagonals AC and BD be perpendicular.
Given that the opposite sides AB and DC are not parallel to each other. Perpendic-
ular bisectors of sides AB and DC intersect at a point P inside of ABCD. Prove that a
circle can be circumscribed around quadrilateral ABCD, if and only if the areas of
triangles ABP and CDP are equal.
8.1.12. Given a point D inside of the acute triangle ABC, such that DA  DB  AB
+ DB  DC  BC + DC  DA  CA ¼ AB  BC  CA. Find the locus of point D.
8.1.13. A convex polygon is broken by nonintersecting diagonals into triangles.
Prove that the sum of inradiuses of all these triangles does not depend on the way of
breaking, if and only if the convex polygon is inscribed. (The diagonals may have
common endpoints).
8.1.14. Given points A1, B1, and C1 on the sides BC, CA, and AB of triangle ABC,
respectively, such that ∠AB1C1 + ∠ BC1A1 + ∠ CA1B1 ¼ 180 . Given also that the
(a) areas,
(b) perimeters,
(c) inradiuses
of triangles AB1C1, BC1A1, and CA1B1 are equal. Prove that A1 , B1 and C1 are the
midpoints of sides BC, CA, and AB.
8.1 Application of Geometric Inequalities for Solving Geometric problems 415

8.1.15. Given points D and E on sides AC and BC of triangle ABC, respectively, such
that ∠CDE ∠CED
∠BDE ¼ ∠AED . Is it true that ABC is an isosceles triangle, if AE and BD are the

(a) altitudes,
(b) medians,
(c) bisectors
of ABC?
8.1.16. (a) Prove that if for the circumscribed quadrilateral ABCD the condition
p2 ¼ AC2 + BD2 holds true, where p is the semiperimeter of the quadrilateral,
then it is a rhombus.
(b) Let quadrilateral ABCD be inscribed in circle ω. Let E be the intersection point
of rays AB, DC, and F be the intersection point of rays BC, AD. Given that
AC ⊥ EF. Prove that segment AC is a diameter of the circle ω.

8.1.17. In triangle ABC the bisectors of angles ∠ABC and ∠BCA intersect sides CA
and AB at points D and E, respectively. Let
(a) AE ¼ BD and AD ¼ CE,

CE ¼ AE and ∠A  36 .
(b) AD BD

Find the angles of triangle ABC.


8.1.18. Let two isosceles triangles ABC and ADC be constructed on the base AC in
different half-planes, such that ∠ADC ¼ 3 ∠ ACB. Let AE be the bisector of triangle
ABC and F be the intersection point of segments DE and AC. Prove that CEF is an
isosceles triangle.
8.1.19. Given a convex hexagon, such that for its each pair of opposite sides the
following condition holds true: the ratio of the distance between the midpoints of
pffiffi
these sides to the sum of the lengths of these sides is equal to 23. Prove that all
angles of the hexagon are equal. (A convex hexagon ABCDEF has three pairs of
opposite sides: AB and DE, BC and EF, CD and FA).
8.1.20. Prove that one can circumscribe a circle around the convex quadrilateral
ABCD, if and only if the common tangent of the incircles of triangles ABD and
ACD, different from AD, is parallel to BC.
8.1.21. (a) For triangle ABC, prove that haa  12 ctg α2.
(b) Let circle ω be inscribed in quadrilateral ABCD and I be the center of ω. Given
that (AI + DI)2 + (BI + CI)2 ¼ (AB + CD)2. Prove that ABCD is an isosceles trap-
ezoid or a square.
(c) Let point I be inside of a convex quadrilateral ABCD. Given that ∠BIC ¼ ∠ BAI
+ ∠ CDI, ∠ CID ¼ ∠ CBI + ∠ DAI, and ∠AIB + ∠ CID ¼ 180 . Prove that one
can inscribe a circle in quadrilateral ABCD.
416 8 Some Applications of Geometric Inequalities

8.1.22. (a) Let ABCDEF be a convex hexagon, such that AD ¼ BC + EF,


BE ¼ AF + CD, CF ¼ AB + DE. Prove that DE
AB
¼ CD
AF ¼ BC.
EF

(b) Let ABCDEF be a convex hexagon, such that the sum of the distances of each
interior point to the six sides (AB, BC, CD, DE, EF, and FA) is equal to the sum
of the distances between the midpoints of AB and DE, BC and EF, CD and FA.
Prove that ABCDEF is a cyclic hexagon.
8.1.23. Let equilateral triangles ACB0 and BDC0 are drawn on the diagonals of a
convex quadrilateral ABCD, such that points B, B0 are on the same side of AC, and
points C, C0 are on the same side of BD. Find ∠BAD + ∠ CDA, if B0 C0 ¼ AB + CD.

Solutions

8.1.1. Note that ΔMAB ¼ ΔMCD, consequently, MA ¼ MD.


If MA > AD, then ∠MAD ¼ ∠ MDA > ∠ AMD. Therefore ∠MAD > 60 , thus
∠MAB < 30 and MA > AB. Hence ∠ABM > 75 . This leads to a contradiction.
Similarly, we can prove that if MA < AD, then ∠ABM < 75 .
We deduce that MA ¼ AD, which means that triangle AMD is equilateral.
8.1.2. If BD > 1 ¼ CD, then ∠DCB > ∠ DBC ¼ 30 . Let point E be on the segment
BD and BE ¼ 1, thus ∠BAD > ∠ BAE ¼ 30 . We obtain that ∠A + ∠ C > 60 . This
leads to a contradiction.
Similarly, one can prove that if BD < 1, then ∠A + ∠ C < 60 .
Hence, BD ¼ 1, therefore AD ¼ 2.
One can easily verify that there exists a triangle satisfying these conditions.
8.1.3. Note that ∠EFC ¼ 20 .
If EC > AE ¼ EF, then ∠ECF < ∠ EFC ¼ 20 and ∠ECA < ∠ EAC ¼ 75 .
Consequently, ∠ACB ¼ ∠ ECF + ∠ ECA < 95 . This leads to a contradiction.
Similarly, one can prove that if EC < AE, then ∠ACB > 95 .
Since ∠ACB ¼ 95 , hence EC ¼ AE. Thus ∠AEC ¼ 30 .
One can easily verify that there exists a triangle satisfying these conditions.
8.1.4. If BD > AB ¼ BC, then ∠BDA < ∠ BAD. Consequently, ∠BDA < 57.5 and
∠BDC < ∠ BCD. Hence ∠BDC < 72.5 , therefore ∠ADC < 57.5 + 72.5 ¼ 130 .
This leads to a contradiction.
Similarly, one can prove that if BD < AB, then ∠ADC > 130 . On the other
hand, since ∠ADC ¼ 130 , then we have that BD ¼ AB. Consequently, ∠A ¼ 57.5 ,
∠C ¼ 72.5 . This ends the proof.
8.1.5. Let BC ¼ a, thus AB ¼ 2a > BC. Therefore, one can take on side AB a point D,
such that ∠DCA ¼ ∠ A ¼ α. Thus AD ¼ DC ¼ b and ∠CDB ¼ 2α.
8.1 Application of Geometric Inequalities for Solving Geometric problems 417

If b > a, then b > a > 2a  b. Therefore for triangle BDC, we have the inequality
α + 30 > 2α > 150  3α. This leads to a contradiction.
If b < a, then b < a < 2a  b. Hence α + 30 < 2α < 150  3α.
This leads to a contradiction.
If b ¼ a, then triangle BDC is equilateral. Therefore ∠A ¼ 30 , ∠B ¼ 60 ,
∠C ¼ 90 . This ends the proof.
8.1.6. Let N be a point symmetric to point M with respect to line AC.
We have that ∠NAC ¼ ∠ MAC ¼ 80 . Consequently, N lies on line AB. Since
∠NCA ¼ ∠ MCA ¼ 30 and MC ¼ NC, then triangle MNC is equilateral.
If BM < MN ¼ MC, then ∠NBM > ∠ MNB ¼ 10 and ∠MBC > ∠ MCB ¼ 20 .
Therefore, ∠ABC > 30 . This leads to a contradiction.
Similarly, one can prove that if BM > MN ¼ MC, then ∠ABC < 30 .
This leads to a contradiction. Consequently, BM ¼ MN. Hence ∠AMB ¼ 150 .
8.1.7. Let N be a point symmetric to point M with respect to line AC, and lines AC,
MN intersect at point K, which means MK ⊥ AC and MK ¼ NK. Note that N lies on
line AB.
If BM > MC, then ∠MCB > ∠ MBC ¼ 20 . Since MK ¼ MN 2 ¼ 2 > 2 , then
BM MC
 
∠MCK > 30 . Consequently, ∠ACB > 50 . This leads to a contradiction.
Similarly, one can prove that if BM < MC, then ∠ACB < 50 .
This leads to a contradiction.
Hence BM ¼ MC, ∠MCB ¼ 20 , and ∠AMC ¼ 70 . This ends the proof.
8.1.8. Let O and O1 be the incenter and circumcenter of triangle ABC, respectively.
By the triangle inequality, R ¼ AO  AO1 + O1O  (la  r) + r ¼ R.
Hence, la ¼ AO1 + r, O1O ¼ r, and O1 2 [AO], consequently, ∠B ¼ ∠ C and
∠A ¼ 90 . Therefore ∠B ¼ ∠ C ¼ 45 . This ends the proof.
8.1.9. If BC > AB, then AE < CF (see problem 7.1.63). Similarly, if BC < AB, then
AE > CF. Hence BC ¼ AB. This ends the proof.
8.1.10. Let quadrilateral ABCD be circumscribed, thus AB + CD ¼ AD + BC. We
have that AM ¼ AB þ AC 2
 BC
¼ AD þ AC
2
 CD
¼ AQ. Similarly, one can prove that
CN ¼ CP. Consequently,
     
∠QMN þ ∠QPN ¼ 180  ∠AMQ  ∠BMN þ 180  ∠CPN  ∠DPQ
∠A þ ∠B ∠C þ ∠D 
¼ þ ¼ 180 :
2 2

Hence quadrilateral MNPQ is inscribed.


If AB + CD > AD + BC, then AM > AQ and CP > CN. Therefore, ∠AQM >
 
∠ AMQ and ∠CNP > ∠ CPN. Hence ∠AMQ < 90  ∠A ∠C
2 and ∠CPN < 90  2 .
Consequently,
418 8 Some Applications of Geometric Inequalities

   
 ∠B  ∠D
∠QMN þ ∠QPN ¼ 90  ∠AMQ þ þ 90  ∠CPN þ
   2  2
∠A ∠B ∠C ∠D 
> þ þ þ ¼ 180 :
2 2 2 2

Similarly, one can prove that, if AB + CD < AD + BC, then ∠QMN +


∠ QPN < 180 .
Thus, if ∠QMN + ∠ QPN ¼ 180 , then AB + CD ¼ AD + BC. This means that, if
quadrilateral MNPQ is inscribed, then quadrilateral ABCD is circumscribed.
This ends the proof.
8.1.11. Let quadrilateral ABCD be inscribed, thus point P is the center of the
circumcircle of ABCD.
Consequently, PA ¼ PB ¼ PC ¼ PD and ∠APB + ∠ CPD ¼ 2 ∠ BCA + 2
∠ CBD ¼ 180 . Hence, it follows that SABP ¼ 12 PA  PB sin ∠APB ¼ 12 PC
PD sin ∠CPD ¼ SCDP .
Let in the convex quadrilateral ABCD diagonals AC and BD are perpendicular.
Let the perpendicular bisectors to sides AB and DC intersect at point P inside
ABCD. We need to prove that if PA > PD, then SABP > SCDP.
Let E 2 AC, F 2 BD, and PE k BD, PF k AC, thus AE > EC, BF > FD.
Let C1 be a point symmetric to point A with respect to point E, and D1 be a point
symmetric to point B with respect to point F. Thus, it follows that
PC1 ¼ PA ¼ PB ¼ PD1 and segment PM intersects line CD, where M is the mid-
point of segment C1D1. Therefore PM > PH, where PH ⊥ CD, H 2 CD.
We have that C1D1 > CD, consequently, SABP ¼ SC1 D1 P ¼ 12 C1 D1  PM > 12 CD
PH ¼ SCDP .
It is clear that if PA < PD, then SABP < SCDP. Thus, if SABP ¼ SCDP, then
PA ¼ PD. Therefore, quadrilateral ABCD is inscribed. This ends the proof.
8.1.12. According to problem 4.1.8c, we have that DA  DB  AB + DB  DC  BC +
DC  DA  CA  AB  BC  CA, the equality holds true, if
! 
! 
!
A1 B1  D1 C1 þ B1 C1  D1 A1 þ A1 C1  D1 B1 ¼ ~
0, ð8:1Þ

where D1, A1, B1, C1, are H images of points D, A, B, C (see the proof of problems
4.1.8a,c).
If D1 is the incenter of triangle A1B1C1, then condition (8.1) is satisfied (see the
proof of problem
 ! 4.1.19). Let O beany point, then condition(8.1) can be rewritten
! ! ! ! !
as A1 B1  OC1  OD1 þ B1 C1  OA1  OD1 þ A1 C1  OB1  OD1 ¼ ~ 0.
! ! ! !
Therefore, OD1 ¼ A1 B1 OCA1 1þB 1 C1 OA1 þA1 C1 OB1
B1 þB1 C1 þA1 C1 . Thus, condition (8.1) is holds
true only for one point D1 (the incenter of triangle A1B1C1).
8.1 Application of Geometric Inequalities for Solving Geometric problems 419

We have that ∠D1A1C1 ¼ ∠ D1A1B1 and ∠D1A1C1 ¼ ∠ DCA, ∠ D1A1B1 ¼


∠ DBA. Hence ∠DCA ¼ ∠ DBA. Similarly, we obtain that ∠DBC ¼ ∠ DAC and
∠DAB ¼ ∠ DCB.
Consequently, ∠DAC þ ∠DAB þ ∠DBA ¼ 12 ð∠A þ ∠B þ ∠CÞ ¼ 90 . Hence,
BD ⊥ AC. In the same way, we deduce that AD ⊥ BC and CD ⊥ AB.
Hence, point D is the orthocenter of triangle ABC.
This ends the proof.
8.1.13. Let the convex polygon A1A2 . . . An be inscribed. Let A1A2 . . . An be broken
by nonintersecting diagonals into k triangles. Therefore, we have that
∠A1 + ∠ A2 + . . . + ∠ An ¼ 180 k and ∠A1 + ∠ A2 + . . . + ∠ An ¼ 180 (n  2).
Thus, it follows that k ¼ n  2.
Let the inradiuses of these triangles be equal to r1 , r2 , . . . , rn  2, and R be the
circumradius of polygon A1A2 . . . An.
Using problem 5.3.6 for all these n  2 triangles, one can easily obtain that
_
n  2 þ rR1 þ . . . þ rn2
R ¼ cos α1 þ cos α2 þ . . . þ cos αn , where αi ¼ 12 Ai A iþ1 ,
_
i ¼ 1 , . . . , n and An + 1  A1, and the arc Ai A iþ1 does not contain the vertices of
polygon A1A2 . . . An other than Ai and Ai + 1. Consequently, r1 + . . . + rn  2 does not
depend on the way of breaking.
Let the convex polygon A1A2 . . . An (n  4) be such that for any way of breaking
it by nonintersecting diagonals into triangles, the sum of the inradiuses of these
triangles is the same. Then, note that quadrilateral A1A2A3A4 also satisfies that
condition and using problem 6.1.12b we deduce that quadrilateral A1A2A3A4 is
inscribed.
Similarly, one can prove that for n  5 quadrilateral AiAi + 1Ai + 2Ai + 3, i ¼ 2 , . . . ,
n  3, is inscribed, consequently, polygon A1A2 . . . An is inscribed. This ends the
proof.
8.1.14. Let A0 , B0 , C0 be the midpoints of sides BC, CA, AB, respectively, and let
points A1, A0 be different. Let point A1 belong to segment BA0. If B1 belongs to
segment CB0, then C1 belongs to segment BC0. Otherwise, we have that
∠C ¼ ∠ AB0C0  ∠ AB1C1, ∠A ¼ ∠ BC0A0  ∠ BC1A1, and ∠B ¼ ∠ B0A0C >
∠ CA1B1. Thus, 180 ¼ ∠ C + ∠ A + ∠ B > ∠ AB1C1 + ∠ BC1A1 + ∠ CA1B1.
This leads to a contradiction.
We obtain that ΔBC1A1  ΔBC0A0 ¼ ΔAC0B0 ΔAC1B1, thus conditions (a),
(b), and (c) are wrong (see problem 2.1.1 and the proof of problem 6.1.8b).
If B1 belongs to segment AB0, and C1 belongs to segment AC0, then
ΔAB1C1 ΔAB0C0 ¼ ΔCA0B0  ΔCA1B1. This leads to a contradiction.
If B1 belongs to segment AB0, and C1 belongs to segment BC0, then
ΔBC1A1  ΔBC0A0 ¼ ΔCA0B0  ΔCA1B1. This leads to a contradiction.
Similarly, one can prove that if A1 belongs to segment CA0, then this case also
leads to a contradiction.
Hence points A1, B1 and C1 are the midpoints of sides BC, CA, and AB.
∠CDE
8.1.15. Let ¼ k, ∠BDE ¼ y, ∠AED ¼ x, thus ∠CDE ¼ ky and ∠CED ¼ kx.
∠BDE
420 8 Some Applications of Geometric Inequalities

Figure 8.1 C

kx
ky E
D y x
M

A B

(a) We have that kx þ x ¼ π2 ¼ ky þ y. Consequently, x ¼ y. Since points D and


E are on the circle with a diameter AB, then x ¼ π2  ∠A, y ¼ π2  ∠B.
Hence, ∠A ¼ ∠ B, therefore AC ¼ BC.
(b) We have that ∠A ¼ ky, ∠B ¼ kx, ∠MAB ¼ x, ∠ MBA ¼ y, x ¼ y.
Let x  y.
If x > y, then AC > BC and MB > MA (see Figure 8.1).
 
4 2AB2 þ 2BC2  AC2 2AC2 þ 2AB2  BC2
Thus, MB  MA ¼
2 2
 < 0.
9 4 4
This leads to a contradiction.
Hence, x ¼ y, then ∠A ¼ ∠ B. Therefore, AC ¼ BC.
(c) Note that ∠A ¼ 2(ky  x) and ∠B ¼ 2(kx  y), ∠C ¼ π  kx  ky. Consequently,
2(ky  x) + 2(kx  y) + π  kx  ky ¼ π, kx + ky ¼ 2(x + y), k ¼ 2.
Let N be the intersection point of the bisectors of triangle CDE, thus CN and CM
are bisectors of angle ACB. Consequently, C, N, M belong to the same line.
Since MN ⊥ DE, then CM ⊥ DE. Hence π2  2y ¼ π2  2x, y ¼ x, thus ∠A ¼ ∠ B.
This ends the proof.
8.1.16. (a) Let A1, B1, C1, D1 be the midpoints of the sides AB, BC, CD, DA,
respectively. Then, since quadrilateral A1B1C1D1 is a parallelogram, we have that
AC2 þ BD2
2A1 B21 þ 2B1 C21 ¼ A1 C21 þ B1 D21 . Therefore, we obtain that ¼
2
A1 C1 þ B1 D1 .
2 2

According to problem 1.1.9a, we have that A1 C1  p2 , B1 D1  p2 . Consequently,


AC2 + BD2  p2. The equality holds true, if AB k B1D1 k CD and BC k A1C1 k AD,
which means that ABCD is a parallelogram and as AB + CD ¼ BC + AD, then
AB ¼ BC ¼ CD ¼ AD. This means that ABCD is a rhombus.
(b) Note that ∠A + ∠ D < 180 , ∠A + ∠ B < 180 , consequently 2 ∠ A + ∠ B +
∠ D < 360 . Hence, it follows that ∠A < 90 . Segment AC intersects segment
EF, consequently AEF is an acute triangle.
8.1 Application of Geometric Inequalities for Solving Geometric problems 421

Figure 8.2 E

B
B0

A
C0 C К

D0
D

Let C0 be the orthocenter of that triangle. If points C0 and C do not coincide, then
∠B < 90 and ∠D < 90 or ∠B > 90 and ∠D > 90 . Hence, it follows that
180 ¼ ∠ B + ∠ D < 180 (see Figure 8.2) or 180 ¼ ∠ B + ∠ D > 180 .
Both these inequalities are wrong. Thus, points C0 and C coincide. Hence, it
follows that ∠B ¼ 90 , which means that AC is the diameter of circle ω.
8.1.17. (a) (Proof of Hayk Sedrakyan, ninth grade) Let BC ¼ a, AB ¼ c, AC ¼ b
∠A ¼ 2α, ∠ B ¼ 2β, ∠ C ¼ 2γ.
bc bc
We have that BD ¼ AE ¼ and EC ¼ AD ¼ . On the other hand,
aþb aþc
BD ¼ AB  BC  AD  DC, EC ¼ BC  AC  AE  EB.
2 2

b2 c ab2 c2 b c2 a
Therefore, 2
¼a 2
, 2
¼a .
ð a þ bÞ ða þ cÞ ða þ cÞ ð a þ bÞ 2
x2 y
Let b ¼ xa and c ¼ ya, then from the last equalities, we deduce that
ð x þ 1Þ 2
x2 xy2 y2
¼1 2
and 2
¼1 . As b + c > a, then x + y > 1.
ð y þ 1Þ ð y þ 1Þ ð x þ 1Þ 2
x2 y xy2 y2 x2
Note that 2
 2
¼ 2
 , consequently, (y  x)
ð x þ 1Þ ð y þ 1Þ ð x þ 1Þ ð y þ 1Þ 2
(x 2y 2  xy) ¼ (y  x)(y 3 + xy 2 + x 2y + y 3 + 2(y 2 + xy + x 2) + y + x).
Hence y ¼ x or x2y2 ¼ y3 + xy2 + x2y + x3 + 2y2 + 3xy + 2x2 + x + y.
Otherwise, x2y2 > xy2 + yx2 + 3xy or xy > x + y + 3 > 4. We have obtained that
bc > 4a2. By the law of sines, it follows that sin2β sin 2γ > 4sin22α. Note that
BD AE sin 2α sin γ
¼ , hence ¼ . Therefore, sin2β sin 2γ > 4 sin β sin γ or
AD EC sin β sin 2α
cosβ cos γ > 1. This leads to a contradiction.
422 8 Some Applications of Geometric Inequalities

We have that x ¼ y, which means that b ¼ c. Thus, AD ¼ AE ¼ BD, consequently,


β ¼ γ ¼ 2α. Hence, ∠A ¼ 36 , ∠ B ¼ 72 , ∠ C ¼ 72 .
AD CE
(b) See the notations of problem 8.1.17a. We have that 2α  36 and ¼ .
BD AE
sin β sin 2α
Hence ¼ , sinβ sin γ ¼ sin22α.
sin 2α sin γ
Let β ¼ φ + x, γ ¼ φ  x, where φ < x < φ, β + γ  72 , φ  36 .
We have that sin(φ + x) sin (φ  x) ¼ sin24φ, consequently, cos2x ¼
2sin24φ + cos 2φ.
Note that 2sin24φ + cos 2φ  1, since φ  4φ  180  φ, sin4φ  sin φ.
Therefore cos2x  1. Consequently, x ¼ 0 and 4φ ¼ 180  φ.
Hence ∠A ¼ 36 , ∠B ¼ 72 , ∠C ¼ 72 . This ends the proof.
 
8.1.18. Let ∠ACB ¼ γ, thus ∠DAE ¼ 90  γ, ∠DCE ¼ 90  2γ , ∠AEC ¼ 180 
2 . If DE > AD, then from triangles ADE and DEC, we obtain that


∠AED < ∠ DAE ¼ 90  γ, ∠DEC < ∠DCE ¼ 90  2γ . Therefore, it follows

that ∠AEC < 180  3γ2 . This leads to a contradiction. Similarly, we obtain that,
if DE < AD, then this also leads to a contradiction.

Hence DE ¼ DC. Thus, it follows that ∠DEC ¼ 90  2γ .

Consequently, ∠EFC ¼ 90  2γ , therefore CE ¼ CF. This ends the proof.
8.1.19. Let M be the intersection point of diagonals AD, CF (see Figure 8.3), and
points P and Q be the midpoints of sides CD and AF, respectively.
pffiffiffi pffiffiffi pffiffiffi
3 3 3
As MP þ MQ  PQ ¼ CD þ AF, then either MP  CD or
pffiffiffi 2 2 2
3
MQ  AF.
2 pffiffiffi
3
Let MP  CD, we need to prove that ∠CMD  60 . Indeed, if
pffiffiffi 2
3
MP ¼ CD, then using the law of cosines and the formula for the median, we
2
obtain that

MC2 þ MD2  CD2 CD2 CD2 1


cos ∠CMD ¼ ¼  2 ¼ :
2  MC  MD 2  MC  MD M C þ MD 2 2

Figure 8.3 C P D

B
M E

A Q F
8.1 Application of Geometric Inequalities for Solving Geometric problems 423

pffiffiffi
 3
Consequently, ∠CMD  60 . If MP > CD let us take a point M0 on segment
pffiffiffi 2
3
PM, such that M0 P ¼ CD. Thus, as we have proven above, ∠CM0D  60 .
2
We have that ∠CMD < ∠ CM0D, hence ∠CMD < 60 .
Similarly, we obtain that ∠BNC  60 and ∠BKA  60 , where N ¼ CF \ BE,
K ¼ AD \ BE. Since ∠CMD + ∠ BNC + ∠ BKA ¼ 180 , then ∠CMD ¼

∠ BNC ¼ ∠ BKA ¼ 60 . Note that, the equality ∠CMD ¼ 60 is possible if MP
pffiffiffi pffiffiffi
3 3
þMQ ¼ CD þ AF and MC ¼ MD. Thus, we deduce that ∠MCD ¼
2 2 pffiffiffi
3
∠ DMC ¼ 60 , MQ ¼ AF, hence ∠MAF ¼ ∠ MFA ¼ 60 .
2
Similarly, we obtain that ∠NBC ¼ ∠ NCB ¼ ∠ NEF ¼ ∠ NFE ¼ 60 and
∠KAB ¼ ∠ KBA ¼ ∠ KED ¼ ∠ KDE ¼ 60 , hence ∠A ¼ ∠ B ¼ . . . ¼ ∠ F ¼ 120 .
See also problem 4.1.4b.
8.1.20. Let us denote the incenters of triangles ABD and ACD by O1 and O2,
respectively, and the common tangent of these incircles, different from AD, by
l (see Figure 8.4).
Note that l and AD are symmetric to each other with respect to O1O2, thus l and
BC are parallel, if and only if ∠CBD  ∠ ADB ¼ 2(∠O2O1D  ∠ O1DA). This
means that

∠CBD ¼ 2∠O2 O1 D: ð8:2Þ

C
l

O2
O1

A D

Figure 8.4
424 8 Some Applications of Geometric Inequalities

If ABCD is an inscribed quadrilateral, then ∠ABD ¼ ∠ ACD. Thus, it follows


that ∠AO1 D ¼ 90 þ 12 ∠ABD ¼ 90 þ 12 ∠ACD ¼ ∠AO2 D. Consequently, points
A, O1, O2, D belong to the same circle. Therefore, we have obtained that
∠CBD ¼ ∠ CAD ¼ 2 ∠ O2AD ¼ 2 ∠ O2O1D, which means that l k BC.
Let l k BC, prove that quadrilateral ABCD is inscribed.
Denote by ∠CAD ¼ α, ∠CAB ¼ α1, ∠ABD ¼ β, ∠DBC ¼ β1, ∠ACB ¼ γ,
∠ACD ¼ γ 1, ∠CDB ¼ δ, ∠BDA ¼ δ1.
According to the law of sines, we have that

AB BC CD DA sin γ sin δ sin α sin β


1¼    ¼    :
BC CD DA AB sin α1 sin β1 sin γ 1 sin δ1

Consequently,

sin α sin β sin γ sin δ ¼ sin α1 sin β1 sin γ 1 sin δ1 : ð8:3Þ


β
We have that l k BC, thus according to (8.2) we obtain that ∠O2 O1 D ¼ 1 and
2
γ
∠AO2 O1 ¼ (the proof is similar to the proof of (8.2)).
2
α α1 β
On the other hand, ∠O2 AD ¼ , ∠O2 AO1 ¼ , ∠AO1 D ¼ 90 þ ,
2 2 2
 γ1 δ1 δ
∠AO2 D ¼ 90 þ , ∠O1 DA ¼ , ∠O2 DO1 ¼ . Now, by (8.3), for the quadri-
2 2 2
lateral AO1O2D we obtain that

α β γ δ α1 β γ δ1
sin cos sin sin ¼ sin sin 1 cos 1 sin : ð8:4Þ
2 2 2 2 2 2 2 2

Dividing equality (8.3) by equality (8.4), we deduce that

α β γ δ α1 β γ δ1
cos sin cos cos ¼ cos cos 1 sin 1 cos : ð8:5Þ
2 2 2 2 2 2 2 2

Now, let us prove that β ¼ γ 1. Indeed, let β 6¼ γ 1, thus without loss of generality
one can assume that β < γ 1. In that case point C is inside of the circumcircle of
triangle ABD. Therefore, α > β1 and γ > δ1, thus from (8.4) we deduce that
cos β2 sin 2δ < sin α21 cos γ21 , while from (8.5) we have that cos α21 sin γ21 < cos 2δ sin β2.
α1 γ 1
Adding these two last inequalities, we obtain that sin δβ 2 < sin 2 .
This leads to a contradiction, as δ  β ¼ α1  γ 1
Therefore, β ¼ γ 1. Thus, ABCD is an inscribed quadrilateral.
This ends the proof.
(c) We are going to use the following lemma.
8.1 Application of Geometric Inequalities for Solving Geometric problems 425

Figure 8.5

Lemma Let point P be inside of a convex quadrilateral ABCD (see Figure 8.5).
Given that ∠PAD + ∠ PDA + ∠ PBC + ∠ PCB  180 and ∠BPC ¼ ∠ BAP
+ ∠ CDP. Prove that AD + BC  AB + CD.
Proof of the Lemma Indeed, let ∠BAP ¼ x, then ∠CDP ¼ ∠ BPC  x.
We have that
qffiffiffiffiffiffiffiffiffiffiffiffiffiffiffiffiffiffiffiffiffiffiffiffiffiffiffiffiffiffiffiffiffiffiffiffiffiffiffiffiffiffiffiffiffiffiffiffiffiffiffiffiffiffiffiffiffiffiffiffiffiffiffiffiffiffiffiffiffiffiffiffiffiffiffiffiffiffiffiffiffiffiffi
jAP cos x þ DP cos ð∠BPC  xÞj  ðAP þ DP cos ∠BPCÞ2 þ ðDP sin ∠BPCÞ2 ¼
pffiffiffiffiffiffiffiffiffiffiffiffiffiffiffiffiffiffiffiffiffiffiffiffiffiffiffiffiffiffiffiffiffiffiffiffiffiffiffiffiffiffiffiffiffiffiffiffiffiffiffiffiffiffiffiffiffiffiffiffiffiffiffiffiffiffiffi pffiffiffiffiffiffiffiffiffiffiffiffiffiffiffiffiffiffiffiffiffiffiffiffiffiffiffiffiffiffiffiffiffiffiffiffiffiffiffiffiffiffiffiffiffiffiffiffiffiffiffiffiffiffiffiffiffiffiffiffiffiffiffiffiffiffiffiffi
¼ AP2 þ DP2 þ 2AP  DP cos ∠BPC  AP2 þ DP2  2AP  DP cos ∠APD ¼ AD,

therefore

jAP cos x þ DP cos ð∠BPC  xÞj  AD: ð8:6Þ

According to (8.6), we obtain that AD  DN  AM and BC  BM  CN. Hence,


we deduce that AD + BC  AB + CD.
According to the lemma, we have that AD + BC  AB + CD and AB + CD  AD
+ BC. Therefore AB + CD ¼ AD + BC.
Remark One can prove that point I is the incenter of quadrilateral ABCD.
8.1.22. (a) We need to prove that for any hexagon ABCDEF, the inequality (AB
+ DE)2 + (AF + CD)2 + (BC + EF)2  AD2 + BE2 + CF2 holds true and that the equal-
! ! ! ! ! ! ! !
ity holds true, if and only if AB "" ED , CD "" AF , BC "" FE , and AB þ CD
!
þ EF ¼ ~ 0.
426 8 Some Applications of Geometric Inequalities

! ! ! ! !
a, BC ¼ ~
Indeed, let AB ¼ ~ b, CD ¼ ~ c, DE ¼ ~d, EF ¼ ~ e. Thus, we have that







AB þ DE ¼ j~aj þ
~ a ~
d

~ d
, BC þ EF ¼
~ ej 
~
b
þ j~ b ~e
,




ð8:7Þ



CD þ AF ¼ j~cj þ
~a þ~b þ~ c þ~ d þ~ a þ~
e

~ b þ~c þ~ c þ~ d þ~e
:

Therefore, we deduce that

ðAB þ DEÞ2 þ ðAF þ CDÞ2 þ ðBC þ EFÞ2 


 2  2  2
 ~ a ~
d þ ~ e þ ~
b ~ a þ~
b þ~c þ~c þ~
d þ~e ¼
 2  2  2
¼ ~ a þ~
b þ~c þ ~ b þ~c þ~
d þ ~ c þ~ e þ ð~
d þ~ a þ~ e Þ2 
c þ~
 2  2  2
 ~ a þ~
b þ~c þ ~ b þ~c þ~
d þ ~ c þ~d þ~
e ¼ AD2 þ BE2 þ CF2 ,

hence

ðAB þ DEÞ2 þ ðAF þ CDÞ2 þ ðBC þ EFÞ2  AD2 þ BE2 þ CF2 : ð8:8Þ

Note that the equality in (8.8) holds true, if and only if it holds true in the
inequalities of (8.7) and~a þ~c þ~e ¼~
0, which means that~a "# ~d, ~
b "# ~e,~ a þ~
c "" ~ b
þ~ ~
c þ d þ~ e and ~a þ~ c þ~ ~
e ¼ 0.
From the assumptions of the problem, it follows that (AB + DE)2 + (AF + CD)2
+ (BC + EF)2 ¼ AD2 + BE2 + CF2. Therefore, AB k ED, BC k EF, CD k AF and
! ! ! ~
AB þ CD þ EF ¼ 0 .
Consider now parallelogram ABKF, then CDEK is also a parallelogram, conse-
quently, K 2 BE and K 2 CF.
FK KE EF AB CD EF
Thus ΔEKF
ΔBKC, hence ¼ ¼ , or ¼ ¼ .
KC BK BC DE AF BC
This ends the proof.
Second Solution Let AD ¼ p, BE ¼ q, CF ¼ r, and BE \ CF ¼ M. As q + r ¼ BM
+ ME + CM + MF ¼ BM + CM + ME + MF > BC + EF ¼ AD ¼ p, then q + r > p.
Similarly, we obtain that p + q > r, p + r > q. Thus, it follows that one can
construct a triangle with sides p, q, r. Note that α  α1 (see Figures 8.6 and 8.7).
Indeed, if α < α1, then

cos α > cos α1 : ð8:9Þ

Let us consider parallelogram BCKE (Figure 8.6). We have that FK2 ¼ r2 + q2


 2rq cos α1 > r2 + q2  2rq cos α ¼ p2, therefore FK > p. On the other hand
FK  FE + EK ¼ FE + BC ¼ AD ¼ p. This leads to a contradiction.
Similarly, we obtain that β  β1 and γ  γ 1. As α + β + γ ¼ 180 ¼ α1 + β1 + γ 1,
then α ¼ α1 , β ¼ β1 , γ ¼ γ 1.
8.1 Application of Geometric Inequalities for Solving Geometric problems 427

Figure 8.6

Figure 8.7

We have that α ¼ α1, therefore FK ¼ p ¼ FE + EK, which means that FE||BC.


As β ¼ β1, then AD||FE (Figure 8.6).
Similarly, we obtain that CD||BE||AF and AB||CF||DE, then quadrilaterals ABMF
EF ME MF
and DCME are parallelograms. As ΔFME
ΔBMC, then ¼ ¼ , or
BC BM CM
EF CD AB
¼ ¼ :
BC AF DE
(b) Let diagonals AD and CF, CF and BE, BE and AD intersect at points M, N, P,
respectively (Figure 8.8). Let point O intersects the bisector of triangle MNP, if
points M, N, P coincide, then point O coincides with point M.
Let A1, B1, C1, D1, E1, F1 be the midpoints of sides AB, BC, CD, DE, EF, FA of a
hexagon and da, db, dc, dd, de, df be the distances from point O to lines AB, BC, CD,
DE, EF, FA, respectively.
Note that A0D0  da + dd, B0E0  db + de, C0F0  dc + df, therefore

A0 D0 þ B0 E0 þ C0 F0  da þ db þ dc þ dd þ de þ d f : ð8:10Þ

On the other hand, we have that ∠A1A0D0  90 and ∠D1D0A0  90 , as in any
triangle the bisector is between the altitude and the median (if the bisector, altitude,
and median are drawn from the same vertex of a triangle).
Therefore, the projection of segment A1D1 on line A0D0 contains segment A0D0,
which means that A1D1  A0D0.
Similarly, we obtain that B1E1  B0E0, C1F1  C0F0.
428 8 Some Applications of Geometric Inequalities

Figure 8.8

Hence, from the last three inequalities and (8.10), we obtain that

A1 D1 þ B1 E1 þ C1 F1  da þ db þ dc þ dd þ de þ d f : ð8:11Þ

From the assumptions of the problem, it follows that (8.11) is an equality.


Note that, it holds true if points A1, B1, C1, D1, E1, F1 coincide with points A0, B0,
C0, D0, E0, F0, respectively.
Then, we obtain that lines B0C0, C0F0, A0D0 are the mid-perpendiculars of
segments BC and EF, CD and AF, DE and AB, respectively.
Then, point O is equidistant from points A, B, C, D, E, F. This means that
ABCDEF is a cyclic hexagon.
8.1.23. Let us externally construct an equilateral triangle BCF on side BC of a
convex quadrilateral ABCD.
Note that ∠FBC ¼ 60 ¼ ∠ C0 BD, therefore ∠FBC0 ¼ ∠ CBD. On the other
hand, we have that BF ¼ BC and BC0 ¼ BD, then ΔBFC0 ¼ ΔBCD.
Thus, it follows that FC0 ¼ CD and ∠BFC0 ¼ ∠ BCD. Similarly, we obtain that
B F ¼ AB and ∠B0 FC ¼ ∠ ABC (Figure 8.9).
0

We have that B0 C0 ¼ AB + CD, therefore B0 C0 ¼ B0 F + FC0 .


Hence, it follows that point F is on segment B0 C0 .
We deduce that ∠B0 FC + ∠ BFC0 ¼ 180 + ∠ BFC ¼ 240 .
Thus, we obtain that ∠BCD + ∠ ABC ¼ ∠ BFC0 + ∠ B0 FC ¼ 240 and ∠BAD +
∠ CDA ¼ 120 .

Problems for Self-Study

8.1.24. Let O be the intersection point of diagonals AC and BD of a convex


quadrilateral ABCD. Given that BO ¼ OD and AB + BC ¼ AD + DC.
Prove that either BD ⊥ AC or ABCD is a parallelogram.
8.1.25. Prove that, if in triangle ABC holds true p ¼ 2R + r, then triangle ABC is
right angled.
8.1 Application of Geometric Inequalities for Solving Geometric problems 429

Figure 8.9

Hint See problems 5.5.7a and 5.5.8a.


 pffiffiffi
8.1.26. Find the angles of triangle ABC, if la ¼ R  1 þ 2 r.
8.1.27. Let ABCD be a convex quadrilateral. Prove that if MNPQ is an inscribed
quadrilateral, then RSTU is also an inscribed quadrilateral, where M, N and P, Q are
tangential points of incircles of triangles ABC and ACD with sides AB, BC and CD,
AD, respectively, R, S and T, U are the tangential points of the incircles of triangles
ABD and BCD with sides AD, AB and BC, CD, respectively.
Hint See problem 8.1.10.
8.1.28. Prove that if in triangle ABC holds true la ¼ lb, then a ¼ b.
8.1.29. The altitudes drawn from the vertices A and C of triangle ABC intersect the
median BM at points E and F, respectively. Given that AE ¼ CF. Prove that ABC is
an isosceles triangle.
8.1.30. Prove that, if three bisectors of one triangle are equal, respectively to three
bisectors of another triangle, then these triangles are congruent.
8.1.31. The base A1A2 . . . An of pyramid PA1A2 . . . An is a regular polygon. Prove
that if ∠PA1A2 ¼ ∠ PA2A3 ¼ . . . ∠ PAn  1An ¼ ∠ PAnA1, then the pyramid is
regular.
8.1.32. Let O be the intersection point of diagonals AC and BD of a convex
quadrilateral ABCD. Given that the perimeters of all triangles AOB, BOC, COD,
DOA are equal. Prove that ABCD is a rhombus.
8.1.33. Let O be the intersection point of the diagonals of a convex quadrilateral
ABCD. Prove that if the inradiuses of triangles AOB, BOC, COD, and DOA are
equal to each other, then ABCD is a rhombus.
8.1.34. Given points D, E on sides AB, AC of triangle ABC, respectively. Prove that
if DP ¼ PE and AB ¼ AC, where P is the intersection point of segments BE and CD,
then BP ¼ PC.
Hint If BP > PC, then consider a point B0 on ray PC, such that PB0 ¼ PB. Thus,
∠PCB > ∠ PBC and ∠PCE > ∠ PB0 E ¼ ∠ PBD.
430 8 Some Applications of Geometric Inequalities

8.2 Using Geometric Inequalities for Proving Algebraic


Inequalities

8.2.1. Prove the following inequalities


(a) |sinx|  |x|,
π
(b) tgx > x, where 0 < x < ,
2
π
(c) α  sin α < β  sin β, where 0 < α < β < ,
2
π
(d) tgα  α < tgβ  β, where 0 < α < β < ,
2
sin α sin β π
(e) > , where 0 < α < β < ,
α β 2
2 π
(f) sin x > x, where 0 < x < ,
π 2
tgα tgβ π
(g) < , where 0 < α < β < ,
α β pffiffiffi 2
3 3
(h) sin α þ sin β þ sin γ  , where α, β, γ > 0 and α + β + γ ¼ π.
2
8.2.2. Prove the inequalities
π
(a) 2þ 2 sin x cos y þ 2 sin y cos z > sin 2x þ sin 2y þ sin 2z, where 0 < x < y
< z < π2,
pffiffiffiffiffiffiffiffiffiffiffiffiffi pffiffiffiffiffiffiffiffiffiffiffiffiffiffiffi qffiffiffiffiffiffiffiffiffiffiffiffiffiffiffiffiffiffiffiffiffiffiffiffiffiffiffi
(b) 0:785n2  n < n2  1 þ n2  22 þ . . . þ n2  ðn  1Þ2 < 0:79n2 , where
n 2 N.
1h X n
1þh
8.2.3. Prove that the inequality < x2i ðx2iþ1  x2i1 Þ < holds true
2 i¼1
2
for real numbers 0 ¼ x1 < x2 < . . . < x2n < x2n + 1 ¼ 1, such that xi + 1  xi  h,
1  i  2n.
8.2.4. Let the real numbers a1, . . . , an, an + 1 and b1, . . . , bn be such that 0  bk  1,
k ¼ 1, . . . , n, and a1  a2  . . .  an + 1 ¼ 0. Let [b1 + . . . + bn] ¼ p. Prove that
a1b 1 + a 2b 2 + . . . + a nb n  a 1 + a 2 + . . . + a p + a p + 1.
8.2.5. Prove the inequalities
(a) a2 + b2  2ab, where a, b > 0,
(b) a3 + b3 + c3  3abc, where a, b, c > 0,
sffiffiffiffiffiffiffiffiffiffiffiffiffiffiffi
2 pffiffiffiffiffi a þ b a2 þ b2
(c)  ab   , where a, b > 0.
1 1 2 2
þ
a b
8.2 Using Geometric Inequalities for Proving Algebraic Inequalities 431

pffiffiffi pffiffiffiffiffiffiffiffiffiffiffiffiffiffiffi pffiffiffiffiffiffiffiffiffiffiffiffiffiffiffi 2


8.2.6. Prove that 6 3aðb þ cÞ  a2 þ c2 þ a2 þ b2 þ b þ c , where a, b,
c > 0.
8.2.7. Prove the inequalities
(a) ((x + y)(y+ z)(x +z))2  xyz(2x +y + z)(2y + x + z)(2z
 + x + y), where x, y, z > 0,
 a b  c aþbþc
(b) 1 þ 1þ 1þ 2 1þ p ffiffiffiffiffiffiffi , where a, b, c > 0,
b c a 3
abc
1 1 1 9
(c) 2
þ 2
þ 2
 , where x, y, z > 0 and x + y + z ¼ 1.
ðxy þ zÞ ðxz þ yÞ ðyz þ xÞ 16xyz
qffiffiffiffiffiffi qffiffiffiffiffiffi qffiffiffiffiffiffi
8.2.8. Prove the inequality 2a
aþb þ 2b
bþc þ 2c
cþa  3, where a, b, c > 0.

8.2.9. Find the maximal constant number C, such that the inequality ðx þ yÞðy þ zÞ
pffiffiffiffiffiffiffi
ðz þ xÞ  C xyz holds true for all x, y, z > 0 and x + y + z ¼ 1.
8.2.10. Prove that
bþc ðy þ zÞ þ cþa ðx þ zÞ þ aþb ðx þ yÞ  3, where a + b, b + c, c + a > 0, x, y,
a b c

z > 0, and xy + yz + zx ¼ 3.
8.2.11. Prove that
pffiffiffiffiffiffiffiffiffiffiffiffiffiffiffiffiffiffiffiffiffiffiffiffiffi pffiffiffiffiffiffiffiffiffiffiffiffiffiffiffiffiffiffiffiffiffiffiffiffi pffiffiffiffiffiffiffiffiffiffiffiffiffiffiffiffiffiffiffiffiffiffiffiffi
xy x2 þ xy þ y2 þ yz y2 þ yz þ z2 þ zx x2 þ xz þ z2
pffiffiffiffiffiffiffiffiffiffiffiffiffiffiffiffiffiffiffiffiffiffiffiffiffiffiffiffiffiffiffiffiffiffiffiffiffiffiffiffiffiffiffiffiffiffiffiffiffiffiffiffiffiffiffiffiffiffiffiffiffiffiffiffiffiffiffiffiffiffiffiffiffiffiffiffiffiffiffiffiffiffiffi
 ðx2 þ xy þ y2 Þðy2 þ yz þ z2 Þðx2 þ xz þ z2 Þ, where x, y, z > 0.
8.2.12. Prove that
pffiffiffiffiffiffiffiffiffiffiffiffiffiffiffiffiffiffiffiffiffiffiffiffiffi pffiffiffiffiffiffiffiffiffiffiffiffiffiffiffiffiffiffiffiffiffiffiffiffi pffiffiffiffiffiffiffiffiffiffiffiffiffiffiffiffiffiffiffiffiffiffiffiffi
x2 þ xy þ y2 þ y2 þ yz þ z2 þ x2 þ xz þ z2
pffiffiffiffiffiffiffiffiffiffiffiffiffiffiffiffiffiffiffiffiffiffiffiffiffiffiffiffiffiffiffiffiffiffiffiffiffiffiffiffiffiffiffiffiffiffiffiffiffiffiffiffiffiffiffiffiffiffiffiffiffiffiffiffiffiffiffiffi
 5x2 þ 5y2 þ 5z2 þ 4xy þ 4yz þ 4xz, where x, y, z > 0.
8.2.13. Prove that
pffiffiffiffiffiffiffiffiffiffiffiffiffiffiffiffiffiffiffiffiffiffiffiffiffi pffiffiffiffiffiffiffiffiffiffiffiffiffiffiffiffiffiffiffiffiffiffiffiffi pffiffiffiffiffiffiffiffiffiffiffiffiffiffiffiffiffiffiffiffiffiffiffiffi pffiffi3
x2 þ xy þ y2  y2 þ yz þ z2  z2 þ zx þ x2  8 ðx þ y þ zÞ3 , where x > 0,
y > 0, z > 0.
8.2.14. Prove that
αβγ ðx þ y þ zÞ2
αyz þ βzx þ γxy  , where α > 0, β > 0,
2αβ þ 2βγ þ 2γα  α2  β2  γ 2
γ > 0 and 2αβ + 2βγ + 2γα  α2  β2  γ 2 > 0.
8.2.15. Prove that
p ffiffiffiffiffiffiffiffiffiffiffiffiffiffiffiffiffiffiffiffiffiffiffiffiffiffiffiffiffiffiffiffiffiffiffiffiffiffiffiffiffiffi pffiffiffiffiffiffiffiffiffiffiffiffiffiffiffiffiffiffiffiffiffiffiffiffiffiffiffiffiffiffiffiffiffiffi
3
2ðx3 þ y3 þ z3 þ xyzÞ  x2 þ y2 þ z2 þ yz, where y > 0, z > 0 and
max (x, y, z) ¼ x.
432 8 Some Applications of Geometric Inequalities

Solutions

8.2.1. (a) As sin(x) ¼  sin x and |sinx|  1, then it is sufficient to prove the
inequality for 0 < x < 1. We have that (see Figure 8.10) SΔAOB ¼
OA2
2 OA sin x ¼ 2 , S sec AOB ¼ 2  x ¼ 2, since SΔAOB < SsecAOB. Then, sinx < x,
1 2 sin x x

hence at 0 < x < 1, we have that |sinx| < |x|.


(b) We have that (see Figure 8.10)

SAOC ¼ OAOC
2 ¼ 12 tgx, since SΔAOC > SsecAOB. Therefore, tgx > x.
(c) We have that (see Figure 8.11) SsegAB < SsegAC.

Therefore, α 2sin α < β 2sin β, hence α  sin α < β  sin β.

Figure 8.10

Figure 8.11
8.2 Using Geometric Inequalities for Proving Algebraic Inequalities 433

(d) We have that S 0 _ < S 0 _ (see Figure 8.11), consequently tgαα


2 < 2 . Hence
tgββ
B BA C CA
tgα  α < tgβ  β.
(e) We have that

sin β sin ðβ  αÞ cos α þ cos ðβ  αÞ sin α


¼ ¼
β  β 
sin α α 1 sin α 1 sin α
¼ sin ðβ  αÞ þ α cos ðβ  αÞ  < ððβ  αÞ þ αÞ  ¼
α tgα β α β α

(see problems 8.2.1a and b).


(f) We have that

π π π
sin sin  x cos x þ sin x cos x
2 2 2
π ¼ π ¼
2  π 2 x π  2
sin x sin x π 2
¼ sin x þ x cos x  < xþx  ,
x 2 tgx 2 π x 2 π

2
hence sin x > x.
π
tgβ cos ðβ  αÞ 1
(g) We have that ¼ tg ðβ  αÞ þ 1 > tg ðβ  αÞ  þ1>
tgα cos β sin α sin α
βα tgα tgβ
þ 1, hence < .
α α β pffiffiffi
3 3
(h) Let α  β  γ. If α ¼ γ, then sin α þ sin β þ sin γ ¼ .
2
π
If α 6¼ γ, then α < < γ. Let the triangle with angles α, β, γ is inscribed in the
3
circle with the radius 0.5 (see Figure 8.12).
β π β
As ∠DD0 C ¼ ∠DAC ¼ α þ and ∠CDD0 ¼ þ , then ∠DD0 C < ∠ CDD0 .
2 3 2
π  
Therefore CD < CD0 , which means that sin α þ sin γ < sin þ sin α þ γ  π3 .
 3
π π π
In the same way, we obtain that sin β þ sin α þ γ   sin þ sin ,
3 3 3
consequently,
π  π π
sin α þ sin γ þ sin β < sin þ sin β þ sin α þ γ   3 sin :
3 3 3
pffiffiffi
3 3
Hence, sin α þ sin γ þ sin β < .
2
8.2.2. (a) Note that S1 þ S2 þ S3 < π4 (see Figure 8.13) and S1 ¼ sin x cos x,
S2 ¼ cos y(siny  sin x), S3 ¼ cos z(sinz  sin y).
434 8 Some Applications of Geometric Inequalities

Figure 8.12 D

b b
2 2


B
b
b

p
g a
3
C A

Figure 8.13
D(cosz;sinz)
S3 C(cosy;siny)
S2
B(cosx;sinx)
y S1
x z
A(1;0)

Therefore, π4 > sin x cos x þ cos yð sin y  sin xÞ þ cos zð sin z  sin yÞ, hence
π
2 þ 2 sin x cos y þ 2 sin y cos z > sin 2x þ sin 2y þ sin 2z.
pffiffiffiffiffiffiffiffiffiffiffiffiffi
(b) Note that S1 þ S2 þ . . . þ Sn1 < πn4 (see Figure 8.14a), S1 ¼ n2  1 ,
2

pffiffiffiffiffiffiffiffiffiffiffiffiffiffiffi qffiffiffiffiffiffiffiffiffiffiffiffiffiffiffiffiffiffiffiffiffiffiffiffiffiffiffi
S2 ¼ n2  22 , . . . , , Sn1 ¼ n2  ðn  1Þ2 .

pffiffiffiffiffiffiffiffiffiffiffiffiffi pffiffiffiffiffiffiffiffiffiffiffiffiffiffi2ffi qffiffiffiffiffiffiffiffiffiffiffiffiffiffiffiffiffiffiffiffiffiffiffiffiffiffiffi


Therefore, n  1 þ n  2 þ . . . þ n2  ðn  1Þ2 < π4 n2 < 0:79n2 .
2 2
8.2 Using Geometric Inequalities for Proving Algebraic Inequalities 435

Figure 8.14

Figure 8.15

Note that S0 þ S1 þ ::: þ Sn1 > π4 n2 (see Figure 8.14b) and S0 ¼ n,


pffiffiffiffiffiffiffiffiffiffiffiffiffi pffiffiffiffiffiffiffiffiffiffiffiffiffiffiffi qffiffiffiffiffiffiffiffiffiffiffiffiffiffiffiffiffiffiffiffiffiffiffiffiffiffiffi
S1 ¼ n2  1 S2 ¼ n2  22 , . . . , Sn1 ¼ n2  ðn  1Þ2 , consequently,
pffiffiffiffiffiffiffiffiffiffiffiffiffi pffiffiffiffiffiffiffiffiffiffiffiffiffiffi2ffi qffiffiffiffiffiffiffiffiffiffiffiffiffiffiffiffiffiffiffiffiffiffiffiffiffiffiffi
n2  1 þ n2  2 þ . . . þ n2  ðn  1Þ2 > π4 n2  n > 0:785n2  n.

8.2.3. Note that (see Figure 8.15)

X
n
1 ðx2  x 1 Þ2
x2i ðx2iþ1  x2i1 Þ ¼ S1 þ S2 þ . . . þ Sn < þ þ ...
i¼1
2 2
ðx2n  x2n1 Þ2 1 h 1 h
þ  þ ðx2  x1 þ . . . þ x2n  x2n1 Þ < þ
2 2 2 2 2
436 8 Some Applications of Geometric Inequalities

We have that

1 ðx 1  x0 Þ2 ðx2nþ1  x2n Þ2
S1 þ S2 þ . . . þ Sn >   ...  
2 2 2
1 h 1 h
  ðx1  x0 þ . . . þ x2nþ1  x2n Þ >  :
2 2 2 2

8.2.4. Note that the sum a1b1 + a2b2 + . . . + anbn is equal to the area of the shaded
figure (see Figure 8.16), while the sum a1 + . . . + ap + 1 is equal to the area of the
second figure, containing the first figure.
Therefore, a1b1 + a2b2 + . . . + anbn  a1 + a2 + . . . + ap + ap + 1.
This ends the proof.
2 2
8.2.5. (a) See Figure 8.17. We have that ab  S1 þ S2 ¼ a2 þ b2 .
(b) See Figure 8.18.

Let V 1 ¼ V ABB2 C2 D2 , V 2 ¼ V ADB3 C3 D3 , V 3 ¼ V AA1 B4 C4 D4 .


3 3 3
We have that abc ¼ V  V 1 þ V 2 þ V 3 ¼ a3 þ b3 þ c3 .
x þ y þ z 3
Remark If x, y, z > 0, then xyz  .
p ffiffi
ffi p 3ffiffiffi p ffiffi
It is sufficient to take a ¼ 3 x, b ¼ 3 y, c ¼ 3 z .

Figure 8.16

Figure 8.17
8.2 Using Geometric Inequalities for Proving Algebraic Inequalities 437

B4 c C4

c
C3
B1 C1

A1 D1 D4

B3

c D3
B2 C2 b

a b
B D2 C
a

A b D

Figure 8.18

sffiffiffiffiffiffiffiffiffiffiffiffiffiffiffi
2pffiffiffiffiffi a þ b a2 þ b2
(c) If a ¼ b, then ¼ ab ¼ ¼ .
1 1 2 2
þ
a b
If a 6¼ b, then without loss of generality, one can assume that a > b.
ab pffiffiffiffiffi
Let AC ¼ , CB ¼ ab, ∠C ¼ 90 (see Figure 8.19).
2
aþb ab 2
Thus, AB ¼ and HB ¼ aþb ¼ , consequently, HB < CB < AB, that is,
2 1 1
2 þ
pffiffiffiffiffi aþb a b
2
1 1 < ab < 2 .
þ
a b
Let AC ¼ ab 2q, BC ¼ aþb , ∠C ¼ 90 (see Figure 8.20).
ffiffiffiffiffiffiffiffiffi 2 qffiffiffiffiffiffiffiffiffi
Thus AB ¼ a þb a2 þb2
2
< <
2 aþb
2 , and BC AB, which means that 2 2 .

8.2.6. We have that (see Figure 8.21.)


438 8 Some Applications of Geometric Inequalities

Figure 8.19 A

a-b
2

C ab B

Figure 8.20 A

C B

Figure 8.21 B

m a n

A b D c C
p

rffiffiffiffiffiffiffiffiffiffiffiffiffiffiffiffiffiffiffiffiffiffiffiffiffiffiffiffiffiffiffiffiffiffiffiffiffiffiffiffiffiffiffiffiffiffiffiffiffiffiffiffiffiffiffiffiffiffiffiffiffiffiffiffiffiffiffiffiffiffiffiffiffiffiffiffiffiffiffiffiffiffiffiffiffiffiffiffiffiffiffiffiffiffiffiffiffiffiffiffi
að b þ c Þ 1
SABC ¼ ¼ ðm þ n þ pÞðm þ n  pÞðm  n þ pÞðn þ p  mÞ 
2 ffiffiffiffiffiffiffiffiffiffiffiffiffiffiffiffiffiffiffiffiffiffiffiffiffiffiffiffiffiffiffiffiffiffiffiffiffiffiffiffiffiffiffiffiffiffiffiffiffiffiffiffiffiffiffiffi
r 16
1 m þ n þ p 3 1
 ð m þ n þ pÞ ¼ pffiffiffi3 ðm þ n þ pÞ2 ¼
16 3 4 3
1 pffiffiffiffiffiffiffiffiffiffiffiffiffiffiffi pffiffiffiffiffiffiffiffiffiffiffiffiffiffiffi 2
¼ pffiffiffi3  a þb þ a þc þbþc ,
2 2 2 2
4 3

(see the remark of problem 8.2.5b).


8.2 Using Geometric Inequalities for Proving Algebraic Inequalities 439

Therefore, we obtain that


pffiffiffi pffiffiffiffiffiffiffiffiffiffiffiffiffiffiffi pffiffiffiffiffiffiffiffiffiffiffiffiffiffiffi 2
6 3aðb þ cÞ  a2 þ b2 þ a2 þ c2 þ b þ c . Note that the equality holds
pffiffiffi pffiffiffi
true, if and only if 3b ¼ 3c ¼ a.
8.2.7. (Proof of Hayk Sedrakyan, ninth grade)
(a) Without loss of generality, one can assume that xyz ¼ 1.
Let x + y ¼ a, y + z ¼ b, z + x ¼ c, thus we have to prove that (a + b)(b + c)(a + c) 
a2 b 2 c 2 .
Since a, b, c are the sides of some triangle, its area denoted by S, we have that S2
a þ b þ c a þ b  c a  b þ c a þ b þ c a þ b þ c
¼    ¼ and a2b2c2 ¼ 16S2R2.
2 2 2 2 2
Therefore, we have to prove that 2(sinα + sin β + sin γ)  (sinα + sin β)(sinα
+ sin γ)(sinβ + sin γ).
According to the remarks of problems 8.2.5b and 8.2.1h, it follows that
 3
ð sin α þ sin βÞð sin α þ sin γ Þð sin β þ sin γ Þ  2ð sin αþ sin
3
βþ sin γ Þ
¼
8
¼ ð sin α þ sin β þ sin γ Þ  ð sin α þ sin β þ sin γ Þ2 
27
8 27
 ð sin α þ sin β þ sin γ Þ  ¼ 2ð sin α þ sin β þ sin γ Þ:
27 4

(b) Without loss of generality, one can assume that abc ¼ 1.


Let a + b ¼ x, b + c ¼ y, a + c ¼ z, then x, y, z are the sides of some triangle of the
pffiffiffi
area S ¼ p (see the proof of problem 8.2.7a).
We have to prove that xyz  2 + x + y + z or 2RS  1 + p.
pffiffiffiffiffiffiffiffiffiffiffiffiffiffiffiffi
According to problems 8.2.1h and 5.3.1, we have that 2RS ¼ 4R2  S2 ¼
rffiffiffiffiffiffiffiffiffiffiffiffiffiffiffiffiffiffiffiffiffiffiffiffiffiffiffiffiffiffiffiffiffiffiffiffiffiffiffiffiffiffiffiffiffiffiffiffiffiffiffiffiffiffiffiffiffiffiffiffiffiffiffiffiffiffiffiffiffiffiffiffiffiffi
qffiffiffiffiffiffiffi qffiffiffiffiffiffiffiffiffiffiffiffiffiffiffi ffi
pffiffiffiffiffiffiffiffiffiffi pffiffiffiffiffi
2 27p2 þ63p 3 3 xyz
4R2 p  4  27 4 3
p  48p 27  27  p þ 1, as p ¼ xþyþz
2  2  3, 1 ¼
xþyz
2  yþzx
2  2
xþzy
 xyz
8 (see the proof of problem 7.1.13).

(c) We have to prove that

1 1 1 9
2 2
þ 2 2
þ 2 2

ð1  x Þ ð1  yÞ ð1  zÞ ð1  xÞ ð1  yÞ ð1  zÞ 16xyz

or

16ðx þ y þ zÞxyz ðx þ zÞ2 þ ðy þ zÞ2 þ ðx þ yÞ2  9ðx þ zÞ2 ðy þ zÞ2 ðx þ yÞ2 :
440 8 Some Applications of Geometric Inequalities

Figure 8.22 B

2x R

O
G

x R

A M C

Let x + y ¼ a, y + z ¼ b, x + z ¼ c, where xyz ¼ 1. Therefore, S2 ¼ p, thus we have


to prove that 8(a + b + c)(a2 + b2 + c2)  9a2b2c2, or equivalently, a2 + b2 + c2  9R2
(see problem 5.1.1).
According to Stuart’s  theorem, wehave that (see Figure 8.22)
2
2c2 þ2a2 b2 2
þb2 þc2
GO2 ¼ 13  R2 þ 23  R2  b4  29 4 ¼ R2  a 9  0. Hence, a2
+ b2 + c2  9R2.
pffiffiffiffiffiffiffiffiffiffiffiffiffiffiffi pffiffiffiffiffiffiffiffiffiffiffiffiffiffiffi pffiffiffiffiffiffiffiffiffiffiffiffiffiffiffi  
8.2.8. Let abþbcþac
a ¼ tgα, abþbcþac
b ¼ tgβ, abþbcþac
c ¼ tgγ, where α, β, γ 2 0; π2 .
Note that tgα + tgβ + tgγ ¼ tgα  tgβ  tgγ.
tgαþtgβ
Therefore, it follows that 1tgαtgβ ¼ tgγ, or tg(α + β) ¼ tg(π  γ), hence α + β
+ γ ¼ π.
We have that
rffiffiffiffiffiffiffiffiffiffiffi rffiffiffiffiffiffiffiffiffiffiffi rffiffiffiffiffiffiffiffiffiffiffi vffiffiffiffiffiffiffiffiffiffiffiffiffiffiffi v u 2
ffiffiffiffiffiffiffiffiffiffiffiffiffiffiffi vffiffiffiffiffiffiffiffiffiffiffiffiffiffiffi
2a 2b 2c u 2 u u 2
þ þ ¼u t þ u þ u
tgγ ¼
aþb bþc cþa tgα t tgβ t
1þ 1þ 1þ
tgβ tgγ tgα
1  p ffiffiffiffiffiffiffiffiffiffiffiffi
ffi p ffiffiffiffiffiffiffiffiffiffiffiffi
ffi p ffiffiffiffiffiffiffiffiffiffiffiffi
¼ pffiffiffiffiffiffiffiffiffiffiffiffiffiffiffiffiffiffiffiffiffiffiffiffiffiffiffiffiffi sin 2α  sin β þ sin 2β  sin γ þ sin 2γ  sin α 
sin α sin β sin γ
1 pffiffiffiffiffiffiffiffiffiffiffiffiffiffiffiffiffiffiffiffiffiffiffiffiffiffiffiffiffiffiffiffiffiffiffiffiffiffiffiffiffiffiffiffiffiffiffiffiffiffiffiffiffiffiffiffiffiffiffiffiffiffiffiffiffiffiffiffiffiffiffiffiffiffiffiffiffiffiffiffiffiffiffiffiffiffiffiffiffiffiffiffiffiffiffiffiffiffiffi
 pffiffiffiffiffiffiffiffiffiffiffiffiffiffiffiffiffiffiffiffiffiffiffiffiffiffiffiffiffi ð sin 2α þ sin 2β þ sin 2γ Þðsin 2 α þ sin 2 β þ sin 2 γ Þ ¼
sin α sin β sin γ
pffiffiffiffiffiffiffiffiffiffiffiffiffiffiffiffiffiffiffiffiffiffiffiffiffiffiffiffiffiffiffiffiffiffiffiffiffiffiffiffiffiffiffiffiffiffiffiffiffiffi
¼ 4ðsin 2 α þ sin 2 β þ sin 2 γ Þ  3

(see problems 5.3.19b and 5.1.1).


qffiffiffiffiffiffi qffiffiffiffiffiffi qffiffiffiffiffiffi
Hence, aþb2a
þ bþc
2b
þ cþa
2c
 3.
This ends the proof.
8.2 Using Geometric Inequalities for Proving Algebraic Inequalities 441

pffiffi
8.2.9. For x ¼ y ¼ z ¼ 13, we have that C  8 9 3.
We need to prove that for x > 0, y > 0, z > 0, the following inequality holds true
pffiffiffi qffiffiffiffiffiffiffiffiffiffiffiffiffiffiffiffiffiffiffiffiffiffiffiffiffiffiffiffiffiffi
8 3
ðx þ yÞðy þ zÞðz þ xÞ  xyzðx þ y þ zÞ3 : ð8:12Þ
9

Without  of generality, one can assume that x ¼ tgα, y ¼ tgβ, z ¼ tgγ, where
 πloss
α, β, γ 2 0; 2 and α, β, γ are the angles of some triangle.
 
Indeed, denote by x ¼ tgα, y ¼ tgβ, z ¼ tgγ, where α, β, γ 2 0; π2 and we can
assume that x + y + z ¼ xyz, then α, β, γ are the angles of some triangle.
Then, inequality (8.12) is equivalent to the following inequality sin α  sin β
pffiffi
 sin γ  3 8 3 (see problem 5.1.12).
If x, y, z > 0 and x + y + z ¼ 1, then from inequality (8.12), we obtain that
pffiffi
pffiffiffiffiffiffiffi
ðx þ yÞðy þ zÞðz þ xÞ  8 9 3 xyz. This means that the greatest value of C is equal
pffiffi
to 8 9 3.
This ends the proof.
Remark Similarly, one can prove that if x, y, z > 0, x + y + z ¼ 1 and k  12, then
(x + y)(y + z)(z + x)  8  27k  1(xyz)k.
8.2.10. Denote by b + c ¼ m, c + a ¼ n, a + b ¼ k, then a ¼ nþkm
2 , b ¼ mþkn
2 ,
c ¼ 2 , note that for m, n, k, x, y, z > 0.
nþmk

We need to prove that

n m k m k n
ðy þ zÞ þ ðx þ zÞ þ ðy þ zÞ þ ðx þ yÞ þ ðx þ zÞ þ ðx þ yÞ 
m n m k n k
 6 þ 2x þ 2y þ 2z:

As
n m pffiffiffiffiffiffiffiffiffiffiffiffiffiffiffiffiffiffiffiffiffiffiffiffiffiffiffiffi
ðy þ zÞ þ ðx þ zÞ  2 ðy þ zÞðx þ zÞ,
m n
k m pffiffiffiffiffiffiffiffiffiffiffiffiffiffiffiffiffiffiffiffiffiffiffiffiffiffiffiffi
ðy þ zÞ þ ðx þ yÞ  2 ðy þ zÞðx þ yÞ,
m k
k n pffiffiffiffiffiffiffiffiffiffiffiffiffiffiffiffiffiffiffiffiffiffiffiffiffiffiffiffi
ðx þ zÞ þ ðx þ yÞ  2 ðx þ zÞðx þ yÞ,
n k

then it if sufficient to prove that


pffiffiffiffiffiffiffiffiffiffiffiffiffiffiffiffiffiffiffiffiffiffiffiffiffiffiffiffi pffiffiffiffiffiffiffiffiffiffiffiffiffiffiffiffiffiffiffiffiffiffiffiffiffiffiffiffi pffiffiffiffiffiffiffiffiffiffiffiffiffiffiffiffiffiffiffiffiffiffiffiffiffiffiffiffi
ðx þ yÞðy þ zÞ þ ðx þ yÞðx þ zÞ þ ðx þ zÞðy þ zÞ  3 þ x þ y þ z

or
442 8 Some Applications of Geometric Inequalities

pffiffiffiffiffiffiffiffiffiffiffiffiffi pffiffiffiffiffiffiffiffiffiffiffiffiffi pffiffiffiffiffiffiffiffiffiffiffiffi


x2 þ 3 þ y2 þ 3 þ z2 þ 3  3 þ x þ y þ z: ð8:13Þ
pffiffiffi pffiffiffi pffiffiffi  
Denote by x ¼ 3ctgα, y ¼ 3ctgβ, z ¼ 3ctgγ, where α, β, γ 2 0; π2 .
From the condition xy + yz + zx ¼ 3, we deduce that α + β + γ ¼ π.
sin α þ
1
Then, inequality (8.13) is equivalent to the following inequality
pffiffiffi pffiffiffi
sin β þ sin y 
1 1
3 þ ctgα þ ctgβ þ ctgγ, or tg α2 þ tg β2 þ tg 2γ  3:
The last inequality holds true, according to problem 5.1.9a.
8.2.11. Let us consider points M, A, B, C on plane, such that MA ¼ x, MB ¼ y,
MC ¼ z and ∠AMB ¼ ∠ BMC ¼ ∠ CMA ¼ 120 .
According to problem 4.1.8c (or 1.1.14i), we have that xyAB + yzBC
pffiffiffiffiffiffiffiffiffiffiffiffiffiffiffiffiffiffiffiffiffiffiffiffiffi pffiffiffiffiffiffiffiffiffiffiffiffiffiffiffiffiffiffiffiffiffiffiffiffi
+ zxAC  AB  BC  AC. Note that AB ¼ x2 þ xy þ y2 , BC ¼ y2 þ yz þ z2 , AC
pffiffiffiffiffiffiffiffiffiffiffiffiffiffiffiffiffiffiffiffiffiffiffiffi pffiffiffiffiffiffiffiffiffiffiffiffiffiffiffiffiffiffiffiffiffiffiffiffiffi pffiffiffiffiffiffiffiffiffiffiffiffiffiffiffiffiffiffiffiffiffiffiffiffi
¼ x2 þ xz þ z2 , therefore xy x2 þ xy þ y2 þ yz y2 þ yz þ z2 þ
pffiffiffiffiffiffiffiffiffiffiffiffiffiffiffiffiffiffiffiffiffiffiffiffi pffiffiffiffiffiffiffiffiffiffiffiffiffiffiffiffiffiffiffiffiffiffiffiffiffiffiffiffiffiffiffiffiffiffiffiffiffiffiffiffiffiffiffiffiffiffiffiffiffiffiffiffiffiffiffiffiffiffiffiffiffiffiffiffiffiffiffiffiffiffiffiffiffiffiffiffiffiffiffiffiffiffiffi
zx x2 þ xz þ z2  ðx2 þ xy þ y2 Þðy2 þ yz þ z2 Þðx2 þ xz þ z2 Þ:
8.2.12. We need to prove that
pffiffiffiffiffiffiffiffiffiffiffiffiffiffiffiffiffiffiffiffiffiffiffiffiffi pffiffiffiffiffiffiffiffiffiffiffiffiffiffiffiffiffiffiffiffiffiffiffiffi pffiffiffiffiffiffiffiffiffiffiffiffiffiffiffiffiffiffiffiffiffiffiffiffi pffiffiffiffiffiffiffiffiffiffiffiffiffiffiffiffiffiffiffiffiffiffiffiffi
2x2 þ xy þ y2  y2 þ yz þ z2 þ 2 y2 þ yz þ z2  z2 þ zx þ x2
pffiffiffiffiffiffiffiffiffiffiffiffiffiffiffiffiffiffiffiffiffiffiffiffiffi pffiffiffiffiffiffiffiffiffiffiffiffiffiffiffiffiffiffiffiffiffiffiffiffi
þ2 x2 þ xy þ y2  z2 þ zx þ x2 
 3x2 þ 3y2 þ 3z2 þ 3xy þ 3yz þ 3zx,

or equivalently
pffiffiffiffiffiffiffiffiffiffiffiffiffiffiffiffiffiffiffiffiffiffiffiffiffi pffiffiffiffiffiffiffiffiffiffiffiffiffiffiffiffiffiffiffiffiffiffiffiffi pffiffiffiffiffiffiffiffiffiffiffiffiffiffiffiffiffiffiffiffiffiffiffiffi pffiffiffiffiffiffiffiffiffiffiffiffiffiffiffiffiffiffiffiffiffiffiffiffi
2 x2p xy þ y2  ffiy2pþffiffiffiffiffiffiffiffiffiffiffiffiffiffiffiffiffiffiffiffiffiffiffiffi
þffiffiffiffiffiffiffiffiffiffiffiffiffiffiffiffiffiffiffiffiffiffiffiffi yz þ z2 þ 2 y2 þ yz þ z2  z2 þ zx þ x2
þ2 x2 þ xy þ y2  z2 þ zx þ x2 
pffiffiffiffiffiffiffiffiffiffiffiffiffiffiffiffiffiffiffiffiffiffiffiffiffi 2 pffiffiffiffiffiffiffiffiffiffiffiffiffiffiffiffiffiffiffiffiffiffiffiffi 2 pffiffiffiffiffiffiffiffiffiffiffiffiffiffiffiffiffiffiffiffiffiffiffiffi2 
 x2 þ xy þ y2 þ y2 þ yz þ z2 þ z2 þ zx þ x2  ðx þ y þ zÞ2 :

Let us consider points M, A, B, C on plane, such that MA ¼ x, MB ¼ y, MC ¼ z,


and ∠AMB ¼ ∠ BMC ¼ ∠ CMA ¼ 120 .
Hence, we need to prove that 2ca + 2ab + 2bc  a2  b2  c2  (MA + MB + MC)2,
pffiffiffiffiffiffiffiffiffiffiffiffiffiffiffiffiffiffiffiffiffiffiffiffi pffiffiffiffiffiffiffiffiffiffiffiffiffiffiffiffiffiffiffiffiffiffiffiffi pffiffiffiffiffiffiffiffiffiffiffiffiffiffiffiffiffiffiffiffiffiffiffiffiffi
where a ¼ y2 þ yz þ z2 , b ¼ z2 þ zx þ x2 , c ¼ x2 þ xy þ y2 :
Note that
4( p  a)( p  b) + 4( p  b)( p  c) + 4( p  c)( p  a) ¼ 2ac + 2ab + 2bc  a2  b2
 c2, where p ¼ a þ 2b þ c : It is sufficient to prove that
pffiffiffiffiffiffiffiffiffiffiffiffiffiffiffiffiffiffiffiffiffiffiffiffiffiffiffiffiffiffiffiffiffiffiffiffiffiffiffiffiffiffiffiffiffiffiffiffiffiffiffiffiffiffiffiffiffiffiffiffiffiffiffiffiffiffiffiffiffiffiffiffiffiffiffiffiffiffiffiffiffiffiffiffiffiffiffiffiffiffiffiffiffiffiffiffiffiffi
2 ðp  aÞðp  bÞ þ ðp  bÞðp  cÞ þ ðp  cÞðp  aÞ  MA þ MB þ MC:

The last inequality holds true according to problem 7.1.107c.


This ends the proof.
8.2.13. Let us consider equilateral triangle ABC and point P inside of it, such that
PA1 ¼ x, PB1 ¼ y, PC1 ¼ z (Figure 1.7). Thus, it follows that PA ¼
8.2 Using Geometric Inequalities for Proving Algebraic Inequalities 443

pffiffiffiffiffiffiffiffiffiffiffiffiffiffiffiffiffiffiffiffiffiffiffiffi pffiffiffiffiffiffiffiffiffiffiffiffiffiffiffiffiffiffiffiffiffiffiffiffi pffiffiffiffiffiffiffiffiffiffiffiffiffiffiffiffiffiffiffiffiffiffiffiffiffi


y2 þ yz þ z2 , PB ¼ z2 þ zx þ x2 , PC ¼ x2 þ xy þ y2 , and according to prob-
pffiffiffiffiffiffiffiffiffiffiffiffiffiffiffiffiffiffiffiffiffiffiffiffiffi pffiffiffiffiffiffiffiffiffiffiffiffiffiffiffiffiffiffiffiffiffiffiffiffi pffiffiffiffiffiffiffiffiffiffiffiffiffiffiffiffiffiffiffiffiffiffiffiffi
lem 7.1.67b, we have that x2 þ xy þ y2  y2 þ yz þ z2  z2 þ zx þ x2 
pffiffi
3
8 ðx þ y þ zÞ :
3

This ends the proof.


pffiffiffi pffiffiffi pffiffiffi
8.2.14. We have that 2αβ þ 2βγ þ 2γα  α2  β2  γ 2 ¼ αþ βþ γ
 pffiffiffi pffiffiffi pffiffiffipffiffiffi pffiffiffi pffiffiffipffiffiffi pffiffiffi pffiffiffi
 αþ βþ γ α βþ γ α þ β  γ > 0, therefore
pffiffiffi pffiffiffi pffiffiffi pffiffiffi pffiffiffi pffiffiffi pffiffiffi pffiffiffi pffiffiffi
β þ γ > α, α þ γ > β , α þ β > γ :
pffiffiffi pffiffiffi pffiffiffi
Consider points M, A, B, C on plane, such that BC ¼ α, AC ¼ β, AB ¼ γ ,
and MA ¼ MB ¼ MC, From problem 4.1.8а, it follows that (x + y + z)2MA2  αyz
+ βzx + γxy.
pffiffi pffiffi pffiffi pffiffiffiffiffiffiffiffiffiffiffiffiffiffiffiffiffiffiffiffiffiffiffiffiffiffiffiffiffiffiffiffiffiffiffiffiffiffiffiffiffiffiffiffiffiffiffiffiffiffiffiffiffiffiffiffiffiffiffiffiffiffiffiffiffi
α β  γ
We have that MA ¼ 4ðABCÞ and 4ðABCÞ ¼ 2αβ þ 2βγ þ 2γα  α2  β2  γ 2 :
2
αβγ ðx þ y þ zÞ
Thus, it follows that αyz þ βzx þ γxy  2αβ þ 2βγ þ 2γα  α2  β2  γ 2
:
This ends the proof.

 x + y + z + yz ¼ 1.
2 2 2
8.2.15. Without loss of generality, one can assume πthat
Denote by y ¼ cos β, z ¼ cos γ, where β, γ 2 0; 2 :
Note that 4x2  x2 + y2 + z2 + yz ¼ 1 and y2 + z2 < 1.
 
Therefore, x  12 and cos 2 β < 1  cos 2 γ ¼ cos 2 π2  γ :
Thus, it follows that β þ γ > π2 :
Let α ¼ π  β  γ, then cos2α + cos2β + cos2γ + 2 cos α cos β cos γ ¼ 1 (see the
proof of problem 5.1.2). We deduce that
x2 + y2 + z2 + 2xyz  x2 + y2 + z2 + yz, and
x2 + y2 + z2 + yz ¼ 1 ¼ cos2α + cos2β + cos2γ + 2 cos α cos β cos γ.
Hence, we obtain that x  cos α.
This ends the proof.

Problems for Self-Study

8.2.16. Prove the inequalities


(a) x3 + y3 + z3 + 3xyz  x2y + y2x + y2z + z2y + z2x + x2z, where x, y, z > 0,
(b) a4 + b4 + c4 + 3(abc)4/3  2(a2b2 + b2c2 + a2c2), where a, b, c > 0,
pffiffiffiffiffiffiffi pffiffiffi pffiffiffi2 pffiffiffi pffiffiffi2 pffiffiffi pffiffiffi 2
(c) a þ 3b þ c  3 abc 13 a  b þ b  c þ ð c  aÞ , where a, b,
c > 0,
(d) ða þ b þ cÞða  b þ cÞ þ ða  b þ cÞða þ b  cÞ þ ða þ b  cÞða þ b þ cÞ 
pffiffiffiffiffiffiffipffiffiffi pffiffiffi pffiffiffi
 abc a þ b þ c , where a, b, c > 0.
Hint (b) In inequality 8.2.10a, take x ¼ a4/3, y ¼ b4/3, z ¼ c4/3 and use inequality
8.2.5c.
444 8 Some Applications of Geometric Inequalities

pffiffiffiffiffiffiffi qffiffiffiffiffiffiffi pffiffiffiffiffiffiffi



bþc þ þ > 2, where a, b, c > 0.
a b c
8.2.17. Prove the inequality cþa aþb

8.2.18. Prove the inequality (a2 + b2 + c2)(a + b  c)(b + c  a)(c + a  b)  abc(ab


+ bc + ac), where a, b, c > 0.
Hint See the proof of problem 7.1.29p.

8.2.19. Prove the inequality x2 þ xy1 þ y2 þ y2 þ yz1 þ z2 þ z2 þ zx1 þ x2  ðx þ y9þ zÞ2 , where x, y,
z > 0.
Hint See the proof of problem 7.1.107.
8.2.20. Prove that if x, y, z > 0 and x2 + y2 + z2 + 2xyz ¼ 1, then
(a) xyz  18,
(b) x þ y þ z  32,
(c) xy þ yz þ xz  34  x2 þ y2 þ z2 ,
(d) xy þ yz þ xz  12 þ 2xyz.
 
Hint Let x ¼ tgα, y ¼ tgβ, z ¼ tgγ, where α, β, γ 2 0; π2 , then α + β + γ ¼ π.
8.2.21. Prove the inequality
pffiffiffiffiffiffiffiffiffiffiffiffiffi pffiffiffiffiffiffiffiffiffiffiffiffiffi pffiffiffiffiffiffiffiffiffiffiffiffi
xy þ yz þ xz  3 þ 1 þ x2 þ 1 þ y2 þ 1 þ z2 , where x, y, z > 0 and x + y
+ z ¼ xyz.
 
Hint Let x ¼ tgα, y ¼ tgβ, z ¼ tgγ, where α, β, γ 2 0; π2 , then α + β + γ ¼ π.
8.2.22. Prove that if x, y, z > 0 and xy + yz + zx + 2xyz ¼ 1, then
(a) xyz  18,
(b) x þ y þ z  32,
(c) 1x þ 1y þ 1z  4ðx þ y þ zÞ,
2
(d) 1x þ 1y þ 1z  4ðx þ y þ zÞ  ðz2z1 Þ
ð2zþ1Þ, where z ¼ max (x, y, z).
 
Hint Let xy ¼ cos2α, yz ¼ cos2β, x z ¼ cos2γ, where α, β, γ 2 0; π2 .
Then α + β + γ ¼ π.
8.2.23. Prove that, if x, y, z > 0 and xyz ¼ x + y + z + 2, then
(a) xy + yz + zx  2(x + y + z),
pffiffiffi pffiffiffi pffiffi pffiffiffiffiffiffiffi
(b) x þ y þ z  32 xyz.
 
Hint Let 1
xy ¼ cos 2 α, yz1 ¼ cos 2 β, x1z ¼ cos 2 γ, where α, β, γ 2 0; π2 , then α + β
+ γ ¼ π.
8.2.24. Prove the inequality
64(x + y + z)3xyz  27(x + y)2(y + z)2(x + z)2, where x, y, z > 0.
Hint Let x + y ¼ a, y + z ¼ b, x + z ¼ c.
8.2 Using Geometric Inequalities for Proving Algebraic Inequalities 445

8.2.25. Prove the inequality


(3  2a)(3  2b)(3  2c)  a2b2c2, where a, b, c > 0 and a + b + c ¼ 3.
8.2.26. Prove the inequality

x1 x2 cos φ1 þ x2 x3 cos φ2 þ ::: þ xn1 xn cos φn1 þ xn x1 cos φn 


π 
 cos x21 þ x22 þ ::: þ x2n ,
n
where n  2 and φ1 + φ2 + . . . + φn ¼ π.
Hint See the proof of problem 5.5.31.
8.2.27. Prove the inequalities
pffiffiffiffiffiffiffiffiffiffiffiffiffiffiffiffiffiffiffiffiffiffiffiffiffiffi pffiffiffiffiffiffiffiffiffiffiffiffiffiffiffiffiffiffiffiffiffiffiffiffiffi pffiffiffiffiffiffiffiffiffiffiffiffiffiffiffiffiffiffiffiffiffiffiffiffiffi pffiffiffiffiffiffiffiffiffiffiffiffiffiffiffiffiffiffiffiffiffiffiffiffiffiffi
(a) a2 þ ab þ b2 þ b2 þ bc þ c2 þ c2 þ ac þ a2  3 ab þ bc þ ac, where
a, b, c > 0,
pffiffiffiffiffiffiffiffiffiffiffiffiffiffiffiffiffiffiffiffiffiffiffiffiffiffi pffiffiffiffiffiffiffiffiffiffiffiffiffiffiffiffiffiffiffiffiffiffiffiffiffi pffiffiffiffiffiffiffiffiffiffiffiffiffiffiffiffiffiffiffiffiffiffiffiffiffi pffiffiffi
(b) a2 þ ab þ b2 þ b2 þ bc þ c2 þ c2 þ ac þ a2  3ða þ b þ cÞ, where a,
b, c > 0.
8.2.28. Prove the inequality
qffiffiffiffiffiffiffiffiffiffiffiffiffiffiffiffiffiffiffiffiffiffiffiffiffiffiffiffiffiffiffiffiffiffiffiffi p3 ffiffi
1
3 3 abc þ 6ða þ b þ cÞ abc3 , where a, b, c > 0 and ab + bc + ca ¼ 1.
 
Hint Denote by a ¼ ctgα, b ¼ ctgβ, c ¼ ctgγ, where α, β, γ 2 0; π2 .
8.2.29. Prove that
ðxy þ yz þ zxÞ 1
ðxþyÞ2
þ ðyþz
1
Þ 2 þ
1
ðxþzÞ2  94, where x, y, z > 0.

Hint See problem 5.5.10c.


Basic Notations

ρ(X, l ) The distance from the point X to the line l.


ρ(X, Π) The distance from the point X to the plane Π.
AB # CD The segments AB and CD are parallel and equal.
supX Supremum of the set X.
MN The points M and N coincide.
M=6 N The points M and N do not coincide.
(ABC) The plane containing points A , B , C.
Φ  Φ1 All points of the figure Φ are inside the figure Φ1.
Φ  Φ1 The figure Φ is covered by the figure Φ1.
AB \ CD ¼ X The lines AB and CD intersect at point X.
_
ABC The arc ABC.
[AB] The segment AB.
SA1 A2 ...An The area of the polygon A1A2 . . . An.
PA1 A2 ...An The perimeter of the polygon A1A2 . . . An.
ω(O, R) A circle with a center O and radius R.
R The set of real numbers.
[a] The whole part of the number a.
{a} The fractional part of the number a.

© Springer International Publishing AG 2017 447


H. Sedrakyan, N. Sedrakyan, Geometric Inequalities, Problem Books
in Mathematics, DOI 10.1007/978-3-319-55080-0
448 Basic Notations

Notation for the Elements of Triangle ABC

a,b,c The lengths of the sides BC , AC , AB.


p The half-perimeter of the triangle.
SABC or S The area of the triangle.
α,β,γ The values of the angles at the vertices A , B , C.
ma , mb , mc The lengths of the medians drawn from the vertices A , B , C.
ha , hb , hc The lengths of the altitudes drawn from the vertices A , B , C.
la , l b , l c The lengths of the bisectors drawn from the vertices A , B , C.
r and R The inradius and circumradius of the triangle.
ra , rb , rc The radiuses of the excircles.
References

1. Andreescu, T., Feng, Z.: Mathematical Olympiads, problems and solutions from around the
world. Mathematical Association of America, Washington, DC (2000)
2. Bottemi, O., Djorjevic, R., Janic, R., Mitrinovoc, D., Vasic, P.: Geometric inequalities.
Wolters-Noordhoff Publishing, Groningen (1969)
3. Branzei, D.: Notes on geometry. Editura Paralela 45, Bucharest (1999)
4. Galperin, G., Tolpygo, A.: Moscow mathematical Olympiads. Moscow Education, Moscow
(1986)
5. Hadwiger, H., Debrunner, H., Klee, V.: Combinatorial geometry in the plane. Holt, Rinehart
and Winston, New York/Chicago/San Fransisco/Toronto/London (1964)
6. Honsberger, R.: Mathemathical morsels. Mathematical Association of America, Washington,
DC (1978)
7. Kalinin, A., Tereshin, D.: Stereometry 11. Moscow Publication of MPhTI, Moscow (2001)
8. Locmele, A., Palma, I., Ramana, L., Andzans, A., Largbeldts, T.: Neruvadibu pieradisanas
metodes. Krauklitis, Riga (1997).
9. Prasolov, V.: Problems in planimetry. Nauka, Moscow (1995)
10. Scopec, Z.: Geometrical miniatures. Moscow Education, Moscow (1990)
11. Sedrakyan, N.: Created by the students of Armenia. Yerevan State University Press, Yerevan
(1997)
12. Sedrakyan, N., Sedrakyan, H.: Inequalities. Methods of proving 1. Kyowoo Publishing, South
Korea (2015)
13. Sedrakyan, N., Sedrakyan, H.: Inequalities. Methods of proving 2. Kyowoo Publishing, South
Korea (2015)
14. Sharygin, I.: Geometry (from an educational problem to creative). Moscow Drofa, Moscow
(1997)
15. Shklarsky, D., Chentzov, N., Yaglom, I.: Geometric inequalities and problems on a maximum
and minimum. Nauka, Moscow (1970)
16. Shklarsky, D., Chentzov, N., Yaglom, I.: Geometric estimations and problems from combi-
natorial geometry. Nauka, Moscow (1974)

© Springer International Publishing AG 2017 449


H. Sedrakyan, N. Sedrakyan, Geometric Inequalities, Problem Books
in Mathematics, DOI 10.1007/978-3-319-55080-0

Potrebbero piacerti anche